EEVblog Electronics Community Forum

EEVblog => The AmpHour Radio Show => Topic started by: bsfeechannel on October 21, 2021, 03:16:01 pm

Title: #562 – Electroboom!
Post by: bsfeechannel on October 21, 2021, 03:16:01 pm
Dave, KVL doesn't hold under a varying magnetic field.

Repeat it until enlightened.

Mehdi now comes with yet another explanation for his lack (which he admits) of understanding of electromagnetism : the old vs new definition (i.e. what he thinks is the definition) of voltage.

As for Lewin's experiment, it was just Faraday's experiment with fancy test equipment. Faraday did what Lewin did, using an unshielded galvanometer with completely unshielded probes and using an electromagnet to generate the varying magnetic field.

If Lewin was wong in anything, Faraday was also wrong. And that's not the case for about the last two centuries.

So, give up this nonsense already. Please.
Title: Re: #562 – Electroboom!
Post by: jesuscf on November 03, 2021, 05:22:25 am
Dave, KVL doesn't hold under a varying magnetic field.

Really?  So according to the statement, AC circuits (with transformers and such) can not be solved using KVL!  You better go tell the whole electric power industry that they have been analyzing circuits under the effects of magnetic fields incorrectly for the last 160 years!!!
Title: Re: #562 – Electroboom!
Post by: bsfeechannel on November 05, 2021, 01:07:34 pm
Dave, KVL doesn't hold under a varying magnetic field.

Really?

Really.

Quote
So according to the statement, AC circuits (with transformers and such) can not be solved using KVL!

AC circuits can be solved with KVL as long as you maintain all the varying magnetic fields away from the path of the analyzed circuit. This is what happens with a transformer for instance. All the varying magnetic fields are inside the transformer core, not inside the path of the circuit.

Quote
You better go tell the whole electric power industry that they have been analyzing circuits under the effects of magnetic fields incorrectly for the last 160 years!!!

I don't need to. The whole power electric industry never made the mistake of analyzing a circuit under a varying magnetic field using KVL. In fact the whole power electric industry started out because of Faraday's discoveries which was solely explained by Maxwell's equations. And that's what we use to this very day.
Title: Re: #562 – Electroboom!
Post by: thinkfat on November 05, 2021, 04:12:26 pm
I think Lewins model on which he applied KVL was incorrect. The lines in a circuit diagram don't interact with a magnetic field.

If you watch the original lecture, the error creeps in when he assumes "A1=A2" and "D1=D2", but this just doesn't hold.
Title: Re: #562 – Electroboom!
Post by: bsfeechannel on November 05, 2021, 08:30:39 pm
I think Lewins model on which he applied KVL was incorrect. The lines in a circuit diagram don't interact with a magnetic field.

If you watch the original lecture, the error creeps in when he assumes "A1=A2" and "D1=D2", but this just doesn't hold.

What is absolutely amusing about the people who can't accept that KVL doesn't hold under a varying magnetic field is the disparity of explanations.

For Mehdi at first, Lewin's experiment was flawed because of bad probing. When he tried to replicate the experiment and couldn't get a different result no matter how good probing he did, he tried to reinvent the theory creating what he calls the new definition of voltage.

Dave came with a Deus ex-machina called "Practical Engineering". It's a completely irrational explanation with absolutely no basis on science.

Cyriel Mabilde came with the "masked" EMF, which he states is much simpler to understand than the real thing which is Faraday's law.

Others tried to manipulate the math to make it appear that the voltages found in such a circuit actually add up to zero.

Now Lewin doesn't know how to apply Kirchhoff's law to a circuit, and, worse, doesn't know how to draw diagrams.

So summarizing, KVL always holds because Lewin didn't know how to probe the circuit, because he's adherent to the "old definition of voltage", because he's a physicist and not a practical engineer, because he doesn'1 know how to manipulate the math so that you make Maxwell's equation of magnetic inductance to always show a zero on one o the sides of the equal sign and finally because he doesn't understand how to model circuits and draw diagrams.

The truth is that "KVL always hold" is a myth, brilliantly debunked by Lewin. And up to this day no one could show that those two resistors in a loop under a varying magnetic field will not show different voltages, no matter what.

You can fool an audience. But, as Richard Feynman said, nature cannot be fooled.
Title: Re: #562 – Electroboom!
Post by: armandine2 on November 05, 2021, 08:40:09 pm
brilliantly debunked by Lewin.

experimentally?
Title: Re: #562 – Electroboom!
Post by: bsfeechannel on November 05, 2021, 08:44:10 pm
Yes. Try it yourself.
Title: Re: #562 – Electroboom!
Post by: thinkfat on November 05, 2021, 10:57:54 pm
I think Lewins model on which he applied KVL was incorrect. The lines in a circuit diagram don't interact with a magnetic field.

If you watch the original lecture, the error creeps in when he assumes "A1=A2" and "D1=D2", but this just doesn't hold.

Now Lewin doesn't know how to apply Kirchhoff's law to a circuit, and, worse, doesn't know how to draw diagrams.


Precisely. You cannot on one hand assume those "wires" in the circuit be "ideal" and effectively form one "node", and at the same time expect them to span up a area for the magnetic flux to pass through. It doesn't work that way. These circuit lines are dimensionless. They have zero length. The area is zero, therefore, and no magnetic flux passing through them. Besides, KVL is a mere calculation tool for network analysis. If your result doesn't match the measurements, clearly something is wrong with the model. But not with the method as such. That's a bit like blaming spice for not matching reality when you forgot to put an element.
Title: Re: #562 – Electroboom!
Post by: bsfeechannel on November 06, 2021, 02:15:29 am
Precisely. You cannot on one hand assume those "wires" in the circuit be "ideal" and effectively form one "node", and at the same time expect them to span up a area for the magnetic flux to pass through. It doesn't work that way. These circuit lines are dimensionless. They have zero length. The area is zero, therefore, and no magnetic flux passing through them. Besides, KVL is a mere calculation tool for network analysis. If your result doesn't match the measurements, clearly something is wrong with the model. But not with the method as such. That's a bit like blaming spice for not matching reality when you forgot to put an element.

The myth says that KVL ALWAYS holds even when a circuit is under the effect of a varying magnetic field. Lewin showed at least one circuit where KVL doesn't hold. Everyone repeated his experiment and obtained the same result. Conclusion: KVL does NOT ALWAYS hold. End of story.

If Lewin didn't wear the appropriate hairstyle or used a chalk of the wrong color, that's absolutely irrelevant.
Title: Re: #562 – Electroboom!
Post by: thinkfat on November 06, 2021, 12:25:55 pm
Do you have an actual argument or are you just forever going to repeat yourself?

There is nothing wrong with the experiment showing Faradays Law in action. There is also nothing wrong with the probing or the instruments. That is undeniable. But if you then incorrectly translate that into a circuit diagram and blindly apply KVL, making wrong assumptions, then of course you will not get the correct result.

Do you understand that a circuit diagram assumes lumped elements? So where's the element that corresponds to the "coil", the area the magnetic flux permeates? At the very least you'd have to model that with an ideal current source that represents the current induced by the magnetic flux. But lo and behold, the internal resistance of an ideal current source in infinite! Does Lewins' "A1=A2" and "D1=D2" still hold?
Title: Re: #562 – Electroboom!
Post by: bsfeechannel on November 06, 2021, 03:15:10 pm
That's precisely your, Mehdi's, Dave's and everybody-else-that-insist-that-KVL-always-hold's mistake.

KVL says that the voltages measured along the path of a mesh add up to zero.

But Lewin showed at least one circuit where the voltages do not add up to zero.

Therefore KVL doesn't always hold. Period.

There's nothing you can do to remedy this. It is not about Lewin. It is the way nature works. Go to your lab, set up the same experiment, measure the voltages around the circuit and see for yourself. We are glad that those who repeated the experiment were precisely the guys who thought that Lewin had somehow cheated or blundered the experiment.

They're all dead inside now. All of them.

You cannot solve such circuit using KVL. It is impossible. You'll have to resort to the full monty and use Maxwell's equations at least to calculate the EMF produced by the varying magnetic field.

After that, you can of course devise an equivalent circuit where instead of a varying magnetic field producing the extra EMF upon the entire circuit, you have a battery, a generator, a transformer, or any other equivalent lumped (i.e. localized) component to produce the same EMF and get the exact same effect on the other components. In that case, you can solve the equivalent circuit using KVL because you theoretically removed the varying field from the circuit and stashed it away in the equivalent component.

But that is just a theoretical trick that has a lot of caveats.

One of them is that you will not find this extra EMF on the circuit. Not even if you fart your way through the Bohemian Rapsody.

So how's that possible? How can we have, so to speak, a spooky "component" that produces an EMF in a circuit, but is not present there? This seems to violate the principle of conservation of energy, doesn't it? Those are legit questions. But to answer them you need to abandon KVL, which these people are not prepared to accept. To reconcile their cognitive bias with the real phenomenon that contradicts it, they create all the irrational arguments I listed above and more.

Title: Re: #562 – Electroboom!
Post by: jesuscf on November 06, 2021, 03:37:09 pm
Quote
You better go tell the whole electric power industry that they have been analyzing circuits under the effects of magnetic fields incorrectly for the last 160 years!!!
I don't need to. The whole power electric industry never made the mistake of analyzing a circuit under a varying magnetic field using KVL. In fact the whole power electric industry started out because of Faraday's discoveries which was solely explained by Maxwell's equations. And that's what we use to this very day.

Right there is proof that you have no idea of what you are talking about!  If (and that is a big IF) you are an electrical/electronics engineer, you are embarrassing yourself.  If your background is in physics, then I know were you are coming from, and your ignorance is understandable.
Title: Re: #562 – Electroboom!
Post by: thinkfat on November 06, 2021, 05:17:44 pm
That's precisely your, Mehdi's, Dave's and everybody-else-that-insist-that-KVL-always-hold's mistake.

KVL says that the voltages measured along the path of a mesh add up to zero.

But Lewin showed at least one circuit where the voltages do not add up to zero.


Only because he omitted a critical element of the circuit. His diagram was simply incomplete: he drew a line in the diagram and then promptly neglected that it's a real, physical wire and not an ideal "net" you would find in a schematic.

You cannot solve such circuit using KVL. It is impossible. You'll have to resort to the full monty and use Maxwell's equations at least to calculate the EMF produced by the varying magnetic field.

After that, you can of course devise an equivalent circuit where instead of a varying magnetic field producing the extra EMF upon the entire circuit, you have a battery, a generator, a transformer, or any other equivalent lumped (i.e. localized) component to produce the same EMF and get the exact same effect on the other components. In that case, you can solve the equivalent circuit using KVL because you theoretically removed the varying field from the circuit and stashed it away in the equivalent component.

But that is just a theoretical trick that has a lot of caveats.


And that is exactly the error that Dr. Lewin made: He drew up a circuit, but it was not the equivalent of his experiment and then tried to solve that with KVL, and of course it failed.
Solving an equivalent circuit is not a theoretical trick, it is exactly how network analysis works. How do you calculate a circuit that has a BJT in it? With an equivalent circuit using e.g. the Ebers-Moll transistor model.


So how's that possible? How can we have, so to speak, a spooky "component" that produces an EMF in a circuit, but is not present there? This seems to violate the principle of conservation of energy, doesn't it? Those are legit questions. But to answer them you need to abandon KVL, which these people are not prepared to accept. To reconcile their cognitive bias with the real phenomenon that contradicts it, they create all the irrational arguments I listed above and more.

That's exactly your problem. There cannot be a "spooky component" that is not present but causes an effect. You have to have it in your model, otherwise all calculations are nonsense. Yes, you cannot calculate current induced by magnetic flux with KVL, yes you need to solve the Maxwell-Faraday equation to calculate the EMF, but after you've done that, you cannot simply forget about it and omit if from your circuit and postulate: "current flows in this circuit but without anything that causes it".
Title: Re: #562 – Electroboom!
Post by: jesuscf on November 06, 2021, 07:00:40 pm

So how's that possible? How can we have, so to speak, a spooky "component" that produces an EMF in a circuit, but is not present there? This seems to violate the principle of conservation of energy, doesn't it? Those are legit questions. But to answer them you need to abandon KVL, which these people are not prepared to accept. To reconcile their cognitive bias with the real phenomenon that contradicts it, they create all the irrational arguments I listed above and more.

That's exactly your problem. There cannot be a "spooky component" that is not present but causes an effect. You have to have it in your model, otherwise all calculations are nonsense. Yes, you cannot calculate current induced by magnetic flux with KVL, yes you need to solve the Maxwell-Faraday equation to calculate the EMF, but after you've done that, you cannot simply forget about it and omit if from your circuit and postulate: "current flows in this circuit but without anything that causes it".

bsfeechannel understanding of KVL is stuck in 1845.
Title: Re: #562 – Electroboom!
Post by: bsfeechannel on November 06, 2021, 10:40:11 pm
Right there is proof that you have no idea of what you are talking about!  If (and that is a big IF) you are an electrical/electronics engineer, you are embarrassing yourself.  If your background is in physics, then I know were you are coming from, and your ignorance is understandable.

Ah the ad hominem argument. Let's add it to the list above.

KVL always holds because in the eyes of EEVBlog forum member jesuscf, EEVBlog member bsfeechannel is perhaps not an engineer, but probably a physicist.
Title: Re: #562 – Electroboom!
Post by: bsfeechannel on November 06, 2021, 10:45:28 pm
bsfeechannel understanding of KVL is stuck in 1845.

Ooh! I didn't see that one.

KVL always holds because bsfeechannel understanding of KVL is stuck in 1845.
Title: Re: #562 – Electroboom!
Post by: jesuscf on November 07, 2021, 12:27:34 am
Right there is proof that you have no idea of what you are talking about!  If (and that is a big IF) you are an electrical/electronics engineer, you are embarrassing yourself.  If your background is in physics, then I know were you are coming from, and your ignorance is understandable.

Ah the ad hominem argument. Let's add it to the list above.

KVL always holds because in the eyes of EEVBlog forum member jesuscf, EEVBlog member bsfeechannel is perhaps not an engineer, but probably a physicist.

Hey bsfeechannel, I am honestly curious about your credentials.  Are you an electrical/electronics engineer with a degree from a  university (or similar) or not?

I'll need you to prove it by solving "problem #24" from your hero Dr. Lewin, but as an electrical/electronics engineer will do:

https://www.youtube.com/watch?v=7Y7FZ_AtrEs (https://www.youtube.com/watch?v=7Y7FZ_AtrEs)



Title: Re: #562 – Electroboom!
Post by: bsfeechannel on November 07, 2021, 03:54:47 am
But Lewin showed at least one circuit where the voltages do not add up to zero.
Only because he omitted a critical element of the circuit.

He, and everyone who repeated the experiment, measured the voltages and they didn't add up to zero. That's how he showed it.

Quote
His diagram was simply incomplete: he drew a line in the diagram and then promptly neglected that it's a real, physical wire and not an ideal "net" you would find in a schematic.

That didn't change the outcome of the experiment. Neither on his bench, nor on the bench of everyone else's. I. e., the voltages didn't add up to zero.

Quote
And that is exactly the error that Dr. Lewin made: He drew up a circuit, but it was not the equivalent of his experiment and then tried to solve that with KVL, and of course it failed.

The circuit had just two resistors and nothing else. And his model predicted exactly what happened in practice. KVL failed on the board. And then failed on the bench.

Quote
Solving an equivalent circuit is not a theoretical trick, it is exactly how network analysis works. How do you calculate a circuit that has a BJT in it? With an equivalent circuit using e.g. the Ebers-Moll transistor model.

You do not expect to find two diodes and two current sources inside a transistor, do you?

Quote
That's exactly your problem. There cannot be a "spooky component" that is not present but causes an effect.

Exactly. So what is going on?

Quote
You have to have it in your model, otherwise all calculations are nonsense.

Perhaps, what you need to have in your model is not a component. It is something else.

Quote
Yes, you cannot calculate current induced by magnetic flux with KVL, yes you need to solve the Maxwell-Faraday equation to calculate the EMF,

Perfect.

Quote
but after you've done that, you cannot simply forget about it and omit if from your circuit and postulate: "current flows in this circuit but without anything that causes it".

Who said that?





Title: Re: #562 – Electroboom!
Post by: bsfeechannel on November 07, 2021, 04:30:41 am
Hey bsfeechannel, I am honestly curious about your credentials.

Thank you for your interest in my credentials.

Quote
Are you an electrical/electronics engineer with a degree from a  university (or similar) or not?

I think that's irrelevant for the present discussion.

Quote
I'll need you to prove it by solving "problem #24" from your hero Dr. Lewin, but as an electrical/electronics engineer will do:

Lewin is not my "hero". I don't subscribe to his channel, nor follow him anywhere on the social networks.

He just happened to show a physical phenomenon whose understanding is very important for electronics engineering.
Title: Re: #562 – Electroboom!
Post by: jesuscf on November 07, 2021, 04:34:26 am
You do not expect to find two diodes and two current sources inside a transistor, do you?

Actually you do! As a matter of fact you can use the BE or BC junction of a BJT as a diode as represented in the Ebers-Moll model!  As for the current sources, that is the whole point of BJTs!!! So I don't see what you are trying to imply here.  Oh, wait, I know what were you may be going: because Kirchhoff didn't know about the physics of semiconductors, KVL can not be applied when you have semiconductors in a circuit???  As ridiculous at it sounds it is the same argument you are using with Faraday's law and KVL after all.
Title: Re: #562 – Electroboom!
Post by: bdunham7 on November 07, 2021, 04:35:51 am
The circuit had just two resistors and nothing else. And his model predicted exactly what happened in practice. KVL failed on the board. And then failed on the bench.

I really don't want to wade into this mess, but the glaring issue for me in the original was the fact that Dr. Lewin made a dramatic point of saying (and drawing) that the two voltage measuring devices were connected to the same points and that they would demonstrate the the voltage from one point to another was 'path dependent' by displaying different values.  As a practical matter, if there are two different voltages between the two points depending on which branch you follow, how do the oscilloscopes 'know' which branch they are measuring?  Is that determined by where they are physically placed?  Are they briefed beforehand?  If you want to debate, correct or wrangle about anything I've said, please answer this question first as I have no desire to argue this issue until that is cleared up.

And as a theoretical matter, the voltage between two points can never be 'path dependent', that's ridiculous.  Voltage is at its core an absolute value, we just typically use and measure relative values because, well, circuits and current.

Now as for KVL and magnetic fields, you can all have at it, but there's one thing I haven't seen mentioned, so someone point it out if I've missed it:  Change in flux through a loop causes EMF, EMF causes current to flow, that current then causes......counter EMF?  No?  Anyway, connecting two test instruments to different points on a wire with current flowing in it and in a changing magnetic field and then claiming they are connected to the 'same' point makes me not want to try to solve the problem.

And for whoever posted the 5-resistor video, maybe I've misunderstood, but I seem to see another glaring error regarding the supposed symmetry of the current between the L-lower and R-upper resistors regardless of the resistance.  Try 0R for the right upper one and see if that holds! 

So no need for a pissing match, if someone can explain where I'm wrong here I'll listen.   

Title: Re: #562 – Electroboom!
Post by: jesuscf on November 07, 2021, 04:37:15 am
Hey bsfeechannel, I am honestly curious about your credentials.

Thank you for your interest in my credentials.

Quote
Are you an electrical/electronics engineer with a degree from a  university (or similar) or not?

I think that's irrelevant for the present discussion.

Quote
I'll need you to prove it by solving "problem #24" from your hero Dr. Lewin, but as an electrical/electronics engineer will do:

Lewin is not my "hero". I don't subscribe to his channel, nor follow him anywhere on the social networks.

He just happened to show a physical phenomenon whose understanding is very important for electronics engineering.

Then, solve the f***ing circuit!!!  Show me that you know what you are talking about.  It really only takes a couple of minutes.
Title: Re: #562 – Electroboom!
Post by: jesuscf on November 07, 2021, 05:07:17 am
And for whoever posted the 5-resistor video, maybe I've misunderstood, but I seem to see another glaring error regarding the supposed symmetry of the current between the L-lower and R-upper resistors regardless of the resistance.  Try 0R for the right upper one and see if that holds! 

It is the way Lewin solves the circuit and the pomposity of "his" solution as he follows up with three more videos more that the circuit and explanation itself!  It really takes around 2 minutes to properly solve such a trivial circuit without any trickery whatsoever.  I'll post the solution here tomorrow if bsfeechannel doesn't do it first.

EDIT: the solution is attached.  A day has passed and bsfeechannel is still talking BS but it is not capable of solving a very simple circuit.
Title: Re: #562 – Electroboom!
Post by: bsfeechannel on November 07, 2021, 06:10:51 am
As a practical matter, if there are two different voltages between the two points depending on which branch you follow, how do the oscilloscopes 'know' which branch they are measuring?

Good question. For your answer see below.

Quote
Is that determined by where they are physically placed?

Yes. That's why we study vector calculus at any engineering graduation course before we study electromagnetism. Because to understand this bleep you need to think "fourth-dimentionally". I.e. you need to understand that electricity and magnetism are not phenomena confined to electronic components, and how this thing behave in space.

So, frame of references, relative positions, relative velocities, paths, rates of change. All of that counts.

Quote
Are they briefed beforehand?

Nope. That would break causality.

Quote
If you want to debate, correct or wrangle about anything I've said, please answer this question first as I have no desire to argue this issue until that is cleared up.

The oscilloscopes "know" what branch they are measuring because they form a loop with each resistor. If you pause Lewin's presentation on Youtube ( /watch?v=nGQbA2jwkWI ) at 41:54, you'll see that the scope on the right forms a loop with R2. In that loop, there's no varying magnetic field. So, all the voltages will add up to zero according to Faraday's law and, in this case, to KVL, which is nothing more than a special case of Faraday's law when you have no varying magnetic field inside the path of the circuit. So the voltage on the right scope will have to be exactly the voltage on R2.

The same thing is happening to the left scope and R1.

Quote
And as a theoretical matter, the voltage between two points can never be 'path dependent', that's ridiculous.  Voltage is at its core an absolute value, we just typically use and measure relative values because, well, circuits and current.

Voltage can be path dependent if you are dealing with a non conservative electric field (i.e. one generated by a varying magnetic field).

Quote
Now as for KVL and magnetic fields, you can all have at it, but there's one thing I haven't seen mentioned, so someone point it out if I've missed it:  Change in flux through a loop causes EMF, EMF causes current to flow, that current then causes......counter EMF?  No?

That current will produce a magnetic field. While this current, and the corresponding magnetic field are varying, they will produce a counter EMF. The amplitude  will depend on the intensity of this magnetic field and its rate of change. But if you conveniently choose resistors that will allow a relatively low current, this counter EMF will be very low and can be neglected.

Quote
Anyway, connecting two test instruments to different points on a wire with current flowing in it and in a changing magnetic field and then claiming they are connected to the 'same' point makes me not want to try to solve the problem.

Since the wires have very low resistance compared to the resistors, they can be considered practically dead shorts.
Title: Re: #562 – Electroboom!
Post by: thinkfat on November 07, 2021, 07:35:26 am
I just have to ask: do you fundamentally understand the Maxwell-Faraday Equation? I have my doubts. Otherwise you would not claim that the circuit consists of only two resistors connected by wires that can be seen as "dead shorts" and will have no voltage across them.

Do you agree that the Maxwell-Faraday equation relates a time varying magnetic flux to an electric field? Do you agree that this changing flux _causes_ an electric field in the wire? Do you agree that an electric field is a difference in potential? Do you agree that a potential difference is measurable as a voltage between the wire ends? This is very much how a transformer works.

If you agree on all this, then you should be able to see that Dr. Lewins circuit is not just two resistors. It is in fact a transformer (or maybe generator) with the single secondary winding cut open in two places to insert resistors. And it is the voltage across those two wire that you need to take into account when you add up all the voltages in the loop.
Title: Re: #562 – Electroboom!
Post by: bsfeechannel on November 07, 2021, 08:53:53 am
I just have to ask: do you fundamentally understand the Maxwell-Faraday Equation? I have my doubts. Otherwise you would not claim that the circuit consists of only two resistors connected by wires that can be seen as "dead shorts" and will have no voltage across them.

Do you agree that the Maxwell-Faraday equation relates a time varying magnetic flux to an electric field? Do you agree that this changing flux _causes_ an electric field in the wire? Do you agree that an electric field is a difference in potential? Do you agree that a potential difference is measurable as a voltage between the wire ends?

OK. So let's suppose that the wires in Lewin's circuit have a 0.1 ohm resistance. R1=100 Ω and R2=900 Ω. So, around the loop, we will have 100 + 900 + 0.1 + 0.1 = 1000.2 ohms. Now let's suppose that the EMF generated by the varying magnetic field induces a current of, say, 1mA. Multiplying that current by the resistance of each wire (0.1 Ω) will give us 100 µV on them. The voltage on R1 will be 100 mV, while on R2 we will have 900 mV. Adding all voltages up, we will have 1.0002 V. So there you have it. The voltages around the loop will still not be adding up to zero.

If you don't believe the math. Build the circuit and measure those voltages. In fact, Cyriel Mabilde did exactly that and since the missing EMF didn't appear on the wires, he concocted the concept of the "masked EMF" that affects the probes of the meter only when you measure the voltages on the wires, but not on the resistors.  :-// This "masked EMF" is what would fool your meter and would sort of "mask" the missing EMF that is somewhere hidden in the wires.

Pseudo-science galore.

Quote
This is very much how a transformer works.

If you agree on all this, then you should be able to see that Dr. Lewins circuit is not just two resistors. It is in fact a transformer (or maybe generator) with the single secondary winding cut open in two places to insert resistors.


Yes. The secondary of a transformer is just a very low resistance wire shorting out a load. There are no other components in this kind of circuit. The wire and the load surround an area where you have a varying magnetic field. If you measure the voltage on the load and the voltage on the wire, they do not add up to zero.

Lewin used two resistors to enhance the effect, but he could very well have used just a resistor and a piece of wire. That would require a more sensitive scope to measure the voltage on the wire due to the low resistance, but it would obviously not be impossible. The voltages though would not still add up to zero.

Try it yourself. I encourage you. It will be fun to have your mind blown and a long cherished myth shattered to pieces. Things that look like black magic in electronics will suddenly start to make sense. You'll like it.

Quote
And it is the voltage across those two wire that you need to take into account when you add up all the voltages in the loop.

Yes I did and they still do not add up to zero.
Title: Re: #562 – Electroboom!
Post by: jesuscf on November 07, 2021, 03:38:46 pm
OK. So let's suppose that the wires in Lewin's circuit have a 0.1 ohm resistance. R1=100 Ω and R2=900 Ω. So, around the loop, we will have 100 + 900 + 0.1 + 0.1 = 1000.2 ohms. Now let's suppose that the EMF generated by the varying magnetic field induces a current of, say, 1mA. Multiplying that current by the resistance of each wire (0.1 Ω) will give us 100 µV on them. The voltage on R1 will be 100 mV, while on R2 we will have 900 mV. Adding all voltages up, we will have 1.0002 V. So there you have it. The voltages around the loop will still not be adding up to zero.

Now let assume that the resistors in Lewin's circuit are of the wire bound kind.  Oopsie daisies!  As you can see we can assume lots of things.  But when electroboom did the experiment he accounted for many of this things... and KVL worked as expected:

https://www.youtube.com/watch?v=0TTEFF0D8SA (https://www.youtube.com/watch?v=0TTEFF0D8SA)

One more thing:  bsfeechannel,  I called you out and you didn't deliver.  You have no idea of what you are talking about.  Talking to you is exactly like talking to a flat earther or an antivaxer.  So I decided to go thunderf00t on you call out all your BS. 

Title: Re: #562 – Electroboom!
Post by: thinkfat on November 07, 2021, 05:02:01 pm
I started writing a detailed response. But I think I let the posts of @bsfeechannel just stand on their own. I mean, just look at the calculations.

I conclude as follows:  :wtf: :palm: and will now stop  |O.
Have a nice day.
Title: Re: #562 – Electroboom!
Post by: jesuscf on November 07, 2021, 05:10:52 pm
I started writing a detailed response. But I think I let the posts of @bsfeechannel just stand on their own. I mean, just look at the calculations.

I conclude as follows:  :wtf: :palm: and will now stop  |O.
Have a nice day.

We have to keep calling him out and pointing out bsfeechannel's BS!  In a previous thread on the same subject about two/three years ago a snowflake got offended because I called him out (it may even have been bsfeechannel) so I stopped doing that.   That was a mistake.  BS must be pointed out and identified every time it shows up!  :box:
Title: Re: #562 – Electroboom!
Post by: bdunham7 on November 07, 2021, 05:17:00 pm
Yes. That's why we study vector calculus at any engineering graduation course before we study electromagnetism. Because to understand this bleep you need to think "fourth-dimentionally". I.e. you need to understand that electricity and magnetism are not phenomena confined to electronic components, and how this thing behave in space.

So, frame of references, relative positions, relative velocities, paths, rates of change. All of that counts.

That's three lines of blather that doesn't answer my question.  You're telling me that the display of my oscilloscope depends on not only how I connect it, but where I physically place it.  Although that may be true at some very minor level due to interference and other effects, it is nonsense here.  Suppose I had long cables on the two scopes and swapped their physical positions left and right, well out of reach of any magnetic field from the experiment.  Now they read the other way?  How about if I take the leads out perpendicularly and use two channels of one scope?  How about if I just use one scope?  These are just simple thought experiments that seem to me to reduce the experiment as claimed to an absurdity.  I know some people have looked at this experimentally and I have no comment on those arguments because I haven't looked at them closely.

Quote
The oscilloscopes "know" what branch they are measuring because they form a loop with each resistor. If you pause Lewin's presentation on Youtube ( /watch?v=nGQbA2jwkWI ) at 41:54, you'll see that the scope on the right forms a loop with R2. In that loop, there's no varying magnetic field. So, all the voltages will add up to zero according to Faraday's law and, in this case, to KVL, which is nothing more than a special case of Faraday's law when you have no varying magnetic field inside the path of the circuit. So the voltage on the right scope will have to be exactly the voltage on R2.

OK, if you are depending on the physical layout of the test leads, rather than the ultimate location of the oscilloscope (a much less ridiculous choice) than you need to reexamine the statement that there is no flux through that outer loop.  In order for the solenoid to induce a current in the loop, there has to be a net flux change inside the loop, but those flux lines have to eventually wrap around and go back to their opposite pole.  If they do that inside the loop, then they cancel out the net flux.  If they do that anywhere outside the loop, then your 'no varying magnetic field in the loop' becomes very questionable and would need to be measured by making another loop as physically close as possible but with a separate resistor not connected to the inner loop. 

Quote
Voltage can be path dependent if you are dealing with a non conservative electric field (i.e. one generated by a varying magnetic field).

Yes, I know the concepts and math (maybe once upon a time) when you are defining voltage potential as the work required to move a charge and so on.  There are multiple ways to define voltage, and when you factor in time variance, sometimes they don't add up.  If you are going to demonstrate a particular theory (path dependence) by relying on a test instrument, you need to consider what it is that the test instrument actually shows. 

As a thought experiment, envision an 'absolute' leaf electrometer that has one leaf and a charged plate is held parallel to the leaf and insulated from all surroundings.  The charged plate will generate a constant electric field which will cause the leaf to move according to the charge on itself.  Since the leaf will have a capacitance, the charge and voltage will be proportional and V = Q/C.  Thus I have a one-leaded absolute voltmeter.  It might need some calibrating, but it will now show the absolute (meaning without reference or referred to an electrically neutral object, not unsigned) voltage on its leaf without needing reference to anything else.  If I put two of these at different points in a circuit, I can observe both and then determine the difference.  If they are connected to the same point, then they have to show the same value because if not, current would flow from one to the other.  So if I put two of them at each point in the circuit, the ones that are connected to the same point must read the same simply because there's nothing to cause them to read otherwise.  That's all assuming there are no external electric or magnetic fields affecting the instruments--which is, of course a very problematic assumption in a time-variant system.

Now if you look at how most actual voltage measuring instruments work, whether they are an analog meter or oscilloscope, they measure the difference in absolute potential across their two input terminals.  They can do this either by reacting to electric fields directly or by allowing a small amount of current to flow.  In other words, the ideal voltmeter, however it works, reacts just like my hypothetical electrometer pair.   The voltmeter doesn't actually care about path dependence or anything else in the DUT, just about the potentials presented at its inputs.  It just measures the difference between two scalar quantities.

Quote
Since the wires have very low resistance compared to the resistors, they can be considered practically dead shorts.

Since this a time-variant system, the wires also have inductance.  I suspect that both the inductance and resistance are low enough in this case that they don't matter, but without numbers I can't say.  I think we can agree that the oscilloscopes read what they do because they are reading more or less the voltage drop across each resistor that results from the induced current.  Without examining Lewin's apparatus or experimenting myself, I couldn't say exactly how that was achieved.  A lot of the attempted explanations and experiments that have been shown regarding this seem as flawed as the original, but I'm pretty sure the answer is simply that there is another layer or two of complexity beyond the simple path-dependence that Lewin was demonstrating. 
Title: Re: #562 – Electroboom!
Post by: bdunham7 on November 07, 2021, 05:29:24 pm
Yes I did and they still do not add up to zero.

So this is why I am so reluctant to wade into this argument--I can't figure out what the dispute is.  It seems that the issue of why the oscilloscopes react the way they do in the example and 'voltages adding up' are somehow conflated? 

As for voltages adding up to zero, I don't know why anyone would think that they should.  So lets make a theoretical loop out of 100 1K resistors and apply a changing magnetic field strong enough to induce a 1mA current.  Now put 100 cheap voltmeters on each resistor (or measure the voltage thermally or whatever suits the imagination) and they should all read 1 volt, right?  Now pick a point and count around the circle, does it add up to zero or 100?  Why is this a question?  Or am I missing the point? 
Title: Re: #562 – Electroboom!
Post by: rfeecs on November 07, 2021, 05:38:48 pm
Wow.  The argument that never dies.

There (at least) two ways of modeling this:

Maxwell's equations model the electric and magnetic vector fields at every point in space.

Circuit theory uses a lumped circuit to model the voltages and currents.  KVL and KCL always hold in lumped circuit models.

Here are some great videos that explain it all:

https://www.youtube.com/watch?v=OmlnGei1xo8&t=2s (https://www.youtube.com/watch?v=OmlnGei1xo8&t=2s)

https://www.youtube.com/watch?v=u6ud7JD0fV4&t=2s (https://www.youtube.com/watch?v=u6ud7JD0fV4&t=2s)
Title: Re: #562 – Electroboom!
Post by: bdunham7 on November 07, 2021, 05:46:18 pm
BS must be pointed out and identified every time it shows up! 

Perhaps, but it should be done with cogent and persuasive argument.  I'm not sure that is always happening here.  We could start by clarifying what we agree and disagree on as that seems to be not clear to me in this case.
Title: Re: #562 – Electroboom!
Post by: thinkfat on November 07, 2021, 06:09:08 pm
Please, don't say "a current is induced". Maxwell-Faraday clearly tells us that there is an electric field resulting from the changing magnetic flux. Current is what results out of Ohms law.
Title: Re: #562 – Electroboom!
Post by: bdunham7 on November 07, 2021, 07:07:30 pm
Please, don't say "a current is induced". Maxwell-Faraday clearly tells us that there is an electric field resulting from the changing magnetic flux. Current is what results out of Ohms law.

Yeah, OK in this case where everything is being discussed and hotly disputed perhaps I should break that down into two steps.  But 'inducing a current' is hardly an unusual way of expressing the concept in general.
Title: Re: #562 – Electroboom!
Post by: bsfeechannel on November 08, 2021, 12:35:32 am
That's three lines of blather that doesn't answer my question.

The three lines of blather is not your answer. It's just a comment. You can ignore it if you will.

Quote
You're telling me that the display of my oscilloscope depends on not only how I connect it, but where I physically place it.

I'm not telling you. Nature is.

Quote
Although that may be true at some very minor level due to interference and other effects, it is nonsense here.

What you call the study of this very minor level interference and other effects is what is called the whole electronics engineering degree. And yes, it is difficult to understand.

Quote
Suppose I had long cables on the two scopes and swapped their physical positions left and right, well out of reach of any magnetic field from the experiment.  Now they read the other way?

That depends. With which of the resistors do your scope probes form a loop?

Quote
How about if I take the leads out perpendicularly and use two channels of one scope?
 

Perpendicularly to what? We are talking about a three dimensional space.

Quote
How about if I just use one scope?

You mean to measure each voltage separately? Please clarify.

Quote
These are just simple thought experiments that seem to me to reduce the experiment as claimed to an absurdity. I know some people have looked at this experimentally and I have no comment on those arguments because I haven't looked at them closely.

The problem with electromagnetism is that to have even a superficial understanding of it you have to go down a very deep rabbit hole, which most people don't care to do.

Quote
OK, if you are depending on the physical layout of the test leads, rather than the ultimate location of the oscilloscope (a much less ridiculous choice) than you need to reexamine the statement that there is no flux through that outer loop.

To be rigorous, you have to consider the position of the scope too. If it is inside the varying field, things change.

Quote
In order for the solenoid to induce a current in the loop, there has to be a net flux change inside the loop, but those flux lines have to eventually wrap around and go back to their opposite pole.  If they do that inside the loop, then they cancel out the net flux.  If they do that anywhere outside the loop, then your 'no varying magnetic field in the loop' becomes very questionable and would need to be measured by making another loop as physically close as possible but with a separate resistor not connected to the inner loop. 

Very pertinent question. Lewin took that precaution and demonstrated both theoretically and experimentally that the magnetic field outside the solenoid, where its length is much grater than its diameter, is negligible. See his lecture about it [ https://youtu.be/MXuZ1SRjpqk ]. So, from a practical point of view, the field outside the solenoid can be considered zero.

Quote
Now if you look at how most actual voltage measuring instruments work, whether they are an analog meter or oscilloscope, they measure the difference in absolute potential across their two input terminals.  They can do this either by reacting to electric fields directly or by allowing a small amount of current to flow.  In other words, the ideal voltmeter, however it works, reacts just like my hypothetical electrometer pair.   The voltmeter doesn't actually care about path dependence or anything else in the DUT, just about the potentials presented at its inputs.  It just measures the difference between two scalar quantities.

You are assuming your meter doesn't care about path dependence, but numerous experiments show otherwise. In Lewin's experiment, R1 and R2 are subject to electric fields of different intensities (assuming the resistors have the same size). And they can even be calculated. That's why the meters are showing different voltages (assuming there's no varying magnetic field in the loop made by the meter, the probes and the DUT).

If R1 and R2 were connected in parallel and attached to a battery, the voltages would be the same no matter what, because they would be subject to electric fields of the same intensity.

Stop for a moment and think. The voltages between points A and D in Lewin's experiment cannot be the same for the two resistors, otherwise they would have different currents flowing through them, which is impossible.


Quote
Since this a time-variant system, the wires also have inductance.  I suspect that both the inductance and resistance are low enough in this case that they don't matter, but without numbers I can't say.  I think we can agree that the oscilloscopes read what they do because they are reading more or less the voltage drop across each resistor that results from the induced current.  Without examining Lewin's apparatus or experimenting myself, I couldn't say exactly how that was achieved.  A lot of the attempted explanations and experiments that have been shown regarding this seem as flawed as the original, but I'm pretty sure the answer is simply that there is another layer or two of complexity beyond the simple path-dependence that Lewin was demonstrating.

Mehdi's experiment was spot on. His conclusion not quite.
Title: Re: #562 – Electroboom!
Post by: bsfeechannel on November 08, 2021, 01:48:08 am
Wow.  The argument that never dies.

Every trade has its fair share of myths and in all cases they perpetuate because they are uttered by popularizers. We can't be naïve to think that we will manage to reverse the spreading of those myths, but I think it is always important to debunk them when necessary, so people can see the truth and decide if they will accept it or not.

Quote
There (at least) two ways of modeling this:

Maxwell's equations model the electric and magnetic vector fields at every point in space.

Circuit theory uses a lumped circuit to model the voltages and currents.  KVL and KCL always hold in lumped circuit models.

Lewin's circuit is simply unlumpable, because you cannot confine the varying magnetic field somewhere outside the path of the circuit.

Quote
Here are some great videos that explain it all:

Great. Indeed. The theory, the experiments and even the numerical simulations agree and show that KVL doesn't hold under a varying magnetic field. I saw the comments for the first video and I was glad to see that people repeated the experiment, checked the theory and confirmed that this myth is what it is: a myth.

Some even are casting doubts about Mehdi's integrity, which is a good thing. In science the only word of authority is that of nature's.
Title: Re: #562 – Electroboom!
Post by: bsfeechannel on November 08, 2021, 01:52:27 am
I started writing a detailed response. But I think I let the posts of @bsfeechannel just stand on their own. I mean, just look at the calculations.

What is wrong with the calculations?
Title: Re: #562 – Electroboom!
Post by: bsfeechannel on November 08, 2021, 02:21:54 am
But when electroboom did the experiment he accounted for many of this things... and KVL worked as expected:

Nope. Lewin used the theory to predict that the two resistors would have different voltages. Mehdi' experiment showed that Lewin's prediction was right. Mehdi admitted that explicitly.

Then Mehdi said that although the experiment matched exactly what Lewin predicted, he, Mehdi, thought that Lewin's was wrong.

This is a case of doublethink, where someone simultaneously accepts two mutually contradictory beliefs as correct, often in contravention to one's own sense of reality.

Quote
One more thing:  bsfeechannel,  I called you out and you didn't deliver. 

Not interested in your piss contest.

Quote
You have no idea of what you are talking about.  Talking to you is exactly like talking to a flat earther or an antivaxer.  So I decided to go thunderf00t on you call out all your BS.

Be my guest.

Title: Re: #562 – Electroboom!
Post by: jesuscf on November 08, 2021, 02:51:12 am
But when electroboom did the experiment he accounted for many of this things... and KVL worked as expected:

Nope. Lewin used the theory to predict that the two resistors would have different voltages. Mehdi' experiment showed that Lewin's prediction was right. Mehdi admitted that explicitly.

Then Mehdi said that although the experiment matched exactly what Lewin predicted, he, Mehdi, thought that Lewin's was wrong.

This is a case of doublethink, where someone simultaneously accepts two mutually contradictory beliefs as correct, often in contravention to one's own sense of reality.

Quote
One more thing:  bsfeechannel,  I called you out and you didn't deliver. 

Not interested in your piss contest.

Quote
You have no idea of what you are talking about.  Talking to you is exactly like talking to a flat earther or an antivaxer.  So I decided to go thunderf00t on you call out all your BS.

Be my guest.

Hey bsfeechannel, do you have an oscilloscope?   Oh wait, do you even know how to use an oscilloscope?  Do you have any idea of what is the deal with oscilloscope probes?  Did you have an instrumentation course at the university?  I know the answers already: no, no, no, and no.  Because you are a fake!   Your knowledge of these very elemental subjects is minimal at best and shows in your ridiculous comments.

For everyone else reading, who are you going to trust?  Electroboom whom uses multiple oscilloscopes in a regular basis or Lewin that had no idea on what is going on with the oscilloscope and often calls it a voltmeter?



 
Title: Re: #562 – Electroboom!
Post by: bdunham7 on November 08, 2021, 03:13:57 am
For everyone else reading, who are you going to trust?  Electroboom whom uses multiple oscilloscopes in a regular basis or Lewin that had no idea on what is going on with the oscilloscope and often calls it a voltmeter?

Maybe lighten up a bit?  An oscilloscope IS a voltmeter!  And 'trust' isn't the issue, I'm (maybe) interested in what is going on, not comparing the trustworthiness of two rather flamboyant showmen.
Title: Re: #562 – Electroboom!
Post by: jesuscf on November 08, 2021, 03:38:26 am
For everyone else reading, who are you going to trust?  Electroboom whom uses multiple oscilloscopes in a regular basis or Lewin that had no idea on what is going on with the oscilloscope and often calls it a voltmeter?

Maybe lighten up a bit?  An oscilloscope IS a voltmeter!  And 'trust' isn't the issue, I'm (maybe) interested in what is going on, not comparing the trustworthiness of two rather flamboyant showmen.

Yes, and oscilloscope measures voltage, but in the profession we don't call it a voltmeter.  If I say: "I used the voltmeter" to measure something, the first thing that comes to your mind is not an oscilloscope for sure!  If you are old enough, not even a multimeter comes to your mind when somebody says 'voltmeter'.  As for lighten up a bit: NO!  Charlatans like bsfeechannel must be confronted.
Title: Re: #562 – Electroboom!
Post by: bdunham7 on November 08, 2021, 03:48:22 am
Very pertinent question. Lewin took that precaution and demonstrated both theoretically and experimentally that the magnetic field outside the solenoid, where its length is much grater than its diameter, is negligible. See his lecture about it [ https://youtu.be/MXuZ1SRjpqk ]. So, from a practical point of view, the field outside the solenoid can be considered zero.

I'll have to look at that. 

Quote
You are assuming your meter doesn't care about path dependence, but numerous experiments show otherwise. In Lewin's experiment, R1 and R2 are subject to electric fields of different intensities (assuming the resistors have the same size). And they can even be calculated. That's why the meters are showing different voltages (assuming there's no varying magnetic field in the loop made by the meter, the probes and the DUT).

If R1 and R2 were connected in parallel and attached to a battery, the voltages would be the same no matter what, because they would be subject to electric fields of the same intensity.

Stop for a moment and think. The voltages between points A and D in Lewin's experiment cannot be the same for the two resistors, otherwise they would have different currents flowing through them, which is impossible.

Without doing a blow by blow response, I'll point out again that there seems to be a lack of clarity about what we agree on and what is in question.

I don't think that there is any dispute that the two resistors will have voltages across them of opposite sign and different magnitude.  Is that point in contention?

It seems fairly clear to me, and it seems to be fairly explicitly admitted in the second video from MIT (perhaps where Lewin got the idea) posted by rfeecs that the 'path dependence' vis a vis the voltmeters is not the path within the loop, but rather the positioning of the test leads going to the voltmeter.  If you want to convince me otherwise, you would have to address my absolute voltmeter thought experiment.  As far as positioning the scope, imagine a two-channel scope positioned well above the apparatus, not to the R1 or R2 side but right between them, far enough away that any electric or magnetic field is truly negligible.  Now everything is identical except the test leads going to the apparatus and their position is the only variable.

As for the rest of the issues and whether the voltages at the resistors must equal the voltage at the test points simply because wires are 'dead shorts' is something that I think involves--as I said previously--another layer of complexity.  'Voltage ambiguity' isn't really a workable concept, thus much of the consternation among the people trying (and mostly failing!) to explain this.  The first video posted by rfeecs seems to bring some concepts in worth looking at, but I can't say I've really considered it enough to comment. 

One thing to consider is your repeated assertion that you can't have a voltage differential in the wires because they have low resistance.  So are there any conditions where that statement is not true?  Are there conditions where a straight piece of wire will have a voltage differential from end-to-end?
Title: Re: #562 – Electroboom!
Post by: bdunham7 on November 08, 2021, 03:59:38 am
NO!  Charlatans like bsfeechannel must be confronted.

Well then help me understand what goes on in the experiment we're debating.

Title: Re: #562 – Electroboom!
Post by: bsfeechannel on November 08, 2021, 07:21:00 am
It seems fairly clear to me, and it seems to be fairly explicitly admitted in the second video from MIT (perhaps where Lewin got the idea) posted by rfeecs that the 'path dependence' vis a vis the voltmeters is not the path within the loop, but rather the positioning of the test leads going to the voltmeter.

R1 and R2 establish two different paths. The positioning of the probes is a consequence of not letting the varying magnetic field induce other voltages in the loop that you will have when the probes are attached to the resistors. So there you have your path dependence.

Quote
As far as positioning the scope, imagine a two-channel scope positioned well above the apparatus, not to the R1 or R2 side but right between them, far enough away that any electric or magnetic field is truly negligible.  Now everything is identical except the test leads going to the apparatus and their position is the only variable.

Got it. And that's a very good question. If you position your scope well above the solenoid of instance, with the probes at a right angle with the plane of the circuit, the loop formed by the resistor and the probes will be traversed by the magnetic field. The voltages will be identical. But they will not be the EMF. They'll be of a value between the voltage of both resistors.

Quote
As for the rest of the issues and whether the voltages at the resistors must equal the voltage at the test points simply because wires are 'dead shorts' is something that I think involves--as I said previously--another layer of complexity.

You can get rid of the dead shorts if you attach two resistive wires of different resistances in the same loop configuration. The result will obviously the same.

Quote
'Voltage ambiguity' isn't really a workable concept, thus much of the consternation among the people trying (and mostly failing!) to explain this.

It is not an easy phenomenon to explain because it is unintuitive.

Quote
One thing to consider is your repeated assertion that you can't have a voltage differential in the wires because they have low resistance.  So are there any conditions where that statement is not true? Are there conditions where a straight piece of wire will have a voltage differential from end-to-end?
 
Yes. When the wires are moving perpendicular to the magnetic field.

But in Lewin's circuit the wires are static in relation to the frame of reference. So the voltage across the wires will obey ohms law.

One misconception is that since the secondary of a transformer is just a wire, and I can measure a voltage when I attach a meter to the terminals, it is the wire that is generating the voltage.

What the wire is doing is to set a boundary condition around the loop where the rotational electric field will be forbidden to exist. This will concentrate the field between the terminals. What you are measuring is a voltage produced by an electric field that only exists between the terminals. It doesn't exist in the wire. So it cannot produce a voltage across it.

When you attach a load to the secondary and current flows, then you'll have an electric field inside the wire that will produce a voltage which will be proportional the current times the resistance of the wire. This electric field will have to be discounted from the field at the terminals, because the integral of the field around the loop is proportional to the derivative of the magnetic field ( I.e. the field in the wire plus the field in the load must add up to the induced EMF no matter what).
Title: Re: #562 – Electroboom!
Post by: thinkfat on November 08, 2021, 08:09:49 am
I just cannot let this slide:


Quote
One thing to consider is your repeated assertion that you can't have a voltage differential in the wires because they have low resistance.  So are there any conditions where that statement is not true? Are there conditions where a straight piece of wire will have a voltage differential from end-to-end?
 
Yes. When the wires are moving perpendicular to the magnetic field.

But in Lewin's circuit the wires are static in relation to the frame of reference. So the voltage across the wires will obey ohms law.

This is plain nonsense. It doesn't matter if the wires are moving or not. You will have a voltage whenever the magnetic flux changes. There is no term in the Maxwell-Faraday equation that has any component of spatial displacement.

Quote
One misconception is that since the secondary of a transformer is just a wire, and I can measure a voltage when I attach a meter to the terminals, it is the wire that is generating the voltage.

What the wire is doing is to set a boundary condition around the loop where the rotational electric field will be forbidden to exist. This will concentrate the field between the terminals. What you are measuring is a voltage produced by an electric field that only exists between the terminals. It doesn't exist in the wire. So it cannot produce a voltage across it.

But if there is no field "in the wire", how does a Variac work? Or any tapped transformer? Surely the voltage at the tap cannot be due to Ohms law if there's no current flowing? And if you connect the outer terminals of the transformer to a load, does that mean the field inside the wire collapses? Then the voltage at the tap would also collapse. But quite obviously, this isn't the case.

Quote
When you attach a load to the secondary and current flows, then you'll have an electric field inside the wire that will produce a voltage which will be proportional the current times the resistance of the wire. This electric field will have to be discounted from the field at the terminals, because the integral of the field around the loop is proportional to the derivative of the magnetic field ( I.e. the field in the wire plus the field in the load must add up to the induced EMF no matter what).

See above to understand why this cannot be a proper explanation.
Title: Re: #562 – Electroboom!
Post by: bsfeechannel on November 08, 2021, 09:30:48 am
This is plain nonsense. It doesn't matter if the wires are moving or not. You will have a voltage whenever the magnetic flux changes. There is no term in the Maxwell-Faraday equation that has any component of spatial displacement.

Nope. You'll not have an electric field inside the wire if it is static with no current flowing through it. If it is moving, the charges in the conductor will experience the Lorentz force and you'll have an electric field inside it.

Quote
But if there is no field "in the wire", how does a Variac work? Or any tapped transformer? Surely the voltage at the tap cannot be due to Ohms law if there's no current flowing? And if you connect the outer terminals of the transformer to a load, does that mean the field inside the wire collapses? Then the voltage at the tap would also collapse. But quite obviously, this isn't the case.

A transformer winding is just a series of loops connected in series. The electric field resides in the space between each turn.

Quote
See above to understand why this cannot be a proper explanation.

See above to understand what is really going on in a transformer.
Title: Re: #562 – Electroboom!
Post by: jesuscf on November 08, 2021, 09:49:55 am
A transformer winding is just a series of loops connected in series. The electric field resides in the space between each turn.

WHAT!  This quote needs to be preserved for posterity!  :-DD
Title: Re: #562 – Electroboom!
Post by: tszaboo on November 08, 2021, 09:54:50 am
KVL and KCL are special cases of the Maxwell's equations with some assumptions.
Maxwell's equations always hold, if we assume if there is no quantum physics shenanigans.
KVL always hold, if we assume a few things. How to derive it, what to assume? Google it, or open your university books, because in every half decent university they teach this shit, and I am amazed that people still talk about this. I am not surprised if a physics professor in some US university would go ahead and "discover on his own" that KVL suddenly doesn't hold. What I'm amazed that they let this professor teach electronics to students, and had nobody around tell him that "Da! It is in the coursework for the engineers, haven't you read it?"
Title: Re: #562 – Electroboom!
Post by: thinkfat on November 08, 2021, 11:51:46 am
A transformer winding is just a series of loops connected in series. The electric field resides in the space between each turn.

WHAT!  This quote needs to be preserved for posterity!  :-DD

Yeah, it's getting more and more interesting with each new post. Now only Lorentz' force creates an electric field but not a changing magnetic flux (despite Maxwell-Faraday telling the opposite), and now you only have an electric field in the space between the transformer windings. I really wonder how antennas ever worked.

Besides, this contradicts his own statement:
Quote
What the wire is doing is to set a boundary condition around the loop where the rotational electric field will be forbidden to exist. This will concentrate the field between the terminals. What you are measuring is a voltage produced by an electric field that only exists between the terminals. It doesn't exist in the wire. So it cannot produce a voltage across it.

If the field "concentrates between the terminals" it should be nowhere else. Not between the windings either, because clearly they are connected to each other and thus there should not be a "boundary condition" where the "rotational electric field" (what?) is "forbidden to exist".

Next argument will be that the field only exists when you connect a meter to measure it, which would violate causality.
Title: Re: #562 – Electroboom!
Post by: bsfeechannel on November 08, 2021, 01:15:10 pm
Yeah, it's getting more and more interesting with each new post.

Of course it is. Electromagnetism is interesting.

Quote
Now only Lorentz' force creates an electric field

Nope. Inside a moving wire perpendicular to a magnetic field, the Lorentz force will displace the charges inside a conductor. Since the magnetic field is static, it doesn't induce any other electric field. So you will have a net electric field inside the conductor produced by the charge displacement.

Quote
but not a changing magnetic flux (despite Maxwell-Faraday telling the opposite),

A changing magnetic flux will induce a voltage inside the wire, however the displacement of charges will counteract this field, resulting in a net electric field inside the conductor with zero intensity.

Quote
and now you only have an electric field in the space between the transformer windings.

Precisely. One of the main causes of failure in transformers is the breakdown of insulation between adjacent turns. This is because the field resides between the turns, and produces arching. That would not be possible if the electric field was inside the wire.

Quote
I really wonder how antennas ever worked.

It is easy. Study electromagnetism for real. It will take you years to master the trade. But its worthwhile. And forget those bloggers that pretend to teach you this complicated matter in a 12-minute video plagued with all kinds of pseudo-scientific claims.

Quote
Besides, this contradicts his own statement:

If the field "concentrates between the terminals" it should be nowhere else. Not between the windings either, because clearly they are connected to each other and thus there should not be a "boundary condition" where the "rotational electric field" (what?) is "forbidden to exist".

I said concentrate, not is absent elsewhere. This is very easy to understand. Look at the picture below. It is the "front view" of a turn. As you extend the left terminal on top of the right terminal to wind the second turn, you see that the electric field accommodates conveniently between the turns. It is not shown in the picture, but assume that the wires have an insulation.

(https://www.eevblog.com/forum/amphour/562-electroboom!/?action=dlattach;attach=1318763;image)

And it is this property that is used for instance with a Variac. The electric field between the turns add up between the terminal of the first turn until the tap. And that's how you get your variable voltage. I'm omitting the resistive wiper that gets in contact with several adjacent turns, but you get the idea.

(https://www.eevblog.com/forum/amphour/562-electroboom!/?action=dlattach;attach=1318769;image)

Quote
Next argument will be that the field only exists when you connect a meter to measure it, which would violate causality.

Your comment is violating causality, since I haven't seen yet what will be your next expression of absolute lack of understanding of electromagnetism.
Title: Re: #562 – Electroboom!
Post by: thinkfat on November 08, 2021, 02:30:12 pm
So, let there be a single loop of wire and subject it to the same changing magnetic flux as the transformer in your example above. Let d be the length of wire needed for a single winding in said transformer. What is the voltage between two points of distance d on the single loop of wire?

Title: Re: #562 – Electroboom!
Post by: bdunham7 on November 08, 2021, 03:20:45 pm
Yes. When the wires are moving perpendicular to the magnetic field.

But in Lewin's circuit the wires are static in relation to the frame of reference. So the voltage across the wires will obey ohms law.

OK, so let's imagine I connect my scope to the ends of the top wire (A1 and A2 IIRC) and I repeat the experiment with the full loop in place and then with the resistors disconnected so that I only have the wire with open ends.  Then I do the same routine with my magic electroscopes.  What do I see?
Title: Re: #562 – Electroboom!
Post by: Sredni on November 08, 2021, 11:29:05 pm
I no longer post here, but...

https://electronics.stackexchange.com/questions/506590/can-two-voltmeters-connected-to-the-same-terminals-show-different-values-circui?noredirect=1&lq=1
https://electronics.stackexchange.com/questions/551519/assigning-a-notion-of-voltage-even-when-there-is-a-changing-magnetic-field/551548#551548
https://electronics.stackexchange.com/questions/551244/what-would-a-voltmeter-measure-if-you-had-an-electromotive-force-generated-by-a/551428#551428

there are slight difference with the terminology, and I need to fix (at least) a missing sign but the gist is there.
For people who want to see: the key is in the fields.

Title: Re: #562 – Electroboom!
Post by: jesuscf on November 09, 2021, 05:14:00 pm
KVL and KCL are special cases of the Maxwell's equations with some assumptions.
Maxwell's equations always hold, if we assume if there is no quantum physics shenanigans.
KVL always hold, if we assume a few things. How to derive it, what to assume? Google it, or open your university books, because in every half decent university they teach this shit, and I am amazed that people still talk about this. I am not surprised if a physics professor in some US university would go ahead and "discover on his own" that KVL suddenly doesn't hold. What I'm amazed that they let this professor teach electronics to students, and had nobody around tell him that "Da! It is in the coursework for the engineers, haven't you read it?"

This is by far the best summary on this 'debate' I've read!   I got tired of pointing out the literature on the debated subject and be dismissed out of hand by un-educated blockheads.  Even Lewin at some point in one of his videos says that every book in the matter is wrong and that is a "crime against humanity" or something along those lines not to teach his BS.  Somehow Lewin believes that he is better than hundred (if not thousands) of experts in the field!
Title: Re: #562 – Electroboom!
Post by: bsfeechannel on November 09, 2021, 07:59:06 pm
I got tired of pointing out the literature on the debated subject and be dismissed out of hand by un-educated blockheads.

You got tired because you don't understand the theory and you dismiss the evidence that contradicts your claims.
Title: Re: #562 – Electroboom!
Post by: jesuscf on November 10, 2021, 12:19:03 am
I got tired of pointing out the literature on the debated subject and be dismissed out of hand by un-educated blockheads.

You got tired because you don't understand the theory and you dismiss the evidence that contradicts your claims.

Says the person with no credentials that dropped this jewel:

"A transformer winding is just a series of loops connected in series. The electric field resides in the space between each turn."

Title: Re: #562 – Electroboom!
Post by: bsfeechannel on November 10, 2021, 12:54:03 am
Let me summarize the discussion for you:

I trust Mehdi. Lewin is full of BS and everyone that shows any practical evidence that Mehdi's claims are false and Lewin's claim correspond to reality is a blockhead.

Only irrational arguments. No scientific or engineering arguments and no evidence.

You contribute nothing to any discussion.
Title: Re: #562 – Electroboom!
Post by: jesuscf on November 10, 2021, 01:22:07 am
Let me summarize the discussion for you:

I trust Mehdi. Lewin is full of BS and everyone that shows any practical evidence that Mehdi's claims are false and Lewin's claim correspond to reality is a blockhead.

Only irrational arguments. No scientific or engineering arguments and no evidence.

You contribute nothing to any discussion.

Once again... Says the person with no credentials that dropped this jewel:

"A transformer winding is just a series of loops connected in series. The electric field resides in the space between each turn."
Title: Re: #562 – Electroboom!
Post by: Sredni on November 10, 2021, 01:30:23 am
Once again... Says the person with no credentials that dropped this jewel:

"A transformer winding is just a series of loops connected in series. The electric field resides in the space between each turn."

Well, but that is true.
You can read and learn, if you want.
http://web.mit.edu/6.013_book/www/chapter10/10.1.html (http://web.mit.edu/6.013_book/www/chapter10/10.1.html)
Title: Re: #562 – Electroboom!
Post by: jesuscf on November 10, 2021, 01:37:47 am
Once again... Says the person with no credentials that dropped this jewel:

"A transformer winding is just a series of loops connected in series. The electric field resides in the space between each turn."

Well, but that is true.
You can read and learn, if you want.
http://web.mit.edu/6.013_book/www/chapter10/10.1.html (http://web.mit.edu/6.013_book/www/chapter10/10.1.html)

So if there is only one loop, no electric field?
Title: Re: #562 – Electroboom!
Post by: Sredni on November 10, 2021, 01:56:14 am
You can read and learn, if you want.
http://web.mit.edu/6.013_book/www/chapter10/10.1.html (http://web.mit.edu/6.013_book/www/chapter10/10.1.html)

So if there is only one loop, no electric field?

Oh, dear... you did not even open the link, did you?
If there is only one loop the field is still in the space. This time is mostly between the terminals.
There literally is a picture of the field for a single turn coil at that link.

I should have written "You can learn, if you can".
Title: Re: #562 – Electroboom!
Post by: bdunham7 on November 10, 2021, 02:28:32 am
Once again... Says the person with no credentials that dropped this jewel:

"A transformer winding is just a series of loops connected in series. The electric field resides in the space between each turn."

What is the issue with that?
Title: Re: #562 – Electroboom!
Post by: bsfeechannel on November 10, 2021, 02:55:33 am
The issue is that he thinks that the voltage produced at the terminals of a loop of wire under a varying magnetic field is located inside the wire.

He doesn't know that the voltage is produced in the gap between the terminals. Typical KVLer, who thinks there must be some kind of battery inside the wire pulling the charges along.

He also doesn't understand that a coil of wire is just a bunch of loops in series. Go figure.
Title: Re: #562 – Electroboom!
Post by: jesuscf on November 10, 2021, 03:30:55 am
So which one is the correct one?  This one?

He doesn't know that the voltage is produced in the gap between the terminals. Typical KVLer, who thinks there must be some kind of battery inside the wire pulling the charges along.

Or this one?

A transformer winding is just a series of loops connected in series. The electric field resides in the space between each turn.




Title: Re: #562 – Electroboom!
Post by: jesuscf on November 10, 2021, 03:45:50 am
Once again... Says the person with no credentials that dropped this jewel:

"A transformer winding is just a series of loops connected in series. The electric field resides in the space between each turn."

Well, but that is true.
You can read and learn, if you want.
http://web.mit.edu/6.013_book/www/chapter10/10.1.html (http://web.mit.edu/6.013_book/www/chapter10/10.1.html)

There are a lot of idealizations in the link provided: "Magnetoquasistatic Electric Fields in Systems of Perfect Conductors".  I hope one day we have perfect conductors!
Title: Re: #562 – Electroboom!
Post by: bsfeechannel on November 10, 2021, 05:37:55 am
So which one is the correct one?  This one?

He doesn't know that the voltage is produced in the gap between the terminals. Typical KVLer, who thinks there must be some kind of battery inside the wire pulling the charges along.

Or this one?

A transformer winding is just a series of loops connected in series. The electric field resides in the space between each turn.

Both are correct. When you have just one loop, the electric field resides between the terminals. When you overlap the terminals to start the second loop on top of the first, the electric field accommodates to reside between the turns, as I showed you here (https://www.eevblog.com/forum/amphour/562-electroboom!/msg3799322/#msg3799322).
Title: Re: #562 – Electroboom!
Post by: bsfeechannel on November 10, 2021, 05:45:04 am
I hope one day we have perfect conductors!

Haven't you heard? It's called superconductors. And any model is made of ideal components.
Title: Re: #562 – Electroboom!
Post by: thinkfat on November 10, 2021, 11:22:40 am
That's a lot of stuff to digest. But one aspect seems obvious, no matter which "cult" you adhere to: In order to actually observe the path dependence of the voltage between A and D, you need to probe in a way that allows "selecting" the path you want to observe. Watching the MIT video that was linked here by @rfeecs, it is obvious that this selection is done only by the layout of the probe wires. If I understood the reasoning correctly, the selector is not the magnetic flux, because the paths that lead to the instrument are outside of it. What is also clear from the MIT video is that there is no dependence on how closely the probe wires follow the loop, any path on the "same side" as the voltage to be observed is OK. "Same side" seems to be "geometrically on the same side relative to the magnetic flux". That means the chosen path must never encompass any area that is inside the magnetic flux. As long as you stick to that, you're good.

Apart from that the explanation in the video is pretty clear, I think: The paths C1 and C2 are not encompassing the magnetic flux, so the line integral over all electric fields along C1 or C2 sum up to 0. The only observable voltage there is whatever the resistor in the path drops. That is true even when you argue that C1 and C2 are still subjected to the magnetic flux change, because they don't encompass it. If you look at the rotational electric field, any path you walk that doesn't go "around the center" of the field sums up to 0.

What I still have a hard time coming to terms with is that there is no voltage contribution by the other possible path through the second resistor, and no contribution from the loop wire itself. But I think the other video posted by @rfeecs (from Silicon Soup) shows that there is in fact a contribution by the other possible path but as this path also captures the EMF (because it encompasses the magnetic flux), the sum of all voltages is again only the voltage dropped by the resistor in the "chosen" path.

Now, why is there no voltage contribution by the wire itself, this is the most counter-intuitive aspect and the core of the claim that KVL doesn't hold. "Silicon Soup" explains that the rotational electric field caused by the changing magnetic flux and the electric field inside the wire caused by charge separation cancel each other out and the net electric field "in the wire" is 0. D'oh! So it's there but you cannot observe it because there's another electric field of same "strength" just in the "opposite direction". I put that in quotes not to express doubt but to denote that I understand that it's a simplification and we're talking about vectors here. But it is still hard to stomach because you can observe the effect of the charge separation through the voltage measurable at the terminals.

So, is it really "not there" or is it just difficult to find a path along which it would be observable? If there is no field, then what did Mabilde measure in his setup? Because his setup modeled after the McDonald paper shows 0.25V across a quarter segment of the loop, and that is obviously way to much for just Ohms law in action.

I'm still not done thinking this through but at this point I have an inkling that when I'm done, I will have to apologize to @bsfeechannel :palm:

What I'm still chewing on is the Variac example I brought up and the claim that there is an electric field in the wire if it's not stationary, because then it's Lorentz' force and not EMF that is causing the charge separation. An experiment I'd like to see is what happens when you have e.g. a rotating magnet instead of a solenoid, iow a generator instead of a transformer.

But it's not a problem to be wrong, just being stubborn.

PS: after some more headscratching, I think I understood why you cannot see any voltage across the loop wire: every path that involves the piece of wire between the resistors and that does not go around the center of the electric field will sum up to 0. Therefore, if you just naively connect you voltmeter probes to the points "A1" and "A2", that path will only observe the voltage across the wire according to Mr. Ohm. No need to argue with electric charge being counteracted by another electric field.
Title: Re: #562 – Electroboom!
Post by: Sredni on November 10, 2021, 02:18:53 pm
That's a lot of stuff to digest. But one aspect seems obvious, no matter which "cult" you adhere to: In order to actually observe the path dependence of the voltage between A and D, you need to probe in a way that allows "selecting" the path you want to observe.

It's very simple. If the measurement loop formed by voltmeter, probes, and that branch DOES NOT enclose (cut) any (appreciable) variable magnetic flux then the electric fields obeys E = -grad V without the additional term dB/dt and can be seen as conservative. If we limit ourselves to planar circuits to make it easier to see, all paths in the area delimited by that measurement loop give the same value for the path integral of E, so voltage between any two points inside that area is the same no matter how you choose to join them (it only depends on the endpoints): along the measured branch, in the space between them, or along the probes through the voltmeter. I.E. the value shown by the voltmeter is also the voltage computed on the path along the branch, and even across it if you stay in the space that is magnetic-free.

Quote
Watching the MIT video that was linked here by @rfeecs, it is obvious that this selection is done only by the layout of the probe wires. If I understood the reasoning correctly, the selector is not the magnetic flux, because the paths that lead to the instrument are outside of it.

The selector is the presence or better the absence of magnetic flux inside the measurement loop.

Quote
That means the chosen path must never encompass any area that is inside the magnetic flux.

Yes, but you need to think in terms of closed paths. When you put one voltmeter on the outside of the ring, it will form two loops: one with the nearest resistor - that does not enclose the dB/dt region, the other with the farthest resistor - that does enclose the dB/dt region.
The voltages between two points in the region of space delimited by the first loop are all equal, no matter the path - hence the reading on the voltmeter corresponds to the correct voltage of the nearest branch.
The other measurement loop is not reading the correct voltage of the farthest branch, though, because it is affected by the presence of the variable magnetic field linked.
The beauty of it is that [correct voltage of far branch] + [emf with correct sign] = [voltage read by multimeter]

Quote
What I still have a hard time coming to terms with is that there is no voltage contribution by the other possible path through the second resistor

Oh, but there is. The current flowing in the near resistor - the current that gives rise to the voltage you read on the the voltmeter - would not be there if not for the second resistor.

Quote
Now, why is there no voltage contribution by the wire itself, this is the most counter-intuitive aspect and the core of the claim that KVL doesn't hold. "Silicon Soup" explains that the rotational electric field caused by the changing magnetic flux and the electric field inside the wire caused by charge separation cancel each other out and the net electric field "in the wire" is 0. D'oh! So it's there but you cannot observe it because there's another electric field of same "strength" just in the "opposite direction". I put that in quotes not to express doubt but to denote that I understand that it's a simplification and we're talking about vectors here. But it is still hard to stomach because you can observe the effect of the charge separation through the voltage measurable at the terminals.

It is exactly that charge separation that produces the coloumbian field that obliterates the induced field in the wire. If the wire is a perfect conductor, then the total electric field inside is zero. If the wire is a real conductor with high conductivity such as copper then there is a tiny electric field of the order of a handful of microvolts per meter that is compatible with the local form of Ohm's law.

Quote
So, is it really "not there" or is it just difficult to find a path along which it would be observable?

It's not there because the electric field has been cancelled (exactly in a perfect conductor, almost entirely in a good conductor.)

Quote
If there is no field, then what did Mabilde measure in his setup? Because his setup modeled after the McDonald paper shows 0.25V across a quarter segment of the loop, and that is obviously way to much for just Ohms law in action.

Mabilde has put its probes INSIDE the variable magnetic field region. He is cutting flux on purpose to induce a voltage in his measurement loop so that it can cancel the contribute of the induced electric field. This leaves only the contribute of the coloumbian field that is a conservative field. It's a nice technique, but he does not understand that he is measuring a partial contribute only. The field the electrons in the wire and the space experience is the total field

Etot = Eind + Ecoul

He is measuring the effects of Ecoul alone which admit a scalar potential phi. Which is fine if you realize that phi alone is not sufficient to completely describe the system. You also need the vector potential A, as McDonald shows.
What McDonald did was to apply the Helmoltz decomposition of fields to the total electric field and then associate the scalar potential phi to Ecoul and the vector magnetic potential A to Eind.
What the KVLers understood is only half of it. They stopped at phi and thought: "see? the potential is uniquely defined" without realizing that such potential is referred to a part and not all of the electric field.

In one of my answers on EESE I quote a paragraph of Popovic & Popovic where the actual expression of voltage in the presence of a variable magnetic field is given.

Quote
I'm still not done thinking this through but at this point I have an inkling that when I'm done, I will have to apologize to @bsfeechannel :palm:

From time to time you read, on Lewin's YT channel, the posts of someone apologizing to him after realizing he has been right the whole time.

Quote
an electric field in the wire if it's not stationary,

Get hold of Purcell. It has pictures for that.

And for the variac... we know the field is in the space between turns, and that is what you measure with a voltmeter on the outside. But if you have trouble computing that, why don't you use the other measuring loop, the one following the conductor around the core? You just have to apply Faraday and the concept that there is no appreciable voltage drop in the conductor: [voltage measured outside] = [negligible voltage drop in the turns tapped] + [emf with correct sign multiplied by number of turns]

That also explains why you cannot measure partial turn voltage, but only integer multiples.

(Edit: corrected a sentence to make it clear which area, which loop, which points)
Title: Re: #562 – Electroboom!
Post by: Sredni on November 10, 2021, 02:52:17 pm
Once again... Says the person with no credentials that dropped this jewel:

"A transformer winding is just a series of loops connected in series. The electric field resides in the space between each turn."

Well, but that is true.
You can read and learn, if you want.
http://web.mit.edu/6.013_book/www/chapter10/10.1.html (http://web.mit.edu/6.013_book/www/chapter10/10.1.html)

There are a lot of idealizations in the link provided: "Magnetoquasistatic Electric Fields in Systems of Perfect Conductors".  I hope one day we have perfect conductors!

Magneto-quasistatics refers to the way electric and magnetic fields are changing. And it is the settings one uses for all non radiating circuits. Transformers operate in a magneto-quasistatic settings. Antennas do not.
That leaves us with the perfect conductors. Which are used everywhere in physics and electronics to simplify computations - such as... all wires in a circuit schematic are considered perfect conductors. Do you have superconducting ceramics at nearly 0 K in your cellphone charger? I don't think so, and yet I bet the SMPS has been modeled in a SPICE-like program assuming perfect conductors, and modelling losses via lumped resistors.

But, anyway... What do you think would change in the Lewin ring if, instead of a perfect conductor, you had a highly conductive copper conductor?
I tell you what: almost nothing. And certainly nothing of relevance. The only difference is that, instead of zero electric field and zero voltage drop in the conductors, you will see an almost negligible electric field E = j /sigma_copper and an almost negligible voltage drop of a handful of microvolts. Against the hundreds of millivolts of drops at the resistors.
Title: Re: #562 – Electroboom!
Post by: bsfeechannel on November 10, 2021, 03:29:47 pm
"Same side" seems to be "geometrically on the same side relative to the magnetic flux". That means the chosen path must never encompass any area that is inside the magnetic flux. As long as you stick to that, you're good.

You got that right. It all boils down to geometry. Not electricity. Electricity works as usual.

Quote
I will have to apologize to @bsfeechannel :palm:

No worries.
Title: Re: #562 – Electroboom!
Post by: bsfeechannel on November 10, 2021, 03:58:36 pm
OK, so let's imagine I connect my scope to the ends of the top wire (A1 and A2 IIRC) and I repeat the experiment with the full loop in place and then with the resistors disconnected so that I only have the wire with open ends.  Then I do the same routine with my magic electroscopes.  What do I see?

I need to understand what is really bugging you. But before I ask you some questions, allow me to suggest a different analogy.

Mehdi said in this episode of the AmpHour that someone gave him an explanation full of math so he dismissed it. The advantage of math is that it is rigorous. It doesn't admit contradictions, so you are sure your reasoning has no flaws. However math is not very intuitive, so I can understand him.

An analogy is intuitive, but not rigorous. So, let's start with an analogy to give us some insight about what is going on. Then we leave the math as an exercise to cover for any loopholes.

Picture yourself in a boat on a river. This river has an accelerating stream of water at constant rate, which produces a constant force throughout the stream.

(https://www.eevblog.com/forum/amphour/562-electroboom!/?action=dlattach;attach=1320377;image)

Your boat experiences this force, but there's no friction so your boat can move perpendicularly to the stream without effort.

It doesn't matter if your boat chugs from D to A through path 1 or path 2. It'll experience the same force all along. You're path independent. Moreover, if you want to return from A to D, the force produced by the stream will take you there. No need to row your boat.

That's why we can say that point A has a positive potential in relation to D. Because if you are at point A you don't need to spend energy to get to point D. We can say that D has a negative potential in relation to A, because you'll have to overcome the force of the stream. If you go from point D to point A and return to point D again, the sum of the potentials will be zero.

Now your boat is in swirling accelerating waters. It is pretty obvious that if you go from point D to A trough path 1 you will get there effortlessly, but if you choose path 2 your up the creek and you'll need a paddle.

(https://www.eevblog.com/forum/amphour/562-electroboom!/?action=dlattach;attach=1320383;image)

You're now path dependent. We could say that A has a positive "potential" in relation to D, because you can go from A to D through path 2 without spending energy. But, hey, D also has a positive "potential" in relation to A, because you can choose path 1 and get there for free. But if in each case you choose the other path, the potentials are negative. What is the "potential" between A and D? Undefined. Its path dependent.

So if you calculate the sum of the potentials, from D to A through path 1 and from A to D through path 2, it will give you a positive number. You can go anti-clockwise if you will and you'll get a negative number. But NEVER zero.

This has nothing to do with electricity. This is a property associated with any force field. That's why I said we take at least two years in an electronics engineering degree studying with the due rigor all the implications of this kind of thing.

Alright. Now replace the boat with an electric charge, and the moving water with an electric field. Nothing changes. Moving a charge in a circle or circuit of any shape through a conservative field will give us a net zero potential. That's KVL in all its glory.

Moving a charge through a rotational field around a complete circuit will give us a potential that's different than zero. And here is where we pronounce the words that is a blasphemy to any KVLer: KVL becomes bird seed.

In the conservative field, you'll need to spend energy along some path to get from D to A. In the rotational field you don't. Just take path 1 and you're good to go.

That's why in the conservative electric field, you'll need some kind of EMF along the path between D and A. But not in the case of the rotational field.

So you can have two resistors connected in a circuit without any kind of component to overcome the field.

Because in this case the electric field is already circuital.

So in this case where does the energy come from? It comes entirely from the electric and the varying magnetic fields. And here we have to resort to the Poynting vector.
Title: Re: #562 – Electroboom!
Post by: thinkfat on November 10, 2021, 04:11:30 pm
"Same side" seems to be "geometrically on the same side relative to the magnetic flux". That means the chosen path must never encompass any area that is inside the magnetic flux. As long as you stick to that, you're good.

You got that right. It all boils down to geometry. Not electricity. Electricity works as usual.

Quote
I will have to apologize to @bsfeechannel :palm:

No worries.
:phew:
Title: Re: #562 – Electroboom!
Post by: jesuscf on November 10, 2021, 04:20:42 pm
And Lewin's supporters went the obfuscation way again!  Paragraphs and paragraphs of gibberish.  I almost felt for that again. Lets go back to the original problem at hand:  probing the circuit correctly as electroboom did shows that KVL works perfectly in Lewin's experiment.  It is that simple.
Title: Re: #562 – Electroboom!
Post by: bdunham7 on November 10, 2021, 04:21:12 pm
The selector is the presence or better the absence of magnetic flux inside the measurement loop.

OK, no argument that the manifestation is through magnetic flux.

Quote

The beauty of it is that [correct voltage of far branch] + [emf with correct sign] = [voltage read by multimeter]

Alright, IIRC R1 = left, R2 = right, A1 is top of R1.......D2 is bottom of R2.  So if we are measuring R2 with a voltmeter on the right, we are also measuring the equal voltage comprised of the contribution of the loop and R1.  Assuming clockwise current flow, the voltage calculated across R2 is I*R2 and the other matching voltage would be the EMF (A2-A1) - (I*R1) - EMF (D1-D2).  Or, if I've botched the signs, something like that.  Is that right?

Quote
That also explains why you cannot measure partial turn voltage, but only integer multiples.

My understanding of the difficulty of measuring partial turns was a bit different, I think, but perhaps the two can be reconciled--or I'm wrong.  So how does this work when you have only a partial turn or a wire that goes straight through a core like a current transformer? 
Title: Re: #562 – Electroboom!
Post by: thinkfat on November 10, 2021, 05:03:20 pm
@Sredni

You probably missed what I added to my post while you were writing your reply:

PS: after some more headscratching, I think I understood why you cannot see any voltage across the loop wire: every path that involves the piece of wire between the resistors and that does not go around the center of the electric field will sum up to 0. Therefore, if you just naively connect you voltmeter probes to the points "A1" and "A2", that path will only observe the voltage across the wire according to Mr. Ohm. No need to argue with electric charge being counteracted by another electric field.

I think it's really not needed to bring "charge" into the picture to understand why there is no observable voltage between any points of the loop wire (other than those from Mr. Ohms contribution). It's also not needed to bring in the magnetic flux, because once you've established that there's a rotating electric field, which is what Maxwell-Faraday gives you from $${\nabla \times E = -\frac{\partial B}{\partial t}}$$ it's enough to only look at paths through that field to explain each of the observed effects. Of course that is only easy if you only look at the two dimensional plane of the field that that is defined by the "inner loop", but it's enough to explain each of the effects Mehdi/ElectroBOOM was seeing in his experiments.

Title: Re: #562 – Electroboom!
Post by: Sredni on November 10, 2021, 05:23:17 pm
And Lewin's supporters went the obfuscation way again!  Paragraphs and paragraphs of gibberish.

People who studied it, prefer to call it physics.
But I am sure it is gibberish, to you.
Title: Re: #562 – Electroboom!
Post by: jesuscf on November 10, 2021, 05:56:24 pm
And Lewin's supporters went the obfuscation way again!  Paragraphs and paragraphs of gibberish.

People who studied it, prefer to call it physics.
But I am sure it is gibberish, to you.

Yes, "the spherical cow" argument:

Milk production at a dairy farm was low, so the farmer wrote to the local university, asking for help from academia. A multidisciplinary team of professors was assembled, headed by a theoretical physicist, and two weeks of intensive on-site investigation took place. The scholars then returned to the university, notebooks crammed with data, where the task of writing the report was left to the team leader. Shortly thereafter the physicist returned to the farm, saying to the farmer, "I have the solution, but it works only in the case of spherical cows in a vacuum."

https://en.wikipedia.org/wiki/Spherical_cow
Title: Re: #562 – Electroboom!
Post by: Sredni on November 10, 2021, 06:07:02 pm
Alright, IIRC R1 = left, R2 = right, A1 is top of R1.......D2 is bottom of R2.  So if we are measuring R2 with a voltmeter on the right, we are also measuring the equal voltage comprised of the contribution of the loop and R1.  Assuming clockwise current flow, the voltage calculated across R2 is I*R2 and the other matching voltage would be the EMF (A2-A1) - (I*R1) - EMF (D1-D2).  Or, if I've botched the signs, something like that.  Is that right?

I have no idea what the points A2, A1, D2, D1 refer to - probably they are present in the original Lewin drawing but I do not have it at had now. But I am a bit troubled by your use of "EMF (A2-A1)" and I believe therein lies the rub. You are still trying to apply Kirchhoff and you are implicitly assuming the wires are like 'batteries'. No, you need to let go of that because there is no longer the Eind field in the wires. It has been obliterated by the Ecoul field. This is what KVLers have trouble accepting. For example, thinkfat in the message above is trying to avoid bringing the charges into the discussion, but it's all about charges. Literally, all. And I mean "ALL".
You can try to avoid mentioning them directly but current and voltage are derived quantities to the more fundamental physical quantities charge and flux. Sooner or later you have to deal with the complete picture.

What you need to do is to apply the correct laws: KVL in the outer loop (because it is free of varying magnetic flux) and Faraday in the ring (because it links magnetic flux). When you apply Faraday in a circuit you apply the integral form and you put in the EMF linked by the closed (measurement) loop. All of it, because it is the surface integral of the the area whose boundary is your measurement loop.
You can see it how I solved the circuit in my second (or third, I don't remember) post about this topic on EESE linked above.
You can also see it applied by prof. Sam Ben-Yaakov of Ben Gurion University

Intuitive analysis of non conservative electrical circuits and an answer to a Riddle 
https://www.youtube.com/watch?v=pUsdiIl1Kyg (https://www.youtube.com/watch?v=pUsdiIl1Kyg)

As you can see, Ben-Yaakov seems to question Lewin's statement that KVL is for the birds, but only because he acknoweldge that you can still use KVL in the outer loops. But in the ring, and in the measurement loops that run around the core, he uses Faraday.
So KVL is no longer generally applicable. It really is for the birds, when you consider the ring alone.

Quote
So how does this work when you have only a partial turn or a wire that goes straight through a core like a current transformer?

A partial turn of wire is not a closed circuit, so it cannot have a current flowing. What you have is that the Eind field will displace charge, but charge has its own coloumbian field and it will try to resist this displacement. In the end, instant after instant, you reach (with relaxation times) an equilibrium where the induced field Eind in the conductor is exactly countebalanced by the coloumbian field Ecoul generated by the charge that has accumulated at the terminals (and a bit on the lateral surface, depending on geometry).
Things break down when the field change so fast that you cannot reach an equilibrium over the whole extension of the system. But this falls out of the boundaries of magneto-quasistatics. I don't want to go retarded on you.

Again, you cannot do away with the role of charges. And Eind alone tells only half of the story.

Your follow-up question - because I've developed this ability to read minds - is "what about partial inductances? there are books on partial inductance written by prominent scholars in the field of EMC, what about them?"
It is a very useful concept but needs extreme care not to be misunderstood. I consider it a useful bean-counting tool to see where most of the Eind field is captured by a particular loop, but it certainly does not authorize you to think there is a voltage developed along (note that I am using 'along') any part of a coil or conductor in a loop. I had a fourth answer for EESE in the making that will address this topic briefly but I no longer actively participate in the forum. I might make an exception because this madness about KVL is really irking, and the more explanation are out there, the better for all.
Title: Re: #562 – Electroboom!
Post by: thinkfat on November 10, 2021, 06:13:55 pm
My understanding of the difficulty of measuring partial turns was a bit different, I think, but perhaps the two can be reconciled--or I'm wrong.  So how does this work when you have only a partial turn or a wire that goes straight through a core like a current transformer?

I've been thinking about this, too. The issue with partial turns is, IMHO, how do you want to actually measure them? Your probe wire either becomes part of the loop and completes it, or it negates its contribution, depending on whether you route it around the axis of the rotational electrical field (then it completes the turn) or not (then it reverses the partial turn). That's how I explain to myself why it would be impossible to measure partial turns. I have yet to conduct an experiment to confirm this, though.

I haven't got an idea about the wire straight through the core, but at some point you will have to close the loop to add your volt meter.
Title: Re: #562 – Electroboom!
Post by: jesuscf on November 10, 2021, 06:51:15 pm
Here is the best demonstration I have found that Lewin is wrong and Electroboon/Mehdi is right.  The video is by  'fromjesse' and, in my opinion, he deserves more views and subscribers.  The setup is absolutely beautiful, and guess what: KVL holds perfectly.  'fromjesse' calls his setup the Lewin clock.  If you still think Lewin is correct watch this short video, it is just 10 minutes long:

https://www.youtube.com/watch?v=nAsZFP8Cfxk (https://www.youtube.com/watch?v=nAsZFP8Cfxk)

(When he measures, the peak voltage is shown in the top left corner of the oscilloscope screen)

Title: Re: #562 – Electroboom!
Post by: bsfeechannel on November 10, 2021, 07:10:47 pm
The problem with this demonstration is that KVL only "holds" in a very particular placement of the probes. If you try to measure from the outside of the ring the voltages will be different. Confirming what Lewin said: you can measure two different voltages at the same time when you have a varying magnetic field.

And he is really bad probing the circuit, because his probes are forming a loop with the "clock" wire which is inducing a voltage that is proportional to the area of this parasitic loop he created.

There are people pointing out his mistake in the comments, but of course he is rejecting.

What is interesting is that another KVLer, RSD Academy, if I'm not mistaken, said that if Lewin was right, you would connect a meter to a circuit and depending on the position of the probes and the meter, you would read a different voltage, and that's absurd in his view.

But if that is not true, why fromjesse proposed the "right" way to measure the circuit? Shouldn't I put my meter and my probes whatever the way I want and get the same voltage?

So, we don't need Lewin to prove that this is wrong, because the KVLers themselves contradict each other.
Title: Re: #562 – Electroboom!
Post by: bdunham7 on November 10, 2021, 07:15:32 pm
I have no idea what the points A2, A1, D2, D1 refer to - probably they are present in the original Lewin drawing but I do not have it at had now. But I am a bit troubled by your use of "EMF (A2-A1)" and I believe therein lies the rub. You are still trying to apply Kirchhoff and you are implicitly assuming the wires are like 'batteries'. No, you need to let go of that because there is no longer the Eind field in the wires. It has been obliterated by the Ecoul field. This is what KVLers have trouble accepting.

I'm not a 'KVLer' and you may be confusing me with someone else because as far as I know, I haven't really made any significant assertions of any kind here, just asked questions.  What I'm asking is if I have correctly understood your assertion and the reason I used the terms I did is because they are what you used when you said "The beauty of it is that [correct voltage of far branch] + [emf with correct sign] = [voltage read by multimeter".  Here is a quick drawing of my recollection of the diagram in question.

(https://www.eevblog.com/forum/amphour/562-electroboom!/?action=dlattach;attach=1320449;image)

Quote
A partial turn of wire is not a closed circuit, so it cannot have a current flowing.

Why is it relevant whether or not current is flowing?

Quote
But this falls out of the boundaries of magneto-quasistatics.

Your follow-up question - because I've developed this ability to read minds - is "what about partial inductances?

Your Kreskin abilities may be failing you, as I was not thinking that.  I actually don't remember ever hearing the term magneto-quasistatics, but when I saw it in the linked video I instantly understood the idea, or so I think.  I think the concept applies in any near-field case.  So although perhaps a completely rigorous analysis might include a calculation of inductances even if only to be able to definitively dismiss them as negligible, I'm not raising the issue or again, making any assertions at this point.
Title: Re: #562 – Electroboom!
Post by: Sredni on November 10, 2021, 07:23:37 pm
Here is the best demonstration I have found that Lewin is wrong and Electroboon/Mehdi is right.  The video is by  'fromjesse'

This is no different from Mabilde's probing.
Now, don't be alarmed if what follows will sound gibberish (and probably pompous and faggotish) to you.

What the Mabilde probing does is to run the probes at right angle with the induced electric field inside the loop. This will eliminate the contribute of the Eind field along the probes, leaving it untouched in the arc that is probed. BUT both Mabilde and fromjesse are not considering the contribute of the coloumbian electric field Ecoul that is present in the arc AND in the probes. And it is not perpendicular to them,

(https://i.stack.imgur.com/DkJMP.png)

Therefore, they are not cancelling the effects of this component of the field on the probes themselves.

In the end, what they measure in the arcs of conductor is not the actual voltage along (note that I use alond, because path matters) the probed arc, but only the contribute of the coloumbian field along the probes (because in the arc Ecoul = Eind but they are opposite and they cancel so there is no contribute there). And being Ecoul conservative, this value is equal to Ecoul in the arc.

They are measuring a PARTIAL contribute to the actual voltage, namely the part that is ascribed to the coloumbian field Ecoul alone.

Now, voltage is the path integral of the TOTAL electric field. Since the circulation of Etot is not zero, there is no more KVL.
What gives zero circulation if the partial component of the electric field Ecoul. This component admits a potential function that is known as the scalar potential phi, so you can concoct a KSPDL, a Kirchhoff Scalar Potential Difference Law, if you like, but those scalar potential differences are basically useless without the knowledge of the vector potential A. For starters, you cannot make Ohm's law work.
Title: Re: #562 – Electroboom!
Post by: jesuscf on November 10, 2021, 07:24:25 pm
The problem with this demonstration is that KVL only "holds" in a very particular placement of the probes. If you try to measure from the outside of the ring the voltages will be different. Confirming what Lewin said: you can measure two different voltages at the same time when you have a varying magnetic field.

KVL holds for any placement of the probes, if you account for the probes as being part of the circuit.  Check this video by 'RSD Academy' which explains it in more depth.  The conclusion is clear: Lewin messed up the measurement:

https://www.youtube.com/watch?v=LGdN7rDB3XE (https://www.youtube.com/watch?v=LGdN7rDB3XE)
Title: Re: #562 – Electroboom!
Post by: jesuscf on November 10, 2021, 07:33:21 pm
Here is the best demonstration I have found that Lewin is wrong and Electroboon/Mehdi is right.  The video is by  'fromjesse'

This is no different from Mabilde's probing.
Now, don't be alarmed if what follows will sound gibberish (and probably pompous and faggotish) to you.

What the Mabilde probing does is to run the probes at right angle with the induced electric field inside the loop. This will eliminate the contribute of the Eind field along the probes, leaving it untouched in the arc that is probed. BUT they are not considering the contribute of the coloumbian electric field Ecoul that is present in the arc AND in the probes. And it is not perpendicular to them, so they are not cancelling the effects on the probes themselves.

In the end, what they measure in absence of resistors is not the actual voltage along (note that I use alond, because path matters) the probed arc, but only the contribute of the coloumbian field along the probes (because in the arc Ecoul = Eind but they are opposite and they cancel so there is no contribute there). And being Ecoul conservative, this value is equal to Ecoul in the arc.

They are measuring a PARTIAL contribute to the actual voltage, namely the part that is ascribed to the coloumbian field Ecoul alone.

1) Do this experiment falsifies Lewin's experiment?  The answer is yes.

2) Does KVL hold in this experiment?  The answer is also yes.

Title: Re: #562 – Electroboom!
Post by: Sredni on November 10, 2021, 07:48:41 pm
I'm not a 'KVLer'

I did not say you were one. I just stated what KVLers have trouble with.

Quote
(https://www.eevblog.com/forum/amphour/562-electroboom!/?action=dlattach;attach=1320449;image)

And this is what a KVLer would do. Why do you compute partial contribute to the emf at all? And only on the horizontal branches? Write the equations by listing the voltage drops on one side, and the linked emf on the other (zero if absent)

(https://i.stack.imgur.com/155Ic.jpg]https://i.stack.imgur.com/155Ic.jpg)

Quote
Quote
A partial turn of wire is not a closed circuit, so it cannot have a current flowing.
Why is it relevant whether or not current is flowing?

What is the definition of inductance?

Quote
Quote
Your follow-up question - because I've developed this ability to read minds - is "what about partial inductances?
Your Kreskin abilities may be failing you, as I was not thinking that.

It would have come up anyway, eventually, from someone else.

Title: Re: #562 – Electroboom!
Post by: bdunham7 on November 10, 2021, 09:07:00 pm
And this is what a KVLer would do. Why do you compute partial contribute to the emf at all? And only on the horizontal branches? Write the equations by listing the voltage drops on one side, and the linked emf on the other (zero if absent)

It was not my idea to draw the diagram this way, it is how Lewin drew it, AFAIK the 'horizontal branches' aren't really intended to represent the actual physical layout.  As for the way I've written the EMF part, I was simply trying to interpret what you wrote to make sure I understood it correctly.  Are you saying that geometrical accuracy issues aside, you simply cannot break down the EMF into two parts?

Quote
What is the definition of inductance?

Hmmm... L = V/(dI/dt) or something like that?

So where are you going with that?  Are you going to claim that unless current is flowing, you can't have a definite voltage?  In your diagram you refer to current in the outer loops, I1 and I2 and the voltmeters as having a resistance.  What if those were ideal voltmeters with infinite resistance and both I1 and I2 were zero?  Would that change the readings?

Title: Re: #562 – Electroboom!
Post by: thinkfat on November 10, 2021, 09:34:05 pm
And this is what a KVLer would do. Why do you compute partial contribute to the emf at all? And only on the horizontal branches? Write the equations by listing the voltage drops on one side, and the linked emf on the other (zero if absent)

It was not my idea to draw the diagram this way, it is how Lewin drew it, AFAIK the 'horizontal branches' aren't really intended to represent the actual physical layout.  As for the way I've written the EMF part, I was simply trying to interpret what you wrote to make sure I understood it correctly.  Are you saying that geometrical accuracy issues aside, you simply cannot break down the EMF into two parts?


There's also the EMF in the resistors that you need to account for, since they're part of the loop. At this point it doesn't really make sense to break EMF down into parts any more, at least not for the sake of the computation: the total EMF around the loop is given as 1V.
Title: Re: #562 – Electroboom!
Post by: jesuscf on November 10, 2021, 09:36:09 pm
But, anyway... What do you think would change in the Lewin ring if, instead of a perfect conductor, you had a highly conductive copper conductor?
I tell you what: almost nothing. And certainly nothing of relevance. The only difference is that, instead of zero electric field and zero voltage drop in the conductors, you will see an almost negligible electric field E = j /sigma_copper and an almost negligible voltage drop of a handful of microvolts. Against the hundreds of millivolts of drops at the resistors.

Really, almost nothing?  The attached image is from 'fromjess' experiment which shows a significant voltage drop between two points in the ring.  Hard to see, but in the attached image I think it is 184mV.  You can clearly see from the video that you can measure a sizeable voltage between any two arbitrary points around the copper ring.  As he says it in the video "so we can measure positive and negative voltages all around this dial".  How do explain that now?
Title: Re: #562 – Electroboom!
Post by: Sredni on November 10, 2021, 09:49:13 pm
It was not my idea to draw the diagram this way, it is how Lewin drew it, AFAIK the 'horizontal branches' aren't really intended to represent the actual physical layout.

Ok, but the problem with this approach is that in the branches that represent the conductors there is not only Eind (responsible for the EMF part of voltage), but also Ecoul (responsible for the scalar potential difference of voltage).

Quote
As for the way I've written the EMF part, I was simply trying to interpret what you wrote to make sure I understood it correctly.  Are you saying that geometrical accuracy issues aside, you simply cannot break down the EMF into two parts?

You can break the emf contribute apart as much as you like. It's the path integral of the Eind part of Etot and integrals are linear. No problem with that. But you need to consider the other part of the electric field in the wire, Ecoul. That was my objection. So, there is nothing in terms of voltage in the wires. The moment you put a good or perfect conductor on the 'imaginary' path in empty space where you could see the little arrows going in circle, those little arrows gets smashed and obliterated by the field generated by the charges they displaced in the conductor.
This is the crucial point.

Quote
Quote
What is the definition of inductance?
Hmmm... L = V/(dI/dt) or something like that?
So where are you going with that?  Are you going to claim that unless current is flowing, you can't have a definite voltage?

Nope, you can have voltage across the terminals of an open circuited mutual inductance. That sentence of mine was ill conceived. I'll tell you what I was aiming to. Inductance is a geometric property, but the definition is as the coefficient between the flux linked and the current that generates such flux. You need a surface to define flux, and you need a closed path (not necessarily in matter) to define the boundary of the surface cutting the flux.
The V = L di/dt formula comes out from applying Faraday's law to the surface of the coil that is bounded by the closed path formed by joining the path inside the coil's conductor and the jump at the terminals. In the end, you find that voltage computed as path integral across the gap is L di/dt, but voltage computed as path integral along the conductor is zero. Because voltage computed as path integral along the closed path formed by the two is the time derivative of surface integral of the magnetic field.

KVL breaks right into the [edit: lumped element] definition of inductance: between the same two points you can have 0V (along the conductor) and, for example, 120V (across the terminals).
(Incidentally this is something that Bob Duhamel has shown not to be able to conceive, to the point that he had to distort what Lewin said in his lecture).

To go back to the inductance of a partial turn, in order to talk about the flux linked you need to define a surface. So, what is the surface enclosed by a partial turn? It's a very delicate concept and it is better to leave it alone or we would need a new thread only for that.
Title: Re: #562 – Electroboom!
Post by: bdunham7 on November 10, 2021, 09:53:08 pm
There's also the EMF in the resistors that you need to account for, since they're part of the loop. At this point it doesn't really make sense to break EMF down into parts any more, at least not for the sake of the computation: the total EMF around the loop is given as 1V.

The resistors can be arbitrarily small.  Whether breaking down the EMF parts makes sense depends on what you are trying to analyze.  In the current context, I thought it was worth the inquiry.
Title: Re: #562 – Electroboom!
Post by: Sredni on November 10, 2021, 09:55:18 pm
But, anyway... What do you think would change in the Lewin ring if, instead of a perfect conductor, you had a highly conductive copper conductor?
I tell you what: almost nothing. And certainly nothing of relevance. The only difference is that, instead of zero electric field and zero voltage drop in the conductors, you will see an almost negligible electric field E = j /sigma_copper and an almost negligible voltage drop of a handful of microvolts. Against the hundreds of millivolts of drops at the resistors.

Really, almost nothing?  The attached image is from 'fromjess' experiment which shows a significant voltage drop between two points in the ring.  Hard to see, but in the attached image I think it is 184mV.  You can clearly see from the video that you can measure a sizeable voltage between any two arbitrary points around the copper ring.  As he says it in the video "so we can measure positive and negative voltages all around this dial".  How do explain that now?

That measurement has nothing to do with the fact that the ring is made of a finite conductivity conductor. You would have basically the same measure even if they were superconducting traces.

And I have already explained why he gets those reading in my answer to you above.
It is not my fault if you cannot make sense of it.
Both fromjesse and RSD Academy resort to deleting comments that tell them they are wrong, which goes to show how insecure they are about what they dish out to their viewers.
Title: Re: #562 – Electroboom!
Post by: bdunham7 on November 10, 2021, 10:22:21 pm
Picture yourself in a boat on a river.

I had already envisioned a more apt analogy, a circular canal with a circulating stream of water.  Obviously there are two ways from any one point to any other--upstream and downstream.

I don't think there's any confusion about the fact that the voltages across the resistors are what they are nor that a charge starting at A will take different amounts of work to get it to D depending on which resistor you take it through.  What is at issue is that the 'path dependence' as stated refers to the positioning of the test leads and it appears, although I haven't seen this stated as such or proven mathematically, that if all of the circuitry including the test leads are in the same plane, then although there are infinitely many possible paths for the test leads, all of those paths end up being exactly equal to one branch or the other depending on which resistor the leads form a closed loop with that excludes the center ring.

For me this raises questions of manifestation and definition.  A grand law like Faraday's is all well and good (and I'm not doubting it in any way) but even when such laws are universally satisfied, the minions of nature have to go out and push and tug on actual particles to make them obey those laws.  Those forces don't read the laws, the laws are a result of observation and math--usually integral calculus--where someone has figured out a consistency that can be used to simplify more advanced calculations.  And then there's the definition of voltage itself, which I'll leave alone for now.  So in this case, a charge going from A to D will experience a different net total of forces on it during the trip (work) depending on the path it takes.  The main questions seem to be where and how those forces are applied and then how and why the system would react to changes in the path, such as taking the test leads out of the plane. 

Title: Re: #562 – Electroboom!
Post by: bsfeechannel on November 10, 2021, 10:39:11 pm
KVL holds for any placement of the probes, if you account for the probes as being part of the circuit. 

No. It doesn't. He even shows a video from our fellow EEVBlogger Joeqsmith where he clearly demonstrates that the positioning of probes affects the measurement.

This is what we call cognitive dissonance: KVL holds for any placement of the probes, as long as you place them in a very specific and unique position.

Give me a break.

Quote
Check this video by 'RDS Academy' which explains it in more depth.  The conclusion is clear: Lewin messed up the measurement:

He concludes that Lewin doesn't know Ohms law. I wonder how could he have fooled MIT, which I thought was one of the most, if not the most, prestigious technology institutes in the world, for 43 years, as he doesn't know Ohms law, doesn't know how to model a circuit, doesn't know how Kirchhoff law works, doesn't know how to probe a circuit.

How could they have given him these awards:

1978 – NASA Award for Exceptional Scientific Achievement
1984 – Alexander von Humboldt Award
1984 – Guggenheim Fellowship
1984 – MIT Science Council Prize for Excellence in Undergraduate Teaching
1988 – MIT Department of Physics W. Buechner Teaching Prize
1991 – Alexander von Humboldt Award (again)
1997 – NASA Group Achievement Award for the Discovery of the Bursting Pulsar
2003 – MIT Everett Moore Baker Memorial Award for Excellence in Undergraduate Teaching
2011 – first recipient of the Educator Award for OpenCourseWare Excellence (ACE)

Lewin must be a master con man, no doubt. Or the MIT is a joint. Or both.

The RDS Academy dude also accuses Lewin of defying the scientific and engineering establishment and textbooks. I've never seen a reputable textbook both of engineering and physics which claims KVL holds under a varying magnetic field. In fact I've seen exactly the opposite. They call our attention to the fact that this cannot happen.
Title: Re: #562 – Electroboom!
Post by: jesuscf on November 10, 2021, 11:05:41 pm
But, anyway... What do you think would change in the Lewin ring if, instead of a perfect conductor, you had a highly conductive copper conductor?
I tell you what: almost nothing. And certainly nothing of relevance. The only difference is that, instead of zero electric field and zero voltage drop in the conductors, you will see an almost negligible electric field E = j /sigma_copper and an almost negligible voltage drop of a handful of microvolts. Against the hundreds of millivolts of drops at the resistors.

Really, almost nothing?  The attached image is from 'fromjess' experiment which shows a significant voltage drop between two points in the ring.  Hard to see, but in the attached image I think it is 184mV.  You can clearly see from the video that you can measure a sizeable voltage between any two arbitrary points around the copper ring.  As he says it in the video "so we can measure positive and negative voltages all around this dial".  How do explain that now?

That measurement has nothing to do with the fact that the ring is made of a finite conductivity conductor. You would have basically the same measure even if they were superconducting traces.

And I have already explained why he gets those reading in my answer to you above.
It is not my fault if you cannot make sense of it.
Both fromjesse and RSD Academy resort to deleting comments that tell them they are wrong, which goes to show how insecure they are about what they dish out to their viewers.

So, what is wrong with measuring voltages correctly?  The experiment is there: the voltages add to zero.  KVL holds if you know how to measure voltages in a varying magnetic field.  Lewin didn't measure in his experiment correctly and he got the wrong conclusions from his results.  RSD Academy summarizes it nicely:

"Dr. Lewin is disagreeing with the vast majority of the scientific establishment, he's disagreeing with the vast majority of textbooks, he's  disagreeing with the vast majority of professors of both electrical engineering and physics and his postulation that Kirchhoff's voltage law doesn't hold is based on a incorrect application of ohm's law not knowing how ohm's law has to be applied to a voltage source.  Then he performs an experiment to prove his premise but he doesn't take precautions to make sure that the magnetic fields don't affect his measurements."

Maybe fromjesse and RSD Academy are erasing your comments because you are, you know, perhaps, incorrect...  Anyhow: assume we have a spherical cow in a vacuum...
Title: Re: #562 – Electroboom!
Post by: jesuscf on November 10, 2021, 11:10:11 pm
KVL holds for any placement of the probes, if you account for the probes as being part of the circuit. 

No. It doesn't. He even shows a video from our fellow EEVBlogger Joeqsmith where he clearly demonstrates that the positioning of probes affects the measurement.

This is what we call cognitive dissonance: KVL holds for any placement of the probes, as long as you place them in a very specific and unique position.

Give me a break.

Quote
Check this video by 'RDS Academy' which explains it in more depth.  The conclusion is clear: Lewin messed up the measurement:

He concludes that Lewin doesn't know Ohms law. I wonder how could he have fooled MIT, which I thought was one of the most, if not the most, prestigious technology institutes in the world, for 43 years, as he doesn't know Ohms law, doesn't know how to model a circuit, doesn't know how Kirchhoff law works, doesn't know how to probe a circuit.

How could they have given him these awards:

1978 – NASA Award for Exceptional Scientific Achievement
1984 – Alexander von Humboldt Award
1984 – Guggenheim Fellowship
1984 – MIT Science Council Prize for Excellence in Undergraduate Teaching
1988 – MIT Department of Physics W. Buechner Teaching Prize
1991 – Alexander von Humboldt Award (again)
1997 – NASA Group Achievement Award for the Discovery of the Bursting Pulsar
2003 – MIT Everett Moore Baker Memorial Award for Excellence in Undergraduate Teaching
2011 – first recipient of the Educator Award for OpenCourseWare Excellence (ACE)

Lewin must be a master con man, no doubt. Or the MIT is a joint. Or both.

The RDS Academy dude also accuses Lewin of defying the scientific and engineering establishment and textbooks. I've never seen a reputable textbook both of engineering and physics which claims KVL holds under a varying magnetic field. In fact I've seen exactly the opposite. They call our attention to the fact that this cannot happen.

You forgot this 'achievement': Victim In Walter Lewin Online Course Sexual Harassment Case Comes Forward

https://www.huffpost.com/entry/walter-lewin-sexual-harassment-mit_n_6532698 (https://www.huffpost.com/entry/walter-lewin-sexual-harassment-mit_n_6532698)

EDIT: Sorry, I forgot.  Did you see Lewin solving problem #24?  The impression I have is that he has a very basic knowledge (at best) of electric circuits.

Title: Re: #562 – Electroboom!
Post by: bsfeechannel on November 10, 2021, 11:47:35 pm

You forgot this 'achievement': Victim In Walter Lewin Online Course Sexual Harassment Case Comes Forward

https://www.huffpost.com/entry/walter-lewin-sexual-harassment-mit_n_6532698 (https://www.huffpost.com/entry/walter-lewin-sexual-harassment-mit_n_6532698)

So what? Does this mean he doesn't know ohms law? What happens is that Lewin exposed how most people in our field take electromagnetism for granted and don't really understand the full implications of the phenomenon. This left many uncomfortable. And now they want to redeem themselves by treating him as a scapegoat. That backfired by the looks of it. 
Title: Re: #562 – Electroboom!
Post by: bsfeechannel on November 11, 2021, 12:00:40 am
Really, almost nothing?  The attached image is from 'fromjess' experiment which shows a significant voltage drop between two points in the ring.  Hard to see, but in the attached image I think it is 184mV.  You can clearly see from the video that you can measure a sizeable voltage between any two arbitrary points around the copper ring.  As he says it in the video "so we can measure positive and negative voltages all around this dial".  How do explain that now?

What is funny is that for some magic reason the copper rings generate voltages, while the resistors drop them. As if the copper rings were not resistors themselves.

I wonder what would happen if if the resistors took up all the circumference of the rings, leaving no space for the probes to move.

So much for he Lewin clock. Entertaining. But hardly scientific.
Title: Re: #562 – Electroboom!
Post by: bsfeechannel on November 11, 2021, 12:11:41 am
I had already envisioned a more apt analogy, a circular canal with a circulating stream of water.

Glad that you have already understood it intuitively.

Quote
The main questions seem to be where and how those forces are applied and then how and why the system would react to changes in the path, such as taking the test leads out of the plane.

Well, if you really want to go down the rabbit hole, there's no shortcut. You'll have to resort to vector calculus. This is where the intuitive approach starts to fail, I'm afraid.
Title: Re: #562 – Electroboom!
Post by: jesuscf on November 11, 2021, 12:18:29 am
Really, almost nothing?  The attached image is from 'fromjess' experiment which shows a significant voltage drop between two points in the ring.  Hard to see, but in the attached image I think it is 184mV.  You can clearly see from the video that you can measure a sizeable voltage between any two arbitrary points around the copper ring.  As he says it in the video "so we can measure positive and negative voltages all around this dial".  How do explain that now?

What is funny is that for some magic reason the copper rings generate voltages, while the resistors drop them. As if the copper rings were not resistors themselves.

I wonder what would happen if if the resistors took up all the circumference of the rings, leaving no space for the probes to move.

So much for he Lewin clock. Entertaining. But hardly scientific.

Any arbitrary section of the copper ring under the influence of the external varying magnetic field behaves as non-ideal voltage source.

EDIT: both the copper rings and the resistors behave as non-ideal voltage sources.  The resistors equivalent circuit will be a very small voltage source with a large resistance in series. The copper (or arbitrary segment of copper) equivalent circuit will be a large voltage source in series with a small series resistance.
Title: Re: #562 – Electroboom!
Post by: bdunham7 on November 11, 2021, 12:56:18 am
To go back to the inductance of a partial turn, in order to talk about the flux linked you need to define a surface. So, what is the surface enclosed by a partial turn? It's a very delicate concept and it is better to leave it alone or we would need a new thread only for that.

OK, so then lets not talk about flux and surface.  Consider a straight conductor going through a toroid core, as you would see in a current transformer.  An increasing current in the coil of the current transformer should result in voltage at the ends of the rods, no?
Title: Re: #562 – Electroboom!
Post by: rfeecs on November 11, 2021, 01:13:38 am

What is funny is that for some magic reason the copper rings generate voltages, while the resistors drop them. As if the copper rings were not resistors themselves.

I wonder what would happen if if the resistors took up all the circumference of the rings, leaving no space for the probes to move.


That is explained here:

https://web.mit.edu/8.02/www/Spring02/lectures/lecsup4-1.pdf (https://web.mit.edu/8.02/www/Spring02/lectures/lecsup4-1.pdf)
Title: Re: #562 – Electroboom!
Post by: bsfeechannel on November 11, 2021, 01:17:03 am
I was being sarcastic. Anyway, thanks.
Title: Re: #562 – Electroboom!
Post by: Sredni on November 11, 2021, 01:25:38 am
To go back to the inductance of a partial turn, in order to talk about the flux linked you need to define a surface. So, what is the surface enclosed by a partial turn? It's a very delicate concept and it is better to leave it alone or we would need a new thread only for that.

OK, so then lets not talk about flux and surface.  Consider a straight conductor going through a toroid core, as you would see in a current transformer.  An increasing current in the coil of the current transformer should result in voltage at the ends of the rods, no?

Straight conductors always pose problems when inductance (external, either self- or mutual) is involved.
Make a diagram with relative dimensions specified. Off the top of my head, if I understood what you mean (this is a current transformer in reverse, where a current goes through the detecting coil and you expect a charge displacement in the rod along the axis), the changing flux will produce a changing induced electric field Eind along the axis of the torus. And this will displace the charges in the metallic conductor the rod is made of, causing accumulation of positive charges on one extreme and negative on the other. At a given instant in time, the electric field of the displaced charge will be equal and opposite to the Eind field in the rod.
Total electric field in the rod is zero, and you will observe a voltage, but to measure it correctly - apart from an insanely high impedance voltmeter - you have to place the voltmeter in the 'hole' of the torus, so as not to link any of the flux in the core.

But, please, produce a drawing.

(Why this 'degenerate' example? Have you already solved all doubts about the circular ring?)
Edit: this is a bit stranger than I first thought - I am not sure there could be a voltage across the straight wire in that position. Can we bend it a little?
Edit 2: relative dimensions of the torus and the wire, which wire has current impressed.
There is no picture attached in your post below.
Title: Re: #562 – Electroboom!
Post by: jesuscf on November 11, 2021, 01:33:29 am
I was being sarcastic. Anyway, thanks.

Sorry, I thought you were asking sincerely.  A resistor under the influence of the external varying magnetic field also behaves as non-ideal voltage source.
Title: Re: #562 – Electroboom!
Post by: bdunham7 on November 11, 2021, 01:41:17 am
Total electric field in the rod is zero, and you will observe a voltage, but to measure it correctly - apart from an insanely high impedance voltmeter - you have to place the voltmeter in the 'hole' of the torus, so as not to link any of the flux in the core.

But, please, produce a drawing.

(Why this 'degenerate' example? Have you already solved all doubts about the circular ring?)

Why does the voltmeter impedance matter?  If it does, just consider the ideal voltmeter with infinite impedance, since this is theoretical and we have ideal components elsewhere.  If that is problematic, then voltmeters can be provided with some pretty high input impedances, certainly high enough to not matter much in this case.

Drawing attached.  I don't know that I have any doubts about the original circular ring, I think the issues lie elsewhere.  But please, tell me if the drawing matches what you were thinking and responding to.

Edit:  I'm sorry, I didn't specify actual numerical dimensions because I didn't have any specific ones in mind.  Which dimensions are necessary?
Title: Re: #562 – Electroboom!
Post by: Sredni on November 11, 2021, 02:53:57 am
There is no drawing attached that I can see.
You probably did not see the edits I made in my previous message.

Anyway, the more I think about it, the more I hate partial coils, especially straight wires. We need to go around the core to see a voltage that we can measure in the gap. When there are only partial turns, we usually complete the link with the probes, so we cannot measure the voltage of a partial turn (the multitap transformer discussed in another page). We can compute the contribution of partial inductance as path integrals of the vector potential. I am not sure it's possible to measure partial contributions.
Title: Re: #562 – Electroboom!
Post by: bdunham7 on November 11, 2021, 04:03:14 am
Sorry, here's the drawing.  I'd like the rod to stay straight for now just so I can see where this goes. 

Dimension, lets have the rod be 1cm diameter x 100cm long, the torus can be 10cm OD, 8cm ID and 1cm height.  There can be 10 windings and the current can be increasing at 1000A/s.  Or any other numbers that are convenient.  The rod can be copper or iron.

Quote
We can compute the contribution of partial inductance as path integrals of the vector potential. I am not sure it's possible to measure partial contributions.

How you define and measure voltage is certainly germane.  But saying you can't measure the voltage of part of a turn, presumably because your test leads are subject to a field that exactly counteracts the partial contribution is different than saying that the voltage isn't there.  That's a definitional issue.  I have a couple of ideas, but I'd like your take on this first.

(https://www.eevblog.com/forum/amphour/562-electroboom!/?action=dlattach;attach=1320656;image)

Title: Re: #562 – Electroboom!
Post by: Sredni on November 11, 2021, 05:18:36 am
there are too many degeneration to give a quick answer. Partial turn, straight wire, in the middle of the torus... I need to work out the actual induced field of a toroidal core.

Partial turn is a problem because it does not link any flux per se.
Straight wire is a problem because it does not have external inductance.
Middle of the torus is a problem because if we consider a filamentary current, the field is curl free... and this is related to the fact that there is no area associated to a segment. And if I go to infinity, we have all planes to choose.

Interesting problem, but unnecessary to understand the Faraday-Kirchhoff dispute.

I need to think about it. For the moment I tell you this about the inability to measure.
You can have a voltage across a straight piece of wire in a nonconservative field due to the displaced charge, but you might not be able to measure it because you always complete a loop when you place the instrument across it. So you end up either linking the whole emf or none of it. This is why I hate partial turns. But when you complete the loop with your probes and the internal resistance of the voltmeter, you remove the charge from the extremes of your piece of wire and place them at the interfaces with the internal resistance of the voltmeter. So, there no longer is a voltage once you close the circuit, and even with an infinite resistance voltmeter, you still have to complete the circuit with the probes so you won't be able to read for example 1/10th of the emf. You will read the whole emf.

And this is what happens with the voltmeters at the exterior of Lewin's ring as well. When you consider the loop formed by the farthest resistor, you complete the half ring with your probe and read the whole EMF: and in fact the voltage of the far resistor is altered by one emf; we say we use Faraday. When you consider the loop formed by the nearest resistor, you complete the other half-ring with your probes but this time you do not link the emf (or, if you will, you link 0 times the EMF), and we can be sure that the voltmeter reads the voltage across that branch; we say we use KVL on this loop.

Would it be the same to consider a straight rod near an infinite solenoid? For that I have the analytical expressions of the fields, it's a tad less degenerate case.
I'll have some sleep first.

Title: Re: #562 – Electroboom!
Post by: bsfeechannel on November 11, 2021, 07:40:29 am
A resistor under the influence of the external varying magnetic field also behaves as non-ideal voltage source.

But, but, but, but fromjesse said that the copper rings generate voltages, while the resistors drop it! How can I properly learn Ohms law, KVL, good probing and oscilloscope operation if you guys keep contradicting each other? Aw, unbelievable!
Title: Re: #562 – Electroboom!
Post by: bsfeechannel on November 11, 2021, 08:09:44 am
Interesting problem, but unnecessary to understand the Faraday-Kirchhoff dispute.

I guess this is an important issue since KVLers out there are claiming that the wires in the loop are standalone inductors that generate voltage to the circuit. This may come from the fact that you can actually calculate the inductance of a straight stretch of wire. You can but there's an implicit assumption that they do not consider. Since the the magnetic field intensity generated by the current in the wire is inversely proportional to the distance from the wire, just consider a rectangle whose one of the sides is the wire, and whose width is a distance for which the magnetic field intensity is negligible. That's the area you will consider for the calculation of the induced voltages around this rectangle. So you are assuming that this piece of wire will be somehow part of a circuit whose return path will be sufficiently away from it.

A stretch of wire very close to a return ground plane for instance will have to be calculated taking into consideration this geometry and will of course have an inductance that will be very different from the same piece wire hanging out in the breeze.

In short, the wire is never consider alone. It is implicitly considered part of a complete loop.
Title: Re: #562 – Electroboom!
Post by: thinkfat on November 11, 2021, 09:11:09 am
So, what is wrong with measuring voltages correctly?  The experiment is there: the voltages add to zero.  KVL holds if you know how to measure voltages in a varying magnetic field.  Lewin didn't measure in his experiment correctly and he got the wrong conclusions from his results.  RSD Academy summarizes it nicely:

"Dr. Lewin is disagreeing with the vast majority of the scientific establishment, he's disagreeing with the vast majority of textbooks, he's  disagreeing with the vast majority of professors of both electrical engineering and physics and his postulation that Kirchhoff's voltage law doesn't hold is based on a incorrect application of ohm's law not knowing how ohm's law has to be applied to a voltage source.  Then he performs an experiment to prove his premise but he doesn't take precautions to make sure that the magnetic fields don't affect his measurements."

I don't think there's a "correct" way or a "wrong" way to measure the voltages. Every outcome of the experiments, be it from Lewin or "Electroboom" or Mabilde or "fromjesse" can be explained with Faradays law. What is also undeniable is that the measurement instruments and their arrangement are inevitably part of the experiment. There is no "magic probe" you could poke anywhere and have a result that is independent of the setup.

Fun fact: in Lewins experiment, and also in the MIT Courseware video, the probe leads _may_ be affected by the magnetic flux, but since the path they're taking is only through a "conservative" region of the electric field, the effect cancels itself out.

Anyway, this is becoming really quite academic. I understand why engineering minded people try to avoid this stuff like the plague, after all the whole electronic industry is trying hard to confine those effects within the boundaries of components and rather work with transformers described by datasheet values instead of diving into vector analysis and calculus. It is really only to keep the "magic" contained in small boxes that can be linked together in a way that lets you forget that fields exist and only need circuit theory and KVL to get by. As soon as Physics enters the stage, the hurt starts. That's why RF and "high speed design" is considered "black magic", because you cannot get by without considering Physics. I think it's good that there are Dr. Lewins out there to remind us that blind application of method will fool you.

PS: Dr. Lewin is not incorrectly applying Ohms law. He just doesn't consider any discrete part of the loop being a "voltage source", because he doesn't attempt to "lump" the circuit to apply KVL.
Title: Re: #562 – Electroboom!
Post by: thinkfat on November 11, 2021, 10:55:39 am
Humor me, guys. See the attached picture. All wires are actual wires and interact with fields, but have negligible resistance. R1 can be any value, I don't think it matters. Assume EMF=1V. Predict the voltages shown on V1 and V2, using KVL and Faradays Law.
Title: Re: #562 – Electroboom!
Post by: rsjsouza on November 11, 2021, 02:14:20 pm
(this post is just to tag this thread to show up in my updated threads list, which I have been following with interest. Nothing new I can contribute at the time. Sorry about the noise. Carry on...)
Title: Re: #562 – Electroboom!
Post by: bdunham7 on November 11, 2021, 03:14:56 pm
In short, the wire is never consider alone. It is implicitly considered part of a complete loop.

That may make your theory and math work out more clearly to you, but I think it is a mistake.  It's true that, for example, a wire going straight through a current transformer is considered to be 1 turn because there is typically a return wire somewhere, but that's not why it actually works.  The interaction between a varying magnetic field and the charges in a wire is a local (microscopically so) effect and Faraday's Law is a mathematically proven observation that says that for a given area and a varying total flux through the area, the EMF around the perimeter area adds up to a number, and that number only depends on the rate of change of the total flux.  So, for example, it doesn't matter where in the toroid of the current transformer the wire is, it still reads the same because...math.  And in the Lewin device, it doesn't matter if the solenoid is off center, the total EMF on the ring adds up to the same amount.  That doesn't mean that the EMF is always evenly distributed nor that we can always consider it only as a total around the perimeter.  It so happens that for a CT and for the Lewin device with resistors, perhaps you can ignore local differentials.  Can we imagine a circuit where the local variations do matter? 
Title: Re: #562 – Electroboom!
Post by: jesuscf on November 11, 2021, 04:39:19 pm
A resistor under the influence of the external varying magnetic field also behaves as non-ideal voltage source.

But, but, but, but fromjesse said that the copper rings generate voltages, while the resistors drop it! How can I properly learn Ohms law, KVL, good probing and oscilloscope operation if you guys keep contradicting each other? Aw, unbelievable!

Nice try at deflection.  There is no contradiction.  Here it is again so you can understand:

1) thick wire = large voltage; very small resistance

2) tiny resistor = very small voltage; large resistance

That is why fromjesse used SMT resistors and wide copper tape.  Did Lewin used tiny resistors and thick wire in his experiment?  I doubt it!  This should also answer your previous question about making a ring with only resistors in series.


Title: Re: #562 – Electroboom!
Post by: Sredni on November 11, 2021, 05:18:42 pm
In short, the wire is never consider alone. It is implicitly considered part of a complete loop.
That may make your theory and math work out more clearly to you, but I think it is a mistake.  It's true that, for example, a wire going straight through a current transformer is considered to be 1 turn because there is typically a return wire somewhere, but that's not why it actually works.  The interaction between a varying magnetic field and the charges in a wire is a local (microscopically so) effect

This locality thing is something I hear from time to time. Some of the people who bring it up tend to think to Maxwell's equations as vector algebraic equations. They are not. They are vector partial differential equations, and as such they need boundary conditions.
So, if your experiment with the straight wire was just to show that (dang, I was trying to get a quantitative answer...), well I'm sorry but I have to inform you that the information about a piece of wire alone is not sufficient to predict how it will behave in a complete circuit. You can treat it as it is and show that there is charge displacement - probably with a sensitive electrometer. But when you put it in a circuit it's the whole circuit that dictates how it will behave. For example, a piece of straight wire inside the variable magnetic zone can have current flowing from extreme A to extreme B if it part of a certain loop, and current flowing from B to A if it is part of another loop. In any case, the charge that was there when the rod was stand-alone, is no longer there when it's part of a circuit.

I stand by my position: you can see charge at the extremes, but if you try to measure the voltage with a voltmeter you will necessarily end up linking integer multiples of the EMF. And this applies to, say, a 3/4 loop of wire around the core. There's no need to invoke the degenerate case of straight wire. Any partial turn will do.

As I said before, the concept of partial inductance is a very delicate one and that is why I am reluctant (pun!!!) to discuss it in this context. It will confuse KVLers even more and it will lead to endless thought experiments that seem to validate the idea that the wires behaves as batteries. They do not. Look at the fields. Look at the total electric field in your circuit or even rod and compute the path integral to find the voltage. There is the answer.


Title: Re: #562 – Electroboom!
Post by: bdunham7 on November 11, 2021, 05:25:39 pm
but if you try to measure the voltage with a voltmeter you will necessarily end up linking integer multiples of the EMF.

OK, last question before I actually present something on the locality issue.

Do you need to use a voltmeter (with test leads) to measure voltage?

 >:D

Title: Re: #562 – Electroboom!
Post by: Sredni on November 11, 2021, 05:26:17 pm
Interesting problem, but unnecessary to understand the Faraday-Kirchhoff dispute.
I guess this is an important issue since KVLers out there are claiming that the wires in the loop are standalone inductors that generate voltage to the circuit. This may come from the fact that you can actually calculate the inductance of a straight stretch of wire.

Yes, I understand that the point he's trying to discuss is that "if it behaves as a tiny battery because it displace charges, then why can't wires be thought as batteries". But it does not matter the the piece of wire be straight and in the dead center of the torus. I was thinking how to derive the whole Eind field of a toroidal transfomer with increasing current in the whole space. At this point, if the objection he is trying to make is the one above, I can use a piece of wire in the field of an infinitely long solenoid. The answer is the same: the wire will experience the Eind rotational field, it will displace charge (with relaxation times, instant after instant) and it will make the total electric field inside it zero. Voltage computed as path integral of Etot along the rod is zero. Voltage computed as path integral of Etot in the space around the rod from the same endpoints will not be zero EDIT post-nap still be zero on paths that do not go around the core, and will be the full EMF for paths that do go around the core. See my answer post-nap on the next page.
But when the rod is part of a closed circuit there no longer is charge at its extremes, it has been 'conducted' away to eventual discontinuities in resistivity or permeability. So the tiny battery is no more. You can see the charge at the open terminals of the coil the tiny rod belongs to, or at the interfaces with the resistor that represents either the load or the internal impedance of the voltmeter. But tiny rod is no more a battery.

Regarding partial inductance, I reiterate my reluctance in discussing it in this venue. It will only be a distraction. But I do agree with you (and Bruce Archambeault) that "the concept of inductance, without defining a complete loop of current, is completely meaningless!". I could make only one exception with regard to the straight wire: it does have an internal inductance per unit length. The reason internal inductance can be defined is that you can limit the surface through which you consider the flux: it's delimited by the lateral surface of the rod itself. For a cylindrical conductor you get the well know mu/8pi henries per meter. But, before the KVLers get too excited - that is self-inductance and is usually much smaller than the external inductance which, in Lewin's experiment, has negligible effects.
And yes, I agree on the method to compute partial inductance of a segment of wire, even if I prefer converting the surface integral into the path integral of A (which ends up vanishing at infinity and give no contribution of the 'lateral' sides). But a segment in the dead center of a toroid left me a bit confused as to which direction to use to go to infinity. Never mind.
To sum up my opinion on partial inductance: to me it is just a bean-counting tool that helps identifying which parts of a loop (for which we can talk about proper inductance) contribute the most EMF. But that EMF - as we know - is no longer there once you consider the interaction with the charges that have been displaced.
Title: Re: #562 – Electroboom!
Post by: rfeecs on November 11, 2021, 05:27:04 pm
In short, the wire is never consider alone. It is implicitly considered part of a complete loop.

That may make your theory and math work out more clearly to you, but I think it is a mistake.  It's true that, for example, a wire going straight through a current transformer is considered to be 1 turn because there is typically a return wire somewhere, but that's not why it actually works.  The interaction between a varying magnetic field and the charges in a wire is a local (microscopically so) effect and Faraday's Law is a mathematically proven observation that says that for a given area and a varying total flux through the area, the EMF around the perimeter area adds up to a number, and that number only depends on the rate of change of the total flux.

Yes, so to consider it locally you look more at the differential form of Maxell's equations instead of the integral form.

For your example with the wire segment in the middle of the toroid coil, consider the plane going through the wire.  The electric field goes in a circular direction around the magnetic cores, so two sets of ellipse shapes.  If the wire is at the center, the electric field there would be straight along the wire.

The wire is a good conductor, so it has net zero electric field inside the wire.  At the surface the tangential field must be zero.  So the charge in the wire must arrange itself to counteract the electric field.  This results in the charges moving to the ends of the wire.  One end will be positively charged, the other negative.

So now you know what the fields are and what the charge distribution is.  Problem solved.

What about voltage?  The voltage depends on path.  If you measure the voltage from one end of the wire to the other through the wire, it is zero.  If you measure by taking a path around the magnetic core, you will get the EMF.  If you measure by taking a path that does not go around a magnetic core, you get zero.

By "measure"  you could use a voltmeter, then the "path" is determined by the path of the test leads.

Another theoretical way to "measure" is to take a test charge and move it along a path and integrate the force with distance to get the potential difference.  This is E dot dl.  You don't need an actual wire to "measure" this way, and you don't need a closed loop.

Looks like Sredni posted while I was typing this so he beat me to it.
Title: Re: #562 – Electroboom!
Post by: Sredni on November 11, 2021, 05:32:17 pm
but if you try to measure the voltage with a voltmeter you will necessarily end up linking integer multiples of the EMF.

OK, last question before I actually present something on the locality issue.

Do you need to use a voltmeter (with test leads) to measure voltage?

 >:D

If you are able to measure intensity and direction of the electric field in all points of the path you want the voltage computed without disturbing the fields, no. You can create an instrument that computes the integral for you.
In circuits you usually attach a voltmeter to measure a voltage. What instrument do you use on your bench?
Title: Re: #562 – Electroboom!
Post by: bdunham7 on November 11, 2021, 06:06:04 pm
For your example with the wire segment in the middle of the toroid coil, consider the plane going through the wire.  The electric field goes in a circular direction around the magnetic cores, so two sets of ellipse shapes.  If the wire is at the center, the electric field there would be straight along the wire.

The wire is a good conductor, so it has net zero electric field inside the wire.  At the surface the tangential field must be zero.  So the charge in the wire must arrange itself to counteract the electric field.  This results in the charges moving to the ends of the wire.  One end will be positively charged, the other negative.

So now you know what the fields are and what the charge distribution is.  Problem solved.

What about voltage?  The voltage depends on path

So there's the crux of the problem.  My straight wire, or any partial loop for that matter, ends up with charged ends, which outside of the particular conditions at hand would be considered to be a voltage and could be measured with traditional test instruments.  So what is a voltage? 
Title: Re: #562 – Electroboom!
Post by: Sredni on November 11, 2021, 06:09:20 pm
So, for example, it doesn't matter where in the toroid of the current transformer the wire is, it still reads the same because...math.  And in the Lewin device, it doesn't matter if the solenoid is off center, the total EMF on the ring adds up to the same amount.  That doesn't mean that the EMF is always evenly distributed nor that we can always consider it only as a total around the perimeter.

Who ever says that the EMF is evenly distributed? What is evenly distributed in the conductor is the total electric field. If the ring is off center so that Eind is not tangentially directed in the ring then you will have surface charge on the lateral surface of the conductor that will produce the correct Ecoul that will obliterate it.

We always get to the same point: the induced field is no longer there, in the conductor.
When you and bsfeechannel talked about the channel with rotating current model, you find an effective way to exemplify path dependance, but do not be fooled, that model does not exemplifies what happens in the ring. The resistors (or the gap in the naked coil) will have charge that will neutralize Eind in the conductor, and this is something that is not modeled in the rotating current model, because you would need something that neutralized the flow in the channel.
(I'm writing this in a hurry because I have things to do, but I hope it is clear that what happens in the ring, and what is sorely missed in the KVLers' view, is this compensation effect by the displaced charge. That is the constantly missing piece to correctly interpret the phenomenon.)
Title: Re: #562 – Electroboom!
Post by: Sredni on November 11, 2021, 06:20:09 pm
So there's the crux of the problem.  My straight wire, or any partial loop for that matter, ends up with charged ends, which outside of the particular conditions at hand would be considered to be a voltage and could be measured with traditional test instruments.  So what is a voltage?

Please modify your post to include the "traditional test instrument" and the measuring procedure adopted.

Voltage is (minus) the path integral of the total electric field along a particular path.
(The total electric field is the field that you experience in that point, in that instant of time and in this contenxt is the vectorial sum of Eind and Ecoul).
Title: Re: #562 – Electroboom!
Post by: jesuscf on November 11, 2021, 06:36:21 pm
In short, the wire is never consider alone. It is implicitly considered part of a complete loop.

What about induction heating?  Where is the complete loop?

https://www.youtube.com/watch?v=8i2OVqWo9s0 (https://www.youtube.com/watch?v=8i2OVqWo9s0)
Title: Re: #562 – Electroboom!
Post by: bdunham7 on November 11, 2021, 06:41:06 pm
Please modify your post to include the "traditional test instrument" and the measuring procedure adopted.

What I mean is that if you had the same wire ends but absent any external fields and achieved the exact same concentration of charge at the ends by some other means (like a battery in the middle of the wire), and measuring the voltage with a plain voltmeter.  Before you protest about the battery, I'm not interested in the means, just the concentration (distribution) of charge--which should be definite, even if we have difficulty measuring it.

Voltage is (minus) the path integral of the total electric field along a particular path.
(The total electric field is the field that you experience in that point, in that instant of time and in this contenxt is the vectorial sum of Eind and Ecoul).

[/quote]

That is one definition of voltage.  Can you think of another?  Like you, I'm busy for a while with real work.  I'll try and actually post something tonight worth thinking about instead of posing silly questions.
Title: Re: #562 – Electroboom!
Post by: jesuscf on November 11, 2021, 07:01:59 pm
What I mean is that if you had the same wire ends but absent any external fields and achieved the exact same concentration of charge at the ends by some other means (like a battery in the middle of the wire), and measuring the voltage with a plain voltmeter.  Before you protest about the battery, I'm not interested in the means, just the concentration (distribution) of charge--which should be definite, even if we have difficulty measuring it.

I know the question is not directed at me, but would some sort of thermoelectric emf do?  Because I showed that very example to the same blockheads before and nothing happened.

https://www.eevblog.com/forum/chat/does-kirchhoffs-law-hold-disagreeing-with-a-master/msg2351850/#msg2351850 (https://www.eevblog.com/forum/chat/does-kirchhoffs-law-hold-disagreeing-with-a-master/msg2351850/#msg2351850)
Title: Re: #562 – Electroboom!
Post by: thinkfat on November 11, 2021, 09:05:14 pm
Hm, I think I got my "tapped transformer" figured out. It's actually quite simple, again looking only at the rotational electrical field:

Let's say I have N (N being any even number) turns around a core with a given magnetic flux, the flux causing "some constant EMF". At N/2 turns the transformer winding is tapped.

"N turns" just means walking around the axis of the rotational electric field N times, each completed loop adds "EMF" volts. So after N turns you have N x "EMF", and obviously after N/2 turns you have N/2 x "EMF".

If you think about it, this opens up some interesting transformer "hacks": Say you have a toroidal transformer in your parts bin and the secondary voltage is just a bit too high. It should be possible to correct the voltage without cutting open the transformer and removing turns, by just adding some turns on the outside but in "reverse" direction.
Title: Re: #562 – Electroboom!
Post by: Sredni on November 11, 2021, 09:14:36 pm
Ok, I knew I had to sleep over it. One night didn't do, but a little five minute nap seemed to have done the trick.

Here is your answer to the partial turn question.
The rod will develop a charge at its extreme, there will be a net electric field around the rod, but no voltage if the rod does not go around the core.
As I had written months ago in my answers on SE, the total electric field behaves as a conservative field in all regions of space that do not contain the variable magnetic flux region. So, your rod that sits in a cylinder of space that does not go around the core, will behave as if it were subjected to an electric field produced by sources and sinks of field.

The answer to your question is therefore: at a given instant in time the rod will develop surface charge that will make the electric field inside exactly zero (no matter the resistivity, in this open circuit case). Now, on the exterior there will be a net nonzero electric field Etot, but the surface will be 'equipotential', meaning that the path integral of the electric field on any path going from one point on the rod to any other point on the rod will be zero. Just like in the electrostatic case. (Incidentally, this is the real magic: path integrals that are independent of path. How cool is that?)

And in fact, if I put the voltmeter with its probes inside the torus hole, or anyway in such a manner that the measuremente loop (voltmeter + probes + rod) does not enclose any changing flux, I will read the actual voltage along the rod and also in the space around the rod where the measurement loop is. And that voltage is zero.
Of course if your measurement loop includes the core, then you will read one EMF. But that is not the voltage along (and around, if nearby) the rod.

Sorry for the delay. Quality sleep is everything.



Now, to address your last post...

1) You did not name the instrument, nor did you show the measurement procedure. Did you realize that?
2) You say
" if you had the same wire ends but absent any external fields and achieved the exact same concentration of charge at the ends by some other means"
So, you want to take away half of the physical excitation that make your system what it is. Sure, if you pin the charges with nails and turn off the current in the transformer so as to kill the flux, you will see an electric field in the space where the rod was, and you will measure a voltage that will be different from zero. But the configuration of the electric field will be different from that around the rod when the transformer is powered.
And I know where you are going (much in the same way that yesterday I knew we would have ended up discussing partial inductances :-) ). You want to define voltage as the scalar potential difference.
You are free to do that, and some do, but you must be aware that with that definition of voltage you can't do anything useful without supplying the magnetic vector potential A. The reason, as I said before, is that it the scalar electric potential is insufficient to completely characterize the system. You cannot even apply Ohm's law.

And electrons, in the rod, in the coil, in the space around it, respond to the TOTAL electric field, not the partial component Ecoul (that is responsible for the scalar potential part of the voltage).
Title: Re: #562 – Electroboom!
Post by: Sredni on November 11, 2021, 09:21:40 pm
Humor me, guys. See the attached picture. All wires are actual wires and interact with fields, but have negligible resistance. R1 can be any value, I don't think it matters. Assume EMF=1V. Predict the voltages shown on V1 and V2, using KVL and Faradays Law.

V1 is zero, V2 is 1V. (didn't check the sign just saw the thumbnail), might as well be -1V
It's easy if you check if your measurement loop (voltmeter + probes + branch you want to know the voltage of) contains or does not contain the flux.

Now I have a question for you.
Suppose I place a loop made with one resistor, the voltmeter, and its probes, near the magnetic flux region. Would you spend even a microsecond of your time to check that the induced field is being compensated? Or would you rather go: "nah, there is no flux in my measurement loop, I can apply KVL, no problem!"

EDIT: Yep, the transformer trick would do. IIRC Big Clive used the whole secondary of a transformer to step down the voltage. From 240-->12V to 240 ---> 240-12V or something like that. Caveat: the current must be sustained by both windings.

In principle we could use the radial probing to extract a finely varying voltage from a solenoidal coil immersed in a magnetic field. Instead of going one turn at the time, we can have a continuous (well, only marred by asperities in contact) voltage from start to finish. Mechanical nightmare, of course.
Title: Re: #562 – Electroboom!
Post by: bdunham7 on November 11, 2021, 11:49:43 pm
I know the question is not directed at me, but would some sort of thermoelectric emf do?  Because I showed that very example to the same blockheads before and nothing happened.

I don't know, but I'd like to use as many concepts that we all agree on as I can.
Title: Re: #562 – Electroboom!
Post by: bdunham7 on November 11, 2021, 11:55:10 pm
1) You did not name the instrument, nor did you show the measurement procedure. Did you realize that?

I didn't have one in mind.  An infinite impedance voltmeter with test leads.  Or negligibly close to infinite impedance. 

Quote
And I know where you are going (much in the same way that yesterday I knew we would have ended up discussing partial inductances :-) ). You want to define voltage as the scalar potential difference.
You are free to do that, and some do, but you must be aware that with that definition of voltage you can't do anything useful without supplying the magnetic vector potential A. The reason, as I said before, is that it the scalar electric potential is insufficient to completely characterize the system. You cannot even apply Ohm's law.

You're catching on!  And Ohms law indeed may not always work.  So what?  What I'm getting at isn't really physics, it is understanding the nature of the disagreement between the 'KVLers' and the 'Lewinites'.  I think I've figured it out, so I'm now going to write a very long post.

Title: Re: #562 – Electroboom!
Post by: jesuscf on November 12, 2021, 12:07:34 am
Suppose I place a loop made with one resistor, the voltmeter, and its probes, near the magnetic flux region. Would you spend even a microsecond of your time to check that the induced field is being compensated? Or would you rather go: "nah, there is no flux in my measurement loop, I can apply KVL, no problem!"

Do you mean a loop like the one in the attached picture?  If so, and if you measure correctly, KVL works perfectly, once again!
Title: Re: #562 – Electroboom!
Post by: bdunham7 on November 12, 2021, 01:07:50 am
OK, I've asked a lot of pointed questions, so it's time for me to stick my neck out and say something.  First, when I first saw this debacle, I didn't really understand it, but it seemed like a non-issue to me.  Still, such debates should be about learning and discovery, not belittling, name-calling and all the sorts of ugly pettiness that have shown up.  It doesn't help that both Lewin and Mehdi are of the attention-seeking type.  What struck me as interesting is that of all the stuff that has been posted here, Youtube, wherever, it never becomes clear what the disagreement is about.  People say Lewin is 'wrong', others say he is 'right'.  What does that even mean?  Did he rig his demonstration?  Here's my interpretation of the disagreement (warning: there's no real resolution), anyone who differs please be specific about which point we depart at! 

So here's a drawing just like the myriads of others we've all seen:

(https://www.eevblog.com/forum/amphour/562-electroboom!/?action=dlattach;attach=1321133;image)

I think we all agree that in this diagram, both V1 and V2 will read the same voltage.  I think both camps also agree that voltages V(A0-A2) and V(A0-AV2) are the same.  The point of departure is that the 'KVLers' would say that the two voltages are some number and they cancel each other out, so the both voltmeters read just the voltage across R2.  That number, of course, would vary with the position of R2 because it posits the existence of a continual voltage gradient on the wire.  The 'Lewinites' seem to be asserting that both voltages are zero and that the position of A0 doesn't matter. 

Using the definition of voltage as being the work required to move a test charge along a path, it seem pretty clear that going from A0 to D0 through R1 takes a different amount of work than going through R2.  Thus they have different voltages and of opposite sign.  However, when we are measuring with a voltmeter, neither of those paths is what we are referring to--it is the path of the test leads going to the voltmeter.  (I initially stuck on this point!)  I haven't seen any mathematical proof, but it appears that things work out so that even though there are infinitely many paths to arrange those test leads, if you stay in-plane (or possibly even not, but I haven't thought about that) and don't cross the ring, your voltmeter will always read either the voltage across R2 or R1, without any intermediate results.  Again, either there is no induced voltage on the test leads nor (A0-A2) and (D0-D2), or they always add up to the same number and cancel.

 Some have tried probing out-of-plane perpendicular to the ring and gotten different results, but the counter is that this method somehow 'ignores' something--the induced EMF field--and thus is wrong.  It has been asserted here that you can't have a voltage on a conductor that is a partial turn or a partial ring.  Again, there are two apparent explanations, one that until the loop is complete there is no voltage and the other that there is a voltage, but you can't measure it because any probe you use will either cancel or complete the loop. 

I earlier had alluded to the concept of 'absolute' voltage.  It might sound silly, but I checked to make sure that Lewin had actually taught this earlier in the course.  In electrostatics, you learn and then apparently promptly forget that a conductive sphere of radius r will have a capacitance of C=4*pi*E0r.  And, since Q=CV, V=Q/C.  There's a perfectly legitimate definition of voltage derived directly from fundamental constants.  Now before the howls of protest start, please remember I'm trying to resolve the issue of what the disagreement is about, not who is right.

(https://www.eevblog.com/forum/amphour/562-electroboom!/?action=dlattach;attach=1321139;image)

So now I'm going to measure voltage in a different way, one that is never path dependent--brass balls.  I'm going to take two hollow brass balls of a precisely known radius r hanging from an insulated string.  Since I know r, I know C.  I'm going to arrange a partial turn or partial ring or even a straight rod, going through a changing magnetic field, then extend the ends out to the point where the fields are negligible.  Then while the field is in state of linear change  (call this the MQS period, or Magneto-Quasistatic period) I'm going to touch the brass balls to the ends of the rod so that they accumulate a charge, keeping them there until equilibrium is reached.  Then I will separate them from the rod before the MQS period ends.  Next I will take the brass balls back to my Couloumb meter lab where I will proceed to precisely determine what the charge was on each ball, then I will know each voltage from V=Q/C.  The difference between the two is my voltage, by this definition, between the two points.  If I use this method on the Lewin ring, I think the results will match the perpendicular probing method.

So that, IMO, is the nature of the disagreement.  As others have said, in a conductor in a varying field, there will be two forces EIND and ECOUL that will will balance out in the MQS domain, because that is what conductors do--there can be no electric field in a conductor.  However, that doesn't negate the fact that there is a charge imbalance over the conductor.  When it comes to partial turns, I think the mathematical types here have done themselves a disservice by insisting that we only consider full loops because any voltage measurement will make a complete loop.  The complete loop doesn't matter if the gap is beyond the non-negligible area of flux.  If you took the rod in my example and bent it into a U-shape and then extended it far enough out so that the flux from the torus was negligible, you would get the same reading from a voltmeter as  you would from the brass ball technique.  Completing the loop satisfies the math, but doesn't matter in the real world because the work has been done locally in the area near the torus.

So that post wasn't as long as I thought it would be.  Lewin is not 'wrong'.  But Lewin is wrong!  According to my absolute definition, it is not KVL that fails in a varying magnetic field, it is the definition of voltage being the integral of E dot dL (sorry--I can't find notation symbols) that fails in a non-conservative field.  I can hear howls of protest.  But think about it!

Title: Re: #562 – Electroboom!
Post by: Sredni on November 12, 2021, 01:14:46 am
Suppose I place a loop made with one resistor, the voltmeter, and its probes, near the magnetic flux region. Would you spend even a microsecond of your time to check that the induced field is being compensated? Or would you rather go: "nah, there is no flux in my measurement loop, I can apply KVL, no problem!"

Do you mean a loop like the one in the attached picture?  If so, and if you measure correctly, KVL works perfectly, once again!

You are the one calling others 'blockheads'.
Seems to me you have yet to finish coloring your EM book.

We are talking infinitely long solenoids and toroids to confine the magnetic flux in the core. And you come up with a pancake?
Don't you even realize what is wrong with your setup?
Title: Re: #562 – Electroboom!
Post by: Sredni on November 12, 2021, 01:24:26 am
It has been asserted here that you can't have a voltage on a conductor that is a partial turn or a partial ring.

Sorry, you are mistaken. You can surely have a voltage on a partial ring, but you need to have the ring plus the path you are referring it to go around the magnetic flux region. This is how transformer work, and where the L di/dt formula for inductors is derived.
I haven't read the rest yet, but I have to stop you right here.

I'll tell you what my crystal ball forecasts: that your method to measure voltage across the rod will force you to move the sensing instrument on a path that, together with the rod, will form a closed path around the variable magnetic region.

Now, I'll read the rest.
<pause>

Oh. I read the rest. It started interesting, but then... I find many points that do not ring right, but I'll start with this.

Forget nonconservative fields. Put your rod in a conservative field, an electrostatic field. It will 'charge up' by induction. You will have opposite charge at the extremes. Now measure your absolute voltage with your method. It should be zero.
Is it zero?
Title: Re: #562 – Electroboom!
Post by: bdunham7 on November 12, 2021, 01:33:36 am
I'll tell you what my crystal ball forecasts: that your method to measure voltage across the rod will force you to move the sensing instrument on a path that, together with the rod, will form a closed path around the variable magnetic region.

No, your insistence on a closed path is your downfall.  You only insist on it because it makes your math work. 
Title: Re: #562 – Electroboom!
Post by: jesuscf on November 12, 2021, 01:35:54 am
Suppose I place a loop made with one resistor, the voltmeter, and its probes, near the magnetic flux region. Would you spend even a microsecond of your time to check that the induced field is being compensated? Or would you rather go: "nah, there is no flux in my measurement loop, I can apply KVL, no problem!"

Do you mean a loop like the one in the attached picture?  If so, and if you measure correctly, KVL works perfectly, once again!

You are the one calling others 'blockheads'.
Seems to me you have yet to finish coloring your EM book.

We are talking infinitely long solenoids and toroids to confine the magnetic flux in the core. And you come up with a pancake?
Don't you even realize what is wrong with your setup?

Sredni you are talking about many things at once.  The trademark of a bullshitter!  Paragraphs and paragraphs: just deflection and more deflection.  Concentrate in the point at hand:  did Lewin measured the circuit correctly or not?  Are his conclusions correct? The answer is NO for both questions as Electroboom and others have demonstrated.

By the way have you done the experiment yourself?  Where is your setup?  Can you show us?

EDIT: Here is another video from 'fromjesse'.  The title says it all: "The Lewin loop inside an iron core - KVL still holds"

https://www.youtube.com/watch?v=iDWv8QJrzUo (https://www.youtube.com/watch?v=iDWv8QJrzUo)



Title: Re: #562 – Electroboom!
Post by: Sredni on November 12, 2021, 01:51:45 am
No, your insistence on a closed path is your downfall.  You only insist on it because it makes your math work.

It's all Stokes' fault. I ain't do nothin'!!!
And it's not my math. I wish, oh how I wish it was. And it's not even Lewin's.
KVLers seem to think Lewin has come up with some extravagant theory, but this is not his theory: it's plain and simple classical electrodynamics. When Lewin said 'all textbook are wrong' he meant introductory books talking about how to solve LR circuit. The 5 + 3 - 8 = 0 vs 5 + 3 = 8 way to apply KVL or Faraday in lumped dynamic circuits. There are tons of EM books that agree with Lewin on this matter. Purcell is one of them. He treats the ring explicitly. Haus and Melcher is another one. Faria is another. I can go on an on... There are video from professors of MIT, Purdue University, Cornell University that apply Faraday the way Lewin does, and that say that voltage is no longer unique in a nonconservative settings.
(And who do you have? A comedian and three guys in a garage?)

Please, open an EM book. One that is not about coloring pictures.

And yes, I did the experiment myself. My transformer is in the guest room, under a nightstand; my Lewin ring is in a box where I keep jumpers and connectors; my scope in on my desk. Why? Do you doubt the result of the measurement? Everybody agrees on those. Why wasting time showing another identical experiment?
Title: Re: #562 – Electroboom!
Post by: bdunham7 on November 12, 2021, 01:58:04 am
Forget nonconservative fields. Put your rod in a conservative field, an electrostatic field. It will 'charge up' by induction. You will have opposite charge at the extremes. Now measure your absolute voltage with your method. It should be zero.
Is it zero?

Why would it be zero?
Title: Re: #562 – Electroboom!
Post by: jesuscf on November 12, 2021, 02:53:15 am

KVLers seem to think Lewin has come up with some extravagant theory, but this is not his theory: it's plain and simple classical electrodynamics. When Lewin said 'all textbook are wrong' he meant introductory books talking about how to solve LR circuit. The 5 + 3 - 8 = 0 vs 5 + 3 = 8 way to apply KVL or Faraday in lumped dynamic circuits. There are tons of EM books that agree with Lewin on this matter. Purcell is one of them. He treats the ring explicitly. Haus and Melcher is another one. Faria is another. I can go on an on... There are video from professors of MIT, Purdue University, Cornell University that apply Faraday the way Lewin does, and that say that voltage is no longer unique in a nonconservative settings.
(And who do you have? A comedian and three guys in a garage?)

So how comes that when you eliminate the influence of the varying magnetic field on the measuring instrument KVL works perfectly?  Here are the results from 'fromjesse' (from his garage I guess?):

Left resistor: -61mV
Right resistor: -660mV

Total voltage dropped: -721mV

Top copper: 378mV
Bottom copper 350mV

Total voltage induced 728 mV

Total sum: 7 mV (which within the tolerance of the measuring instrument)

Please, open an EM book. One that is not about coloring pictures.

I have!  As a matter of fact, I tediously transcribed a whole section of one (with equations and figures!) in a similar thread a couple of years back. You know, just to make sure everybody was talking about the same f*king thing:

https://www.eevblog.com/forum/chat/does-kirchhoffs-law-hold-disagreeing-with-a-master/msg2352807/#msg2352807 (https://www.eevblog.com/forum/chat/does-kirchhoffs-law-hold-disagreeing-with-a-master/msg2352807/#msg2352807)

And yes, I did the experiment myself. My transformer is in the guest room, under a nightstand; my Lewin ring is in a box where I keep jumpers and connectors; my scope in on my desk. Why? Do you doubt the result of the measurement? Everybody agrees on those. Why wasting time showing another identical experiment?

I asked because you were very quick into mocking my setup!  In a previous message it was somehow suggested that a ring made of resistors will behave differently than a copper ring.  I was pretty sure it will work exactly the same, so I made one.  And yes, KVL works (again)!

https://www.eevblog.com/forum/amphour/562-electroboom (https://www.eevblog.com/forum/amphour/562-electroboom)!/msg3806177/#msg3806177
Title: Re: #562 – Electroboom!
Post by: bsfeechannel on November 12, 2021, 03:48:25 am
What struck me as interesting is that of all the stuff that has been posted here, Youtube, wherever, it never becomes clear what the disagreement is about.

It is very simple. Look at your straight wire again at an arbitrary position near a varying magnetic field.

(https://www.eevblog.com/forum/amphour/562-electroboom!/?action=dlattach;attach=1321190;image)

We see you understand that the field inside the wire will be zero, so no voltage inside the wire. But you have charges of opposite polarity on each end.

What is voltage? It is the potential energy electric charges have to go from one point to another. The positive charges (using conventional current) won't be able to go through the red no-no path because you can see you have lines of non-conservative electric field opposing their way (I am omitting the conservative field generated by those charges).  So, the voltage between the two ends of the wire will be zero through these paths. For you to make them go, you would have to provide an additional electric field in this region. This is KVL.

However, if they "choose" the green yes path, they'll succeed because you have lines of electric field actually going from the positive to the negative charges. So their voltages will not be zero. You don't need to apply an additional field to move them.

What KVLers are saying essentially is that this voltage is zero, i.e., the charges can't go through the green path and that the electric field that is moving them must be inside the wire. Which is absurd.

The addition of meter probes doesn't affect what you are going to measure through the red or green paths. And what is more annoying is that when KVLers measure the voltages through the green path and the red path and encounter exactly what the theory predicts, they say that the theory is wrong.
Title: Re: #562 – Electroboom!
Post by: bdunham7 on November 12, 2021, 03:59:52 am
What is voltage? It is the potential energy electric charges have to go from one point to another.

That's one definition.  The whole point of my post is that there is another, whether you believe it to be an acceptable one or not.  The nature of the disagreement, IMO, is that your KVLers seem to have instinctively adopted the definition that matches my explanation of absolute voltage.  In KVL-world, your definition of voltage is what fails in a non-conservative field, not KVL. 

Quote
The addition of meter probes doesn't affect what you are going to measure through the red or green paths. And what is more annoying is that when KVLers measure the voltages through the green path and the red path and encounter exactly what the theory predicts, they say that the theory is wrong.

I think the KVLers have settled on the approach of using test probes that are perpendicular to the plane (it's not a two-dimensional universe)  and extending those leads far enough out that the rotational E-field is negligible.  So no in-plane paths.  This eliminates the 'interference' that would otherwise prevent an accurate measurement. 
Title: Re: #562 – Electroboom!
Post by: Sredni on November 12, 2021, 04:22:56 am
Forget nonconservative fields. Put your rod in a conservative field, an electrostatic field. It will 'charge up' by induction. You will have opposite charge at the extremes. Now measure your absolute voltage with your method. It should be zero.
Is it zero?
Why would it be zero?

Well, in my universe conductors in electrostatics are equipotentials. (It has something to do with a certain path independence...)
Is it not the case in your universe, with your definition of voltage?
Because if it is not, you are gonna rewrite a great deal of physics.

So, will your method give zero voltage between two points on the surface of a conductor, in electrostatic conditions? Can you prove it?
Title: Re: #562 – Electroboom!
Post by: bdunham7 on November 12, 2021, 04:27:05 am
There are tons of EM books that agree with Lewin on this matter. Purcell is one of them. He treats the ring explicitly. Haus and Melcher is another one. Faria is another. I can go on an on... There are video from professors of MIT, Purdue University, Cornell University that apply Faraday the way Lewin does, and that say that voltage is no longer unique in a nonconservative settings.
(And who do you have? A comedian and three guys in a garage?)

I'll assume that the rest of your comments aren't directed at me.  No longer unique?  Sort of reminds me of the Second Vatican Council abolishing purgatory. 

I'm agnostic, I don't believe there are 'right' and 'wrong' models in physics (OK, there are some wrong ones  :)  )  rather there are different models that allow you to solve different problems.  Actually, now that I think about it, perhaps ALL models are wrong, it's just that some of them are close enough that they'll do for one purpose or another.
Title: Re: #562 – Electroboom!
Post by: Sredni on November 12, 2021, 04:36:55 am
So how comes that when you eliminate the influence of the varying magnetic field on the measuring instrument KVL works perfectly?  Here are the results from 'fromjesse' (from his garage I guess?):
I have already explained how fromjesse and Mabilde obtain that reading. They are measuring the contribute of the Ecoul field in their probes. And that contribute is equal to the contribute of Ecoul in the arc they measure.
A good setup to find a partial component of the actual voltage.

Quote
Please, open an EM book. One that is not about coloring pictures.

I have!  As a matter of fact, I tediously transcribed a whole section of one (with equations and figures!) in a similar thread a couple of years back. You know, just to make sure everybody was talking about the same f*king thing:

https://www.eevblog.com/forum/chat/does-kirchhoffs-law-hold-disagreeing-with-a-master/msg2352807/#msg2352807 (https://www.eevblog.com/forum/chat/does-kirchhoffs-law-hold-disagreeing-with-a-master/msg2352807/#msg2352807)

Good. I have that book (a little bit too superficial for my tastes but all in all okeyish - I like his circuit book better, the old editions).
Let's see... from what you copied:

Quote
The closed line integral is taken along the filament, directly between the capacitor plates and points 0 and 1, as indicated by the dashed line.  The contribution from the perfectly conducting filament is zero, because tangential E must be zero there; this includes the helix, surprising as that may be.

You do realize that this - and all the rest - is exactly what Lewin says in his video "Kirchhoff for the birds"  https://youtu.be/LzT_YZ0xCFY (https://youtu.be/LzT_YZ0xCFY)  from  minute 27? Integral of E.dl along the coil wire is zero, but voltage across the inductor L di/dt.
Did you not understand it the first time you read it?

And it seems you have yet to realize what is wrong with your pancake setup.
Title: Re: #562 – Electroboom!
Post by: Sredni on November 12, 2021, 04:45:55 am
voltage is no longer unique in a nonconservative settings.
No longer unique?  Sort of reminds me of the Second Vatican Council abolishing purgatory. 

No longer uniquely determined by endpoints alone. You need to specify the path. They are path dependent. The total electric field does not admit a potential function. The circulation of the total electric field is not always zero. rot E !=0.

So, will your definition of voltage give zero for [edit maybe this you did not get: pairs of) points on a conductor in electrostatic conditions? To be more clear when you measure VAB = VB-VA will it always be zero for all pair of points on a conductor?
Title: Re: #562 – Electroboom!
Post by: bsfeechannel on November 12, 2021, 04:57:50 am
That's one definition.  The whole point of my post is that there is another, whether you believe it to be an acceptable one or not.  The nature of the disagreement, IMO, is that your KVLers seem to have instinctively adopted the definition that matches my explanation of absolute voltage.  In KVL-world, your definition of voltage is what fails in a non-conservative field, not KVL. 

I was using YOUR definition of voltage, which really fails in a non conservative field, you're right.

The definition of voltage is the path integral of the electric field, which does not fail for a conservative nor for a non-conservative field.

Quote
I think the KVLers have settled on the approach of using test probes that are perpendicular to the plane (it's not a two-dimensional universe)  and extending those leads far enough out that the rotational E-field is negligible.  So no in-plane paths.  This eliminates the 'interference' that would otherwise prevent an accurate measurement.

There's no "interference" when your probes are in the plane. Between the probes connected to R1 and R2 you'll only find the electric field that are in these resistors and nothing else.

In the KVLer approach, the probes form a loop drenched in the varying magnetic field. Funny that you mentioned a three dimensional world but you failed to consider the three dimensional loop that you configured with that arrangement.

(https://www.eevblog.com/forum/amphour/562-electroboom!/?action=dlattach;attach=1321232;image)

All of the assumptions KVLers make always lead to contradictions.

That's why it is derided and dismissed as a serious theory.
Title: Re: #562 – Electroboom!
Post by: jesuscf on November 12, 2021, 04:59:25 am
So how comes that when you eliminate the influence of the varying magnetic field on the measuring instrument KVL works perfectly?  Here are the results from 'fromjesse' (from his garage I guess?):
I have already explained how fromjesse and Mabilde obtain that reading. They are measuring the contribute of the Ecoul field in their probes. And that contribute is equal to the contribute of Ecoul in the arc they measure.
A good setup to find a partial component of the actual voltage.

It looks to me that your are in denial.  Here, read the transcript (or watch the video) of the conclusion from the video from Bob DuHamel from the YouTube channel RSD Academy.  He can explain it way better than me:

Dr. Lewin is disagreeing with the vast majority of the scientific establishment he's disagreeing with the vast majority of textbooks he is disagreeing with the vast majority of professors of both electrical engineering and physics and his postulation that Kirchhoff's voltage law doesn't hold is based on a incorrect application of ohm's law not knowing how ohm's law has to be applied to a voltage source.

Then he performs an experiment to prove his premise but he doesn't take precautions to make sure that the magnetic fields don't affect his measurements.  In the end he says “look at the math proves I'm right” well the math is right.  He is right about the non-conservative fields and the conservative fields, and if we put a loop of wire in that non-conservative field that loop of wire gets a current and the electric fields in the wire are still non-conservative.  But as soon as we either make a gap in the loop or put some resistance in there now we have a backup of energy where that gap is or where the resistors are, where, we get now a measurable voltage, a measurable difference in potential where we are now going up in potential and then back down in potential and we are no longer non-conservative.  We are now conservative so we end up with a conservative circuit or conservative fields in the circuit even though they're embedded in a non-conservative field.
Title: Re: #562 – Electroboom!
Post by: bsfeechannel on November 12, 2021, 05:32:22 am
There are tons of EM books that agree with Lewin on this matter. Purcell is one of them. He treats the ring explicitly. Haus and Melcher is another one. Faria is another. I can go on an on... There are video from professors of MIT, Purdue University, Cornell University that apply Faraday the way Lewin does, and that say that voltage is no longer unique in a nonconservative settings.
(And who do you have? A comedian and three guys in a garage?)

This does not add to the argument. It is just an observation, but it is interesting to note that the KVLers' champions are always academic and engineering underdogs. They have no paper, scientific or engineering, published by a reputable scientific or engineering publisher with a sound and consistent argument that KVL holds for varying magnetic fields. Just a bunch videos with contradictory statements, Lewin bashing and the usual request for subscriptions, likes, and membership money.
Title: Re: #562 – Electroboom!
Post by: bdunham7 on November 12, 2021, 05:36:10 am
So, will your method give zero voltage between two points on the surface of a conductor, in electrostatic conditions? Can you prove it?

I misunderstood your question, thinking you were referring to a single absolute measurement, not a relative measurement.  My brass balls can give both!

Thank you for asking for proof!  This prevented me from stating what I thought was an obvious answer.  Don't ask what that was, it isn't obvious anymore!

I'm going to have to think about that question.  My only thought so far is that if the result were not zero, I would have a mechanism for perpetually extracting energy from a conservative field.

Title: Re: #562 – Electroboom!
Post by: bdunham7 on November 12, 2021, 05:47:48 am
This does not add to the argument. It is just an observation, but it is interesting to note that the KVLers' champions are always academic and engineering underdogs. They have no paper, scientific or engineering, published by a reputable scientific or engineering publisher with a sound and consistent argument that KVL holds for varying magnetic fields. Just a bunch videos with contradictory statements, Lewin bashing and the usual request for subscriptions, likes, and membership money.

I would like to note that the universe doesn't care about our discussion.  My first impression of the matter was that it was much ado about nothing and just one of those quirks that I don't fully understand but don't need to.  But here we are.  I still think it is much ado about nothing--perhaps no more than a notational convention--but perhaps I and others will learn some stuff in the process.  Perhaps one reason Lewin chose to present this the way he did was to fire up his bored students, perhaps he likes the attention--as seems the case. 
Title: Re: #562 – Electroboom!
Post by: Sredni on November 12, 2021, 05:54:26 am
Here, read the transcript (or watch the video) of the conclusion from the video from Bob DuHamel from the YouTube channel RSD Academy.  He can explain it way better than me:

Dr. Lewin is disagreeing with the vast majority of the scientific establishment he's disagreeing with the vast majority of textbooks he is disagreeing with the vast majority of professors of both electrical engineering and physics and his postulation that Kirchhoff's voltage law doesn't hold is based on a incorrect application of ohm's law not knowing how ohm's law has to be applied to a voltage source.

Then he performs an experiment to prove his premise but he doesn't take precautions to make sure that the magnetic fields don't affect his measurements.  In the end he says “look at the math proves I'm right” well the math is right.  He is right about the non-conservative fields and the conservative fields, and if we put a loop of wire in that non-conservative field that loop of wire gets a current and the electric fields in the wire are still non-conservative.  But as soon as we either make a gap in the loop or put some resistance in there now we have a backup of energy where that gap is or where the resistors are, where, we get now a measurable voltage, a measurable difference in potential where we are now going up in potential and then back down in potential and we are no longer non-conservative.  We are now conservative so we end up with a conservative circuit or conservative fields in the circuit even though they're embedded in a non-conservative field.


And that's a lot of... alternative truths. To say the least.
But you probably won't find many confutations in the comments section, because Duhamel, like fromjesse, delete the comments that tell them they are wrong and why. For example, in his last video "Voltage in a loop is weird", a post by Silicon Soup stating that by putting the probes inside the magnetic region a voltage was induced in the loop has disappeared.

Here's the reason:


https://i.postimg.cc/vZ6BM4Wn/screenshot-3.png

Not so sure what is a "protected man", but I have some idea.

This has a lot in common with cults: silence the critical voices and surround yourself with bootlickers.

(and you have no idea how many other videos on this topic Duhamel has put out, only to redo them to correct the... alternative truths there were pointed out in the comment section. Now some of those videos are no longer listed and so the comments pointing out the... alternatively accurate statements are no longer visible.
Title: Re: #562 – Electroboom!
Post by: jesuscf on November 12, 2021, 05:57:34 am
There are tons of EM books that agree with Lewin on this matter. Purcell is one of them. He treats the ring explicitly. Haus and Melcher is another one. Faria is another. I can go on an on... There are video from professors of MIT, Purdue University, Cornell University that apply Faraday the way Lewin does, and that say that voltage is no longer unique in a nonconservative settings.
(And who do you have? A comedian and three guys in a garage?)

This does not add to the argument. It is just an observation, but it is interesting to note that the KVLers' champions are always academic and engineering underdogs. They have no paper, scientific or engineering, published by a reputable scientific or engineering publisher with a sound and consistent argument that KVL holds for varying magnetic fields. Just a bunch videos with contradictory statements, Lewin bashing and the usual request for subscriptions, likes, and membership money.

Suuure!  Tell that to this measurements from 'fromjesse' and see how they change based in your BS statement above:

Left resistor: -61mV
Right resistor: -660mV

Total voltage dropped: -721mV

Top copper: 378mV
Bottom copper 350mV

Total voltage induced 728 mV

Total sum: 7 mV (which is within the tolerance of the measuring instrument)

Also bsfeechannel, what are your credentials?  Are you an electrical engineer or not?  All of a sudden it became important to you to have a reputation to talk about the subject.

Title: Re: #562 – Electroboom!
Post by: jesuscf on November 12, 2021, 06:07:11 am
And that a lot of... alternative truths. To say the least.
But you probably won't find many confutations in the comments section, because Duhamel, as fromjesse delete the comments that tell them they are wrong and why. For example, in his last video "Voltage in a loop is weird", a post by Silicon Soup stating that by putting the probes inside the magnetic region a voltage was induced in the loop has disappeared.

You can make out all the conspiratorial BS you want.  The results are there, KVL works perfectly when the circuit is in variying magnetic field and you measure correctly, as shown by 'fromjesse' experiment:

Left resistor: -61mV
Right resistor: -660mV

Total voltage dropped: -721mV

Top copper: 378mV
Bottom copper 350mV

Total voltage induced 728 mV

Total sum: 7 mV (which is within the tolerance of the measuring instrument)
Title: Re: #562 – Electroboom!
Post by: bsfeechannel on November 12, 2021, 06:08:20 am
I would like to note that the universe doesn't care about our discussion. 

Why would it?

Quote
My first impression of the matter was that it was much ado about nothing and just one of those quirks that I don't fully understand but don't need to.

For most people, it is irrelevant. No doubt.

Quote
But here we are.  I still think it is much ado about nothing--perhaps no more than a notational convention

It's more serious than that. It's about pseudo-scientific claims.

Quote
--but perhaps I and others will learn some stuff in the process. 

You have no idea. I learned a ton with all of this.

Quote
Perhaps one reason Lewin chose to present this the way he did was to fire up his bored students, perhaps he likes the attention--as seems the case.

Being a scientist, he obviously cares about rigor. But he is right. We engineers tend to cut corners, rely on rules of thumb, and make approximations. Sometimes we even give credit do certain myths. But it's a good thing to be reminded of the true nature of reality.
Title: Re: #562 – Electroboom!
Post by: bsfeechannel on November 12, 2021, 06:30:25 am
Dr. Lewin is disagreeing with the vast majority of the scientific establishment he's disagreeing with the vast majority of textbooks he is disagreeing with the vast majority of professors of both electrical engineering and physics and his postulation that Kirchhoff's voltage law doesn't hold is based on a incorrect application of ohm's law not knowing how ohm's law has to be applied to a voltage source.

Is there some objective data to substantiate this claim? I guess not. But it is interesting to note that guys like Niels Bohr and Richard Feynman disagreed with the vast majority of textbooks and professors of their time.

And they were right.

Quote
And that a lot of... alternative truths. To say the least.

You mean lies? Here's is one. He said that some say that the voltage in a loop of wire closed by a resistor under a varying magnetic field is in the resistor only. He said that this is philosophy and that no one can prove it. Not only the theory predicts that, but also you can hook up a meter across the resistor and measure its voltage, and across the wire and measure zero volts. The voltage is only across the resistor. No metaphysics. Only physics.
Title: Re: #562 – Electroboom!
Post by: bdunham7 on November 12, 2021, 06:32:00 am
It's more serious than that. It's about pseudo-scientific claims.

The noise-to-signal ratio is pretty high, but the perpendicular probing proponents are on to something IMO, although many of them may not know what it is.  And in trying to answer Sredni's last question, it may be slowly dawning on me how to show how the non-conservative example is different.  Hopefully not in a convoluted, roundabout way.
Title: Re: #562 – Electroboom!
Post by: bdunham7 on November 12, 2021, 06:41:47 am
you can hook up a meter across the resistor and measure its voltage, and across the wire and measure zero volts. The voltage is only across the resistor. No metaphysics. Only physics.

Or 'bad probing'.  :-DD

But seriously, you can also hook up a meter and measure voltage on both.

(https://www.eevblog.com/forum/amphour/562-electroboom!/?action=dlattach;attach=1321259;image)
Title: Re: #562 – Electroboom!
Post by: bdunham7 on November 12, 2021, 06:55:45 am
In the KVLer approach, the probes form a loop drenched in the varying magnetic field. Funny that you mentioned a three dimensional world but you failed to consider the three dimensional loop that you configured with that arrangement.

I believe they choose their loop so that it bisects the solenoid so that there is no net flux.  Or at least that is what is attempted and claimed.

Quote
I was using YOUR definition of voltage, which really fails in a non conservative field, you're right.

Fails at what?  The two definitions are irreconcilable as far as I can see in this specific instance.  That's the issue.
Title: Re: #562 – Electroboom!
Post by: jesuscf on November 12, 2021, 07:03:29 am
Not only the theory predicts that, but also you can hook up a meter across the resistor and measure its voltage, and across the wire and measure zero volts. The voltage is only across the resistor. No metaphysics. Only physics.

Wow, really! :palm: You hadn't learn anything after 7 pages of comments!   If your statement is true ("the voltage across the wire is zero volts"), please explain why the voltage across the copper strip in the picture below is 184mv.  Notice that the 'hands' of the Lewin clock are at angle of about 90 degrees; remember that the 'bottom copper' voltage is 350mV.  That is from the video from 'fromjesse' I linked in the previous messages.  By the way, that is exactly the same mistake Lewin makes, because under the influence of a varying magnetic field, a real piece of wire behaves as non-ideal voltage source.


Title: Re: #562 – Electroboom!
Post by: jesuscf on November 12, 2021, 07:13:32 am
In the KVLer approach, the probes form a loop drenched in the varying magnetic field. Funny that you mentioned a three dimensional world but you failed to consider the three dimensional loop that you configured with that arrangement.
I believe they choose their loop so that it bisects the solenoid so that there is no net flux.  Or at least that is what is attempted and claimed.

bdunham7 you are technically correct.  The best kind of correct!!!
Title: Re: #562 – Electroboom!
Post by: thinkfat on November 12, 2021, 08:07:37 am
Humor me, guys. See the attached picture. All wires are actual wires and interact with fields, but have negligible resistance. R1 can be any value, I don't think it matters. Assume EMF=1V. Predict the voltages shown on V1 and V2, using KVL and Faradays Law.

V1 is zero, V2 is 1V. (didn't check the sign just saw the thumbnail), might as well be -1V
It's easy if you check if your measurement loop (voltmeter + probes + branch you want to know the voltage of) contains or does not contain the flux.

I'd say vice versa, because the path through V2 is not in the magnetic flux and the branch voltage is zero, so I'd think V2=0 and V1=1V (or -1V, I didn't check the field orientation either). If I'm wrong, I'd really like to hear your explanation.

Quote
Now I have a question for you.
Suppose I place a loop made with one resistor, the voltmeter, and its probes, near the magnetic flux region. Would you spend even a microsecond of your time to check that the induced field is being compensated? Or would you rather go: "nah, there is no flux in my measurement loop, I can apply KVL, no problem!"

Hm, I'd probably still picture the electric field vortex, because for me that's easier to visualize. But no I'd not spend any time actually checking, I'd know that if the field axis is outside of my loop area, the electric fields must sum up to 0. To me the electric field is somehow more "intuitive" because that's what the charge in the loop is interacting with.

Which is another thing I think I understood now: charge particles that are inert referenced to the magnetic flux are not interacting with the magnetic field, even if it's changing. Charge particles in motion (e.g. when you move the wire) interact with the magnetic field and experience Lorentz' force.

Quote
EDIT: Yep, the transformer trick would do. IIRC Big Clive used the whole secondary of a transformer to step down the voltage. From 240-->12V to 240 ---> 240-12V or something like that. Caveat: the current must be sustained by both windings.

In principle we could use the radial probing to extract a finely varying voltage from a solenoidal coil immersed in a magnetic field. Instead of going one turn at the time, we can have a continuous (well, only marred by asperities in contact) voltage from start to finish. Mechanical nightmare, of course.

I feel the need to add something to the discussion of the actual matter, Dr. Lewins experiment.

I believe what many of us participating in the discussion initially didn't understand, me explicitly included, is that Dr. Lewins experiment must be seen as a whole, including his choice of probing the voltages. The whole idea about formulating a "law" in science is to be able to explain and predict the outcome of an experiment, and as I already said but probably nobody really took notice, the test instrument setup including geometrical arrangement of probe wires is inevitably part of the experiment. Dr. Lewin proposed a circuit and the challenge was to explain the measurements, and quite honestly, you cannot do that with Kirchhoff. It doesn't work.

What Mabilde and "fromjesse" did was change the experiment until they could explain it with KVL. That's like, when you get some task assigned in a test and you cannot solve it, you change the task until you can. I don't think that'll earn you any points.

Mehdi came so close to the truth with his "+ V/2 - V/2" construct, but he was so fixated on saving Kirchhoff that he forgot what the task was.

For me that concludes the discussion. I've learned a lot in the course of it and I have a new understanding of electromagentism through it. I'll go back to the sidelines now for a while.
Title: Re: #562 – Electroboom!
Post by: jesuscf on November 12, 2021, 08:54:55 am
What Mabilde and "fromjesse" did was change the experiment until they could explain it with KVL. That's like, when you get some task assigned in a test and you cannot solve it, you change the task until you can. I don't think that'll earn you any points.

Nope!  I am pretty sure that Mabilde, fromjesse, electroboom, and many other people, including myself, saw Lewin's experiment and immediately thought "bad probing"!  Why?  Because all of us have been there before, many, many times.  If you are not careful setting your measuring equipment you'll pick-up all kinds of unwanted stuff.  You quickly learn that oscilloscopes and their proves are very good at that!

EDIT: in fromjesse experiment there is a moment in which he notices some unwanted spikes in the signal he is measuring.  Then he immediately turns off a light and presto:  the spikes are gone!!!   Raise your had if that has happened to you!
Title: Re: #562 – Electroboom!
Post by: thinkfat on November 12, 2021, 09:49:21 am
What Mabilde and "fromjesse" did was change the experiment until they could explain it with KVL. That's like, when you get some task assigned in a test and you cannot solve it, you change the task until you can. I don't think that'll earn you any points.

Nope!  I am pretty sure that Mabilde, fromjesse, electroboom, and many other people, including myself, saw Lewin's experiment and immediately thought "bad proving"!  Why?  Because all of us have been there before, many, many times.  If you are not careful setting your measuring equipment you'll pick-up all kinds of unwanted stuff.  You quickly learn that oscilloscopes and their proves are very good at that!

If this is the mindset you're stuck with, that's not going to lead anywhere. It's not bad probing, the choice of probing was absolutely deliberate. The probes wires are part of the experiment. If you look at how Mabilde and also Mehdi are meticulously routing the probe wires so that they "hug" the inner loop because they think that's how they pick up "stray magnetic flux" or something, it's absolutely hilarious. They believe they have to, in order to reproduce Dr. Lewins "impossible" measurements. But in reality, you can route the probe wires anywhere you want, provided they don't enclose a non-conservative region. Also Mehdis careful twisting of the probe wires is superfluous, it doesn't change the outcome even slightly. It is not possible to understand the measurements if you don't look at the geometry of both the fields and the path described by the probe wires and where they attach.

For me the eyeopeners were the two videos linked by @rfeecs, the one by "Silicon Soup" that made me understand the rotational e-field, and the MIT courseware video where they basically showed Dr. Lewins experiment with a slightly different setup, using a toroidal core with one winding driven by a generator (like Mabilde) and just two resistors soldered together at their contact wires reaching around the core. Observe how carelessly they handle the probe wires going back to the oscilloscope, and how they explain the results on paper using C1, C2, CC as path designators.
Title: Re: #562 – Electroboom!
Post by: bsfeechannel on November 12, 2021, 02:08:46 pm
Or 'bad probing'.  :-DD

But seriously, you can also hook up a meter and measure voltage on both.


Remember when I told you that engineers take classes in vector calculus? It is so as not to incur in errors in their reasoning when analyzing some phenomenon. Your configuration is measuring only the voltage across the resistor, exactly what you'd measure if your meter and probes were in the plane. In fact any position of the meter and probes in red in the picture below will do that. This is because the area they define together with the resistor is not traversed by the lines of the varying magnetic field. Faraday's law. KVL won't tell you that.

(https://www.eevblog.com/forum/amphour/562-electroboom!/?action=dlattach;attach=1321469;image)
Title: Re: #562 – Electroboom!
Post by: bdunham7 on November 12, 2021, 02:41:33 pm
Forget nonconservative fields. Put your rod in a conservative field, an electrostatic field. It will 'charge up' by induction. You will have opposite charge at the extremes. Now measure your absolute voltage with your method. It should be zero.
Is it zero?

It should measure zero, simply because it needs to.  I have not come up with an elegant, direct proof, but I think I have one that is easy enough to understand.

(https://www.eevblog.com/forum/amphour/562-electroboom!/?action=dlattach;attach=1321478;image)

I realized the field lines may not be exactly as I've drawn, but whatever they are, if I provide a path from each charged end out of the field along a path that is always perpendicular to the field lines and then connect them to a voltmeter and resistor, I can see if a current flows in the resistor.  If it does, I can then extract energy from a static, conservative and irrotational E-field, which violates conservation of energy, among other laws.  IOW, it just isn't done! 

I can do the same with my brass balls and I would expect the same result.  It takes no work to move them on a path that only cuts the field lines at right angles, so if they get charged up in the process, I could do it repeatedly and again, extract energy from a system that we know we can't get energy from.  Another way of looking at it is that the charges will have no force causing them to go from one point to another because the fields are balanced, (Eind + Ecoul = 0) but I'm unable to prove that more directly, especially in the universal case as you demanded. 

Now in the case of the straight wire tangent to a rotational, non-conservative field, things change.  Specifically, we know experimentally that we can take conductors (or brass balls, but I haven't proven that experimentally) out of the field and then extract energy from the system as long as the rotational E-field continues.  We keep hearing that things are different in a non-conservative field, well that is one of them.  If you look at my drawing in my last reply to bsfeechannel, you'll see what I mean--the loop is not 'closed' until you are far enough away that the field is negligible, so the act of closing it only allows current to flow.  If you had the ideal voltmeter and thus did not close the loop, you would still see the voltage.

In conclusion I guess I'm saying that the rules are different for conductors in static, irrotational, conservative fields than they are in dynamic, rotational, non-conservative ones--which we all knew already, right?   :)   Exactly how that manifests itself is something that perhaps someone else can expound on, but the basic facts have been shown experimentally.  The rule that a conductor is an equipotential in a the static, irrotational, conservative field may not apply here and adopting the 'integral loop' rule seems to me to be a crutch for a failed model.  The rotational E-field exerts its force locally and tangentially and the rest of the story is just the process of adding it all up.  The 'closed loop' requirement in general is a red herring and demonstrably false--I could, in theory, use multiple torii and construct a linear accelerator by lining up the relatively straight inner sections of the rotational E-field for as long as I like.  Sorry, the drawing isn't very good....

(https://www.eevblog.com/forum/amphour/562-electroboom!/?action=dlattach;attach=1321484;image)

Edit:  "Linear accelerator" is a flawed statement because I had a conductor in mind, not a loose charged particle.  I'll try to refine what I'm trying to say when I get time.  But the point will be that I can develop an observable, usable and perhaps measurable phenomenon at the ends of the straight conductor without ever contemplating any loop at all, something that can't be done in a static, irrotational conservative field.
Title: Re: #562 – Electroboom!
Post by: bdunham7 on November 12, 2021, 02:52:57 pm
Remember when I told you that engineers take classes in vector calculus? It is so as not to incur in errors in their reasoning when analyzing some phenomenon. Your configuration is measuring only the voltage across the resistor, exactly what you'd measure if your meter and probes were in the plane. In fact any position of the meter and probes in red in the picture below will do that. This is because the area they define together with the resistor is not traversed by the lines of the varying magnetic field. Faraday's law. KVL won't tell you that.

I don't know why, at this point in the discussion, you think this is a revelation to me.  I deliberately drew that diagram to illustrate that the position of the voltmeter doesn't matter, as you've drawn.  I don't need Faraday's Law or KVL to tell that, all I need to know is that I'm out of the region of flux and thus there are no rotational E-fields to become tangential to my wires and push the charges around.  That's the whole point of extending out the partial turn.
Title: Re: #562 – Electroboom!
Post by: bsfeechannel on November 12, 2021, 03:08:47 pm
I believe they choose their loop so that it bisects the solenoid so that there is no net flux.  Or at least that is what is attempted and claimed.

Do the math. You'll see that there is no way to position the probes if their projection crosses the area of the varying magnetic field that can cancel its effects. The only way to do this was shown by Mehdi himself and it is to twist half of the loop so that the line integral of the electric field in one of the halves will cancel out the line integral of the other half.

In which case your meter will measure zero. Duh!

Quote
Quote
I was using YOUR definition of voltage, which really fails in a non conservative field, you're right.

Fails at what?  The two definitions are irreconcilable as far as I can see in this specific instance.  That's the issue.

Nope. YOUR definition of voltage is just a special case of the more general definition of voltage which is the line integral of the electric field along a certain path, be it the field conservative or non-conservative.

They're not irreconcilable.
Title: Re: #562 – Electroboom!
Post by: Sredni on November 12, 2021, 03:09:15 pm
Humor me, guys. See the attached picture. All wires are actual wires and interact with fields, but have negligible resistance. R1 can be any value, I don't think it matters. Assume EMF=1V. Predict the voltages shown on V1 and V2, using KVL and Faradays Law.
V1 is zero, V2 is 1V. (didn't check the sign just saw the thumbnail), might as well be -1V
It's easy if you check if your measurement loop (voltmeter + probes + branch you want to know the voltage of) contains or does not contain the flux.
I'd say vice versa, because the path through V2 is not in the magnetic flux and the branch voltage is zero, so I'd think V2=0 and V1=1V (or -1V, I didn't check the field orientation either). If I'm wrong, I'd really like to hear your explanation.

The explanation is that I need glasses. I only looked at the thumbnail and assumed you had numbered the voltmeters from left to right.  You are correct, and you have understood how to solve this kind of problems.
Here's a graphical depiction of the key, with an open circuit in lieu of the resistor

(https://i.postimg.cc/5yLyGRy0/Voltage-along-a-single-turn-coil.jpg)
Source: https://i.postimg.cc/5yLyGRy0/Voltage-along-a-single-turn-coil.jpg

Quote
Quote
Now I have a question for you.
Suppose I place a loop made with one resistor, the voltmeter, and its probes, near the magnetic flux region. Would you spend even a microsecond of your time to check that the induced field is being compensated? Or would you rather go: "nah, there is no flux in my measurement loop, I can apply KVL, no problem!"
Hm, I'd probably still picture the electric field vortex, because for me that's easier to visualize. But no I'd not spend any time actually checking, I'd know that if the field axis is outside of my loop area, the electric fields must sum up to 0. To me the electric field is somehow more "intuitive" because that's what the charge in the loop is interacting with.

Not sure how you will find an axis with more complex configurations of the fields. My suggestion is to stick with the flux: does it cut the surface bounded by your measurement loop? If yes, then kiss KVL goodbye.

Quote
I believe what many of us participating in the discussion initially didn't understand, me explicitly included, is that Dr. Lewins experiment must be seen as a whole, including his choice of probing the voltages. The whole idea about formulating a "law" in science is to be able to explain and predict the outcome of an experiment, and as I already said but probably nobody really took notice, the test instrument setup including geometrical arrangement of probe wires is inevitably part of the experiment.

well, yes and no. When there is no flux cut, the choice of the probes is irrelevant. But you realized that.
What one sees by looking at the fields is that, if we limit ourselves to the only two possible paths in the circuit joining the points A and B on the diameter (don't know what Lewin calls these points, maybe A and D) it is clear that the path integrals along these two different paths HAVE TO BE different.

(https://i.stack.imgur.com/YzxFb.jpg)
source: https://electronics.stackexchange.com/questions/551244/what-would-a-voltmeter-measure-if-you-had-an-electromotive-force-generated-by-a/551428#551428 (https://electronics.stackexchange.com/questions/551244/what-would-a-voltmeter-measure-if-you-had-an-electromotive-force-generated-by-a/551428#551428)

(I don't understand why the images from postimage.cc do not show up inline, like this one).

Of course you can put voltmeters on the sides, but only the voltmeter forming a measurement loop that does not cut the flux will measure the correct voltage. If voltmeter and resistor run aroung the mag zone, then the voltage shown will be altered by the intercepted EMF.

(https://i.postimg.cc/QtVW6mc9/Two-measuring-loops-one-consistent-result.jpg)
Source: https://i.postimg.cc/QtVW6mc9/Two-measuring-loops-one-consistent-result.jpg


Quote
For me that concludes the discussion. I've learned a lot in the course of it and I have a new understanding of electromagentism through it. I'll go back to the sidelines now for a while.

Welcome to the dark side.
Title: Re: #562 – Electroboom!
Post by: Sredni on November 12, 2021, 03:21:07 pm
Remember when I told you that engineers take classes in vector calculus? It is so as not to incur in errors in their reasoning when analyzing some phenomenon. Your configuration is measuring only the voltage across the resistor, exactly what you'd measure if your meter and probes were in the plane. In fact any position of the meter and probes in red in the picture below will do that. This is because the area they define together with the resistor is not traversed by the lines of the varying magnetic field. Faraday's law. KVL won't tell you that.

I don't know why, at this point in the discussion, you think this is a revelation to me.  I deliberately drew that diagram to illustrate that the position of the voltmeter doesn't matter, as you've drawn.  I don't need Faraday's Law or KVL to tell that, all I need to know is that I'm out of the region of flux and thus there are no rotational E-fields to become tangential to my wires and push the charges around.  That's the whole point of extending out the partial turn.

What do you think the voltmeter would read, in that picture?
Title: Re: #562 – Electroboom!
Post by: bsfeechannel on November 12, 2021, 03:27:10 pm
I don't know why, at this point in the discussion, you think this is a revelation to me.  I deliberately drew that diagram to illustrate that the position of the voltmeter doesn't matter, as you've drawn

But you said that you are measuring the voltage across both the wire and the resistor with that configuration. Or did get it wrong?

Quote
I don't need Faraday's Law or KVL to tell that, all I need to know is that I'm out of the region of flux and thus there are no rotational E-fields to become tangential to my wires and push the charges around. 

Unless you discovered that independently, it is generally attributed to Michael Faraday the discovery of the phenomenon you've just described above.
Title: Re: #562 – Electroboom!
Post by: bdunham7 on November 12, 2021, 03:40:40 pm
Nope. YOUR definition of voltage is just a special case of the more general definition of voltage which is the line integral of the electric field along a certain path, be it the field conservative or non-conservative.

They're not irreconcilable.

By irreconcilable I mean that they aren't going to be the same number in a non-conservative field.  You say "be it conservative or non-conservative", but I think perhaps the Magneto-Quasi-Static idea has fooled you into thinking that the concepts from conservative fields can be applied to non-conservative fields as long as they hold steady for some period of time.  I'm pretty sure that is untrue, but I'm struggling to demonstrate that.
Title: Re: #562 – Electroboom!
Post by: Sredni on November 12, 2021, 03:41:21 pm
Which is another thing I think I understood now: charge particles that are inert referenced to the magnetic flux are not interacting with the magnetic field, even if it's changing.

Whoa! My spider senses are tingling all over.
Care to rethink this?
Title: Re: #562 – Electroboom!
Post by: thinkfat on November 12, 2021, 04:07:05 pm
Which is another thing I think I understood now: charge particles that are inert referenced to the magnetic flux are not interacting with the magnetic field, even if it's changing.

Whoa! My spider senses are tingling all over.
Care to rethink this?

Yes, of course, but I'm pretty sure about that. It explains, IMHO, that Lorentz' force and the electromagnetic force are two independent effects and not just two sides of the same coin. It would also serve as an explanation for why, as @bsfeechannel said, a wire in time varying magnetic flux doesn't have an "inner electric field" while when it is moving in a (constant) magnetic flux, there is an "inner electric field". In the case of the time varying magnetic flux the charge particles interact not with the magnetic field but with the electric field that results from the magnetic flux change. That's how I understood the Maxwell-Faraday equation: change in magnetic flux causes a rotational electric field.
Title: Re: #562 – Electroboom!
Post by: bdunham7 on November 12, 2021, 04:26:22 pm
But you said that you are measuring the voltage across both the wire and the resistor with that configuration. Or did get it wrong?

I'm measuring from one point to another.  In this case, it a happens to be both across the resistor and across the partial turn, outside of any non-conservative field.

Title: Re: #562 – Electroboom!
Post by: bsfeechannel on November 12, 2021, 04:42:57 pm
By irreconcilable I mean that they aren't going to be the same number in a non-conservative field.

It's because YOUR definition of voltage fails. The line integral of the electric field is telling you the right answer. And how do we know it is right? Because you can find absolutely no contradiction between what it predicts and what you measure.

The line integral of the electric field says you'll find different voltages between two points under a non-conservative field. You connect your meters there and BANG! that's what they measure.

Heck, it even gives you the numerical value of what the meters are going to show with absolute accuracy.

YOUR definition of voltage doesn't predict any of that and does not allow me to calculate what the meters are going to measure requiring all sorts of tricks and cheats both speculative and practical to make the experiment conform to your assumptions and it leaves a lot of loose ends that, when tried to be dealt with, lead to contradictions.

The general definition of voltage depicts reality. YOUR definition of voltage depicts the struggle to understand it.

Quote
You say "be it conservative or non-conservative", but I think perhaps the Magneto-Quasi-Static idea has fooled you into thinking that the concepts from conservative fields can be applied to non-conservative fields as long as they hold steady for some period of time.  I'm pretty sure that is untrue, but I'm struggling to demonstrate that.

It is the other way around. The concepts from the non-conservative field can be applied to the conservative field, which is just a special case of the former when you do not have a varying magnetic field. The quasi static is there to guarantee that the wavelengths associated with the time-variations are much greater than the dimensions of our geometry, otherwise things get really hairy.
Title: Re: #562 – Electroboom!
Post by: jesuscf on November 12, 2021, 04:44:55 pm
I believe what many of us participating in the discussion initially didn't understand, me explicitly included, is that Dr. Lewins experiment must be seen as a whole, including his choice of probing the voltages. The whole idea about formulating a "law" in science is to be able to explain and predict the outcome of an experiment, and as I already said but probably nobody really took notice, the test instrument setup including geometrical arrangement of probe wires is inevitably part of the experiment. Dr. Lewin proposed a circuit and the challenge was to explain the measurements, and quite honestly, you cannot do that with Kirchhoff. It doesn't work.

I beg to disagree.  A proper experiment would had included many measurements from different geometric probing configurations.  Lewin  purposefully picked a configuration that he believed would eliminate the effect of the varying magnetic field in the measurement equipment while measuring the voltage between the top and the bottom of the ring.   He ended up cancelling the effect of the magnetic field both in the probes and in the ring!  That is equivalent to measure the voltage across the resistors directly.

Lewin then assumes that the voltage across any two points in the ring wire is zero volts, because he applies ohms law and the wire resistance is almost zero, and he goes AHA! KVL doesn't work!!!  The problem here is that Lewin forgot that a piece of wire in a circuit under the influence a varying magnetic field doesn't behave as zero ohm resistor but as non-ideal voltage source.  When you account for that extra little piece of information, all of a sudden KVL works perfectly, no matter the probing geometry, if you include the probes as part of your circuit.
Title: Re: #562 – Electroboom!
Post by: bdunham7 on November 12, 2021, 05:34:12 pm
The general definition of voltage depicts reality. YOUR definition of voltage depicts the struggle to understand it.

It's not my definition of voltage, it is a definition that was taught earlier in Lewin's course, among a zillion other places that it pops up.

If you posit that a conductor is an equipotential even in a quasi-static but rotational, non-conservative field then you are saying that my example with the torus, winding and straight rod results in an equipotential over the entire surface of that conductive rod.  Is that an accurate assessment of your position?
Title: Re: #562 – Electroboom!
Post by: bsfeechannel on November 12, 2021, 06:52:21 pm
But you said that you are measuring the voltage across both the wire and the resistor with that configuration. Or did I get it wrong?

I'm measuring from one point to another.  In this case, it a happens to be both across the resistor and across the partial turn, outside of any non-conservative field.

You are just measuring a voltage due to a conservative field that is present in the resistor and between the terminals of the wire. Assuming the wire has no resistance, you can slide the meter all the way down to the left, just before you cross the lines of the mag field. You're measuring nothing else. You're not measuring the voltage due to the field in the wire. For that you need to place your meter anywhwere three-dimensionally speaking to the left of the field.
Title: Re: #562 – Electroboom!
Post by: Sredni on November 12, 2021, 06:53:56 pm
Which is another thing I think I understood now: charge particles that are inert referenced to the magnetic flux are not interacting with the magnetic field, even if it's changing.

Whoa! My spider senses are tingling all over.
Care to rethink this?
Yes, of course, but I'm pretty sure about that. It explains, IMHO, that Lorentz' force and the electromagnetic force are two independent effects and not just two sides of the same coin.

Well,

F = q ( E + v x B)

if B = 0 or v = 0 (or v//B) we kiss goodbye to Lorentz's force. We are left with E.
And a variable magnetic field in the core will be associated with a circulating Eind field around the core. So, a charge nearby will feel the force and will accelerate in the tangential direction. The changing magnetic field makes the particle move. I can call that 'an interaction'.
They built accelerators based on this principles: the betatrons.
See, for example, http://web.mit.edu/course/22/22.09/ClassHandouts/Charged%20Particle%20Accel/CHAP11.PDF (http://web.mit.edu/course/22/22.09/ClassHandouts/Charged%20Particle%20Accel/CHAP11.PDF)

Quote
"A toroidal vacuum chamber encircles the core of a large magnet. The magnetic field is produced by pulsed coils; the magnetic flux inside the radius of the vacuum chamber changes with time. Increasing flux generates an azimuthal electric field which accelerates electrons in the chamber.
In the absence of an air gap, there is little magnetic flux outside the core."

I think you should reconsider.
(Also magnetic and electric effects are the two faces of the same coin. A magnetic field is but an electric field in another frame of reference. But I'm not gonna touch that in here. But if you are interested there is a new book with a nonconventional approach

Kjell Prytz
Electrodynamics: The Field-Free Approach:
Electrostatics, Magnetism, Induction, Relativity and Field Theory
Undergraduate Lecture Notes in Physics
2015, Spinger

He does not even use fields. Just forces between charges. It needs a bit of editing in my opinion - looks like a work in progress, with some of the theory discussed in the exercises and apparent gaps in the text but it makes a very interesting read. I am pretty sure bsfeechannel and rsfeec will find it interesting)
Title: Re: #562 – Electroboom!
Post by: bsfeechannel on November 12, 2021, 06:54:47 pm

It's not my definition of voltage, it is a definition that was taught earlier in Lewin's course, among a zillion other places that it pops up.

Care to provide a reference to it in Lewin's course, please?
Title: Re: #562 – Electroboom!
Post by: bdunham7 on November 12, 2021, 07:06:19 pm
You are just measuring a voltage due to a conservative field that is present in the resistor and between the terminals of the wire. Assuming the wire has no resistance, you can slide the meter all the way down to the left, just before you cross the lines of the mag field. You're measuring nothing else. You're not measuring the voltage due to the field in the wire. For that you need to place your meter anywhwere three-dimensionally speaking to the left of the field.

How do you define the term 'equipotential'?

Let's assume I'm measuring between any two points with an analog voltmeter, just to make things a bit clearer.  This would be a galvanometer in series with a resistor.  So if I connect such a voltmeter to two points, current will not flow if they are equipotential, but will if there is a potential there.  You can hide the entire apparatus from me, but as long as my meter and test leads are free from any external fields, B or E, I will get a certain result that will indicate whether those two points are equipotential or not.  Free of external fields, there is no dispute over what voltage is.  I don't need to have any knowledge of the source or cause of the voltage, or lack thereof.  Or do you disagree with that as well?
Title: Re: #562 – Electroboom!
Post by: bdunham7 on November 12, 2021, 07:08:25 pm
Care to provide a reference to it in Lewin's course, please?

Maybe I can look a bit later, but which part do you doubt?   C = 4 * pi * E0 * r  or Q = CV?
Title: Re: #562 – Electroboom!
Post by: bsfeechannel on November 12, 2021, 07:29:35 pm
I'll tell you what my crystal ball forecasts: that your method to measure voltage across the rod will force you to move the sensing instrument on a path that, together with the rod, will form a closed path around the variable magnetic region.

No, your insistence on a closed path is your downfall.  You only insist on it because it makes your math work.

The insistence in the closed path is because we are talking about circuits, which by definition are closed paths.
Title: Re: #562 – Electroboom!
Post by: bdunham7 on November 12, 2021, 07:46:12 pm
The insistence in the closed path is because we are talking about circuits, which by definition are closed paths.

Your arguments are growing feeble.  I'm disappointed!

Obviously a debate over KVL necessarily refers to a closed path, but what we seem to be debating is related to specific parts of that path and I see no problem in breaking them down.

Title: Re: #562 – Electroboom!
Post by: bsfeechannel on November 12, 2021, 10:39:55 pm
How do you define the term 'equipotential'?

Let's assume I'm measuring between any two points with an analog voltmeter, just to make things a bit clearer.  This would be a galvanometer in series with a resistor.  So if I connect such a voltmeter to two points, current will not flow if they are equipotential, but will if there is a potential there.  You can hide the entire apparatus from me, but as long as my meter and test leads are free from any external fields, B or E, I will get a certain result that will indicate whether those two points are equipotential or not.  Free of external fields, there is no dispute over what voltage is.  I don't need to have any knowledge of the source or cause of the voltage, or lack thereof.  Or do you disagree with that as well?

Suppose I agree. Please, go on.

Care to provide a reference to it in Lewin's course, please?

Maybe I can look a bit later, but which part do you doubt?   C = 4 * pi * E0 * r  or Q = CV?

Where, when, what did Lewin exactly say? You need to support your claims with evidence, otherwise you'll have a feeble argument. As for your electrostatic equations, please enlighten me as to what this has to do with a rotational electric field along the path of a circuit. I read your posts, obviously, but nothing in them gave me a clue about the fact that you'll not have undefined voltages between two points under a varying magnetic field.
Title: Re: #562 – Electroboom!
Post by: Sredni on November 13, 2021, 12:21:26 am
It should measure zero, simply because it needs to
Emphasis mine.

Ok, I have re-read the description of your concept of 'absolute voltage', the description of 'the machine' and how it should work in a nonconservative and in a conservative field. I... I...

I need a shot of tequila.

<pause>

I don't know where to begin. Let me say that your instrument appears to run on a lot of wishful thinking. You use the formula for self-capacitance to infer voltage? But is your sphere alone in the vacuum of space? I don't think so. First off, your machine requires two spheres, so you will have to account for the field lines going from one sphere to the other. Then you have the object you want to measure the charge of that will interact with those field lines, and the source of the field, and the whole planet beneath your feet. You will have a full fledged matrix of coefficients of capacitance to handle - and that alone would kill the independence of your instrument.

And that thing about the field lines being perpendicular to the path of the spheres? Wishful thinking at its best. The moment the field induces charges on your rod, they will distort the field - not only inside the body to create the zero field, but also outside. And the distortion will be significant because the field lines needs to be perpendicular to the surface of the conductor.
And then your spheres will have induced charge themselves, that will disturb the field even more and change the charge distribution of the rod as you move near and away from it.
And why should the charge displaced on the rod jump on the spheres (which will have their own induced charge already?)

And what do you want to do in the nonconservative case? "extend the ends out to the point where the fields are negligible"??? What fields? Not the B field, it was never outside the core to begin with. So it must be the electric field. It can't be Ecoul, since it is generated by the charges themselves. Must be the induced field Eind, then. And you expect to see the displaced charge where there is no Eind field? If there is no more Eind field, what keeps the charges segregated at the extremes of the rod? All that charges of the same sign crammed together at one extreme? Wouldn't you think they'd repel each other?

And then you bring the alleged charge transferred on the spheres back together in the same place where you can measure it? My crystal ball was right. You are closing the path, after all (but let's not talk about this now). Even admitting you can magically duplicate (this could actually work) or take the charge of one extreme of the rod and the charge on the other extreme and then infer the voltage between them - you would see a voltage between them even in the electrostatic case. (because you have removed the external field Eext that causes the electrostatic induction of the rod). So, should we also say that there is a voltage between two points of a conductor in an electrostatic field?

Tequila is my lady, tonight.
Title: Re: #562 – Electroboom!
Post by: bdunham7 on November 13, 2021, 12:23:49 am
Yes, of course, but I'm pretty sure about that. It explains, IMHO, that Lorentz' force and the electromagnetic force are two independent effects and not just two sides of the same coin. It would also serve as an explanation for why, as @bsfeechannel said, a wire in time varying magnetic flux doesn't have an "inner electric field" while when it is moving in a (constant) magnetic flux, there is an "inner electric field". In the case of the time varying magnetic flux the charge particles interact not with the magnetic field but with the electric field that results from the magnetic flux change. That's how I understood the Maxwell-Faraday equation: change in magnetic flux causes a rotational electric field.

I can't prove it at this time,  but I think that doesn't work.  It posits two different types of EMF resulting from non-conservative forces (energy can be extracted) one with a so-called inner E-field in a conductor and presumably a potential gradient and the other with no inner E-field and an equipotential conductor.  Both EMF forces push on charges, what is different about them that would allow that result?  Remember, in both cases the charges are being pushed out and around to potentially (!) do work, whereas in the static, irrotational conservative field, that can't happen.
Title: Re: #562 – Electroboom!
Post by: bdunham7 on November 13, 2021, 01:48:52 am
First off, your machine requires two spheres, so you will have to account for the field lines going from one sphere to the other. Then you have the object you want to measure the charge of that will interact with those field lines, and the source of the field, and the whole planet beneath your feet.

Actually, it only requires one sphere.  The 'machine' will measure Q precisely by adding or subtracting electrons until the sphere is exactly neutral.  The two-sphere model allows simultaneous measurements, so I can equate it to an ideal voltmeter in the right conditions.  As for the difficulties, experimental physics isn't simple!  Look at what Millikan did 100+ years ago.

Quote
And that thing about the field lines being perpendicular to the path of the spheres? Wishful thinking at its best. The moment the field induces charges on your rod, they will distort the field - not only inside the body to create the zero field, but also outside. And the distortion will be significant because the field lines needs to be perpendicular to the surface of the conductor.

Yes, idea requires me to move the spheres in contact and then back out to a field-free zone by only crossing field lines at right angles, or IOW, along an equipotential.  Difficult, but I don't think theoretically impossible.  Or if it is technically theoretically impossible due to the perpendicular field issue, at least possible to get infinitesmally close.

Quote
And then your spheres will have induced charge themselves, that will disturb the field even more and change the charge distribution of the rod as you move near and away from it.
And why should the charge displaced on the rod jump on the spheres (which will have their own induced charge already?)

Well there's the thing--I think the charges won't jump on the sphere because they're happy where they are.   Just like a the voltmeter that I drew above the static field won't show anything.  But I couldn't see an easy way to prove that.  And as you pointed out, I'd need to prove that in the general case.

Quote
And what do you want to do in the nonconservative case? "extend the ends out to the point where the fields are negligible"??? What fields? Not the B field, it was never outside the core to begin with. So it must be the electric field. It can't be Ecoul, since it is generated by the charges themselves. Must be the induced field Eind, then. And you expect to see the displaced charge where there is no Eind field? If there is no more Eind field, what keeps the charges segregated at the extremes of the rod? All that charges of the same sign crammed together at one extreme? Wouldn't you think they'd repel each other?

Yes, they will repel each other until they are evenly distributed in the no-field zone.  And yes, I mean Eind, the rotational E-field around the flux.  If I have, for example, a U-shaped rod that goes partly around the flux and then extends out a ways, I would expect to see a gradient in the area subject to the rotational E-field and then two long ends with evenly distributed but not neutral charge.  Thus when I test the ends of that rod, I will see a potential between them regardless of what method I use because there's no external field to worry about.

Quote
And then you bring the alleged charge transferred on the spheres back together in the same place where you can measure it? My crystal ball was right. You are closing the path, after all (but let's not talk about this now). Even admitting you can magically duplicate (this could actually work) or take the charge of one extreme of the rod and the charge on the other extreme and then infer the voltage between them - you would see a voltage between them even in the electrostatic case. (because you have removed the external field Eext that causes the electrostatic induction of the rod). So, should we also say that there is a voltage between two points of a conductor in an electrostatic field?

See, I can predict your thinking as well--the closed path.  That's why I went to the extra effort of measuring each balls absolute charge independently of the other.  I can actually not close the path.  But IMO it doesn't matter, closing the path far away from the action has negligible effect and you shouldn't rely on technicalities, infinitesmals or magic to make a practical theory work. 

Quote
Tequila is my lady, tonight.

Scotch for me!
Title: Re: #562 – Electroboom!
Post by: bsfeechannel on November 13, 2021, 03:23:00 am
As for the difficulties, experimental physics isn't simple!
Quote
Difficult, but I don't think theoretically impossible.
Quote
But I couldn't see an easy way to prove that.
Quote
But IMO it doesn't matter, closing the path far away from the action has negligible effect and you shouldn't rely on technicalities, infinitesmals or magic to make a practical theory work. 

So let me get this straight. You can't prove none of your claims and you want to reinvent the wheel with a "practical" theory that doesn't rely on "technicalities, infinitesimals or magic" that again you can't even demonstrate in practice because "experimental physics isn't simple".

You said you're not a KVLer, but you talk and think like one. Don't get me wrong, but your posts look like a bunch of delusional speculations.

On the other hand, Faraday's law of induction, which renders KVL useless for varying magnetic fields, is not only easily provable, but also easily demonstrated by anyone with a spool of wire, a battery and a meter. That's how Faraday himself discovered the phenomenon. You don't need a fancy physics lab.

As for the practical side of it, Maxwell's equation for Faraday's law is what is used everyday in the world to calculate the number of turns of the primary of any transformer. For a linear transformer, for example the peak voltage per turn is calculated as follows:

Vpeak = 2πf*Bmax*A

You can easily recognize it. Vpeak is the line integral of E around the turn. 2πf is the result of the time derivative of cos(2πf), Bmax is the peak magnetic field just before saturation and A is the transversal area of the core. Define the peak voltage on the primary and divide it by Vpeak and you have the number of turns. That simple. No need for brass balls.

There are billions of them in the world today. There's nothing more practical and consistent than this.
Title: Re: #562 – Electroboom!
Post by: bdunham7 on November 13, 2021, 04:29:37 am
So let me get this straight. You can't prove none of your claims and you want to reinvent the wheel with a "practical" theory that doesn't rely on "technicalities, infinitesimals or magic" that again you can't even demonstrate in practice because "experimental physics isn't simple".

Sir, this is EEVBlog.

Quote
On the other hand, Faraday's law of induction, which renders KVL useless for varying magnetic fields, is not only easily provable, but also easily demonstrated by anyone with a spool of wire, a battery and a meter. That's how Faraday himself discovered the phenomenon. You don't need a fancy physics lab.

I can't imagine how you could possibly read a denial of Faraday's Law in anything I've written.  If some of it seems too far out there, address a  few simpler ideas.

Start with the long U-shaped conductor.  Is there potential across the ends or not?  If so, how goes your claim of equipotentiality in non-conservative fields?  If not, why not? 

As I was responding to Sredni and thinkfat it occured to me that there's another issue with the equipotentiality argument.  In the static, conservative, irrotational field, there isn't an E-field inside the conductor because the charges instantly rearrange themselves to oppose it.  Thus there is no net force on any charge within the conductor.  In the case of a charge moving in a magnetic field, you accept that there is an non-conservative field inside the conductor acting on the charges, right?  But then we get to the MQS system, and even though there is clearly a local force acting on charges inside the conductor--which is the EMF--and those forces continually push charges through the conductor, that the conductor is nonetheless equipotential in the same way as in the static case, and for the same reason--there's no electric field in a conductor.  But what I'm showing you with my U-shaped conductor example is that while the charges will rearrange themselves to counter any net electric field in the conductor, the result may not be an equipotential surface.  Instead, the charges are pumped one way or another, with the result that one end has a different potential than the other. 

Here is a non-closed path example without any as-of-yet uninvented machines, except I'm not sure they have electroscopes this good.  What happens when the current is turned on?

(https://www.eevblog.com/forum/amphour/562-electroboom!/?action=dlattach;attach=1321814;image)
Title: Re: #562 – Electroboom!
Post by: Sredni on November 13, 2021, 06:22:01 am
Start with the long U-shaped conductor.  Is there potential across the ends or not?

You are asking the wrong question. It's probably my fault because in a previous post I used the term 'equipotential' in quotes to mean 'the voltage difference between two points is zero' (inside a certain cylinder), so let me explain again.
You can no longer talk about potential function for the total electric field. It has no longer sense because the path integral of the total electric field depends on the path joining two points.
Potential no mas.

So your question has no sense. You should ask, instead: is there a voltage across the ends?
And the answer is:

IT DEPENDS ON THE PATH

What path? The path along which you want voltage, which is a function of the endpoints AND THE PATH, to be evaluated.
For all paths that together with the conductor (let me rephrase: together with any path inside the conductor that joins point A and B - it does not matter which because the electric field inside is exactly zero if there is a gap)--- let me repeat: for all paths that together with the conductor form a closed path that DOES NOT go around the dB/dt region, the voltage difference is ZERO.
For all paths that, together with the conductor, DO GO around the dB/dt region, the voltage difference is 1 EMF (or -1 depending on how we go around, or a multiple of the EMF if we go around several times).

Tomorrow I will add a couple of pictures that hopefully will clarify what I wrote.

Quote
As I was responding to Sredni and thinkfat it occured to me that there's another issue with the equipotentiality argument.  In the static, conservative, irrotational field, there isn't an E-field inside the conductor because the charges instantly rearrange themselves to oppose it.  Thus there is no net force on any charge within the conductor.  In the case of a charge moving in a magnetic field, you accept that there is an non-conservative field inside the conductor acting on the charges, right?  But then we get to the MQS system, and even though there is clearly a local force acting on charges inside the conductor--which is the EMF--

Stop. This is wrong.
It is NOT the EMF. We always end up here: forgetting the role of the displaced charge. The EMF -  or better, the induced electric field Eind, is compensated in the conductor by the coloumbian field Ecoul. If there is a gap and no current is flowing, this is always exactly true.
If there is a current flowing (because we closed the circuit) then it is zero only if the conductor is perfect (zero resistivity, infinite conductivity). Charges move without a field. In practice, with real conductors, the compensation is not perfect and there is a residual field E in the conductor, directed along the conductor and whose value complies with the local form of Ohm's law: E = j / sigma_copper.

How does it follow the conductor? Answer: surface charge. (Read here, if you are interested https://electronics.stackexchange.com/questions/532541/is-the-electric-field-in-a-wire-constant/532550#532550 )

So, it is not Eind that is acting on the carriers. It is the combined effect of the EMF (represented by Eind) and the displaced charge (represented by Ecoul).

Quote
and those forces continually push charges through the conductor, that the conductor is nonetheless equipotential in the same way as in the static case, and for the same reason--there's no electric field in a conductor.

and this is again (EDIT: in general) wrong for the reason above. But, if you consider a region of space that contains all of your piece of conductor and none of the magnetic flux, like a cylinder around the rod in the middle of the torus, in that region of space all paths you can imagine will never enclose the magnetic flux region. in this sense you can consider it as equipotential. Shrink the surface to include only the conductor if you want. But the moment you consider a path the forms, with the conductor, a closed path that cuts the flux - NO MORE.

(I have a picture to visualize this as well - but I drank too much tequila to operate a scanner)
Title: Re: #562 – Electroboom!
Post by: thinkfat on November 13, 2021, 07:41:32 am
I believe what many of us participating in the discussion initially didn't understand, me explicitly included, is that Dr. Lewins experiment must be seen as a whole, including his choice of probing the voltages. The whole idea about formulating a "law" in science is to be able to explain and predict the outcome of an experiment, and as I already said but probably nobody really took notice, the test instrument setup including geometrical arrangement of probe wires is inevitably part of the experiment. Dr. Lewin proposed a circuit and the challenge was to explain the measurements, and quite honestly, you cannot do that with Kirchhoff. It doesn't work.

I beg to disagree.  A proper experiment would had included many measurements from different geometric probing configurations. Lewin purposefully picked a configuration that he believed would eliminate the effect of the varying magnetic field in the measurement equipment while measuring the voltage between the top and the bottom of the ring.   He ended up cancelling the effect of the magnetic field both in the probes and in the ring! That is equivalent to measure the voltage across the resistors directly.

Lewin then assumes that the voltage across any two points in the ring wire is zero volts, because he applies ohms law and the wire resistance is almost zero, and he goes AHA! KVL doesn't work!!!  The problem here is that Lewin forgot that a piece of wire in a circuit under the influence a varying magnetic field doesn't behave as zero ohm resistor but as non-ideal voltage source.  When you account for that extra little piece of information, all of a sudden KVL works perfectly, no matter the probing geometry, if you include the probes as part of your circuit.

I suggest watching that other MIT video, https://youtu.be/u6ud7JD0fV4. The thing is, the measurement loops C1 and C2 are not influenced by any magnetic flux. The demonstration even uses a toroid core to make sure that there is no flux outside of the core. There's nothing you'd need to cancel or compensate for. And the setup is not even very peculiar, the probe wires are just laying there on the bench with no particular care taken to fixate them anywhere. They're just "flopping around in the breeze".

But anyway, would you be able to explain what the oscilloscope shows in that video with just KVL?

I actually like that setup, because the core is a solid loop, there is no point in arguing "stray magnetic fields" and no way to contort the setup to "eliminate bad probing".
Title: Re: #562 – Electroboom!
Post by: thinkfat on November 13, 2021, 09:02:15 am
Quote
"A toroidal vacuum chamber encircles the core of a large magnet. The magnetic field is produced by pulsed coils; the magnetic flux inside the radius of the vacuum chamber changes with time. Increasing flux generates an azimuthal electric field which accelerates electrons in the chamber.
In the absence of an air gap, there is little magnetic flux outside the core."

Emphasis mine, this is what I was referring to. It's the electric field the electrons interact with.
Title: Re: #562 – Electroboom!
Post by: jesuscf on November 13, 2021, 09:48:04 am
I suggest watching that other MIT video, https://youtu.be/u6ud7JD0fV4. (https://youtu.be/u6ud7JD0fV4.) The thing is, the measurement loops C1 and C2 are not influenced by any magnetic flux. The demonstration even uses a toroid core to make sure that there is no flux outside of the core. There's nothing you'd need to cancel or compensate for. And the setup is not even very peculiar, the probe wires are just laying there on the bench with no particular care taken to fixate them anywhere. They're just "flopping around in the breeze".

But anyway, would you be able to explain what the oscilloscope shows in that video with just KVL?

I actually like that setup, because the core is a solid loop, there is no point in arguing "stray magnetic fields" and no way to contort the setup to "eliminate bad probing".

It is the same experiment and they are making exactly the same mistake Lewin made!!!  What induces the voltage in the wires is the rate of change of magnetic flux through the loops, even if the magnetic flux is confined in the core.   Adding a core to confine the flux inside the loops increases the efficiency of the transformer as no flux is wasted outside, but it doesn't change  the experiment.  They are still assuming that the wires in the secondary behave as short circuits with zero volts across them, while under the influence of external varying magnetic field.  That assumption is not correct.  Under the influence of an external magnetic field those secondary wires behave as a non-ideal voltage sources (or put in other words, the wire has both an 'inductance' and 'resistance').  The resistors in the loops also behaving as non-ideal voltage sources (they also have 'inductance' and 'resistance').

Check this video from 'fromjesse' with the very same experiment where he explains much better than me what is going on.  The title of the video says it all: "The Lewin loop inside an iron core - KVL still holds":

https://www.youtube.com/watch?v=iDWv8QJrzUo (https://www.youtube.com/watch?v=iDWv8QJrzUo)

Also, I am attaching a couple of figures.  The first figure shows the circuit Lewin think he was dealing with.  The second figure shows the actual circuit Lewin should had used, based on his original experiment.  When you prove correctly with the oscilloscope you get rid of the induced voltages in series with the proves in the second figure, but the induced voltages in the wires connecting the resistors remain.
Title: Re: #562 – Electroboom!
Post by: thinkfat on November 13, 2021, 02:43:37 pm
Check this video from 'fromjesse' with the very same experiment where he explains much better than me what is going on.  The title of the video says it all: "The Lewin loop inside an iron core - KVL still holds":
https://www.youtube.com/watch?v=iDWv8QJrzUo (https://www.youtube.com/watch?v=iDWv8QJrzUo)

I'm afraid this won't make your argument stronger. You really need to take geometry and the fields into account and where the magnetic flux "flows".

The core is split, it's two toroids which both carry about half of the total flux each. His "Lewin loop" is around the center column where the two toroids join, same as the yellow reference loop, they both experience the total magnetic flux. However, when he measures the voltage across the "red" parts, his probe leads and the red wire form a loop around one of the two toroids, encompassing half of the magnetic flux. So what he actually measures is not the voltage across that red wire, it is the EMF in his measurement loop, which is, since it experiences about half of the total flux, half of the voltage induced in his reference loop.

PS: this would be a brilliant experiment: Use a split core with different permeability in each of the toroids, so that the flux is no longer evenly split. I wonder if the voltages would still add up ;)
Title: Re: #562 – Electroboom!
Post by: bsfeechannel on November 13, 2021, 05:03:04 pm
Sir, this is EEVBlog.

I didn't mean you cannot discuss such things here. What I meant is to call your attention to the fact that you need to provide evidence and proof to your claims.

Quote
I can't imagine how you could possibly read a denial of Faraday's Law in anything I've written.  If some of it seems too far out there, address a  few simpler ideas.

I'm not saying you're denying Faraday's law. Perhaps you don't understand the full implications of it yet. But that's not your fault.

Quote
Here is a non-closed path example without any as-of-yet uninvented machines, except I'm not sure they have electroscopes this good.  What happens when the current is turned on?

(https://www.eevblog.com/forum/amphour/562-electroboom!/?action=dlattach;attach=1321814;image)

OK. Let's suppose that you produce a sawtooth magnetic field pulse in the torus, which your drawing seems to indicate. We will have a square pulse voltage between the terminals of the wire. While the square pulse exists, let's suppose your electroscopes indicate the displacement of charges at the terminals of the wire.

Where do we go from here?
Title: Re: #562 – Electroboom!
Post by: Sredni on November 13, 2021, 05:11:30 pm
Quote
"A toroidal vacuum chamber encircles the core of a large magnet. The magnetic field is produced by pulsed coils; the magnetic flux inside the radius of the vacuum chamber changes with time. Increasing flux generates an azimuthal electric field which accelerates electrons in the chamber.
In the absence of an air gap, there is little magnetic flux outside the core."

Emphasis mine, this is what I was referring to. It's the electric field the electrons interact with.

But it's not independent from the changing magnetic field. The way I see it is they are two manifestations of the same phenomenon. (Like in the propagation of an EM wave, it's not that the changing B field causes a changing E field that causes... ). But anyway, we can consider this just nitpicking.
It appears to me that you now understand Faraday's law in full.

I cannot help imagining Lewin stroking his hands with satisfaction while, in a black hood and cloak, reads your exchange with jesuscf and mutters "Well done, my apprentice... Hihihihi.... Now, strike him down with your path integral!"

May the nonconservative force be with you.

Title: Re: #562 – Electroboom!
Post by: bdunham7 on November 13, 2021, 05:35:24 pm
IT DEPENDS ON THE PATH

Of course it does.  And if I were measuring an AAA cell and I looped one test lead through a core with dB/dt, I might not get 1.5V.  But the problem I posed specified two points out of any external fields.  The voltage across those should be unambiguous if your path is also out of those fields.  This was all in response to bsfeechannel's assertions regarding partial turns and the electroscope example was to show that there is an observable potential even without a closed path. 

Quote
Stop. This is wrong.
It is NOT the EMF. We always end up here: forgetting the role of the displaced charge. The EMF -  or better, the induced electric field Eind, is compensated in the conductor by the coloumbian field Ecoul. If there is a gap and no current is flowing, this is always exactly true.

Well, reading the first line of Wikipedia on 'EMF' I'm not sure they agree, but since you've been using Eind and Ecoul and those are very nice, clear terms, I'll use them instead and I'll avoid the term 'EMF'. 

Quote
So, it is not Eind that is acting on the carriers. It is the combined effect of the EMF (represented by Eind) and the displaced charge (represented by Ecoul).

To put this in context, I was comparing three phenomena- a conductor in a static, irrotational conservative field, a conductor in a rotational non-conservative field derived from dB/dt and a conductor moving across magnetic lines as in a generator, where the charges experience the Lorentz force.  So in the first case, I think we all agree with your assertion.  Apparently, and don't attribute this assertion to me, the third case does result in a potential gradient over the length of the conductor.  Is that right or wrong?  And if it is right, then how do you differentiate the effect of the Lorentz force from the local Eind force in the second example?
[/quote]

As for the rest of what you responded to, you may have misunderstood--I was asking bsfeechannel if what I wrote was his understanding, and since you appear to disagree with the statement, if he disavows it as well then we're all on the same page.

We may be straying from the Lewin experiment a bit in these discussions but I think we need to in order to understand where the dispute lies--which, if you remember, was my original intent.  So if you have an open ended conductor and over some portion of that conductor you apply an electric field along it(Eind, the charges instantly rearrange to oppose and cancel the field.  How is that different from inserting a battery in the conductor?  Won't the charges also rearrange to oppose the battery voltage?  Is the conductor equipotential?  And before you accuse me of being a KVLer or a battery moron, I'm not claiming battery equivalence and I do have a further point that I'm working on.
Title: Re: #562 – Electroboom!
Post by: rfclown on November 13, 2021, 05:53:43 pm
...
The circuit had just two resistors and nothing else. And his model predicted exactly what happened in practice. KVL failed on the board. And then failed on the bench.
...

Wow. What a strange thread. It isn't "just two resistors and nothing else", it has a closed loop of wire in a varying magnetic field. It has a transformer (as has been pointed out over and over here). If the model doesn't include the transformer, it is an incomplete model.
Title: Re: #562 – Electroboom!
Post by: bsfeechannel on November 13, 2021, 05:56:21 pm

The core is split, it's two toroids which both carry about half of the total flux each. His "Lewin loop" is around the center column where the two toroids join, same as the yellow reference loop, they both experience the total magnetic flux. However, when he measures the voltage across the "red" parts, his probe leads and the red wire form a loop around one of the two toroids, encompassing half of the magnetic flux. So what he actually measures is not the voltage across that red wire, it is the EMF in his measurement loop, which is, since it experiences about half of the total flux, half of the voltage induced in his reference loop.

Exactly. What he had was NOT Lewin's setup. The meters that were measuring the voltages across the wires were forming additional loops around half of the magnetic field in the opposite direction. No wonder he got negative voltages.

(https://www.eevblog.com/forum/amphour/562-electroboom!/?action=dlattach;attach=1322216;image])

Quote
PS: this would be a brilliant experiment: Use a split core with different permeability in each of the toroids, so that the flux is no longer evenly split. I wonder if the voltages would still add up ;)

What is funny is that he was told by user Copernico Felinis to use a torus, but the poor bugger refused saying that the leads of the meters would develop a voltage that would mask the real voltages he thinks are in the wires.

This unfortunate experimenter doesn't know that if he measures the voltages the same way he did for the EI core transformer, they will actually add up to zero. Only that the bottom meter will exhibit zero volts, while the top meter will show 200mV, proving that the voltages are not in the wire.

(https://www.eevblog.com/forum/amphour/562-electroboom!/?action=dlattach;attach=1322222;image)
Title: Re: #562 – Electroboom!
Post by: rfeecs on November 13, 2021, 06:00:44 pm
a piece of wire in a circuit under the influence a varying magnetic field doesn't behave as zero ohm resistor but as non-ideal voltage source.

This is the crux of the argument.

Clearly the wire was breaking the law (KVL?) by operating under the influence.  :-DD

Is there voltage "in the wires"?  There is no electric field in the wires or tangential to the surface of the wires, so physics says the answer is no.

But there is a charge separation between the ends of the wires so something is going on.

One can make a lumped model by representing this charge separation by a battery in series with the wire.  Now with a lumped model, KVL works fine.

But if we try to measure the voltage across the wire, we get zero volts.  However we can get a voltage if we position the wires differently.  If we do it just right, then we can get the numbers that make KVL work again.

I'm sure people who make a lumped model know that there is no battery in the wire.  They are just making a model.  It isn't reality.

Forget the measuring equipment and just look at the loop by itself.  Calculate the voltage along the path of the circuit.  The voltage across the wires are zero.

Choose a different path and you may get a different answer.  Because voltage depends on path in this case.
Title: Re: #562 – Electroboom!
Post by: bdunham7 on November 13, 2021, 06:01:37 pm
Where do we go from here?

Why here, of course!

(https://www.eevblog.com/forum/amphour/562-electroboom!/?action=dlattach;attach=1322228;image)

You tap into the Lewin ring and since the rotational E-field from the flux doesn't magically end at the ring, you need to run a wire out some distance perpendicularly to any E-field, so that there is no Eind.  Once you are far enough that all fields are negligibly low, you then connect an electroscope.  You can then connect the electroscopes to points A, B, C and D and see what results you get.  What do you expect to see?
Title: Re: #562 – Electroboom!
Post by: bsfeechannel on November 13, 2021, 06:09:43 pm
It isn't "just two resistors and nothing else", it has a closed loop of wire in a varying magnetic field. It has a transformer (as has been pointed out over and over here). If the model doesn't include the transformer, it is an incomplete model.

What transformer? Replace the solenoid by a moving magnet, or by the radiation coming from a radio station. The voltages in your circuit will not add up to zero. This was not only proven theoretically but practically. It is the principle behind the reception of every radio.

And we're not discussing the model. We are discussing if it is possible to measure two different voltages at the same two points by the same or different but identical meters on a circuit subjected to an externally generated varying magnetic field.

It is. Period.
Title: Re: #562 – Electroboom!
Post by: rfclown on November 13, 2021, 06:19:28 pm
It isn't "just two resistors and nothing else", it has a closed loop of wire in a varying magnetic field. It has a transformer (as has been pointed out over and over here). If the model doesn't include the transformer, it is an incomplete model.

What transformer? Replace the solenoid by a moving magnet, or by the radiation coming from a radio station. The voltages in your circuit will not add up to zero. This was not only proven theoretically but practically. It is the principle behind the reception of every radio.

And we're not discussing the model. We are discussing if it is possible to measure two different voltages at the same two points by the same or different but identical meters on a circuit subjected to an externally generated varying magnetic field.

It is. Period.

What transformer? The transformer that is there. What is a transformer? Two coils with a coupled field. It is a circuit element. If you neglect a circuit element in a circuit diagram, it is incorrect. It is no more of an error than if there was a AC (or DC) source in the loop and you neglect to put it in the circuit diagram. It is obvious that this tread isn't going to convince anyone of anything.
Title: Re: #562 – Electroboom!
Post by: jesuscf on November 13, 2021, 06:25:07 pm
I'm afraid this won't make your argument stronger. You really need to take geometry and the fields into account and where the magnetic flux "flows".

The core is split, it's two toroids which both carry about half of the total flux each. His "Lewin loop" is around the center column where the two toroids join, same as the yellow reference loop, they both experience the total magnetic flux. However, when he measures the voltage across the "red" parts, his probe leads and the red wire form a loop around one of the two toroids, encompassing half of the magnetic flux. So what he actually measures is not the voltage across that red wire, it is the EMF in his measurement loop, which is, since it experiences about half of the total flux, half of the voltage induced in his reference loop.

PS: this would be a brilliant experiment: Use a split core with different permeability in each of the toroids, so that the flux is no longer evenly split. I wonder if the voltages would still add up ;)

Yes, you are right, in 'fromjesse' experiment a loop is formed with the bottom red wire with half the magnetic flux, but a loop is also formed with the top red wire with also half the magnetic flux! As a result the induced voltages due to the half magnetic flux in the red wires cancel each other!   As for the test leads of the voltmeter, a voltage is induced in one of the leads and an opposite voltage is induced in the other lead and they cancel each other as well!  The setup ends up measuring only the effect of the central core flux in the bottom red wire.
Title: Re: #562 – Electroboom!
Post by: rfeecs on November 13, 2021, 06:29:16 pm
One can make a lumped model by representing this charge separation by a battery in series with the wire.  Now with a lumped model, KVL works fine.

In my mind where this modeling method breaks down badly is in the case of the uniform resistive loop.  Here the loop is just a high resistance wire, for example.

If the loop is uniform, then there is no static charge anywhere in the loop.  So where to put the battery?

You could just stick one battery at one spot in the loop.  To me, this is a half decent approximation.  It works everywhere except the section of the loop including the battery.

But people are modeling this as lots of little batteries alternating with lots of little resistors.  So the voltage through the loop goes up through the batteries and down through the resistors all around the loop.  So the voltage between any two points on the loop is zero.

At the least this model is utterly confusing.  The voltage between two points is zero?  But what happened to ohm's law?  V=IR?  What about the power?  The resistor is going to get hot, like an induction heater isn't it?  I thought P=(I squared)R, or (V squared)/R.  But there's zero voltage.  So where's the power coming from?  (I know, the batteries).  Not a very good model.
Title: Re: #562 – Electroboom!
Post by: bdunham7 on November 13, 2021, 06:47:47 pm
But people are modeling this as lots of little batteries alternating with lots of little resistors.  So the voltage through the loop goes up through the batteries and down through the resistors all around the loop.  So the voltage between any two points on the loop is zero.

At the least this model is utterly confusing.  The voltage between two points is zero?  But what happened to ohm's law?  V=IR?  What about the power?  The resistor is going to get hot, like an induction heater isn't it?  I thought P=(I squared)R, or (V squared)/R.  But there's zero voltage.  So where's the power coming from?  (I know, the batteries).  Not a very good model.

The problem with trying to model something you don't understand with elements that you do understand and think are equivalent is that if you don't understand it in the first place, how can you know that your model is actually equivalent?

And this issue doesn't just apply to the 'KVLers', although most of them seem messy and unclear, which is why I've devoted my arguments to the issue of what fundamental disagreement causes the two camps to differ.  Unfortunately I'm figuring this out as I go with a pretty rusty education and a bit of spare time. 
Title: Re: #562 – Electroboom!
Post by: rfclown on November 13, 2021, 06:50:11 pm
One can make a lumped model by representing this charge separation by a battery in series with the wire.  Now with a lumped model, KVL works fine.

In my mind where this modeling method breaks down badly is in the case of the uniform resistive loop.  Here the loop is just a high resistance wire, for example.

If the loop is uniform, then there is no static charge anywhere in the loop.  So where to put the battery?

You could just stick one battery at one spot in the loop.  To me, this is a half decent approximation.  It works everywhere except the section of the loop including the battery.

But people are modeling this as lots of little batteries alternating with lots of little resistors.  So the voltage through the loop goes up through the batteries and down through the resistors all around the loop.  So the voltage between any two points on the loop is zero.

At the least this model is utterly confusing.  The voltage between two points is zero?  But what happened to ohm's law?  V=IR?  What about the power?  The resistor is going to get hot, like an induction heater isn't it?  I thought P=(I squared)R, or (V squared)/R.  But there's zero voltage.  So where's the power coming from?  (I know, the batteries).  Not a very good model.

This to me is one of the few posts in this thread to invoke some thinking, not just a reaction. What is being induced in the coupled circuit is a current, not a voltage. A better model might be a current source. You can't distrubute that along the wire length like you can a voltage source, but it removes the problem mentioned here with using the distributed voltage source model.
Title: Re: #562 – Electroboom!
Post by: bdunham7 on November 13, 2021, 07:01:51 pm
What is being induced in the coupled circuit is a current, not a voltage.

Really?  You have a rotational E-field that exerts a force on the charge.  There's no guarantee that they'll move and if there is another counterforce, they won't.  A current source implies that they move no matter what.
Title: Re: #562 – Electroboom!
Post by: jesuscf on November 13, 2021, 07:04:00 pm
What transformer? The transformer that is there. What is a transformer? Two coils with a coupled field. It is a circuit element. If you neglect a circuit element in a circuit diagram, it is incorrect. It is no more of an error than if there was a AC (or DC) source in the loop and you neglect to put it in the circuit diagram. It is obvious that this tread isn't going to convince anyone of anything.

This is a nice summary of the state of this tread.
Title: Re: #562 – Electroboom!
Post by: rfclown on November 13, 2021, 07:28:06 pm
What is being induced in the coupled circuit is a current, not a voltage.

Really?  You have a rotational E-field that exerts a force on the charge.  There's no guarantee that they'll move and if there is another counterforce, they won't.  A current source implies that they move no matter what.

You may be right, I was just replying off the top of my head, but a current source model doesn't imply current. If I draw a current course with nothing attached to it, it is assumed that no current flows. If you get into a debate over that, it will be simillar to this thread.
Title: Re: #562 – Electroboom!
Post by: jesuscf on November 13, 2021, 07:32:58 pm
At the least this model is utterly confusing.  The voltage between two points is zero?  But what happened to ohm's law?  V=IR?  What about the power?  The resistor is going to get hot, like an induction heater isn't it?  I thought P=(I squared)R, or (V squared)/R.  But there's zero voltage.  So where's the power coming from?  (I know, the batteries).  Not a very good model.

There is a very nice video explaining all of this from Bob DuHamel (RSD Academy) that I watched after following a link in a YouTube comment.  The video is not listed and I can not find it know.  As soon as I find it, I'll post the link here.

In an unbroken loop of wire under the influence of a varying magnetic field, each segment of arbitrary length of wire behaves as a voltage source in series with a resistance.  The induced voltage in that segment is consumed by the resistance in that segment.  When we add them up, the net contribution is zero.  That is why if we measure voltage (correctly) across two arbitrary points of the loop of the unbroken wire the voltage we get is zero.  If we open the loop, now the generated voltage at the terminals of the open loop doesn't match the consumed voltage (the current through the resistors is zero) and we get a net voltage output.

The same happens if you add a resistor between the open terminals: the net voltage drop inside the loop doesn't match the voltage generated and we get a net voltage across the external resistor.  If you make a closed loop out of resistors of the same value instead of wire, you'll observe exactly the same behavior!  The net voltage generated in each resistor is consumed by the voltage drop in the resistor and the net effect is zero.  In both cases, (wire loop and resistor loop) KVL always holds. 

Your observation of the loop getting hot is correct!  That is why is not a good idea to short circuit the secondary of a transformer!

EDIT: Here is the video I mentioned above.

https://www.youtube.com/watch?v=-QE8h-UjW2c (https://www.youtube.com/watch?v=-QE8h-UjW2c)

Title: Re: #562 – Electroboom!
Post by: bdunham7 on November 13, 2021, 07:39:01 pm
You may be right, I was just replying off the top of my head, but a current source model doesn't imply current. If I draw a current course with nothing attached to it, it is assumed that no current flows. If you get into a debate over that, it will be simillar to this thread.

As we both know, a real-life current source will have a maximum compliance voltage above which it is no longer a current source.  However, if you are using an ideal current source in a theoretical model, the voltage goes to infinity and your model breaks, unless you are modeling an electric arc.  No assumptions should be made about a broken model, you need to revise it so that works or it is meaningless. 

There's a lot of broken models in this discussion, and even more messy ones.
Title: Re: #562 – Electroboom!
Post by: bsfeechannel on November 13, 2021, 07:42:40 pm
What transformer? The transformer that is there. What is a transformer? Two coils with a coupled field. It is a circuit element. If you neglect a circuit element in a circuit diagram, it is incorrect. It is no more of an error than if there was a AC (or DC) source in the loop and you neglect to put it in the circuit diagram.

A transformer is a lumped component. It is a black box with three, four or more terminals. We know that we excite the primary and we get voltages on the secondary.

But, what is really going on inside it? Do the wires generate voltage? If they do, why is not possible to measure that voltage? Or better, why do we get an undefined measurement? If the wires don't generate voltage, what is in fact generating this voltage?

Quote
It is obvious that this tread isn't going to convince anyone of anything.

I haven't seen a single person that understood that the voltages are not in the wires and were convinced that they were. But seen lots who realized where the misconception is, changed sides and never came back.
Title: Re: #562 – Electroboom!
Post by: bsfeechannel on November 13, 2021, 07:47:10 pm

Why here, of course!

(https://www.eevblog.com/forum/amphour/562-electroboom!/?action=dlattach;attach=1322228;image)

You tap into the Lewin ring and since the rotational E-field from the flux doesn't magically end at the ring, you need to run a wire out some distance perpendicularly to any E-field, so that there is no Eind.  Once you are far enough that all fields are negligibly low, you then connect an electroscope.  You can then connect the electroscopes to points A, B, C and D and see what results you get.  What do you expect to see?

Are you trying to measure the contribution of the displacement charges to the Coulomb field inside the wire?
Title: Re: #562 – Electroboom!
Post by: bdunham7 on November 13, 2021, 07:55:16 pm
Are you trying to measure the contribution of the displacement charges to the field inside the wire?

Sigh.  That is not my ultimate intent nor am I sure that this will do exactly that--although perhaps it boils down to the same thing.  This is not the last step.  So, do I see varying electroscope readings at A,B,C and D or are they all zero?
Title: Re: #562 – Electroboom!
Post by: jesuscf on November 13, 2021, 08:02:20 pm
The core is split, it's two toroids which both carry about half of the total flux each. His "Lewin loop" is around the center column where the two toroids join, same as the yellow reference loop, they both experience the total magnetic flux. However, when he measures the voltage across the "red" parts, his probe leads and the red wire form a loop around one of the two toroids, encompassing half of the magnetic flux. So what he actually measures is not the voltage across that red wire, it is the EMF in his measurement loop, which is, since it experiences about half of the total flux, half of the voltage induced in his reference loop.
Exactly. What he had was NOT Lewin's setup. The meters that were measuring the voltages across the wires were forming additional loops around half of the magnetic field in the opposite direction. No wonder he got negative voltages.

bsfeechannel, in how many loops is the voltmeter circuit in the 'fromjesse' experiment as shown in the figure below?  (Tip: the answer is in a previous post)
Title: Re: #562 – Electroboom!
Post by: jesuscf on November 13, 2021, 08:36:37 pm
Quote
"A toroidal vacuum chamber encircles the core of a large magnet. The magnetic field is produced by pulsed coils; the magnetic flux inside the radius of the vacuum chamber changes with time. Increasing flux generates an azimuthal electric field which accelerates electrons in the chamber.
In the absence of an air gap, there is little magnetic flux outside the core."

Emphasis mine, this is what I was referring to. It's the electric field the electrons interact with.

But it's not independent from the changing magnetic field. The way I see it is they are two manifestations of the same phenomenon. (Like in the propagation of an EM wave, it's not that the changing B field causes a changing E field that causes... ). But anyway, we can consider this just nitpicking.
It appears to me that you now understand Faraday's law in full.

I cannot help imagining Lewin stroking his hands with satisfaction while, in a black hood and cloak, reads your exchange with jesuscf and mutters "Well done, my apprentice... Hihihihi.... Now, strike him down with your path integral!"

May the nonconservative force be with you.

Really?  In the attached diagram courtesy of bsfeechannel, in how may loops is the voltmeter?  Is there some EMF cancellation going on?  Here is a tip: if you draw a circuit and KVL doesn't add up, then you are missing something in the circuit!
Title: Re: #562 – Electroboom!
Post by: thinkfat on November 13, 2021, 08:51:10 pm
The core is split, it's two toroids which both carry about half of the total flux each. His "Lewin loop" is around the center column where the two toroids join, same as the yellow reference loop, they both experience the total magnetic flux. However, when he measures the voltage across the "red" parts, his probe leads and the red wire form a loop around one of the two toroids, encompassing half of the magnetic flux. So what he actually measures is not the voltage across that red wire, it is the EMF in his measurement loop, which is, since it experiences about half of the total flux, half of the voltage induced in his reference loop.
Exactly. What he had was NOT Lewin's setup. The meters that were measuring the voltages across the wires were forming additional loops around half of the magnetic field in the opposite direction. No wonder he got negative voltages.

bsfeechannel, in how many loops is the voltmeter circuit in the 'fromjesse' experiment as shown in the figure below?  (Tip: the answer is in a previous post)

In one loop only. No matter how you see it. Especially if you argue that the wire is a voltage source - the wire will "enforce" a voltage across itself and anything happening in any other part of the circuit will not matter.

There is no other path going through the volt meter, and if the magnetic flux wasn't evenly distributed between the two toroids, you would even measure different voltages "across" each of the red wires.
Title: Re: #562 – Electroboom!
Post by: bsfeechannel on November 14, 2021, 01:20:49 am
Sigh.  That is not my ultimate intent nor am I sure that this will do exactly that--although perhaps it boils down to the same thing.  This is not the last step.  So, do I see varying electroscope readings at A,B,C and D or are they all zero?

I'm not sure, so I'll bet they will show different readings.
Title: Re: #562 – Electroboom!
Post by: Sredni on November 14, 2021, 03:40:07 am
IT DEPENDS ON THE PATH
Of course it does. 

But then why do you ask, and i quote, "Is there potential across the ends or not?" without specifying the path?
Anyway, my yesterday answer was focused on the the long straight conductor through the center of the torus. For some reason (well, I had your sphere machine in mind) I thought you were talking about that all along and that the long U shaped path was just another question at the end (I even skipped it as non related). So, there has been a misunderstanding but what I wrote in my previous answer still applies. It is nevertheless clear that with that U shape the "potential across" meant the voltage on a path joining them in the proximity of the terminals. Sincere apologies for this misunderstanding on my part.

I nevertheless confirm that you should not use the word potential and in general - and even in this case because there are paths that can be run around the core.

The pictures I had drawn yesterday are mainly centered on the partial turn that according to some represent the 'elementary battery', but at this point I might as well post them. The second one was about the nearly complete but still partial turn and that is basically the same as your U shaped turn.
What I wrote in the previous answer still apply - but I would have worded it in a different manner. Can I blame the tequila?

Ok, here is the partial turn and some paths

(https://i.postimg.cc/jqJ7FwKh/screenshot.png)
https://i.postimg.cc/jqJ7FwKh/screenshot.png

and here is still another partial turn that runs almost - but not - completely around the core. You can consider this a representation of your long U coil.

(https://i.postimg.cc/QN5Fq2k4/screenshot-2.png)
https://i.postimg.cc/QN5Fq2k4/screenshot-2.png

As you can see it all depends on the path. And the need for a closed (mathematical, non necessarily material) path is that it's the only kind of path that allows me to apply Faraday's law to get the answer. We already discussed that the local form of Maxwell equations doesn't really change this, because they are partial differential equations that requiire boundary conditions. (Also look up at the definition of curl and divergence and you will see that surfaces and boundary paths, volumes and boundary surfaces are built in into the equations.)

Now, let me answer your question about the U coil. You ask about the electrometers at the end and I have seen the experiment performed with FET electrometers. They put them near an outlet and they sense the (alternating) accumulation of charge at the terminals. The core is in the transformer on a pole near the house in the US, or even a mile away in a big transformer that delivers power to whole city blocks in some parts of the rest of the world. So, I'd say that with a linearly increasing excitation you will see the leaf open - for as long as you can ramp up the current in the primary.

What I am perplexed about is your fixation with 'where there are no fields'. If the charges are there, there is at least their field, so you can't do away with it. If there are no fields to push and keep them there, then the charges are no reason to accumulate there.

Quote
To put this in context, I was comparing three phenomena- a conductor in a static, irrotational conservative field, a conductor in a rotational non-conservative field derived from dB/dt and a conductor moving across magnetic lines as in a generator, where the charges experience the Lorentz force.  So in the first case, I think we all agree with your assertion.  Apparently, and don't attribute this assertion to me, the third case does result in a potential gradient over the length of the conductor.  Is that right or wrong?  And if it is right, then how do you differentiate the effect of the Lorentz force from the local Eind force in the second example?

Well, yesterday I also did a couple of pictures for the straight rod near a variable magnetic flux region. They are no different from the ones above, I only separated the paths that give zero voltage, like these

(https://i.postimg.cc/cJ2JRhKG/screenshot-4.png)
https://i.postimg.cc/cJ2JRhKG/screenshot-4.png

from the ones that, by linking the emf, give a voltage different from zero (and a multiple of the EMF)

(https://i.postimg.cc/1zk9nzT9/screenshot-5.png)
https://i.postimg.cc/1zk9nzT9/screenshot-5.png

The second picture is the reason I wrote - adding in the edit: IN GENERAL

Quote
"and this is again (EDIT: in general) wrong for the reason above."

The paths above show that in general VBA can be nonzero.
Immediately after I added (emphasy added now):

Quote
"But, if you consider a region of space that contains all of your piece of conductor and none of the magnetic flux, like a cylinder around the rod in the middle of the torus, in that region of space all paths you can imagine will never enclose the magnetic flux region. in this sense you can consider it as equipotential."

And this is basically the situation depicted in the former picture. If I consider paths that do not run around the variable magnetic region. I can consider - AND PLEASE NOTE THAT I AM PUTTING IT IN QUOTES - the conductor as 'equipotential', meaning that the voltage (difference) between any two points of the conductor (and in particular on the surface) is zero. This is apparently indistinguishable from the conductor polarized by immersion in the electrostatic field generated by two charged plates.

I believe that there could be a distinction, tho. It's in the nature - I would say 'the shape'  - of the field. In both cases, if you stay in your 'safe space' or 'magnetic free bubble' you can consider the conductor 'equipotential' (THE QUOTES! THE QUOTES!!!), but I seriously doubt that you could manage to make the field lines (and presumably the distribution of charge on the surface) the same in both cases (maybe a few lines will be reasonable close but then the fields will differ). What I mean is that the shape of the field lines coming out of charged plates will be straight if the plates are parallel, with the field magnitude constant, or hyperbolic shaped if the plates are at an angle, but you probably won't be able to replicate both the shape of the lines and the way the field increase or decrease on large portions of space. (I am not 100% positive.)
This in turn means that even the distorted field will be different.

But from the point of view of an observer of a polarized body that is in a room next to the room where a giant solenoid is being powered, the voltage along any path he can devise inside the room will be zero.
Does this answer your question?

I am not going to touch the motional case even with a ten foot pole. We are already wasting too much space and time to address the Lewin ring and we would fall down a relativistic rabbit hole with no end. I suggest you look up Purcell for an introduction to that kind of stuff.
Title: Re: #562 – Electroboom!
Post by: bdunham7 on November 14, 2021, 03:53:25 am
Can I blame the tequila?

You may.

Quote
What I am perplexed about is your fixation with 'where there are no fields'. If the charges are there, there is at least their field, so you can't do away with it. If there are no fields to push and keep them there, then the charges are no reason to accumulate there.

A very good point, one which occurred to me several hours ago as I was writing another response.  I'm going to have to be very careful how I state the case, but it isn't fatal to the concept that I'm trying to get across or at least I don't think so yet.

Quote
I am not going to touch the motional case even with a ten foot pole. We are already wasting too much space and time to address the Lewin ring and we would fall down a relativistic rabbit hole with no end. I suggest you look up Purcell for an introduction to that kind of stuff.

OK, it's a sideshow anyway.  But can you have a glance at my last diagram in my reply to bfseechannel and see what you think the electroscopes would show at A, B, C and D in that case?

I'll try to read and digest the rest of what you wrote tomorrow.  I don't think it is a waste of space or time if it advances anyone's understanding (mine, for example) as opposed to degenerating into a food fight.
Title: Re: #562 – Electroboom!
Post by: Sredni on November 14, 2021, 04:15:01 am
I can answer for a ring of finite dimensions, where the fields are nonzero.
In general, unless the ring is perfectly circular and isotropic - without gaps and localized resistors - the symmetry is broken and surface charge will show up not only at the terminals of a gap (or at the discontinuities when the copper becomes the highly resistive material of a resistor) but also on the lateral surface. So, there will be in general be charge on the conductor even in that position because the coloumbian field has to compensate the perfectly rotating Eind field in the conductor.

The amount of surface charge necessary to do this is in general unbelievably small. With ordinary circuits it can be just a handful of electron charges! Therefore, while the charge would be there on the surface, I seriously doubt it could be measured with a leaf electrometer, or any electrometer that will interact with the field.

And if you make the ring so big that the field becomes negligible you are only making matters worse.
Jefimenko, for lumpled circuits in DC, had to use high voltage generators to get pictures of the electric field in the space between the conductors.

Obviously it is implied that you use perfect conductors with zero resistivity, but you need to clarify this concept of vanishing field.
In reality, for a ring big enough the resistance of the ring would make the current so small that you won't be able to see anything at all, even at the terminals (imagine a ring one light yeat in radius from Lewin's solenoid).
Title: Re: #562 – Electroboom!
Post by: bdunham7 on November 14, 2021, 04:40:38 am
In reality, for a ring big enough the resistance of the ring would make the current so small that you won't be able to see anything at all, even at the terminals (imagine a ring one light yeat in radius from Lewin's solenoid).

OK, fair enough.  I wasn't specific about my 'negligible fields' or the length of the wire.  For concentric rings, Faraday's law dictates that all of the rings have the same total EMF Eind so you can think of that as volts per radian--if the total EMF Eind is 1 volt, there are 1/(2 * pi) volts/radian. If I've flubbed that, well..Scotch.

I'm only thinking of a few meters of wire and while there still will be a circular field that only diminishes with the radius, I can arrange the wire and electroscope to be as perpendicular to those fields as possible so they have minimal effect.  I think I only need them to diminish to small fraction of the field near the ring.  If I want to try them in positions that are 20 degrees apart on the ring, as long as the entire apparatus stays within a 1 degree arc of the perpendicular line, I have at most a 5% error due to the external field at the electroscope.  As for the sensitivity of the electroscopes, these are theoretical electroscopes, so assume they have a minimum sensitivity of a small fraction of the total ring EMF.
Title: Re: #562 – Electroboom!
Post by: Sredni on November 14, 2021, 05:11:30 am
The reason you always link the same EMF, which is the circulation of the Eind field on a closed path when the ring is not there, is that outside of the the circular solenoid the Eind field decreases with 1/r, while the perimeter of the region of space you will be putting your circular ring will increase with r. The magnitude of the Eind field along a circle is constant. So when you integrate on a bigger ring, you have a smaller field Eind but a bigger path that compensate each other to give the same value for the EMF. In principle, you will link the same EMF even at 10 light years.

Let's see: 1V EMF at 10 cm radius means and electric field Eind of 1V/(2 pi 0.1m) = 1.59 V/m
Let's make the ring 1 m of radius and the Eind field becomes 0.159 V/m.
At 10 m it will be 15 mV per meter.

But that field Eind will no longer be there once you put your copper ring with the resistors in the circular path of radius 10cm or 1m or 10m. It will be cancelled by and equal and opposite electric field generated by the charges that it initially displaced and after a handful of nanoseconds have crammed up (for the most part) at the interfaces with the resistors. 
These charges will build up the field inside the resistors and that is why sometimes Lewin said that all the emf shows up at the resistors: you have 1V along the whole circle, and nearly 0.9V appears where the 900 ohm resistor is, and nearly 0.1V appears where the 100 ohm resistor is. In the wire there is cancellation and all that is left in copper is of the order of the millivolt (corresponding to a field of some tens of microvolt per meter.)
EDIT: and then near the ring you will still see a distorted induced field that will interfere with your instrument.

Some residual charge will be also present on the surface of the conductor to steer the compensating field, but as I said, I would not expect anything you could measure with a leaf electrometer. It might be interesting, but not easy, to compute the density of charge on the lateral surface but to what end?
Title: Re: #562 – Electroboom!
Post by: Sredni on November 14, 2021, 03:05:32 pm
I don't think it is a waste of space or time if it advances anyone's understanding (mine, for example) as opposed to degenerating into a food fight.

This is the only post I'll made related to the motional case. And it's a silent one.

(https://i.postimg.cc/m2QQ0KYk/Conservativs-vs-Rotational-vs-Motional-small.png)
https://i.postimg.cc/m2QQ0KYk/Conservativs-vs-Rotational-vs-Motional-small.png
Title: Re: #562 – Electroboom!
Post by: rfeecs on November 14, 2021, 05:53:42 pm
I'm only thinking of a few meters of wire and while there still will be a circular field that only diminishes with the radius

Your forgetting that the core is a toroid.  Looking at the cross section view, there are two cores.  So as you move away there are two sets of fields that rapidly cancel.

Another thought experiment would be to spit the wire in the middle, move it out of the field and then join it back together.  Then you can look at the coulomb potential only.

Or you can think of the E and B fields and the charge and current as the only primary quantities that matter; and consider the potentials as just mathematical conveniences.
Title: Re: #562 – Electroboom!
Post by: Sredni on November 14, 2021, 07:14:40 pm
Your forgetting that the core is a toroid. 

You're right. I keep bringing up the infinite solenoid to exploit the symmetry and the simple analytical expression of the field.  I think we should all stick with that to make things simpler and consider the toroid as a "poor man's folded infinite solenoid" for lab experiments with voltages and currents in the ring.
Title: Re: #562 – Electroboom!
Post by: Jesse Gordon on November 15, 2021, 08:54:30 pm
Your forgetting that the core is a toroid. 

You're right. I keep bringing up the infinite solenoid to exploit the symmetry and the simple analytical expression of the field.  I think we should all stick with that to make things simpler and consider the toroid as a "poor man's folded infinite solenoid" for lab experiments with voltages and currents in the ring.

The difficulty with the toroid is that the folded nature makes some hands-on experimental observations difficult.

For smallbrains like me, we need simple observable reality or we don't feel it's real.

I am, however, sorely tempted to use a carbide or diamond bit to drill a small hole in the side of a formerly toroidal core to facilitate such measurements  :-DD :-DD :-DD
Title: Re: #562 – Electroboom!
Post by: bsfeechannel on November 15, 2021, 09:23:47 pm
Helmholtz is your friend.
Title: Re: #562 – Electroboom!
Post by: Jesse Gordon on November 15, 2021, 10:05:36 pm
It isn't "just two resistors and nothing else", it has a closed loop of wire in a varying magnetic field. It has a transformer (as has been pointed out over and over here). If the model doesn't include the transformer, it is an incomplete model.

What transformer? Replace the solenoid by a moving magnet, or by the radiation coming from a radio station. The voltages in your circuit will not add up to zero. This was not only proven theoretically but practically. It is the principle behind the reception of every radio.

And we're not discussing the model. We are discussing if it is possible to measure two different voltages at the same two points by the same or different but identical meters on a circuit subjected to an externally generated varying magnetic field.

It is. Period.

What transformer? The transformer that is there. What is a transformer? Two coils with a coupled field. It is a circuit element. If you neglect a circuit element in a circuit diagram, it is incorrect. It is no more of an error than if there was a AC (or DC) source in the loop and you neglect to put it in the circuit diagram. It is obvious that this tread isn't going to convince anyone of anything.

Indeed sir! This entire topic has me baffled.

Folks have argued so strongly that KVL cannot work in Lewin's circuit.

Dr. McDonald said "Lewin's circuit is within the range of applicability of Kirchhoff's loop equations, which can be used to predict measurements by the 'voltmeters' in the experiment. "

Dr. Belcher cited Feynman and said "In this sense, KVL holds, as argued by Mehdi Sadaghdar ..."

So at a very least, we ALL ought to be able to see that it is a very complex topic and not even MIT/Princeton physics professors agree on it.

(However, I do note that the two who said "Mehdi is right" are thoughtful meticulous individuals who did some research to come to their conclusion, whereas Dr. Lewin just dismissed any questions out of hand and stuff.  If I was going purely on the odds, Belcher & McDonald would be much more likely to be right because they are willing to question their assumptions and sort through things.)

I'm with Dr. McDonald and Dr. Belcher.

As for me, I have no degrees and am no math wiz. But I have a volt meter and scopes and stuff, and feel like I understand the basics of EE. When I watch Lewin's lectures (and other physics lectures) it all makes sense. But when he did the thing with the Lewin Loop, it seemed odd. So I measured.

As a ham radio guy, I know there are cases where KVL does not hold, or at least it's difficult to measure, because of the non-finite speed of light.
(i.e. when your volt meter leads become feed lines, you then have to look at the propagation, and two voltages may not arrive at the correct time, so a phase shift may give the appearance of voltages not adding up. However, I'll give 100 fresh never used blank pixels (the picture element kind) to the first person who can present me ANY scenario for which the best and proper application of KVL fails.)

But in cases where the basic laws tell us how we can unambiguously probe, and where the wavelength far exceeds the size of our apparatus, KVL seems to hold so far as I have measured.

What baffles me is that all kinds of people crop up to argue, but I'm not even sure what they are arguing. I'm not sure they know what they are arguing.

Usually, when I have a disagreement with someone, it's not too long before I understand where I went wrong, or they understand where they went wrong.

But in this case, even after months of discussion with very intelligent people, none have been able to actually put into words something that helps me see where I've gone wrong, if such is the case.

I almost feel like it's some vague technicality that nobody can put their hands on.

And it's obviously a subtle point, because even Dr. McDonald and Dr. Belcher disagreed with the Master Lewin.

One interesting clue is that over the course of my discussion with various people, we would each make contradictory predictions about real-world measurements, so I would build a contraption to measure it, and my predictions were right each time. So this shows that the people arguing against KVL did not have a full understanding, or they would have predicted correctly. But the fact that their predictions were wrong didn't phase them, so I don't know what to say.

I also find it odd how many strange assumptions seem to be made. Nobody seems to know that resistors, capacitors, and inductors all have resistance, capacitance, and inductance (excepting of course super conductors.)

I find it odd that people seem to think that all voltage drop must be purely ohmic. It's almost like there are two definitions for voltage - one being the kind that a volt meter reads, and another kind that differs from what a volt meter reads - but by now somebody should have said "Hey Jesse, problem is you're using a volt meter. That's not the volts we're talking about...."

I also find it odd that so many people call it a measurement when there is an undocumented error offset added to the "measurement." When you have two unknowns, you are not measuring either of them.

As for me, so far no body has actually shown how Lewin's loop breaks KVL, except by putting undocumented secondary windings in series with the volt meter leads. Doh.

One very kind and sincere fellow and I had a very long discussion on it. At one point I basically said "So if Lewin can fail KVL with undocumented probe paths, then let's see how MaxEQ handles the same thing. I'll tell you the volts per turn, and I will tell you that the volt meter will be on the LEFT side of my solenoid, but, like Lewin, I will not tell you the locations  or path of my probe leads, and you can calculate the voltage I will read.." He couldn't, so I guess MaxEQ fails too?

The fact is, that all measurements fail when you have an unknown variable offset error inline with your meter.

And I have asked so many of them "What about a regular iron core transformer, can it's outputs be used in a loop and have KVL hold?" it is very hard getting them to answer that. Obviously the output of a regular iron core transformer will behave the same as a pair of DDS waveform generators making the same waveform.

I really wish the people who say that KVL fails Lewin could put it into simple words that I could understand.

They say that the measured voltage is path dependent. But if the probes follow a path that prevents or cancels induced voltage, then it's the same as if the magnetic field were contained entirely within the lumped component.


 :-// And after all this, not one correctly lumped correctly probed working model has been presented which shows KVL to fail. I say it's high time. :-//

And after all, KVL does require lumped components. *always*. It cannot be decried as having failed when it's not even KVL that is being tried. If you made a KVL loop out of open wire wound resistors, but instead of measuring voltages always at the ends of the resistors you measured at random locations inside the resistor, of course KVL will fail, even on a DC circuit - but it's not KVL that failed, it was the lumping that failed, it's only a strawman argument.

MaxEQ also fails if you don't model reality. All of physics fails if it's not modeling reality.

Thus I ask myself, what exactly is the actual question at hand?
Title: Re: #562 – Electroboom!
Post by: Sredni on November 15, 2021, 10:06:35 pm
The difficulty with the toroid is that the folded nature makes some hands-on experimental observations difficult.

How so? I have seen this experiment done on toroidal transformers (MIT), and even on EI transformers (Sam Ben-Yaakov, Ben Gurion University). The EI are 'doubly folded': you just need to be careful not to enclose the 'legs' where the flux in your measurement loop.

Quote
I am, however, sorely tempted to use a carbide or diamond bit to drill a small hole in the side of a formerly toroidal core to facilitate such measurements  :-DD :-DD :-DD

Why on earth would you want to do that? You need to avoid the magnetic flux region.
Please explain.
Title: Re: #562 – Electroboom!
Post by: Jesse Gordon on November 15, 2021, 10:42:03 pm
...
But you probably won't find many confutations in the comments section, because Duhamel, like fromjesse, delete the comments that tell them they are wrong and why. For example, in his last video "Voltage in a loop is weird", a post by Silicon Soup stating that by putting the probes inside the magnetic region a voltage was induced in the loop has disappeared.

Here's the reason:

https://i.postimg.cc/vZ6BM4Wn/screenshot-3.png
...

Just to set the record straight, your statement is false.

Neither I nor Duhamel delete comments just because they disagree with us.

I don't know whether Bob deleted any specific comments, but I don't recall deleting any specific comments, but what I did do is ban ONE particular person (felinus retardus) who was being insulting to my viewers as well as myself. I did not delete all of his comments, if you go look they are still there. In fact, I see his comments on Bob's videos right now and goodness knows his comments are all over my videos right now, as well as comments from other people who disagree with me.

In fact, if you look at my videos on this topic, the bulk of the comments ARE from people who disagree with me, and their comments are still all there.

Neither Bob nor I delete comments just because they disagree with us.

But that one user that you ask about is just mean -- and that's why I banned them(?) and that's why Bob banned him, because he was doing the same thing there.

I actually unbanned felinus retardus at one time and he immediately started up with the insults again, so I banned him again. That interchange is still public as well.

So please, don't accuse us of that, especially when you can literally look at the video comments and see lots of people disagreeing, including comments going back nearly to the upload date in the case of my videos.

Let's try to figure out the issue. Obviously there are very sincere people on all sides of this, trying to make sense of it.

If it was a simple matter and only pure idiots like me got confused by it, then Professor Dr. Kirk McDonald PhD of Princeton wouldn't say IN WRITING "Lewin's circuit is within the range of applicability of Kirchhoff's loop equations, which can be used to predict measurements by the 'voltmeters' in the experiment. "

And Professor Dr.  John W. Belcher PhD of MIT wouldn't cite Feynman and then say IN WRITING "In this sense, KVL holds, as argued by Mehdi Sadaghdar ..."

Perhaps show some patience for those you disagree with?

Thank you.
Title: Re: #562 – Electroboom!
Post by: Jesse Gordon on November 15, 2021, 10:53:01 pm
The difficulty with the toroid is that the folded nature makes some hands-on experimental observations difficult.

How so? I have seen this experiment done on toroidal transformers (MIT), and even on EI transformers (Sam Ben-Yaakov, Ben Gurion University). The EI are 'doubly folded': you just need to be careful not to enclose the 'legs' where the flux in your measurement loop.

Quote
I am, however, sorely tempted to use a carbide or diamond bit to drill a small hole in the side of a formerly toroidal core to facilitate such measurements  :-DD :-DD :-DD

Why on earth would you want to do that? You need to avoid the magnetic flux region.
Please explain.


Let's start at a very simple place where my small brain gets it. Ok?

Let's say I have two small but powerful battery operated optically synchronized DDS waveform generators each generating a 60Hz 100mV AC RMS sinewave.
Further, let's say I have a 100 ohm resistor and a 1000 ohm resistor and I use these four components to form a series loop, alternating resistors and DDS units.

If I take a four channel fully isolated input scope and connect each of my four elements to their own scope input with positive-clockwise polarity, will the sum be zero? Let's say I turn on the math channel on the scope, and sum all the inputs, will it be a straight line?

KVL will hold fine in this case, right?
Title: Re: #562 – Electroboom!
Post by: Jesse Gordon on November 15, 2021, 11:07:36 pm
...
PS: this would be a brilliant experiment: Use a split core with different permeability in each of the toroids, so that the flux is no longer evenly split. I wonder if the voltages would still add up ;)

Thank you for the invite here my friend!

And yes, the two toroids are slightly different from eachother. They actually really are two toroids in this particular transformer, each made from a looong strip of sheet metal wound up, and there are two of them in there, butted up.

And evidently, one has a little more iron to it because it has more voltage across it.

But regardless, the sum still adds up to "zero." (within the resolution limitation of my volt meter.)

However, if you would like, I would be glad to do an experiment literally with two different toroids which are of significantly different sizes, and do the same test to show you that even then, all the voltages add up to zero going clockwise around the loop.

Would that be meaningful to you? Do you think the voltage would no longer add up to zero?
Title: Re: #562 – Electroboom!
Post by: Sredni on November 16, 2021, 01:46:28 am
Ok, let's set the record straight. This will be my last post on the topic of your disappearing comments and Bob's merry deletions, but I really need to set the record actually straight.

...
But you probably won't find many confutations in the comments section, because Duhamel, like fromjesse, delete the comments that tell them they are wrong and why. For example, in his last video "Voltage in a loop is weird", a post by Silicon Soup stating that by putting the probes inside the magnetic region a voltage was induced in the loop has disappeared.

Here's the reason:

https://i.postimg.cc/vZ6BM4Wn/screenshot-3.png
...
Just to set the record straight, your statement is false.
Neither I nor Duhamel delete comments just because they disagree with us.
I don't know whether Bob deleted any specific comments,

You literally contradicted yourself from one sentence to the other.
Sentence two says "Duhamel does not delete comments because..."
Sentence three says "I do not know whether Bob deleted any specific comments..."
Did you talk to Bob expressely about this? Sentence three says you didn't. Also says you seem fine about talking of what you do not know.

Record straightened: I can recall at least three posts that disappeared from RSD A. videos. One is the comment from Silicon Soup, stating that by running the probes in the magnetic flux region he was intercepting flux and this inducing a voltage. It was deleted along with a timestamped comment by Copernico Felinis (or, by using your non-insulting name: felinus retardus). Another one is a comment of I believe Mick Vall (Or Mark Fruchtman, a name like that... M---- --a---) where he told Bob that he was wrong and he should not be teaching this wrong concepts because it would cause confusion in students (and he also explained why and where he was wrong). The whole thread disappeared, along with other comments discussing technicalities.
So, keep your "false" to you.
And then there is that exchange with Mr. Anderson. It tells all you need to know.

Quote
but I don't recall deleting any specific comments, but what I did do is ban ONE particular person (felinus retardus) who was being insulting to my viewers as well as myself. I did not delete all of his comments,
But you did not publish some?

I guess that, regarding the insults, you meant comments like this one  - in that case I can understand

(https://i.postimg.cc/N081Kppk/screenshot-fromjesse-insults.png)
Source: your channel - image here https://i.postimg.cc/N081Kppk/screenshot-fromjesse-insults.png (https://i.postimg.cc/N081Kppk/screenshot-fromjesse-insults.png)

Oh, my bad. That was Samuel Snerden cold open post and he was talking about Lewin. Did you ban him as well, for insulting your viewers? But of course not. Lewin is certainly not a viewer of yours.
Also about insults, and then I'll drop it definitively: I have been through the exchange you are having with ThinkFat. In just two or three comments you manage to call bsfeechannell: "clueless", "totally idiotic", "either he's fulla BS or he's not being honest", "your hero the ignorant bsfeechannel", "he's either ignorant or disingeneous".

Apart from that thing about pots and kettles, I don't think your ban was motivated by insults. You seem pretty comfortable with insults in your channel. I believe it's something else. I wonder what that could be.  :-//
Maybe when you are cornered, you get insecure?

Quote
Let's try to figure out the issue.

Will you start insulting people in here, as well?
If not, I can try to explain what is wrong with what you believe.
Namely:

1. That sentence of Belcher is about the RLC lumped circuit of section 10. Read pages 15 and 16. It's not about the unlumpable Lewin ring.

2. The note of McDonald (it's not a paper, it's a note for his students) has been through many revisions, so you should also specify the date it was last updated. The most recent I have on my laptop says November 14, 2018. And he uses a definition of "voltage drop" that 'others' (namely the IEC) call 'scalar potential difference'. Therefore he is talking about the component of the voltage that is solely determined by the conservative electric field generated by the distribution of charge displaced by Eind. It's just that, and he says so on page 10, after formula (35). The scalar PD alone is insufficient to describe the physical system. In fact, you cannot even apply Ohm's law to that 'voltage drop' as he calls it. And he acknowledges that voltmeters do not measure that 'voltage drop' of his, but the path integral of the total electric field Etot (as Belcher says, as well).
Therefore, when stores will sell AC voltmeters that can read that scalar potential difference, that definition of his will come very handy. Till then, a lot of people prefer to use actual voltage, the one that can be measured by voltmeters and that is path-dependent. It is also much easier to treat.

3. By reading your thread with ThinkFat for the video "The Lewin loop inside an iron core - KVL still holds" it appears to me you have problems with basic electromagnetism. You say con can "correctly lump" the circuit, but Lewin's ring is not lumpable. You also seem to think that if your "volt meter leads do not cut through any changing magnetic fields [...], according to Faraday's law [...] there will be no voltage induced along my volt meter leads".
Do you really think that in order to induce a voltage, the conductors need to pass through the variable magnetic field region?
Is that the reason you want to drill a hole into the toroidal core?
You also seem anxious about looking inside a transformer. But that is the crux of the problem: Lewin's ring has placed two resistor inside the coil of a single turn transformer, creating a circuit that cannot be model with a lumped component.
Title: Re: #562 – Electroboom!
Post by: bsfeechannel on November 16, 2021, 02:13:37 am
Dr. McDonald said "Lewin's circuit is within the range of applicability of Kirchhoff's loop equations, which can be used to predict measurements by the 'voltmeters' in the experiment. "

You said you have no degrees and you're no math wiz. Probably you didn't read the entire paper by McDonald, or if you did, you didn't understand it.

What he says is that, specifically for Lewin's circuit, it is possible to consider the EMF as a generator element of an equivalent circuit and still apply KVL EQUATIONS. Which is true.

But he stresses that this EMF is nowhere to be found with voltmeters in the circuit.

In Lewin’s example, the magnetic flux in the primary solenoid may well be within a small coil, but the secondary consists of only a single “turn”, so the associated inductive EMF is not well localized, but rather is distributed around the entire secondary loop. Then, since inductive EMF’s are associated with a vector potential, rather than a scalar potential, it can be misleading to interpret the inductive EMF as related to a “voltage”.

Lewin's beef is that people read that you can use KVL EQUATIONS to calculate the voltage across the resistors and try to find this EMF with voltmeters. You'll never find it. People say that the voltages are in the wires. McDonald denies that. It's all over the circuit, and it is not a voltage. So voltmeters won't measure it. (In other words, this circuit is "unlumpable", and modeling it so as to make KVL work is just a math trick).

MacDonald says more.

Kirchhoff’s (extended) loop equation (1) does not apply to all possible circuits, and gives a poor description of circuits whose size is not small compared to relevant wavelengths, in which effects of radiation and retardation can be important. Examples such as Lewin’s in which the self inductance of the entire loop could be important must be treated with care.

So Kirchhoff's law doesn't always hold, does it?

He calculates and confirm that the two voltmeters in Lewin's experiment will show two different voltages even though connected to the same points in the circuit and then declares:

These results were validated by experiment during Lewin’s lecture demonstration.

Quote
"In this sense, KVL holds, as argued by Mehdi Sadaghdar ..."

In what sense? Have you read the whole paper? He explicitly said that the voltage through an inductor is zero. Of course it is! It's just a piece of wire! But across the terminals of the inductor it is defined by its inductance (path dependence of voltage). And IN THIS SENSE, KVL holds. Which is true.

And here we come to the Mehdi problem.

Mehdi claims that KVL ALWAYS holds. Which is not true. He claims that Belcher agrees with him. He doesn't. Belcher says KVL only holds for specific conditions. He says that Lewin is wrong and invokes McDonald. MacDonald doesn't say Lewin is wrong anywhere in his paper. His argument is because he thinks that Lewin presented his circuit as a paradox that cannot be solved in the confines of Kirchhoff's equations. He shows it actually can. But in fact there's no paradox--that's Lewin's argument--when you realize the circuit is immersed in a non-conservative field, which is a much broader concept, that allows you to understand the problems to which McDonald says Kirchhoff's equations can't be applied.

Mehdi claims the voltages are in the wires, that Lewin doesn't know how to probe his circuit and many other irrational and nonfactual assertions. We just can't accept that.

Title: Re: #562 – Electroboom!
Post by: bdunham7 on November 16, 2021, 02:32:43 am
Therefore, when stores will sell AC voltmeters that can read that scalar potential difference, that definition of his will come very handy. Till then, a lot of people prefer to use actual voltage, the one that can be measured by voltmeters and that is path-dependent. It is also much easier to treat.

I haven't time for a long post as I sip my Scotch and fix some power sensors but I have two observations.  The first is that in fact the vast majority of times that a voltmeter is used, the user is indeed attempting to measure that 'scalar potential' and would regard any electric fields along the path that are attributable to any other cause to be 'interference'.  And there are a lot of ways of dealing with interference.  I would say that most of the KVLer/YouTubers attempts are exactly that--an attempt to find a path where that circular electric field has zero net contribution.  Mathematically it seems obvious that in-plane without any sort of shielding--has anyone tried shielding?--with a uniform circular field, such a path does not exist.  But in the real world, we usually actually do find such a path, when we can't things gets interesting. 

Quote
creating a circuit that cannot be model with a lumped component.

Second, you can lump it just fine--around the loop you  have two arbitrarily small resistors with -0.9 and -0.1 volts and two semicircles of wire with 0.5 volts each, with the caveat that in the planar paper universe that the problem exists, there is a circular electric field about the center of the loop that corresponds to 1/360 volts per radian, and thus any attempt to measure voltage with a normal voltmeter with test leads will result in 'interference' of 1/360 volts per degree of difference in the radial position of the two test lead ends.  You may not agree with that way of thinking and you may even be right, but the model still works.

Now I'd also like to point out that if you did not know the total loop EMF, I'm quite sure you would be using a KVL-like method of determining it.  You would measure the voltage across the arbitrarily small resistors  (no E * dL, no 'interference', measuring only the scalar potential across the resistor), add those up KVL-style and then assume, correctly of course, that the total EMF Eindon the loop is equal to that.  So instead of the two semicircular wires with 0.5V, you have KVL + superimposition, meaning superimposition of the EMF Eind.  (-0.9 - 0.1 + 1 = 0). 





Title: Re: #562 – Electroboom!
Post by: bdunham7 on November 16, 2021, 02:52:00 am
It's all over the circuit, and it is not a voltage. So voltmeters won't measure it.

It is all over the circuit, but as for it not being a voltage that is a very confusing, very unhelpful and vast oversimplification.  The reason a voltmeter won't measure it is simply because in the problem as drawn, there is no way to connect test leads without getting an exactly equal Eind in the test leads as you have in the wire.  If a path existed without that issue, you could measure the voltage from Ecoul directly.  Note that I am not making any statement here about the existence of such a path.  Your insistence on only full turns is simply the same assertion, only allowing that for exact full turns, there is such a path.

Quote
He calculates and confirm that the two voltmeters in Lewin's experiment will show two different voltages even though connected to the same points in the circuit...

The reason for that is the presence (not absence, as has been asserted elsewhere) of dB/dt in the measuring loop.  The way Lewin presented this confused me and many other people, and that is probably the main reason people are all upset and call him a fraud.  His whole demonstration is designed to provoke rather than explain, IMO--although I can't say whether that is good or bad.

Quote
And here we come to the Mehdi problem.

I haven't watched, but I suppose he didn't clear anything up?  SiliconSoup's presentation was much more helpful although I still think it is possible to think about the problem somewhat differently and still not be 'wrong'.
Title: Re: #562 – Electroboom!
Post by: Sredni on November 16, 2021, 03:00:49 am
in fact the vast majority of times that a voltmeter is used, the user is indeed attempting to measure that 'scalar potential' and would regard any electric fields along the path that are attributable to any other cause to be 'interference'. 

In general that's the spirit.  And in fact, all you need to make sure in order for the voltage shown by your voltmeter to be the same as the voltage 'along' (and very often 'across') the branch of circuit you are probing, is making sure there are no variable magnetic fields in your measuring loop - the one including said branch.
That is why the voltmeters outside the ring measure correctly the voltages along (and across for any path that does not run around the core) the branch with the nearest resistor.
The other measurement loop is marred with an 'interfering voltage' of the EMF of one full turn.

Quote
And there are a lot of ways of dealing with interference.  I would say that most of the KVLer/YouTubers attempts are exactly that--an attempt to find a path where that circular electric field has zero net contribution.

I am well aware of that: they are trying to strip the contribute of the Eind field to only get the contribute of the Ecoul field alone, as if the displaced charge was held in place (instant after instant) by nails, or some superglue. The conservative Ecoul gives a path integral that admit a potential function, what we call scalar potential difference. It is not path dependent and you can apply your KSPDL to that, if it makes you happy.
But removing ALL contributes of Eind means you are no longer considering the physical system in its entirety: the Eind is what makes the charge accumulate in the ring, and along with Ecoul makes the current flow in the ring. It's not like avoiding external, spurious, unwanted, interference from your measurement loop. It's removing the juice that makes your circuit tick.

Decomposing Etot into Ecoul and Eind - and then considering the respective electric scalar potential phi and magnetic vector potential A is nothing new under the sun (it's the Helmoltz decomposition of a vector field into irrotational and solenoidal parts). But your system requires both these fields (or their associated potentials)  to be described. It's fine if you know what you are doing, but pretending the scalar potential difference is the actual voltage (integral of E.dl) is misleading to say the least.

Quote
Quote
creating a circuit that cannot be model with a lumped component.
Second, you can lump it just fine

No, you can't.
In order to lump it you need to devise a circuit path that does not include the variable magnetic region.
What you have in mind - one or multiple 'partial inductor' cannot satisfy that condition. You must split the magnetic region in two or four region in order for the circuit path to pass in between them.
No can do.

What you create is a lumped circuit, different from the unlumpable Lewin ring.
It will give you the correct current and the correct voltages across the resistors but does not correctly describe the voltage along the arcs. You can at this point replace the ring with a single lumped coil. You will see a massive jump in voltage.
How many times I had to explain this...
Title: Re: #562 – Electroboom!
Post by: bdunham7 on November 16, 2021, 03:23:20 am
That is why the voltmeters outside the ring measure correctly the voltages along (and across for any path that does not run around the core) the branch with the nearest resistor.
The other measurement loop is marred with an 'interfering voltage' of the EMF of one full turn.

No, the voltmeter measures the EMF of the outside loop--the far side of the ring plus its own test leads, less the voltage drop across the far resistor.  The near resistor only determines the loop current (along with the far one) and thus the voltage drop across the far resistor.   :)

This is where the second Lewin 'explanation' diagram was confusing, because it showed three loops and three EMF circles in each one.  The thing to point out if one is trying to make the situation clear is that Eind is the same across the test leads as it is across the span that includes the near resistor from one connection point to another, and that this Eind is not the same as the voltage drop across the near resistor.

(https://www.eevblog.com/forum/amphour/562-electroboom!/?action=dlattach;attach=1324184;image)

Quote
It will give you the correct current and the correct voltages across the resistors but does not correctly describe the voltage along the arcs.

If the wire section is a lumped component, we only worry about the voltage at the ends.  I wouldn't claim that a lumped model could possibly address the issue of the voltage gradient or equipotentiality of the wire itself.
Title: Re: #562 – Electroboom!
Post by: Sredni on November 16, 2021, 03:31:13 am
That is why the voltmeters outside the ring measure correctly the voltages along (and across for any path that does not run around the core) the branch with the nearest resistor.
The other measurement loop is marred with an 'interfering voltage' of the EMF of one full turn.
No, the voltmeter measures the EMF of the outside loop--the far side of the ring plus its own test leads, less the voltage drop across the far resistor. 

Isn't that what I said about the other loop? You measure the correct voltage along the far resistor branch with 'an error' of one turn EMF. And this value is exactly equal to the correct voltage along the near resistor branch. Didn't anybody read the links I've posted??? They linked pages full of pictures.

Quote
This is where the second Lewin 'explanation' diagram was confusing, because it showed three loops and three EMF circles in each one.

Could it be those were the mesh currents?

Title: Re: #562 – Electroboom!
Post by: bsfeechannel on November 16, 2021, 03:51:19 am
His whole demonstration is designed to provoke rather than explain

Oh yes, he left it as homework for his students to solve. Those who wrote in the report that KVL always holds or that he bad-probed got an F.

Quote
I haven't watched, but I suppose he didn't clear anything up?

Mehdi said once he lives off Youtube, so he must do anything to generate views. His commitment is with his viewership. I can't blame him for that. That's what Youtube is all about. But I think that in the case of his diatribe with Lewin, he really jumped the shark.
Title: Re: #562 – Electroboom!
Post by: bdunham7 on November 16, 2021, 04:02:29 am
But I think that in the case of his diatribe with Lewin, he really jumped the shark.

Yes, it's like "Ow My Balls!"* with some wires thrown in.  But Lewin isn't any less of a showman. 

* Idiocracy (2006).  If you have not seen this, you must.  It is painful but prophetic.
Title: Re: #562 – Electroboom!
Post by: Jesse Gordon on November 16, 2021, 05:19:22 am
Ok, let's set the record straight. This will be my last post on the topic of your disappearing comments and Bob's merry deletions, but I really need to set the record actually straight.

Thank you, thank you, thank you!

...
But you probably won't find many confutations in the comments section, because Duhamel, like fromjesse, delete the comments that tell them they are wrong and why. For example, in his last video "Voltage in a loop is weird", a post by Silicon Soup stating that by putting the probes inside the magnetic region a voltage was induced in the loop has disappeared.

Here's the reason:

https://i.postimg.cc/vZ6BM4Wn/screenshot-3.png
...
Just to set the record straight, your statement is false.
Neither I nor Duhamel delete comments just because they disagree with us.
I don't know whether Bob deleted any specific comments,

You literally contradicted yourself from one sentence to the other.
Sentence two says "Duhamel does not delete comments because..."
Sentence three says "I do not know whether Bob deleted any specific comments..."
Did you talk to Bob expressely about this? Sentence three says you didn't. Also says you seem fine about talking of what you do not know.


If you had quoted me in context, it would not falsely appear that I contradicted myself.

I have not talked to Bob about it, but your statement was a blanket statement that THERE WOULD BE SEEN NO CONFUTATIONS  in the comments on my channel or Bob's channel because you said (implicitly) we deleted all disagreeing comments.

Your statement was out right false, and observably false, because there are still dissenting comments on both his channel and mine.

Record straightened: I can recall at least three posts that disappeared from RSD A. videos. One is the comment from Silicon Soup, stating that by running the probes in the magnetic flux region he was intercepting flux and this inducing a voltage. It was deleted along with a timestamped comment by Copernico Felinis (or, by using your non-insulting name: felinus retardus). Another one is a comment of I believe Mick Vall (Or Mark Fruchtman, a name like that... M---- --a---) where he told Bob that he was wrong and he should not be teaching this wrong concepts because it would cause confusion in students (and he also explained why and where he was wrong). The whole thread disappeared, along with other comments discussing technicalities.

He may have removed some entire conversations, I don't know. But I do know that he didn't remove all disagreeing comments as you stated, and that is an observable fact.

So, keep your "false" to you.

If you had said that Bob had deleted SOME users, or SOME disagreeing comments, but left others, then I'd have had no problem with your claim.
If you had said that I had banned you but not others, then I'd have no problem.

And then there is that exchange with Mr. Anderson. It tells all you need to know.
I don't know what you're talking about.

Quote
but I don't recall deleting any specific comments, but what I did do is ban ONE particular person (felinus retardus) who was being insulting to my viewers as well as myself. I did not delete all of his comments,
But you did not publish some?
I don't remember not publishing any. Most of my videos have un-moderated comments, but sometimes youtube flags them and I have to go in and approve them.
If you left a comment after I banned you, then it may not have been published and it probably didn't even show up in my unpublished comments.

I guess that, regarding the insults, you meant comments like this one  - in that case I can understand

(https://i.postimg.cc/N081Kppk/screenshot-fromjesse-insults.png)
Source: your channel - image here https://i.postimg.cc/N081Kppk/screenshot-fromjesse-insults.png (https://i.postimg.cc/N081Kppk/screenshot-fromjesse-insults.png)

Oh, my bad. That was Samuel Snerden cold open post and he was talking about Lewin. Did you ban him as well, for insulting your viewers? But of course not. Lewin is certainly not a viewer of yours.

Lewin has never left any comment. If he wants to come comment under my videos then he can complain about Samuel Snerden's comments.

Also about insults, and then I'll drop it definitively: I have been through the exchange you are having with ThinkFat. In just two or three comments you manage to call bsfeechannell: "clueless", "totally idiotic", "either he's fulla BS or he's not being honest", "your hero the ignorant bsfeechannel", "he's either ignorant or disingeneous".

Again, if you quoted me in context, you'd see why. The chap literally responded thus:

bsfeechannel responded to "A resistor under the influence of the external varying magnetic field also behaves as non-ideal voltage source" by saying  "But, but, but, but fromjesse said that the copper rings generate voltages, while the resistors drop it! How can I properly learn Ohms law, KVL, good probing and oscilloscope operation if you guys keep contradicting each other? Aw, unbelievable!"

Doesn't he know that resistors, capacitors, and inductors all have resistance, inductance, and capacitance? (excepting superconductors which have no resistance)

Apart from that thing about pots and kettles, I don't think your ban was motivated by insults. You seem pretty comfortable with insults in your channel. I believe it's something else. I wonder what that could be.  :-//

Well, you're the ONLY one I've banned on that topic. I banned a few pharmacy spammers but that's something else.
You can look and there are LOTS AND LOTS of comments of all the regulars like Trevor and Melo and I don't remember who else. They argued hard and long -- but didn't resort to insulting me, and I never banned them. You're the only one I banned.

Maybe when you are cornered, you get insecure?
Nope, look at all the other guys who were much more meticulous and thorough - they would be the ones that cornered me and caused insecurity. But you can see that the discussions go on for a LONG time, and they are still there, and those users are not banned.

Quote
Let's try to figure out the issue.

Will you start insulting people in here, as well?
If not, I can try to explain what is wrong with what you believe.
Namely:

1. That sentence of Belcher is about the RLC lumped circuit of section 10. Read pages 15 and 16. It's not about the unlumpable Lewin ring.


(https://i.postimg.cc/Yr9hvQtH/Screenshot-2021-11-15-at-20-13-02-Microsoft-Word-Faradays-Law-Mehdi-Dr-John-Belcher-on-Faradays-Law.png)

That's where I was quoting from. Wouldn't you say that he clearly describes that there are two different attributes, both of which use the unit volt, but one of which is always zero across an inductor and one which is what a volt meter reads, which is how KVL holds as argued by Mehdi?

He says "Thus with Feynman et al.’s definition, the sum of all the voltage differences around the circuit is zero (that is, KVL holds) "


2. The note of McDonald (it's not a paper, it's a note for his students) has been through many revisions, so you should also specify the date it was last updated. The most recent I have on my laptop says November 14, 2018. And he uses a definition of "voltage drop" that 'others' (namely the IEC) call 'scalar potential difference'. Therefore he is talking about the component of the voltage that is solely determined by the conservative electric field generated by the distribution of charge displaced by Eind. It's just that, and he says so on page 10, after formula (35). The scalar PD alone is insufficient to describe the physical system. In fact, you cannot even apply Ohm's law to that 'voltage drop' as he calls it. And he acknowledges that voltmeters do not measure that 'voltage drop' of his, but the path integral of the total electric field Etot (as Belcher says, as well).
Therefore, when stores will sell AC voltmeters that can read that scalar potential difference, that definition of his will come very handy. Till then, a lot of people prefer to use actual voltage, the one that can be measured by voltmeters and that is path-dependent. It is also much easier to treat.

Yes, I have the same November 14th 2018.

And yes, Dr. McDonald cleary states:

"Lewin’s circuit is within the range of applicability of Kirchhoff’s loop equations, which can be used to predict measurements by the “voltmeters” in the experiment."

What gives? You say Lewin's circuit is not within the range of applicability of Kirchhoff's loop equations. McDonald says it is.
He literally says it. In writing.

3. By reading your thread with ThinkFat for the video "The Lewin loop inside an iron core - KVL still holds" it appears to me you have problems with basic electromagnetism.

Naturally, but then again, to me, it looks like you don't understand basic electromagnetism. That's why I'm here trying to learn.

You say con can "correctly lump" the circuit, but Lewin's ring is not lumpable.

Huh? I never said Lewin's ring was not lumpable.
McDonald said it is within the range of applicability of Kirchhoff's loop equations.

Are you saying it's not lumpable, but it's still within the range of applicability of Kirchhoff's loop equations?

You also seem to think that if your "volt meter leads do not cut through any changing magnetic fields [...], according to Faraday's law [...] there will be no voltage induced along my volt meter leads".
Do you really think that in order to induce a voltage, the conductors need to pass through the variable magnetic field region?

I think that a wire running axially along the magnetic "lines of force" will have no voltage induced in it,  and no force exerted on it. That's why voice-coil actuators have the winding running at right angles to both the axis of movement and the axis of magnetic "lines of force."

Is that the reason you want to drill a hole into the toroidal core?
LOL Didn't you see the laughing face emoticons after that statement?

The point of the hole would be to allow measuring a half-turn. Look at it like this, here's a cross section at the toroid with the hole shown as a pipe:
(d|b)  Parenthesis=primary winding, db=core material, and |=wire through the core.

The current in the "(" half of the winding will be flowing UP, and the current in the ")" half of the winding will be flowing DOWN, which means that the voltage induced along the length of "|" will be zero, thus allowing us to measure the voltage of a half of a turn hahahahahaha.

And by the way, since the core hogs the vast majority of the magnetic flux due to it's high permeability, the magnetic flux in the drilled hole would be pretty small, unless operating beyond the saturation point of the now narrowed cross section of the core.

You also seem anxious about looking inside a transformer.
It was a half-turn-on-a-toroid joke, man.
It's because people were trying to get me to do the EI-Core experiment on a toroid knowing that I couldn't do a fractional turn.

But that is the crux of the problem: Lewin's ring has placed two resistor inside the coil of a single turn transformer, creating a circuit that cannot be model with a lumped component.

Why can't it be modeled as several lumped components? Didn't Dr. McDonald say it could be?

Faraday's law is clear that:

Voltage generated = -N * ((delta(B*A)) / delta-t)

where N=Turns
B=Magnetic Flux
A=Area

Sooo, if we have a circular transformer winding composed of two copper windings of nearly of half a turn, and two resistors which take up almost no turn, why can't it be modeled as two half turns and two resistors?

If you want to get really into the details, sure the resistor might be 0.5% of a turn, so would be modeled as 0.5% of a turn plus its resistance.
And yes, the copper traces have some resistance, so you would model them as a winding plus a couple milliohms.
And of course both the resistor and the copper trace have some parasitic capacitance as well, so if you want to be really accurate you can model that as well.

But for the sake of discussion, the stray attributes are very small compared to the primary attributes or induced voltage and ohmic voltage drop.

N need not bee an integer.

Continuously variable variacs have been around forever - I've got one, real pretty, all silver plated, you turn the knob, and the wiper slides or rolls along the winding providing a movable  tap at any of an infinite number of positions, limited only by the smoothness of the winding wire and the wiper roller.
Title: Re: #562 – Electroboom!
Post by: bdunham7 on November 16, 2021, 05:41:54 am
Continuously variable variacs have been around forever - I've got one, real pretty, all silver plated, you turn the knob, and the wiper slides or rolls along the winding providing a movable  tap at any of an infinite number of positions, limited only by the smoothness of the winding wire and the wiper roller.

Could you show a picture or a link?  I've never seen a variac like that.   I assume you mean a line-voltage variac?
Title: Re: #562 – Electroboom!
Post by: Jesse Gordon on November 16, 2021, 07:47:21 am
Dr. McDonald said "Lewin's circuit is within the range of applicability of Kirchhoff's loop equations, which can be used to predict measurements by the 'voltmeters' in the experiment. "

You said you have no degrees and you're no math wiz.

At least I admit it.

Probably you didn't read the entire paper by McDonald, or if you did, you didn't understand it.

I did read and did my best to understand it.

What he says is that, specifically for Lewin's circuit, it is possible to consider the EMF as a generator element of an equivalent circuit and still apply KVL EQUATIONS. Which is true.

Note, by the way, I don't think McDonald used the term "KVL Equations." I think he seems to be saying "Kirchhoff's  loop equations." That sounds to me like the equations which define KVL.

So we can apply Kirchoff's Loop equations just not KVL itself? Hmmmm. What's left beyond the equations?  What are the Kirchhoff's loop equations, if not the heart of KVL itself?

Isn't the Kirchhoff's loop equation Σ(V1,V2,...,Vn) = 0 ?

What Dr. McDonald actually said is "Lewin’s circuit is within the range of applicability of Kirchhoff’s loop equations, which can be used to predict measurements by the 'voltmeters' in the experiment."

So isn't he saying that we can use Kirchhoff's loop equations to predict the measurements by the volt meters?

But he stresses that this EMF is nowhere to be found with voltmeters in the circuit.

In Lewin’s example, the magnetic flux in the primary solenoid may well be within a small coil, but the secondary consists of only a single “turn”, so the associated inductive EMF is not well localized, but rather is distributed around the entire secondary loop. Then, since inductive EMF’s are associated with a vector potential, rather than a scalar potential, it can be misleading to interpret the inductive EMF as related to a “voltage”.

Wait a second. Where does he stress that the EMF is no where to be found with the voltmeters in the circuit? He specifically says that the EMF, which by Faradays law is -L(dI/dt), is read by the volt meter.

The part you are quoting simply says that the inductive EMF is not well localized, and that it can be misleading to interpret the inductive EMF as related to "Voltage."

He didn't say it IS misleading, but that it CAN be misleading.

Of course it can be misleading because if you are sloppy how you run your probes you may get an ambiguous reading, which is no reading at all. If you are measuring two  unknowns, you are measuring neither.

But it doesn't have to be misleading, according to him.

How did you get from "can be misleading" to "EMF is nowhere to be found with the voltmeters in the circuit?"

Lewin's beef is that people read that you can use KVL EQUATIONS to calculate the voltage across the resistors and try to find this EMF with voltmeters. You'll never find it.

But I do find that voltage with volt meters.


People say that the voltages are in the wires. McDonald denies that. It's all over the circuit, and it is not a voltage. So voltmeters won't measure it. (In other words, this circuit is "unlumpable", and modeling it so as to make KVL work is just a math trick).

I never said "the voltage was in the wires."

If it's unlumpable, then why did McDonald say that it was within the range of applicability or Kirchhoff's loop equations? Is not that equation Σ(V1,V2,...,Vn) = 0 ?

Regardless of the dB field distribution, at the end of the day, a transformer's wires emerge far from within the influence of the dB and run out to power loads, where they do have an unambiguous voltage which can be unambiguously measured with a volt meter and used to power energy consumers.

MacDonald says more.

Kirchhoff’s (extended) loop equation (1) does not apply to all possible circuits, and gives a poor description of circuits whose size is not small compared to relevant wavelengths, in which effects of radiation and retardation can be important. Examples such as Lewin’s in which the self inductance of the entire loop could be important must be treated with care.

So Kirchhoff's law doesn't always hold, does it?

I never said that there aren't cases where KVL fails, but.....

 :scared: Woah, is this what I've been overlooking?  :scared:

Were the wavelengths in Lewin's loop or in my shortwave transmitter (Lewin Clock) experiment small compared to the test apparatus?

 :palm: That's something... wow.. Is that what I've been overlooking this whole time?  :palm:

He calculates and confirm that the two voltmeters in Lewin's experiment will show two different voltages even though connected to the same points in the circuit and then declares:

These results were validated by experiment during Lewin’s lecture demonstration.

Of course, I myself verified that if I ran my volt meter leads through the core of the transformer I could get all sorts of errors too.

Quote
"In this sense, KVL holds, as argued by Mehdi Sadaghdar ..."

In what sense? Have you read the whole paper? He explicitly said that the voltage through an inductor is zero. Of course it is! It's just a piece of wire! But across the terminals of the inductor it is defined by its inductance (path dependence of voltage). And IN THIS SENSE, KVL holds. Which is true.

Wait, if the voltage across the winding is zero, and the voltage across the resistors is non-zero, then how the tarzan can the sum be zero?

It looks to me like he's saying that -L(dI/dt) does not represent -∫E.dl

(NOTE: I copy pasted integral and close loop/contour integral symbols. If you don't see one of the two before occurrences of "E.dl" then it didn't paste correctly.Let me know. Looks fine to me.)

It looks to me like he's saying that Feynman stated explicitly that -∫E.dl is through an inductor, but that the voltage difference across the inductor, which they correctly identify as EMF, is ∮E.dl, which by Faraday's Law is -L(dI/dt).

He goes on to state that this quantity  ∮E.dl or -L(dI/dt) has NOTHING to do with -∫E.d, even though the latter does have the unit volts.

In summary, what I take this to mean:

1:  -∫E.dl  through an inductor is zero for a super conductor.

2: ∮E.dl is the voltage difference across the inductor, and is equal to Faraday's law -L(dI/dt), and can be measured with a volt meter.

3: -∫E.dl has nothing to do with ∮E.dl

Do you agree that Dr. Belcher stated the above 3 numbered points?

It sure looks to me like he's saying that the ∮E.dl of the windings will have a voltage differential which, when algebraically summed with the voltages across the resistors, will equal zero, and that KVL thus holds true.

Quote
"In this sense, KVL holds, as argued by Mehdi Sadaghdar ..."
...
He explicitly said that the voltage through an inductor is zero.
...

No, he said that the -∫E.dl (which has the unit of volts) is zero, but that the ∮E.dl (which also has the unit of volts) is the volts that the volt meter reads.

Is this what the whole shebang is about? ∮E.dl vs -∫E.dl? Two different unrelated quantities, both of unit type volts?

Are you saying that KVL must only be used with -∫E.dl and not ∮E.dl?

And here we come to the Mehdi problem.

Mehdi claims that KVL ALWAYS holds. Which is not true.

"Always" as in are you saying that he claims that there are absolutely no circumstances that KVL fails? Got a link to him saying that?

But anyway, it doesn't really matter, I'm not Mehdi, and I don't claim that there is no possible situation where KVL will fail.

He claims that Belcher agrees with him. He doesn't.

Well, did Mehdi claim that Belcher agrees with him on every single point?

Belcher sort of did agree with Mehdi on some points, otherwise he wouldn't have said "In this sense, KVL holds, as argued by Mehdi..."

That is some level of agreement. Did Mehdi overstate that? Got a link?

But it doesn't really matter, and I'm not Mehdi. I'm not stating that Belcher did or did not COMPLETELY agree with EVERYTHING Mehdi said, I'm only stating that he did agree on the point that KVL holds, as argued by Mehdi.

Belcher says KVL only holds for specific conditions. He says that Lewin is wrong and invokes McDonald. MacDonald doesn't say Lewin is wrong anywhere in his paper.

Out of professional courtesy, he may not name Lewin and say the exact words "Lewin is wrong" however Lewin said Mehdi was wrong to claim that KVL holds, and Dr. Belcher says that Mehdi was right in arguing that KVL holds, so that sort of makes Lewin wrong according to Belcher - they can't both be right.

Furthermore, Lewin stated that the voltage across the wire was zero volts. Belcher says "No, -∫E.dl is zero. But the voltage that your scopes measure is measuring  ∮E.dl which is the voltage difference across the wire, which is not zero."

So while he was polite about it, he does seem to saying that Lewin was wrong, even though he doesn't name him like  Dr. McDonald did.

His argument is because he thinks that Lewin presented his circuit as a paradox that cannot be solved in the confines of Kirchhoff's equations. He shows it actually can. But in fact there's no paradox--that's Lewin's argument--when you realize the circuit is immersed in a non-conservative field, which is a much broader concept, that allows you to understand the problems to which McDonald says Kirchhoff's equations can't be applied.

Mehdi claims the voltages are in the wires, that Lewin doesn't know how to probe his circuit and many other irrational and nonfactual assertions. We just can't accept that.

Noooo of course we can't accept any of that, nooooo!

Except those of us who aren't we.  :-DD

But maybe all of that is above my skill level.

Let's start at a very simple place where my small brain gets it. Ok?

Let's say I have two small but powerful battery operated optically synchronized DDS waveform generators each generating a 60Hz 100mV AC RMS sinewave.
Further, let's say I have a 100 ohm resistor and a 1000 ohm resistor and I use these four components to form a series loop, alternating resistors and DDS units.

If I take a four channel fully isolated input scope and connect each of my four elements to their own scope input with positive-clockwise polarity, will the sum be zero? Let's say I turn on the math channel on the scope, and sum all the inputs, will it be a straight line?

KVL will hold fine in this case, right?
Title: Re: #562 – Electroboom!
Post by: thinkfat on November 16, 2021, 08:13:09 am
Welcome to our little "Understanding Faradays' Law" support group. Please have a seat.

...
PS: this would be a brilliant experiment: Use a split core with different permeability in each of the toroids, so that the flux is no longer evenly split. I wonder if the voltages would still add up ;)

Thank you for the invite here my friend!

And yes, the two toroids are slightly different from eachother. They actually really are two toroids in this particular transformer, each made from a looong strip of sheet metal wound up, and there are two of them in there, butted up.

And evidently, one has a little more iron to it because it has more voltage across it.

But regardless, the sum still adds up to "zero." (within the resolution limitation of my volt meter.)

However, if you would like, I would be glad to do an experiment literally with two different toroids which are of significantly different sizes, and do the same test to show you that even then, all the voltages add up to zero going clockwise around the loop.

Would that be meaningful to you? Do you think the voltage would no longer add up to zero?

The voltages would still add up, I give you that. My mistake, I wasn't thinking straight.

But doesn't it feel odd how suddenly half of the Lewin Loop in your experiment has a vastly different voltage "across" the same red wire than before? Because the Lewin Loop, and also the yellow reference loop still experience the same flux as before (of course presuming the total magnetic flux didn't change).

And I have another thought experiment for you: what if you replaced the two resistors with just wires, what would you measure around the loop?
I can actually predict it: the voltage across the two wires replacing the resistors would read "zero", while across the other two, the red wires of your original experiment, you would still read roughly 100mV.

That is a little hard to explain with your claim that none of your probing wires "cross" any magnetic flux, right?
Title: Re: #562 – Electroboom!
Post by: Jesse Gordon on November 16, 2021, 09:35:57 am
Welcome to our little "Understanding Faradays' Law" support group. Please have a seat.

...
PS: this would be a brilliant experiment: Use a split core with different permeability in each of the toroids, so that the flux is no longer evenly split. I wonder if the voltages would still add up ;)

Thank you for the invite here my friend!

And yes, the two toroids are slightly different from eachother. They actually really are two toroids in this particular transformer, each made from a looong strip of sheet metal wound up, and there are two of them in there, butted up.

And evidently, one has a little more iron to it because it has more voltage across it.

But regardless, the sum still adds up to "zero." (within the resolution limitation of my volt meter.)

However, if you would like, I would be glad to do an experiment literally with two different toroids which are of significantly different sizes, and do the same test to show you that even then, all the voltages add up to zero going clockwise around the loop.

Would that be meaningful to you? Do you think the voltage would no longer add up to zero?

The voltages would still add up, I give you that. My mistake, I wasn't thinking straight.


That's what I'm struggling with. I have seen this forum full of small mistakes like that which tells me I ought to not just accept statements seen here without understanding them.


But doesn't it feel odd how suddenly half of the Lewin Loop in your experiment has a vastly different voltage "across" the same red wire than before? Because the Lewin Loop, and also the yellow reference loop still experience the same flux as before (of course presuming the total magnetic flux didn't change).

I'm not sure what you're asking here, please explain your setup in more detail if I misunderstand.

ASSUMING that you're talking about the "KVL holds in iron core transformer" video except with two toroids of vastly different permeability, and you're asking whether it "feels odd" how one of the red wires has less voltage on it, NO, Not at all:

Like I keep yammering on about, model reality! model reality!

The two toroids having different permeability (given the same number of primary windings) will have different inductive reactance, which is like AC-resistance. So just like two resistors of different values in series, the two toroids will have different volts/turn when unloaded or not significantly loaded.

The two toroid's primaries are electrically in series, same current flowing through both of them, and the one with more permeability will have a higher inductive reactance and more inductive voltage drop which means more unloaded volts per turn.

I keep saying unloaded because if you put a secondary winding and a load on the larger inductor, then the primary current will increase and the primary voltage on the bigger one will drop and the primary voltage on the second one will rise.

It doesn't feel weird at all to me.

Does it feel weird to you?

And I have another thought experiment for you: what if you replaced the two resistors with just wires, what would you measure around the loop?
I can actually predict it: the voltage across the two wires replacing the resistors would read "zero", while across the other two, the red wires of your original experiment, you would still read roughly 100mV.

Instead of using my little red wires, I used a single length of 14AWG  in a loop, and soldered the ends together with 3/4" overlap so I wouldn't have any contact resistance issues to worry about, like this: https://postimg.cc/nXD7ynQf

I cannot run it for very long because the wire starts getting hot.

However:
Measuring across where the red wires used to be gives about +50mv.
Measuring across where the resistors used to be gives about -50mv.

(NOTE: Signs are to indicate the phase! I was using an AC volt meter, but I also determined the phase.)

Measuring from two points exactly opposite on the copper ring gives about 0mv.

The voltage dropped on the exposed part of the wire is opposite phase as compared to the voltage developed  on the wire which passes through the core.

To determine whether the voltage drop across the place where the resistors were was resistive or inductive, I put on a twisted pair as you can see, and it read the same ~50mv that the volt meter leads do.

It looks like we basically have essentially four resistors in series, two passing through the core and 2 not passing through the core.

A total of 200mv is induced, and each of the four "resisters" drops 50mv.

There would be 100mv induced in each resistor which passes through the core, but due to the current, there is a 50mv ohmic voltage drop in the opposite direction.

And since Σ(50,50,-50,-50)=0 it looks like KVL still holds.

That is a little hard to explain with your claim that none of your probing wires "cross" any magnetic flux, right?

I don't see why. The two ends of the "resistor" that is inside the core stick out, and those are the output wires to my transformer. They have ∮E.dl voltage across the ends of the wire.
By definition, a closed path core is equivalent to an infinitely long solenoid, right? So it makes sense that if the solenoid wraps around the straight wire, the straight wire effectively does exactly one turn around the infinitely long solenoid.

For the sake of KVL, it seems to model that way just fine.
Title: Re: #562 – Electroboom!
Post by: thinkfat on November 16, 2021, 10:18:32 am
Instead of using my little red wires, I used a single length of 14AWG  in a loop, and soldered the ends together with 3/4" overlap so I wouldn't have any contact resistance issues to worry about, like this: https://postimg.cc/nXD7ynQf

I cannot run it for very long because the wire starts getting hot.

The wire getting hot means there's a significant contribution by Mr. Ohm and so we can no longer neglect him.

EDIT (sorry, I pushed the POST button to early)

Quote
However:
Measuring across where the red wires used to be gives about +50mv.
Measuring across where the resistors used to be gives about -50mv.

(NOTE: Signs are to indicate the phase! I was using an AC volt meter, but I also determined the phase.)

Measuring from two points exactly opposite on the copper ring gives about 0mv.



You get 50mV from the voltage drop due to the wire resistance, -100mV from the EMF in the volt meter loop, resulting in -50mV total.

You get 0mV across two opposite points on the loop, but only in one case, if your volt meter loop encloses a part of the core (linking the flux and having an induced EMF) and also the ohmic voltage drop exactly counters the EMF in the volt meter loop. But it will be path dependent, in any other case you will get a non-zero voltage reading.
Title: Re: #562 – Electroboom!
Post by: bsfeechannel on November 16, 2021, 01:33:18 pm
Yes, it's like "Ow My Balls!"* with some wires thrown in.  But Lewin isn't any less of a showman. 

That's where the similarities between the two end. Every reputable author says the same thing about trying to apply KVL to a circuit subjected to a non-conservative field: caveat emptor, if you try to find the inductive EMF in the circuit as a voltage you'll come a gutser. But, people shrug and don't pay attention. Lewin does the same the other authors do, only that he goes one step further and asks his students to explain WHY exactly you can't find the "missing" "voltage". Wouldn't expect less from an MIT-quality lecture.

Mehdi does differently. He says that this "voltage" is really somewhere in the circuit (sometimes in the wire, sometimes in the resistors) and through some kind of clever technique you will be able to find it. Those who believe his false claims set out to devise all kinds of pseudo-scientific and contradictory explanations and "experiments" to try to confirm the word of their idol.

But Mehdi does worse. He divides the community and incites people against each other, all in the name of a polemic with the only intent of increasing his viewership.

Fortunately, there are courageous people who are turning this into an opportunity to debunk this bunk and introduce people to the tried and tested concepts behind the electromagnetic phenomenon.

Title: Re: #562 – Electroboom!
Post by: Sredni on November 16, 2021, 08:35:18 pm
Quote
1. That sentence of Belcher is about the RLC lumped circuit of section 10. Read pages 15 and 16. It's not about the unlumpable Lewin ring.
That's where I was quoting from. Wouldn't you say that he clearly describes that there are two different attributes, both of which use the unit volt, but one of which is always zero across an inductor and one which is what a volt meter reads, which is how KVL holds as argued by Mehdi?
He says "Thus with Feynman et al.’s definition, the sum of all the voltage differences around the circuit is zero (that is, KVL holds) "

Ok, let me join the dots for you.

About that sentence by Belcher
You already posted the two relevant pages of Belcher's note. Let me show you that the partial sentence you keep repeating ("In this sense, KVL holds as argued by Mehdi Sagadhdar, but..." - oh, no! You stop right before the 'but') is not about the unlumpable Lewin's ring, but another, lumpable, circuit. You need to read carefully, lest you mistake similar words, like "many" with "any", or something like that.
The famous sentence appears in section 11 on page 16, but it is related to a discussion that began in section 10, on page 15.

Source: Belcher's note available on Mehdi's YT channel. Let's see what section 10 is about, first.

Quote
10. Another circuit
(https://i.postimg.cc/DZwxkkyJ/screenshot-3.png)
link to image: https://i.postimg.cc/DZwxkkyJ/screenshot-3.png (https://i.postimg.cc/DZwxkkyJ/screenshot-3.png)

The experiment above was not considered in the video, but I offer it up as an exercise for the reader. I have a “one-loop” inductor, with a capacitor and resistor, as shown, where I assume the self-magnetic field is only non-zero inside the loop. The battery is shown with the positive terminal down, which will result in a current flow counterclockwise around the circuit, giving a self-magnetic field out of the paper.

At the top of p. 15, he writes
(https://i.postimg.cc/vTphV47f/screenshot-4.png)
https://i.postimg.cc/vTphV47f/screenshot-4.png (https://i.postimg.cc/vTphV47f/screenshot-4.png)

Take note of the equation, You will see it repeated shortly. For the time being, let's look at the other picture:

(https://i.postimg.cc/ZqCLVPfp/screenshot-5.png)
https://i.postimg.cc/ZqCLVPfp/screenshot-5.png (https://i.postimg.cc/ZqCLVPfp/screenshot-5.png)

Oh, look: same circuit, same test points to which attach the probes and different voltages depending on whether the voltmeter is on the left or on the right of the loop. I wonder where I have ever seen that before.
Oh yes, Lewin's ring. There is a difference though: Lewin's ring is not lumpable, while this RLC circuit with lumped R, L and C component is lumpable.

How so? What you need to do to consider it lumped is to devise a circuit path that DOES NOT CONTAIN the changing magnetic flux region in its interior (technically, we should say that the closed circuit loop does not cut any net flux - but in 2D it's easier to say what I just wrote). If you can do that, then all voltages along any path you could imagine in the area enclosed by your circuit path, will be path independent (they will only depend on endpoints).
How can you find such a circuit path? Easy: instead of having the circuit path following the coil's filament, we consider the circuit path jumping in the space between the coil terminals. As explained by Feynman, shown by Hayt (posted an excerpt just a few pages above in this thread), detailed by Ramo Whinnery and VanDuzer, exemplified by Purcell, and Haus and Melcher, and Faria, and in the published papers by Romer, Roche, Nicholson, and on and on and on... now you can apply KVL to your freshly lumped circuit and be happy. It's fake, because voltage will still in general depend on path, but in our little bubble we can pretend it doesn't. In that sense, in that little safe space we just cut out from the rest of the real world, KVL holds.
Let me rephrase it: if you can hide the changing magnetic flux inside a lumped component, and you do not look inside it (meaning: your circuit path does not go inside those forbidden zone components and thus it is impossible to go around the dB/dt region) then the "amended" KVL holds.

Yet, whenever you consider circuit paths that go around the magnetic region, -pouff- the dream shatters. For example, if your circuit path consists of only one inductor and the jump at its terminal, you will see KVL die in the definition itself of voltage across the terminals of the lumped inductor. If you accept the definition of voltage as (minus) the path integral of the total electric field (such as that given by the IEC), the following paragraph says just that

(https://i.postimg.cc/5y3vxj7f/screenshot-6.png)
https://i.postimg.cc/5y3vxj7f/screenshot-6.png (https://i.postimg.cc/5y3vxj7f/screenshot-6.png)

i.e. you have a voltage -Ldi/dt across the terminals of the (self-)inductor, but zero voltage along the filament of its coil. Voltage referred to the same points A and B is different depending on the particular path you refer it to. Exactly what Lewin says in his video "Kirchhoff law is for the birds=https://youtu.be/LzT_YZ0xCFY (http://Kirchhoff law is for the birds=https://youtu.be/LzT_YZ0xCFY)", at minute 33:

Quote
"And now I'm going to cause you some sleepless nights. And I'm going to attach here a voltmeter... And now I'm going to ask you what would this voltmeter show. And you'll probably say: 'well didn't you say that the integral of E dot dl through this wire is zero?' So you will think that V here is zero. But that's not true. You know what V is going to be? It's going to be + L di/dt."

Oh, look! Belcher and Lewin are saying the very same thing.
Anyway, let's go back to Belcher's note and the infamous sentence KVL keep waving like a victory flag. To show that Belcher was talking about the lumped RLC circuit, here is the rest of page 15

(https://i.postimg.cc/dV77fWYs/screenshot-9.png)
https://i.postimg.cc/dV77fWYs/screenshot-9.png (https://i.postimg.cc/dV77fWYs/screenshot-9.png)

Do you recognize the equation +V - IR - Q/C - Ldi/dt = 0 ? It's the equation for the series lumped RLC circuit.
This part of Belcher's note is about the series lumped RLC circuit where L is the self inductance. NOT the unlumpable LEWIN's RING (where, incidentally but not importantly, the self inductance is negligible). It is about the different approach IN LUMPED CIRCUIT THEORY between properly applying Faraday (using a circuit path the goes through the filament and accounting for the magnetic flux intercepted by the path: 5 + 3 + 0 = 8 ) and using the 'amended version of KVL' (using a circuit path that jumps at the inductor terminals and pretending -L di/dt is a potential difference to save KVL: 5 + 3 - 8 = 0).

One can still save KVL for lumped circuits by bringing the rhs (the surface integral of B.dS) to the left side (and pretending it is a path integral of E.dl).
The trick works ONLY IF you can devise a circuit path that DOES NOT contain the variable magnetic flux region.

You can do it for the lumpable RLC circuit.
And this is the "amended", or "modified", or "extended" or "new" KVL that is so very often badly explained in high school and in first, second year introductory uni books (they use the 5 + 3 - 8 = 0 formalism that erroneously lead students to think there is a voltage buildup in the wires of a coil - Lewin says that "the physics stinks").

You CANNOT DO IT for Lewin's ring.
Why? Because it is required that the two resistor be on the opposite sides of the variable magnetic region. So, your circuit path is BY DEFINITION required to contain the magnetic flux region. The circuit is unlumpable.

(https://i.stack.imgur.com/HXv9x.jpg)
source: https://electronics.stackexchange.com/questions/551244/what-would-a-voltmeter-measure-if-you-had-an-electromotive-force-generated-by-a/551428#551428 (https://electronics.stackexchange.com/questions/551244/what-would-a-voltmeter-measure-if-you-had-an-electromotive-force-generated-by-a/551428#551428)

And as such, it cannot be properly modeled by lumped transformer models, without changing the true nature of the system (i.e. without introducing jumps in voltage whenever you encounter one of these lumped coils).

(https://i.postimg.cc/YCKNjzg2/Spot-the-differences.png)
source: https://i.postimg.cc/YCKNjzg2/Spot-the-differences.png (https://i.postimg.cc/YCKNjzg2/Spot-the-differences.png)

In order to model it with two or four lumped coils, the magnetic field region must be splitted to accomodate a circuit path that does not include any of it. I used the same 'stellated' path style used by Feynman in figure 22-9 at page 22-7 of his second volume of lectures.

Some ask: "don't you see the transformer?" Well, when you are inside the coil, when your circuit path go around the dB/dt coil you no longer have the luxury of using a lumped component model (the secondary of a transformer to which you can apply KVL) for the coil. You have an unlumpable circuit and you need to deal with it accordingly: ditch KVL and use Faraday.

I have a crystal ball that almost never fails me. It is now telling me that, despite being shown here that Belcher was not talking about Lewin's ring when he wrote that sentence, you will nevertheless keep use that very sentence in the future to show that Belcher agrees with Mehdi on Lewin being wrong on the Lewin ring. Just like Mehdi pretendend both Belcher and Feynman agrees with him on all the line, including Lewin's ring. By using your terminology: he is either "clueless", or "ignorant" or "not being honest". In any case: disappointing.

And I have not even mentioned the other note written by Belcher many years ago with Lewin, where they discuss the RL circuit and the "Kirchhoff second law modified for inductors", saying (regarding the 5 + 3 - 8 = 0 formalism)

(https://i.postimg.cc/zGMhDsjH/screenshot-7.png)
source: Lewin and Belcher note for 802.11, updated by Lewin to add pointer to Giancoli and to his lecture.
snippet url: https://i.postimg.cc/zGMhDsjH/screenshot-7.png (https://i.postimg.cc/zGMhDsjH/screenshot-7.png)

which also contains the treatment of the ring made of uniform resistive material and of two resistive halves in a variable magnetic field. Even there, you will see that Belcher's and Lewin's view are the same.

Next stop: a bit more detail on McDonald's note.
(but first, a few fun posts )





Title: Re: #562 – Electroboom!
Post by: Jesse Gordon on November 16, 2021, 08:59:10 pm
Instead of using my little red wires, I used a single length of 14AWG  in a loop, and soldered the ends together with 3/4" overlap so I wouldn't have any contact resistance issues to worry about, like this: https://postimg.cc/nXD7ynQf

I cannot run it for very long because the wire starts getting hot.

The wire getting hot means there's a significant contribution by Mr. Ohm and so we can no longer neglect him.

EDIT (sorry, I pushed the POST button to early)

haha yeah, I saw your initial post and was tempted to reply "I love the way you drive home your point... But what is it? hahaha" but I figured you weren't done so I'd let you finish  :-DD

Quote
However:
Measuring across where the red wires used to be gives about +50mv.
Measuring across where the resistors used to be gives about -50mv.

(NOTE: Signs are to indicate the phase! I was using an AC volt meter, but I also determined the phase.)

Measuring from two points exactly opposite on the copper ring gives about 0mv.



You get 50mV from the voltage drop due to the wire resistance, -100mV from the EMF in the volt meter loop, resulting in -50mV total.

You get 0mV across two opposite points on the loop, but only in one case, if your volt meter loop encloses a part of the core (linking the flux and having an induced EMF) and also the ohmic voltage drop exactly counters the EMF in the volt meter loop. But it will be path dependent, in any other case you will get a non-zero voltage reading.

But your prediction was unambiguously wrong. Remember, you predicted that there would be 0v across the wires replacing the resistors, and about 100mv measured 'around the ends' where the red wires used to be.

What gives? Your odds aren't very good here. First you suggested that maybe vastly different sizes of toroids in my EI-Core configuration might not sum to zero in the "KVL in an iron core" setup. You were wrong, sure, everyone makes a mistake.

(Ha! I guess technically, my EI core is actually two rectangular toroids butted up, so it's more of an OO core configuration hahahahahaha. Irrelevant though.)

Now you've made yet another prediction, you said I'd get around 0,100,0,100 mv on my shorted winding. I got around 50,50,50,50 which is not even arguably close to what you predicted.

It'd be one thing if I got like, oh, say 10mv on the outside-the-core sections and around 90mv on the through-the-core section. You could have argued you were right. But you weren't even close.

You also asked if it "feels odd" to me that the unloaded volts/turn might be vastly different for two series toroids with same number of primary turns but vastly different permeabilities.
I answered, but I also ask if it feels odd to you.

So, does it feel odd to you? Or did it?

If it didn't feel odd to you, then why'd you ask? Just hoping I wouldn't know the answer?

If it did feel odd to you, then that's a third prediction you "made," or rather suggested, which was also wrong.

Also, above you say that "But it will be path dependent." What will be path dependent? What path? The path of my volt meter leads? Or the path of the shorted winding?

By the way, have you read Dr. Belchers writup on the subject yet?

He goes over the idea of an unloaded transformer having voltage.

He basically cites Faraday's law and says that if you want to reassure yourself that the electric field actually exists, just increase the output voltage of the transformer to where it jumps an arc. He says you will see a spark and that PROVES (his word, my emphasis) that there is an electric field there, whether you put a volt meter onto the circuit or not.

Others here have likewise used similar electroscope/statically deflected oscilloscopes as thought experiments to show that the EMF is still there even if no current is flowing as a result of it.

In fact, I think maybe this is what's getting everyone in knots:

Dr. Belcher writes:

Quote
(https://i.postimg.cc/jdTDgf6D/Screenshot-2021-11-16-at-12-06-04-Microsoft-Word-Faradays-Law-Mehdi-Dr-John-Belcher-on-Faradays-Law.png)

At 4:31 into the video, Mehdi draws the open circuit to the left above.  There is no current flowing
because the curcuit is open.  Let’s discuss this situation before we put the voltmeter into the circuit. 
There will be zero electric field in the wires, because in the wires the induced electric field exactly
cancels the coulomb electric field, as we saw before.

And everybody stops reading there and declares that a transformer does not have a voltage output.

But Dr. Belcher continues immediately following the above quote:

Quote
But we will still see a charge accumulation on the ends of the wire, as shown in the schematic to the right above.
The upper end of the wire will be charged negatively and the lower end of the wire will be charged positively.
Moreover we can calculate the potential difference across the gap by using Faraday’s Law (which applies for
any open surface and its bounding contour, where or not there are any wires around.  Going around the circle
counterclockwise gives us (l  is the height of the gap)

(https://i.postimg.cc/QxFckh0C/Screenshot-2021-11-16-at-11-59-16-Microsoft-Word-Faradays-Law-Mehdi-Dr-John-Belcher-on-Faradays-Law.png)

If you want to reassure yourself that this electric field actually exists, make (https://i.postimg.cc/T1LhmYfX/Screenshot-2021-11-16-at-12-00-09-Microsoft-Word-Faradays-Law-Mehdi-Dr-John-Belcher-on-Faradays-Law.png) larger than the
breakdown voltage in air, about a million volts per meter, and you will see a spark across the gap that
proves that there is an electric field there, whether you put a voltmeter into the circuit or not.


So there. A transformer secondary puts out a voltage. KVL depends on guess what, VOLTAGE. (Word Kirchhoff's, emphasis mine.)

So why the tarzan can't Kirchhoff's VOLTAGE Law be applied to a transformer secondary as a lumped element?

So what is this whole debacle about? Is it about whether a transformer output has a voltage because "There will be zero electric field in the wires, because in the wires the induced electric field exactly cancels the coulomb electric field?"

Look at what Dr. Belcher says:

Quote

Thus with Feynman et al.’s definition, the sum of all the voltage differences around the circuit is zero (that is, KVL holds)
+V - IR - Q/C - L(dI/dt) = 0, but the first three terms here are the -∫E.dl through the various circuit
elements, and the last term has nothing to do with the -∫E.dl through the inductor, which is zero.

Do you see that last term, which Belcher says has NOTHING (his word, my emphasis) to do with the first 3 terms?

Do you see that last term? That's Faraday's law.

I think you're ignoring Faraday's law, which is, according to Belcher, part of the deal here.

Of course if you ignore Faraday's law, then you're going to say that there's no voltage on the output winding of a transformer, because Faradays law is the term which describes the induced voltage difference across the ends of the winding!

Am I all wrong? What gives? What do you take Belcher to be saying here?

Thank you.
Title: Re: #562 – Electroboom!
Post by: Jesse Gordon on November 16, 2021, 09:20:40 pm
Quote
1. That sentence of Belcher is about the RLC lumped circuit of section 10. Read pages 15 and 16. It's not about the unlumpable Lewin ring.

That's where I was quoting from. Wouldn't you say that he clearly describes that there are two different attributes, both of which use the unit volt, but one of which is always zero across an inductor and one which is what a volt meter reads, which is how KVL holds as argued by Mehdi?
He says "Thus with Feynman et al.’s definition, the sum of all the voltage differences around the circuit is zero (that is, KVL holds) "


Ok, let me join the dots for you.

About that sentence by Belcher
You already posted the two relevant pages of Belcher's note. Let me show you that the partial sentence you keep repeating ("In this sense, KVL holds as argued by Mehdi Sagadhdar, but..." - oh, no! You stop right before the 'but') is not about the unlumpable Lewin's ring, but another, lumpable, circuit. You need to read carefully, lest you mistake similar words, like "many" with "any", or something like that.
The famous sentence appears in section 11 on page 16, but it is related to a discussion that began in section 10, on page 15.

Source: Belcher's note available on Mehdi's YT channel. Let's see what section 10 is about, first.

Quote
10. Another circuit
([url]https://i.postimg.cc/DZwxkkyJ/screenshot-3.png[/url])
link to image: [url]https://i.postimg.cc/DZwxkkyJ/screenshot-3.png[/url] ([url]https://i.postimg.cc/DZwxkkyJ/screenshot-3.png[/url])

The experiment above was not considered in the video, but I offer it up as an exercise for the reader. I have a “one-loop” inductor, with a capacitor and resistor, as shown, where I assume the self-magnetic field is only non-zero inside the loop. The battery is shown with the positive terminal down, which will result in a current flow counterclockwise around the circuit, giving a self-magnetic field out of the paper.


At the top of p. 15, he writes
([url]https://i.postimg.cc/vTphV47f/screenshot-4.png[/url])
[url]https://i.postimg.cc/vTphV47f/screenshot-4.png[/url] ([url]https://i.postimg.cc/vTphV47f/screenshot-4.png[/url])

Take note of the equation, You will see it repeated shortly. For the time being, let's look at the other picture:

([url]https://i.postimg.cc/ZqCLVPfp/screenshot-5.png[/url])
[url]https://i.postimg.cc/ZqCLVPfp/screenshot-5.png[/url] ([url]https://i.postimg.cc/ZqCLVPfp/screenshot-5.png[/url])

Oh, look: same circuit, same test points to which attach the probes and different voltages depending on whether the voltmeter is on the left or on the right of the loop. I wonder where I have ever seen that before.
Oh yes, Lewin's ring. There is a difference though: Lewin's ring is not lumpable, while this RLC circuit with lumped R, L and C component is lumpable.

How so? What you need to do to consider it lumped is to devise a circuit path that DOES NOT CONTAIN the changing magnetic flux region in its interior (technically, we should say that the closed circuit loop does not cut any net flux - but in 2D it's easier to say what I just wrote). If you can do that, then all voltages along any path you could imagine in the area enclosed by your circuit path, will be path independent (they will only depend on endpoints).
How can you find such a circuit path? Easy: instead of having the circuit path following the coil's filament, we consider the circuit path jumping in the space between the coil terminals. As explained by Feynman, shown by Hayt (posted an excerpt just a few pages above in this thread), detailed by Ramo Whinnery and VanDuzer, exemplified by Purcell, and Haus and Melcher, and Faria, and in the published papers by Romer, Roche, Nicholson, and on and on and on... now you can apply KVL to your freshly lumped circuit and be happy. It's fake, because voltage will still in general depend on path, but in our little bubble we can pretend it doesn't. In that sense, in that little safe space we just cut out from the rest of the real world, KVL holds.
Let me rephrase it: if you can hide the changing magnetic flux inside a lumped component, and you do not look inside it (meaning: your circuit path does not go inside those forbidden zone components and thus it is impossible to go around the dB/dt region) then the "amended" KVL holds.

Yet, whenever you consider circuit paths that go around the magnetic region, -pouff- the dream shatters. For example, if your circuit path consists of only one inductor and the jump at its terminal, you will see KVL die in the definition itself of voltage across the terminals of the lumped inductor. If you accept the definition of voltage as (minus) the path integral of the total electric field (such as that given by the IEC), the following paragraph says just that

([url]https://i.postimg.cc/5y3vxj7f/screenshot-6.png[/url])
[url]https://i.postimg.cc/5y3vxj7f/screenshot-6.png[/url] ([url]https://i.postimg.cc/5y3vxj7f/screenshot-6.png[/url])

i.e. you have a voltage -Ldi/dt across the terminals of the (self-)inductor, but zero voltage along the filament of its coil. Voltage referred to the same points A and B is different depending on the particular path you refer it to. Exactly what Lewin says in his video "Kirchhoff law is for the birds=https://youtu.be/LzT_YZ0xCFY ([url]http://Kirchhoff[/url] law is for the birds=https://youtu.be/LzT_YZ0xCFY)", at minute 33:

Quote
"And now I'm going to cause you some sleepless nights. And I'm going to attach here a voltmeter... And now I'm going to ask you what would this voltmeter show. And you'll probably say: 'well didn't you say that the integral of E dot dl through this wire is zero?' So you will think that V here is zero. But that's not true. You know what V is going to be? It's going to be + L di/dt."


Oh, look! Belcher and Lewin are saying the very same thing.
Anyway, let's go back to Belcher's note and the infamous sentence KVL keep waving like a victory flag. To show that Belcher was talking about the lumped RLC circuit, here is the rest of page 15

([url]https://i.postimg.cc/dV77fWYs/screenshot-9.png[/url])
[url]https://i.postimg.cc/dV77fWYs/screenshot-9.png[/url] ([url]https://i.postimg.cc/dV77fWYs/screenshot-9.png[/url])

Do you recognize the equation +V - IR - Q/C - Ldi/dt = 0 ? It's the equation for the series lumped RLC circuit.
This part of Belcher's note is about the series lumped RLC circuit where L is the self inductance. NOT the unlumpable LEWIN's RING (where, incidentally but not importantly, the self inductance is negligible). It is about the different approach IN LUMPED CIRCUIT THEORY between properly applying Faraday (using a circuit path the goes through the filament and accounting for the magnetic flux intercepted by the path: 5 + 3 + 0 = 8 ) and using the 'amended version of KVL' (using a circuit path that jumps at the inductor terminals and pretending -L di/dt is a potential difference to save KVL: 5 + 3 - 8 = 0).

One can still save KVL for lumped circuits by bringing the rhs (the surface integral of B.dS) to the left side (and pretending it is a path integral of E.dl).
The trick works ONLY IF you can devise a circuit path that DOES NOT contain the variable magnetic flux region.

You can do it for the lumpable RLC circuit.
And this is the "amended", or "modified", or "extended" or "new" KVL that is so very often badly explained in high school and in first, second year introductory uni books (they use the 5 + 3 - 8 = 0 formalism that erroneously lead students to think there is a voltage buildup in the wires of a coil - Lewin says that "the physics stinks").

You CANNOT DO IT for Lewin's ring.
Why? Because it is required that the two resistor be on the opposite sides of the variable magnetic region. So, your circuit path is BY DEFINITION required to contain the magnetic flux region. The circuit is unlumpable.

([url]https://i.stack.imgur.com/HXv9x.jpg[/url])
source: [url]https://electronics.stackexchange.com/questions/551244/what-would-a-voltmeter-measure-if-you-had-an-electromotive-force-generated-by-a/551428#551428[/url] ([url]https://electronics.stackexchange.com/questions/551244/what-would-a-voltmeter-measure-if-you-had-an-electromotive-force-generated-by-a/551428#551428[/url])

And as such, it cannot be properly modeled by lumped transformer models, without changing the true nature of the system (i.e. without introducing jumps in voltage whenever you encounter one of these lumped coils).

([url]https://i.postimg.cc/YCKNjzg2/Spot-the-differences.png[/url])
source: [url]https://i.postimg.cc/YCKNjzg2/Spot-the-differences.png[/url] ([url]https://i.postimg.cc/YCKNjzg2/Spot-the-differences.png[/url])

In order to model it with two or four lumped coils, the magnetic field region must be splitted to accomodate a circuit path that does not include any of it. I used the same 'stellated' path style used by Feynman in figure 22-9 at page 22-7 of his second volume of lectures.

Some ask: "don't you see the transformer?" Well, when you are inside the coil, when your circuit path go around the dB/dt coil you no longer have the luxury of using a lumped component model (the secondary of a transformer to which you can apply KVL) for the coil. You have an unlumpable circuit and you need to deal with it accordingly: ditch KVL and use Faraday.

I have a crystal ball that almost never fails me. It is now telling me that, despite being shown here that Belcher was not talking about Lewin's ring when he wrote that sentence, you will nevertheless keep use that very sentence in the future to show that Belcher agrees with Mehdi on Lewin being wrong on the Lewin ring. Just like Mehdi pretendend both Belcher and Feynman agrees with him on all the line, including Lewin's ring. By using your terminology: he is either "clueless", or "ignorant" or "not being honest". In any case: disappointing.

And I have not even mentioned the other note written by Belcher many years ago with Lewin, where they discuss the RL circuit and the "Kirchhoff second law modified for inductors", saying (regarding the 5 + 3 - 8 = 0 formalism)

([url]https://i.postimg.cc/zGMhDsjH/screenshot-7.png[/url])
source: Lewin and Belcher note for 802.11, updated by Lewin to add pointer to Giancoli and to his lecture.
snippet url: [url]https://i.postimg.cc/zGMhDsjH/screenshot-7.png[/url] ([url]https://i.postimg.cc/zGMhDsjH/screenshot-7.png[/url])

which also contains the treatment of the ring made of uniform resistive material and of two resistive halves in a variable magnetic field. Even there, you will see that Belcher's and Lewin's view are the same.

Next stop: a bit more detail on McDonald's note.
(but first, a few fun posts )


Thank you! I have to run to work, I'll re-read this later.

But to clarify two things, are we in agreement then that in cases where an inductor or transformer output has it's dB/dt contained entirely within itself, it can be perfectly lumped as an element for the sake of KVL?

So I don't need to keep arguing with people about whether my KVL in an iron core transformer is valid? It's a transformer, the magnetic circuit of the core contains the dB/dt and the leads that come out of it can be considered output wires from a black box and can be considered a voltage source just like any other for the sake of KVL?

(Assuming, of course, the frequencies involved in the test are of a significantly larger wavelength than the apparatus itself.)

So many people have been arguing so many angles, it would help me a lot to just get that cleared up.

Also, if I understand correctly, you're saying that my Lewin Clock would be lumpable only at the resistors because that's the only place I can measure a voltage difference without running my leads around an area of dB/dt, and that the ∮E.dl or more specifically the Faraday Law Induced Voltage -L(dI/dt) across the windings is there and is a voltage but cannot be unambiguously measured due to the practical issues of dB/dt being enclosed in my volt meter lead path and inducing an error? Do we agree on that too?

Thanks!
Title: Re: #562 – Electroboom!
Post by: Sredni on November 16, 2021, 09:35:21 pm
But to clarify two things, are we in agreement then that in cases where an inductor or transformer output has it's dB/dt contained entirely within itself, it can be perfectly lumped as an element for the sake of KVL?

So I don't need to keep arguing with people about whether my KVL in an iron core transformer is valid?

Oh, for Stokes' sakes!
No, you can't use KVL on the circuit running around a core. You are putting your circuit INSIDE the transformer. How can you possibly think you can still consider the contribute of the cut flux as lumpable? The circuit with the two resistors that is your circuit, and identifies your circuit path is going around a nonzero net flux.

You have a fundamental misconception about Faraday's law. It's not your probe wires that need not to 'cut' throught the flux, it's the surface delimited by the closed curve formed by your probe wires (which are arcs, segments, do not have an area) AND the branch of circuit you are probing that does not have to cut any flux line.
EDIT: and of course, in the case of the circuit it's the circuit path itself that must not go around the flux region. Picture it this way: consider the circuit path as rigid iron wire, and then apply an elastic membrane. No net flux should go through that membrane.

Look at what you did with your EI transformer: if you had that membrane on your circuit path, it would be rammed by the central leg like there's no tomorrow.

Quote
Also, if I understand correctly, you're saying that my Lewin Clock would be lumpable only at the resistors because that's the only place I can measure a voltage difference without running my leads around an area of dB/dt,

The Lewin clock AS  A CIRCUIT is not lumpable.
You can apply KVL at the measurement loops that do not cut the field lines (I would not even say you "lumped the loop", I don't think it even has a meaning). In particular you can apply KVL to the measurement loops formed by: voltmeter on the left, its probes and the resistor on the left. And analogously to the right.
Buy you cannot apply it to the loop formed by left voltmeter, its probes and the right branch with the right resistor.
And you cannot apply it to the Lewin ring itself.
Title: Re: #562 – Electroboom!
Post by: Sredni on November 17, 2021, 12:57:00 am
Dr. Belcher writes:
Quote
(https://i.postimg.cc/jdTDgf6D/Screenshot-2021-11-16-at-12-06-04-Microsoft-Word-Faradays-Law-Mehdi-Dr-John-Belcher-on-Faradays-Law.png)
At 4:31 into the video, Mehdi draws the open circuit to the left above.  There is no current flowing
because the curcuit is open.  Let’s discuss this situation before we put the voltmeter into the circuit. 
There will be zero electric field in the wires, because in the wires the induced electric field exactly
cancels the coulomb electric field, as we saw before.
And everybody stops reading there and declares that a transformer does not have a voltage output.

I am sure that in the universe where Spock has a goatee everybody will say that.
In this universe, though, I have heard it say from one person only, and it's also on video. At 28:10 of "Kirchhoff's Voltage Law Fails, or does it - Who is Right" the one published on September 16, 2021

https://www.youtube.com/watch?v=LGdN7rDB3XE (https://www.youtube.com/watch?v=LGdN7rDB3XE)

(I need to specify because some three or four videos have been removed from its listing - and I wouldn't be surprised if he pulled the remaining ones...), this gentleman, immediately after having shown a clip of Lewin saying "what is the E field in the wire of the self-inductor? What do I hear? What do you say it is? It's zero! Exactly because there is no resistance",  shows a surprised face and says "regardless of the fact that I can put an oscilloscope there and measure - here is a picture of my own oscilloscope - that the voltage DOES NOT EXISTS because Ohm's law says it can't."

EDIT: to be clear, here Bob is asserting the Lewin says that the voltage across a coil is zero, because Ohm's law says it has to be zero. While anyone with an oscilloscope knows better that a MIT professor, because when they put their probes across the terminals they can see a wiggly line on a little screen. And all of this despite the fact that just a few minutes later in Lewin's video, Lewin explains that the voltage across the inductor is + L di/dt - while the integral of E.dl in the wire is zero (because if the resistance of the wire is zero, Ohm's law says the field has to be zero)

Well, guess what - neglecting the difference between saying "there is no voltage"/"there is no voltage output" and "voltage is zero", what you and Bob do not understand is that, since voltage can be multivalued, you can have both:

Voltage on the path that goes through the coil ('voltage along the coil') IS ZERO.
Voltage on the path that jumps between terminals ('voltage across the coil') is + L di/dt, a nonzero value whose waveform you can observe on the oscilloscope screen.

It is amazing how certain people cannot manage this thing.

"Can you believe it? I made this experiment to measure the energy per unit mass necessary to take this 1kg pellet from ground to the first floor. And I found that when I pushed the pellet along a waxed slide the energy per unit mass required was different from that for the slide with one inch of honey on its surface!!!"
"Wait, what??? You get different values for the energy per unit mass between the same starting and ending point??? That's impossible. You must have done something wrong!"
"I know, right? This is not logical. There must be something wrong with the measurement procedure..."
"Certainly a probing error. Let's see on YouTube if someone in a garage has an explanation."

Oh, dear...
Title: Re: #562 – Electroboom!
Post by: Jesse Gordon on November 17, 2021, 05:00:06 am
But to clarify two things, are we in agreement then that in cases where an inductor or transformer output has it's dB/dt contained entirely within itself, it can be perfectly lumped as an element for the sake of KVL?

So I don't need to keep arguing with people about whether my KVL in an iron core transformer is valid?

Oh, for Stokes' sakes!
No, you can't use KVL on the circuit running around a core.

My apologies, I may not have explained my question clearly.

Please allow me try again.

Let's say I have a toroid transformer, or an EI transformer. It's got a primary and two secondaries.

The two secondaries are individual separate isolated output windings.

Each secondary produces 100mV AC RMS 60hz.

Nothing fancy, no components laced through the core, no half turns, nothing fancy.

An ordinary step down mains powered 60hz transformer.

That's all, just an ordinary power transformer, two secondaries each producing 100mV AC RMS 60hz.

Can I consider these two 100mV AC RMS 60hz signals to be lumped element voltage sources coming from a self contained black box, and may I use them as lumped elements to form a loop with the two secondaries and two resistors all in series, and will KVL hold for this setup?

Thank you!
Title: Re: #562 – Electroboom!
Post by: Sredni on November 17, 2021, 05:23:15 am
Let's say I have a toroid transformer, or an EI transformer. It's got a primary and two secondaries.
The two secondaries are individual separate isolated output windings.
Each secondary produces 100mV AC RMS 60hz.

That's all, just an ordinary power transformer, two secondaries each producing 100mV AC RMS 60hz.

Can I consider these two 100mV AC RMS 60hz signals to be lumped element voltage sources coming from a self contained black box, and may I use them as lumped elements to form a loop with the two secondaries and two resistors all in series, and will KVL hold for this setup?

You mean two secondaries in series? Sure, it's the first figure in the bottom row. The one that says "two secondaries in series".
And if you want to use four secondaries, second picture in the bottom row. The one that says "four secondaries in series".

(https://i.postimg.cc/YCKNjzg2/Spot-the-differences.png)
https://i.postimg.cc/YCKNjzg2/Spot-the-differences.png

Maybe the image is too small and you didn't see it, in my previous post?
Title: Re: #562 – Electroboom!
Post by: Jesse Gordon on November 17, 2021, 05:59:36 am
Let's say I have a toroid transformer, or an EI transformer. It's got a primary and two secondaries.
The two secondaries are individual separate isolated output windings.
Each secondary produces 100mV AC RMS 60hz.

That's all, just an ordinary power transformer, two secondaries each producing 100mV AC RMS 60hz.

Can I consider these two 100mV AC RMS 60hz signals to be lumped element voltage sources coming from a self contained black box, and may I use them as lumped elements to form a loop with the two secondaries and two resistors all in series, and will KVL hold for this setup?

You mean two secondaries in series? Sure, it's the first figure in the bottom row. The one that says "two secondaries in series".
And if you want to use four secondaries, second picture in the bottom row. The one that says "four secondaries in series".

(https://i.postimg.cc/JnvnsgYG/Spot-the-differences.png)
https://i.postimg.cc/JnvnsgYG/Spot-the-differences.png

Maybe the image is too small and you didn't see it, in my previous post?

Thank you!
Title: Re: #562 – Electroboom!
Post by: Jesse Gordon on November 17, 2021, 06:29:30 am
...  But Lewin isn't any less of a showman.  ...

Professor Dr. Walter Lewin a showman? Naaaaah. Just no way. No.

But he will show you something so amazing that you will be telling your grandchildren about it!
In fact, it is probably the only time in your life that you will see a transformer!  :-DD
Title: Re: #562 – Electroboom!
Post by: Jesse Gordon on November 17, 2021, 06:34:27 am
Yes, it's like "Ow My Balls!"* with some wires thrown in.  But Lewin isn't any less of a showman. 

That's where the similarities between the two end. Every reputable author says the same thing about trying to apply KVL to a circuit subjected to a non-conservative field: caveat emptor, if you try to find the inductive EMF in the circuit as a voltage you'll come a gutser. But, people shrug and don't pay attention. Lewin does the same the other authors do, only that he goes one step further and asks his students to explain WHY exactly you can't find the "missing" "voltage". Wouldn't expect less from an MIT-quality lecture.

Mehdi does differently. He says that this "voltage" is really somewhere in the circuit (sometimes in the wire, sometimes in the resistors) and through some kind of clever technique you will be able to find it. Those who believe his false claims set out to devise all kinds of pseudo-scientific and contradictory explanations and "experiments" to try to confirm the word of their idol.

But Mehdi does worse. He divides the community and incites people against each other, all in the name of a polemic with the only intent of increasing his viewership.

Fortunately, there are courageous people who are turning this into an opportunity to debunk this bunk and introduce people to the tried and tested concepts behind the electromagnetic phenomenon.

Thank you for being one of those courageous people!

So to clarify, I'm trying to get some coherence on this one point:

If I have a regular EI or toroid transformer and it's got two secondary windings, I can use those two winding outputs as "two wire black box components" in a loop with resistors and stuff and they qualify as lumpable elements because the magnetic flux is (almost completely) entirely within the iron core, and thus, I can consider them lumped elements in a loop and apply KVL and KVL will hold, right?

Thank you!
Title: Re: #562 – Electroboom!
Post by: thinkfat on November 17, 2021, 07:50:54 am
Instead of using my little red wires, I used a single length of 14AWG  in a loop, and soldered the ends together with 3/4" overlap so I wouldn't have any contact resistance issues to worry about, like this: https://postimg.cc/nXD7ynQf

I cannot run it for very long because the wire starts getting hot.

The wire getting hot means there's a significant contribution by Mr. Ohm and so we can no longer neglect him.

EDIT (sorry, I pushed the POST button to early)

haha yeah, I saw your initial post and was tempted to reply "I love the way you drive home your point... But what is it? hahaha" but I figured you weren't done so I'd let you finish  :-DD

Quote
However:
Measuring across where the red wires used to be gives about +50mv.
Measuring across where the resistors used to be gives about -50mv.

(NOTE: Signs are to indicate the phase! I was using an AC volt meter, but I also determined the phase.)

Measuring from two points exactly opposite on the copper ring gives about 0mv.



You get 50mV from the voltage drop due to the wire resistance, -100mV from the EMF in the volt meter loop, resulting in -50mV total.

You get 0mV across two opposite points on the loop, but only in one case, if your volt meter loop encloses a part of the core (linking the flux and having an induced EMF) and also the ohmic voltage drop exactly counters the EMF in the volt meter loop. But it will be path dependent, in any other case you will get a non-zero voltage reading.

But your prediction was unambiguously wrong. Remember, you predicted that there would be 0v across the wires replacing the resistors, and about 100mv measured 'around the ends' where the red wires used to be.

My prediction was under the assumption that Mr. Ohm can be ignored. Obviously he wasn't agreeing with that. But that doesn't invalidate my point.

Quote
What gives? Your odds aren't very good here. First you suggested that maybe vastly different sizes of toroids in my EI-Core configuration might not sum to zero in the "KVL in an iron core" setup. You were wrong, sure, everyone makes a mistake.
The odds are entirely in my favor, though you are yet unable to see it.

Quote
Now you've made yet another prediction, you said I'd get around 0,100,0,100 mv on my shorted winding. I got around 50,50,50,50 which is not even arguably close to what you predicted.

And I explained why. Besides you got 50, -50, 50, -50 around the loop, mind the phase, and I also explained why. Anyway, you should have gotten a total of 200mV around the inner loop, according to Mr. Faraday, and you didn't, this should make you think. But apparently it doesn't. The reason you're getting 0V is because you're neglecting to account for parts of the EMF in your setup. Had you probed correctly, the total around the inner wire loop would have been the expected 200mV,

Quote
Also, above you say that "But it will be path dependent." What will be path dependent? What path? The path of my volt meter leads? Or the path of the shorted winding?

For the sake of your measurements, only the path of your voltmeter loops are relevant.

Quote
By the way, have you read Dr. Belchers writup on the subject yet?

He goes over the idea of an unloaded transformer having voltage.

He basically cites Faraday's law and says that if you want to reassure yourself that the electric field actually exists, just increase the output voltage of the transformer to where it jumps an arc. He says you will see a spark and that PROVES (his word, my emphasis) that there is an electric field there, whether you put a volt meter onto the circuit or not.


But the question was never if there's an electric field at the terminals of a transformer or not. You will always have an electric field at discontinuities around a loop.
I'll ignore some part of your quoting Dr. Belcher because what he writes there is undisputed.

Quote
So why the tarzan can't Kirchhoff's VOLTAGE Law be applied to a transformer secondary as a lumped element?

You can, but only as long the transformer can be completely regarded as a black box which is fully described by its datasheet values. However, in your experiment, you chose measurement paths that are inside the transformer and exposed to magnetic flux and thus this simplification is not valid.

Quote
So what is this whole debacle about? Is it about whether a transformer output has a voltage because "There will be zero electric field in the wires, because in the wires the induced electric field exactly cancels the coulomb electric field?"

Look at what Dr. Belcher says:

Quote

Thus with Feynman et al.’s definition, the sum of all the voltage differences around the circuit is zero (that is, KVL holds)
+V - IR - Q/C - L(dI/dt) = 0, but the first three terms here are the -∫E.dl through the various circuit
elements, and the last term has nothing to do with the -∫E.dl through the inductor, which is zero.

Do you see that last term, which Belcher says has NOTHING (his word, my emphasis) to do with the first 3 terms?

Do you see that last term? That's Faraday's law.

I think you're ignoring Faraday's law, which is, according to Belcher, part of the deal here.

Of course if you ignore Faraday's law, then you're going to say that there's no voltage on the output winding of a transformer, because Faradays law is the term which describes the induced voltage difference across the ends of the winding!

Am I all wrong? What gives? What do you take Belcher to be saying here?

Thank you.

Dr. Belcher says that if you want KVL to hold, you need to subtract the EMF in the loop. That's what I pointed out above: The EMF induced in your inner wire loop is 200mV, so if you sum up all the electric fields in the loop they should add up to the EMF. But in your case they added up to 0V, which is obviously wrong according to Dr. Belcher.

PS: I object to Dr. Belchers statement that the EMF is a voltage difference.
Title: Re: #562 – Electroboom!
Post by: Jesse Gordon on November 17, 2021, 09:44:48 am
Quote
1. That sentence of Belcher is about the RLC lumped circuit of section 10. Read pages 15 and 16. It's not about the unlumpable Lewin ring.
That's where I was quoting from. Wouldn't you say that he clearly describes that there are two different attributes, both of which use the unit volt, but one of which is always zero across an inductor and one which is what a volt meter reads, which is how KVL holds as argued by Mehdi?
He says "Thus with Feynman et al.’s definition, the sum of all the voltage differences around the circuit is zero (that is, KVL holds) "

Ok, let me join the dots for you.

About that sentence by Belcher
You already posted the two relevant pages of Belcher's note. Let me show you that the partial sentence you keep repeating ("In this sense, KVL holds as argued by Mehdi Sagadhdar, but..." - oh, no! You stop right before the 'but') is not about the unlumpable Lewin's ring, but another, lumpable, circuit.

The "but" doesn't have anything to do with it not being lumpable.

And I just re-read  Belcher's writeup, and I think you're wrong in saying that "In this sense, KVL holds as argued by Mehdi" was said about the RLC circuit that Belchar shows.

Specifically, Belcher goes over Lewin's loop specifically ( two resistors in a loop on opposite sides) then also goes over several specific loop configurations that Mehdi showed.

Then Belcher gives an RLC loop diagram that Mehdi did NOT use.

Belcher then goes onto a new Section,  "11 KVL" which makes one reference to the Non-Mehdi RLC loop above just to say that a common misconception holds that the -L(dI/dt) voltage read in the above RLC diagram represents -∫E.dl

In no other way does section "11 KVL" tie it'self to (or exclude itself from) any specific loop topology. In fact, it specifically says "As argued by Mehdi" which means section "11 KVL" CANNOT be about the RLC loop, because Mehdi didn't argue anything about that loop.


I stopped at the "but" because it just says that "one must always remember that the voltage difference across the inductor is defined in a very different way compared to the voltage difference across the other three elements."

It's still a voltage difference though.


I really don't see Belcher addressing the issue of probing or lumping at all.

It looks like he's just saying that even though a superconducting wire has no voltage drop due to -∫E.dl, it does have voltage drop due to ∮E.dl or Faradays law, and thus, KVL holds, that is to say, once the ∮E.dl is taken into account, then yes, the sum  of the voltage differences around the circuit is zero, and that KVL holds.

He says nothing about whether it is testable in the real world, but he says it holds.

How did you get from "but" to believing that Becher is saying that it Lewin's loop is unlumpable? Or that his section "11 KVL" does not apply to Lewin's loop?

After all, the entire basis for his conclusion in "11 KVL" is that ∮E.dl does give the voltage across the wires in order for all the voltage differences around the loop to sum to zero: Just because Lewin's loop may be difficult or impossible to measure in the real world doesn't mean that ∮E.dl  suddenly vanishes.

The famous sentence appears in section 11 on page 16, but it is related to a discussion that began in section 10, on page 15.

Source: Belcher's note available on Mehdi's YT channel. Let's see what section 10 is about, first.

Quote
10. Another circuit
(https://i.postimg.cc/DZwxkkyJ/screenshot-3.png)
link to image: https://i.postimg.cc/DZwxkkyJ/screenshot-3.png (https://i.postimg.cc/DZwxkkyJ/screenshot-3.png)

The experiment above was not considered in the video, but I offer it up as an exercise for the reader. I have a “one-loop” inductor, with a capacitor and resistor, as shown, where I assume the self-magnetic field is only non-zero inside the loop. The battery is shown with the positive terminal down, which will result in a current flow counterclockwise around the circuit, giving a self-magnetic field out of the paper.

At the top of p. 15, he writes
(https://i.postimg.cc/vTphV47f/screenshot-4.png)
https://i.postimg.cc/vTphV47f/screenshot-4.png (https://i.postimg.cc/vTphV47f/screenshot-4.png)

Take note of the equation, You will see it repeated shortly. For the time being, let's look at the other picture:

(https://i.postimg.cc/ZqCLVPfp/screenshot-5.png)
https://i.postimg.cc/ZqCLVPfp/screenshot-5.png (https://i.postimg.cc/ZqCLVPfp/screenshot-5.png)

Oh, look: same circuit, same test points to which attach the probes and different voltages depending on whether the voltmeter is on the left or on the right of the loop. I wonder where I have ever seen that before.
Oh yes, Lewin's ring. There is a difference though: Lewin's ring is not lumpable, while this RLC circuit with lumped R, L and C component is lumpable.

How so? What you need to do to consider it lumped is to devise a circuit path that DOES NOT CONTAIN the changing magnetic flux region in its interior (technically, we should say that the closed circuit loop does not cut any net flux - but in 2D it's easier to say what I just wrote). If you can do that, then all voltages along any path you could imagine in the area enclosed by your circuit path, will be path independent (they will only depend on endpoints).
How can you find such a circuit path? Easy: instead of having the circuit path following the coil's filament, we consider the circuit path jumping in the space between the coil terminals. As explained by Feynman, shown by Hayt (posted an excerpt just a few pages above in this thread), detailed by Ramo Whinnery and VanDuzer, exemplified by Purcell, and Haus and Melcher, and Faria, and in the published papers by Romer, Roche, Nicholson, and on and on and on... now you can apply KVL to your freshly lumped circuit and be happy. It's fake, because voltage will still in general depend on path, but in our little bubble we can pretend it doesn't. In that sense, in that little safe space we just cut out from the rest of the real world, KVL holds.
Let me rephrase it: if you can hide the changing magnetic flux inside a lumped component, and you do not look inside it (meaning: your circuit path does not go inside those forbidden zone components and thus it is impossible to go around the dB/dt region) then the "amended" KVL holds.

Yet, whenever you consider circuit paths that go around the magnetic region, -pouff- the dream shatters. For example, if your circuit path consists of only one inductor and the jump at its terminal, you will see KVL die in the definition itself of voltage across the terminals of the lumped inductor. If you accept the definition of voltage as (minus) the path integral of the total electric field (such as that given by the IEC),

Are you saying that the IEC defines volts in such a way that transformers do not have an output voltage?

That's silly.

Why do you bring up the IEC? They know better than Belcher?

Poor old Faraday, eh.

All the abuse he gets, good thing he made himself a shield.

the following paragraph says just that

(https://i.postimg.cc/5y3vxj7f/screenshot-6.png)
https://i.postimg.cc/5y3vxj7f/screenshot-6.png (https://i.postimg.cc/5y3vxj7f/screenshot-6.png)

i.e. you have a voltage -Ldi/dt across the terminals of the (self-)inductor, but zero voltage along the filament of its coil. Voltage referred to the same points A and B is different depending on the particular path you refer it to. Exactly what Lewin says in his video "Kirchhoff law is for the birds=https://youtu.be/LzT_YZ0xCFY (http://Kirchhoff law is for the birds=https://youtu.be/LzT_YZ0xCFY)", at minute 33:

Quote
"And now I'm going to cause you some sleepless nights. And I'm going to attach here a voltmeter... And now I'm going to ask you what would this voltmeter show. And you'll probably say: 'well didn't you say that the integral of E dot dl through this wire is zero?' So you will think that V here is zero. But that's not true. You know what V is going to be? It's going to be + L di/dt."

Oh, look! Belcher and Lewin are saying the very same thing.
Anyway, let's go back to Belcher's note and the infamous sentence KVL keep waving like a victory flag. To show that Belcher was talking about the lumped RLC circuit, here is the rest of page 15

(https://i.postimg.cc/dV77fWYs/screenshot-9.png)
https://i.postimg.cc/dV77fWYs/screenshot-9.png (https://i.postimg.cc/dV77fWYs/screenshot-9.png)

Do you recognize the equation +V - IR - Q/C - Ldi/dt = 0 ? It's the equation for the series lumped RLC circuit.
This part of Belcher's note is about the series lumped RLC circuit where L is the self inductance. NOT the unlumpable LEWIN's RING (where, incidentally but not importantly, the self inductance is negligible). It is about the different approach IN LUMPED CIRCUIT THEORY between properly applying Faraday (using a circuit path the goes through the filament and accounting for the magnetic flux intercepted by the path: 5 + 3 + 0 = 8 ) and using the 'amended version of KVL' (using a circuit path that jumps at the inductor terminals and pretending -L di/dt is a potential difference to save KVL: 5 + 3 - 8 = 0).

Belcher is explaining the common misconception, and showing that L(dI/dt) is COMPLETELY SEPARATE from -∫E.dl and that when Faraday's law is a term, then in fact the voltage difference across the inductor portions summed with the voltage difference across the non-inductor portions will be zero.

One can still save KVL for lumped circuits by bringing the rhs (the surface integral of B.dS) to the left side (and pretending it is a path integral of E.dl).
The trick works ONLY IF you can devise a circuit path that DOES NOT contain the variable magnetic flux region.

You can do it for the lumpable RLC circuit.
And this is the "amended", or "modified", or "extended" or "new" KVL that is so very often badly explained in high school and in first, second year introductory uni books (they use the 5 + 3 - 8 = 0 formalism that erroneously lead students to think there is a voltage buildup in the wires of a coil - Lewin says that "the physics stinks").

You CANNOT DO IT for Lewin's ring.
Why? Because it is required that the two resistor be on the opposite sides of the variable magnetic region. So, your circuit path is BY DEFINITION required to contain the magnetic flux region. The circuit is unlumpable.

(https://i.stack.imgur.com/HXv9x.jpg)
source: https://electronics.stackexchange.com/questions/551244/what-would-a-voltmeter-measure-if-you-had-an-electromotive-force-generated-by-a/551428#551428 (https://electronics.stackexchange.com/questions/551244/what-would-a-voltmeter-measure-if-you-had-an-electromotive-force-generated-by-a/551428#551428)

And as such, it cannot be properly modeled by lumped transformer models, without changing the true nature of the system (i.e. without introducing jumps in voltage whenever you encounter one of these lumped coils).

(https://i.postimg.cc/YCKNjzg2/Spot-the-differences.png)
source: https://i.postimg.cc/YCKNjzg2/Spot-the-differences.png (https://i.postimg.cc/YCKNjzg2/Spot-the-differences.png)

In order to model it with two or four lumped coils, the magnetic field region must be splitted to accomodate a circuit path that does not include any of it. I used the same 'stellated' path style used by Feynman in figure 22-9 at page 22-7 of his second volume of lectures.

Some ask: "don't you see the transformer?" Well, when you are inside the coil, when your circuit path go around the dB/dt coil you no longer have the luxury of using a lumped component model (the secondary of a transformer to which you can apply KVL) for the coil. You have an unlumpable circuit and you need to deal with it accordingly: ditch KVL and use Faraday.

I have a crystal ball that almost never fails me. It is now telling me that, despite being shown here that Belcher was not talking about Lewin's ring when he wrote that sentence, you will nevertheless keep use that very sentence in the future to show that Belcher agrees with Mehdi on Lewin being wrong on the Lewin ring. Just like Mehdi pretendend both Belcher and Feynman agrees with him on all the line, including Lewin's ring. By using your terminology: he is either "clueless", or "ignorant" or "not being honest". In any case: disappointing.

Yeah yeah yeah, you always say that.

I could just as easily say that my crystalball tells me that despite you being shown what Belcher said, you will continue to go on misrepresenting it blah blah blah.

Anyway, uhhh, it looks to me like Belcher is quoting Feynman, and in fact is representing Feynman's definition as +V - IR - Q/C - L (dI/dt) = 0.

If Belcher is talking about a loop topology that Mehdi didn't argue about, how then can Belcher say "As argued by Mehdi?"

What did Belcher mean? "As Mehdi would have argued had he argued with a loop with a capacitor?" Bleh.

If you back up and look at section 10, it looks to me like Belcher probably introduced the LRC loop topology specifically to match Feyman's definition.

The important take away though is that the fourth term, which is -L(dI/dt) (Faraday's Law) and IS included in all of Lewin's and Mehdi's loops.

I really don't see a strong argument to be made there. Just because Mehdi's and Lewin's loops may not contain a term for a capacitor, that doesn't mean that they don't contain the term for Faraday's Law.

The whole point of "11 KVL" is that due to -L(dI/dt), there is an induced voltage in the opposite polarity of the restively dropped voltage, and the sum of the two categories of voltage will be zero.

That will be true for any loop which has -L(dI/dt) and which has resistance.

Which all of the loops have.

And the rest of the point of "11 KVL" is that even though -∫E.dl must be zero for a superconductor, - L (dI/dt) is COMPLETELY separate from -∫E.dl, and - L (dI/dt) is NOT zero for a superconductor enclosing dB/dt.

And since Lewin's and Mehdi's loops all are (for the sake of discussion) superconductors enclosing dB/dt, the volts across the ends of the wires is non-zero due to the term for Faraday's law.

Belcher isn't even talking about lumbability.


And I have not even mentioned the other note written by Belcher many years ago with Lewin, where they discuss the RL circuit and the "Kirchhoff second law modified for inductors", saying (regarding the 5 + 3 - 8 = 0 formalism)

(https://i.postimg.cc/zGMhDsjH/screenshot-7.png)
source: Lewin and Belcher note for 802.11, updated by Lewin to add pointer to Giancoli and to his lecture.
snippet url: https://i.postimg.cc/zGMhDsjH/screenshot-7.png (https://i.postimg.cc/zGMhDsjH/screenshot-7.png)

LOL That reads like Lewin's writing, not like Dr. Belcher's.

which also contains the treatment of the ring made of uniform resistive material and of two resistive halves in a variable magnetic field. Even there, you will see that Belcher's and Lewin's view are the same.

Next stop: a bit more detail on McDonald's note.
(but first, a few fun posts )


In summary, in as few but succinct words as you can muster, what exactly are you trying to say about Belcher and Mehdi?

Mehdi DID NOT argue for KVL on any loop with capacitor in it. So how can that be the one that "Mehdi argued" for?

And what is your point with Feynman's definition of voltage? Did not Belcher point out that Faraday's law is what we're interested in, and that it is separate from -∫E.dl?
And did not Belcher show Faraday's Law in Lewin's and Mehdi's loops as well?

Does Belcher talk about lumpability anywhere?

Do you consider your arguments on Belcher's meaning to be weak, medium, or strong? (I guess "strong" would be if Belcher specifically stated what  argument of Mehdi's he was supporting.)

What argument of Mehdi's is Belcher supporting?

Oh yeah, and you were going to gaze deeply into that  crystalball of yours and tell me what McDonald "really meant" when he said that Lewin's loop was within the range of applicability to Kirchhoff's Voltage Equations.

My crystalball tells me that yours is going to mess up McDonalds words too :DD


Well, at least we agree that transformer windings have voltage, unless you're the IEC, and that as long as there is a way to measure the voltage across the wire without our measurement being tainted by stray magnetic fields, then KVL will be observably true.

And we agree that there are situations where probing can be difficult in areas of stray magneticfields.

We disagree about what Dr. Belcher and Dr. McDonald meant.
We may also disagree about our ability to eliminate said influences of stray magnetic fields.


Can we agree at this point that whether or not it can be measured in the real world, the L(dI/dt) voltage differential across the ends of the wire segments in Lewin's loop summed with the resistive losses in the same loop do add up to zero?

As a separate question, if yes to above:

Can we agree that in Lewin's loop, even if it's not possible to measure it, according to Feynman, the  sum of the voltage differences around the loop  is zero?
Title: Re: #562 – Electroboom!
Post by: Jesse Gordon on November 17, 2021, 10:15:28 am
Instead of using my little red wires, I used a single length of 14AWG  in a loop, and soldered the ends together with 3/4" overlap so I wouldn't have any contact resistance issues to worry about, like this: https://postimg.cc/nXD7ynQf

I cannot run it for very long because the wire starts getting hot.

The wire getting hot means there's a significant contribution by Mr. Ohm and so we can no longer neglect him.

EDIT (sorry, I pushed the POST button to early)

haha yeah, I saw your initial post and was tempted to reply "I love the way you drive home your point... But what is it? hahaha" but I figured you weren't done so I'd let you finish  :-DD

Quote
However:
Measuring across where the red wires used to be gives about +50mv.
Measuring across where the resistors used to be gives about -50mv.

(NOTE: Signs are to indicate the phase! I was using an AC volt meter, but I also determined the phase.)

Measuring from two points exactly opposite on the copper ring gives about 0mv.



You get 50mV from the voltage drop due to the wire resistance, -100mV from the EMF in the volt meter loop, resulting in -50mV total.

You get 0mV across two opposite points on the loop, but only in one case, if your volt meter loop encloses a part of the core (linking the flux and having an induced EMF) and also the ohmic voltage drop exactly counters the EMF in the volt meter loop. But it will be path dependent, in any other case you will get a non-zero voltage reading.

But your prediction was unambiguously wrong. Remember, you predicted that there would be 0v across the wires replacing the resistors, and about 100mv measured 'around the ends' where the red wires used to be.

My prediction was under the assumption that Mr. Ohm can be ignored. Obviously he wasn't agreeing with that. But that doesn't invalidate my point.

You made no point, except that I did not understand reality. You made a prediction of what reality was gonna be. But your prediction was wrong, and since your prediction being right was the force of your point, you have no point.

You simply did not understand reality, which is why your prediction was wrong.

Quote
What gives? Your odds aren't very good here. First you suggested that maybe vastly different sizes of toroids in my EI-Core configuration might not sum to zero in the "KVL in an iron core" setup. You were wrong, sure, everyone makes a mistake.
The odds are entirely in my favor, though you are yet unable to see it.
By "odds" I really meant that your batting average is pretty poor at this point.
So far it's 3 for 3 incorrectly predicting reality.
Quote
Now you've made yet another prediction, you said I'd get around 0,100,0,100 mv on my shorted winding. I got around 50,50,50,50 which is not even arguably close to what you predicted.

And I explained why. Besides you got 50, -50, 50, -50 around the loop, mind the phase, and I also explained why. Anyway, you should have gotten a total of 200mV around the inner loop, according to Mr. Faraday, and you didn't, this should make you think. But apparently it doesn't. The reason you're getting 0V is because you're neglecting to account for parts of the EMF in your setup. Had you probed correctly, the total around the inner wire loop would have been the expected 200mV,

I did get 200mv, I just didn't measure all of it.

Look at it this way:

200mv is induced due to Faraday's law, right?

And 200mv is lost due to resistance.  Right?

Let's say The 100mv and 100mv is induced where the wire passes through the transformer core.

But the resistive loss is evenly distributed around the copper ring, right?

And due to the geometry of the copper ring, my four measuring points were basically at quarters.

In other words, the purely resistive part outside the transformer was about 1/4 of the length and 1/4 of the length.

And the portion passing through the core was also about 1/4 and 1/4 of the length.

Thus, each quarter of the copper ring had about 50mv dropped. Ya?

Outside the core, I would just be measuring the ohmic drop.

But the quarter of the loop which passes through the core would be 100mv Faraday Law voltage minus 50mv ohmic drop, which gives 50.

So there IS 200mv generated, and 200mv lost, it's just not generated evenly around the loop, but it's lost evenly around the loop.

Right or wrong, you gotta admit it does add up, and it does seem to make sense.

Like I said before, resistors, capacitors, and inductors ALL have ALL THREE of inductance, capacitance, and resistance. (excepting of course superconductors which have no resistance.)
Normally the named attribute is the only major attribute (i.e. a resistor has very little capacitance or inductance) but in cases like we did with the copper wire loop, we have to model in the inductive and resistive components to our inductors. Model it as an inductor with a built in series resistor. That's exactly what it is.

Quote
Also, above you say that "But it will be path dependent." What will be path dependent? What path? The path of my volt meter leads? Or the path of the shorted winding?

For the sake of your measurements, only the path of your voltmeter loops are relevant.

Quote
By the way, have you read Dr. Belchers writup on the subject yet?

He goes over the idea of an unloaded transformer having voltage.

He basically cites Faraday's law and says that if you want to reassure yourself that the electric field actually exists, just increase the output voltage of the transformer to where it jumps an arc. He says you will see a spark and that PROVES (his word, my emphasis) that there is an electric field there, whether you put a volt meter onto the circuit or not.


But the question was never if there's an electric field at the terminals of a transformer or not. You will always have an electric field at discontinuities around a loop.
I'll ignore some part of your quoting Dr. Belcher because what he writes there is undisputed.

Quote
So why the tarzan can't Kirchhoff's VOLTAGE Law be applied to a transformer secondary as a lumped element?

You can, but only as long the transformer can be completely regarded as a black box which is fully described by its datasheet values. However, in your experiment, you chose measurement paths that are inside the transformer and exposed to magnetic flux and thus this simplification is not valid.


Are you talking about my EI-Core transformer with the two red secondary windings?

Because Sredni has said that an EI-Core transformer's secondary winding outputs qualify as black box because the dB/dt is contained (for all practical purposes) entirely inside the iron core.

You need to take that up with him if you think he's wrong.

But think about it. If you have a toroidal transformer, and you pass a wire once through the center to form a secondary, it will have EXACTLY a 1 turn secondary. It doesn't matter what else you do, that wire will still "enclose" the EXACT same one turn of dB/dt. You can take a magnetic field meter and prove that there is no dB/dt outside the toroid's surface.

Because there is no dB/dt outside the toroid, changing the path of the leads can neither add to nor subtract from the dB/dt being experienced by that one turn.

Like I said, please take up with Sredni.

btw, what about a transformer like this? https://postimg.cc/VdKPNnSB

It's a bobbin design, but the ends of the bobbin mushroom out and fold back to enclose the entire transformer, and the wires come out holes.

Would that make a good lumped element for a KVL loop?

Quote
So what is this whole debacle about? Is it about whether a transformer output has a voltage because "There will be zero electric field in the wires, because in the wires the induced electric field exactly cancels the coulomb electric field?"

Look at what Dr. Belcher says:

Quote

Thus with Feynman et al.’s definition, the sum of all the voltage differences around the circuit is zero (that is, KVL holds)
+V - IR - Q/C - L(dI/dt) = 0, but the first three terms here are the -∫E.dl through the various circuit
elements, and the last term has nothing to do with the -∫E.dl through the inductor, which is zero.

Do you see that last term, which Belcher says has NOTHING (his word, my emphasis) to do with the first 3 terms?

Do you see that last term? That's Faraday's law.

I think you're ignoring Faraday's law, which is, according to Belcher, part of the deal here.

Of course if you ignore Faraday's law, then you're going to say that there's no voltage on the output winding of a transformer, because Faradays law is the term which describes the induced voltage difference across the ends of the winding!

Am I all wrong? What gives? What do you take Belcher to be saying here?

Thank you.

Dr. Belcher says that if you want KVL to hold, you need to subtract the EMF in the loop. That's what I pointed out above: The EMF induced in your inner wire loop is 200mV, so if you sum up all the electric fields in the loop they should add up to the EMF. But in your case they added up to 0V, which is obviously wrong according to Dr. Belcher.

No, because again, like I keep saying MODEL REALITY!

Reality:

I have a loop consisting of 2 resistors and 2 inductors.
But the inductors have the same resistance as the 2 resistors.

Let's say for the sake of argument, that my loop resistance is 16 mohms.

The resistors have zero volts induced on them, and are 4mohm.
The inductors have 100mv induced in them, but are also 4mohm.

The total loop resistance is 16mohm, and the current is 12.5 amps.

So as you can see, the two inductors each have 100mv induced across them, but 50mv of that is lost due to resistive heating.

So again, if you model each inductor as a resistor+inductor lumped element, (WHICH, by the way, is what you always have to do in the real world when you need good accuracy because components always have some parasitic attributes) then it all adds up and KVL holds just fine.




Title: Re: #562 – Electroboom!
Post by: thinkfat on November 17, 2021, 11:08:47 am
Quote
Are you talking about my EI-Core transformer with the two red secondary windings?

Because Sredni has said that an EI-Core transformer's secondary winding outputs qualify as black box because the dB/dt is contained (for all practical purposes) entirely inside the iron core.

You're still completely disregarding that you have multiple loops in your EI-Core transformer experiment.

@Sredni isn't wrong, you just have troubles understanding what he said because you only hear what seems to suit your understanding. That's called "confirmation bias". That transfomer is a black box only at its terminals. But your measurements are not at its terminals.

Quote
Let's say The 100mv and 100mv is induced where the wire passes through the transformer core.
And there's your error, right there. This is complete BS.
Title: Re: #562 – Electroboom!
Post by: Sredni on November 17, 2021, 05:14:51 pm
About that sentence by Belcher
Let me show you that the partial sentence you keep repeating ("In this sense, KVL holds as argued by Mehdi Sagadhdar, but..." - oh, no! You stop right before the 'but') is not about the unlumpable Lewin's ring, but another, lumpable, circuit.
The "but" doesn't have anything to do with it not being lumpable.
And I just re-read  Belcher's writeup, and I think you're wrong in saying that "In this sense, KVL holds as argued by Mehdi" was said about the RLC circuit that Belchar shows.

See, I told you my crystal ball almost never fails me.
You really don't see it! Do you?
You read the sentence

Quote from: John Belcher
"In this sense, KVL holds as argued by Mehdi Sagadhdar, but one must always remember that the voltage difference across the inductor is defined in a very different way compared to the voltage difference across the other three elements."

and just before the but, you go on a celebratory trip and shut down. And yet, the circuit it is referred to is the lumpable RLC circuit whose equation is just above. Let's see what the but says:

Quote from: John Belcher
"...but one must always remember that the voltage difference across the inductor is defined in a very different way compared to the voltage difference across the other three elements."

And what could possibly be the other three elements? Well, the generator, the resistor and the capacitor, of course. He is talking about a lumpable circuit that, once lumped, allows for KVL to be applied. That sentence, it is one sentence, is not about Lewin's ring. (Also because in Lewin's ring the inductance L is neglible and one could also perform the experiment using a falling magnet, thus eliminating the secondary coil and the related mutual inductance as well - but let's forget about this).

Quote from: Jesse Gordon

If Belcher is talking about a loop topology that Mehdi didn't argue about, how then can Belcher say "As argued by Mehdi?" What did Belcher mean? "As Mehdi would have argued had he argued with a loop with a capacitor?" 
[...]
Mehdi DID NOT argue for KVL on any loop with capacitor in it. So how can that be the one that "Mehdi argued" for? 
[...]
What argument of Mehdi's is Belcher supporting? 

Dang, you really nailed me, here.
If only if could find a part in Mehdi's first video where he considers a lumped circuit and applies the 'modified KVL' to it, and hopefully says something along the lines of "KVL holds"...
Oh, let see...
At 8:21 in his first video on the topic, Mehdi draws his conclusions about a LUMPABLE circuit. This one:

(https://i.postimg.cc/zBSSpd8x/screenshot.png)
https://i.postimg.cc/zBSSpd8x/screenshot.png

and he says:

Quote from: Electroboom
"So you see that the voltage across both R1 and R2 is equal to the voltage across R1 plus the voltage across R2 which is the same voltage ACROSS the loop. "

Like you and Bob, he cannot understand that voltage, being path dependent in the presence of variable magnetic fields, can have different values depending (doh!) on the path and he is stuck with the terminology used in lumped circuit theory (where we consider the voltage across the terminals). In this case, since we are in the presence of a variable magnetic field, voltage depends on path and in particular: the voltage ACROSS the loop is Vr1+Vr2, while the voltage ALONG the loop is zero.
But the way he put resistors on one side makes this a lumpable circuit. In fact, it can be lumped by considering a circuit path that jumps at the terminals so that there is no variable magnetic field enclosed by said path. If we look at the circuit this way, the only voltage we need to consider in our limited safe space of lumped circuit theory is the voltage ACROSS the loop, which is not zero.

But Mehdi is clearly oblivious of all this, and keeps thinking that there can be only one value for the voltage between two points

Quote from: Electroboom
"This shows that the voltage ACROSS the loop is not zero, unlike what we've thought, but is equal to Vr1 plus Vr2. The loop is the secondary of a transformer, with the primary being my coil."

Well, no shit, Sherlock! That is a lumpable circuit and as such it can be modeled with lumped components. And sure enough, he produces a LUMPED circuit model:

(https://i.postimg.cc/285FFFJx/screenshot-2.png)
https://i.postimg.cc/285FFFJx/screenshot-2.png

A model that exhibit a massive jump in voltage when you encounter the lumped symbol of the inductor. Lumped circuit theory can only model the jump in voltage at the terminals because you are not allowed to look inside the transformer, BUT the voltage ALONG the loop is still zero (or, to be precise, the negligible ohmic loss in copper).

Let's hear it straight from the horse's mouth. I mean, from Mehdi, at 8:41:

Quote from: Electroboom
"Dr. Lewin's model misses a transformer in the loop, with the voltage across the winding equal to Vr1 plus Vr2. KVL HOLDS"

Well, we have now established that Mehdi talked about a LUMPED circuit in his first video (even though he does not realize that it IS NOT Lewin's ring, which is unlumpable exactly because the resistors are required to be on the opposite sides of the magnetic flux region). And that he applied KVL to said lumped circuit (which we can do, in the 'modified' form. Too bad it is not Lewin's ring). And he also says "KVL holds"!

In lumped circuits, such as this one (which is NOT Lewin's ring, let me repeat it) and the series RLC circuit considered by Belcher at page 15, we can make Faraday's law look like it's KVL. We just need to take the surface integral on the right, change its sign and put it in the lhs.
What have we done, by doing so? We started with a formula that says the circulation of E is NOT zero (when the circuit path follows the filament of the coil and thus INCLUDES the variable dB/dt region) and by cutting away the part of path the follows the coil and replacing it with the jump at its terminals we ended up with a formula that says that the circulation of E is ZERO (because the modified circuit path now skips - DOES NOT INCLUDE - the variable dB/dt) region.
In this modified circuit - THAT DOES NOT INCLUDE THE dB/dt REGION - KVL holds in the sense intended by Mehdi, but the voltage ACROSS the inductor is the result of the application of Faraday's law and the exclusion of its filament from the circuit's path.

And this is the key: in order to pull this trick (which I call the 'amended KVL') you need to be able to find a circuit path that DOES NOT INCLUDE THE VARIABLE MAGNETIC REGION AT ITS INTERIOR. You can do it when you put both resistors on the same side of the ring. You CANNOT do it for Lewin's ring (because Lewin's ring is Lewin's ring ONLY IF THE RESISTORS ARE ON THE OPPOSITE SIDES OF THE VARIABLE MAGNETIC FIELD REGION AND THE CIRCUIT PATH INCLUDES THAT REGION).
Now try to focus:

You cannot exclude the variable magnetic flux region from the circuit path of a circuit that requires its circuit path to contain the variable magnetic flux region. No matter how hard you try.

Geometry matters, exactly because in the case of Lewin's ring we are talking about a non lumpable circuit whose circuit path is required to circle a variable magnetic region.

P.S
Incidentally, the last sentence in Belcher's note is the following:

Quote from: John Belcher
"An excellent discussion of “electromotive force” (a terrible and misleading name) is given in the text by Griffiths, Introduction to Electrodynamics 4th Edition."

and guess what? Griffiths has the Romer-Lewin ring in one of his problems. Problem 7.50 uses the same diagram as Romer's paper. And in the solution he states... "Notice that V1 != V2, even though they are connected to the same points!"
Well, it really looks like the reference Belcher suggests has been written by another of Lewin's minions.


Title: Re: #562 – Electroboom!
Post by: Jesse Gordon on November 17, 2021, 07:30:07 pm
About that sentence by Belcher
Let me show you that the partial sentence you keep repeating ("In this sense, KVL holds as argued by Mehdi Sagadhdar, but..." - oh, no! You stop right before the 'but') is not about the unlumpable Lewin's ring, but another, lumpable, circuit.

The "but" doesn't have anything to do with it not being lumpable.
And I just re-read  Belcher's writeup, and I think you're wrong in saying that "In this sense, KVL holds as argued by Mehdi" was said about the RLC circuit that Belchar shows.


See, I told you my crystal ball almost never fails me.
You really don't see it! Do you?
You read the sentence

Quote from: John Belcher
"In this sense, KVL holds as argued by Mehdi Sagadhdar, but one must always remember that the voltage difference across the inductor is defined in a very different way compared to the voltage difference across the other three elements."


and just before the but, you go on a celebratory trip and shut down. And yet, the circuit it is referred to is the lumpable RLC circuit whose equation is just above. Let's see what the but says:

Quote from: John Belcher
"...but one must always remember that the voltage difference across the inductor is defined in a very different way compared to the voltage difference across the other three elements."


And what could possibly be the other three elements? Well, the generator, the resistor and the capacitor, of course. He is talking about a lumpable circuit that, once lumped, allows for KVL to be applied. That sentence, it is one sentence, is not about Lewin's ring. (Also because in Lewin's ring the inductance L is neglible and one could also perform the experiment using a falling magnet, thus eliminating the secondary coil and the related mutual inductance as well - but let's forget about this).

Quote from: Jesse Gordon

If Belcher is talking about a loop topology that Mehdi didn't argue about, how then can Belcher say "As argued by Mehdi?" What did Belcher mean? "As Mehdi would have argued had he argued with a loop with a capacitor?" 
[...]
Mehdi DID NOT argue for KVL on any loop with capacitor in it. So how can that be the one that "Mehdi argued" for? 
[...]
What argument of Mehdi's is Belcher supporting? 


Dang, you really nailed me, here.
If only if could find a part in Mehdi's first video where he considers a lumped circuit and applies the 'modified KVL' to it, and hopefully says something along the lines of "KVL holds"...
Oh, let see...
At 8:21 in his first video on the topic, Mehdi draws his conclusions about a LUMPABLE circuit. This one:

([url]https://i.postimg.cc/zBSSpd8x/screenshot.png[/url])
[url]https://i.postimg.cc/zBSSpd8x/screenshot.png[/url]

and he says:

Quote from: Electroboom
"So you see that the voltage across both R1 and R2 is equal to the voltage across R1 plus the voltage across R2 which is the same voltage ACROSS the loop. "


Like you and Bob, he cannot understand that voltage, being path dependent in the presence of variable magnetic fields, can have different values depending (doh!) on the path and he is stuck with the terminology used in lumped circuit theory (where we consider the voltage across the terminals). In this case, since we are in the presence of a variable magnetic field, voltage depends on path and in particular: the voltage ACROSS the loop is Vr1+Vr2, while the voltage ALONG the loop is zero.
But the way he put resistors on one side makes this a lumpable circuit. In fact, it can be lumped by considering a circuit path that jumps at the terminals so that there is no variable magnetic field enclosed by said path. If we look at the circuit this way, the only voltage we need to consider in our limited safe space of lumped circuit theory is the voltage ACROSS the loop, which is not zero.

But Mehdi is clearly oblivious of all this, and keeps thinking that there can be only one value for the voltage between two points

Quote from: Electroboom
"This shows that the voltage ACROSS the loop is not zero, unlike what we've thought, but is equal to Vr1 plus Vr2. The loop is the secondary of a transformer, with the primary being my coil."


Well, no shit, Sherlock! That is a lumpable circuit and as such it can be modeled with lumped components. And sure enough, he produces a LUMPED circuit model:

([url]https://i.postimg.cc/285FFFJx/screenshot-2.png[/url])
[url]https://i.postimg.cc/285FFFJx/screenshot-2.png[/url]

A model that exhibit a massive jump in voltage when you encounter the lumped symbol of the inductor. Lumped circuit theory can only model the jump in voltage at the terminals because you are not allowed to look inside the transformer, BUT the voltage ALONG the loop is still zero (or, to be precise, the negligible ohmic loss in copper).

Let's hear it straight from the horse's mouth. I mean, from Mehdi, at 8:41:

Quote from: Electroboom
"Dr. Lewin's model misses a transformer in the loop, with the voltage across the winding equal to Vr1 plus Vr2. KVL HOLDS"


Well, we have now established that Mehdi talked about a LUMPED circuit in his first video (even though he does not realize that it IS NOT Lewin's ring, which is unlumpable exactly because the resistors are required to be on the opposite sides of the magnetic flux region). And that he applied KVL to said lumped circuit (which we can do, in the 'modified' form. Too bad it is not Lewin's ring). And he also says "KVL holds"!

In lumped circuits, such as this one (which is NOT Lewin's ring, let me repeat it) and the series RLC circuit considered by Belcher at page 15, we can make Faraday's law look like it's KVL. We just need to take the surface integral on the right, change its sign and put it in the lhs.
What have we done, by doing so? We started with a formula that says the circulation of E is NOT zero (when the circuit path follows the filament of the coil and thus INCLUDES the variable dB/dt region) and by cutting away the part of path the follows the coil and replacing it with the jump at its terminals we ended up with a formula that says that the circulation of E is ZERO (because the modified circuit path now skips - DOES NOT INCLUDE - the variable dB/dt) region.
In this modified circuit - THAT DOES NOT INCLUDE THE dB/dt REGION - KVL holds in the sense intended by Mehdi, but the voltage ACROSS the inductor is the result of the application of Faraday's law and the exclusion of its filament from the circuit's path.

And this is the key: in order to pull this trick (which I call the 'amended KVL') you need to be able to find a circuit path that DOES NOT INCLUDE THE VARIABLE MAGNETIC REGION AT ITS INTERIOR. You can do it when you put both resistors on the same side of the ring. You CANNOT do it for Lewin's ring (because Lewin's ring is Lewin's ring ONLY IF THE RESISTORS ARE ON THE OPPOSITE SIDES OF THE VARIABLE MAGNETIC FIELD REGION AND THE CIRCUIT PATH INCLUDES THAT REGION).
Now try to focus:

You cannot exclude the variable magnetic flux region from the circuit path of a circuit that requires its circuit path to contain the variable magnetic flux region. No matter how hard you try.

Geometry matters, exactly because in the case of Lewin's ring we are talking about a non lumpable circuit whose circuit path is required to circle a variable magnetic region.

P.S
Incidentally, the last sentence in Belcher's note is the following:

Quote from: John Belcher
"An excellent discussion of “electromotive force” (a terrible and misleading name) is given in the text by Griffiths, Introduction to Electrodynamics 4th Edition."


and guess what? Griffiths has the Romer-Lewin ring in one of his problems. Problem 7.50 uses the same diagram as Romer's paper. And in the solution he states... "Notice that V1 != V2, even though they are connected to the same points!"
Well, it really looks like the reference Belcher suggests has been written by another of Lewin's minions.


Wow, all that and you didn't really answer my question. You never answer a question head-on, do you  :-DD

Exactly what argument did Mehdi make that Belcher agreed with?

Are you saying that the ONLY Mehdi-argument that Belcher agreed with was a fully lumpable circuit?
Title: Re: #562 – Electroboom!
Post by: Jesse Gordon on November 17, 2021, 07:39:22 pm
Quote
Are you talking about my EI-Core transformer with the two red secondary windings?

Because Sredni has said that an EI-Core transformer's secondary winding outputs qualify as black box because the dB/dt is contained (for all practical purposes) entirely inside the iron core.

You're still completely disregarding that you have multiple loops in your EI-Core transformer experiment.

@Sredni isn't wrong, you just have troubles understanding what he said because you only hear what seems to suit your understanding. That's called "confirmation bias". That transfomer is a black box only at its terminals. But your measurements are not at its terminals.

Quote
Let's say The 100mv and 100mv is induced where the wire passes through the transformer core.
And there's your error, right there. This is complete BS.


To clarify regarding my EI-Core transformer, with the two red wires that pass through the core, are you saying that if I connected my volt meter leads across the ends of one of those red wires, are you saying that I could change my reading by changing the path of the volt meter leads as I moved them around the outside of the transformer?

Make another prediction. I got an EI-Core. I got a volt meter. I can pass a wire through one side, the other side, or both sides if you like.

Tell me where I should run the secondary turn wire, where I should connect my volt meter leads, and what I should do to witness this alleged path-dependence.

I think you're wrong. Because the dB/dt is contained "entirely" within the iron core, changing the path of the leads external to the core can make no difference in measured voltaged because it can make no difference in the quantity of area of enclosing dB/dt, because in order to change the enclosed area of dB/dt, the volt meter lead would have to physically pass through part of the iron core.

But if you think you're right, make another prediction here. If you can get 4 correct predictions in a row, then you can bring your average above 50%  :-DD

Exactly how would I witness this alleged path-dependence you assert?
Title: Re: #562 – Electroboom!
Post by: thinkfat on November 17, 2021, 08:22:41 pm
Quote
Are you talking about my EI-Core transformer with the two red secondary windings?

Because Sredni has said that an EI-Core transformer's secondary winding outputs qualify as black box because the dB/dt is contained (for all practical purposes) entirely inside the iron core.

You're still completely disregarding that you have multiple loops in your EI-Core transformer experiment.

@Sredni isn't wrong, you just have troubles understanding what he said because you only hear what seems to suit your understanding. That's called "confirmation bias". That transfomer is a black box only at its terminals. But your measurements are not at its terminals.

Quote
Let's say The 100mv and 100mv is induced where the wire passes through the transformer core.
And there's your error, right there. This is complete BS.


To clarify regarding my EI-Core transformer, with the two red wires that pass through the core, are you saying that if I connected my volt meter leads across the ends of one of those red wires, are you saying that I could change my reading by changing the path of the volt meter leads as I moved them around the outside of the transformer?

Make another prediction. I got an EI-Core. I got a volt meter. I can pass a wire through one side, the other side, or both sides if you like.

Tell me where I should run the secondary turn wire, where I should connect my volt meter leads, and what I should do to witness this alleged path-dependence.

I think you're wrong. Because the dB/dt is contained "entirely" within the iron core, changing the path of the leads external to the core can make no difference in measured voltaged because it can make no difference in the quantity of area of enclosing dB/dt, because in order to change the enclosed area of dB/dt, the volt meter lead would have to physically pass through part of the iron core.

Exactly how would I witness this alleged path-dependence you assert?

I apologize in advance for the quality of the drawing I whipped up in "Libreoffice Draw". I'm not really an artistic person and this stupid program isn't helping me be, either.

The attached drawing is a cross section through your EI-Core transformer with your Lewin-Loop.

"V1" is connected like you're probing right now. The volt meter reads 100mV or thereabouts.

"V2" is connected through a different path. I predict that the volt meter will read 0mV (or close to it).

There you go, knock yourself out.
Title: Re: #562 – Electroboom!
Post by: Jesse Gordon on November 17, 2021, 09:28:37 pm
PS: I object to Dr. Belchers statement that the EMF is a voltage difference.

Wow, Dr. thinkphat, your objections are duly noted.

Actually, Dr. Belcher was quoting Dr. Feynman.

Dr. Belcher writes:

Quote

One textbook that makes perfectly clear what the “voltage across the inductor” is is the textbook by
Feynman, Leighton, and Sands, The Feynman Lectures on Physics (Addison‐Wesley, Reading, MA 1964),
Vol II, p22‐2).  In that textbook, the authors state explicitly that the -∫E.dl through the inductor must
be zero, and they define the voltage difference across the inductor (which the correctly identify as an
“electromotive force”, as ∮E.dl, which by Faraday’s Law is -L(dI/dt).

So there you have it. Feynman, Leighton, and Sands also identify the voltage difference across the inductor as an EMF.

I suppose you object to that too?

Besides, so far, 3 out of 3 of your predictions failed the reality check. Dr. Belchers on the other hand seem to pass the reality check. I think I'll go with what he says on that matter.
(And one out of one logic attempted failed as well.)
Title: Re: #562 – Electroboom!
Post by: Jesse Gordon on November 17, 2021, 09:42:13 pm
Quote
Are you talking about my EI-Core transformer with the two red secondary windings?

Because Sredni has said that an EI-Core transformer's secondary winding outputs qualify as black box because the dB/dt is contained (for all practical purposes) entirely inside the iron core.

You're still completely disregarding that you have multiple loops in your EI-Core transformer experiment.

@Sredni isn't wrong, you just have troubles understanding what he said because you only hear what seems to suit your understanding. That's called "confirmation bias". That transfomer is a black box only at its terminals. But your measurements are not at its terminals.

Quote
Let's say The 100mv and 100mv is induced where the wire passes through the transformer core.
And there's your error, right there. This is complete BS.


To clarify regarding my EI-Core transformer, with the two red wires that pass through the core, are you saying that if I connected my volt meter leads across the ends of one of those red wires, are you saying that I could change my reading by changing the path of the volt meter leads as I moved them around the outside of the transformer?

Make another prediction. I got an EI-Core. I got a volt meter. I can pass a wire through one side, the other side, or both sides if you like.

Tell me where I should run the secondary turn wire, where I should connect my volt meter leads, and what I should do to witness this alleged path-dependence.

I think you're wrong. Because the dB/dt is contained "entirely" within the iron core, changing the path of the leads external to the core can make no difference in measured voltaged because it can make no difference in the quantity of area of enclosing dB/dt, because in order to change the enclosed area of dB/dt, the volt meter lead would have to physically pass through part of the iron core.

Exactly how would I witness this alleged path-dependence you assert?

I apologize in advance for the quality of the drawing I whipped up in "Libreoffice Draw". I'm not really an artistic person and this stupid program isn't helping me be, either.

The attached drawing is a cross section through your EI-Core transformer with your Lewin-Loop.

"V1" is connected like you're probing right now. The volt meter reads 100mV or thereabouts.

"V2" is connected through a different path. I predict that the volt meter will read 0mV (or close to it).

There you go, knock yourself out.
(https://i.postimg.cc/Kvrx3DNc/Faraday.png)

What are you talking about? I don't have a third secondary winding. You have drawn a third secondary winding.
Obviously if you put in a third secondary winding and wire it in series with another secondary winding, you can cause the voltages to either add or subtract.
You've basically formed a fake secondary winding with zero dB/dt enclosed.


That is NOT MY SETUP.

I AM NOT running my volt meter leads through the core of the transformer. If I did, I'd be adding more secondary windings.

What sort of diversion is this? Can't you see that according to Faraday's law the one wire is a clockwise turn and the other is a counter clockwise turn, and that the voltage across one will be the negative of the other, and when summed they will be zero?

You saw the video, I simply have two single-pass secondary windings. That is all.

Where the tarzan did you get the idea that I had a third volt meter lead run through the transformer, thus forming a third secondary winding?

But FOR A GIVEN SECONDARY WINDING, without changing the number of turns between tests, why on earth isn't my transformer secondary windings considered a black box for the sake of KVL?

You're grabbing at straws my friend, or in this case, you're grabbing at secondary windings.

Dude, look. The ability to change the number of turns on the secondary doesn't mean that for a given number of turns the output of the secondary is not a lumpable voltage source which can be considered a black box.
Title: Re: #562 – Electroboom!
Post by: thinkfat on November 17, 2021, 10:01:00 pm
...

I deleted your incoherent ranting.

You asked for an example how you would be able to "experience" the path dependence. I provided a drawing.

I predict that V1 will show 100mV, V2 will show 0V. What else are you asking? It is the same probing points, just connected to the volt meter through different paths. This is what "path dependent" is about.

The path going through V1 is linking the flux in the core, the path through V2 is not. V1 displays an EMF in the circuit going around one leg of the EI-Core, V2 displays just the voltage drop in the red wire according to Mr. Ohm, which will be (close to) 0V.

But your reaction shows quite clearly that you don't have the slightest idea how a transformer works, and what makes it work.

Really, this feels like discussing with a flatearther.
Title: Re: #562 – Electroboom!
Post by: thinkfat on November 17, 2021, 10:17:03 pm
That is NOT MY SETUP.

See the attached screenshot from our discussion on YT, from the comment section of your video (saved "just in case").
Title: Re: #562 – Electroboom!
Post by: thinkfat on November 17, 2021, 10:31:03 pm
That is NOT MY SETUP.

See the attached screenshot from our discussion on YT, from the comment section of your video (saved "just in case").

See the attached screenshot for your wiring setup. Clearly the red wires are inside the transformer. How on earth can you expect this not picking up an induced EMF when you just connect the probing leads on the outside, at both ends of the red wires? Do you really think you have to sling a wire around the core to create a "secondary"?
Title: Re: #562 – Electroboom!
Post by: thinkfat on November 17, 2021, 10:42:08 pm
That is NOT MY SETUP.

See the attached screenshot from our discussion on YT, from the comment section of your video (saved "just in case").

See the attached screenshot for your wiring setup. Clearly the red wires are inside the transformer. How on earth can you expect this not picking up an induced EMF when you just connect the probing leads on the outside, at both ends of the red wires? Do you really think you have to sling a wire around the core to create a "secondary"?

And for the sake of completeness, another screenshot showing how you connect the probing leads, closing the loop, red wire on one side of the core, probe leads on the other side of the core.
Title: Re: #562 – Electroboom!
Post by: Sredni on November 17, 2021, 10:58:56 pm
[...]
Mehdi talked about a LUMPED circuit in his first video (even though he does not realize that it IS NOT Lewin's ring, which is unlumpable exactly because the resistors are required to be on the opposite sides of the magnetic flux region). And that he applied KVL to said lumped circuit (which we can do, in the 'modified' form. Too bad it is not Lewin's ring). And he also says "KVL holds"!
[...]

Wow, all that and you didn't really answer my question. You never answer a question head-on, do you  :-DD
Exactly what argument did Mehdi make that Belcher agreed with?
Are you saying that the ONLY Mehdi-argument that Belcher agreed with was a fully lumpable circuit?

You are absolutely right. I committed an error. My bad.
I should have known better: if you are not capable of understanding a single sentence because it has a subordinate, there would be no hope that you could understand several paragraphs all in one post.
Let me use a single line. I'll try to simplify it for you:

Mehdi talked about a LUMPED circuit in his first video, he applied (extended) KVL to it and said "KVL holds".

Well, he was right. I too agree with Mehdi that KVL holds when applied to circuits with lumped inductors. And if you actually read my previous post you will see that I wrote the same sentence Belcher wrote. Am  I agreeing with Mehdi on everything now?





Seriously dude, you really, really, really don't understand Faraday's law. You should look at surfaces, not segment of wire.
Title: Re: #562 – Electroboom!
Post by: Sredni on November 18, 2021, 12:02:23 am
Jesse, you are aware that this is how you expect to measure the voltage in the conductors?

(https://i.postimg.cc/B61Zt8Nx/Bad-probing.jpg)
https://i.postimg.cc/B61Zt8Nx/Bad-probing.jpg

(it's a negative of a drawing on paper, therefore the filled probe tips are the black ones - I did not pay excessive attention at the signs but they should be right. Also I chose 100 mV for the central leg and 50 mV for the other two. And I did not care to look up the values for the resistor - fill them in, if you want)
Does it not strike you as strange that if you remove the ring, leaving only the one conductor you expect to measure the voltage of, you still measure half the EMF of the central leg (50 mV)?
Not a bit?

Moreover, you can slide the probes along the conductors and you would still always read the same voltages. Does it not strike you as curious that the voltage drop for the wires is the same whether you consider the full branch or one millimeter of conductor?

The left and right voltmeters form flux-free loops with the nearest resistor branches, and therefore measure the correct branch voltages.
Title: Re: #562 – Electroboom!
Post by: Jesse Gordon on November 18, 2021, 03:37:10 am
[...]
Mehdi talked about a LUMPED circuit in his first video (even though he does not realize that it IS NOT Lewin's ring, which is unlumpable exactly because the resistors are required to be on the opposite sides of the magnetic flux region). And that he applied KVL to said lumped circuit (which we can do, in the 'modified' form. Too bad it is not Lewin's ring). And he also says "KVL holds"!
[...]

Wow, all that and you didn't really answer my question. You never answer a question head-on, do you  :-DD
Exactly what argument did Mehdi make that Belcher agreed with?
Are you saying that the ONLY Mehdi-argument that Belcher agreed with was a fully lumpable circuit?

You are absolutely right. I committed an error. My bad.
I should have known better: if you are not capable of understanding a single sentence because it has a subordinate, there would be no hope that you could understand several paragraphs all in one post.
Let me use a single line. I'll try to simplify it for you:

Mehdi talked about a LUMPED circuit in his first video, he applied (extended) KVL to it and said "KVL holds".

Well, he was right. I too agree with Mehdi that KVL holds when applied to circuits with lumped inductors. And if you actually read my previous post you will see that I wrote the same sentence Belcher wrote. Am  I agreeing with Mehdi on everything now?


Seriously dude, you really, really, really don't understand Faraday's law. You should look at surfaces, not segment of wire.

So you're saying that Mehdi argued that KVL holds for a bunch of circuits including lumpable and unlumpable, and Belcher only agreed with the argument for Mehdi's lumpable circuit?
(but neglected to say that he disagreed with any of the rest..)

And even Lewin would have agreed that a lumpable circult works with KVL, so there was no argument between Mehdi and Lewin on lumpable circuits.

So if the one circuit which Lewin and Mehdi agreed about was not even an argument, then why the tarzan did Belcher say "As ARGUED by Mehdi?"

And besides, Belcher's argument was not based on lumpability. It was based on the term for Faraday's law -- which was in every one of Mehdi's loops.

I do notice that again you didn't answer my question head-on.

The fact is that Belcher said "As argued by Mehdi" and I asked you "What argument did Mehdi make that Belcher is agreeing with.

And then you go on talking about what Mehdi TALKED about in one of his videos - but that's not the argument I asked for.


Title: Re: #562 – Electroboom!
Post by: Jesse Gordon on November 18, 2021, 03:50:43 am
Jesse, you are aware that this is how you expect to measure the voltage in the conductors?

(https://i.postimg.cc/B61Zt8Nx/Bad-probing.jpg)
https://i.postimg.cc/B61Zt8Nx/Bad-probing.jpg

(it's a negative of a drawing on paper, therefore the filled probe tips are the black ones - I did not pay excessive attention at the signs but they should be right. Also I chose 100 mV for the central leg and 50 mV for the other two. And I did not care to look up the values for the resistor - fill them in, if you want)
Does it not strike you as strange that if you remove the ring, leaving only the one conductor you expect to measure the voltage of, you still measure half the EMF of the central leg (50 mV)?
Not a bit?

Moreover, you can slide the probes along the conductors and you would still always read the same voltages. Does it not strike you as curious that the voltage drop for the wires is the same whether you consider the full branch or one millimeter of conductor?

The left and right voltmeters form flux-free loops with the nearest resistor branches, and therefore measure the correct branch voltages.

Are you saying that the two red wires passing through the two halves of my transformer core are not lumpable voltage sources?

But it looks like if I had one wire that looped through both halves, then it's lumpable?

Questions:

Answering these will help me understand what you're trying to say.

1:If I pass my secondary through BOTH halves for a total of 200mv, will the secondary then be a lumpable voltage sources?

2: If instead of an EI-Core, I had a toroid, and I pass a wire through the center, would that secondary be lumpable?

3: If instead of an EI-Core, I had two nearly identical toroids with the primaries in series and the same number of primary turns on each, and I passed a secondary wire through each of the toroids, would those two outputs be lumpable voltage sources?

4: If instead of an EI-Core, I had two nearly identical toroids side by side like OO with the primary wound through both of them,  and I passed a secondary wire through each of the toroids, would those two secondaries be lumpable voltage sources?

Title: Re: #562 – Electroboom!
Post by: Jesse Gordon on November 18, 2021, 04:07:59 am
...

I deleted your incoherent ranting.

You asked for an example how you would be able to "experience" the path dependence. I provided a drawing.


You and I already have always agreed that adding or subtracting turns from a transformer changes the voltage.

But that's not what this is about. I asked how the path of the wires OUTSIDE my transformer could affect the reading.

But I guess you're basically admitting that the only way my secondary output windings are unlumpable is if the number of turns randomly changes, otherwise you wouldn't have had to add in another secondary winding to induce a different voltage reading.

If you honestly didn't understand my question, this is it, clarified:

FOR A GIVEN AND FIXED NUMBER OF SECONDARY TURNS ON AN IRON CORE TRANSFORMER NOT OPERATED NEAR SATURATION WITH A CLOSED MAGNETIC CIRCUIT CORE, How can I witness the measured voltage changing due to the path of my VOLT METER LEADS (Not the fixed internal path of the fixed windings)?

I think your answer is that the voltage measured on my two secondaries is unambiguous and lumpable so long as the number of turns doesn't magically change itself at random times.

I predict that V1 will show 100mV, V2 will show 0V. What else are you asking? It is the same probing points, just connected to the volt meter through different paths. This is what "path dependent" is about.

The path going through V1 is linking the flux in the core, the path through V2 is not. V1 displays an EMF in the circuit going around one leg of the EI-Core, V2 displays just the voltage drop in the red wire according to Mr. Ohm, which will be (close to) 0V.

But your reaction shows quite clearly that you don't have the slightest idea how a transformer works, and what makes it work.

Really, this feels like discussing with a flatearther.

Please don't say that. Don't think for a moment that the feeling isn't mutual.
The fact is that two sincere people can have wildly different understandings and though they both try to do their best, it feels to each like the other guy is crazy.

But we continue on in hopes that we will be able to learn from eachother.
Title: Re: #562 – Electroboom!
Post by: Jesse Gordon on November 18, 2021, 04:11:50 am
That is NOT MY SETUP.

See the attached screenshot from our discussion on YT, from the comment section of your video (saved "just in case").

See the attached screenshot for your wiring setup. Clearly the red wires are inside the transformer. How on earth can you expect this not picking up an induced EMF when you just connect the probing leads on the outside, at both ends of the red wires? Do you really think you have to sling a wire around the core to create a "secondary"?

I take it you are saying that my two red wires are not lumpable voltage sources...? Wouldn't this be evidenced by a change in volt meter reading by where I hung my volt meter leads OUTSIDE the transformer?

Questions:

Answering these will help me understand what you're trying to say.

1:If I pass my secondary through BOTH halves for a total of 200mv, will the secondary then be a lumpable voltage sources?

2: If instead of an EI-Core, I had a toroid, and I pass a wire through the center, would that secondary be lumpable?

3: If instead of an EI-Core, I had two nearly identical toroids with the primaries in series and the same number of primary turns on each, and I passed a secondary wire through each of the toroids, would those two outputs be lumpable voltage sources?

4: If instead of an EI-Core, I had two nearly identical toroids side by side like OO with the primary wound through both of them,  and I passed a secondary wire through each of the toroids, would those two secondaries be lumpable voltage sources?
Title: Re: #562 – Electroboom!
Post by: Jesse Gordon on November 18, 2021, 04:37:21 am
Continuously variable variacs have been around forever - I've got one, real pretty, all silver plated, you turn the knob, and the wiper slides or rolls along the winding providing a movable  tap at any of an infinite number of positions, limited only by the smoothness of the winding wire and the wiper roller.

Could you show a picture or a link?  I've never seen a variac like that.   I assume you mean a line-voltage variac?

"Variac" may not be the most common term for the specific construction I mention, but the function is identical, and anyone who's seen one would know exactly what I meant I suppose.

Mine's not a line-voltage unit, and in fact, most are much smaller. It's a ham radio thing - but they are an inductor with a movable tap which is continuously movable.

Here's an example: http://k3rrr.com/cool-antenna-tuner-images-3/ (http://k3rrr.com/cool-antenna-tuner-images-3/)

You can see it's got the two end-of-winding terminals and the moving wiper terminal. Really cool bit of hardware from the ham radio days :D
Title: Re: #562 – Electroboom!
Post by: EEVblog on November 18, 2021, 04:43:47 am
I had no idea the KVL thing was raging on the forum again!  :popcorn:
Title: Re: #562 – Electroboom!
Post by: rfclown on November 18, 2021, 04:51:08 am

Really, this feels like discussing with a flatearther.

Please don't say that. Don't think for a moment that the feeling isn't mutual.
...

With both sides feeling like they are debating flat earthers.
Title: Re: #562 – Electroboom!
Post by: thinkfat on November 18, 2021, 07:58:31 am
I take it you are saying that my two red wires are not lumpable voltage sources...? Wouldn't this be evidenced by a change in volt meter reading by where I hung my volt meter leads OUTSIDE the transformer?

That's what I'm saying. There is no discernible region in the circuit formed by your volt meter and the red wire that could be thought of as a voltage source. It is certainly not the red wire, nor is it the segment of the wire that happens to cross the core.

Quote
Questions:

Answering these will help me understand what you're trying to say.

1:If I pass my secondary through BOTH halves for a total of 200mv, will the secondary then be a lumpable voltage sources?

2: If instead of an EI-Core, I had a toroid, and I pass a wire through the center, would that secondary be lumpable?

3: If instead of an EI-Core, I had two nearly identical toroids with the primaries in series and the same number of primary turns on each, and I passed a secondary wire through each of the toroids, would those two outputs be lumpable voltage sources?

4: If instead of an EI-Core, I had two nearly identical toroids side by side like OO with the primary wound through both of them,  and I passed a secondary wire through each of the toroids, would those two secondaries be lumpable voltage sources?

I cannot answer that. I have trouble understanding what actual geometry you are describing. What do you mean by "passing a wire through each of the toroids"?
I think we need to go back to the basics of Faradays Law to align our mutual understanding. The EI-Core arrangement is already too complex. The EI geometry of the core allows for too many possibilities to shoot oneself in the foot and we should abandon it for the discussion. A single toroid core and a setup like in the following video would be much easier.

https://www.youtube.com/watch?v=u6ud7JD0fV4 (https://www.youtube.com/watch?v=u6ud7JD0fV4)

Regarding the possibility of "lumping" a transformer winding, I can only repeat: you can do that only if nothing that is connected to the winding forms a circuit that is "coupling" with the magnetic flux going through the transformer. You EI-Core setup violates that condition and therefore can not be lumped. Your Lewin Loop is a "secondary", but it is in its entirety including the resistors and therefore all measurements taken on parts of the loop are path-dependent. The circuit given by the red wire and your volt meter is also a "secondary" and cannot be lumped.
Title: Re: #562 – Electroboom!
Post by: Jesse Gordon on November 18, 2021, 08:36:20 am
Questions:

Answering these will help me understand what you're trying to say.

1:If I pass my secondary through BOTH halves for a total of 200mv, will the secondary then be a lumpable voltage sources?

2: If instead of an EI-Core, I had a toroid, and I pass a wire through the center, would that secondary be lumpable?

3: If instead of an EI-Core, I had two nearly identical toroids with the primaries in series and the same number of primary turns on each, and I passed a secondary wire through each of the toroids, would those two outputs be lumpable voltage sources?

4: If instead of an EI-Core, I had two nearly identical toroids side by side like OO with the primary wound through both of them,  and I passed a secondary wire through each of the toroids, would those two secondaries be lumpable voltage sources?

I cannot answer that. I have trouble understanding what actual geometry you are describing.

Uhh, well work with me. It's not that complex. It's also not a trick question.

What do you mean by "passing a wire through each of the toroids"?

Oh dear. Ok. A toroidal transformer has exactly ONE opening in the core, as the core takes on the shape of a torus.
When you want to add a new secondary winding to a toroidal transformer, you thread that new secondary winding wire through that one hole in the center of the core, and voilà! you have a new secondary winding! If you want more turns to your new secondary, loop it through a few more times. 

You seriously don't know what it means to pass a wire through a hole in a toroid? Should I have said "Thread it through?" or "Place the wire so that it follows a path through the hole in the toroidal transformer core?"

I think we need to go back to the basics of Faradays Law to align our mutual understanding.The EI-Core arrangement is already too complex. The EI geometry of the core allows for too many possibilities to shoot oneself in the foot and we should abandon it for the discussion.

Dude, an EI-Core is not complex! It's just two toroids side by side with the primary windings in series!

If you call that complex I guess we're in trouble.

A single toroid core and a setup like in the following video would be much easier.

Since you understand a simple toroidal transformer, at least answer THAT one of my questions!

I'll repeat it and clarify it  so you realize we're talking about transformers here:

2: If I had a toroid based transformer, and I pass a wire through the center of the toroid to form a new secondary,, would that new secondary be a lumpable black box voltage source for the sake of KVL?

Thank you!
Title: Re: #562 – Electroboom!
Post by: Jesse Gordon on November 18, 2021, 08:45:43 am
TO: thinkfat, Sredni, bsfeechannel

You guys are constantly telling me that I don't know what I'm talking about and that you do know what you're talking about. But anybody can say that. But not anybody can prove it.

Please solve the following scenario to 10 digits past the decimal point and post your results so that I know you actually know something about this stuff.

Also please explain the math you used to arrive at your results, and where various amounts of voltage are lost, if any.

It was tough for me, but since you know more than me about this it should be a snap. I used nice clean numbers as provided data.

I'm sure you do know how to do this, but how do I know you do? We're just anonymous usernames flitting around cyberspace and I'm here possibly taking advice from people who present themselves as knowing astronomically more about the topic than me, so it would be a real nice thing for me to see you strut some of your stuff on the following challenge.

You could be MIT Professor triple PhD of physics, world leader on the topic.

Or you could be an ordinary guy like me trying to make sense of things.

You claiming you got great knowledge, understanding,  or credentials doesn't help me a lot - but if you can solve this for me, then at least I'll know you know your stuff, or at least I'll know you know how to figure out real-world stuff.

Once you've each presented your results, I'll present my results and you all can tell me how I did it wrong if that brings you pleasure.

Thank you!

(https://i.postimg.cc/j2gpWPqv/Transformer.png)
Title: Re: #562 – Electroboom!
Post by: Sredni on November 18, 2021, 09:02:58 am
About lumped and not lumped

For the other forum fellows who have not yet deciphered your view: Jesse here, wants to treat the two red straight segment of wire in his setup as two distinct lumped transformers.

Are you saying that the two red wires passing through the two halves of my transformer core are not lumpable voltage sources?

Well, it is kinda lumpable, and if you can attach it to a circuit that can be shrunk (lumped) to a point (this requires that there be no flux caught inside), you can consider it a lumped component in a lumped circuit.
Problem is, you cannot have a spatially extended component along your lumped circuit path, so you at least need to have the terminals close together, ideally separated by an infinitesimal distance.
In this picture I used the terms lumpable, kinda lumpable and unlumpable

(https://i.stack.imgur.com/HXv9x.jpg)
lumpable kindalumpable unlumpable

But maybe it would be preferable to use: lumped, lumpable, and unlumpable. ( It's always after you have scanned them that you notice these things...)

In order to make that lumpable system of yours
(https://i.postimg.cc/gjhfcHXZ/screenshot-10.png)
Pic torus with straight wire and circuit missing a coil
link https://i.postimg.cc/gjhfcHXZ/screenshot-10.png

into a lumped transformer you just have to solve the problem of how to put those terminals close together. How do you plan to do it? There are two options

Option 1: You modify your component to fit the circuit
Take one on the other side to almost meet the other, or nearly join them together halfway. Now you can actually see a voltage in the space between terminals.

(https://i.postimg.cc/g2mfdRYx/screenshot-11.png)
Pic bent wire around torus: change the component to fit the circuit
link https://i.postimg.cc/g2mfdRYx/screenshot-11.png

And if you stay outside your black box and you do not run around the flux it will be always the same. Notiche how the charges have moved from the conductor's extreme to the resistor.

Option 2: You modify your circuit to fit the component
You insist in calling that piece of wire a transformer? Well, suit yourself, I will not alter it at all. But I still need to put it into the circuit, so I make connections to its end terminals

(https://i.postimg.cc/7LLdJkJD/screenshot-12.png)
Pic change the circuit to fit the component
link https://i.postimg.cc/7LLdJkJD/screenshot-12.png

and now you have your component unaltered attached to your circuit. The circuit path (highlighted here by the dotted line) is now including the component and along with it, the flux lines. And since your circuit path encompasses a variable magnetic flux, the circulation of the electric field can no longer be zero. This means that the path integral of the electric circuit depends on the path and therefore, voltage is path dependent. Kirchhoff's loop law is for the birds.

There is also the 'wishful thinking' option:
You just ignore component and circuit boundaries and cram everything together, hoping for the best. Let's apply this to your circuit inside the EI transformer.
Here on the left is the circuit with an elastic mebrane on it. It is integer and pristine. Look how happy it is, singing one of Madonna's earliest greatest hits.
But then you put it inside the EI transformer. Do you really think you can do that without breaking the membrane? Look what you did to your little circuit's membrane:

(https://i.postimg.cc/65LmH3dt/screenshot-13.png)
fig rammed to death
link https://i.postimg.cc/65LmH3dt/screenshot-13.png

I really hope you had proof of previous consent, because you broke the law: Kirchhoff's law, to be specific.

(https://i.postimg.cc/xd7xkbSy/screenshot-17.png)
Just look at the circuit path.
link https://i.postimg.cc/xd7xkbSy/screenshot-17.png

The broken membrane is proof that your circuit cut the flux lines. KVL is for the birds. Also, since your circuit path is going around the changing flux with no way to exclude it, your circuit is unlumpable. Lewin would be proud of you.



Bonus Track: What's the deal with the straight partial 'turn'?
I believe I figured you out now.
You are a lumper who believes that when a piece of wire 'goes through the hole' in the transformer, it develops a voltage, that is nonzero, irregardless of the path it is computed on! As if the hole was some kind of portal to another dimension, like a stargate.
No. If it intercepts the induced electric field, the wire gets polarized, and you might see charges of opposite sign at the extremities, but the voltage between point A at one end and point B at the other end is not unique: it depends on the path.

Along all these paths, the voltage (i.e. the work done per unit charge that is compute as minus path integral of E.dl) is zero (just like with a polarized conductor in an electrostatic field - see my previous silent post)

(https://i.postimg.cc/mZ7XDDpC/screenshot-14.png)
Fig straight wire and paths zero v
link  https://i.postimg.cc/mZ7XDDpC/screenshot-14.png

Along this other type of paths, paths that together with the piece of wire form a loop that runs around the changing flux, the voltage is nonzero ( one turn emf, so to speak). There is your transformer.

(https://i.postimg.cc/HL70hZfX/screenshot-15.png)
Fig straight wire and paths emf
link https://i.postimg.cc/HL70hZfX/screenshot-15.png

But how can you make it lumped / part of a lumped circuit? Bend the conductor a little more so as to bring the terminals almost together, as shown above. Now you can insert it into a shrinkable circuit and, if the circuit path does not include variable flux you have a lumped circuit where KVL holds. You will see a jump in voltage at the component's terminals. The displaced charge is not there, though, it is at the resistor's boundaries if there is one, or facing the gap if it is open circuited.

Which side is the hole in the core?
Incidentally, this is the equivalent of what you want to consider a lumped transformer with the toroidal or EI cores,  but with the core in the form of an infinitely long cylinder where the primary is an infinitely long solenoid. So, you have a piece of wire, and the 'hole' is basically the whole space. Where is left of the hole, or the right of the hole? Is the wire inside the core, or outside of it?

(https://i.postimg.cc/qvWyyczR/screenshot-16.png)
Pic straight wire solenoid
link https://i.postimg.cc/qvWyyczR/screenshot-16.png

Do you still see a lumped transformer? Or in order to call that piece of polarized wire a transformer you first need to bend it and bring the terminals together in a way that the wire almost encircles the core? 
 
Title: Re: #562 – Electroboom!
Post by: thinkfat on November 18, 2021, 09:29:50 am
What do you mean by "passing a wire through each of the toroids"?

Oh dear. Ok. A toroidal transformer has exactly ONE opening in the core, as the core takes on the shape of a torus.
When you want to add a new secondary winding to a toroidal transformer, you thread that new secondary winding wire through that one hole in the center of the core, and voilà! you have a new secondary winding! If you want more turns to your new secondary, loop it through a few more times. 

You seriously don't know what it means to pass a wire through a hole in a toroid? Should I have said "Thread it through?" or "Place the wire so that it follows a path through the hole in the toroidal transformer core?"

I think we need to go back to the basics of Faradays Law to align our mutual understanding.The EI-Core arrangement is already too complex. The EI geometry of the core allows for too many possibilities to shoot oneself in the foot and we should abandon it for the discussion.

Dude, an EI-Core is not complex! It's just two toroids side by side with the primary windings in series!

If you call that complex I guess we're in trouble.


It appears to be too complex for you, I was trying to reduce the complexity for your sake, not mine.
The term "threading a wire trough" already implies that you misunderstand how the loop/winding/secondary is formed.
In this matter, understanding the geometry is key. I needed to make sure that I get what you mean.

A single toroid core and a setup like in the following video would be much easier.

Since you understand a simple toroidal transformer, at least answer THAT one of my questions!

I'll repeat it and clarify it  so you realize we're talking about transformers here:

2: If I had a toroid based transformer, and I pass a wire through the center of the toroid to form a new secondary,, would that new secondary be a lumpable black box voltage source for the sake of KVL?

Yes, the complete "winding" in its entirety is a lumpable black box voltage source for everything that happens outside of the transformer.
Title: Re: #562 – Electroboom!
Post by: bsfeechannel on November 18, 2021, 04:33:21 pm

Really, this feels like discussing with a flatearther.

Please don't say that. Don't think for a moment that the feeling isn't mutual.
...
With both sides feeling like they are debating flat earthers.

And the rest not having a clue what is really being discussed.

In short it is like this.
Lewin: KVL doesn't hold for circuits immersed in a non-conservative electric field, here is the theory to support this claim and here is the demonstration.
McDonald: Your calculations and your experiment are spot-on, Lewin, but don't say KVL doesn't hold because, although it really doesn't describe your circuit very well, I can still use KVL equations to arrive at the same results. Here is the calculations. It is a math trick used to solve various problems in electromagnetism.
Lewin: Yeah. I know many authors use that trick and even call the attention to the fact that you will not find the EMF as a voltage in the circuit, but people don't pay attention to that and try to invent all kinds of unscientific explanations to prove that there is a voltage there hidden, or somehow masked, in the circuit.
McDonald: I agree, this can be misleading sometimes.
Mehdi (intruding on adults' conversation): Hey Lewin, I think you're wrong and I have McDonald's paper to prove it. Your experiment is flawed.
Lewin: Talk to Belcher. I'm retired.
Mehdi: Belcher, please, say I'm right, say I'm right, please, please, please. I'm in desperate need of viewership and I need to convey the idea that I'm an expert electromagnetism.
Belcher: Alright, Mehdi. You're right, BUT, and that's a big BUT, only when it comes to circuits with lumped components and no varying magnetic fields inside the path of the circuit.
Mehdi Woohoo! You see audience? I'm right. Who said that? An MIT professor and the paper by McDonald. Lewin is wrong. Now subscribe, hit the like bottom, pour your sweat money into my Patreon account and buy the sponsor's shit.

People do exactly that and prove how humans failed as a species.
Title: Re: #562 – Electroboom!
Post by: jesuscf on November 18, 2021, 05:41:25 pm
Lewin: KVL doesn't hold for circuits immersed in a non-conservative electric field, here is the theory to support this claim and here is the demonstration.

What non-conservative electric field are you talking about?
Title: Re: #562 – Electroboom!
Post by: Sredni on November 18, 2021, 06:09:33 pm
Lewin: KVL doesn't hold for circuits immersed in a non-conservative electric field, here is the theory to support this claim and here is the demonstration.
What non-conservative electric field are you talking about?

(https://i.postimg.cc/GmtYp52x/screenshot.png)
https://i.postimg.cc/GmtYp52x/screenshot.png

 :-DD
Bsfeechannel has omitted Feynman: "Belcher and Feynman agree with me".
Title: Re: #562 – Electroboom!
Post by: thinkfat on November 18, 2021, 06:35:33 pm
I had no idea the KVL thing was raging on the forum again!  :popcorn:
This must be the longest thread on the AmpHour topic by a huge margin.
Well done :clap:
Title: Re: #562 – Electroboom!
Post by: bdunham7 on November 18, 2021, 06:44:32 pm
What non-conservative electric field are you talking about?

(https://www.eevblog.com/forum/amphour/562-electroboom!/?action=dlattach;attach=1326467;image)
Title: Re: #562 – Electroboom!
Post by: Jesse Gordon on November 18, 2021, 08:11:23 pm
Do you still see a lumped transformer? Or you need to bend it and bring the terminals together in a way that the wire almost encircles the core?

I'm waiting to see when your pics show up to reply in whole, but I have a couple questions.

If I have a toroidal transformer, and I thread a wire once through the hole to form my secondary "winding," does anything else matter?

if I connect my volt meter leads to the ends of my new secondary "winding," and read exactly 1.000000 volts, will the reading I get change depending on how I shift around the physical positions of my volt meter leads?

In other words, assuming ONE TURN ONLY on the secondary, does the path of my volt meter leads OUTSIDE THE CORE have ANY affect on the measured voltage?

Or is one pass through the center of a toroid the same as EXACTLY one turn around an infinitely long solenoid?

The way I see it, other than the crucial pass through the hole in the core, the path of my leads do not matter, because the entirety of the dB/dt is contained within the core, so even though you draw my wires outside the core in different places to show it forming various sized loops around the cross section of the core, the fact is that the cross sectional area of the core never changes and neither does dB or dt, so d(BA)/dt is exactly the same regardless of how tightly my secondary hugs the core -- d(BA)/dt is exactly the same whether my secondary is a straight wire that passes once through and goes all the way around the world as it is if that one turn wraps tightly around the side of the core, crosses over itself.

Right?

Or are you saying that these two setups would read different voltages: (Assuming an ideal uniform core and no resistive losses in the windings and infinitely high impedance volt meters.)
(https://i.postimg.cc/RZp1Ykwb/20211118-115444.jpg)

Are you saying V1 and V2 will read differently?
Title: Re: #562 – Electroboom!
Post by: Jesse Gordon on November 18, 2021, 08:15:43 pm
Yes, the complete "winding" in its entirety is a lumpable black box voltage source for everything that happens outside of the transformer.

Thank you!

So then tell me please, in the following diagram, Which of V1 and V2 are lumpable? Both,? Neither? Which?

 (https://i.postimg.cc/RZp1Ykwb/20211118-115444.jpg)
Title: Re: #562 – Electroboom!
Post by: jesuscf on November 18, 2021, 08:24:53 pm
What non-conservative electric field are you talking about?

So, what is generating the external electric field?  Is the circuit under test between the plates of a giant capacitor with a varying voltage applied to it?  I thought that in Lewin's experiment we had an varying external magnetic field generated by a coil.  That is why I am asking.
Title: Re: #562 – Electroboom!
Post by: Jesse Gordon on November 18, 2021, 09:19:43 pm

Really, this feels like discussing with a flatearther.

Please don't say that. Don't think for a moment that the feeling isn't mutual.
...
With both sides feeling like they are debating flat earthers.

And the rest not having a clue what is really being discussed.

In short it is like this.
Lewin: KVL doesn't hold for circuits immersed in a non-conservative electric field, here is the theory to support this claim and here is the demonstration.
McDonald: Your calculations and your experiment are spot-on, Lewin, but don't say KVL doesn't hold because, although it really doesn't describe your circuit very well, I can still use KVL equations to arrive at the same results. Here is the calculations. It is a math trick used to solve various problems in electromagnetism.
Lewin: Yeah. I know many authors use that trick and even call the attention to the fact that you will not find the EMF as a voltage in the circuit, but people don't pay attention to that and try to invent all kinds of unscientific explanations to prove that there is a voltage there hidden, or somehow masked, in the circuit.
McDonald: I agree, this can be misleading sometimes.
Mehdi (intruding on adults' conversation): Hey Lewin, I think you're wrong and I have McDonald's paper to prove it. Your experiment is flawed.
Lewin: Talk to Belcher. I'm retired.
Mehdi: Belcher, please, say I'm right, say I'm right, please, please, please. I'm in desperate need of viewership and I need to convey the idea that I'm an expert electromagnetism.
Belcher: Alright, Mehdi. You're right, BUT, and that's a big BUT, only when it comes to circuits with lumped components and no varying magnetic fields inside the path of the circuit.
Mehdi Woohoo! You see audience? I'm right. Who said that? An MIT professor and the paper by McDonald. Lewin is wrong. Now subscribe, hit the like bottom, pour your sweat money into my Patreon account and buy the sponsor's shit.

People do exactly that and prove how humans failed as a species.



Or maybe it's like this:

Lewin: Hello, hello, hello! Today I am going to show you something amazing! Something so amazing that you will be telling your grandchildren about it!
Lewin: In fact, it is so amazing that this is probably the only time in your entire life that you will see it!

He then presents an air core transformer, which he has incorrectly drawn on the chalkboard, and declares:

Lewin: KVL IS FOR THE BIRDS! All the textbooks are wrong! Only I am the source of truth on this matter, and even my fellow professors think I CHEATED! That shows you what THEY do!! They must be all cheaters! Yes, the whole world is insane but me!

Mehdi shocks himself a few times to see if this newfound knowledge will sink in and make sense. But it doesn't, so like any seeker of truth worth his sparks, he shocks himself a few more times and maybe melts down some clip leads, and then gets busy doing experiments, and finds that Lewin was incorrectly probing.

Mehdi: Professor Lewin, good sir, I tried my best to understand this, but I think you're not probing correctly. What am I missing?

Lewin: FLAT EARTHER! FLAT EARTHER! I do not argue with flat earthers! I won't even watch your video, but you're just wrong! KVL IS FOR THE BIRDS!

Mehdi: Professor McDonald, Professor Belcher, I'm really trying my best to understand this, and my fingers hurt and it smells like burnt clip leads in here. It looks to me like when correctly probing, KVL holds fine. It looks like Professor Lewin did not account for the dB/dt in the loop formed by his volt meter leads, which he did not depict as being magnetically coupled to his loop under test. But he won't even watch my video. What am I missing here?

McDonald: Yeah, he probably can't stand to see you shocking yourself and burning up perfectly good clip leads.
McDonald: Also, Dr. Lewin is as much of a showman as you. He uses diversion and an obsolete version of a "law" to create an apparent paradox.
McDonald: But you are right, Lewin’s circuit is within the range of applicability of Kirchhoff’s loop equations, which
can be used to predict measurements by the “voltmeters” in the experiment.

Belcher: Lewin has fallen for a very common misconception where he thinks that the -L(dI/dt) voltage read by volt meters represents -∫E.dl through the inductor, but that's false. Feynman is very clear that while -∫E.dl through a superconductor is zero, the voltage difference across the inductor is defined as ∮E.dl  (which he says is EMF) -- and this is Faradays law -L(dI/dt).
Belcher: According to Feynman's definition, the sum of all the voltage differences around the circuit is zero (that is, KVL holds) +V - IR - Q/C - L(dI/dt)=0, but the first three terms here are the -∫E.dl through various circuit elements, and the last term has NOTHING to do with the -L(dI/dt) through the inductor, which is the part that Lewin is all hung up on about being zero. The -∫E.dl term is zero, but it's not the only term: The remaining term, -L(dI/dt), is not zero, and he's ignoring that.
Belcher:  In that sense, you correctly argue that KVL holds. Just keep in mind that because the inductor is so different from the other elements, it is defined differently. But it's still a unit of volts, it's still a real measurable -- and lumpable voltage for the sake of KVL.

Mehdi shocks himself and burns up some more wires.





Also, Hey, did you see my request to you here? https://www.eevblog.com/forum/amphour/562-electroboom!/msg3820280/#msg3820280 (https://www.eevblog.com/forum/amphour/562-electroboom!/msg3820280/#msg3820280)

Edit: Fixed the link.
Title: Re: #562 – Electroboom!
Post by: jesuscf on November 19, 2021, 12:16:24 am
McDonald: ... He uses diversion and an obsolete version of a "law" to create an apparent paradox.

That is the root of the controversy.  Everything else is noise.

Lewin thinks that this is the definition of KVL:

\$\oint_{}^{}E.dL=0\$

While this is the accepted definition of KVL (since at least 1909, the earliest reference I found, but must likely older than that):

\$emf=\oint_{}^{}E.dL=-\frac{d }{dt}\int_{S}^{}B.dS\$

And before the usual blockheads start arguing with me, read the 5 pages I took from "Engineering Electromagnetics" by Hayt and Buck (8th edition) where you can find a better explanation of what I wrote above.  In one of the paragraphs, the authors even point out the need for an updated definition of KVL (despite Lewin saying than nobody does it, and it is crime against humanity or something dramatic along those lines):

"Equation (20) is true for static fields, but we will see in Chapter 9 that Faraday demonstrated it was incomplete when time-varying magnetic fields were present. One of Maxwell’s greatest contributions to electromagnetic theory was in showing that a time-varying electric field produces a magnetic field, and therefore we should expect to find later that Eq. (20) is not correct when either E or the magnetic field varies with time."

.
.
.

"Equation (20) is therefore just a more general form of Kirchhoff’s circuital law for voltages, more general in that we can apply it to any region where an electric field exists and we are not restricted to a conventional circuit composed of wires, resistances, and batteries. Equation (20) must be amended before we can apply it to time-varying fields."

It is the amended equation (the one that includes Faraday law) that nowadays is known as KVL.









Title: Re: #562 – Electroboom!
Post by: Sredni on November 19, 2021, 12:17:56 am
Mehdi talked about a LUMPED circuit in his first video, he applied (extended) KVL to it and said "KVL holds".
So if the one circuit which Lewin and Mehdi agreed about was not even an argument, then why the tarzan did Belcher say "As ARGUED by Mehdi?"

Here, read and learn your own language:

https://www.thefreedictionary.com/argue (https://www.thefreedictionary.com/argue)

Look at the usage examples and don't stop to celebrate once you find your confirmation bias.

Quote
So you're saying that Mehdi argued that KVL holds for a bunch of circuits including lumpable and unlumpable, and Belcher only agreed with the argument for Mehdi's lumpable circuit?
(but neglected to say that he disagreed with any of the rest..)

That sentence written by Belcher is explicitly referencing the RLC lumped circuit. It comes after a sentence that explicitly includes the equation of that lumped RLC circuit. And it is closed by a subordinate that explicitly references the other three elements of that lumped RLC circuit.
My take is that Belcher was talking arguing about that lumped RLC circuit.

But in the universe where Spock has grown a goatee... who knows?
Title: Re: #562 – Electroboom!
Post by: Sredni on November 19, 2021, 12:27:40 am
updated

About lumped and not lumped
Title: Re: #562 – Electroboom!
Post by: Sredni on November 19, 2021, 12:47:09 am
McDonald: ... He uses diversion and an obsolete version of a "law" to create an apparent paradox.

That is the root of the controversy.  Everything else is noise.

Lewin thinks that this is the definition of KVL:

\$\oint_{}^{}E.dL=0\$

While this is the accepted definition of KVL (since at least 1909, the earliest reference I found, but must likely older than that):

\$emf=\oint_{}^{}E.dL=-\frac{d }{dt}\int_{S}^{}B.dS\$

And before the usual blockheads start arguing with me, read the 5 pages I took from "Engineering Electromagnetics" by Hayt and Buck (8th edition)

You call others 'blockheads' but after 13 pages in this thread, and nearly forty in the previous one you have not yet understood that Hayt is talking about the 'amended KVL' that can be applied to circuits with inductors and that is not disputed here (if not for the semantics 5 + 3 + 0 = 8 vs 5 + 3 - 8 = 0 ).
Do you really think you are the only one to have read Hayt?
Title: Re: #562 – Electroboom!
Post by: bdunham7 on November 19, 2021, 12:52:14 am
So, what is generating the external electric field?  Is the circuit under test between the plates of a giant capacitor with a varying voltage applied to it?  I thought that in Lewin's experiment we had an varying external magnetic field generated by a coil.  That is why I am asking.

There is a long core with a varying magnetic flux at the center of the ring.  The magnetic field is only varying within the core.  I had pointed out that the return flux of the solenoid would generate a varying magnetic field as well, but apparently that has been shown both mathematically and experimentally to be very low.  I haven't seen or measured the apparatus myself, so I'll assume that is correct--even if there were such a contribution, it would be a lot lower than the contribution directly from the core.  So the circuit itself is not 'immersed' in a varying magnetic field of any significance.

This varying flux creates a rotational electric field concentrically around it.  This rotational field is said to have 'curl', and that makes it non-conservative, which essentially means that you can lose or gain energy when you go in a circle and come back to the same spot.  In a field with no curl, coming back to the same spot will always result in no net work--that is conservative.

Given your previous comments I would have assumed you already knew all that, so that's why I'm wondering why you are asking.  Have I misunderstood?

Title: Re: #562 – Electroboom!
Post by: jesuscf on November 19, 2021, 01:11:44 am
McDonald: ... He uses diversion and an obsolete version of a "law" to create an apparent paradox.

That is the root of the controversy.  Everything else is noise.

Lewin thinks that this is the definition of KVL:

\$\oint_{}^{}E.dL=0\$

While this is the accepted definition of KVL (since at least 1909, the earliest reference I found, but must likely older than that):

\$emf=\oint_{}^{}E.dL=-\frac{d }{dt}\int_{S}^{}B.dS\$

And before the usual blockheads start arguing with me, read the 5 pages I took from "Engineering Electromagnetics" by Hayt and Buck (8th edition)

You call others 'blockheads' but after 13 pages in this thread, and nearly forty in the previous one you have not yet understood that Hayt is talking about the 'amended KVL' that can be applied to circuits with inductors and that is not disputed here (if not for the semantics 5 + 3 + 0 = 8 vs 5 + 3 - 8 = 0 ).
Do you really think you are the only one to have read Hayt?

Really? So you are admitting that the very first post in this thread is incorrect?  Here it is for reference:

Dave, KVL doesn't hold under a varying magnetic field.
Title: Re: #562 – Electroboom!
Post by: Sredni on November 19, 2021, 01:37:56 am
You call others 'blockheads' but after 13 pages in this thread, and nearly forty in the previous one you have not yet understood that Hayt is talking about the 'amended KVL' that can be applied to circuits with inductors and that is not disputed here (if not for the semantics 5 + 3 + 0 = 8 vs 5 + 3 - 8 = 0 ).
Do you really think you are the only one to have read Hayt?

Really? So you are admitting that the very first post in this thread is incorrect?  Here it is for reference:

Dave, KVL doesn't hold under a varying magnetic field.

Inside your circuit path. Or inside any loop (such as a measurement loop) you wanna apply the voltage loop rule to.
But if you can hide the flux inside an inductor and skip it, by considering your circuit path jumping at the terminals, then you end up with a circuit path that is flux free. There you can apply 'modified', 'extended', 'new', 'upgraded', 'enhanced' KVL by treating the surface integral on the rhs AS IF it were a path integral on the left. So, you go from 5 + 3 + 0 = 8 to 5 + 3 - 8 = 0 . And you can feel like you're in high school again.

But for circuits where it is not possible to 'hide' the flux inside a 'detached' component, like Lewins ring WHOSE CIRCUIT PATH IS REQUIRED TO RUN AROUND THE CHANGING FLUX - IT IS A GEOMETRIC CONSTRAINT IMPOSED BY DESIGN - you can no longer pull the 'extended KVL' trick. You have to fold back to Faraday.
Title: Re: #562 – Electroboom!
Post by: EEVblog on November 19, 2021, 01:56:40 am
Who was it again in the original KVL forum thread that deleted all their (excellent) contributions?
Title: Re: #562 – Electroboom!
Post by: jesuscf on November 19, 2021, 01:59:25 am
You call others 'blockheads' but after 13 pages in this thread, and nearly forty in the previous one you have not yet understood that Hayt is talking about the 'amended KVL' that can be applied to circuits with inductors and that is not disputed here (if not for the semantics 5 + 3 + 0 = 8 vs 5 + 3 - 8 = 0 ).
Do you really think you are the only one to have read Hayt?

Really? So you are admitting that the very first post in this thread is incorrect?  Here it is for reference:

Dave, KVL doesn't hold under a varying magnetic field.

Inside your circuit path. Or inside any loop (such as a measurement loop) you wanna apply the voltage loop rule to.
But if you can hide the flux inside an inductor and skip it, by considering your circuit path jumping at the terminals, then you end up with a circuit path that is flux free. There you can apply 'modified', 'extended', 'new', 'upgraded', 'enhanced' KVL by treating the surface integral on the rhs AS IF it were a path integral on the left. So, you go from 5 + 3 + 0 = 8 to 5 + 3 - 8 = 0 . And you can feel like you're in high school again.

But for circuits where it is not possible to 'hide' the flux inside a 'detached' component, like Lewins ring WHOSE CIRCUIT PATH IS REQUIRED TO RUN AROUND THE CHANGING FLUX - IT IS A GEOMETRIC CONSTRAINT IMPOSED BY DESIGN - you can no longer pull the 'extended KVL' trick. You have to fold back to Faraday.

Yes, we can use KVL (the current version, not the one from the mid 1800) if you account for every contributing induced voltage and every voltage drop.   In the attached figure, Lewin is claiming to measure voltage Vxy from the left with 'voltmeter' V1 and to measure voltage Vxy from the right  with 'voltmeter' V2.  He is amazed that the measurements are different, then he concludes that KVL doesn't work!  But in reality what he is measuring is voltage Vkm with 'voltmeter' V1 and voltage Vhj with 'voltmeter' V2.  He didn't add the induced voltages Va, Vb, Vc, and Vd to the circuit diagram as if they didn't exist.
Title: Re: #562 – Electroboom!
Post by: Sredni on November 19, 2021, 02:30:25 am
Who was it again in the original KVL forum thread that deleted all their (excellent) contributions?

The problem is that jesuscf had read them before I deleted them. Those contribution did not do any good to him, it appears (well, back then IIRC he was arguing that Lewin's ring was a transmission line). And anyway I collected them in my answers on Stack Exchange. Do you think he read them?
Anyway, I explained why I did what I did back then. There is no greater sin in my eyes than destroying something I put my time into, for free (even if it was collateral damage). Criticize all you want, no problem. Call me names, it's fun for me either. But if my efforts are done for nothing... The ferret giveth, the ferret taketh.

Anyway, I am not going to delete any of my last posts. I made an exception because it is a shame to see so many people misguided and the one person who tried to show them the beauty of EM vilified (Lewin), but also because I am literally intrigued by the the phenomena (self-bias, exclusion of critical voices, and mostly the general gaslighting that social media are perfecting in the last years - thumbs down, anyone? ).

I wonder if Mehdi (and I put Belcher into this as well, for not being more explicit) realize the damage he has done and is doing (not about Lewin, but the misconceptions that get amplified and propagated). McDonald, on the other hand has just put forward a legitimate approach (whose exposition I personally don't like, but that's just a matter of personal tastes), but maybe he could have avoided acting surprised of the 'paradox', considering the history of misunderstanding behind it. He practically ended up being the one spare climatologist that published a paper against climate change.


Title: Re: #562 – Electroboom!
Post by: jesuscf on November 19, 2021, 05:22:48 am
Who was it again in the original KVL forum thread that deleted all their (excellent) contributions?
The problem is that jesuscf had read them before I deleted them. Those contribution did not do any good to him, it appears (well, back then IIRC he was arguing that Lewin's ring was a transmission line).

I said what???  My posts are still there; I didn't erase them, so please let me know were I said that...  If I remember correctly I said that a long, long wire can be modeled as a transmission line.  Transmission lines are often modeled as lumped pi circuits.  I even pointed out that Hayt explicitly used KVL to develop such model.
Title: Re: #562 – Electroboom!
Post by: thinkfat on November 19, 2021, 08:28:32 am
Also, Hey, did you see my request to you here? https://www.eevblog.com/forum/amphour/562-electroboom!/msg3820280/#msg3820280 (https://www.eevblog.com/forum/amphour/562-electroboom!/msg3820280/#msg3820280)

Yes, I saw it. I'll give it a shot.

Since you've formulated an ideal transformer, the easiest way to calculate the result is to lump the resistance of the wires and use "reflection" to transform all resistors and voltage sources over to the secondary side. The result is attached.

I'd like to add, though, that this has nothing to do with what we're discussing here. This "assignment" is so remote from Physics that you need to know nothing about Faraday or Maxwell, just some basic rules of circuit theory. But that's what engineers are being taught today in their first or maybe second year: how to work with equivalences so that you don't need to get into the nitty gritty of the REAL WORLD you're demanding to model in every other post.
Title: Re: #562 – Electroboom!
Post by: Jesse Gordon on November 19, 2021, 10:06:21 am
McDonald: ... He uses diversion and an obsolete version of a "law" to create an apparent paradox.

That is the root of the controversy.  Everything else is noise.

Lewin thinks that this is the definition of KVL:

\$\oint_{}^{}E.dL=0\$

While this is the accepted definition of KVL (since at least 1909, the earliest reference I found, but must likely older than that):

\$emf=\oint_{}^{}E.dL=-\frac{d }{dt}\int_{S}^{}B.dS\$

Wait. You mean the Lewies on youtube have been badgering me for a year because Lewin was using a 110+ year outdated definition?  |O

*Gasp* Why couldn't they just tell me it was a definition game the whole time? :palm:

I guess that explains why they are so reluctant to actually answer practical real life type transformer questions.

And before the usual blockheads start arguing with me, read ....

The way they don't answer my questions, I thought they couldn't read! :-DD

So far most of them won't respond to my challenge (https://www.eevblog.com/forum/amphour/562-electroboom!/msg3820280/#msg3820280)
(Kudos to Thinkfat who did respond after a while!)

I want to know if they actually know this stuff better than me, so I made a problem for them to solve, so far silence on that topic from most of them.
Title: Re: #562 – Electroboom!
Post by: Jesse Gordon on November 19, 2021, 10:14:57 am
Also, Hey, did you see my request to you here? https://www.eevblog.com/forum/amphour/562-electroboom!/msg3820280/#msg3820280 (https://www.eevblog.com/forum/amphour/562-electroboom!/msg3820280/#msg3820280)

Yes, I saw it. I'll give it a shot.

Since you've formulated an ideal transformer, the easiest way to calculate the result is to lump the resistance of the wires and use "reflection" to transform all resistors and voltage sources over to the secondary side. The result is attached.

I'd like to add, though, that this has nothing to do with what we're discussing here. This "assignment" is so remote from Physics that you need to know nothing about Faraday or Maxwell, just some basic rules of circuit theory. But that's what engineers are being taught today in their first or maybe second year: how to work with equivalences so that you don't need to get into the nitty gritty of the REAL WORLD you're demanding to model in every other post.
(https://i.postimg.cc/vHF2XXjf/IMG-20211119-084936.jpg)

Thank you!

It does have to do with some of what we're talking about - it is related to the experiment with the copper loop secondary winding we did the other day.

But the point is to give you three an opportunity to demonstrate that you do know something about the topic.

Now let's see if the other two fellas come up with the same answer as you. As I said, once all three have shared their answer, I'll share mine. If they don't do so in a couple days, I'll just assume they don't know how to do it and share mine anyway.
Title: Re: #562 – Electroboom!
Post by: thinkfat on November 19, 2021, 11:05:57 am
Thank you!

It does have to do with some of what we're talking about - it is related to the experiment with the copper loop secondary winding we did the other day.

But the point is to give you three an opportunity to demonstrate that you do know something about the topic.

Now let's see if the other two fellas come up with the same answer as you. As I said, once all three have shared their answer, I'll share mine. If they don't do so in a couple days, I'll just assume they don't know how to do it and share mine anyway.

It has some resemblance to the copper loop experiment. But it is already so heavily simplified and distorted that it's useless to argue about what you actually measured and how.
I also think that what you'd learn from anyone being able to solve this "challenge" or not has anything to do with their understanding of Faradays Law, because, well, it's not relevant.

Anyway, my turn now:

See the attached arrangement. If it looks familiar, this is your EI-core setup, just with a toroidal core instead.

What voltages would you measure across the resistors? What voltages would you measure across the wires? Show the arrangement of your probe wires.

The actual value of R is not relevant, assume it is large enough to make the resistance of the wiring negligible. Assume the magnetic flux in the core corresponds to a value of "1V" induced in the secondary.
Also assume the length of the wires between the resistors is identical, so that it looks nice and symmetric (hint: it doesn't actually matter, I'm just a sucker for symmetric arrangements).
Title: Re: #562 – Electroboom!
Post by: bsfeechannel on November 19, 2021, 11:11:04 am

Or maybe it's like this:

Lewin: Hello, hello, hello! Today I am going to show you something amazing! Something so amazing that you will be telling your grandchildren about it!
Lewin: In fact, it is so amazing that this is probably the only time in your entire life that you will see it!

He then presents an air core transformer, which he has incorrectly drawn on the chalkboard, and declares:

Lewin: KVL IS FOR THE BIRDS! All the textbooks are wrong! Only I am the source of truth on this matter, and even my fellow professors think I CHEATED! That shows you what THEY do!! They must be all cheaters! Yes, the whole world is insane but me!

Mehdi shocks himself a few times to see if this newfound knowledge will sink in and make sense. But it doesn't, so like any seeker of truth worth his sparks, he shocks himself a few more times and maybe melts down some clip leads, and then gets busy doing experiments, and finds that Lewin was incorrectly probing.

Mehdi: Professor Lewin, good sir, I tried my best to understand this, but I think you're not probing correctly. What am I missing?

Lewin: FLAT EARTHER! FLAT EARTHER! I do not argue with flat earthers! I won't even watch your video, but you're just wrong! KVL IS FOR THE BIRDS!

Mehdi: Professor McDonald, Professor Belcher, I'm really trying my best to understand this, and my fingers hurt and it smells like burnt clip leads in here. It looks to me like when correctly probing, KVL holds fine. It looks like Professor Lewin did not account for the dB/dt in the loop formed by his volt meter leads, which he did not depict as being magnetically coupled to his loop under test. But he won't even watch my video. What am I missing here?

McDonald: Yeah, he probably can't stand to see you shocking yourself and burning up perfectly good clip leads.
McDonald: Also, Dr. Lewin is as much of a showman as you. He uses diversion and an obsolete version of a "law" to create an apparent paradox.
McDonald: But you are right, Lewin’s circuit is within the range of applicability of Kirchhoff’s loop equations, which
can be used to predict measurements by the “voltmeters” in the experiment.

Belcher: Lewin has fallen for a very common misconception where he thinks that the -L(dI/dt) voltage read by volt meters represents -∫E.dl through the inductor, but that's false. Feynman is very clear that while -∫E.dl through a superconductor is zero, the voltage difference across the inductor is defined as ∮E.dl  (which he says is EMF) -- and this is Faradays law -L(dI/dt).
Belcher: According to Feynman's definition, the sum of all the voltage differences around the circuit is zero (that is, KVL holds) +V - IR - Q/C - L(dI/dt)=0, but the first three terms here are the -∫E.dl through various circuit elements, and the last term has NOTHING to do with the -L(dI/dt) through the inductor, which is the part that Lewin is all hung up on about being zero. The -∫E.dl term is zero, but it's not the only term: The remaining term, -L(dI/dt), is not zero, and he's ignoring that.
Belcher:  In that sense, you correctly argue that KVL holds. Just keep in mind that because the inductor is so different from the other elements, it is defined differently. But it's still a unit of volts, it's still a real measurable -- and lumpable voltage for the sake of KVL.

Mehdi shocks himself and burns up some more wires.


I’d say you’d make an excellent Hollywood script  writer, if we didn’t know who invented all the myth above for the sole aim of boosting his own viewership, at the expense of truth. But we’re not buying it because serious science and engineering.

As you keep forwarding this misinformation, you’ll not advance a single bit in the understanding of the electromagnetic phenomenon, exactly as Mehdi declares every time, including in this very interview.

Quote
Also, Hey, did you see my request to you here? https://www.eevblog.com/forum/amphour/562-electroboom!/msg3820280/#msg3820280 (https://www.eevblog.com/forum/amphour/562-electroboom!/msg3820280/#msg3820280)

Your question is not aimed at honestly trying to understand the issue. If it were, you would have already heeded what we’ve written all along this thread.
Title: Re: #562 – Electroboom!
Post by: Jesse Gordon on November 19, 2021, 11:12:21 am
About lumped and not lumped

For the other forum fellows who have not yet deciphered your view: Jesse here, wants to treat the two red straight segment of wire in his setup as two distinct lumped transformers.

Are you saying that the two red wires passing through the two halves of my transformer core are not lumpable voltage sources?


Well, it is kinda lumpable, and if you can attach it to a circuit that can be shrunk (lumped) to a point (this requires that there be no flux caught inside), you can consider it a lumped component in a lumped circuit.
Problem is, you cannot have a spatially extended component along your lumped circuit path, so you at least need to have the terminals close together, ideally separated by an infinitesimal distance.
In this picture I used the terms lumpable, kinda lumpable and unlumpable

([url]https://i.stack.imgur.com/HXv9x.jpg[/url])
lumpable kindalumpable unlumpable

But maybe it would be preferable to use: lumped, lumpable, and unlumpable. ( It's always after you have scanned them that you notice these things...)

In order to make that lumpable system of yours
([url]https://i.postimg.cc/gjhfcHXZ/screenshot-10.png[/url])
Pic torus with straight wire and circuit missing a coil
link [url]https://i.postimg.cc/gjhfcHXZ/screenshot-10.png[/url]

into a lumped transformer you just have to solve the problem of how to put those terminals close together. How do you plan to do it? There are two options

Option 1: You modify your component to fit the circuit
Take one on the other side to almost meet the other, or nearly join them together halfway. Now you can actually see a voltage in the space between terminals.

([url]https://i.postimg.cc/g2mfdRYx/screenshot-11.png[/url])
Pic bent wire around torus: change the component to fit the circuit
link [url]https://i.postimg.cc/g2mfdRYx/screenshot-11.png[/url]

And if you stay outside your black box and you do not run around the flux it will be always the same. Notiche how the charges have moved from the conductor's extreme to the resistor.

Option 2: You modify your circuit to fit the component
You insist in calling that piece of wire a transformer? Well, suit yourself, I will not alter it at all. But I still need to put it into the circuit, so I make connections to its end terminals

([url]https://i.postimg.cc/7LLdJkJD/screenshot-12.png[/url])
Pic change the circuit to fit the component
link [url]https://i.postimg.cc/7LLdJkJD/screenshot-12.png[/url]

and now you have your component unaltered attached to your circuit. The circuit path (highlighted here by the dotted line) is now including the component and along with it, the flux lines. And since your circuit path encompasses a variable magnetic flux, the circulation of the electric field can no longer be zero. This means that the path integral of the electric circuit depends on the path and therefore, voltage is path dependent. Kirchhoff's loop law is for the birds.

There is also the 'wishful thinking' option:
You just ignore component and circuit boundaries and cram everything together, hoping for the best. Let's apply this to your circuit inside the EI transformer.
Here on the left is the circuit with an elastic mebrane on it. It is integer and pristine. Look how happy it is, singing one of Madonna's earliest greatest hits.
But then you put it inside the EI transformer. Do you really think you can do that without breaking the membrane? Look what you did to your little circuit's membrane:

([url]https://i.postimg.cc/65LmH3dt/screenshot-13.png[/url])
fig rammed to death
link [url]https://i.postimg.cc/65LmH3dt/screenshot-13.png[/url]

I really hope you had proof of previous consent, because you broke the law: Kirchhoff's law, to be specific.

([url]https://i.postimg.cc/xd7xkbSy/screenshot-17.png[/url])
Just look at the circuit path.
link [url]https://i.postimg.cc/xd7xkbSy/screenshot-17.png[/url]

The broken membrane is proof that your circuit cut the flux lines. KVL is for the birds. Also, since your circuit path is going around the changing flux with no way to exclude it, your circuit is unlumpable. Lewin would be proud of you.



Bonus Track: What's the deal with the straight partial 'turn'?
I believe I figured you out now.
You are a lumper who believes that when a piece of wire 'goes through the hole' in the transformer, it develops a voltage, that is nonzero, irregardless of the path it is computed on! As if the hole was some kind of portal to another dimension, like a stargate.
No. If it intercepts the induced electric field, the wire gets polarized, and you might see charges of opposite sign at the extremities, but the voltage between point A at one end and point B at the other end is not unique: it depends on the path.

Along all these paths, the voltage (i.e. the work done per unit charge that is compute as minus path integral of E.dl) is zero (just like with a polarized conductor in an electrostatic field - see my previous silent post)

([url]https://i.postimg.cc/mZ7XDDpC/screenshot-14.png[/url])
Fig straight wire and paths zero v
link  [url]https://i.postimg.cc/mZ7XDDpC/screenshot-14.png[/url]

Along this other type of paths, paths that together with the piece of wire form a loop that runs around the changing flux, the voltage is nonzero ( one turn emf, so to speak). There is your transformer.

([url]https://i.postimg.cc/HL70hZfX/screenshot-15.png[/url])
Fig straight wire and paths emf
link [url]https://i.postimg.cc/HL70hZfX/screenshot-15.png[/url]

But how can you make it lumped / part of a lumped circuit? Bend the conductor a little more so as to bring the terminals almost together, as shown above. Now you can insert it into a shrinkable circuit and, if the circuit path does not include variable flux you have a lumped circuit where KVL holds. You will see a jump in voltage at the component's terminals. The displaced charge is not there, though, it is at the resistor's boundaries if there is one, or facing the gap if it is open circuited.

Which side is the hole in the core?
Incidentally, this is the equivalent of what you want to consider a lumped transformer with the toroidal or EI cores,  but with the core in the form of an infinitely long cylinder where the primary is an infinitely long solenoid. So, you have a piece of wire, and the 'hole' is basically the whole space. Where is left of the hole, or the right of the hole? Is the wire inside the core, or outside of it?

([url]https://i.postimg.cc/qvWyyczR/screenshot-16.png[/url])
Pic straight wire solenoid
link [url]https://i.postimg.cc/qvWyyczR/screenshot-16.png[/url]

Do you still see a lumped transformer? Or in order to call that piece of polarized wire a transformer you first need to bend it and bring the terminals together in a way that the wire almost encircles the core?


First of all, let me say THANK YOU for drawing all those very nice diagrams, and laying out your post so nicely.
I really honestly am genuinely grateful.

I do have one question though, regarding:

But how can you make it lumped / part of a lumped circuit? Bend the conductor a little more so as to bring the terminals almost together, as shown above. Now you can insert it into a shrinkable circuit and, if the circuit path does not include variable flux you have a lumped circuit where KVL holds. You will see a jump in voltage at the component's terminals. The displaced charge is not there, though, it is at the resistor's boundaries if there is one, or facing the gap if it is open circuited.

(Bold mine.)

Do I understand correctly that the voltage across the resistor (and consequently the heat produced by the resistor) will change when I go from Option 1 configuration and Option 2 configuration?

For the sake of discussion, let's say I took a 5ft length of wire, passed it through the core once and formed a large round loop with it and my volt meter, and wrote that voltage down.
Then let's say I took that same wire and hugged the core tightly and twisted it all the way to the volt meter so it was a twisted pair from the core to the volt meter, and measured the volts:

Here's what I mean:

(https://i.postimg.cc/fTgyDNp0/20211119-030105.jpg)

Would the two measurements be different? or the same?

If V1 is different than V2, what is the general magnitude and sign of the difference? Are we talking like a few percent? or a part per million?

(The core is small, about 3 inches in it's longest dimension.)

In metric, the core's maximum dimension is roughly 8cm and the length of the wire is about 150cm.

So it's not like I have a HUGE core in mind.

Thank you very much!
Title: Re: #562 – Electroboom!
Post by: Sredni on November 19, 2021, 03:29:44 pm
Also, Hey, did you see my request to you here? https://www.eevblog.com/forum/amphour/562-electroboom!/msg3820280/#msg3820280 (https://www.eevblog.com/forum/amphour/562-electroboom!/msg3820280/#msg3820280)
Yes, I saw it. I'll give it a shot.

Please, don't. He's just trying to pollute the thread.
He probably pulled out an old high school problem he had solved, and now wants us to waste out time on it to prove what?
As if one could not google how to computed inductances values, or how to model a transformer with lumped parameters? I did this kind of exercises when I was seventeen, and I hated them. Especially the vector diagrams, they are so ugly and lack any kind of beauty.
Besides having nothing to do with the unlumpable Lewin's ring, his problem is not even interesting because he doesn't even model the hysterisis. And with all the simplification assumptions one has to make to get to a lumped model, he wants what? Ten significative digits?

Please, move the problem in the beginner section of the forum, if you want to solve it. Here it will only dilute the thread.

Edit: I had one sentence unfinished
Title: Re: #562 – Electroboom!
Post by: thinkfat on November 19, 2021, 04:01:27 pm
Yes, I saw it. I'll give it a shot.
Please, don't. He's just trying to pollute the thread.
He probably pulled out an old high school problem he had solved, and now wants us to waste out time on it to prove what?
As if one could not google how to computed inductances values, or how to model a transformer with lumped parameters? I did t
Besides having nothing to do with the unlumpable Lewin's ring, his problem is not even interesting because he doesn't even model the hysterisis. And with all the simplification assumptions one has to make to get to a lumped model, he wants what? Ten significative digits?

Please, move the problem in the beginner section of the forum, if you want to solve it. Here it will only dilute the thread.

Why? Problem and solution nicely demonstrate his level of expertise. That can't hurt.
Besides, the solution (trivial, 3-resistor circuit plus voltage source) and all two lines of Ohms law are already attached to my post for anyone wanting to calculate the result to 10 digits. I don't feel like wasting space for this in the beginner section.
Title: Re: #562 – Electroboom!
Post by: Sredni on November 19, 2021, 04:04:08 pm
About lumped and not lumped
For the other forum fellows who have not yet deciphered your view: Jesse here, wants to treat the two red straight segment of wire in his setup as two distinct lumped transformers.
Are you saying that the two red wires passing through the two halves of my transformer core are not lumpable voltage sources?

I see you are capable of quoting and entire post. Good. How about (re-)reading it, now? Because to answer this question I should rewrite it as it is.

Quote
Do I understand correctly that the voltage across the resistor (and consequently the heat produced by the resistor) will change when I go from Option 1 configuration and Option 2 configuration?

Of course you don't understand correctly.
Here is the sentence you highlighted. I will toss in the next sentence too because it's black friday month.

"You will see a jump in voltage at the component's terminals. The displaced charge is not there, though, it is at the resistor's boundaries if there is one, or facing the gap if it is open circuited."

Now, what makes you think that the component is the resistor? Especially when the sentence immediately after says "The displaced charge is not there, though, it is at the resistor's boundaries if there is one, or facing the gap if it is open circuited."

If "NOT THERE" means "AT THE RESISTOR'S", how can "there" mean "at the resistor's"?

Title: Re: #562 – Electroboom!
Post by: Sredni on November 19, 2021, 04:18:19 pm
Yes, I saw it. I'll give it a shot.
Please, don't. He's just trying to pollute the thread.

Please, move the problem in the beginner section of the forum, if you want to solve it. Here it will only dilute the thread.

Why? Problem and solution nicely demonstrate his level of expertise. That can't hurt.
Because it will uselessly waste space here.
Quote
Besides, the solution (trivial, 3-resistor circuit plus voltage source)

You need to model more than that. If you find it, have a look at K. C. A. Smith & R. E. Alley "Electrical Circuits, an introduction", chapter 4, section 9 "Single-phase power transformers". It has the simplest treatment of 'real' transformers I have ever seen.

He will drag you down in a series of dozens of post, arguing about any simplificative assumption he did not make in his high-school exercise, and defending any simplification he made, no matter how unrealistic that may be (like--- this looks like a welder's transformer and he's not considering saturation?). It will go on forever.

Remember, I have a crystal ball that almost never fails me.

Edit: grammar! and a little addition.
Title: Re: #562 – Electroboom!
Post by: thinkfat on November 19, 2021, 07:11:36 pm
You need to model more than that.
Nah, not really. Look at the constraints given (ideal core, infinite permeability, perfect coupling) and imagine what that does to the standard "non-ideal transformer" model. So much for "dealing with the real world".
Title: Re: #562 – Electroboom!
Post by: Jesse Gordon on November 19, 2021, 07:33:39 pm

Or maybe it's like this:

Lewin: Hello, hello, hello! Today I am going to show you something amazing! Something so amazing that you will be telling your grandchildren about it!
Lewin: In fact, it is so amazing that this is probably the only time in your entire life that you will see it!

He then presents an air core transformer, which he has incorrectly drawn on the chalkboard, and declares:

Lewin: KVL IS FOR THE BIRDS! All the textbooks are wrong! Only I am the source of truth on this matter, and even my fellow professors think I CHEATED! That shows you what THEY do!! They must be all cheaters! Yes, the whole world is insane but me!

Mehdi shocks himself a few times to see if this newfound knowledge will sink in and make sense. But it doesn't, so like any seeker of truth worth his sparks, he shocks himself a few more times and maybe melts down some clip leads, and then gets busy doing experiments, and finds that Lewin was incorrectly probing.

Mehdi: Professor Lewin, good sir, I tried my best to understand this, but I think you're not probing correctly. What am I missing?

Lewin: FLAT EARTHER! FLAT EARTHER! I do not argue with flat earthers! I won't even watch your video, but you're just wrong! KVL IS FOR THE BIRDS!

Mehdi: Professor McDonald, Professor Belcher, I'm really trying my best to understand this, and my fingers hurt and it smells like burnt clip leads in here. It looks to me like when correctly probing, KVL holds fine. It looks like Professor Lewin did not account for the dB/dt in the loop formed by his volt meter leads, which he did not depict as being magnetically coupled to his loop under test. But he won't even watch my video. What am I missing here?

McDonald: Yeah, he probably can't stand to see you shocking yourself and burning up perfectly good clip leads.
McDonald: Also, Dr. Lewin is as much of a showman as you. He uses diversion and an obsolete version of a "law" to create an apparent paradox.
McDonald: But you are right, Lewin’s circuit is within the range of applicability of Kirchhoff’s loop equations, which
can be used to predict measurements by the “voltmeters” in the experiment.

Belcher: Lewin has fallen for a very common misconception where he thinks that the -L(dI/dt) voltage read by volt meters represents -∫E.dl through the inductor, but that's false. Feynman is very clear that while -∫E.dl through a superconductor is zero, the voltage difference across the inductor is defined as ∮E.dl  (which he says is EMF) -- and this is Faradays law -L(dI/dt).
Belcher: According to Feynman's definition, the sum of all the voltage differences around the circuit is zero (that is, KVL holds) +V - IR - Q/C - L(dI/dt)=0, but the first three terms here are the -∫E.dl through various circuit elements, and the last term has NOTHING to do with the -L(dI/dt) through the inductor, which is the part that Lewin is all hung up on about being zero. The -∫E.dl term is zero, but it's not the only term: The remaining term, -L(dI/dt), is not zero, and he's ignoring that.
Belcher:  In that sense, you correctly argue that KVL holds. Just keep in mind that because the inductor is so different from the other elements, it is defined differently. But it's still a unit of volts, it's still a real measurable -- and lumpable voltage for the sake of KVL.

Mehdi shocks himself and burns up some more wires.


I’d say you’d make an excellent Hollywood script  writer, if we didn’t know who invented all the myth above for the sole aim of boosting his own viewership, at the expense of truth. But we’re not buying it because serious science and engineering.

As you keep forwarding this misinformation, you’ll not advance a single bit in the understanding of the electromagnetic phenomenon, exactly as Mehdi declares every time, including in this very interview.

Quote
Also, Hey, did you see my request to you here? https://www.eevblog.com/forum/amphour/562-electroboom!/msg3820280/#msg3820280 (https://www.eevblog.com/forum/amphour/562-electroboom!/msg3820280/#msg3820280)

Your question is not aimed at honestly trying to understand the issue. If it were, you would have already heeded what we’ve written all along this thread.

Hey, I thought your highly biased skit was pretty good too! Maybe we should make a movie?  :-DD

You say that you think I "keep forwarding this misinformation." If you think that's true, then why not try to set me straight in the most effective way of actually answering my questions so I can understand better? You dodge every question you can, and you avoid the rest.

While I have no degrees and I'm certainly not a math wiz, I have a nack for making videos that illustrate my point. As long as I think my point is true, I continue to do so, and then you have to deal with the people who see the videos.

Your best bet for solving that problem is to actually work with me, at my level, to show me exactly where I went wrong -- not pasting large sections of whatnot and then refusing to answer questions when I'm confused by what you're actually saying.

My challenge of the 100v:10v stepdown transformer was aimed at helping me understand who here knows what they are talking about.

Turns out only Thinkfat has a clue, the other two of you couldn't solve it. And that's what I wanted to know.
Title: Re: #562 – Electroboom!
Post by: Jesse Gordon on November 19, 2021, 07:36:14 pm
Yes, I saw it. I'll give it a shot.
Please, don't. He's just trying to pollute the thread.

Please, move the problem in the beginner section of the forum, if you want to solve it. Here it will only dilute the thread.

Why? Problem and solution nicely demonstrate his level of expertise. That can't hurt.
Because it will uselessly waste space here.
Quote
Besides, the solution (trivial, 3-resistor circuit plus voltage source)

You need to model more than that. If you find it, have a look at K. C. A. Smith & R. E. Alley "Electrical Circuits, an introduction", chapter 4, section 9 "Single-phase power transformers". It has the simplest treatment of 'real' transformers I have ever seen.

He will drag you down in a series of dozens of post, arguing about any simplificative assumption he did not make in his high-school exercise, and defending any simplification he made, no matter how unrealistic that may be (like--- this looks like a welder's transformer and he's not considering saturation?). It will go on forever.

Remember, I have a crystal ball that almost never fails me.

Edit: grammar! and a little addition.


Well your crystal ball failed you this time.

Thinkfat solved it correctly and with elegance.  Evidently he read the notes in the challenge and understood them and he came up with the same result as I did which also agrees 3 digits past the decimal with an online circuit simulator I tried (which only shows 3 digits past the decimal).

What exactly more needed to be modeled?
Title: Re: #562 – Electroboom!
Post by: Jesse Gordon on November 19, 2021, 08:00:46 pm
Also, Hey, did you see my request to you here? https://www.eevblog.com/forum/amphour/562-electroboom!/msg3820280/#msg3820280 (https://www.eevblog.com/forum/amphour/562-electroboom!/msg3820280/#msg3820280)

Yes, I saw it. I'll give it a shot.

Since you've formulated an ideal transformer, the easiest way to calculate the result is to lump the resistance of the wires and use "reflection" to transform all resistors and voltage sources over to the secondary side. The result is attached.

I'd like to add, though, that this has nothing to do with what we're discussing here. This "assignment" is so remote from Physics that you need to know nothing about Faraday or Maxwell, just some basic rules of circuit theory. But that's what engineers are being taught today in their first or maybe second year: how to work with equivalences so that you don't need to get into the nitty gritty of the REAL WORLD you're demanding to model in every other post.

Alright, the jury is in! bsfeechannel and Sredni couldn't solve it.

You solved it absolutely correctly, and much more elegantly than I did.

You know this stuff better than them, they should be listening to you, not you listening to them.

I solved it this way:

primary_voltage=100
resistor_ohms=0.2
primary_turns=100
secondary_turns=10
resistance_per_foot=0.001
feet_per_turn=1

primary_resistance=resistance_per_foot*feet_per_turn*primary_turns
secondary_resistance=resistance_per_foot*feet_per_turn*secondary_turns

resistor_volts=(primary_voltage*resistor_ohms)/(resistor_ohms*(primary_turns/secondary_turns)+secondary_resistance*(primary_turns/secondary_turns)+primary_resistance*(secondary_turns/primary_turns))

The circuitlab.com online simulator said 9.479v (I don't have an account there but they let you use it free for like 12 moves or something then they demand money.)

My answer is 9.4786729858
and your answer (10/.211)*.2=9.4786729858 as well.
(Rounded to 10 digits past the decimal in both cases.)

I now know that you actually do know more about transformers than I do!

Thank you!
Title: Re: #562 – Electroboom!
Post by: Jesse Gordon on November 19, 2021, 08:15:20 pm
He will drag you down in a series of dozens of post, arguing about any simplificative assumption he did not make in his high-school exercise, and defending any simplification he made, no matter how unrealistic that may be (like--- this looks like a welder's transformer and he's not considering saturation?). It will go on forever.

Remember, I have a crystal ball that almost never fails me.

Try holding that crystal ball up to your computer screen, see if it works like a magnifying glass, because you obviously hadn't read what I wrote in my challenge  :-DD

The fact is I very clearly stated that the core had infinite permeability and was ideal, that the coupling was perfect and ideal, and that the AC generator was ideal, and that the resistor was ideal.

It was NOT a hard problem to solve for someone who knows even a tiny bit more about transformers than I do.

I picked that level of complexity because it was right at my limit, so if someone solved it better than me I know they are better than me, and if they can't solve it then I know they are no better than me.

As it turns out, the one who has been more sincere and more willing to try and be helpful is the one who solved it. Interesting.

Speaking of crystal balls, I got a couple of those 200mm K9 glass balls for various science experiments, and they do make nice magnifying glasses. Works for woodburning too.
Title: Re: #562 – Electroboom!
Post by: Sredni on November 19, 2021, 08:31:37 pm
He will drag you down in a series of dozens of post, arguing about any simplificative assumption he did not make in his high-school exercise, and defending any simplification he made, no matter how unrealistic that may be
The fact is I very clearly stated that the core had infinite permeability and was ideal, that the coupling was perfect and ideal, and that the AC generator was ideal, and that the resistor was ideal.

I told ya.
Defending any simplification he made, no matter how unrealistic that may be.
The famous 'real world' where cores have infinite permeability, there is no hysteresis, no saturation, ideal coupling and the generators are perfect generators with no internal resistance whatsoever and infinite compliance that can drive even an inductance made of one turn without being loaded by the near zero reactance.

It's a strange 'real world' that in which you live in.
Now I am really wondering if you ever modeled a real transformer. But never mind. Can we consider this parentheses closed?
Do you feel better now? Has this helped you resetting to default so that you can bring up the same objections over and over again?

Because I tell you what: what I write, and the drawings I am making, I am not doing that for you. I know you are a lost cause.

Can we get back to Lewin's ring now, and to the simple one turn transformer you have yet to understand the working of?

Edit: clause ->cause
Title: Re: #562 – Electroboom!
Post by: Jesse Gordon on November 19, 2021, 09:16:14 pm
Anyway, my turn now:

See the attached arrangement. If it looks familiar, this is your EI-core setup, just with a toroidal core instead.

What voltages would you measure across the resistors? What voltages would you measure across the wires? Show the arrangement of your probe wires.

The actual value of R is not relevant, assume it is large enough to make the resistance of the wiring negligible. Assume the magnetic flux in the core corresponds to a value of "1V" induced in the secondary.
Also assume the length of the wires between the resistors is identical, so that it looks nice and symmetric (hint: it doesn't actually matter, I'm just a sucker for symmetric arrangements).
(https://i.postimg.cc/CMm4MXwV/IMG-20211119-112430.jpg)

Below: Starting at the top volt meter, going clockwise: 0v, -1/3v, -0v, 1/3v.

Obviously, the 2R is not just a resistor, it also serves as a secondary winding -- albeit a secondary winding with a high resistance -- like the 100V:10V transformer problem you solved for, except the primary winding has more resistance than the load resistor.

(https://i.postimg.cc/rs3cCg2F/20211119-122528.jpg)


And also, nobody has directly answered the following for me yet, do you mind clarifying? Thank you!

(https://i.postimg.cc/fTgyDNp0/20211119-030105.jpg)

Above: Will V1 and V2 read the same?
Title: Re: #562 – Electroboom!
Post by: Jesse Gordon on November 19, 2021, 09:20:57 pm
About lumped and not lumped
For the other forum fellows who have not yet deciphered your view: Jesse here, wants to treat the two red straight segment of wire in his setup as two distinct lumped transformers.
Are you saying that the two red wires passing through the two halves of my transformer core are not lumpable voltage sources?

I see you are capable of quoting and entire post. Good. How about (re-)reading it, now? Because to answer this question I should rewrite it as it is.

Quote
Do I understand correctly that the voltage across the resistor (and consequently the heat produced by the resistor) will change when I go from Option 1 configuration and Option 2 configuration?

Of course you don't understand correctly.
Here is the sentence you highlighted. I will toss in the next sentence too because it's black friday month.

"You will see a jump in voltage at the component's terminals. The displaced charge is not there, though, it is at the resistor's boundaries if there is one, or facing the gap if it is open circuited."

Now, what makes you think that the component is the resistor? Especially when the sentence immediately after says "The displaced charge is not there, though, it is at the resistor's boundaries if there is one, or facing the gap if it is open circuited."

If "NOT THERE" means "AT THE RESISTOR'S", how can "there" mean "at the resistor's"?

Look, I'm sure you'd make a great Hollywood scriptwriter. In fact, your sketches are so good you could make great storyboards too.

Your little stories are a lot of fun to read, but the problem is they are rather ambiguous to someone with my skill level. I'd think maybe I was just really dumb, but then when I try to get clarification with simple unambiguous questions, I get more drama, not answers.

Perhaps I misunderstood some of what you said in your nice long post with lots of pictures. But I'm doing the best I can. If you can spin all these great yarns and write and conduct a 3 page forum film, why not just answer the simple questions, like this:

(https://i.postimg.cc/fTgyDNp0/20211119-030105.jpg) 

Will V1 and V2 read the same? (Neglecting of course any non-uniformity of the core and neglecting also the added capacitance of the twisted pair.....)
Title: Re: #562 – Electroboom!
Post by: Sredni on November 19, 2021, 09:33:03 pm
Sigh...

YES. V1 and V2 will be the same.
Title: Re: #562 – Electroboom!
Post by: thinkfat on November 19, 2021, 09:37:58 pm
Obviously, the 2R is not just a resistor, it also serves as a secondary winding -- albeit a secondary winding with a high resistance -- like the 100V:10V transformer problem you solved for, except the primary winding has more resistance than the load resistor.
(https://i.postimg.cc/rs3cCg2F/20211119-122528.jpg)

I'm sorry, I understand I may need to clarify "R" and "2R". I hoped you'd understand that "2R" means that this resistor has a value that is "two times R". In other words, the second ("inner" if you will) resistor has double the resistance of the first ("outer") resistor. Would you correct your analysis?
Title: Re: #562 – Electroboom!
Post by: Jesse Gordon on November 19, 2021, 09:58:09 pm
(https://i.postimg.cc/fTgyDNp0/20211119-030105.jpg)
Sigh...

YES. V1 and V2 will be the same.

So the V1 configuration and the V2 configuration are functionally identical, and yet  V2 is lumpable and V1 is not?

Do I understand correctly that if I had a loop formed by resistors and either V1 or V2, that I could use the V2 secondary and KVL holds, but if instead I used the V1 configuration, then KVL fails?

Does KVL fail because the volt meter readings around the path would no longer sum to zero? or because of a technicality of the definition of voltage that changes when those two wires aren't twisted together (or at least run very close to eachother)?

Thank you!
Title: Re: #562 – Electroboom!
Post by: Jesse Gordon on November 19, 2021, 10:19:54 pm
Obviously, the 2R is not just a resistor, it also serves as a secondary winding -- albeit a secondary winding with a high resistance -- like the 100V:10V transformer problem you solved for, except the primary winding has more resistance than the load resistor.
(https://i.postimg.cc/rs3cCg2F/20211119-122528.jpg)

I'm sorry, I understand I may need to clarify "R" and "2R". I hoped you'd understand that "2R" means that this resistor has a value that is "two times R". In other words, the second ("inner" if you will) resistor has double the resistance of the first ("outer") resistor. Would you correct your analysis?

Thank you for checking, but no, I understood that 2R is a resistor with twice the resistance of R.

As long as you are OK with how I've drawn the toroid core as a cut-away strictly for clarity (but assume it's a full toroid)  and the physical locations and polarities of my 4 volt meters and the physical path of their leads and the physical location of the two resistors,  then I see no reason to change my analysis.

I honestly believe that if I were to set up an actual test and measure with a real volt meter as shown in my diagram above, I would get the indicated readings. (Of course the sign indicates phase, not actual DC voltage offset.)

EDIT PS: I'm super curious about what your prediction is...! Any chance you might tell me? I put my prediction on the line.. /edit

Thank you!

Sednri says that both V1 and V2 would read the same, in the following diagram, do you agree?:

(https://i.postimg.cc/fTgyDNp0/20211119-030105.jpg)
Title: Re: #562 – Electroboom!
Post by: Sredni on November 19, 2021, 10:59:20 pm
(https://i.postimg.cc/fTgyDNp0/20211119-030105.jpg)
Sigh...
YES. V1 and V2 will be the same.
So the V1 configuration and the V2 configuration are functionally identical, and yet  V2 is lumpable and V1 is not?

Do I understand correctly that if I had a loop formed by resistors and either V1 or V2, that I could use the V2 secondary and KVL holds, but if instead I used the V1 configuration, then KVL fails?

You might have noticed that in my posts I sometimes write "circuit path" and "in the circuit" in bold, or in italics.
So, let's try this.
Pick a colored pen or pencil and draw a finely dotted line following the circuit path for both circuits, following the examples I gave in my "About Lumped and non Lumped" post. Then select two points A and B at random on both circuits - in the same position for both.

Post it, and then I will show you when and where the voltage can be non unique and KVL dies.

Edit: plurals!
Title: Re: #562 – Electroboom!
Post by: Jesse Gordon on November 20, 2021, 12:32:38 am
(https://i.postimg.cc/fTgyDNp0/20211119-030105.jpg)
Sigh...
YES. V1 and V2 will be the same.
So the V1 configuration and the V2 configuration are functionally identical, and yet  V2 is lumpable and V1 is not?

Do I understand correctly that if I had a loop formed by resistors and either V1 or V2, that I could use the V2 secondary and KVL holds, but if instead I used the V1 configuration, then KVL fails?

You might have noticed that in my posts I sometimes write "circuit path" and "in the circuit" in bold, or in italics.
So, let's try this.
Pick a colored pen or pencil and draw a finely dotted line following the circuit path for both circuits, following the examples I gave in my "About Lumped and non Lumped" post. Then select two points A and B at random on both circuits - in the same position for both.

Post it, and then I will show you when and where the voltage can be non unique and KVL dies.

Edit: plurals!

Why are you so reluctant to answer my questions? That's how I learn best -- understand how reality is, then learn why. You're trying to tell me why without even showing that we're talking about the same reality.

I saw your very nice diagram that showed dots on the loop which did not contain within its area dB/dt (lumped) and I saw how you had dots all through the whole loop when the wires did not come near eachother between the transformer and volt meter (unlumpable).

So consider my entire V1 path to have dots if that's what you're asking.

And now you say that the voltage can be non-unique. What do you mean? Do you mean that for configuration V1, my volt meter can read different voltages depending on something?
Or do you mean that the voltage is non-unique in that it's also the same voltage measured in V2? non-unique means there could be another one like it.
But another what? Another location/configuration? Or another voltage?

Are you talking about measured voltages being non-unique? or calculated voltages?

Every time I ask questions, none of them are asked and more nebulous statements are given which only raise more questions.

Can we just agree that in my V1 and V2 case, FOR THE CASE OF ACTUALLY MEASURING WITH A VOLT METER, KVL will appear to hold, in that the measured values will all sum to zero?

Or are you saying that using V1 in a loop would NOT sum to zero as measured with a volt meter?

Then you can go on to explain why, by some definition or technicality, KVL isn't actually holding, even though my experiments with a volt meter make it look like it's holding.

Thank you.
Title: Re: #562 – Electroboom!
Post by: jesuscf on November 20, 2021, 12:57:22 am
So, what is generating the external electric field?  Is the circuit under test between the plates of a giant capacitor with a varying voltage applied to it?  I thought that in Lewin's experiment we had an varying external magnetic field generated by a coil.  That is why I am asking.

There is a long core with a varying magnetic flux at the center of the ring.  The magnetic field is only varying within the core.  I had pointed out that the return flux of the solenoid would generate a varying magnetic field as well, but apparently that has been shown both mathematically and experimentally to be very low.  I haven't seen or measured the apparatus myself, so I'll assume that is correct--even if there were such a contribution, it would be a lot lower than the contribution directly from the core.  So the circuit itself is not 'immersed' in a varying magnetic field of any significance.

This varying flux creates a rotational electric field concentrically around it.  This rotational field is said to have 'curl', and that makes it non-conservative, which essentially means that you can lose or gain energy when you go in a circle and come back to the same spot.  In a field with no curl, coming back to the same spot will always result in no net work--that is conservative.

Given your previous comments I would have assumed you already knew all that, so that's why I'm wondering why you are asking.  Have I misunderstood?

What you wrote there is not describing Lewin's experiment at all!
Title: Re: #562 – Electroboom!
Post by: Sredni on November 20, 2021, 12:58:15 am
never mind.
here's the picture.

(https://i.postimg.cc/15n8XGXJ/Voltage-can-be-path-dependent.jpg)
https://i.postimg.cc/15n8XGXJ/Voltage-can-be-path-dependent.jpg

(I cannot see the pics from postimg from this system, so I don't know if you posted yours yet. Incidentally this is the reason I post the links under every picture - so that I can know there is one)

Ok. You did not post any image, but it's fine. Now, the colored paths are the paths along which you compute the voltage. Voltage is a path integral: you move on a path, consider a small displacement dl and compute the scalar product of E.dl - keep summing up for all infinitesimal intervals the path has been partioned into and then add a minus sign to get the voltage.
This IS the voltage a voltmeter with infinite impedance and zero resistivity probes will measure if the probes followed one of those colored paths and the voltmeter was somewhere along it (ideally without occupying any space).

(I tossed in a few 'external paths, for free)

If the dB/dt zone is OUTSIDE your circuit's premises, voltages along any path between two points A and B will be the same.
If the dB/dt zone in INSIDE your circuit's premises, the paths that cut through it or go 'on the other side' will see different contributes from the flux than those that do not cut it and are on the 'good side'.

Do I have to explain why, if the voltage along two different paths joining the same points A and B are different, then KVL dies?
Title: Re: #562 – Electroboom!
Post by: Jesse Gordon on November 20, 2021, 02:34:28 am
never mind.
here's the picture.

(https://i.postimg.cc/15n8XGXJ/Voltage-can-be-path-dependent.jpg)
https://i.postimg.cc/15n8XGXJ/Voltage-can-be-path-dependent.jpg

(I cannot see the pics from postimg from this system, so I don't know if you posted yours yet. Incidentally this is the reason I post the links under every picture - so that I can know there is one)

Ok. You did not post any image, but it's fine. Now, the colored paths are the paths along which you compute the voltage. Voltage is a path integral: you move on a path, consider a small displacement dl and compute the scalar product of E.dl - keep summing up for all infinitesimal intervals the path has been partioned into and then add a minus sign to get the voltage.
This IS the voltage a voltmeter with infinite impedance and zero resistivity probes will measure if the probes followed one of those colored paths and the voltmeter was somewhere along it (ideally without occupying any space).

(I tossed in a few 'external paths, for free)

If the dB/dt zone is OUTSIDE your circuit's premises, voltages along any path between two points A and B will be the same.
If the dB/dt zone in INSIDE your circuit's premises, the paths that cut through it or go 'on the other side' will see different contributes from the flux than those that do not cut it and are on the 'good side'.

Do I have to explain why, if the voltage along two different paths joining the same points A and B are different, then KVL dies?

Sorry, I had no idea you couldn't see postimg inlines. This is the "V1, V2" diagram I've been referencing, I thought you could see it: https://i.postimg.cc/fTgyDNp0/20211119-030105.jpg
If I ever say "In the diagram below" and you don't see a diagram, please let me know!!!!

Of course I know that if an unambiguous voltage reading cannot be obtained across the terminals of an element due to either the element or the instrument it  will make not only KVL fail but all analyses of all types which would depend on an unambiguous reading. Like I've been saying all along, if you're measuring two unknowns, you're measuring neither of them.


So I look at your diagram, are we still talking about a toroid?

It looks like you're showing the wire passing through the physical volume of the cross section of the core, as if there were holes drilled in the core..... Or did we leave off talking about toroid transformers and I didn't notice? If so, what are we looking at here? an air core transformer?

OF COURSE if you drill holes in your toroid cross section and run your wire through those drilled holes you will get various fractions of a turn of voltage differences.
(That's why I was joking about drilling a hole in a toroid to get a half-turn as a joke a week ago.)

It's the same if you added or removed turns from your secondary between measuring it's voltage and measuring voltage across other components in the loop.

Please explain, are we still talking about a toroid?

If we are talking about a toroid, then are you saying that the only ambiguity is if the volt meter leads have some magical property which allows them to occupy the exact same physical space as the core material?

Would it be fair then to say that in the real world, where wires and ferrite/iron cannot exist in the same physical space, that we don't have to worry about our volt meter leads accidentally ending up half way through the cross section of a toroid, and thus in the real world, KVL holds fine with the outputs of toroid transformers in both the V1 and V2 configurations in my diagram? https://i.postimg.cc/fTgyDNp0/20211119-030105.jpg

(https://i.postimg.cc/fTgyDNp0/20211119-030105.jpg)

Can we agree that in my diagram above, both V1 and V2 configurations using REAL PHYSICAL TOROIDS AND WIRES, and real volt meters, KVL would give every appearance of holding in real life?

Once we agree on observable reality then we can talk about what and why behind it. But talking about what and why is meaningless if we aren't even talking about the same observable points of evidence.

Thank you.

EDIT: PS: If you're saying that my volt meter reading can change due to the changing of the path of the wires OUTSIDE THE CORE(I can't believe I have to say that..), then I need to know that so I can test it out and see for myself. Thanks!
Title: Re: #562 – Electroboom!
Post by: Sredni on November 20, 2021, 03:36:34 am
I can see the images just fine from other systems. I use postimg. But I cannot see them from THIS system.
You seem to lack the capacity to discern between levels of gray. It's either all white or all black.
It feels like being in that 70s movie, "The day of the dolphin".

Yes, I was talking about the toroid because that is the way we chose to steer the flux. In the case matter is in the way we can still compute voltage as path integral, by knowing the field. Don't like the toroid in the way? Make a gap. Slide your circuit in there through the gap (remember the elastic membrane? It will be rammed by the flux just like before (yes, yes, there will be fringing, I don't care, what counts it's the flux inside the circuit). Don't like the gap? Use a magnet and shoot it through your circuit - there will be flux lines and we are there.
Now, let me guess, the next objection will be "but if I shoot a magnet through the circuit, how can I be so fast to read the voltmeter?"
So, use a solenoid.
You built a pancake solenoid to put under your Lewin ring, right? You put the hands of the clock in a way that you can access the space where the magnetic field change, right? I would tell you to do the experiment with that, but your system has so much flux leakage that you can only try the orange paths. In fact, your clock hands are examples of orange paths, with the voltmeter at the center.

What is the next objection? That you cannot put a voltmeter inside a circuit branch with a resistor because you cannot drill a hole along the conductor and the resistor and there are no microscopic voltmeters to fit such a tight space?

Do you really think that measurement instruments only works by implementing verbatim the definition of the variable they measure?
Do amperometers count the single electrons and use a tiny stopwatch to measure the current?
Do they necessarily have to sit inside the branch they measure current? Ever seen a current clamp? Never heard of Hall effect?

It is fascinating to see the amount of irrational objection you need to throw to persist in your religious blindness.

-
Anyway, take one green path, and one purple path, where you can place your voltmeter even if there is a torus. Along one you read 5V and along the other your read 0V. KVL dies there. Wanna see in the circuit: the branch with the resistor is a green path, the branch with the conductor alone is the purple path. One gives 5V and one gives 0V - and you can show that it has to be that way because if you compute the fields, these values are what you get by computing the path integrals.
Everything checks.

And what is the sum of voltages if you circle the ring? 5V + 0V = 5V, or circulation of E along the ring = EMF.

KVL says: circulation of E along the ring = 0
Faradays says: circulation of E along the ring = EMF

I wonder who is right.
Title: Re: #562 – Electroboom!
Post by: bsfeechannel on November 20, 2021, 04:21:48 am
My challenge of the 100v:10v stepdown transformer was aimed at helping me understand who here knows what they are talking about.

OK. So you want an answer to your question? Here it is: stop trying to understand who here knows what they are talking about. There's no guru in science. Mehdi said that he wished Feynman was alive to answer his questions about electromagnetism. Turns out we don't need Feynman. We have nature. Nature told Feynman what he knew and it is apt to answer whatever questions we may have.

You KVLers are obsessed with ascertaining who is wrong and who is right. Forget about that.

Who says KVL doesn't always hold is nature, not Lewin.

There can be no electric fields inside a static conductor, except the one to sustain a voltage as a function of its current as per Ohm's law. This is one of the first things we learn in whatever electromagnetism course out there.

The conclusions of your experiment are wrong because you don't listen to what the phenomenon right in front of you is telling. The only thing you listen to is "Lewin is wrong so I must be right". You ain't gonna get anywhere that way.
Title: Re: #562 – Electroboom!
Post by: Jesse Gordon on November 20, 2021, 05:04:24 am
I can see the images just fine from other systems. I use postimg. But I cannot see them from THIS system.
You seem to lack the capacity to discern between levels of gray. It's either all white or all black.
It feels like being in that 70s movie, "The day of the dolphin".

Yes, I was talking about the toroid because that is the way we chose to steer the flux. In the case matter is in the way we can still compute voltage as path integral, by knowing the field. Don't like the toroid in the way? Make a gap. Slide your circuit in there through the gap (remember the elastic membrane? It will be rammed by the flux just like before (yes, yes, there will be fringing, I don't care, what counts it's the flux inside the circuit). Don't like the gap? Use a magnet and shoot it through your circuit - there will be flux lines and we are there.
Now, let me guess, the next objection will be "but if I shoot a magnet through the circuit, how can I be so fast to read the voltmeter?"
So, use a solenoid.
You built a pancake solenoid to put under your Lewin ring, right? You put the hands of the clock in a way that you can access the space where the magnetic field change, right? I would tell you to do the experiment with that, but your system has so much flux leakage that you can only try the orange paths. In fact, your clock hands are examples of orange paths, with the voltmeter at the center.

What is the next objection? That you cannot put a voltmeter inside a circuit branch with a resistor because you cannot drill a hole along the conductor and the resistor and there are no microscopic voltmeters to fit such a tight space?

Do you really think that measurement instruments only works by implementing verbatim the definition of the variable they measure?
Do amperometers count the single electrons and use a tiny stopwatch to measure the current?
Do they necessarily have to sit inside the branch they measure current? Ever seen a current clamp? Never heard of Hall effect?

It is fascinating to see the amount of irrational objection you need to throw to persist in your religious blindness.

-
Anyway, take one green path, and one purple path, where you can place your voltmeter even if there is a torus. Along one you read 5V and along the other your read 0V. KVL dies there. Wanna see in the circuit: the branch with the resistor is a green path, the branch with the conductor alone is the purple path. One gives 5V and one gives 0V - and you can show that it has to be that way because if you compute the fields, these values are what you get by computing the path integrals.
Everything checks.

And what is the sum of voltages if you circle the ring? 5V + 0V = 5V, or circulation of E along the ring = EMF.

KVL says: circulation of E along the ring = 0
Faradays says: circulation of E along the ring = EMF

I wonder who is right.


I don't know what you mean when you say "THIS system." Your computer? Your internet system? The EEVBlog system? Your Screen reader? Do you have one of those qwerty monitors?  :-DD

It is fascinating to see the amount of irrational objection you need to throw to persist in your religious blindness.

Actually you're describing yourself. Do you know how many times I've specifically clearly asked you whether configuration V1 and V2 would cause KVL to appear to hold when measured with a volt meter as described in the following diagram? https://i.postimg.cc/fTgyDNp0/20211119-030105.jpg

You bob and weave and rant, but you will not answer that question. It's like trying to get a pious priest to cut lose a string of profanities from the podium.
You really seem to have a religious aversion to admitting that KVL *WOULD* give every appearance of holding for both my V1 and V2 configurations as measured with a volt meter.

I even keep giving you the "out" of saying that KVL is only appearing to hold in this situation. You can just say "Yes it appears to hold, but...." then we can talk about the but.

Look at your leadin:

Do you really think that measurement instruments only works by implementing verbatim the definition of the variable they measure?
Do amperometers count the single electrons and use a tiny stopwatch to measure the current?
Do they necessarily have to sit inside the branch they measure current? Ever seen a current clamp? Never heard of Hall effect?

From that I would say that you do know that KVL would give every appearance of holding, but you can't bring yourself to say it, so you're preparing to argue that even though KVL appears to be holding, it's not actually holding because the meters aren't measuring what I think they are measuring.

Since you asked, I have 5  AC/DC amp clamps ranging from a 2 amp to a 2000 amp unit. I've also worked with hall effect sensors in various projects over the years.
I also have 4 CRT type analog oscilloscopes, and I think the 3 Tektronix ones of them work. The 4th is an old heathkit that was all tube based, it doesn't work. All of them are statically deflected, which is interesting because that actually does measure voltage without an electrical current flowing across the measuring element - the beam of electrons is steered due to the current-less electric field.
Also 2 junky Siglent 2GGS/s DSO scopes, a 2 chan and a 4 chan.

I once offered to send one of the old Tek units for free to a guy in Germany if he paid shipping, he was happy till he found out how much it cost to ship a 50lb large chunk of vintage metal equipment LOL.

Why won't you just admit that KVL will appear to hold in my V1 and V2 configurations in the above linked diagram if measured with actual volt meters, then we can move on to why it's not?
If you haven't the ability to solve a simple ideal transformer step down problem, and you haven't the honesty to admit that KVL will appear to work in a case where you know it will appear to work, then how can I trust you to tell me anything else straight?

Thing is, I can hook up some toroid transformers, make a loop, and measure around the loop, and KVL will appear to hold. So can you. Why not admit it?

Once I see that you have the integrity to admit to observable reality, then you can start telling me what's going on behind that observable reality.

But so far, your entire performance looks to me wild savage religious dance to try and save your Lewin.

As for me, it's not religious. It's just me sayin' Hey, I got volt meters and transformers, I made a loop and measured around the loop and summed the voltages, and it all summed to zero.

You think you can show me that KVL actually fails int he real world with toroidal transformer secondaries? Then for pete's sake DO IT! Otherwise, stop telling me it fails in that situation.

Anyway, take one green path, and one purple path, where you can place your voltmeter even if there is a torus. Along one you read 5V and along the other your read 0V. KVL dies there. Wanna see in the circuit: the branch with the resistor is a green path, the branch with the conductor alone is the purple path. One gives 5V and one gives 0V - and you can show that it has to be that way because if you compute the fields, these values are what you get by computing the path integrals.
Everything checks.

And what is the sum of voltages if you circle the ring? 5V + 0V = 5V, or circulation of E along the ring = EMF.

I'm not exactly sure what topology you have in mind for me to try. I could take some guesses, but wouldn't you know it, KVL would appear to work and you'd say I did it wrong.

So let's do it this way.

I have two nice EI-Core transformers with 200mv/turn, or 100mv/half-turn. If  you want me to use a toroid, I can just use half of them.

Or if I have to use a toroid I can get one off ebay.

Please draw me the circuit topology that you want me to test, the one that you think will fail KVL.

Thank you.



Title: Re: #562 – Electroboom!
Post by: jesuscf on November 20, 2021, 05:32:46 am
KVL says: circulation of E along the ring = 0
Faradays says: circulation of E along the ring = EMF

Suuure! Because when Kirchhoff postulated his 'voltage law', he didn't have a emf in the circuit.  Are you even aware that there are more than one way of generating an emf and using it in a circuit?
Title: Re: #562 – Electroboom!
Post by: Sredni on November 20, 2021, 05:37:11 am

You really seem to have a religious aversion to admitting that KVL *WOULD* give every appearance of holding for both my V1 and V2 configurations as measured with a volt meter.


Inner peace.

Inner peace.



Inner peace.


Ok, let's see...

Is FIVE equal to ZERO?

Because if five equals zero, then KVL holds in the nonlumped circuit.
If five is not equal to zero, then KVL does not hold in the nonlumped circuit.
I say that five is not equale to zero, and therefore KVL does not hold in the nonlumped circuit.

So, question - and only this question I would like you to answer:

Is FIVE equal to ZERO?

Title: Re: #562 – Electroboom!
Post by: Sredni on November 20, 2021, 05:52:37 am
KVL says: circulation of E along the ring = 0
Faradays says: circulation of E along the ring = EMF

Suuure! Because when Kirchhoff postulated his 'voltage law', he didn't have a emf in the circuit.  Are you even aware that there more more than one way of generating an emf and using it in a circuit?

Kirchhoff experimented with lumped sources of EMF. Batteries, that are localized in the circuit. In his statement he used the words "auf dem Wege", which means along the way, on the path. Can you locate it on the path, anywhere?

The EMF due to a changing magnetic field is essentially a relativistic effect, and it does not appear anywhere in the circuit. And it is also the manifestation of an actual electric field, the electric field Eind that causes charges to move (edit: I am talking about the surface and interface charge that follows the gradients in permeability and conductivity). It is not just something that happens to have dimensions of a voltage, it is actually a 'component' of the quantity voltage. That electric field, the induced electric field Eind becomes a component of the total electric field and, together with the conservative field Ecoul of the displaced charge, result in the field inside the material that follows Ohm's law (in its local form).
And since electrons only experience the total electric field, there is no way for them to know which portion came from the displaced charges and which came from the changing magnetic field.
You will never find the emf along the circuit path, like the lumped EMF Kirchhoff experimented with.

The EMF from a changing magnetic field is the only one to appear on the rhs - it's a direct consequence of Faraday's law, one of Maxwell's equations that expresses a fundamental property of the electromagnetic field.
Title: Re: #562 – Electroboom!
Post by: Jesse Gordon on November 20, 2021, 05:55:18 am
My challenge of the 100v:10v stepdown transformer was aimed at helping me understand who here knows what they are talking about.

OK. So you want an answer to your question? Here it is: stop trying to understand who here knows what they are talking about. There's no guru in science. Mehdi said that he wished Feynman was alive to answer his questions about electromagnetism. Turns out we don't need Feynman. We have nature. Nature told Feynman what he knew and it is apt to answer whatever questions we may have.

You KVLers are obsessed with ascertaining who is wrong and who is right. Forget about that.

Who says KVL doesn't always hold is nature, not Lewin.

There can be no electric fields inside a static conductor, except the one to sustain a voltage as a function of its current as per Ohm's law. This is one of the first things we learn in whatever electromagnetism course out there.

The conclusions of your experiment are wrong because you don't listen to what the phenomenon right in front of you is telling. The only thing you listen to is "Lewin is wrong so I must be right". You ain't gonna get anywhere that way.

I can't help but chuckle at the contradiction you present.

On one hand, you're telling me we don't need no gurus because nature tells us.

Then you end off talking about what the first thing you learned in electromagnetism courses.

So which is it? Nature? Or the gurus that write the books and teach the courses?

All I'm asking is if you think there is a loop topology involving the output windings of regular closed-magnetic-circuit transformers as elements in a multi-element loop which cause KVL to fail, I want to know about it because I want to build such a loop and see KVL fail it with my own eyes.

There can be no electric fields inside a static conductor, except the one to sustain a voltage as a function of its current as per Ohm's law.

And supposing the conductor is in a changing magnetic field? Why are you bringing up static fields when we're talking about AC transformers?
The fact that static electric fields cannot cause a sustained voltage along a superconductor simply does not mean that a dynamic magnetic field cannot cause a dynamic voltage difference per Faradays Law across the ends of the conductor.

As best as I can tell, both Dr. Belcher and Dr. McDonald disagree with you.

Belcher seems to quote Feynman and claim that the -∫E.dl=0 for a superconductor, but says that the full equation for the loop includes -L(dI/dt) which has nothing to do with the -∫E.dl term, and that -L(dI/dt) is NOT always zero in a changing magnetic field, and so therefore a wire in a changing magnetic field (even a super conductor) can in fact have voltage across it for completely non-ohmic reasons.

Like I said, what I want is what nobody has been able to provide for me yet. That is a loop of elements consisting of resistors and one or more transformer winding which cause KVL to fail visible when I test around the loop with a volt meter following the rules for KVL.

So far? Nada.

There can be no electric fields inside a static conductor, except the one to sustain a voltage as a function of its current as per Ohm's law.

That's worth commenting on again. An entire thread about AC transformers, and you bring up static electric fields. Really?

You very well know that your argument is absolutely useless when it comes to dynamic magnetic fields otherwise  transformers  wouldn't put out voltage.

This is totally amazing. You really don't know this stuff, do you? Why on earth would you bring up static fields? Did you not know? Or did you think I wouldn't know the difference?
What, is this a poker match? Why not just be honest, give your best evidence, and let the cards fall where they may?


OK. So you want an answer to your question? Here it is: stop trying to understand who here knows what they are talking about.
I'm figuring it out. thinkfat knows more than you or snedri about transformers. He solved the puzzle, and did it right sharp too.

There's no guru in science.
I'll allow you to be the prime authority on that regarding only yourself. But I agree, you're not him if there is one  :-DD

You KVLers are obsessed with ascertaining who is wrong and who is right. Forget about that.
I'm just looking for the truth. People tell me my transformer secondaries won't hold up as a lumped element in a loop for KVL. But when I test with real volt meters, they seem to show that KVL holds.

Who says KVL doesn't always hold is nature

And where is this nature when I turn on my volt meter? Why will nobody give me an example of KVL failing due to the secondary winding of a normal closed-magnetic-circuit transformer being part of the loop?

The conclusions of your experiment are wrong because you don't listen to what the phenomenon right in front of you is telling.

Thank you Dr. bsfeechannel!

But don't feel bad if I am skeptical of your opinion about whether my experiment is wrong -- you couldn't even solve a loaded transformer voltage question that I eventually solved. And you know if I solved it (even if it took me hours) it can't be too hard, right?

In all seriousness though, why not show me a topology of resistors and toroidal transformer secondaries in a loop where KVL will fail as measured by my volt meter?
Title: Re: #562 – Electroboom!
Post by: Jesse Gordon on November 20, 2021, 06:00:21 am
KVL says: circulation of E along the ring = 0
Faradays says: circulation of E along the ring = EMF

Suuure! Because when Kirchhoff postulated his 'voltage law', he didn't have a emf in the circuit.  Are you even aware that there more more than one way of generating an emf and using it in a circuit?

Kirchhoff experimented with lumped sources of EMF. Batteries, that are localized in the circuit. In his statement he used the words "auf dem Wege", which means along the way, on the path. Can you locate it on the path, anywhere?

The EMF due to a changing magnetic field is essentially a relativistic effect, and it does not appear anywhere in the circuit. And it is also the manifestation of an actual electric field, the electric field that causes charges to move. It is not just something that happens to have dimensions of a voltage, it is actually a 'component' of the quantity voltage. That electric field, the induced electric field Eind becomes a component of the total electric field, together with the conservative field of the displaced charge.
And since electron only experience the total electric field, there is no way for them to know which portion came from the displaced charges and which came from the changing magnetic field.
You will never find the emf along the circuit path, like the lumped EMF Kirchhoff experimented with.

The EMF from a changing magnetic field is the only one to appear on the rhs - it's a direct consequence of Faraday's law, one of Maxwell's equations that expresses a fundamental property of the electromagnetic field.

Are you arguing that the only reason KVL doesn't hold for a loop with a transformer secondary as an element is because Kirchhoff didn't mention?

In other words, yes, KVL appears to hold for loops with transformer secondaries in it, but since Kirchhoff didn't specifically mention that at the time, then it's not technically  holding?

A big definition word game?
Title: Re: #562 – Electroboom!
Post by: Jesse Gordon on November 20, 2021, 06:06:54 am

You really seem to have a religious aversion to admitting that KVL *WOULD* give every appearance of holding for both my V1 and V2 configurations as measured with a volt meter.


Inner peace.

Inner peace.



Inner peace.


Ok, let's see...

Is FIVE equal to ZERO?

Because if five equals zero, then KVL holds in the nonlumped circuit.
If five is not equal to zero, then KVL does not hold in the nonlumped circuit.
I say that five is not equale to zero, and therefore KVL does not hold in the nonlumped circuit.

So, question - and only this question I would like you to answer:

Is FIVE equal to ZERO?

For exceptionally small versions of five and exceptionally large versions of zero, possibly  :-DD :-DD :-DD :-DD :-DD

And the way you refuse to provide a diagram for me to try which will cause KVL to fail, I think maybe for you 5 might equal 0.


Look my friend, I really appreciate your effort. I really do.

But your inability to grasp the basic concepts plus your refusal to answer questions which you think will undermine your house of cards makes your argument extremely weak and unconvincing.

But carry on, it is your argument to make, weak as it may be.

If you ever figure out how nature can speak to me through my volt meter, please do tell me. Until then, it looks to me like KVL holds up just fine even when some of the elements are transformer secondaries.
Title: Re: #562 – Electroboom!
Post by: Sredni on November 20, 2021, 06:18:13 am
Dude, I have supplied plenty of diagrams, if you don't understand them it's not my fault.
Put one of your voltmeters along a green path and the other along a purple path.

For the rest, try to realize that Faraday's law is one of Maxwell's equation and as such the EMF due to a changing magnetic field is not like other forms of emf.

rot E = -dB/dt

becomes (in stationary conditions - meaning we don't move things around)

circulation of E = - d/dt flux of B

Can you compute the circulation of a vector field if I give you the configuration?
Title: Re: #562 – Electroboom!
Post by: Jesse Gordon on November 20, 2021, 07:34:05 am
Dude, I have supplied plenty of diagrams, if you don't understand them it's not my fault.
Put one of your voltmeters along a green path and the other along a purple path.

For the rest, try to realize that Faraday's law is one of Maxwell's equation and as such the EMF due to a changing magnetic field is not like other forms of emf.

rot E = -dB/dt

becomes (in stationary conditions - meaning we don't move things around)

circulation of E = - d/dt flux of B

Can you compute the circulation of a vector field if I give you the configuration?

Probably not, but you'd have to try me for either of us to know for sure. I already told you I'm no math wiz. And I know you're not either.

I answered yet another of your questions, now it's your turn to answer one of mine.

What voltages would you say each of these 4 volt meters reads in the diagram below?
https://i.postimg.cc/15gbsCmz/20211119-232948.jpg
(https://i.postimg.cc/15gbsCmz/20211119-232948.jpg)

You can throw all the math at me you want but that's not going to prove that KVL actually fails. To make a convincing argument, you need to either:

A) Show that KVL actually fails in the diagram I have provided or some other loop diagram you may provide which uses resistors and the secondary windings from closed-magnetic-circuit-core transformers. I want to build an actual loop of resistors and transformer secondaries and measure the voltage differences across each element and have it not sum up to zero.

or

B) Admit that KVL would appear to hold perfectly as measured with a volt meter in the diagram I provided.

Once you do one of those things, then you can begin to make your case about why KVL is failing even though it appears according to the volt meter to hold.

But until you connect with reality and either admit it least APPEARS to a volt meter to work on the diagram I have provided, OR show me a configuration that I can actually test with my volt meter and have it not appear to hold, then you're refusing to connect with the observable level of reality.


And I go on and say as I told you months ago "Nobody has an answer."

Dude, I have supplied plenty of diagrams, if you don't understand them it's not my fault.

You have shared lots of wonderful diagrams, it's just that none of them were a diagram showing a loop of elements made of resistors and transformer secondaries which would allow me to add up the measured voltages and see KVL fail.

None of the diagrams you shared were the one I'm asking for.

If you went to the grocery store and asked for milk, and they brought you 50 dozen eggs, so you asked for milk again, and they said "Look! We brought you 50 items! Why aren't you happy?"

Same thing. I ask for a diagram that depicts a loop with resistors and transformer secondaries where I can measure the difference across each element and the sum is not zero.

Seriously, your diagrams are NOT what I'm asking for. Not one of them. Ever.


You can consider me a lost cause if you want, but that doesn't change the fact that you couldn't solve the transformer quiz and that you repeatedly refused to answer simple questions and use all sorts of diversionary angles.

If you want to reach people like me, you need to meet them where they are at -- at the level of their volt meter.

In the mean time, we go on to spread the truth that KVL holds when you correctly probe for loops who's size is much smaller than the wavelength of the frequency involved.
More and more people are making videos on the topic, demonstrating Lewin's error and some of them already know way more math than you and me, and the ones that don't know the math aren't going to be helped by your mathematical definition games.

They are going to be influenced by the low-level information about guys like Mehdi and all the other guys taking the practical approach -- because most people identify much more strongly with the practical approach, and won't be helped by you quoting math you don't understand either.

By the way, could YOU  compute the circulation of a vector field if given the configuration? Since you couldn't compute the output voltage of the 100v transformer, I bet not. Or, if you can compute the circulation of a vector field, then I must be able to too since I could solve the transformer quiz and you couldn't.

But I digress.

You're not reaching nearly as many people as you could if you'd just be honest about the simple stuff.

Again, would the sum(V1,V2,V3) = 0 in this circuit here: https://i.postimg.cc/15gbsCmz/20211119-232948.jpg
Would V4 be different than V3?


Thank you.
Title: Re: #562 – Electroboom!
Post by: thinkfat on November 20, 2021, 07:44:11 am
@Sredni
It's no use to throw stuff at Jesse that belongs to your world. He will either ignore it or twist it around until it fits. You will need to find something in his world that can unambiguously be shown to conflict with the reality, which is for him with a volt meter or a scope.

But it might be that he's just too far gone to be saved.
Title: Re: #562 – Electroboom!
Post by: jesuscf on November 20, 2021, 07:46:11 am
@Sredni
It's no use to throw stuff at Jesse that belongs to your world. He will either ignore it or twist it around until it fits. You will need to find something in his world that can unambiguously be shown to conflict with the reality, which is for him with a volt meter or a scope.

But it might be that he's just too far gone to be saved.

Suuure!  Can you please show us your experimental data?
Title: Re: #562 – Electroboom!
Post by: thinkfat on November 20, 2021, 11:29:53 am
But it might be that he's just too far gone to be saved.

Let me elaborate, based on his arrangement:
(https://i.postimg.cc/rs3cCg2F/20211119-122528.jpg)

Jesse believes, and all his measurements confirm it, that:

Voltage is induced locally only where "stuff" is "crossing" the magnetic flux, e.g. in a transformer, only the part of the windings that are "inside" the core contribute to the voltage. Hence only the "2R" resistor acts as a "secondary"

That explains how he gets 0V across the wires and 1/3V across "R". Those are not "inside" the core and so they are no voltage sources.
And since the "2R" resistor for him is a local voltage source, he subtracts the 1V induced EMF from its "ohmic" voltage drop (due to the 1/3R current) and arrives at -1/3V.
The probe wires for the rightmost volt meter are arranged so that, in his world, they don't form a "secondary" because, well, they're not "inside" the core. For him that's the only correct way.
Would I suggest a different path for the probe wires, one that doesn't form a loop around the core, he would reject it because then the wires would necessarily "cross" the inside of the core and form a "secondary" that would compensate the voltage generated in "2R", skewing the measurement.

And guess what: the measurements are "correct" even in my world. Presented with above setup, I will come to the same prediction. Of course I would arrive there without ignoring/violating Maxwell-Faraday, but these mean nothing to Jesse (he believes in a completely distorted version of it).

So for him it all adds up nicely and even makes KVL work. And that's why I likened him to a flatearther: He has made up a framework for himself that is complete and can be proven by observation. Of course Belcher and McDonald would cringe at the sight and Lewin would laugh his arse off.

So, to save Jesse (and the cause), the only way is to disprove experimentally something that must hold in his framework. That's how science has worked forever: find the "purple cow" that must not exist. Bombarding him with "truth" like a Missionary will not work.
Title: Re: #562 – Electroboom!
Post by: jesuscf on November 20, 2021, 05:07:57 pm
You will never find the emf along the circuit path, like the lumped EMF Kirchhoff experimented with.

You are so wrong on this one that you are even disagreeing with Lewin and everyone else for that matter!
Title: Re: #562 – Electroboom!
Post by: Jesse Gordon on November 20, 2021, 07:02:46 pm
@Sredni
It's no use to throw stuff at Jesse that belongs to your world. He will either ignore it or twist it around until it fits. You will need to find something in his world that can unambiguously be shown to conflict with the reality, which is for him with a volt meter or a scope.

What does belong in his world? He couldn't solve the 10:1 loaded transformer question, like you expertly did. He doesn't seem to be able to solve much of anything, he won't answer questions about reality head on, he avoids them like the plague, he doesn't seem to understand the math that Belcher talks about.

But it might be that he's just too far gone to be saved.

You know it's actually funny you should say that. I have a real life flat earth friend and I've been arguing online with flat earthers for 3 years.

I have been gaslit by the best of them, I am very familiar with all the evasive antics I see performed by the Lewinites in this thread.

In the course of my personal study into the topic, I have actually performed a modified Cavendish experiment and measured gravity, I've used aircraft gyroscopes to measure earth rotation, and I've used surveyor's instruments (which I got off ebay!) to measure earth curve from 8 miles to 105 miles distance, from observer elevations of 50ft to 8900ft, from target heights of 0ft to 14000ft. I did these because flat earthers always say "You don't know that, you're just parroting mainstream lies!" so I tested the claims myself. (BTW, earth is a globe, it does spin, and gravity is real, and I can prove it.)

Do you know what flat earthers say after I show them video evidence of all my observations?

Do you know what the say when they realize the evidence is on my side?

They say "You're too far gone."

If I may say so, since you brought up the subject, talking with Lewinites is in fact very much like talking with flat earthers. Flat earthers have no appreciation of the real testable world. They have their own framework that makes sense to them based on a bunch of misunderstandings of observable reality, because, like lewinites, they absolutely refuse to look at observable reality.

They also refuse to answer questions head-on. They try to avoid the questions.

One of their biggest "proofs of a flat earth" presented by flat earthers is that under certain atmospheric conditions, you can see much farther over water than should be possible on a globe. Of course they don't believe in refraction, and even though they can see drastic refraction evidence in their photos, they still refuse to consider that. Look at this:
https://i.imgur.com/Odrs9tn.jpg

Even though there is ample evidence of refraction distorting the apparent vertical position in their own evidence, they still ignore it, JUST LIKE Lewin ignores the very blatant evidence that his volt meter leads are acting as transformer secondary windings and distorting his reading.

Lewin actually made two mistakes in his setup - one was to hook up the second volt meter backwards. If you're really using KVL,you arrange all the volt meters with positive pointing the same way around the circle, for example, positive clock wise or positive counter clockwise.

Lewin had both volt meters with positive up, or positive down. That's why he got a positive voltage on one and a negative voltage on the other. He totally failed KVL right there. Dude, he had a volt meter on backwards! KVL would have failed even on a resistor+battery circuit with that mistake!

His second mistake was to not consider -L(dI/dt) for the loop formed by his volt meter leads and realize that he was measuring the voltage across the resistors -- which, if his volt meters were connected on the correct polarity, would have summed to 0.1v, which is his induced EMF. Had he also accurately probed his winding segments, he would have also have found the -0.1v of induced EMF and KVL would have appeared to hold there too.


As for yourself, I see you almost made a fourth wrong prediction. You were like "are you SURE you don't want to change your analysis?" You thought I was wrong again, but after being proved wrong 3 times in a row because I have a volt meter and I test claims against observable reality, then when you realized that my analysis agreed with observable reality, now you're not even responding to me, you're just calling me "too far gone."

If anyone is too far gone, it is you, because you are unable to say "Hey you know what, yes I was wrong about a bunch of stuff because I really don't know as much as I thought."

But personally, I don't think you're too far gone. You're still learning. After a while you will come to grips with observable reality, you and I will be able to agree that KVL at the very least appears to hold, then we can talk about why it's not actually holding by some specific definition.


The sad part is once we agree on the basics of observable reality then that opens up the path for us to discuss what might actually be going on and the math behind it. But for some reason Lewinites cannot simply say "Hey, you're right, if measured with a volt meter, KVL appears to hold. But here's why it is not actually holding..." But we can never get to that point because Lewinites cannot come to grips with observable reality, for whatever it is.

And that is why I have reason to believe they are wrong about the theoretical angle because if they can't come to grips with the observable reality that is available to anybody with a volt meter, then why should I think they even understand the more complex stuff?

The fact is that there is a lot about the topic that you do not know. But you are learning. Keep up the good work!
There's also a lot I don't know about the topic, especially in the calculus area. I told you I'm no math wiz. We can learn from eachother, I know the real-world level pretty well, and presumably you know the math level pretty well....?
Title: Re: #562 – Electroboom!
Post by: bsfeechannel on November 20, 2021, 08:01:50 pm
So which is it? Nature? Or the gurus that write the books and teach the courses?

Nature.

In an engineering degree we test the theories in the lab systematically. One by one. We don't treat the laws of physics as a precept given by some scientific "authority". There's no authority in science, except that of nature.

Quote
Why are you bringing up static fields when we're talking about AC transformers?

You don't even know how to ask the right questions and you think you already know more than Lewin.

I said static wire. Not static field. Static in relation to a frame of reference.

Quote
But don't feel bad if I am skeptical of your opinion about whether my experiment is wrong

It's not my opinion. You'll find it eventually. But it's a good thing that you are skeptical.
Title: Re: #562 – Electroboom!
Post by: bdunham7 on November 20, 2021, 08:35:21 pm
KVL at the very least appears to hold, then we can talk about why it's not actually holding by some specific definition.

I don't know if you read anything I posted earlier in this thread, but my only intent was to try to explain that very difference, which in my view is that you have a different "KVLer" notion of 'voltage' from the one presumed by  the "Lewinites".  Sredni refers to this as the 'scalar potential', which may be the commonly accepted term for it.  In the vast majority of circumstances the two definitions seem to coincide, the times when they do not are what is being discussed in this thread.

One of the sticking points for me is the seeming assertion that the 'scalar potential' is a meaningless value that is physically impossible to measure and therefore is not worthy of discussion and is not a valid definition of anything, let alone voltage.  In the simple planar world of the Lewin ring and using voltmeters with conductive wire test leads, that is true and mathematically demonstrable.  Outside of that world, I'm not so sure.  I tried to show some examples of what I meant, but I may not have succeeded in convincing anyone.  Anyhow, good luck.  I'm agnostic on the issue and I generally don't have any trouble measuring stuff.  Unless you go over a few GHz, then I'm out of both equipment and theoretical understanding.
Title: Re: #562 – Electroboom!
Post by: Jesse Gordon on November 20, 2021, 08:44:11 pm
But it might be that he's just too far gone to be saved.

Let me elaborate, based on his arrangement:
(https://i.postimg.cc/rs3cCg2F/20211119-122528.jpg)

Jesse believes, and all his measurements confirm it, that:

Voltage is induced locally only where "stuff" is "crossing" the magnetic flux, e.g. in a transformer, only the part of the windings that are "inside" the core contribute to the voltage. Hence only the "2R" resistor acts as a "secondary"

That's how reality presents itself to the volt meter. I set up a ring just like you said. I couldn't find in my junkbox two resistors in a 1:2 resistance ratio, but I found 3 resistors of 51 ohms each, so I literally had 1R on one side, and 2R on the other side, just like you said. I had it super symmetrical, just like you wanted. Here's a picture.
For the picture, I rotated the ring so you could see all 3 resistors, but for the actual test I put the 2R exactly inside the core just like you drew.
(https://i.postimg.cc/CxTLN3Py/20211119-140703.jpg)

That is observable reality. The fact that it takes this many pages of argument for you to realize that means you started out missing a ton of vital relevant information on the topic!

That explains how he gets 0V across the wires and 1/3V across "R". Those are not "inside" the core and so they are no voltage sources.
And since the "2R" resistor for him is a local voltage source, he subtracts the 1V induced EMF from its "ohmic" voltage drop (due to the 1/3R current) and arrives at -1/3V.

I didn't subtract, I measured.

I get 0v across the wires because you have to look at the wire I'm measuring and my volt meter leads as a loop. There is zero dB/dt inside that loop, so the volts is going to be zero.

The probe wires for the rightmost volt meter are arranged so that, in his world, they don't form a "secondary" because, well, they're not "inside" the core. For him that's the only correct way.

Of course. The transformer is a black box with wires coming  out from it. My volt meter is not inside the transformer, why should I model it other than reality?

I know you wanted me to run my volt meter leads through the transformer core as a secondary winding, but you didn't want me to model in that new secondary. When your model and your real circuit are different from each other, not only does KVL fail but all other laws which depend on correct modeling fail as well.

Would I suggest a different path for the probe wires, one that doesn't form a loop around the core, he would reject it because then the wires would necessarily "cross" the inside of the core and form a "secondary" that would compensate the voltage generated in "2R", skewing the measurement.

Yes, you wanted me to run a parallel winding. All I would have measured was IR losses, because two windings following the same path have the same voltage induced across them, as if it was a single fatter wire..

(Which was the same mistake Lewin made: He may have had his volt meter lead attached at the top of the ring, but it ran exactly along the path of the ring until it reached the resistor, where it then turned and went to the scope. He might as well have just attached the scope to the resistor directly, he would have measured the same thing. He was just measuring the voltage drop across the resistor, nothing else, any engineer worth sparks knows that.)

Now in the case of the yellow diagram above, I could have run another secondary winding turn in series with one of my volt meter leads to either add or subtract 1v to the IR loss across 2R, and if I ran it the direction to add the 1v to the -1/3rd V reading, then I would have been left with just the IR voltage loss of 2R which was 2/3rds of the full loop induced voltage. (And I did test this too, and it is correct.)

And guess what: the measurements are "correct" even in my world. Presented with above setup, I will come to the same prediction. Of course I would arrive there without ignoring/violating Maxwell-Faraday, but these mean nothing to Jesse (he believes in a completely distorted version of it).

Acknowledging observable reality does not ignore Maxwell or Faraday.

Observable reality is just what is. However Maxwell and Faraday describe the inner workings is irrelevant to the fact that in the real world a close-magnetic-circuit-cored transformer models as if the entire voltage induced happens at the point that the winding passes through the core.

I never said that's actually how Faraday or Maxwell describe it, or even that is how it is working - I said that in the real world, it models that way. It measures that way.

Why has it taken this long for you to realize that? Once you realize that in the real world, it models this way, then we can have a fascinating discussion about what's going on mathematically.

So for him it all adds up nicely and even makes KVL work. And that's why I likened him to a flatearther: He has made up a framework for himself that is complete and can be proven by observation. Of course Belcher and McDonald would cringe at the sight and Lewin would laugh his arse off.

Belcher and McDonald cringe at Lewin, frankly. Think about it.
McDonald accused Lewin of using diversion and an outdated "law" to create an apparent paradox.
McDonald must have been really cringed out because for a fellow physics professor to actually accuse his colleague IN WRITING like that is pretty huge. There's obviously some level of professional courtesy that should have refrained McDonald, and I'm sure there was some, but he still called Lewin out.

Belcher was more dignified, but he still said that there was a common misconception and that it was incorrect in stating that the volt meter reading was represented in the -∫E.dl, and was wrong in stating that the ∮E.dl=0 for a superconductor.
Those are the misconceptions that Lewin exhibits. So while Dr. Belcher didn't name Lewin, he described Lewin's claims and called them a misconception. Belcher had to have been cringing at Lewin to write that.

If you think Belcher and McDonald cringe at my claims of observable reality, then I suppose you think they cringe also at Mehdi's claims?

Look how Belcher cringes about Mehdi. He says:

Quote

I am grateful to Mr. Sadaghdar for a number of discussions about Faraday’s Law and KVL, which have improved my
understanding of both.
He also describes Mehdi's series of experiments as "very nice."

And what does Dr. Belcher say about Lewin? As far as I can tell, he doesn't even name him. His fellow physics professor, and doesn't even say one thing about him. The silent treatment.
At the very least he could have also simply said "I would like to thank my colleague Professor Lewin for his contribution to this fascinating discussion." But he didn't even say that in the writup I have. No, Belcher was cringing at Lewin.

At least he could have said SOMETHING to give Lewin some form of credence. But he said not one thing.

Sorry, I don't think I buy your claim that Dr. Belcher would cringe at seeing me claim that close-magnetic-circuit-cored transformers model and measure as if the entire voltage was produced at the point the winding passed through the core.


I didn't make up any framework. I just used my volt meter, resistors, and transformers to find out what observable reality is. I measured the voltage differences around the loop and summed them up just like KVL says to do, and they summed to zero, just like it said they would.

My level-1 "framework" is what I observe. Above that I do my best to understand what's going on at a theoretical level. Maxwell, Faraday, and Kirchhoff described it elegantly. I have no problem with any of them. While I may not understand all of the math involved in all of the calculations, I understand enough to sort of follow along with Belcher & McDonald, and they both seem to be saying that Lewin's loop is within the range of applicability of Kirchhoff's loop equations.

If you guys actually knew what you were talking about, the opening post would have been like "Yes we know that when correctly probed, KVL appears to hold even in a varying magnetic field. But for reason XYZ it's not actually holding..." (or whatever your "but" is, which I don't know because we've been arguing about observable reality the whole time.)

So, to save Jesse (and the cause), the only way is to disprove experimentally something that must hold in his framework. That's how science has worked forever: find the "purple cow" that must not exist.
Yes! Thank you! I do want the truth, but I need to see it with my own eyes, otherwise it's just me believing popular opinion, like all yall.

My "framework" is just what I observe followed by trying to make sense of observable reality.

I welcome any observable proof that any part of my framework is false. I constantly adjust my framework to adopt any new evidence that comes to me.

Bombarding him with "truth" like a Missionary will not work.

That is very interesting that you should put it that way.

A missionary goes and tries to convince people of this faith.

Are you implying that your "knowledge" of KVL is actually a faith?
Title: Re: #562 – Electroboom!
Post by: Jesse Gordon on November 20, 2021, 09:19:18 pm
So which is it? Nature? Or the gurus that write the books and teach the courses?

Nature.

In an engineering degree we test the theories in the lab systematically. One by one. We don't treat the laws of physics as a precept given by some scientific "authority". There's no authority in science, except that of nature.

Quote
Why are you bringing up static fields when we're talking about AC transformers?

You don't even know how to ask the right questions and you think you already know more than Lewin.

I said static wire. Not static field. Static in relation to a frame of reference.

Quote
But don't feel bad if I am skeptical of your opinion about whether my experiment is wrong

It's not my opinion. You'll find it eventually. But it's a good thing that you are skeptical.

Static in relation to a frame of reference?  You mean not physically moving???

Regarding your earlier statement of:
Quote
There can be no electric fields inside a static conductor, except the one to sustain a voltage as a function of its current as per Ohm's law. This is one of the first things we learn in whatever electromagnetism course out there.

I'm not saying that there are electric fields INSIDE a conductor, stationary or otherwise, that we know because the -∫E.dl=0 for a superconductor.
I am talking about the EMF across the ends of a wire according to -L(dI/dt), which Belcher clearly describes as being non-zero in a changing magnetic field even for a superconductor.

That should be the second thing you learned in your electromagnetism course out there.

And wait, why are you talking about whether the conductor is moving or not?

Are you thinking there's a difference between a stationary wire in a dB/dt field, and a moving wire in a stationary and non-uniform B field?
Title: Re: #562 – Electroboom!
Post by: Sredni on November 21, 2021, 01:07:53 am
I had written this post hours ago, before reading Jesse's last rant, and delayed till I had the links and the images, but I did not have to change anything except for a few addition to confirm my deductions.

Jesse believes, and all his measurements confirm it, that:
Voltage is induced locally only where "stuff" is "crossing" the magnetic flux, e.g. in a transformer, only the part of the windings that are "inside" the core contribute to the voltage. Hence only the "2R" resistor acts as a "secondary"

Yes, Jesse is a 'lumper'.
He believes that the core is some sort of magical portal, like a stargate, and when you cross it, when you 'cut through it' with your wires, they magically acquire a voltage, that is there independent of anything else.
But in the case of the infinitely long solenoid, where he cannot find the entrance of the stargate, he is forced to become a 'distributer', because the 'hole' is now the whole space and the magical source of voltage can now be inside the whole length of the conductors and even of the probes. "The probes become the secondary".

Edit: read another subsequent message before posting this and he appears to confirm that he jumps between positions depending on what suits him best at the moment

Quote from: Jesse Gordon
"However Maxwell and Faraday describe the inner workings is irrelevant to the fact that in the real world a close-magnetic-circuit-cored transformer models as if the entire voltage induced happens at the point that the winding passes through the core."

I have a couple of setups he should try to test his belief with his trusty voltmeter.
But the reason I keep answering him is not to convince him - ironically he is more of a flatearther of the flatearthers he mocks on his channel - but (among other things) to observe the rejection mechanisms when he is cornered.
First he needed to reassure himself with that exercise on the transformer and you can see the false equivalence (reiterated in several messages, including the last one)

Quote from: Jesse Gordon
Alright, the jury is in! bsfeechannel and Sredni couldn't solve it.  .
...But don't feel bad if I am skeptical of your opinion about whether my experiment is wrong -- you couldn't even solve a loaded transformer voltage question that I eventually solved. 
...He couldn't solve the 10:1 loaded transformer question, like you expertly did. 
...that doesn't change the fact that you couldn't solve the transformer quiz 
...so if someone solved it better than me I know they are better than me, and if they can't solve it then I know they are no better than me

"See, you did not do my high school exercise, therefore you cannot do it and since I can, the only possible conclusion it that I know all of EM better than you. Even if I have no idea how to compute the circulation of a vector field".

Then he is trying to reject all analytical reasoning because he cannot handle the math.

Quote from: Jesse Gordon
You can throw all the math at me you want but that's not going to prove that KVL actually fails. 

to the point that when asked if 5 = 0 his answer is:

Quote from: Jesse Gordon
For exceptionally small versions of five and exceptionally large versions of zero, possibly 

So, now all proofs he accepts are those that he himself can do with his limited equipment: a pancake solenoid that has a ton of stray field on the outside (it's basically a multifilar coil), and an EI transformer that will have 'returning legs' on both sides of the ring and nearly no space inside the legs. He also once produced a tiny toroidal core in whose hole he could not fit the probes.
And he rejects experiments that could prove him wrong, like building a long solenoid to confine the field and go all around it or even right above it, or using a transformer with a gap big enough to fit the probes...Like Cardinal Bellarmino refused to look into Galileo's telescope.
If you cannot prove he's wrong using the limited setup he has, then he must be right.

Quote from: Jesse Gordon
Would it be fair then to say that in the real world, where wires and ferrite/iron cannot exist in the same physical space, that we don't have to worry about our volt meter leads accidentally ending up half way through the cross section of a toroid, and thus in the real world, KVL holds fine 

(Even tho he seems to have changed language: now KVL 'have the appearance of holding' and from one of the latest posts: 'I never said that's actually how Faraday or Maxwell describe it, or even that is how it is working').  So, at least he appears to be right.


But the most interesting thing I have got from him is his lucid analysis of what in the older thread on this topic I called 'scientific populism'.

Quote from: Jesse Gordon
"In the mean time, we go on to spread the truth that KVL holds when you correctly probe for loops who's size is much smaller than the wavelength of the frequency involved.
More and more people are making videos on the topic, demonstrating Lewin's error and some of them already know way more math than you and me, and the ones that don't know the math aren't going to be helped by your mathematical definition games.
They are going to be influenced by the low-level information about guys like Mehdi and all the other guys taking the practical approach -- because most people identify much more strongly with the practical approach, and won't be helped by you quoting math [...]"

He is 100% right.
This is how politics has always worked: you don't need to tell the truth or correctly solve problems, far from it, you just need to tell your electorate what they want to hear. Make them feel smart, even if they are dumb as rocks, by using their language and indulging their wrong beliefs. They will adore you and they will give you their vote and their money.

Social media have helped transpose this to other fields, unfortunately including science and engineering.
You might think, "in science, wrong beliefs can be proved wrong", but Lewin has shown that no, that is not always the case. And certainly not on youtube, where the mass of ignorant followers steer the herd toward whatever they like best (usually something that the average viewer can understand and relate to).

To make another example that is nowhere near as controversial as Lewin's ring: three years ago, exactly in the same period we were discussing Electroboom's video, Science Asylum made a video on the flow of energy in circuits titled "Energy doesn't flow the way you think" (I noticed today that that video is now called "Circuit energy doesn't flow the way you think")

Science Asylum

source youtube video watch?v=C7tQJ42nGno
(https://i.postimg.cc/NGNJkBzc/screenshot-5.png)

He got the direction of the electric field near the wires wrong by nearly 90 degrees (my crystal ball says that's because he saw a figure with the Poynting vector going into a resistive piece of material and assumed it would have been the same in an highly conductive wire forming a circuit with a battery and a far away resistor). In his video he shows energy coming into his circuit from all around space in all directions. It is wrong.
.
Energy flow goes from the battery, is guided by the wires and then 'plunges' into the resistor (there is even a paper by John D. Jackson about the role of surface and interface charge on this). It is nothing fancy as 'energy comes from all around  the universe' that he tried to push on his ignorant audience.
In the comments immediately after the video, among the the mass of fans whose minds had been blown away by such an unexpected (and false) conclusion, somebody had tried to told him that he was getting the directions all wrong - and by nearly 90 degrees - as shown even on wikipedia, and that the field inside the wires is not stronger in the wire than it is outside (as he wrote in the comments). But he kept insisting that the field in the wire is stronger than outside, thereby confirming he has a deeply wrong conception of the direction of the field near the cables, and consequently of the Poynting vector). You could see his erroneous statements had hundreds of thumbs up and the correct critiques just a handful, if any at all.

Fast forward to yesterday, when Veritasium published a video on the same topic (with the slight twist of how the fields propagate when you flip the switch - there was a question on EE Stack Exchange about this a few months ago, and yes it has all to do with surface charges but I am not completely sure Veritasium got it completely right).

Veritasium: The Big Misconception About Electricity  (negative mine)
source youtube video watch?v=bHIhgxav9LY
(https://i.postimg.cc/fTp815sq/screenshot-6.png)

Veritasium (who actually go to the real experts for his videos) got the directions of the fields right just like the wikipedia image. And what is the comment Science Asylum made to Veritasium video?

Quote from: Science Asylum on YT
"I made a video about this a few years ago and I can assure you that you will be told by viewers that you're wrong (even though you aren't). I'm glad you made the video though. The more of us that cover this topic, the more people we'll be able to reach and educate. "

Read in the replies to Science Asylum comment how people are complimenting him for showing them the truth in that three years prior video (which he never corrected). Despite being dead wrong on the direction of the flow of energy. He tried to defend it in the replies (because someone still pointed that out) saying that he made a direction error

(https://i.postimg.cc/VLrR9Mtq/screenshot-2.png)

(https://i.postimg.cc/kgWvzmfD/screenshot-3.png)

But that direction error is colossal, and change completely the direction of energy flows in his video. Also with zero resistance wires Veritasium directions are perfectly correct, and with resistive copper wire he might be off by a fraction of degree, in the middle of the circuit?.
Science Asylum did not correct the video and now he also writes

(https://i.postimg.cc/ZRs8pKKs/screenshot.png)

He has got it wrong by nearly ninety degrees.
How many points would one get in an exam if they got the direction of fields or forces wrong by nearly 90 degrees? And yet, he maintains he is basically right (yeah, not quite correct either...) and he was wrongly attacked. In the comment section of the video that proves him wrong.

Doublethink.

Moreover, all you can see now is... thumbs up. Both for his wrong video, for his leading comment in his wrong video, and for the comment in Veritasium videos where he is trying to rewrite history. Gaslighting is strong on social media.

We live in an age that mixes Orwell's 1984 with Idiocracy. We are now dealing with "the big dumb brother".
We never went to the Moon, but we found the Reptilians there. Thumbs up.

P.S.
Even worse, I just noticed now, Veritasium gave him his little throbbing heart and commented

(https://i.postimg.cc/CKNHM4yg/screenshot-4.png)

The circle is closed.
Title: Re: #562 – Electroboom!
Post by: Sredni on November 21, 2021, 01:21:43 am
You will never find the emf along the circuit path, like the lumped EMF Kirchhoff experimented with.

You are so wrong on this one that you are even disagreeing with Lewin and everyone else for that matter!

Very well. Since Jesse won't do this experiment, let's see if you are willing to do it (I did it).

Are you a toroidal power transformer owner?
You know one those five inches or so big donuts?
Title: Re: #562 – Electroboom!
Post by: jesuscf on November 21, 2021, 01:35:58 am
You will never find the emf along the circuit path, like the lumped EMF Kirchhoff experimented with.

You are so wrong on this one that you are even disagreeing with Lewin and everyone else for that matter!

Very well. Since Jesse won't do this experiment, let's see if you are willing to do it (I did it).

Are you a toroidal power transformer owner?
You know one those five inches or so big donuts?

Diversion and more diversions.  I have a much better idea.  Here, solve this very simple problem.  Please post your complete solution.  I'll give you a day from now, before I post the solution myself.  This problem should look extremely familiar to anybody reading this forum!

A circular wire ring with a radius of 0.2 m has two resistors attached as shown in the figure.  R1 has a resistance 100Ω and R2 has a resistance of 900Ω.  The resistance of the wire can be ignored. The time varying magnetic flux density as shown in the figure is given by:

 \$
B(t) = 0.7958 \cdot e^{\frac{{ - t}}{{0.1s}}} _{} T
\$

At t=0, determine the voltages across the resistors V1 and V2 (with the polarity as indicated in the figure) and the voltage between nodes A and D (VAD) which are half a circle apart.


Title: Re: #562 – Electroboom!
Post by: Sredni on November 21, 2021, 01:48:19 am
You will never find the emf along the circuit path, like the lumped EMF Kirchhoff experimented with.

You are so wrong on this one that you are even disagreeing with Lewin and everyone else for that matter!

Very well. Since Jesse won't do this experiment, let's see if you are willing to do it (I did it).

Are you a toroidal power transformer owner?
You know one those five inches or so big donuts?

Diversion and more diversions.  I have a much better idea.  Here, solve this very simple problem.  Please post your complete solution.  I'll give you a day from now, before I post the solution myself.  This problem should look extremely familiar to anybody reading this forum!

A circular wire ring with a radius of 0.2 m has two resistors attached as shown in the figure.  R1 has a resistance 100Ω and R2 has a resistance of 900Ω.  The resistance of the wire can be ignored. The time varying magnetic flux density as shown in the figure is given by:

 \$
B(t) = 0.7958 \cdot e^{\frac{{ - t}}{{0.1s}}} _{} T
\$

At t=0, determine the voltages across the resistors V1 and V2 (with the polarity as indicated in the figure) and the voltage between nodes A and D (VAD) which are half a circle apart.

Well, if the field permeates the whole ring the flux is the product of the area of the ring times B which is spatially constant on the whole surface.
Differentiate with respect to t, change sign and you get the EMF. You need to pay attention to signs - what matters is how the flux changes.
Once you know the EMF and its verse (cw or ccwI, you know the current I(t) = EMF(t) / (R1 + R2) with the same verse established for the EMF. It will be exponentially decaying, since the time derivative of a decaying exponential is also a decaying exponential. Then you compute the voltages along the resistor branches by using R * I(t). The sign comes from that of the current and the convention of the sinks.

Fill in the values, if you want. That's a servant job.

Whatever voltage is in the ratio 9:1, like the resistors, and since the current goes down in one branch and up in the other the will appear to have opposite sign.

Happy, now?

Now, do you have a toroidal transformer? Yes or no?

(edit: grammar. It happens when you solve a problem in 40 seconds)
Title: Re: #562 – Electroboom!
Post by: bdunham7 on November 21, 2021, 01:56:02 am
voltage between nodes A and D (VAD) which are half a circle apart.[/b]

As I've stated, the definition of that voltage is the very issue that is not agreed upon.  So if you want a number for an answer, you have to specify your definition or method of measurement.  One definition that might be interesting is the voltage measured by a voltmeter whose leads go in an exact straight line between the points.  You would need a gap or hole in the inductor and core that are providing the magnetic flux, but that may be doable.
Title: Re: #562 – Electroboom!
Post by: Sredni on November 21, 2021, 02:07:25 am
voltage between nodes A and D (VAD) which are half a circle apart.[/b]

As I've stated, the definition of that voltage is the very issue that is not agreed upon.  So if you want a number for an answer, you have to specify your definition or method of measurement.  One definition that might be interesting is the voltage measured by a voltmeter whose leads go in an exact straight line between the points.  You would need a gap or hole in the inductor and core that are providing the magnetic flux, but that may be doable.

Or one could use the definition of voltage given by the International Electrotechnical Committe IEC 60050
https://www.electropedia.org/iev/iev.nsf/display?openform&ievref=121-11-27 (https://www.electropedia.org/iev/iev.nsf/display?openform&ievref=121-11-27)
and specify the path, as I did.
Title: Re: #562 – Electroboom!
Post by: jesuscf on November 21, 2021, 02:10:55 am
You will never find the emf along the circuit path, like the lumped EMF Kirchhoff experimented with.

You are so wrong on this one that you are even disagreeing with Lewin and everyone else for that matter!

Very well. Since Jesse won't do this experiment, let's see if you are willing to do it (I did it).

Are you a toroidal power transformer owner?
You know one those five inches or so big donuts?

Diversion and more diversions.  I have a much better idea.  Here, solve this very simple problem.  Please post your complete solution.  I'll give you a day from now, before I post the solution myself.  This problem should look extremely familiar to anybody reading this forum!

A circular wire ring with a radius of 0.2 m has two resistors attached as shown in the figure.  R1 has a resistance 100Ω and R2 has a resistance of 900Ω.  The resistance of the wire can be ignored. The time varying magnetic flux density as shown in the figure is given by:

 \$
B(t) = 0.7958 \cdot e^{\frac{{ - t}}{{0.1s}}} _{} T
\$

At t=0, determine the voltages across the resistors V1 and V2 (with the polarity as indicated in the figure) and the voltage between nodes A and D (VAD) which are half a circle apart.

Well, if the field permeates the whole ring the flux is the product of the area of the ring times B which is spatially constant on the whole surface.
Differentiate with respect to t, change sign and you get the EMF. You need to pay attention to signs - what matters is how the flux changes.
Once you know the EMF and its verse (cw or ccwI, you know the current I(t) = EMF(t) / (R1 + R2) with the same verse established for the EMF. It will be exponentially decaying, since the time derivative of a decaying exponential is also a decaying exponential. Then you compute the voltages along the resistor branches by using R * I(t). The sign comes from that of the current and the convention of the sinks.

Fill in the values, if you want. That's a servant job.

Whatever voltage is in the ratio 9:1, like the resistors, and since the current goes down in one branch and up in the other the will appear to have opposite sign.

Happy, now?

Now, do you have a toroidal transformer? Yes or no?

(edit: grammar. It happens when you solve a problem in 40 seconds)

I don't have a toroidal transformer.

Can you solve for the value of the voltages in volts?  Also, why are you using KVL (EMF(t)-I(t)R1-I(t)R2=0) to get the solution?  Are you one of the 'birds' now?
Title: Re: #562 – Electroboom!
Post by: bdunham7 on November 21, 2021, 02:22:56 am
Or one could use the definition of voltage given by the International Electrotechnical Committe IEC 60050
https://www.electropedia.org/iev/iev.nsf/display?openform&ievref=121-11-27 (https://www.electropedia.org/iev/iev.nsf/display?openform&ievref=121-11-27)
and specify the path, as I did.

Very true, although you could also take the additional step of requiring the path to be one without any net rotational field and thus equal to the second part of that definition you linked.  You could call the first "Lewinvolts", which can have any value you like depending on the path you take, or the second version "Kirchoffvolts" which always have a definite value between two points.  If such a path cannot be found, then the Kirchoffvoltage is undefined.  Or you could mandate that the path always be a straight line, and that can be the "tunnelvolt".

 Remember, I'm not a KVLer, my only goal is to clarify what you guys are arguing about. :)
Title: Re: #562 – Electroboom!
Post by: Sredni on November 21, 2021, 02:35:11 am

I don't have a toroidal transformer.


How unfortunate.

Quote
Can you solve for the value of the voltages in volts?


Of course I can. It's not that difficult differentiate an exponential.
But I am not going to waste my time doing menial task. I gave you the full procedure to compute the solution for any type of (spatially uniform and ortogonal to the disk) B(t) and for any t.
Wanna play data entry employee? Suit yourself.

And, besides, not doing it will give you the same satisfying sensation Jesse experienced. I cannot differentiate A e^k t!!! You must be right on everything, then.

Quote
Also, why are you using KVL (EMF(t)-I(t)R1-I(t)R2=0) to get the solution?  Are you one of the 'birds' now?

I am not using KVL. It is Faraday:
-path integral of E.dl  is R1 I(t) + R2 I (t)  (these you can locate on the ring)
-d/dt flux of B is the EMF (this you can't, and if you had a toroidal transformer you could witness it with your eyes)

Faraday says, then

     -path integral of E.dl  == -d/dt flux of

which is

     R1 I(t) + R2 I (t) ==  EMF

Faraday! Exactly because around the loop you will find only what's in the lhs.

Title: Re: #562 – Electroboom!
Post by: jesuscf on November 21, 2021, 03:12:12 am

I don't have a toroidal transformer.


How unfortunate.

Quote
Can you solve for the value of the voltages in volts?


Of course I can. It's not that difficult differentiate an exponential.
But I am not going to waste my time doing menial task. I gave you the full procedure to compute the solution for any type of (spatially uniform and ortogonal to the disk) B(t) and for any t.
Wanna play data entry employee? Suit yourself.

And, besides, not doing it will give you the same satisfying sensation Jesse experienced. I cannot differentiate A e^k t!!! You must be right on everything, then.

Quote
Also, why are you using KVL (EMF(t)-I(t)R1-I(t)R2=0) to get the solution?  Are you one of the 'birds' now?

I am not using KVL. It is Faraday:
-path integral of E.dl  is R1 I(t) + R2 I (t)  (these you can locate on the ring)
-d/dt flux of B is the EMF (this you can't, and if you had a toroidal transformer you could witness it with your eyes)

Faraday says, then

     -path integral of E.dl  == -d/dt flux of

which is

     R1 I(t) + R2 I (t) ==  EMF

Faraday! Exactly because around the loop you will find only what's in the lhs.

Got it!  You don't know how to use a calculator.  Also, in case you didn't realize it, Faraday's law is one of the many forms you can encounter KVL.  For example, using your same flawed logic, we can argue that a thermocouple circuit is not solved using KVL but the Seebeck effect.  Imagine if we would have to 'name' the solution of every possible circuit based on the nature of its EMF...


Title: Re: #562 – Electroboom!
Post by: jesuscf on November 21, 2021, 05:15:04 am
Fill in the values, if you want. That's a servant job.

On a side note: I have never met an engineer in my life that would say something like that!
Title: Re: #562 – Electroboom!
Post by: Jesse Gordon on November 21, 2021, 07:35:19 am
I had written this post hours ago, before reading Jesse's last rant, and delayed till I had the links and the images, but I did not have to change anything except for a few addition to confirm my deductions.

Jesse believes, and all his measurements confirm it, that:
Voltage is induced locally only where "stuff" is "crossing" the magnetic flux, e.g. in a transformer, only the part of the windings that are "inside" the core contribute to the voltage. Hence only the "2R" resistor acts as a "secondary"

Yes, Jesse is a 'lumper'.
He believes that the core is some sort of magical portal, like a stargate, and when you cross it, when you 'cut through it' with your wires, they magically acquire a voltage, that is there independent of anything else.
Dude, stop misquoting me or trying to stuff words into my mouth.

Can you even read? I've been saying that IT MODELS AND MEASURES as I describe. If you can show voltage being induced across a wire  from being near an ideal toroid transformer when that wire does not pass through said toroid, then show me!

Otherwise, a toroid models and measures just as I described.

But in the case of the infinitely long solenoid, where he cannot find the entrance of the stargate, he is forced to become a 'distributer', because the 'hole' is now the whole space and the magical source of voltage can now be inside the whole length of the conductors and even of the probes. "The probes become the secondary".

Since an infinitely long solenoid is a mythical creature, then I can just say that my probe wires do not interfere with the physical space of said solenoid, and I can run my probe wires right through the center of the solenoid and still measure the voltage induced across a fractional portion of the secondary turn, just like I did with the Lewin Clock, and KVL still holds.

Edit: read another subsequent message before posting this and he appears to confirm that he jumps between positions depending on what suits him best at the moment

Quote from: Jesse Gordon
"However Maxwell and Faraday describe the inner workings is irrelevant to the fact that in the real world a close-magnetic-circuit-cored transformer models as if the entire voltage induced happens at the point that the winding passes through the core."

I have a couple of setups he should try to test his belief with his trusty voltmeter.

GREAT! SHOW ME! That's what I've been asking for going on a year now!

But the reason I keep answering him is not to convince him - ironically he is more of a flatearther of the flatearthers he mocks on his channel - but (among other things) to observe the rejection mechanisms when he is cornered.

Or maybe I'm observing your rejection mechanisms when you're cornered -- which are glaring bob and weave and an utter refusal to come to grips with even the observable part of reality.

If you can't even admit that the output of a closed-magnetic-circuit-core transformer gives every appearance of working with KVL, even though it's glaringly obvious to anyone with a volt meter and such a transformer, then how am I supposed to trust anything else you say?

In fact, how is anybody else supposed to trust anything else you say?


First he needed to reassure himself with that exercise on the transformer and you can see the false equivalence (reiterated in several messages, including the last one)
You act like there's something wrong with doing experiments. Maybe if you did some you wouldn't be so wrong.
Doing experiments is good. It helps you know if you got your theory right or wrong.

Quote from: Jesse Gordon
Alright, the jury is in! bsfeechannel and Sredni couldn't solve it.  .
...But don't feel bad if I am skeptical of your opinion about whether my experiment is wrong -- you couldn't even solve a loaded transformer voltage question that I eventually solved. 
...He couldn't solve the 10:1 loaded transformer question, like you expertly did. 
...that doesn't change the fact that you couldn't solve the transformer quiz 
...so if someone solved it better than me I know they are better than me, and if they can't solve it then I know they are no better than me

"See, you did not do my high school exercise, therefore you cannot do it and since I can, the only possible conclusion it that I know all of EM better than you. Even if I have no idea how to compute the circulation of a vector field".

Then he is trying to reject all analytical reasoning because he cannot handle the math.

Stop lying about me. I'm not rejecting all analytical reasoning, I'm saying "Dude, get a grip with reality. Admit the undeniable, then let's talk about the math and why you think KVL isn't holding even though it gives every appearance of holding."

But you refuse to deal with reality.

As to the 10:1 transformer quiz, I didn't take that from any book, I made that just for you guys.

It looks to me like you're trying lead us to believe you could have solved it but just didn't want to spend the effort.

I'm not buying that for two reasons: One is that the amount of effort you put in here just replying to me already is HUGE. What's solving one little problem you've been solving since you were 17?

The other reason is you told Thinkfat he was solving it wrong. That tells me you didn't know how to solve it either, else you wouldn't have told him he did it wrong when in fact he did it right.

Quote from: Jesse Gordon
You can throw all the math at me you want but that's not going to prove that KVL actually fails. 

to the point that when asked if 5 = 0 his answer is:

Quote from: Jesse Gordon
For exceptionally small versions of five and exceptionally large versions of zero, possibly 


That's a bloody lie and you know it. You asked a bizarre question because 5 obviously does not equal 0. I answered as I did as a joke, and put FIVE laughing-hard emojis, WHICH YOU EDITED OUT to falsely present my partial statement as if I had said it in seriousness.

Furthermore, you also took two lines from TWO DIFFERENT posts and put them one before the other to give a false context to my joke.

That's dirty dude, you must be desperate.

What I really said was:
Quote
For exceptionally small versions of five and exceptionally large versions of zero, possibly  :-DD :-DD :-DD :-DD :-DD

And you actually represented me as having been serious! And the emojis are encoded as Colon Dash D D, so you literally edited them out of my sentence entirely changing the meaning.

That's so low.

So, now all proofs he accepts are those that he himself can do with his limited equipment: a pancake solenoid that has a ton of stray field on the outside (it's basically a multifilar coil), and an EI transformer that will have 'returning legs' on both sides of the ring and nearly no space inside the legs. He also once produced a tiny toroidal core in whose hole he could not fit the probes.
Why do you blather on?

You try to slant everything in such an accusatory tone.

I am working with what I have, I don't get paid to do this, I use what I got. I can't just go buy stuff all the time because you're too lazy and uneducated to do the experiments yourself.

If you think there's a different experiment that will "show me the light" then by all means tell me about it!

I don't mind building a long solenoid. Obviously it can't be infinitely long, but how long does it need to be? Would 10 feet long be long enough? I got magnet wire. I can wind it nicely on a 10ft long PVC plumbing pipe. I can even space the wire 0.1 inches between turns and drill a ring of holes around the pipe before winding it so I can pass my probe wire right through the middle so I can measure the voltage across fractional sections of a turn of a secondary, like I did with my Lewin Clock.

And he rejects experiments that could prove him wrong, like building a long solenoid to confine the field and go all around it or even right above it, or using a transformer with a gap big enough to fit the probes...

Now you're just whining. I've put out significant effort to personally verify observable reality. I've put out effort to show my observations. What've you done?

Besides, I don't remember anyone asking me to build a longer solenoid.

Are you saying that would solve the problem for me? I don't mind building a longer solenoid, although obviously it cannot be infinitely long. Just how long does it need to be to satisfy you?

If you cannot prove he's wrong using the limited setup he has, then he must be right.
Stop the BS dude. You're grabbing at straws.

I've done the experiments I did because I thought that was a good way to test observable reality.

And I've also been asking for a year for a working example that showed I was wrong. It's not my fault if so far nobody has such an example for me to try out.

And by the way, I'm not the only one who can do experiments either - if YOU think you have an experiment that will prove your point, you could either ask someone else to do it or do it yourself.


Quote from: Jesse Gordon
Would it be fair then to say that in the real world, where wires and ferrite/iron cannot exist in the same physical space, that we don't have to worry about our volt meter leads accidentally ending up half way through the cross section of a toroid, and thus in the real world, KVL holds fine 

(Even tho he seems to have changed language: now KVL 'have the appearance of holding' and from one of the latest posts: 'I never said that's actually how Faraday or Maxwell describe it, or even that is how it is working').  So, at least he appears to be right.

Look bud, to me, it seems like KVL works in the cases I've described. I set up the experiment, I used the volt meter. I observed observable reality.

But you seem absolutely unwilling to admit that KVL holds when using the output windings of closed-magnetic-circuit-core transformers.

So I'm trying to meet you half way and say "Can you at least agree that KVL appears to work in this case.." Then we can talk about why you think it's not actually working.

But you even refuse to admit that KVL so much as appears to be working in said configuration.

You really painted yourself into a corner. Regarding this diagram below:
https://i.postimg.cc/fTgyDNp0/20211119-030105.jpg
Regarding diagram above, you said that V2 would work with KVL, but V1 wouldn't. But you also said they are functionally identical.

So you've got yourself in a pickle where even though the two configurations are functionally identical, somehow the voltages will sum to zero in one case but not in the other.

But the most interesting thing I have got from him is his lucid analysis of what in the older thread on this topic I called 'scientific populism'.

Quote from: Jesse Gordon
"In the mean time, we go on to spread the truth that KVL holds when you correctly probe for loops who's size is much smaller than the wavelength of the frequency involved.
More and more people are making videos on the topic, demonstrating Lewin's error and some of them already know way more math than you and me, and the ones that don't know the math aren't going to be helped by your mathematical definition games.
They are going to be influenced by the low-level information about guys like Mehdi and all the other guys taking the practical approach -- because most people identify much more strongly with the practical approach, and won't be helped by you quoting math [...]"

He is 100% right.
This is how politics has always worked: you don't need to tell the truth or correctly solve problems, far from it, you just need to tell your electorate what they want to hear. Make them feel smart, even if they are dumb as rocks, by using their language and indulging their wrong beliefs. They will adore you and they will give you their vote and their money.

And how do we know that you're not just telling your electorate what they want to hear like any good politician worth his salt?

Like I said before, if people like you want people like me to see you as anything other than a religious zealot for your Lewin belief, then you need to meet them at a level they understand and be willing to answer questions head on and be willing to admit to observable reality.

If you duck and dodge every question and refuse to admit things that are plainly observable, they are gonna think you're a quack.

It's up to you of course, but with the amount of effort you put in, I'd think you'd want good results.

But when we see you bobbing and weaving and avoiding questions  and literally refusing to admit to that which we can observe with our own volt meters, we're gonna think you don't know what you're talking about.

Another thing you can do to help people think you're honest and sincere is do some of the challenges they offer you.

Saying you're not going to do their home work is like the biggest copout ever. It raises red flags all over, it tells people that you probably don't know what you're talking about.

"I could but I'm not gonna."

Yeah, no you can't. If you expect people to believe you're honest sincere and know your stuff, you need to be willing to show them that you know your stuff.

And you don't know your stuff. And you know you don't know it, which is why you're so wary of answering questions.

Look, why can't you agree that in the case of a loop composed of resistors and closed-magnetic-circuit-core transformer secondary windings, KVL APPEARS to hold as measured with a volt meter. Why not just admit it?

Then we can go on to talk about why you think KVL isn't ACTUALLY holding even though it appears to be.

The fact is, even if KVL doesn't hold but only appears to, we wouldn't have had such a long conversation, you would have been like "So yeah, it appears to hold, but it's not holding because XYZ..."

How about it? Can you admit the obvious that everyone else can see? KVL at the very least appears to hold in resistor/transformer-secondary winding loops as described above.

Why not admit it?
Title: Re: #562 – Electroboom!
Post by: Jesse Gordon on November 21, 2021, 08:10:07 am
You will never find the emf along the circuit path, like the lumped EMF Kirchhoff experimented with.

You are so wrong on this one that you are even disagreeing with Lewin and everyone else for that matter!

Very well. Since Jesse won't do this experiment, let's see if you are willing to do it (I did it).

Are you a toroidal power transformer owner?
You know one those five inches or so big donuts?

What experiment was it that I wouldn't do again? The only thing I remember refusing to do was to put a secondary winding in series with one of my volt meters because you would not let me account for it in my model.

Hey, You think you can solve the measured voltage using MaxEQ on my lewin clock if I attache my probes to exactly the top and exactly the bottom?

I can tell you the exact total loop induced voltage, and the exact clock position in degrees relative to the resistors, and can you calculate the voltage I will measure on my probes?
(I will not be using my clock's hands, I will be using clip leads clipped to the edge of the copper foil trace.

The catch is I'm not going to tell you what exact path my lead wires take between the clock face and the volt meter.

So MaxEQ will fail too.

Actually, that's a good idea for a future video. I can set up and calculate the predicted voltage using MaxEQ for a false position of the probe leads and then declare that MaxEQ fails too! MaxEQ is for the birds!

I can be like "Hallo hallo hallo! Today I am going to show you something so amazing that you will be telling your great great great grandchildren about it!"
I can even pin a stale fried egg or doughnut to my shirt pocket and wear a clown hat and it would be a lot of fun.

But look, my point to this is that with bad probing or inaccurate modeling, all of reality whether it's KVL or MaxEQ or Physics or Math or anything - everything fails when false assumptions are taken at the foundational level.

Except maybe in poetry and music, I think anything goes there.

Are there cases that are somewheres near impossible to probe accurately? Of course.
Are there other cases that are just difficult to probe? Of course.

But there's also cases which can be unambiguously probed and readily considered a black-box voltage source (or drop) for the sake of KVL, like resistors, solar cells, peltier junctions, and yes, even the outputs of closed-magnetic-circuit-core transformers.

Why not admit it?
Title: Re: #562 – Electroboom!
Post by: Sredni on November 21, 2021, 03:23:28 pm
Why induction EMF is special...

in case you didn't realize it, Faraday's law is one of the many forms you can encounter KVL. 

No. Faraday's law is a fundamental law of nature, that expresses a fundamental property of the electromagnetic field. It tells you that a changing magnetic field is associated with a curling electric field. There is a reason it is one of four Maxwell's equations, the pillars of ALL classical electrodynamics.

...and why you cannot locate it in the closed circuit
Quote
For example, using your same flawed logic, we can argue that a thermocouple circuit is not solved using KVL but the Seebeck effect.  Imagine if we would have to 'name' the solution of every possible circuit based on the nature of its EMF...

You've got it all backwards.

The other sources of EMF lie along the path ("auf dem Wege", remember?), and as such are taken care of by the circulation of E.dl - which is the path integral extended to the whole circuit - that is on the left-hand side. The only form of EMF that falls into the right-hand side is - you guessed it - electromagnetic induction. It's a special one because it is a manifestation of a property of the electromagnetic field and it has its own Maxwell equation, and this is not the first time I wrote that in this thread.


When the CIRCUIT PATH ITSELF runs around a variable flux region, you cannot hide it inside a component accessible through its terminals, because IT IS YOUR FRIGGIN' CIRCUIT PATH, not a part of it. The circuit is unlumpable and you are stuck with Faraday's law. If you mathematically bring the rhs on the lhs, you cannot make it appear as some contribution on the loop ("auf dem Wege", in case you already forgot) for the circulation of E. It is invisible. All you can see are the contribution to the path integral: the drops at the resistors, the boosts at the batteries, the voltage at the thermocouple, the contribute of the solar cell, and of every other component you placed "auf dem Wege", which means 'on the way', along the path. Repetitia juvant, sometimes.

As I said before, it is unfortunate none of you KVLers have a long solenoid or a toroidal transformer because it would have been fun seeing you trying to locate the EMF contribution that's on the right hand side along the circuit path (remember "auf dem Wege"?)


[note] Yes, you can pull the math trick that McDonald uses, by converting the surface integral of B into the path integral of the magnetic vector potential A and then incorporate that into the path integral on the left-hand side. But that will give you the path integral of the conservative electric field Ecoul alone which does not completely describe the system (only half of it, so to speak).
The conservative PART Ecoul of the electric field E admits an electric scalar potential (which obeys what I would call Kirchhoff Scalar Potential Law, so that you KVLers can be happy with a dummy that resembles the high school rule you cannot do without), BUT you have to keep in mind that such potential alone does not completely describe the system. You need to supply the induced electric field Eind (or the vector potential A), that you just subtracted away. Moreover, the difference in scalar potential is not what is measured by a voltmeter, and you can't apply Ohm's law or Joule's heating law using the scalar potential difference alone (in the presence of variable magnetic fields).
Title: Re: #562 – Electroboom!
Post by: jesuscf on November 21, 2021, 04:33:32 pm
Why induction EMF is special...

in case you didn't realize it, Faraday's law is one of the many forms you can encounter KVL. 


No. Faraday's law is a fundamental law of nature, that expresses a fundamental property of the electromagnetic field. It tells you that a changing magnetic field is associated with a curling electric field. There is a reason it is one of four Maxwell's equations, the pillars of ALL classical electrodynamics.

...and why you cannot locate it in the closed circuit
Quote
For example, using your same flawed logic, we can argue that a thermocouple circuit is not solved using KVL but the Seebeck effect.  Imagine if we would have to 'name' the solution of every possible circuit based on the nature of its EMF...


You've got it all backwards.

The other sources of EMF lie along the path ("auf dem Wege", remember?), and as such are taken care of by the circulation of E.dl - which is the path integral extended to the whole circuit - that is on the left-hand side. The only form of EMF that falls into the right-hand side is - you guessed it - electromagnetic induction. It's a special one because it is a manifestation of a property of the electromagnetic field and it has its own Maxwell equation, and this is not the first time I wrote that in this thread.

  • When the magnetic components can be lumped, we can make - with a mathematical trick (take the term from the right side and move it to the left side by changing the sign) - them appear in the form of path integral as a piece of the circulation of E (I have explained how to do that, as well). You know, when instead of following the filament you consider the jump at the terminals (as explained by Hayt, which you quote but seem not to understand)?
  • But when the magnetic 'component' cannot be lumped - i.e. it is unlumpable as the Romer-Lewin ring - you are stuck with the term on the right hand side without possibility to make it appear as part of the circulation of E, BECAUSE THE PATH INTEGRAL ON THE LEFT IS ALREADY COMPLETE AND THERE IS NOWHERE TO FIT THAT TERM. There is no jump at the terminals because the ring is not... a part (that can be made external) to the ring. [note]

When the CIRCUIT PATH ITSELF runs around a variable flux region, you cannot hide it inside a component accessible through its terminals, because IT IS YOUR FRIGGIN' CIRCUIT PATH, not a part of it. The circuit is unlumpable and you are stuck with Faraday's law. If you mathematically bring the rhs on the lhs, you cannot make it appear as some contribution on the loop ("auf dem Wege", in case you already forgot) for the circulation of E. It is invisible. All you can see are the contribution to the path integral: the drops at the resistors, the boosts at the batteries, the voltage at the thermocouple, the contribute of the solar cell, and of every other component you placed "auf dem Wege", which means 'on the way', along the path. Repetitia juvant, sometimes.

As I said before, it is unfortunate none of you KVLers have a long solenoid or a toroidal transformer because it would have been fun seeing you trying to locate the EMF contribution that's on the right hand side along the circuit path (remember "auf dem Wege"?)


[note] Yes, you can pull the math trick that McDonald uses, by converting the surface integral of B into the path integral of the magnetic vector potential A and then incorporate that into the path integral on the left-hand side. But that will give you the path integral of the conservative electric field Ecoul alone which does not completely describe the system (only half of it, so to speak).
The conservative PART Ecoul of the electric field E admits an electric scalar potential (which obeys what I would call Kirchhoff Scalar Potential Law, so that you KVLers can be happy with a dummy that resembles the high school rule you cannot do without), BUT you have to keep in mind that such potential alone does not completely describe the system. You need to supply the induced electric field Eind (or the vector potential A), that you just subtracted away. Moreover, the difference in scalar potential is not what is measured by a voltmeter, and you can't apply Ohm's law or Joule's heating law using the scalar potential difference alone (in the presence of variable magnetic fields).


Wow, interesting!  Every book I have checked say the induced EMF is lumpable (for example, check the attached figure from Electromagnetics for Engineers by Ulaby).  By the way, when you suggested the solution to the problem I posted, what is the first thing you do?  You lumped the EMF!  At this point whom should we believe, the literature (and experiments) or Sredni?  You should write a paper (or ask one of your servants to write it for you) with your amazing 'discovery' and submit it immediately to the IEEE!

Now, you are imposing a lot of conditions for what and how we have to measure the voltages in the loop.  But those same conditions don't seem to apply when Lewin measured the voltages in his circuit.  Hopefully you can see the massive contradictions you are getting into.


Title: Re: #562 – Electroboom!
Post by: thinkfat on November 21, 2021, 04:45:06 pm
Wow, interesting!  Every book I have checked say the induced EMF is lumpable (for example, check the attached figure from Electromagnetics for Engineers by Ulaby).
That's just dumb. The thing that is being "lumped" here is not the EMF, it's the electric potential that "builds up" along a path (of conductor) that is subjected to the EMF.
Title: Re: #562 – Electroboom!
Post by: jesuscf on November 21, 2021, 04:55:33 pm
Wow, interesting!  Every book I have checked say the induced EMF is lumpable (for example, check the attached figure from Electromagnetics for Engineers by Ulaby).
That's just dumb. The thing that is being "lumped" here is not the EMF, it's the electric potential that "builds up" along a path (of conductor) that is subjected to the EMF.

You are right.  Let us call it that VEMF(t) as the book does.
Title: Re: #562 – Electroboom!
Post by: Sredni on November 21, 2021, 05:52:47 pm
Wow, interesting!  Every book I have checked say the induced EMF is lumpable (for example, check the attached figure from Electromagnetics for Engineers by Ulaby). 

You probably have checked fewer books than I have. Purcell does not say so. Ramo Whinnery and VanDuzer does not say so. Haus and Melcher does not say so. Brandao Faria does not say so...
Could it be that the books you checked were talking about lumped circuits?
Because that's what Hayt was doing - he was showing how to "amend KVL" to make it work with LUMPED inductors. (Edit: Components you access via their closed together terminals.)
And so does Ulaby. I have that book, as well, and in my edition that figure is in section 6.2 "Stationary loop in a time-varying magnetic field": it expressly references a lumped inductor which has terminals.

Quote from: Ulaby
"The transformer EMF is the voltage difference that would appear across the small opening between terminals 1 and 2, ..."

You know why the opening has to be small (and the terminals close together)? Because the circuit you put your component in has to be shrunk to a dimensionless point. Has to be lumped. Ulaby is talking about a lumpable circuit that is being lumped.

Quote
By the way, when you suggested the solution to the problem I posted, what is the first thing you do?  You lumped the EMF!

No, I applied Faraday. I can do that even with lumpable circuits, if I choose not to lump them. Here's one picture from one of my answers on EE Stack Exchange where I properly apply Faraday to a lumpable RLC circuit (the 5 + 3 + 0 = 8 approach)

(https://i.stack.imgur.com/d43IA.png)

and here is how I can turn Faraday into "amended KVL" for the same circuit system - which I now treat as lumped (the 5 + 3 - 8 = 0 approach) by using a circuit path that does not inclued the variable magnetic region

(https://i.stack.imgur.com/yIJqh.png)

If the circuit is lumpABLE I can treat is either as lumpED (applying 'extended KVL') or as NON lumped (applying Faraday).
If the circuit is UNlumpABLE, I am stuck with Faraday.

It's that simple. Really.

Quote
At this point whom should we believe, the literature (and experiments) or Sredni?

You could believe Purcell, Ramo Whinnery Vanduzer, Haus and Melcher, Faria, Romer, Roche, Nicholson, ... basically any professor who explicitly treated the general case, and did not limit their discussion to lumped circuits alone.

Title: Re: #562 – Electroboom!
Post by: jesuscf on November 21, 2021, 08:06:04 pm
Wow, interesting!  Every book I have checked say the induced EMF is lumpable (for example, check the attached figure from Electromagnetics for Engineers by Ulaby). 

You probably have checked fewer books than I have. Purcell does not say so. Ramo Whinnery and VanDuzer does not say so. Haus and Melcher does not say so. Brandao Faria does not say so...
Could it be that the books you checked were talking about lumped circuits?
Because that's what Hayt was doing - he was showing how to "amend KVL" to make it work with LUMPED inductors. (Edit: Components you access via their closed together terminals.)
And so does Ulaby. I have that book, as well, and in my edition that figure is in section 6.2 "Stationary loop in a time-varying magnetic field": it expressly references a lumped inductor which has terminals.

Quote from: Ulaby
"The transformer EMF is the voltage difference that would appear across the small opening between terminals 1 and 2, ..."

You know why the opening has to be small (and the terminals close together)? Because the circuit you put your component in has to be shrunk to a dimensionless point. Has to be lumped. Ulaby is talking about a lumpable circuit that is being lumped.

Quote
By the way, when you suggested the solution to the problem I posted, what is the first thing you do?  You lumped the EMF!

No, I applied Faraday. I can do that even with lumpable circuits, if I choose not to lump them. Here's one picture from one of my answers on EE Stack Exchange where I properly apply Faraday to a lumpable RLC circuit (the 5 + 3 + 0 = 8 approach)

(https://i.stack.imgur.com/d43IA.png)

and here is how I can turn Faraday into "amended KVL" for the same circuit system - which I now treat as lumped (the 5 + 3 - 8 = 0 approach) by using a circuit path that does not inclued the variable magnetic region

(https://i.stack.imgur.com/yIJqh.png)

If the circuit is lumpABLE I can treat is either as lumpED (applying 'extended KVL') or as NON lumped (applying Faraday).
If the circuit is UNlumpABLE, I am stuck with Faraday.

It's that simple. Really.

Quote
At this point whom should we believe, the literature (and experiments) or Sredni?

You could believe Purcell, Ramo Whinnery Vanduzer, Haus and Melcher, Faria, Romer, Roche, Nicholson, ... basically any professor who explicitly treated the general case, and did not limit their discussion to lumped circuits alone.

Fantastic!  Now here is another problem for you.  The same 'Lewin' ring as before with a AA battery in series (as show in the attached figure).  Compute the voltages V1 and V2 at t=0.  Also, the battery, although slowly, is discharging over time, but at t=0 the voltage between the terminals of the battery 1.5V.
Title: Re: #562 – Electroboom!
Post by: Jesse Gordon on November 21, 2021, 08:41:58 pm
voltage between nodes A and D (VAD) which are half a circle apart.[/b]

As I've stated, the definition of that voltage is the very issue that is not agreed upon.  So if you want a number for an answer, you have to specify your definition or method of measurement.  One definition that might be interesting is the voltage measured by a voltmeter whose leads go in an exact straight line between the points.  You would need a gap or hole in the inductor and core that are providing the magnetic flux, but that may be doable.

Or one could use the definition of voltage given by the International Electrotechnical Committe IEC 60050
https://www.electropedia.org/iev/iev.nsf/display?openform&ievref=121-11-27 (https://www.electropedia.org/iev/iev.nsf/display?openform&ievref=121-11-27)
and specify the path, as I did.

So let me ask you this. In the following diagram:
(https://i.postimg.cc/pXFDpYwk/deleteme.jpg)
https://i.postimg.cc/pXFDpYwk/deleteme.jpg (https://i.postimg.cc/pXFDpYwk/deleteme.jpg)
In the above diagram, I show the test circuit.

In the below image:
(https://i.postimg.cc/9QctKwkp/Screenshot-2021-11-21-at-12-23-35-IEC-60050-International-Electrotechnical-Vocabulary-Details-fo.png)
https://i.postimg.cc/9QctKwkp/Screenshot-2021-11-21-at-12-23-35-IEC-60050-International-Electrotechnical-Vocabulary-Details-fo.png (https://i.postimg.cc/9QctKwkp/Screenshot-2021-11-21-at-12-23-35-IEC-60050-International-Electrotechnical-Vocabulary-Details-fo.png)

In the above image, I show a screenshot of the IEC definition you reference.

NOW: Using your provided definition, and the circle path diagram above that, in your opinion or by your calculations, what will be the voltage between the points connected by the non-conductive green line, assuming a 1v/turn induced voltage for the whole loop?

Assume that the upper section encloses exactly half of the dB/dt, and that the wires are superconductors.
Also assume that the solenoid is infinitely long.

As I see it, you have yourself in a bit of a bind.

If you say "Can't be known" then you're obviously using the wrong definition for volts.

If you say zero, then transformers have zero volts output, which is obviously false.

If you say non-zero, then Lewin goofed by not accounting for the voltage induced in his probe leads between the "point of measurement" and the "volt meter."

So what's the voltage across the two wire ends shown by the green path?
Title: Re: #562 – Electroboom!
Post by: bsfeechannel on November 21, 2021, 08:55:18 pm
Wow, interesting!  Every book I have checked say the induced EMF is lumpable

Nope. They don't say that. It's all in your head.

In that specific case, that EMF is "lumpable" because the geometry of the circuit allows it. The varying magnetic field is contained in a specific area of the circuit. What is omitted from that diagram is the path taken to calculate the EMF. The path doesn't encompass the area where the varying mag field is. Note also how the author says the loop is stationary.
Title: Re: #562 – Electroboom!
Post by: jesuscf on November 21, 2021, 09:04:46 pm
Wow, interesting!  Every book I have checked say the induced EMF is lumpable

Nope. They don't say that. It's all in your head.

In that specific case, that EMF is lumpable because the geometry of the circuit allows it. The varying magnetic field is contained in a specific area of the circuit. What is omitted from that diagram is the path taken to calculate the EMF. The path doesn't encompass the area where the varying mag field is. Note also how the author says the loop is stationary.

Sure, why not; show me: in the Lewin's circuit I posted above, solve for V1 and V2 without lumping the EMF.
Title: Re: #562 – Electroboom!
Post by: thinkfat on November 21, 2021, 09:06:07 pm
If you say "Can't be known" then you're obviously using the wrong definition for volts.

If you say zero, then transformers have zero volts output, which is obviously false.

If you say non-zero, then Lewin goofed by not accounting for the voltage induced in his probe leads between the "point of measurement" and the "volt meter."

So what's the voltage across the two wire ends shown by the green path?

I don't think the conclusions you draw from the possible answers are valid.
But I can also predict that you will not understand, and reject, the answer once given.  :-DD
Title: Re: #562 – Electroboom!
Post by: Sredni on November 21, 2021, 09:55:28 pm
No, I applied Faraday. I can do that even with lumpable circuits, if I choose not to lump them. Here's one picture from one of my answers on EE Stack Exchange where I properly apply Faraday to a lumpable RLC circuit (the 5 + 3 + 0 = 8 approach)
(https://i.stack.imgur.com/d43IA.png)

Fantastic!  Now here is another problem for you.  The same 'Lewin' ring as before with a AA battery in series (as show in the attached figure).  Compute the voltages V1 and V2 at t=0.  Also, the battery, although slowly, is discharging over time, but at t=0 the voltage between the terminals of the battery 1.5V.

The solution is in the image you quoted. It's fascinating that you do not realize it.
Just switch the capacitor with a resistor, and there you will see how to apply Faraday to your circuit, and how the right hand side only features the induction EMF, while the battery EMF is accounted for as part of the circulation.
You really can't see it, if you think putting a battery in Lewin's ring can represent some sort of difficulty.

Anyway, I solved step by step (again for the general case, I am not interested in exercises) and I tried to spell out every single step - including sign conventions and the many parts you can break the circulation path integral (something you can read, certainly better explained in Ramo Whinnery and VanDuzer - but you won't read it, right?), and how the direction of the current depends on who wins between the induction EMF and the battery.

But before scanning these two pages, I want that some KVLer do the measurements I propose.
(Post to come tomorrow)
Title: Re: #562 – Electroboom!
Post by: Sredni on November 21, 2021, 10:02:30 pm
If you say "Can't be known" then you're obviously using the wrong definition for volts.

If you say zero, then transformers have zero volts output, which is obviously false.

If you say non-zero, then Lewin goofed by not accounting for the voltage induced in his probe leads between the "point of measurement" and the "volt meter."

So what's the voltage across the two wire ends shown by the green path?

I don't think the conclusions you draw from the possible answers are valid.
But I can also predict that you will not understand, and reject, the answer once given.  :-DD

So, you too have a crystal ball!
Title: Re: #562 – Electroboom!
Post by: jesuscf on November 21, 2021, 10:11:10 pm
No, I applied Faraday. I can do that even with lumpable circuits, if I choose not to lump them. Here's one picture from one of my answers on EE Stack Exchange where I properly apply Faraday to a lumpable RLC circuit (the 5 + 3 + 0 = 8 approach)
(https://i.stack.imgur.com/d43IA.png)

Fantastic!  Now here is another problem for you.  The same 'Lewin' ring as before with a AA battery in series (as show in the attached figure).  Compute the voltages V1 and V2 at t=0.  Also, the battery, although slowly, is discharging over time, but at t=0 the voltage between the terminals of the battery 1.5V.

The solution is in the image you quoted. It's fascinating that you do not realize it.
Just switch the capacitor with a resistor, and there you will see how to apply Faraday to your circuit, and how the right hand side only features the induction EMF, while the battery EMF is accounted for as part of the circulation.
You really can't see it, if you think putting a battery in Lewin's ring can represent some sort of difficulty.

Anyway, I solved step by step (again for the general case, I am not interested in exercises) and I tried to spell out every single step - including sign conventions and the many parts you can break the circulation path integral (something you can read, certainly better explained in Ramo Whinnery and VanDuzer - but you won't read it, right?), and how the direction of the current depends on who wins between the induction EMF and the battery.

But before scanning these two pages, I want that some KVLer do the measurements I propose.
(Post to come tomorrow)

What you are telling me by making an equivalence from my circuit to your circuit is that you lumped the induced EMF and them you used KVL to solve it!  Great to know!!!
Title: Re: #562 – Electroboom!
Post by: Sredni on November 21, 2021, 10:12:05 pm
So let me ask you this. In the following diagram:
(https://i.postimg.cc/pXFDpYwk/deleteme.jpg)
Assume that the upper section encloses exactly half of the dB/dt, and that the wires are superconductors.
Also assume that the solenoid is infinitely long.
So what's the voltage across the two wire ends shown by the green path?

Let's see if you can forecast my answer by looking at this image

(https://i.postimg.cc/15n8XGXJ/Voltage-can-be-path-dependent.jpg)
https://i.postimg.cc/15n8XGXJ/Voltage-can-be-path-dependent.jpg

Some hints:
What color would I use for that kind of path?
What color would I use for the probes in Lewin's setup?
Title: Re: #562 – Electroboom!
Post by: Sredni on November 21, 2021, 10:21:28 pm
What you are telling me by making an equivalence from my circuit to your circuit is that you lumped the induced EMF and them you used KVL to solve it!

You really can't see it, can you?
You're like that guy in Kevin Smith's "Mallrats" movie. The guy who spend hours in front of the stereoscopic poster and everybody else can tell what the image is, while he can't see it.

(http://www.austinchronicle.com/binary/3015/maxresdefault__6_.jpg)
Title: Re: #562 – Electroboom!
Post by: bsfeechannel on November 21, 2021, 10:49:42 pm
McDonald: ... He uses diversion and an obsolete version of a "law" to create an apparent paradox.

That is the root of the controversy.  Everything else is noise.

Lewin thinks that this is the definition of KVL:

\$\oint_{}^{}E.dL=0\$


Aaaaaand Lewin is right. KVL holds for every circuit for which \$\oint_{}^{}E.dL=0\$. If you calculate \$\oint_{}^{}E.dL\$ and it results in zero, you can apply KVL blind-folded. You can even bet your life on it.

So it is ironic that in Lewin's world, KVL always holds. ALWAYS. The only thing you need to do is to find a path for which  \$\oint_{}^{}E.dL=0\$ and you're good to go.

Quote
While this is the accepted definition of KVL (since at least 1909, the earliest reference I found, but must likely older than that):

\$emf=\oint_{}^{}E.dL=-\frac{d }{dt}\int_{S}^{}B.dS\$

Uhh, nope. According to McDonald, this is the more accepted definition of KVL:

0 = ∑loop voltage drops = ∑iIiRr + ∑j( ̇Ij Lj + ∑k ̇IkMjk) + ∑l Ql/Cl −∑mEm  (1)

And he remarks:
Lewin has also supplemented his lectures with two notes [34, 35], where is it claimed that Kirchhoff’s (2nd)
circuit law is that ∮loop E·dl= 0 for any circuit loop, rather than the more standard version, eq. (1), for
(coupled) R-L-C circuits.


Bold letters are mine.

So, it holds with the following caveats:

Caveat #1 Your circuit must be made exclusively of lumped components (R, L, C). I.e. it must be lumpable.
Caveat #2 Some unlumpable circuits can be solved by the equation (1), however the EMF can in no way be measured.
Caveat #3 It doesn't hold for whatever circuits where you have radiation.
Caveat #4 It doesn't hold for whatever circuits where its size is comparable to the wavelength.
Caveat #5 It doesn't take into consideration the self inductance of the rest of the circuit.
Caveat #6 While Lewin's version is absolutely precise. This version is an approximation.
Caveat #7 Mehdi says, however, that this "version" of KVL always holds. Which is not true.
Caveat #8 Mehdi is stupid.
Caveat #9 Stupidity is a moral issue. Not and intellectual one.

So, in Mehdi's world KVL doesn't always hold. The reason why it doesn't requires a little study, but Mehdi wants to convey the idea that you can be an instant expert in electromagnetism without the proper education. Welcome to the 21st century. I'm loving it.
Title: Re: #562 – Electroboom!
Post by: bsfeechannel on November 21, 2021, 11:15:10 pm
I'm not saying that there are electric fields INSIDE a conductor, stationary or otherwise, that we know because the -∫E.dl=0 for a superconductor.
I am talking about the EMF across the ends of a wire according to -L(dI/dt), which Belcher clearly describes as being non-zero in a changing magnetic field even for a superconductor.

Precisely. If you "walk" between the ends through the conductor, you will find zero volts. If you "walk" between the same ends, but now chosing the gap between the ends as your path, you'll find your non-zero voltage. These two voltages are different, although they refer to the same two points.

End of story. Have a good night's sleep.

Quote
That should be the second thing you learned in your electromagnetism course out there.

The sole purpose of an engineering degree is to able to debunk KVLers.

Quote
Are you thinking there's a difference between a stationary wire in a dB/dt field, and a moving wire in a stationary and non-uniform B field?

Before you set out to give answers on Youtube, learn to ask the proper questions.
Title: Re: #562 – Electroboom!
Post by: jesuscf on November 21, 2021, 11:17:49 pm
What you are telling me by making an equivalence from my circuit to your circuit is that you lumped the induced EMF and them you used KVL to solve it!

You really can't see it, can you?
You're like that guy in Kevin Smith's "Mallrats" movie. The guy who spend hours in front of the stereoscopic poster and everybody else can tell what the image is, while he can't see it.


And in the diagram you put there is the electric field in the circuit conservative or non-conservative?  Because it looks to me that you are applying KVL despite having a non-conservative electric field in the circuit, which is correct.
Title: Re: #562 – Electroboom!
Post by: jesuscf on November 21, 2021, 11:21:23 pm
McDonald: ... He uses diversion and an obsolete version of a "law" to create an apparent paradox.

That is the root of the controversy.  Everything else is noise.

Lewin thinks that this is the definition of KVL:

\$\oint_{}^{}E.dL=0\$


Aaaaaand Lewin is right. KVL holds for every circuit for which \$\oint_{}^{}E.dL=0\$. If you calculate \$\oint_{}^{}E.dL\$ and it results in zero, you can apply KVL blind-folded. You can even bet your life on it.

So it is ironic that in Lewin's world, KVL always holds. ALWAYS. The only thing you need to do is to find a path for which  \$\oint_{}^{}E.dL=0\$ and you're good to go.

Quote
While this is the accepted definition of KVL (since at least 1909, the earliest reference I found, but must likely older than that):

\$emf=\oint_{}^{}E.dL=-\frac{d }{dt}\int_{S}^{}B.dS\$

Uhh, nope. According to McDonald, this is the more accepted definition of KVL:

0 = ∑loop voltage drops = ∑iIiRr + ∑j( ̇Ij Lj + ∑k ̇IkMjk) + ∑l Ql/Cl −∑mEm  (1)

And he remarks:
Lewin has also supplemented his lectures with two notes [34, 35], where is it claimed that Kirchhoff’s (2nd)
circuit law is that ∮loop E·dl= 0 for any circuit loop, rather than the more standard version, eq. (1), for
(coupled) R-L-C circuits.


Bold letters are mine.

So, it holds with the following caveats:

Caveat #1 Your circuit must be made exclusively of lumped components (R, L, C). I.e. it must be lumpable.
Caveat #2 Some unlumpable circuits can be solved by the equation (1), however the EMF can in no way be measured.
Caveat #3 It doesn't hold for whatever circuits where you have radiation.
Caveat #4 It doesn't hold for whatever circuits where its size is comparable to the wavelength.
Caveat #5 It doesn't take into consideration the self inductance of the rest of the circuit.
Caveat #6 While Lewin's version is absolutely precise. This version is an approximation.
Caveat #7 Mehdi says, however, that this "version" of KVL always holds. Which is not true.
Caveat #8 Mehdi is stupid.
Caveat #9 Stupidity is a moral issue. Not and intellectual one.

So, in Mehdi's world KVL doesn't always hold. The reason why it doesn't requires a little study, but Mehdi wants to convey the idea that you can be an instant expert in electromagnetism without the proper education. Welcome to the 21st century. I'm loving it.

Excellent!  So according to what you say above KVL, holds for any EMF that is not induced by a varying magnetic field?  If for some reason you have a EMF induced by a magnetic field in series with another source of EMF, what do you do then?  Add then together?  Isn't it that KVL?
Title: Re: #562 – Electroboom!
Post by: jesuscf on November 22, 2021, 12:29:32 am
Here is the solution of the problem I posted yesterday.

The given time varying magnetic flux density is:

\$B(t) = 0.7958 \cdot e^{\frac{{ - t}}{{0.1s}}} _{} T\$

The magnetic flux flowing through the ring is:

\$
\begin{array}{l}
 \Phi  = \int\limits_S {B(t) \cdot dS = } \int\limits_S {0.7958 \cdot e^{\frac{{ - t}}{{0.1s}}} _{} T}  \\
 \Phi  = 0.7958 \cdot e^{\frac{{ - t}}{{0.1s}}} _{} T \cdot \pi  \cdot (0.2)^2 m^2  \\
 \Phi  = 0.1 \cdot e^{\frac{{ - t}}{{0.1s}}} Wb \\
 \end{array}
\$

The corresponding EMF is:

\$
EMF =  - \frac{{d\Phi }}{{dt}} =  - \frac{{d(0.1 \cdot e^{\frac{{ - t}}{{0.1s}}} Wb)}}{{dt}} = \frac{{ - 0.1}}{{ - 0.1s}}e^{\frac{{ - t}}{{0.1s}}} Wb = e^{\frac{{ - t}}{{0.1s}}} V
\$

At t=0 the EMF is then:

\$
EMF = e^{\frac{{ - 0}}{{0.1s}}} V = 1V
\$

Use KVL to find the current, which is flowing clockwise:

\$
I = \frac{{EMF}}{{R_1  + R_2 }} = \frac{{1V}}{{100\Omega  + 900\Omega }} = 1mA
\$

Therefore:

\$
\begin{array}{l}
 V_2  = R_2  \cdot I = 900\Omega  \cdot 1mA = 0.9V \\
 V_1  =  - R_1  \cdot I =  - 100\Omega  \cdot 1mA =  - 0.1V \\
 \end{array}
\$

As for VAD we need to find an equivalent circuit and solve using KVL.  Since one quarter loop connects the '+' terminal of each resistor to node 'A', we can represent that quarter loop as voltage source with one quarter of the total voltage of 0.25V.  The same goes for the '-' terminal of both resistors.  By the way, this is the step that Lewin solved incorrectly!

(https://i.postimg.cc/g051NsTW/Example-KVL-VAD.jpg)

We can find now VAD using KVL on one (or both, it is the same value) of the branches:

\$
 \begin{array}{l}
 V_{AD}  = 0.25V - 0.1V + 0.25V = 0.4V \\
 V_{AD}  =  -0.25 + 0.9V - 0.25V =  0.4V \\
 \end{array}
\$

EDIT: corrected calculation of VAD and added image of equivalent circuit.


Title: Re: #562 – Electroboom!
Post by: Jesse Gordon on November 22, 2021, 03:33:04 am
If you say "Can't be known" then you're obviously using the wrong definition for volts.

If you say zero, then transformers have zero volts output, which is obviously false.

If you say non-zero, then Lewin goofed by not accounting for the voltage induced in his probe leads between the "point of measurement" and the "volt meter."

So what's the voltage across the two wire ends shown by the green path?

I don't think the conclusions you draw from the possible answers are valid.
But I can also predict that you will not understand, and reject, the answer once given.  :-DD

So, you too have a crystal ball!



That answers a lot! It's the three gypsy  Fortune Tellers with their Crystal Balls!

I thought this felt like a three ring circus, and now I know why!

(Lewin rings, of course.)

 :-DD :-DD :-DD :-DD :-DD :-DD
Title: Re: #562 – Electroboom!
Post by: Jesse Gordon on November 22, 2021, 04:01:01 am
I'm not saying that there are electric fields INSIDE a conductor, stationary or otherwise, that we know because the -∫E.dl=0 for a superconductor.
I am talking about the EMF across the ends of a wire according to -L(dI/dt), which Belcher clearly describes as being non-zero in a changing magnetic field even for a superconductor.

Precisely. If you "walk" between the ends through the conductor, you will find zero volts. If you "walk" between the same ends, but now chosing the gap between the ends as your path, you'll find your non-zero voltage. These two voltages are different, although they refer to the same two points.

End of story. Have a good night's sleep.

Quote
That should be the second thing you learned in your electromagnetism course out there.

The sole purpose of an engineering degree is to able to debunk KVLers.

Too bad you didn't get an engineering degree, you might have been able to debunk KVLers if you had!   :-DD

Quote
Are you thinking there's a difference between a stationary wire in a dB/dt field, and a moving wire in a stationary and non-uniform B field?

Before you set out to give answers on Youtube, learn to ask the proper questions.

It's awesome how refuse to answer questions because you know it'd betray your lack of learning.

I know the right questions to ask, but you won't answer them.

Let me try again:

Does KVL hold for a loop consisting of resistors and closed-magnetic-circuit-core transformer secondary windings?

For example, would V1 and V2 in the following diagram work as lumped elements in a loop with some resistors, and would KVL hold?

(https://i.postimg.cc/fTgyDNp0/20211119-030105.jpg)
https://i.postimg.cc/fTgyDNp0/20211119-030105.jpg (https://i.postimg.cc/fTgyDNp0/20211119-030105.jpg)

Specifically, would KVL hold in the following circuit?
(https://i.postimg.cc/15gbsCmz/20211119-232948.jpg)
https://i.postimg.cc/15gbsCmz/20211119-232948.jpg (https://i.postimg.cc/15gbsCmz/20211119-232948.jpg)

Would V1, V2, & V3 sum to zero?

Why are you guys so cagey about admitting that KVL at least APPEARS to hold in this scenario?

I say APPEARS because you won't admit that it DOES hold, so I'm trying to meet you half way, and I'm asking you only to agree to the undeniably obvious fact that at the very least KVL appears to hold as measured with a volt meter.

Once you can come to grips with the plainly observable part of reality then we can discuss why you think it's not ACTUALLY holding even though it gives every appearance of holding.

But if you deny observable reality, why should anybody trust you about the more complex subjective claims you make?

And by the way, it's not ME giving answers on yourtube, it is OBSERVABLE REALITY. I use real test equipment, real transformers, real wires, and real ETC., to show REALITY. That's just what reality is.
Title: Re: #562 – Electroboom!
Post by: Jesse Gordon on November 22, 2021, 04:51:20 am
So let me ask you this. In the following diagram:
(https://i.postimg.cc/pXFDpYwk/deleteme.jpg)
Assume that the upper section encloses exactly half of the dB/dt, and that the wires are superconductors.
Also assume that the solenoid is infinitely long.
So what's the voltage across the two wire ends shown by the green path?

Let's see if you can forecast my answer by looking at this image

(https://i.postimg.cc/15n8XGXJ/Voltage-can-be-path-dependent.jpg)
https://i.postimg.cc/15n8XGXJ/Voltage-can-be-path-dependent.jpg

Some hints:
What color would I use for that kind of path?
What color would I use for the probes in Lewin's setup?

Goodness you dodge and weave when asked questions.

But look at the diagram you provided. From that any reasonable person would conclude that the paths which do not collide with the core material, (i.e. either completely inside or completely outside) are lumped. Only the ones that have an unknown position part way into the dB/dt area should be unlumpable according to your upper diagram.

And yet you show all those paths that are outside and all the paths that are inside as unplumpable. Why aren't they lumpable? They are consistent, unambiguous, and by George they work with KVL as measured with a volt meter.


 Tell me more about your crystal ball. Did you get it in gypsy class?
Title: Re: #562 – Electroboom!
Post by: Sredni on November 22, 2021, 05:26:09 am

Goodness you dodge and weave when asked questions.

But look at the diagram you provided. From that any reasonable person would conclude that the paths which do not collide with the core material, (i.e. either completely inside or completely outside) are lumped.


You know, I don't think you know the meaning of lumped. You try to apply it to paths? What is a lumpable path? I apply it to circuits (you need to lump them, squeeze them to make them as small as a point, dimensionless) or components (same). And why can't you lump a circuit or a component that has a magnetic flux region inside? Because flux needs an area to have a meaning. You can't squeeze it into a point without losing meaning. That's the point (pun intended). But if the component has terminals close together...

But what is a 'lumped path'? Where did you heard of this?
A lumpABLE system can be considered lumpED or UNlumped depending on what you consider its circuit path. If the circuit path contains the magnetic flux region, you cannot squeeze it beyond the perimeter of that region. If the component contain the flux region, you take the terminals out, put them very close together so that they can occupy (so to speak) the space of a point on your circuit and attach them to a gap as wide as a point (so to speak) on your circuit path. Your circuit path, including the connection to the component, is now shrinkable to a point.

What the heck is a 'lumpable path'?

Here is the answer to your previous quiz
You talk about measuring reality, so this is my prediction for your circuit. If you don't like the polarities flip the signs.

(https://i.postimg.cc/Yq1ZDcPY/KVL-works-if-I-leave-out-the-magnetic-region.jpg)
https://i.postimg.cc/Yq1ZDcPY/KVL-works-if-I-leave-out-the-magnetic-region.jpg

Note that you won't be able to measure any voltage in the perfectly conducting wire of the ring, in accordance with Ohm's law. So, are you a lumper, or today you decide that there is voltage in the wires and in the probes? I have to ask.

Now, it's my turn.

Same circuit (you can even use a single resistor if you want. My prediction is that you will observe zero volts in the wires even when the sliders (you sliding the probe tips along the ring) get in contact inside the core.
So how does KVL "appear" to hold?

(https://i.postimg.cc/CKfjJ0PV/KVL-dies-if-the-magnetic-region-is-inside-my-circuit-path.jpg)
https://i.postimg.cc/CKfjJ0PV/KVL-dies-if-the-magnetic-region-is-inside-my-circuit-path.jpg

Now, if today you are still a lumper, please tell me where is the EMF on the ring.
Title: Re: #562 – Electroboom!
Post by: Sredni on November 22, 2021, 05:39:14 am

\$
\begin{array}{l}
 V_2  = R_2  \cdot I = 900\Omega  \cdot 1mA = 0.9V \\
 V_1  =  - R_1  \cdot I =  - 100\Omega  \cdot 1mA =  - 0.1V \\
 \end{array}
\$

As for VAD: any two nodes that are half a circle apart have half of the total EMF.  In this case |VAD|=0.5V.

I see you used Ohm's law to compute the voltage drop along the resistors. So Ohm's law works. Wonderful.
Can you apply Ohm's law to compute the voltage drop across the wire that goes from R1 to point A?
Because we know that from A to D we have a voltage of half a volt, while across R1 we have what 0.1 V (fix the signs as you please while I put popcorns in the microwave). So, what gives?

(you can use the resistivity of copper, or a very low resistance for the wires, for example 1 milliohm, if zero resistance is a problem)
Title: Re: #562 – Electroboom!
Post by: Jesse Gordon on November 22, 2021, 05:48:26 am
Yiiieee! Here comes another KVL'er! Get your crystal balls started, Gentlemen!
(https://i.postimg.cc/5NsPHxQ4/3Gypsies.jpg)
The 3 gypsies, the last bastion of truth.
Title: Re: #562 – Electroboom!
Post by: Jesse Gordon on November 22, 2021, 08:17:51 am

Goodness you dodge and weave when asked questions.

But look at the diagram you provided. From that any reasonable person would conclude that the paths which do not collide with the core material, (i.e. either completely inside or completely outside) are lumped.


You know, I don't think you know the meaning of lumped. You try to apply it to paths? What is a lumpable path? I apply it to circuits (you need to lump them, squeeze them to make them as small as a point, dimensionless) or components (same). And why can't you lump a circuit or a component that has a magnetic flux region inside? Because flux needs an area to have a meaning. You can't squeeze it into a point without losing meaning. That's the point (pun intended). But if the component has terminals close together...

But what is a 'lumped path'? Where did you heard of this?

Again, you're quoting me out of context. My comment was in reply to your diagram where you showed different hypothetical paths within an element:
(https://i.postimg.cc/VvFWycbH/Voltage-can-be-path-dependent.jpg)
https://i.postimg.cc/VvFWycbH/Voltage-can-be-path-dependent.jpg (https://i.postimg.cc/VvFWycbH/Voltage-can-be-path-dependent.jpg)
You clearly show different possible paths within the element, the 5v and 0v paths inside that element are unambiguous,  unvarying, and if that is the path in that element, then it can be considered a lumped element in a KVL loop and KVL will hold.

You already admitted that there is no practical difference between V1 and V2 configurations in this diagram:
(https://i.postimg.cc/fTgyDNp0/20211119-030105.jpg)
https://i.postimg.cc/fTgyDNp0/20211119-030105.jpg (https://i.postimg.cc/fTgyDNp0/20211119-030105.jpg)
You also admitted that V2  in the above diagram would work as a lumped element in a KVL loop.

So if V1 and V2 are functionally identical, and V2 holds in a KVL loop, then why not V1? They have the same voltage. They are functionally identical. What's the difference?


A lumpABLE system can be considered lumpED or UNlumped depending on what you consider its circuit path. If the circuit path contains the magnetic flux region, you cannot squeeze it beyond the perimeter of that region. If the component contain the flux region, you take the terminals out, put them very close together so that they can occupy (so to speak) the space of a point on your circuit and attach them to a gap as wide as a point (so to speak) on your circuit path. Your circuit path, including the connection to the component, is now shrinkable to a point.

If configuration V2 and configuration V1 in my above yellow diagram are functionally identical, then why does it matter?

ARE YOU SAYING THAT THE TERMINALS MUST COME VERY CLOSE TOGETHER OR MY VOLT READINGS WILL BE DIFFERENT?

You already admitted that the V1 and V2 readings in my above yellow diagram would be identical, so why does it matter if the wires coming out of the transformer have to come near eachother? Does that change the voltage? Does it change whether my volt meter readings will all add up to zero around the loop of which that transformer secondary is an element?

What the heck is a 'lumpable path'?

As stated above. You quoted me out of the context of the diagram I was replying to.

Here is the answer to your previous quiz
You talk about measuring reality, so this is my prediction for your circuit. If you don't like the polarities flip the signs.

(https://i.postimg.cc/R04QGyHs/KVL-works-if-I-leave-out-the-magnetic-region.jpg)
https://i.postimg.cc/R04QGyHs/KVL-works-if-I-leave-out-the-magnetic-region.jpg

I see you noticed that you made the same mistake Lewin did and you got one of the volt meters backwards, but at least you caught it in time to put a text note that says I can flip the volt sign.
So yes, I'd like to flip the sign on the left-hand meter which reads 1V. You have to keep the meters all pointing clockwise or counter clockwise -- but all the same way for a given test -- around the loop. That's just the way KVL is. If you mix up your volt meter signs even with pure batteries and resistors, KVL will appear to fail then too.

With all the volt meters pointing plus-clockwise, then you can see that the voltage sums to zero, just like KVL says.

So do we agree that  KVL at least APPEARS to hold on your above diagram, since taken in a positive clockwise series of element voltage difference readings, all the voltage differences sum up to zero?

Note that you won't be able to measure any voltage in the perfectly conducting wire of the ring, in accordance with Ohm's law. So, are you a lumper, or today you decide that there is voltage in the wires and in the probes? I have to ask.

I don't understand the question. I think you're making a dichotomy between a lumper and there being voltage IN the wires.. but I'm not sure what you're trying to ask. If a wire has induced voltage, I'd rather say that it has induced voltage across it.

I don't know what you mean by "lumper" - I say I'm just an observer of reality.  I did not decide anything today.

Try your question again more clearly please.

But maybe this answers you. Like I said a hundred times already, my argument is that in your diagram above (and the others functionally the same), the wire passing through the transformer core MODELS AND MEASURES as if the voltage is induced at the point the wire passes through the inner most area INSIDE the core, and the wires outside the core MODEL AND MEASURE as if they there is no voltage induced across them.

Once you realize these self evidence facts, then I'd love to go on to talk about what you think is really going on.

But getting you to admit to the obvious and undeniable reality of how things model and measure has been like pulling teeth from a whale.

So do we agree then that in a setup as you show in your diagram above, the transformer secondary winding MODELS AND MEASURES as if the entire induced voltage is where the wire passes through the core, and that the wires outside the core MODEL AND MEASURE as if there is no voltage induced in them?

Now, it's my turn.

Same circuit (you can even use a single resistor if you want. My prediction is that you will observe zero volts in the wires even when the sliders (you sliding the probe tips along the ring) get in contact inside the core.
So how does KVL "appear" to hold?

(https://i.postimg.cc/cH5RxJ52/KVL-dies-if-the-magnetic-region-is-inside-my-circuit-path.jpg)
https://i.postimg.cc/cH5RxJ52/KVL-dies-if-the-magnetic-region-is-inside-my-circuit-path.jpg

Now, if today you are still a lumper, please tell me where is the EMF on the ring.

KVL continues to hold once you MODEL REALITY.

The REALITY is that by sliding your volt meter probe through the core, you now have ANOTHER SECONDARY WINDING, AND ANOTHER LOOP!

Like this:

(https://i.postimg.cc/LX6wZVFJ/20211121-235058.jpg)
https://i.postimg.cc/LX6wZVFJ/20211121-235058.jpg (https://i.postimg.cc/LX6wZVFJ/20211121-235058.jpg)

See? You have TWO secondary windings, with the SAME number of turns, and the SAME direction, and they are both having induced across them the SAME VOLTAGE and in the SAME POLARITY at the SAME TIME.

Of course the difference between them will be zero, because 1-1=0. Go figure.

And you don't just have one loop anymore, you have TWO loops. Or three, depending on how you want to model and measure.

If you MODEL REALITY, like I've been saying the whole time, KVL will hold with elements composed of the secondary windings on closed-magnetic-circuit-core transformers.

If you DON"T model reality, then all of physics will fail you, not just KVL with transformers, but KVL without transformers, MaxEQ, Ohms law, EVERYTHING.

I don't know what you mean by "lumper" but the EMF MODELS AND MEASURES as being across the transformer secondary at the very point it crosses through the core on a closed-magnetic-circuit-core transformer.

By the way, if you put two AA batteries minus to minus and measure the  voltage difference between the positive terminals, the difference will also be zero. You could use this same "trick" and add in an undocumented battery into your loop and make KVL fail with just batteries and resistors too, as I'm sure you saw hahahaha   :-DD https://youtu.be/V_Gs8tSqBRY  :-DD

The point is that by adding in additional voltage sources/drops and creating nested loops but pretending there is only one loop will get you in the same trouble with pure batteries and resistors as with a transformer.

So do we agree that if we run two identical parallel windings on a transformer that the voltage difference between them is zero?
Title: Re: #562 – Electroboom!
Post by: Jesse Gordon on November 22, 2021, 09:04:06 am
That's precisely your, Mehdi's, Dave's and everybody-else-that-insist-that-KVL-always-hold's mistake.

KVL says that the voltages measured along the path of a mesh add up to zero.

But Lewin showed at least one circuit where the voltages do not add up to zero.

Therefore KVL doesn't always hold. Period.

There's nothing you can do to remedy this. It is not about Lewin. It is the way nature works. Go to your lab, set up the same experiment, measure the voltages around the circuit and see for yourself. We are glad that those who repeated the experiment were precisely the guys who thought that Lewin had somehow cheated or blundered the experiment.

They're all dead inside now. All of them.

You cannot solve such circuit using KVL. It is impossible. You'll have to resort to the full monty and use Maxwell's equations at least to calculate the EMF produced by the varying magnetic field.

After that, you can of course devise an equivalent circuit where instead of a varying magnetic field producing the extra EMF upon the entire circuit, you have a battery, a generator, a transformer, or any other equivalent lumped (i.e. localized) component to produce the same EMF and get the exact same effect on the other components. In that case, you can solve the equivalent circuit using KVL because you theoretically removed the varying field from the circuit and stashed it away in the equivalent component.

But that is just a theoretical trick that has a lot of caveats.

One of them is that you will not find this extra EMF on the circuit. Not even if you fart your way through the Bohemian Rapsody.

So how's that possible? How can we have, so to speak, a spooky "component" that produces an EMF in a circuit, but is not present there? This seems to violate the principle of conservation of energy, doesn't it? Those are legit questions. But to answer them you need to abandon KVL, which these people are not prepared to accept. To reconcile their cognitive bias with the real phenomenon that contradicts it, they create all the irrational arguments I listed above and more.

I must say, I found the above post charming when I read it. I didn't get sucked into this thread  until late in the game when Thinkfat invited me.

When I read the part about all those sad people who went to their labs and experimented and were then dead inside, all of them, I was like huh? Then I realized that I must be one of them because I went to my lab to experiment too hahahaha!  :-DD :-DD

But my experments showed that KVL holds up just fine! Even at 40Khz in an air core transformer!

If this is being dead inside, I'm sure having a blast. Maybe being dead inside isn't so bad after all.

And when I see how the people who are alive inside behave, dodging questions, jumping all over the place, making the most absurd predictions -- I guess I don't feel too bad about being dead inside!  :-DD :-DD :-DD :-DD

Quote
KVL says that the voltages measured along the path of a mesh add up to zero.

But Lewin showed at least one circuit where the voltages do not add up to zero.

And Lewin did that by not using a MESH.  As you said, KVL requires a MESH - i.e., a loop with no sub loops inside it.

And Lewin used a loop that contained a subloop. KVL didn't fail Lewin, Lewin failed KVL. Lewin failed Physics if you ask me, but you didn't.  :-DD

If you want to use KVL on nested or multiple loops, you have to reduce each child loop to a lumped element, then reduce each next child loop to a lumped element, until ultimately you have only a single loop, or, as you said, a mesh.

Lewin's loop system had multiple active loops which all had voltage differences and he simply did not account for a bunch of them.

If he had considered each of his loops as individual nested meshes,  (i.e. an inner mesh and then his outer loop) then KVL would have held up just fine.

(HEHE and if he hadn't got one of his VOLT METERS ON BACKWARDS!)  :-DD :-DD

Quote
One of them is that you will not find this extra EMF on the circuit. Not even if you fart your way through the Bohemian Rapsody.

So how's that possible? How can we have, so to speak, a spooky "component" that produces an EMF in a circuit, but is not present there?

All you gotta do is measure the output voltage of the transformer, and there's your missing EMF.

If you're talking about trying to reach in through the core with your probe to measure the EMF, here's your answer: When you do that, you're creating ANOTHER SECONDARY with the same phase and voltage:
(https://i.postimg.cc/LX6wZVFJ/20211121-235058.jpg)

(https://i.postimg.cc/rs3cCg2F/20211119-122528.jpg)

Anyway, I guess I'm glad I found out I was one of the "Dead inside" so I know to have a somber moment once in a while in honor of, oh wait, if I'm not dead inside, then who the tarzan is?




Title: Re: #562 – Electroboom!
Post by: bsfeechannel on November 22, 2021, 12:07:52 pm
Too bad you didn't get an engineering degree, you might have been able to debunk KVLers if you had!   :-DD

The bogus claims of your nonsense “experiments” were debunked fair and square 9 days ago. Since then you have been trying to conduct “interviews” with forum members to see if you can somehow disqualify them. Let me tell you, it’s not working.

Quote
I know the right questions to ask, but you won't answer them.

Your questions be like “Wait a minute! Do you believe tomatoes are not animals?”

We can’t help you.
Title: Re: #562 – Electroboom!
Post by: jesuscf on November 22, 2021, 12:47:11 pm

\$
\begin{array}{l}
 V_2  = R_2  \cdot I = 900\Omega  \cdot 1mA = 0.9V \\
 V_1  =  - R_1  \cdot I =  - 100\Omega  \cdot 1mA =  - 0.1V \\
 \end{array}
\$

As for VAD: any two nodes that are half a circle apart have half of the total EMF.  In this case |VAD|=0.5V.

I see you used Ohm's law to compute the voltage drop along the resistors. So Ohm's law works. Wonderful.
Can you apply Ohm's law to compute the voltage drop across the wire that goes from R1 to point A?
Because we know that from A to D we have a voltage of half a volt, while across R1 we have what 0.1 V (fix the signs as you please while I put popcorns in the microwave). So, what gives?

(you can use the resistivity of copper, or a very low resistance for the wires, for example 1 milliohm, if zero resistance is a problem)

This is, by large, the most telling evidence of your ignorance of what is going on in the circuit (or any circuit by that matter!) in the whole thread.    How do you expect me to take you seriously when you don't know the circuital difference between a resistor and a voltage source?  That was explained before in this thread several times so I'll not do it again.
Title: Re: #562 – Electroboom!
Post by: jesuscf on November 22, 2021, 12:53:20 pm
Too bad you didn't get an engineering degree, you might have been able to debunk KVLers if you had!   :-DD

The bogus claims of your nonsense “experiments” were debunked fair and square 9 days ago. Since then you have been trying to conduct “interviews” with forum members to see if you can somehow disqualify them. Let me tell you, it’s not working.

Quote
I know the right questions to ask, but you won't answer them.

Your questions be like “Wait a minute! Do you believe tomatoes are not animals?”

We can’t help you.

Hey bsfeechannel when I was asking you if you have an electrical engineering degree of some sort, it was because I had (and I still have) the impression you didn't know what you were talking about.  Will I ask Jesse Gordon if he has an electrical engineering degree? No!  Jesse Gordon knows what he is talking about!  I don't care where he learned the stuff; he just knows.
Title: Re: #562 – Electroboom!
Post by: bsfeechannel on November 22, 2021, 01:37:48 pm

Jesse Gordon knows what he is talking about!

We also know what he’s talking about.
Title: Re: #562 – Electroboom!
Post by: bsfeechannel on November 22, 2021, 01:46:28 pm
How do you expect me to take you seriously when you don't know the circuital difference between a resistor and a voltage source?

The problem is that wires don’t make that distinction. So, you’re not gonna be fool enough to make it.
Title: Re: #562 – Electroboom!
Post by: jesuscf on November 22, 2021, 02:00:02 pm

Jesse Gordon knows what he is talking about!

We also know what he’s talking about.

No you don't.  You don't even know the basics of electric circuits.
Title: Re: #562 – Electroboom!
Post by: jesuscf on November 22, 2021, 02:02:52 pm
How do you expect me to take you seriously when you don't know the circuital difference between a resistor and a voltage source?

The problem is that wires don’t make that distinction. So, you’re not gonna be fool enough to make it.

Before this thread you knew nothing.  Sadly, there is no progress.  That is the problem of arguing with somebody whose fundamentals are not there.
Title: Re: #562 – Electroboom!
Post by: bsfeechannel on November 22, 2021, 02:14:06 pm
You don't even know the basics of electric circuits.

Says every single debunked KVLer.
Title: Re: #562 – Electroboom!
Post by: bsfeechannel on November 22, 2021, 02:15:42 pm
That is the problem of arguing with somebody whose fundamentals are not there.

I’m not versed in pseudo science. Sorry.
Title: Re: #562 – Electroboom!
Post by: Kalvin on November 22, 2021, 02:17:46 pm
I guess this problem could be modeled and solved using a 3D EM-simulator/solver. Then we could say: Case closed.

Any 3D EM-simulator gurus here?
Title: Re: #562 – Electroboom!
Post by: thinkfat on November 22, 2021, 03:33:09 pm
This has already been done. The video by "Silicon Soup" has been linked in this thread at least twice. It is a very good video as it also shows the electric field component, without which you can not understand the problem at all.
Title: Re: #562 – Electroboom!
Post by: Sredni on November 22, 2021, 05:06:46 pm
Again, you're quoting me out of context.

So many words that have nothing to do with the actual discussion. It's as if you were trying to create a smoke screen to hide the inconsistencies in your approach. The problem is that in almost every paragraph you write there is either a wrong concept, or a misconception, or a red flag betraying your limited knowledge. And the same is true for the two very long posts you wrote yesterday. I will try to collect all (well, most) the misconceptions and the false statement in separate posts by category, so that instead of an endless tit for tat, other users can benefit from a more organic discussion.

Let's start with a simple red flag.

Don't be afraid of the signs!

Regarding the following part of my picture, you wrote (emphasis mine):

(https://i.postimg.cc/Fs4SRYQL/screenshot-21.png)
https://i.postimg.cc/Fs4SRYQL/screenshot-21.png

 
Quote
I see you noticed that you made the same mistake Lewin did and you got one of the volt meters backwards, but at least you caught it in time to put a text note that says I can flip the volt sign.
So yes, I'd like to flip the sign on the left-hand meter which reads 1V. You have to keep the meters all pointing clockwise or counter clockwise -- but all the same way for a given test -- around the loop. That's just the way KVL is. If you mix up your volt meter signs even with pure batteries and resistors, KVL will appear to fail then too. 

Dude, seriously? You cannot handle the relativity of polarities? Apart from the fact that we are using voltmeters in an AC circuits, so the phase information is lost on us... what problem can you have in a circuit with batteries if one voltmeter is flipped? If it measure -0.5V with the probes in that position, it will measure +0.5V with the probes inverted.

You know I can choose whatever sign I want for the current in a circuit, then solve for its value and if it comes down negative it just means that it is flowing in the opposite direction to the one I supposed at first?
I hope you know at least some of the basic, such as how a current divider works, or what it means to load or shunt a generator, because otherwise this will be a very long discussion.

You dragged the discussion we were having here down about ten notches.
Title: Re: #562 – Electroboom!
Post by: Sredni on November 22, 2021, 05:13:31 pm
The ambiguously 'unambiguos' voltages in the two branches

Jesse, referencing this figure

(https://i.postimg.cc/VvFWycbH/Voltage-can-be-path-dependent.jpg)

You wrote (still showing you do not understand what the term 'lumped' really means, but I will get to that in a later, dedicated, post):

You clearly show different possible paths within the element, the 5v and 0v paths inside that element are unambiguous,  unvarying, and if that is the path in that element, then it can be considered a lumped element in a KVL loop and KVL will hold. 

It appears you are referencing the bottom figure, where both 5V and 0V paths are present. Good, Let's forget about the 'ambiguous' orange paths and let's focus on the unambiguous green (5V) and purple (0V) paths. Well, the path in the left branch of the circuit is a purple path that has the unambiguous voltage of 0V; the path in the right branch of the circuit is a green path that has the unambiguos voltage of 5V.

What exactly is the excuse you invoke to deny that two branches of the same circuit, with the same starting and ending points will have different values for the unambiguos voltage associated to the path that goes through them?
Title: Re: #562 – Electroboom!
Post by: Sredni on November 22, 2021, 05:20:10 pm

\$
\begin{array}{l}
 V_2  = R_2  \cdot I = 900\Omega  \cdot 1mA = 0.9V \\
 V_1  =  - R_1  \cdot I =  - 100\Omega  \cdot 1mA =  - 0.1V \\
 \end{array}
\$

As for VAD: any two nodes that are half a circle apart have half of the total EMF.  In this case |VAD|=0.5V.

I see you used Ohm's law to compute the voltage drop along the resistors. So Ohm's law works. Wonderful.
Can you apply Ohm's law to compute the voltage drop across the wire that goes from R1 to point A?
Because we know that from A to D we have a voltage of half a volt, while across R1 we have what 0.1 V (fix the signs as you please while I put popcorns in the microwave). So, what gives?

(you can use the resistivity of copper, or a very low resistance for the wires, for example 1 milliohm, if zero resistance is a problem)

This is, by large, the most telling evidence of your ignorance of what is going on in the circuit (or any circuit by that matter!) in the whole thread.    How do you expect me to take you seriously when you don't know the circuital difference between a resistor and a voltage source?  That was explained before in this thread several times so I'll not do it again.

Good: "the soldier that flees is good for another battle".

Can you shed some light on when exactly the wire becomes a 'voltage source'? Do you believe, like Jesse Gordon, that the wires acquire this magical voltage when they cross the magic portal of the core, or are they always voltage sources? For examples, is a quarter turn wire (open, just a quarter turn) a voltage source when it is inside the hole of the toroidal core? Does it ceases to be a voltage source when it is in front of the toroidal core but does not pass through the hole in the donut?

Teach me.
Title: Re: #562 – Electroboom!
Post by: jesuscf on November 22, 2021, 05:35:49 pm

\$
\begin{array}{l}
 V_2  = R_2  \cdot I = 900\Omega  \cdot 1mA = 0.9V \\
 V_1  =  - R_1  \cdot I =  - 100\Omega  \cdot 1mA =  - 0.1V \\
 \end{array}
\$

As for VAD: any two nodes that are half a circle apart have half of the total EMF.  In this case |VAD|=0.5V.

I see you used Ohm's law to compute the voltage drop along the resistors. So Ohm's law works. Wonderful.
Can you apply Ohm's law to compute the voltage drop across the wire that goes from R1 to point A?
Because we know that from A to D we have a voltage of half a volt, while across R1 we have what 0.1 V (fix the signs as you please while I put popcorns in the microwave). So, what gives?

(you can use the resistivity of copper, or a very low resistance for the wires, for example 1 milliohm, if zero resistance is a problem)

This is, by large, the most telling evidence of your ignorance of what is going on in the circuit (or any circuit by that matter!) in the whole thread.    How do you expect me to take you seriously when you don't know the circuital difference between a resistor and a voltage source?  That was explained before in this thread several times so I'll not do it again.

Good: "the soldier that flees is good for another battle".

Can you shed some light on when exactly the wire becomes a 'voltage source'? Do you believe, like Jesse Gordon, that the wires acquire this magical voltage when they cross the magic portal of the core, or are they always voltage sources? For examples, is a quarter turn wire (open, just a quarter turn) a voltage source when it is inside the hole of the toroidal core? Does it ceases to be a voltage source when it is in front of the toroidal core but does not pass through the hole in the donut?

Teach me.

At this point I am pretty sure you have no formal education in electrical engineering.  But here is the answer anyhow: every piece of wire and every resistor in the loop has an inductance.  That should be enough to understand what is going on.
Title: Re: #562 – Electroboom!
Post by: Kalvin on November 22, 2021, 05:39:17 pm
This has already been done. The video by "Silicon Soup" has been linked in this thread at least twice. It is a very good video as it also shows the electric field component, without which you can not understand the problem at all.

Thanks! Checked the video once, and looks really good. I need to dedicate more time watching it again doing the math.
Title: Re: #562 – Electroboom!
Post by: Jesse Gordon on November 22, 2021, 06:52:22 pm
How do you expect me to take you seriously when you don't know the circuital difference between a resistor and a voltage source?

The problem is that wires don’t make that distinction. So, you’re not gonna be fool enough to make it.

Before this thread you knew nothing.  Sadly, there is no progress.  That is the problem of arguing with somebody whose fundamentals are not there.

Look on the bright side! He did learn that his paradigm is so wrong that he dare not answer questions lest his lack of understanding be revealed!  :-DD
Title: Re: #562 – Electroboom!
Post by: Jesse Gordon on November 22, 2021, 07:02:08 pm
That is the problem of arguing with somebody whose fundamentals are not there.

I’m not versed in pseudo science. Sorry.

Like, for example, the pseudo science that which says KVL measures as holding with a loop formed by resistors and the output windings of closed-magnetic-circuit-core transformers, like this:?
(https://i.postimg.cc/15gbsCmz/20211119-232948.jpg)
https://i.postimg.cc/15gbsCmz/20211119-232948.jpg (https://i.postimg.cc/15gbsCmz/20211119-232948.jpg)

Do you really think it's pseudo science for me to say that if I measure the voltage difference across each element like KVL states and sum up the voltages that it will sum to zero?

Because I tried it. It did sum to zero.

You can't just call observable reality pseudo science because you don't understand it.

IN REALITY, as MEASURED with a volt meter, sum(V1, V2, V3)=0 which means KVL holds in this scenario.
Title: Re: #562 – Electroboom!
Post by: Jesse Gordon on November 22, 2021, 07:09:05 pm
You don't even know the basics of electric circuits.

Says every single debunked KVLer.

"Says every single lewinite."

There. Do you see how incredibly feeble we both sound saying that? I'd have to be dead inside to say that.  :-DD

Instead, I'm going to say this:

I personally used a transformer and a volt meter and I measured around a loop of elements like this:
(https://i.postimg.cc/15gbsCmz/20211119-232948.jpg)

I found, by observation, that KVL HOLDS in this scenario.

Tell me, can you at least agree that KVL at least APPEARS to hold in the case of closed-magnetic-circuit-core transformer secondary windings as elements in a loop, as shown above?

Dude, all the voltage differences across each element sum to zero! That's literally the definition of KVL!

How can you not admit that for closed-magnetic-circuit-core transformer secondary windings, KVL holds with that winding as an element in a loop with other elements?
Title: Re: #562 – Electroboom!
Post by: Jesse Gordon on November 22, 2021, 08:32:37 pm
I guess this problem could be modeled and solved using a 3D EM-simulator/solver. Then we could say: Case closed.

Any 3D EM-simulator gurus here?

Simulators are a wonderful thing, but we should not neglect real-world observed data either.

Here I test with an open air core transformer: https://youtu.be/nAsZFP8Cfxk

Here I test with a closed-magnetic-circuit-core transformer (EI Core) https://youtu.be/iDWv8QJrzUo

And this one is more humor than anything, but it shows what Lewin's mistake would have been like had he been using batteries instead of transformer secondary windings: https://youtu.be/V_Gs8tSqBRY

And here's a longer demonstration showing how you can run your probe leads if you do want to measure voltage differences in a changing magnetic field without having your reading influenced by said changing magnetic field: https://youtu.be/_mzxE_p5dN8


Of course all the accurate simulations give the same results as the real world observations.

I'm really not sure what the argument is even about.

The biggest point seems to be to defend Lewin's honor, and has nothing to do with the technical facts.

Those defending Lewin seem to have little understanding of actual electromagnetics, and generally refuse to even accept that KVL does hold in a loop of elements one of which is a closed-magnetic-circuit-core (toroidal) transformer secondary winding, as shown here:
(https://i.postimg.cc/15gbsCmz/20211119-232948.jpg)

I built a working model and measured around the loop, getting the voltage differences across each element per KVL and they summed to zero.

But the people defending Lewin won't even admit that "Yes, in that case, KVL appears to hold" because they feel they are somehow betraying their master.

What's really odd, is that Lewin wasn't even using a closed-magnetic-circuit transformer, he was using an open air core transformer which complicates things for the uninformed.

So knowing that, I really have no idea why they won't accept that KVL can at the very least appear to hold with an element which is the output of such a transformer as I show in my diagram above.
Title: Re: #562 – Electroboom!
Post by: Jesse Gordon on November 22, 2021, 11:04:31 pm
Again, you're quoting me out of context.

So many words that have nothing to do with the actual discussion. It's as if you were trying to create a smoke screen to hide the inconsistencies in your approach. The problem is that in almost every paragraph you write there is either a wrong concept, or a misconception, or a red flag betraying your limited knowledge. And the same is true for the two very long posts you wrote yesterday. I will try to collect all (well, most) the misconceptions and the false statement in separate posts by category, so that instead of an endless tit for tat, other users can benefit from a more organic discussion.

Let's start with a simple red flag.

Don't be afraid of the signs!

Regarding the following part of my picture, you wrote (emphasis mine):

(https://i.postimg.cc/B6WVgpBm/screenshot-21.png)
https://i.postimg.cc/B6WVgpBm/screenshot-21.png

 
Quote
I see you noticed that you made the same mistake Lewin did and you got one of the volt meters backwards, but at least you caught it in time to put a text note that says I can flip the volt sign.
So yes, I'd like to flip the sign on the left-hand meter which reads 1V. You have to keep the meters all pointing clockwise or counter clockwise -- but all the same way for a given test -- around the loop. That's just the way KVL is. If you mix up your volt meter signs even with pure batteries and resistors, KVL will appear to fail then too. 

Dude, seriously? You cannot handle the relativity of polarities? Apart from the fact that we are using voltmeters in an AC circuits, so the phase information is lost on us... what problem can you have in a circuit with batteries if one voltmeter is flipped? If it measure -0.5V with the probes in that position, it will measure +0.5V with the probes inverted.

Are you dyslexic? or using a screen reader?

I complained about you putting the sign wrong on the "LEFT-HAND meter which reads 1V" and you zoom in on the RIGHT HAND meter which reads 1V?

Dude, SERIOUSLY? Look at your original picture, to which I was replying -- except this is the whole thing:

(https://i.postimg.cc/R04QGyHs/KVL-works-if-I-leave-out-the-magnetic-region.jpg)
https://i.postimg.cc/R04QGyHs/KVL-works-if-I-leave-out-the-magnetic-region.jpg (https://i.postimg.cc/R04QGyHs/KVL-works-if-I-leave-out-the-magnetic-region.jpg)

Take note of this:

<-----LEFT   :scared:   RIGHT------>

I asked about the LEFT-Hand 1V meter, and you cropped it off and showed the RIGHT 1V meter. You're quoting me out of context even if you have to cut away the part of the diagram to which I'm replying!

Quote
You know I can choose whatever sign I want for the current in a circuit, then solve for its value and if it comes down negative it just means that it is flowing in the opposite direction to the one I supposed at first?
I hope you know at least some of the basic, such as how a current divider works, or what it means to load or shunt a generator, because otherwise this will be a very long discussion.

You dragged the discussion we were having here down about ten notches.

Of course I understand signs.

But it just so happens that when applying KVL, you have to have all the volt meters pointing the same clock direction around the loop, and you have the LEFT-HAND 1V (<---) meter pointing the wrong way, which would cause KVL to appear to fail even if it was strictly resistors and batteries.

Lewin ALSO made this same error, having one of HIS volt meters backwards when he purported to measure test KVL. That's why one of his scopes read a positive spike and the other read a negative spike. If he'd had them both clocked the same direction, they'd have both read positive (or both negative), and the voltage across the resistors would have summed to the negative of his induced EMF, and KVL would have held.

So of course I know how signs work. And of course I know that if you hook a volt meter up backwards you can just negate whatever it reads. But when we're modeling, if you hook up the volt meter backwards but then don't tell the model that it's backwards, then you failed KVL, KVL didn't fail you.


You drew 6 volt meters in your loop: One had no polarity indicated, two were positive-leading-clockwise and three of them were negative-leading-clockwise except two.

Remember how they hammer into students to pay attention to the sign? It's got to be second nature by the time you graduate. But you draw a KVL loop and you're over the place.

So sure, you can put your volt meters whatever way you want then flip the signs as needed - but no engineer would draw what you drew.

There's just no way you're an engineer of any sort of the meaning. Unless.... Unless.. Do you drive a locomotive? That might suite you, since you don't have to think about whether you drive on the right or the left.  :-DD

You complain about "So many words that have nothing to do with the actual discussion."

Fine, let's talk about the actual discussion, if you will:

In the diagram below, assuming that all of the volt meters are polarity-clocked the same way, and assuming you ignore the 0.5v meters, will KVL hold?
Will the algebraic sum of all the voltage differences of each element around the loop be zero?

(https://i.postimg.cc/R04QGyHs/KVL-works-if-I-leave-out-the-magnetic-region.jpg)
https://i.postimg.cc/R04QGyHs/KVL-works-if-I-leave-out-the-magnetic-region.jpg (https://i.postimg.cc/R04QGyHs/KVL-works-if-I-leave-out-the-magnetic-region.jpg)
Title: Re: #562 – Electroboom!
Post by: Jesse Gordon on November 22, 2021, 11:13:58 pm
The ambiguously 'unambiguos' voltages in the two branches

Jesse, referencing this figure

(https://i.postimg.cc/VvFWycbH/Voltage-can-be-path-dependent.jpg)

You wrote (still showing you do not understand what the term 'lumped' really means, but I will get to that in a later, dedicated, post):

You clearly show different possible paths within the element, the 5v and 0v paths inside that element are unambiguous,  unvarying, and if that is the path in that element, then it can be considered a lumped element in a KVL loop and KVL will hold. 

It appears you are referencing the bottom figure, where both 5V and 0V paths are present. Good, Let's forget about the 'ambiguous' orange paths and let's focus on the unambiguous green (5V) and purple (0V) paths. Well, the path in the left branch of the circuit is a purple path that has the unambiguous voltage of 0V; the path in the right branch of the circuit is a green path that has the unambiguos voltage of 5V.

What exactly is the excuse you invoke to deny that two branches of the same circuit, with the same starting and ending points will have different values for the unambiguos voltage associated to the path that goes through them?

What do you mean "two branches of the same circuit?" One's the secondary winding on a transformer and the other's not. You no longer have a single loop -- you have multiple loops each with their own voltage sources. Like this:

(https://i.postimg.cc/LX6wZVFJ/20211121-235058.jpg)
https://i.postimg.cc/LX6wZVFJ/20211121-235058.jpg (https://i.postimg.cc/LX6wZVFJ/20211121-235058.jpg)

Like I keep saying, MODEL REALITY. Don't slip in extra elements that you're not modeling for. Don't call it a MESH when you have multiple nested or networked loops.

Title: Re: #562 – Electroboom!
Post by: Sredni on November 23, 2021, 12:56:55 am
This has already been done. The video by "Silicon Soup" has been linked in this thread at least twice. It is a very good video as it also shows the electric field component, without which you can not understand the problem at all.

Thanks! Checked the video once, and looks really good. I need to dedicate more time watching it again doing the math.

Here's a map to navigate these troubled waters:

(https://i.postimg.cc/QdRGMwMM/Map-to-the-Ring-Quest.png)
https://i.postimg.cc/QdRGMwMM/Map-to-the-Ring-Quest.png

It shows the fields inside the ring, alone.
Title: Re: #562 – Electroboom!
Post by: Sredni on November 23, 2021, 01:20:11 am
Don't be afraid of the signs!
You have to keep the meters all pointing clockwise or counter clockwise -- but all the same way for a given test -- around the loop. That's just the way KVL is. If you mix up your volt meter signs even with pure batteries and resistors, KVL will appear to fail then too.
...
I complained about you putting the sign wrong on the "LEFT-HAND meter which reads 1V" and you zoom in on the RIGHT HAND meter which reads 1V?

Well, on the right there is a loop made of voltmeters to which you can apply KVL and one of the voltmeter has a flipped polarity (according to you), I though you referred to that one.
But thank you for confirming that the left voltmeter would read 1V and therefore there is no voltage in the wires.

Quote
But it just so happens that when applying KVL, you have to have all the volt meters pointing the same clock direction around the loop,


This is probably true in schools for children with special needs. I can assure you that among adults, we take whatever polarity comes with the circuit and adapt to that without batting an eye.
You are indeed afraid of the signs.

Quote
Lewin ALSO made this same error, having one of HIS volt meters backwards when he purported to measure test KVL. That's why one of his scopes read a positive spike and the other read a negative spike.
That's not an error. It's a no brainer to automatically flip the signs in your head when the loops are these simple.
I told ya, this was a red flag.

Quote
Fine, let's talk about the actual discussion, if you will:
In the diagram below, assuming that all of the volt meters are polarity-clocked the same way, and assuming you ignore the 0.5v meters, will KVL hold?

Do you really need to ask? Did you not read what I wrote in the top right rectangle?
"KVL works here. Works for every path in the shaded region". Here path means 'closed path' or 'loop' to which apply KVL.
Why do you need to ask, I wonder...

Have you seen the rectangle in the other picture, though?  >:D
Never mind, I will repost it with due highlighting.
Title: Re: #562 – Electroboom!
Post by: Sredni on November 23, 2021, 02:25:10 am
I guess this problem could be modeled and solved using a 3D EM-simulator/solver. Then we could say: Case closed.
Any 3D EM-simulator gurus here?
Simulators are a wonderful thing, but we should not neglect real-world observed data either.
Here I test ---

I am sorry to interrupt your victory lap, but... the same circuit you says it shows that KVL 'appears' to hold, is actually breaking it.

https://i.postimg.cc/9MW4fCCd/screenshot-23.png
(https://i.postimg.cc/9MW4fCCd/screenshot-23.png)

Here is the detail.

(https://i.postimg.cc/QCQnJQRG/screenshot-22.png)
https://i.postimg.cc/QCQnJQRG/screenshot-22.png

And in case you are trobled with the magical entrance of the voltage stargate, here is Lewin's ring itself, with voltmeters all around it, showing that KVL 'appears' to die. Horrible death, poor creature.

(https://i.postimg.cc/9MRf0Msq/screenshot-24.png)
https://i.postimg.cc/9MRf0Msq/screenshot-24.png

(https://i.postimg.cc/bw7CWFpc/screenshot-25.png)
https://i.postimg.cc/bw7CWFpc/screenshot-25.png


Title: Re: #562 – Electroboom!
Post by: jesuscf on November 23, 2021, 03:31:02 am
Can you shed some light on when exactly the wire becomes a 'voltage source'? Do you believe, like Jesse Gordon, that the wires acquire this magical voltage when they cross the magic portal of the core, or are they always voltage sources? For examples, is a quarter turn wire (open, just a quarter turn) a voltage source when it is inside the hole of the toroidal core? Does it ceases to be a voltage source when it is in front of the toroidal core but does not pass through the hole in the donut?

Teach me.

I see you are still spreading BS.  Let me try to explain as simply as a I can what is going on with the ring.

In the problem I posted before replace the wires with the series connection of 178,  2 ohms resistors, half of them in the top, half of them in the bottom.  Also keep the two original resistors of 100 ohms and 900 ohms as before.  That is a total of 180 resistors in series.  The reason for 180 resistors is that the radius of this resistor ring matches very closely the radius of the original ring of 0.2m, if we use 1/4W resistors with a length of approximately 7mm.  The new ring should look somehow like the one in the first attached diagram.  The number of resistors shown in the drawing is smaller because it is very hard to draw a ring with 180 resistors; just remember that the top and bottom red/blue set of series resistors has 89 individual pieces each.  Keep the magnetic flux density the same as before:

\$B(t) = 0.7958 \cdot e^{\frac{{ - t}}{{0.1s}}} _{} T\$

Now find the voltages V1 and V2 at t=0.

Using the exact same calculation as shown before we find that the generated EMF is 1V.  Therefore, each resistor is generating an EMF due to the varying magnetic field of:

\$
EMF_{RESISTOR}  = \frac{{1V}}{{180}} = 5.55555...mV
\$

With that EMF per resistor and its resistance value, we can develop a Thevenin equivalent circuit for each resistor as shown in the second attached figure.  Since we are interested in the voltage across the 100 ohms  and 900 ohms resistors only, we can combine all the Thevenin equivalent circuits for the 2 ohms  resistors into a lumped Thevenin equivalent circuit for which VTH=0.9888V and RTH=356 ohms.  The Thevenin circuits are shown in the second attached figure.

The circuit we have to solve now is shown in the third figure.  As before use KVL to find the current:

\$
I = \frac{{EMF}}{{R_1  + R_2  + R_{LOOP} }} = \frac{{1V}}{{100\Omega  + 900\Omega  + 178*2\Omega }} = 0.73746mA
\$

With the current we find the voltages V1 and V2:

\$
V_1  =  - 0.73746mA \cdot 100\Omega  + 5.555mV =  - 0.06819V
\$
\$
V_2  = 0.73746mA \cdot 900\Omega  - 5.555mV = 0.6582V
\$

Can you see that the voltage across any of the smaller resistance resistors is the induced EMF minus the voltage drop in that resistor due to the actual current?  In this case is 5.555mV-(2 ohms x 0.73746mA)=4.081mV.

Now we can decrease the 2 ohms resistors to any smaller resistance we want and/or add as many resistors as we want to make the loop, and you'll see why we get a voltage across any two points in ring's wire.  Any piece of wire in the original circuit, under the varying magnetic field behaves as voltage source in series with a resistor. 







Title: Re: #562 – Electroboom!
Post by: Jesse Gordon on November 23, 2021, 06:32:10 am
I guess this problem could be modeled and solved using a 3D EM-simulator/solver. Then we could say: Case closed.
Any 3D EM-simulator gurus here?
Simulators are a wonderful thing, but we should not neglect real-world observed data either.
Here I test ---

I am sorry to interrupt your victory lap, but... the same circuit you says it shows that KVL 'appears' to hold, is actually breaking it.

https://i.postimg.cc/9MW4fCCd/screenshot-23.png
(https://i.postimg.cc/9MW4fCCd/screenshot-23.png)

Dude, you full well know that KVL requires A MESH. But here you have let's see, what, two or three secondary windings forming 3 different intertwined loops each with their own voltage source?

You've got FOUR wires coming out of that element! KVL requires exactly two terminals per element.

KVL is not failing you, you are failing KVL.

As I said before, when you run your volt meter leads through the transformer core, you're adding more active loops, more undocumented voltage sources.
And you're trying to measure the transformer's OUTPUT from INSIDE, which doesn't work because you're basically creating another secondary, and trying to measure the voltage across the one secondary minus the voltage across another identical secondary. Doh!

If you were to treat the transformer as a 2 terminal element as KVL requires, then you would measure it's voltage across it's terminals and all the sudden KVL would hold.

See here: OF COURSE if you measure the difference between two identical voltages the difference will be zero:
(https://i.postimg.cc/LX6wZVFJ/20211121-235058.jpg)
https://i.postimg.cc/LX6wZVFJ/20211121-235058.jpg (https://i.postimg.cc/LX6wZVFJ/20211121-235058.jpg)

Doesn't mean that the induced EMF is zero though.

Quote
Here is the detail.

(https://i.postimg.cc/QCQnJQRG/screenshot-22.png)
https://i.postimg.cc/QCQnJQRG/screenshot-22.png
Again, you are trying to measure the transformers OUTPUT from WITHIN the transformer.

You're still comparing the voltage difference between one winding to another identical winding.

To measure the induced EMF, you need to measure around dB/dt, not outside of it.

You're all confused by the fact that some configurations make it difficult to measure induced EMF and so you think that it does not exist just because YOU don't know how to measure it.

But this is easy, all you gotta do is pick two points to be the transformer's terminals and measure across those FROM OUTSIDE the transformer, lest you create more undocumented turns.

Quote
And in case you are trobled with the magical entrance of the voltage stargate, here is Lewin's ring itself, with voltmeters all around it, showing that KVL 'appears' to die. Horrible death, poor creature.

(https://i.postimg.cc/9MRf0Msq/screenshot-24.png)
https://i.postimg.cc/9MRf0Msq/screenshot-24.png

And here again, you're adding a bunch more secondary turns and additional loops, which again, which is not what KVL even claims to cover.
You draw volt meters, but in REALITY, their leads are actually secondary turns on the transformer, and they form their own loops with their own induced voltages which (in the case of an infinitely long solenoid) would be equal in voltage to the section of loop you purport to be measuring, so again, you're trying to measure for a difference between parallel winding with the same number of turns.

If you probe correctly, you will find that whatever point you probe to becomes a transformer terminal or tap terminal, and that whatever subsection you've put your probes on still works just fine as 2 terminal element for KVL, as seen in my Lewin Clock video.

And, I'll give you the benefit of the doubt and assume that you meant to describe an infinitely long toroid, because if it's not infinitely long, none of your volt meters will read zero, as seen in my video on that topic.


Quote
(https://i.postimg.cc/bw7CWFpc/screenshot-25.png)
https://i.postimg.cc/bw7CWFpc/screenshot-25.png


And Dr. McDonald was utterly clear that Lewin's loop was within the scope of applicability of Kirchhoff's Voltage Equations.
Dr. McDonald also said that Lewin used DIVERSION and an outdated "law" to create an apparent paradox.



Dr. Belcher started out with a heartfelt expression of gratitude to Mehdi, and not ONE WORD about Lewin.
Dr. Belcher also quotes Feynman and says that because the ∮E.dl of the windings will have a voltage differential which, when algebraically summed with the voltages across the resistors, will equal zero, and that KVL thus holds as Mehdi argued.

I know you ignore the fact that Dr. McDonald out right stated that Lewin's loop was within the range of applicability of KVL.

I also know that you argue that Dr. Belcher was only agreeing with one small undisputed part of Mehdi's claim, but that does not seem to be the case:
It might help you if Belcher also stated that he agreed with Lewin on some point. But he didn't even mention him.
It might help you if Belcher also stated that he disagreed with Mehdi on some point. But I sure didn't notice that either.

And the very reason that Dr. Belcher gave for agreeing with Mehdi was that Dr. Feynman defines the voltage across an inductor as ∮E.dl, and since all of Mehdi's arguments were based on the induced voltage due to Faraday's law, it looks to me like Belcher was entirely agreeing with Mehdi.

Belcher doesn't get behind Lewin's controversial claim at all, and quite clearly wraps up with what can easily be taken as a very broad stamp of agreement with Mehdi.

And lacking mention of Lewin, that can easily be taken as a broad lack of agreement with Lewin.


But why do you refuse to answer my simple question?

Look: At this point, I'm not asking you to agree to anything that goes on INSIDE a transformer. I'm not asking you to agree to anything relating to how it may be difficult to measure the induced EMF across a partial turn for some transformer core types.

I'm simply asking you to agree that KVL holds as measured by a volt meter if the transformer secondary is simply considered  a two terminal element, with the two terminals being the two ends of the secondary winding where they protrude from the core, as depicted in the below diagram.

Does KVL hold in the following test,where we consider the transformer secondary to be a two-terminal element, with the two terminals being the TWO ends of the secondary winding which protrude from the core?

(https://i.postimg.cc/1tNL2Mmn/20211122-214739.jpg)
https://i.postimg.cc/1tNL2Mmn/20211122-214739.jpg (https://i.postimg.cc/1tNL2Mmn/20211122-214739.jpg)

Once we agree that KVL holds in the above diagram as lumped, then we can get onto your actual argument whatever the tarzan that is.

You do seem quite interested in measuring transformers from the inside, although that may just be a diversion.

Frankly, I expected you to say months ago "Dude it's not about what you do with the wires OUTSIDE the transformer, it's what goes on inside."

But you've been so scared to even agree to the diagram directly above, so I don't know what your issue is.

So how about it? Do you agree that KVL holds in the scenario depicted in the diagram directly above? Or at least holds as measured by a volt meter?

Title: Re: #562 – Electroboom!
Post by: Jesse Gordon on November 23, 2021, 07:13:17 am
Does anybody really know what we're actually arguing about here?

I Think we all agree that a transformer's output winding works fine (or at least measures fine) as an element in a mesh which works with KVL.

That is to say, I don't think anybody is saying that a transformer output cannot be an element in a KVL mesh.

And yet, getting clarification on that is nearly impossible. Nobody wants to exactly confirm nor deny that. And why no? I don't know.

If it's not about a transformer secondary winding's acceptability as an element in a KVL mesh, what then?

Many of the examples given for "KVL failing" are not meshes, but rather multiple nested loops with multiple voltage sources and sinks, and various parts are being cherry picked to give the appearance of KVL failing.  Of course KVL doesn't appear to work with any circuit if you are modeling one thing and measuring a functionally different topology.

Then there is talk about there being no electric field along a wire. But we're not talking about an electric field near the surface of the wire, we're talking about the voltage difference between winding ends (or in some cases center taps or other part-way-taps) of a transformer winding -- actual electrical connections to the winding, either at the ends, or at some points between the ends.

The fact is, when we measure the voltage across the ends of a secondary winding, we measure a voltage. If we have a center tap, we can measure voltage there too.
And if we place our super sharp probe at any point along the winding, it forms an instant electrically connected tap that works just like a manufacturer produced tap, and again, we measure the voltage that we would expect.

Or are we arguing about whether certain transformer types can be probed accurately? Does a difficulty in probing mean there is no voltage there to be measured?

What's odd is that this is not the first conversation I've had about this. Since uploading my Lewin Clock video, I have had some exceedingly long discussions about it, quite probably with people who were more educated than any of us here, but the story was the same - a strong hesitance to answer actual real life questions. I also asked them like "What is it exactly, are we just using different definitions for something and we don't know it?" But they could not explain what we were arguing about either.

There were many many things they could not answer. That's why I ended up making my Lewin Clock and Iron Core videos - because people told me things would be a certain way and I didn't believe them, so I tested, and I was right and they were wrong, so I made a video to show them.

It seems as soon as I try to zero in on one issue, the other guy switches to another. As soon as I try to zero in on the new topic, again, he switches to yet another. It's like a 3 ring circuits.

But there is ONE thing that these people do know, without a shadow of a doubt:

In the words of Silicon Soup, "OF COURSE LEWIN IS RIGHT!"

Is that what this is all about?

What is it we're ACTUALLY arguing about? Why are the Lewinites so unwilling to admit to the observable facts so that we can then concentrate on the more difficult parts of the issue?

EDIT/PS: And I often wonder if the issue comes down to the difference in meaning of -∫E.dl and ∮E.dl.   Are we all just falling for the "Common misconception" that Dr. Belcher talked about in assuming that ∮E.dl=0, or that the -L(dI/dt) voltage read by the voltmeter represents a -∫E.dl through the inductor?  But alas, this has been brought up, and that seems not to help either.
Title: Re: #562 – Electroboom!
Post by: jesuscf on November 23, 2021, 03:19:32 pm
Does anybody really know what we're actually arguing about here?

I Think we all agree that a transformer's output winding works fine (or at least measures fine) as an element in a mesh which works with KVL.

That is to say, I don't think anybody is saying that a transformer output cannot be an element in a KVL mesh.

And yet, getting clarification on that is nearly impossible. Nobody wants to exactly confirm nor deny that. And why no? I don't know.

If it's not about a transformer secondary winding's acceptability as an element in a KVL mesh, what then?

Many of the examples given for "KVL failing" are not meshes, but rather multiple nested loops with multiple voltage sources and sinks, and various parts are being cherry picked to give the appearance of KVL failing.  Of course KVL doesn't appear to work with any circuit if you are modeling one thing and measuring a functionally different topology.

Then there is talk about there being no electric field along a wire. But we're not talking about an electric field near the surface of the wire, we're talking about the voltage difference between winding ends (or in some cases center taps or other part-way-taps) of a transformer winding -- actual electrical connections to the winding, either at the ends, or at some points between the ends.

The fact is, when we measure the voltage across the ends of a secondary winding, we measure a voltage. If we have a center tap, we can measure voltage there too.
And if we place our super sharp probe at any point along the winding, it forms an instant electrically connected tap that works just like a manufacturer produced tap, and again, we measure the voltage that we would expect.

Or are we arguing about whether certain transformer types can be probed accurately? Does a difficulty in probing mean there is no voltage there to be measured?

What's odd is that this is not the first conversation I've had about this. Since uploading my Lewin Clock video, I have had some exceedingly long discussions about it, quite probably with people who were more educated than any of us here, but the story was the same - a strong hesitance to answer actual real life questions. I also asked them like "What is it exactly, are we just using different definitions for something and we don't know it?" But they could not explain what we were arguing about either.

There were many many things they could not answer. That's why I ended up making my Lewin Clock and Iron Core videos - because people told me things would be a certain way and I didn't believe them, so I tested, and I was right and they were wrong, so I made a video to show them.

It seems as soon as I try to zero in on one issue, the other guy switches to another. As soon as I try to zero in on the new topic, again, he switches to yet another. It's like a 3 ring circuits.

But there is ONE thing that these people do know, without a shadow of a doubt:

In the words of Silicon Soup, "OF COURSE LEWIN IS RIGHT!"

Is that what this is all about?

What is it we're ACTUALLY arguing about? Why are the Lewinites so unwilling to admit to the observable facts so that we can then concentrate on the more difficult parts of the issue?

EDIT/PS: And I often wonder if the issue comes down to the difference in meaning of -∫E.dl and ∮E.dl.   Are we all just falling for the "Common misconception" that Dr. Belcher talked about in assuming that ∮E.dl=0, or that the -L(dI/dt) voltage read by the voltmeter represents a -∫E.dl through the inductor?  But alas, this has been brought up, and that seems not to help either.

I think it boils down to the assertion: "KVL doesn't hold in non-conservative fields".  Maybe that was the case in the early to middle 1800s when people had no idea of what was going on and KVL was applied to what people thought were resistive circuits only.  After the contributions of Faraday, Maxwell, and many others they found out that yes, KVL holds in non-conservative fields, but now you have to add to your circuits inductors, capacitors, transformers, transmission lines, non-linear semiconductors models, etc. etc. not just resistors.  And pay attention on how the fields of one element in the circuit affects other elements!  KVL got a lot more complicated, with differential equations everywhere!  It was so bad at some point, that in the early 1900 Charles Proteus Steinmetz (an absolute legend in Electrical Engineering), promoted the use of phasor analysis to avoid the differential equations at least in AC circuits.  Many other techniques were developed ever since to solve these very complicated circuits, which are commonly taught in the electrical/electronics higher education programs.
Title: Re: #562 – Electroboom!
Post by: Sredni on November 23, 2021, 05:39:25 pm
I think it boils down to the assertion: "KVL doesn't hold in non-conservative fields".  Maybe that was the case in the early to middle 1800s

No, it all boils down to certain people being unable to understand what they read, and being incapable of assimilating new concepts that go beyond their limited knowledge. Like trying to explain the orbit of planets with epicycloids: it works numerically, but it betrays a fundamental ignorance of the actual physical phenomena behind the motion.

Now, I have to check with the kitchen, but I am pretty sure I have deja-vus:

(https://i.postimg.cc/0jhNjDdL/screenshot.png)

We already had this discussion on how those pages from Hayt are about the "amended" or "extended" or "new" KVL. Haven't we? Three years have passed, and you still have to understand that it's a couple of centuries we know we can apply a 'modified' version of KVL to circuits with inductors and other magnetic components. The trick is to hide the magnetic flux inside the component SO THAT THE CIRCUIT PATH CAN  SKIP ITS INNARDS AND WE AVOID TO HAVE VARIABLE FLUX LINKED BY OUR CIRCUIT PATH.

You really, really, really CAN NOT UNDERSTAND this simple concept.
Yoo probably lack basic vector algebra skill, maybe you were ill when they explained why you can define a potential function when a vector field is conservative, or like Stoke's theorem it is just beyond you, I can't tell. But you ignore basic math and basic physics - and now you are desperately twisting your epicycloids to explain the retrograde motion of Mars. If you only could understand Newton's theory of gravitation...

You call "BS" the standard theory of classical electrodynamics because you did not study it well enough to understand that your 'tiny batteries in series' model is not a model of the complete system, but just of the mathematical portion that is conservative. Look at the "Map of the Ring Quest" picture I've posted yesterday: do you recognize your model in the rhs?
After I have scanned a few images, I will make a dedicated post  to explain what you have done, how you can see it by manipulating the integral vector equations, and why that model is not a model of your system but only of a part of it.

Here's what my crystal ball forecasts:
You will cursorily read it. You will not understand it. And three years from now, you will still be unable to tell the difference between lumped circuits (to which you can apply 'extended KVL' like Hayt and Ulaby show) and unlumpable circuits (for which not even the 'extended' version of KVL works BECAUSE THE ********** CHANGING FLUX IS LINKED BY YOUR ******* CIRCUIT PATH)
Title: Re: #562 – Electroboom!
Post by: jesuscf on November 23, 2021, 07:18:07 pm
I think it boils down to the assertion: "KVL doesn't hold in non-conservative fields".  Maybe that was the case in the early to middle 1800s

No, it all boils down to certain people being unable to understand what they read, and being incapable of assimilating new concepts that go beyond their limited knowledge. Like trying to explain the orbit of planets with epicycloids: it works numerically, but it betrays a fundamental ignorance of the actual physical phenomena behind the motion.

Now, I have to check with the kitchen, but I am pretty sure I have deja-vus:

(https://i.postimg.cc/0jhNjDdL/screenshot.png)

We already had this discussion on how those pages from Hayt are about the "amended" or "extended" or "new" KVL. Haven't we? Three years have passed, and you still have to understand that it's a couple of centuries we know we can apply a 'modified' version of KVL to circuits with inductors and other magnetic components. The trick is to hide the magnetic flux inside the component SO THAT THE CIRCUIT PATH CAN  SKIP ITS INNARDS AND WE AVOID TO HAVE VARIABLE FLUX LINKED BY OUR CIRCUIT PATH.

You really, really, really CAN NOT UNDERSTAND this simple concept.
Yoo probably lack basic vector algebra skill, maybe you were ill when they explained why you can define a potential function when a vector field is conservative, or like Stoke's theorem it is just beyond you, I can't tell. But you ignore basic math and basic physics - and now you are desperately twisting your epicycloids to explain the retrograde motion of Mars. If you only could understand Newton's theory of gravitation...

You call "BS" the standard theory of classical electrodynamics because you did not study it well enough to understand that your 'tiny batteries in series' model is not a model of the complete system, but just of the mathematical portion that is conservative. Look at the "Map of the Ring Quest" picture I've posted yesterday: do you recognize your model in the rhs?
After I have scanned a few images, I will make a dedicated post  to explain what you have done, how you can see it by manipulating the integral vector equations, and why that model is not a model of your system but only of a part of it.

Here's what my crystal ball forecasts:
You will cursorily read it. You will not understand it. And three years from now, you will still be unable to tell the difference between lumped circuits (to which you can apply 'extended KVL' like Hayt and Ulaby show) and unlumpable circuits (for which not even the 'extended' version of KVL works BECAUSE THE ********** CHANGING FLUX IS LINKED BY YOUR ******* CIRCUIT PATH)

Sure why not.  Now hurry up, You better go tell the whole electric power industry that they have been analyzing circuits under the effects of magnetic fields incorrectly for the last 160 years!!!
Title: Re: #562 – Electroboom!
Post by: Jesse Gordon on November 23, 2021, 07:42:18 pm
I think it boils down to the assertion: "KVL doesn't hold in non-conservative fields".  Maybe that was the case in the early to middle 1800s

No, it all boils down to certain people being unable to understand what they read, and being incapable of assimilating new concepts that go beyond their limited knowledge. Like trying to explain the orbit of planets with epicycloids: it works numerically, but it betrays a fundamental ignorance of the actual physical phenomena behind the motion.

Now, I have to check with the kitchen, but I am pretty sure I have deja-vus:

(https://i.postimg.cc/0jhNjDdL/screenshot.png)

We already had this discussion on how those pages from Hayt are about the "amended" or "extended" or "new" KVL. Haven't we? Three years have passed, and you still have to understand that it's a couple of centuries we know we can apply a 'modified' version of KVL to circuits with inductors and other magnetic components. The trick is to hide the magnetic flux inside the component SO THAT THE CIRCUIT PATH CAN  SKIP ITS INNARDS AND WE AVOID TO HAVE VARIABLE FLUX LINKED BY OUR CIRCUIT PATH.

You really, really, really CAN NOT UNDERSTAND this simple concept.
Yoo probably lack basic vector algebra skill, maybe you were ill when they explained why you can define a potential function when a vector field is conservative, or like Stoke's theorem it is just beyond you, I can't tell. But you ignore basic math and basic physics - and now you are desperately twisting your epicycloids to explain the retrograde motion of Mars. If you only could understand Newton's theory of gravitation...

You call "BS" the standard theory of classical electrodynamics because you did not study it well enough to understand that your 'tiny batteries in series' model is not a model of the complete system, but just of the mathematical portion that is conservative. Look at the "Map of the Ring Quest" picture I've posted yesterday: do you recognize your model in the rhs?
After I have scanned a few images, I will make a dedicated post  to explain what you have done, how you can see it by manipulating the integral vector equations, and why that model is not a model of your system but only of a part of it.

Here's what my crystal ball forecasts:
Ahh yes you three guys and your crystalballs! What gives? I have a pair of 200mm crystalballs, some 100's and some 80's, and they don't work near as good as yours!
Maybe they work better depending on who's operating them?
(https://i.postimg.cc/5NsPHxQ4/3Gypsies.jpg)
https://i.postimg.cc/5NsPHxQ4/3Gypsies.jpg (https://i.postimg.cc/5NsPHxQ4/3Gypsies.jpg)
Above: The 3 gypsies. Last bastion of truth.


Quote
You will cursorily read it. You will not understand it. And three years from now, you will still be unable to tell the difference between lumped circuits (to which you can apply 'extended KVL' like Hayt and Ulaby show) and unlumpable circuits (for which not even the 'extended' version of KVL works BECAUSE THE ********** CHANGING FLUX IS LINKED BY YOUR ******* CIRCUIT PATH)

Wait, so are you admitting that the current version of KVL does work for these cases? Dude, why didn't you tell me that a year ago?

By the way, you admitted that the voltages on V1 and V2 on the below diagram will be the same:
(https://i.postimg.cc/fTgyDNp0/20211119-030105.jpg)
https://i.postimg.cc/fTgyDNp0/20211119-030105.jpg (https://i.postimg.cc/fTgyDNp0/20211119-030105.jpg)

And you also admitted that V2 above would work in a KVL loop, but you claimed that V1 above would not hold in a KVL loop.

And then you absolutely refused to tell me how it is possible that V2 can hold, but V1 won't hold, even though they have the EXACT SAME VOLTAGE on the terminals of the transformer secondary.

You act like bringing the wires of V1 near eachother somehow makes it from unlumpable to lumped, even though there is absolutely ZERO real affect on the operation of the circuit or any of the measurements.

So is it some technicality that you can't put into WORDS?

Why would two electrically IDENTICAL circuits which measure the EXACT SAME voltages in all places be such that one fails KVL and the other doesn't, even though the voltages still sum to zero, just because the physical position of a wire is changed without changing any voltages anywhere?


Title: Re: #562 – Electroboom!
Post by: Jesse Gordon on November 23, 2021, 09:11:09 pm
Does anybody really know what we're actually arguing about here?

I think it boils down to the assertion: "KVL doesn't hold in non-conservative fields".

I can understand why we all could have a complex and fascinating discussion about the conservatism (or lack thereof) regarding the short-open-ended-air-core transformer I used in my Lewin Clock, because it does create a nice large non-conservative field where a slight change in the path along most planes does create a slight change in the measured voltage across two points.

I would expect the Lewinites to disagree with me on whether that can be correctly probed because they do not realize that there are planes within that field that are conservative, thus allowing probing to take place. I have in mind making another video where I would take a 6" diameter loop of wire and cover it with small toroids, allowing me to have the magnetic field confined but still measure voltage at points along the single turn, without having my volt meter leads in a non-conservative field. Or maybe I'll just submerge it in ferrofluid or magnetite powder.

But I digress.

What I don't understand from the Lewinites is their problem with closed-magnetic-circuit-core transformers, like this:

(https://i.postimg.cc/fTgyDNp0/20211119-030105.jpg)
https://i.postimg.cc/fTgyDNp0/20211119-030105.jpg (https://i.postimg.cc/fTgyDNp0/20211119-030105.jpg)

The non-conservative field is now confined to within the solid cross sectional area of the core material itself, which is exclusive to the winding wires.

There is no longer a field that produces a slight change in voltage due to a slight change in path, because the electrical wires and the transformer core material are exclusive of eachother's space because they are both solids, so the wire cannot move into that region of a non-conservative field.

We can change the voltage in one turn steps, but this is very different than a field where a slight change in path causes a slight change in voltage - it's all or nothing. It's more of a logical binary field, or a quantized field, but even then it still really bends out of shape the idea of the word "field" because we are missing the very heart of the idea of a field - a gradient along which any infinite number of values can be found based on the position in that field: There is no longer an infinite number of unique positions and values, and it's not dependent so much of where the path is, but how many times the path circles a certain area.

So yes, there is a non-conservative field, but it is clearly confined cleanly inside the core, and thus, where the wires are running, there is no non-conservative field, and the wires cannot get into the conservative field because they are also a solid.

Sure, adding or removing turns on the transformer changes the voltage, but that's a path inside the transformer, I don't see what that has to do with the voltage on the transformer terminals.

Silicon Soup modeled the forward and reverse E-Field along the wire, but does KVL care about the e-fields along the wire inside the transformer? It cares only of the voltage across the terminals, just like it doesn't care about the chemicals inside a battery or the carbon inside a resistor or if I may succinctly say the strictly solid silicon soup inside a soldered solar cell.

But, OF COURSE LEWIN IS RIGHT. That's a given. Don't dare question that!


So then, are the Lewinites bent ouf of shape because they fear the wire might actually merge into the core material, thus entering that non-conservative field, and thus changing the readings, and thus making KVL fail?

Or are they basically in agreement that KVL would appear to hold fine as measured with volt meters, but because of a technicality, it's not actually holding due to the IEC definition of "Voltage" not accounting for Faraday's law?

The most frustrating thing is they don't even seem to know what they are arguing for or against, so long as Lewin is Right.

And I guess that explains why they can't solve simple engineering problems and why they refuse to answer simple questions or really commit to anything: They know that if they reveal what they (don't) know, their argument will have nothing left.


EDIT/PS: I guess calling the secondary winding on a transformer above a non-conservative field is like calling a battery pack a non-conservative field, because you can change the series-parallel configuration of the cells in the battery pack and thus change the voltage by changing the path inside the battery pack, so it's a non-conservative field, and can't be used with KVL. :-DD
Title: Re: #562 – Electroboom!
Post by: Sredni on November 23, 2021, 09:21:27 pm
You know, it's funny that you keep asking the same question, after I already gave you the answer. Like jesuscf that keeps reverting back to the 5 + 3 + 0 = 8  vs 5 + 3 - 8 = 0 dichotomy for lumpable circuits (that can be treated as lumped or unlumped, depending on what circuit path we choose), and completely disregarding the unlumpable circuits it cannot be applied to.

Guys...
IT'S A SAILBOAT!

(http://www.austinchronicle.com/binary/3015/maxresdefault__6_.jpg)

Focus.
Once you see it, you can't unsee it. That's why you see a lot of KVLers turn to Faradians, but never a Faradian turn KVLer.

You know, one of the reasons I keep pushing these concepts to KVLers is that I wanted to see if I am able to break down this 'unituitive' concept in so elementary little pieces that even a bag of nails could understand it.
I have a bag of nails in my basement but it's deceptively irresponsive. People like you, on the other hand, answer the age old question: "what if a bag of nails could post messages on the Internet?" and serve my purpose just right.

You are being too much repetitive, though. Have patience, and I will repeat the answers to the questions you believe I have not answered again. Since you are jumping and yapping with that circuit, I will postpone Jesuscf explanation of his 'tiny little batteries' model after my forthcoming post on "Lumpable (lumped and unlumped) and unlumpable circuits for Dummies".

Then I will make a post where I collect a selection of the misconceptions you keep writing in your long replies. Just to show you are making up rules that are in your head only.

Then I will post the detailed solution of the Lewin ring with an added battery, to show that all EMFs that are not from EM induction appears in the path integral on the left, and there is no place to fit the EMF on the rhs in the path integral of the total electric field E. You can still do some math manipulation and bring the quantity to the left, but this will leave you with the path integral of the conservative part of the electric field.

And this will take us to the tiny batteries model proposed by jesuscf (and basically what appears in RSD Academy videos  - almost all seven of them, most are hidden now) and why it does not model the entire physical system, but only a part of it.

I also intend to show you what you are measuring with your Lewin Clock. I already wrote that in a reply to jesuscf but I feel you need pictures with vivid colors.
Title: Re: #562 – Electroboom!
Post by: jesuscf on November 23, 2021, 09:37:33 pm
Once you see it, you can't unsee it. That's why you see a lot of KVLers turn to Faradians, but never a Faradian turn KVLer.

Suuure!  In the attached figure is the definition of KVL from the book Electromagnetics by Branislav M. Notaros.  Go argue with the author of that book too!!!  By the way, why did you use the very same definition of KVL to solve the problem I proposed before?
Title: Re: #562 – Electroboom!
Post by: Sredni on November 23, 2021, 09:56:32 pm
Once you see it, you can't unsee it. That's why you see a lot of KVLers turn to Faradians, but never a Faradian turn KVLer.

Suuure!  In the attached figure is the definition of KVL from the book Electromagnetics by Branislav M. Notaros.  Go argue with the author of that book too!!!  By the way, why did you use the very same definition of KVL to solve the problem I proposed before?

You still do not understand, do you?
Well, I don't have that book, but my take is that the author is talking about lumped circuits.
For lumped circuits the circulation of E is zero. Then you can break up the integral into the 'passive' and 'active' parts, the rhs still being zero. Now, by the sheer power of mathematics you can put the parts of the circulation integral corresponding to the passive portions on the left (or the right, if you wish) and the parts of the circulation integral corresponding to the active portions on the right (or the left, if you wish).

All these parts are PART OF THE ONE AND ONLY CIRCULATION OF THE TOTAL ELECTRIC FIELD.

When you have electromagnetic induction, if your circuit is lumpable you can choose

1- to not lump it
then your circuit path goes through the component and your circuit path encloses the magnetic region.
You have to apply faraday and end up with A COMPLETE CIRCULATION PATH INTEGRAL ON THE LEFT, and A SURFACE INTEGRAL ON THE RIGHT.

2- to lump it
you change the circuit path for your circuit: yuo no longer pass throught the component, but JUMP AT THE TERMINALS. Now your new circuit path (for the same physical circuit) skips the magnetic region that can be hidden inside the component. You access the component through the terminals, that's the part of the circulation path that JUMPS from one terminal to the other. So, now the effect of your lumped component is ACCOUNTED FOR IN THE PATH INTEGRAL ON THE LEFT.

And these two methods work for lumpABLE circuit that can be considered at will either lumpED or NOT lumped.

But when you deal with circuits whose circuit path IS REQUIRED TO CONTAIN THE MAGNETIC REGION IN ITS INTERIOR you no longer can afford the luxury of lumping it and treat it as a lumped circuit.
NO MORE KVL.
ONLY FARADAY.

I will make a post "Lumpable (lumped and not lumped) and unlumpable circuits for Dummies"
And I forecast you will still not understand it.
Title: Re: #562 – Electroboom!
Post by: jesuscf on November 23, 2021, 10:49:13 pm
Once you see it, you can't unsee it. That's why you see a lot of KVLers turn to Faradians, but never a Faradian turn KVLer.

Suuure!  In the attached figure is the definition of KVL from the book Electromagnetics by Branislav M. Notaros.  Go argue with the author of that book too!!!  By the way, why did you use the very same definition of KVL to solve the problem I proposed before?

You still do not understand, do you?
Well, I don't have that book, but my take is that the author is talking about lumped circuits.
For lumped circuits the circulation of E is zero. Then you can break up the integral into the 'passive' and 'active' parts, the rhs still being zero. Now, by the sheer power of mathematics you can put the parts of the circulation integral corresponding to the passive portions on the left (or the right, if you wish) and the parts of the circulation integral corresponding to the active portions on the right (or the left, if you wish).

All these parts are PART OF THE ONE AND ONLY CIRCULATION OF THE TOTAL ELECTRIC FIELD.

When you have electromagnetic induction, if your circuit is lumpable you can choose

1- to not lump it
then your circuit path goes through the component and your circuit path encloses the magnetic region.
You have to apply faraday and end up with A COMPLETE CIRCULATION PATH INTEGRAL ON THE LEFT, and A SURFACE INTEGRAL ON THE RIGHT.

2- to lump it
you change the circuit path for your circuit: yuo no longer pass throught the component, but JUMP AT THE TERMINALS. Now your new circuit path (for the same physical circuit) skips the magnetic region that can be hidden inside the component. You access the component through the terminals, that's the part of the circulation path that JUMPS from one terminal to the other. So, now the effect of your lumped component is ACCOUNTED FOR IN THE PATH INTEGRAL ON THE LEFT.

And these two methods work for lumpABLE circuit that can be considered at will either lumpED or NOT lumped.

But when you deal with circuits whose circuit path IS REQUIRED TO CONTAIN THE MAGNETIC REGION IN ITS INTERIOR you no longer can afford the luxury of lumping it and treat it as a lumped circuit.
NO MORE KVL.
ONLY FARADAY.

I will make a post "Lumpable (lumped and not lumped) and unlumpable circuits for Dummies"
And I forecast you will still not understand it.

I'll tell you a little secret, but only if you promise to tell everybody:  if you make the 'lumps' small enough, any circuit is lumpable!
Title: Re: #562 – Electroboom!
Post by: bsfeechannel on November 23, 2021, 10:49:59 pm
No, it all boils down to certain people being unable to understand what they read, and being incapable of assimilating new concepts that go beyond their limited knowledge. Like trying to explain the orbit of planets with epicycloids: it works numerically, but it betrays a fundamental ignorance of the actual physical phenomena behind the motion.

Do you really think that by now these guys haven't already figured out that their rhetoric is pointless? They know darn well that Lewin is right, Mehdi is wrong, KVL doesn't hold under a varying magnetic field, static wires are just low ohm resistors that can't generate voltages, you can measure two different voltages across the same two points in a circuit, etc., etc., etc.

So, if they are not intellectually impaired, why do they keep producing assertions that deny the evidence and reject facts and logic? Even when repeatedly debunked left and right?

As I said in an early post, stupidity is a moral issue. Even when you give them the answer they expect, they'll say you're wrong. This is because they determined arbitrarily that you're wrong. No matter what.

Although discussing with them may sometimes produce an interesting socratic-like dialog, it also may give the occasional reader the impression that their questioning is legitimate.
Title: Re: #562 – Electroboom!
Post by: jesuscf on November 24, 2021, 12:01:32 am
No, it all boils down to certain people being unable to understand what they read, and being incapable of assimilating new concepts that go beyond their limited knowledge. Like trying to explain the orbit of planets with epicycloids: it works numerically, but it betrays a fundamental ignorance of the actual physical phenomena behind the motion.

Do you really think that by now these guys haven't already figured out that their rhetoric is pointless? They know darn well that Lewin is right, Mehdi is wrong, KVL doesn't hold under a varying magnetic field, static wires are just low ohm resistors that can't generate voltages, you can measure two different voltages across the same two points in a circuit, etc., etc., etc.

So, if they are not intellectually impaired, why do they keep producing assertions that deny the evidence and reject facts and logic? Even when repeatedly debunked left and right?

As I said in an early post, stupidity is a moral issue. Even when you give them the answer they expect, they'll say you're wrong. This is because they determined arbitrarily that you're wrong. No matter what.

Although discussing with them may sometimes produce an interesting socratic-like dialog, it also may give the occasional reader the impression that their questioning is legitimate.

You have nothing else but to resort to the "Moral High Ground Fallacy".  On the other hand, the KVL side has provided theoretical results and experimental results that perfectly agree with each other.  What have you provide?  Just a big pile of steaming BS.
Title: Re: #562 – Electroboom!
Post by: thinkfat on November 24, 2021, 05:00:20 pm
You have nothing else but to resort to the "Moral High Ground Fallacy".  On the other hand, the KVL side has provided theoretical results and experimental results that perfectly agree with each other.  What have you provide?  Just a big pile of steaming BS.

If both your theory and experiments depend on each other, that'll do you no good.

Because the experimental data presented here was obtained by more or less blindly poking around circuits with a volt meter, and without any understanding of what is actually going on. So the measurements taken do not support in any form the claim that "KVL holds".


I didn't subtract, I measured.

[...]

I didn't make up any framework. I just used my volt meter, resistors, and transformers to find out what observable reality is.

[...]

My level-1 "framework" is what I observe. Above that I do my best to understand what's going on at a theoretical level. Maxwell, Faraday, and Kirchhoff described it elegantly. I have no problem with any of them.


So, what Jesse does is poking around a circuit with a volt meter without having an idea what it's going to show, and as he sees the measurements he makes up some explanations but doesn't validate them against established science.

In fact he doesn't understand Maxwell-Faraday beyond the level of "volts per turn" and especially doesn't understand that the equation describes a relation between the magnetic flux in the core and an (rotational, non-conservative) electric field around the core that is extending to infinity, this being the reason why there cannot be an "inside" or "outside" of a transformer as he understands it. He doesn't realize it is this electric field that is separating charges in a path of conductor and causing "voltage" and that one needs to consider paths through this electric field to see if a loop of wire is a "transformer secondary" or not. Thus all the conclusions he derives from his observation are worthless.

Allow me to elaborate. Below picture shows the problem I gave Jesse to evaluate. It is a toroidal core with "1V of EMF per turn". A ring of resistors is around the core. One has double the value of the other. Volt meters are connected as shown and all numbers were provided by him. I have only added one additional volt meter showing "2/3V" to demonstrate my point. I've tried visualizing the electric field component with green arrows. RED loops are "hot", voltage is induced (1V). BLUE loops are "cold", no voltage is induced. I took over Jesses notation of a "signed voltage" which here means "phase", because obviously this is all AC and there is no "sign" on a RMS volt meter. But I guess he measured with an oscilloscope and found that the voltage is "inverted" in relation to the voltage across the other resistor.

(https://www.eevblog.com/forum/amphour/562-electroboom!/?action=dlattach;attach=1331327)

Now, to see why the blue loops are indeed cold one needs to know a tiny bit of vector calculus, or just accept the fact that all electric field components along any of the blue loops sum up to zero, in other words:
$${\oint{\mathrm{E}\;\mathrm{d}l} = 0}$$
That looks familiar.

With this in mind it should be easily visible why the red loops are "hot", because the electric field components do not sum up to zero:
$$\oint{\mathrm{E}\;\mathrm{d}l} = -\frac{\mathrm{d}B}{\mathrm{d}t},\; (\mathrm{= EMF = 1V})$$
This should also look familiar.

This tells us that in the blue loops there is no contribution from the magnetic flux in the transformer core and the volt meters in any of them display only what results from the electric currents flowing. That explains why we have 1/3V, 0V, 0V and 2/3V, Ohm's Law lets us expect that. They also sum up to the EMF of 1V which we should expect to find according to Mr. Faraday. These blue loops are those where KVL holds.

Now lets walk through both red loops clockwise. In the R+2R loop the path is with the curl of the electric field, in the 2R loop the path is against the curl, so lets just say we expect an EMF of "-1V" along this path and run with that simplification. We know that the electric field across 2R is 2/3V (measured along a "cold" path) and the EMF is "-1V" and when we sum that up we get to the "-1/3V" that Jesse's magic phase-aware volt meter was showing. Note that I am sticking with Jesse's frame of reference on purpose to explain what he "measures", being fully conscious that it is not a full analogy.

This analysis explains all the measurements without making anyone unhappy except Mr. Kirchhoff, but we can easily accept that, given that the more fundamental laws by Mr. Ohm and Mr. Faraday are satisfied. Jesse's claim that "KVL holds" because all the voltages "sum up to 0" is without basis because of a faulty interpretation of the results obtained.

Now the big question is of course, can KVL be somehow made to work in this arrangement, and for that we need to find an equivalent circuit with lumped elements. That will prove to be difficult, because one cannot find a place where to stick a lumped transformer winding or voltage source and still satisfy all the measurements taken in the various places. We can obviously not put it in the wires between the resistors, because we measure 0V across them. It can also not be in "R" and/or in "2R". If we put a 2/3V source "inside" 2R, that would violate the "-1/3V" reading on the rightmost volt meter.

So, where is it? Apparently it is there, but we cannot pinpoint it and measurements between two identical points show different results depending on how we instrument the circuit (1/3V, 2/3V, -1/3V). For circuit theory and KVL this is a nightmare. That's why there are "equivalent circuits" e.g. for transformers which try to model physics with lumped elements full of imaginary numbers and "magic items" like ideal transformers because they relieve engineers of having to think about physics. And to make KVL work. But now and then, when Sir James Clerk Maxwell makes an appearance, everyone is baffled why their circuits don't work.
Title: Re: #562 – Electroboom!
Post by: Jesse Gordon on November 24, 2021, 07:02:47 pm
You know, it's funny that you keep asking the same question, after I already gave you the answer. Like jesuscf that keeps reverting back to the 5 + 3 + 0 = 8  vs 5 + 3 - 8 = 0 dichotomy for lumpable circuits (that can be treated as lumped or unlumped, depending on what circuit path we choose), and completely disregarding the unlumpable circuits it cannot be applied to.

Guys...
IT'S A SAILBOAT!
(http://www.austinchronicle.com/binary/3015/maxresdefault__6_.jpg)


Then I will make a post where I collect a selection of the misconceptions you keep writing in your long replies. Just to show you are making up rules that are in your head only.


Ahh yes, you, the knower of all truth, the fortune telling Gypsie!
(https://i.postimg.cc/5NsPHxQ4/3Gypsies.jpg)


Quote
I also intend to show you what you are measuring with your Lewin Clock. I already wrote that in a reply to jesuscf but I feel you need pictures with vivid colors.

That'll be a hoot! You can't even solve for the voltage on a resistor driven by a transformer with  an ideal core!
Title: Re: #562 – Electroboom!
Post by: Jesse Gordon on November 24, 2021, 07:20:04 pm
No, it all boils down to certain people being unable to understand what they read, and being incapable of assimilating new concepts that go beyond their limited knowledge. Like trying to explain the orbit of planets with epicycloids: it works numerically, but it betrays a fundamental ignorance of the actual physical phenomena behind the motion.

Do you really think that by now these guys haven't already figured out that their rhetoric is pointless? They know darn well that Lewin is right, Mehdi is wrong, KVL doesn't hold under a varying magnetic field, static wires are just low ohm resistors that can't generate voltages, you can measure two different voltages across the same two points in a circuit, etc., etc., etc.

So, if they are not intellectually impaired, why do they keep producing assertions that deny the evidence and reject facts and logic? Even when repeatedly debunked left and right?

As I said in an early post, stupidity is a moral issue. Even when you give them the answer they expect, they'll say you're wrong. This is because they determined arbitrarily that you're wrong. No matter what.

Although discussing with them may sometimes produce an interesting socratic-like dialog, it also may give the occasional reader the impression that their questioning is legitimate.

That is really really weak, my friend.

You must recognize that it is a complex issue on all sides, otherwise Dr. Belcher wouldn't have been so broad in his stamp of approval of Mehdi and Dr. McDonald wouldn't have outright stated that Lewin's loop was within the scope of applicability of Kirchhoff's Voltage Equations.

If you cannot recognize that it's a complex issue and that all of us are trying to argue for what we really think is the truth, then you probably aren't doing so yourself.

So what then? Do we all just start accusing eachother of being stupid and immoral? What good is that going to do? Both sides can do that equally.

Why not deal with the issue? Why not actually talk about the actual crux of the issue?

Frankly, you're sounding more and more like the "dead inside" you described near the opening of this discussion.

Why not answer the following question?

Q: Does KVL at the very least appear to hold as measured by a volt meter in the following diagram:

(https://i.postimg.cc/1tNL2Mmn/20211122-214739.jpg)

Once you answer that, then we can move on and you can explain what you think is really going on.

But when you won't even come to grips with the observable part of reality, then what makes me think you got the rest right?
Title: Re: #562 – Electroboom!
Post by: Jesse Gordon on November 24, 2021, 07:23:43 pm
And I forecast you will still not understand it.

Dude! That crystalball of yours is getting a flatspot from over use!! Give it a break!
Title: Re: #562 – Electroboom!
Post by: Jesse Gordon on November 24, 2021, 07:25:04 pm
No, it all boils down to certain people being unable to understand what they read, and being incapable of assimilating new concepts that go beyond their limited knowledge. Like trying to explain the orbit of planets with epicycloids: it works numerically, but it betrays a fundamental ignorance of the actual physical phenomena behind the motion.

Do you really think that by now these guys haven't already figured out that their rhetoric is pointless? They know darn well that Lewin is right, Mehdi is wrong, KVL doesn't hold under a varying magnetic field, static wires are just low ohm resistors that can't generate voltages, you can measure two different voltages across the same two points in a circuit, etc., etc., etc.

So, if they are not intellectually impaired, why do they keep producing assertions that deny the evidence and reject facts and logic? Even when repeatedly debunked left and right?

As I said in an early post, stupidity is a moral issue. Even when you give them the answer they expect, they'll say you're wrong. This is because they determined arbitrarily that you're wrong. No matter what.

Although discussing with them may sometimes produce an interesting socratic-like dialog, it also may give the occasional reader the impression that their questioning is legitimate.

You have nothing else but to resort to the "Moral High Ground Fallacy".  On the other hand, the KVL side has provided theoretical results and experimental results that perfectly agree with each other.  What have you provide?  Just a big pile of steaming BS.

It really is their religion, isn't it? I hate to say it, but that seems to be what it comes down to.
Title: Re: #562 – Electroboom!
Post by: jesuscf on November 24, 2021, 07:55:19 pm
You have nothing else but to resort to the "Moral High Ground Fallacy".  On the other hand, the KVL side has provided theoretical results and experimental results that perfectly agree with each other.  What have you provide?  Just a big pile of steaming BS.

If both your theory and experiments depend on each other, that'll do you no good.

Because the experimental data presented here was obtained by more or less blindly poking around circuits with a volt meter, and without any understanding of what is actually going on. So the measurements taken do not support in any form the claim that "KVL holds".

Talk for yourself!  Don't reflect your ignorance into other people.  When I saw Lewin's experiment for the first time I thought immediately: he is ignoring the induced voltage in the scope probes!   The other day I showed Lewin's circuit to a friend of mine and almost instantaneously he also said "he is measuring the voltages incorrectly".  I bet that is the case with the vast majority of people that know how to use an oscilloscope.

As for "If both your theory and experiments depend on each other, that'll do you no good", well, last time I checked that is how science works.  That is exactly how Maxwell equations were derived.


Title: Re: #562 – Electroboom!
Post by: jesuscf on November 24, 2021, 07:57:10 pm
No, it all boils down to certain people being unable to understand what they read, and being incapable of assimilating new concepts that go beyond their limited knowledge. Like trying to explain the orbit of planets with epicycloids: it works numerically, but it betrays a fundamental ignorance of the actual physical phenomena behind the motion.

Do you really think that by now these guys haven't already figured out that their rhetoric is pointless? They know darn well that Lewin is right, Mehdi is wrong, KVL doesn't hold under a varying magnetic field, static wires are just low ohm resistors that can't generate voltages, you can measure two different voltages across the same two points in a circuit, etc., etc., etc.

So, if they are not intellectually impaired, why do they keep producing assertions that deny the evidence and reject facts and logic? Even when repeatedly debunked left and right?

As I said in an early post, stupidity is a moral issue. Even when you give them the answer they expect, they'll say you're wrong. This is because they determined arbitrarily that you're wrong. No matter what.

Although discussing with them may sometimes produce an interesting socratic-like dialog, it also may give the occasional reader the impression that their questioning is legitimate.

You have nothing else but to resort to the "Moral High Ground Fallacy".  On the other hand, the KVL side has provided theoretical results and experimental results that perfectly agree with each other.  What have you provide?  Just a big pile of steaming BS.

It really is their religion, isn't it? I hate to say it, but that seems to be what it comes down to.

Sadly, I think you are correct.
Title: Re: #562 – Electroboom!
Post by: Jesse Gordon on November 24, 2021, 09:59:18 pm
First of all, let me say THANK YOU, you are the only one actually trying to think stuff through on the Lewin side, and I really appreciate that!

You have nothing else but to resort to the "Moral High Ground Fallacy".  On the other hand, the KVL side has provided theoretical results and experimental results that perfectly agree with each other.  What have you provide?  Just a big pile of steaming BS.

If both your theory and experiments depend on each other, that'll do you no good.

Because the experimental data presented here was obtained by more or less blindly poking around circuits with a volt meter, and without any understanding of what is actually going on. So the measurements taken do not support in any form the claim that "KVL holds".


I didn't subtract, I measured.

[...]

I didn't make up any framework. I just used my volt meter, resistors, and transformers to find out what observable reality is.

[...]

My level-1 "framework" is what I observe. Above that I do my best to understand what's going on at a theoretical level. Maxwell, Faraday, and Kirchhoff described it elegantly. I have no problem with any of them.


So, what Jesse does is poking around a circuit with a volt meter without having an idea what it's going to show, and as he sees the measurements he makes up some explanations but doesn't validate them against established science.


Naturally, I object to what I describe as your mischaracterization of my understanding.
 
Quote
In fact he doesn't understand Maxwell-Faraday beyond the level of "volts per turn"

Not true, while I don't understand at the level of Dr. McDonald or Dr. Belcher, I do understand more than just "volts per turn."

As I have repeated so many times, my argument is that a closed-magnetic-circuit-core transformer secondary MODELS AND MEASURES as if "volts per turn" is sufficient to describe it as a 2 terminal element for the sake of KVL.

But I've also always invited you to explain why you think KVL is not holding even though it measures like it is.

Quote
and especially doesn't understand that the equation describes a relation between the magnetic flux in the core and an (rotational, non-conservative) electric field around the core that is extending to infinity,

Explain this some more please. If it is non conservative and extends to infinity, then I can detect it from 1ft away, with no part of my measuring apparatus closer than 1 foot from the toroid under test?

Or do you mean I have to run a wire THROUGH the core to detect this electric field?


Quote
this being the reason why there cannot be an "inside" or "outside" of a transformer as he understands it. He doesn't realize it is this electric field that is separating charges in a path of conductor and causing "voltage" and that one needs to consider paths through this electric field to see if a loop of wire is a "transformer secondary" or not. Thus all the conclusions he derives from his observation are worthless.

Oh come on LOL Worthless?  :-DD

It sounds like you're basically saying that we cannot use the secondary winding of a transformer as a 2 terminal element in a KVL loop with other elements because some of those other elements may be subject to the electric field produced by the transformer and may have voltages induced in them?

I hope not because that's absurd. There is no non-conservative field OUTSIDE a toroid for paths that do not go through the hole: Thus, if there is only one path through the hole in the toroid, and it's the secondary winding, and none of the other parts of the other elements go through the hole in the toroid, then all the other elements are not in a non-conservative field.

And by the way, I asked you several times and you never answered, but I'd still like to know if you think the following transformer core design sends out an E-Field into space, and if so, what if we wrapped it in a superconductor enclosure:

(https://i.postimg.cc/5yD35kYf/20211116-110812.jpg)

But anyway, Snedri already admitted that V2 in the below diagram would be suitable for a KVL loop, and he also admitted that V1 and V2 will have the exact same voltage:

(https://i.postimg.cc/fTgyDNp0/20211119-030105.jpg)

Quote
Allow me to elaborate. Below picture shows the problem I gave Jesse to evaluate. It is a toroidal core with "1V of EMF per turn". A ring of resistors is around the core. One has double the value of the other. Volt meters are connected as shown and all numbers were provided by him. I have only added one additional volt meter showing "2/3V" to demonstrate my point. I've tried visualizing the electric field component with green arrows. RED loops are "hot", voltage is induced (1V). BLUE loops are "cold", no voltage is induced. I took over Jesses notation of a "signed voltage" which here means "phase", because obviously this is all AC and there is no "sign" on a RMS volt meter. But I guess he measured with an oscilloscope and found that the voltage is "inverted" in relation to the voltage across the other resistor.

Yes, I used a scope to determine the phase. This convention was started by Lewin who also used scopes and said "At a given instant in time, ......." so I do the same. At a given instant in time, these are the voltages. A moment before or a moment later, they won't be. But at this instant in time, they are.

Quote
(https://www.eevblog.com/forum/amphour/562-electroboom!/?action=dlattach;attach=1331327)

Now, to see why the blue loops are indeed cold one needs to know a tiny bit of vector calculus, or just accept the fact that all electric field components along any of the blue loops sum up to zero, in other words:
$${\oint{\mathrm{E}\;\mathrm{d}l} = 0}$$
That looks familiar.

With this in mind it should be easily visible why the red loops are "hot", because the electric field components do not sum up to zero:
$$\oint{\mathrm{E}\;\mathrm{d}l} = -\frac{\mathrm{d}B}{\mathrm{d}t},\; (\mathrm{= EMF = 1V})$$
This should also look familiar.

Just because a claim is hard to prove doesn't mean it's false.

In this case, just because a certain loop is hard to probe, it doesn't mean KVL doesn't hold.

Your probing method inherently subtracts induced voltage from your reading, leaving you with only the ohmic losses.

For example in the below diagram, stretch out the wires so the 3 volt meters are a million miles from the toroid. Two of the meters read 1v and the third still reads zero. It's not because there are zero volts induced, it's because there is one volt induced on each secondary and the difference between 1 and 1 is zero.

(https://i.postimg.cc/LX6wZVFJ/20211121-235058.jpg)

You are, buy definition, only measuring ohmic losses, because you are using a method of measuring which specifically subtracts all induced voltage differences.

And besides, KVL specifically requires that all elements have EXACTLY 2 terminals -- so of course when you try to throw in an element with multiple terminals (which is what you have done by running a volt meter lead through the transformer) of course KVL appears to fail because you're using it how it's not meant to work.


I think we can agree now though that simple 2-terminal closed-magnetic-circuit-core transformer secondary windings work fine as an element in a KVL loop with other components? Or do you still disagree with that?

Are you saying that the electric field produced by a toroidal transformer can induce voltages in other elements in the KVL loop, thus causing KVL to fail?

Quote
This tells us that in the blue loops there is no contribution from the magnetic flux in the transformer core and the volt meters in any of them display only what results from the electric currents flowing. That explains why we have 1/3V, 0V, 0V and 2/3V, Ohm's Law lets us expect that. They also sum up to the EMF of 1V which we should expect to find according to Mr. Faraday. These blue loops are those where KVL holds.

Now lets walk through both red loops clockwise. In the R+2R loop the path is with the curl of the electric field, in the 2R loop the path is against the curl, so lets just say we expect an EMF of "-1V" along this path and run with that simplification. We know that the electric field across 2R is 2/3V (measured along a "cold" path) and the EMF is "-1V" and when we sum that up we get to the "-1/3V" that Jesse's magic phase-aware volt meter was showing. Note that I am sticking with Jesse's frame of reference on purpose to explain what he "measures", being fully conscious that it is not a full analogy.

This analysis explains all the measurements without making anyone unhappy except Mr. Kirchhoff, but we can easily accept that, given that the more fundamental laws by Mr. Ohm and Mr. Faraday are satisfied. Jesse's claim that "KVL holds" because all the voltages "sum up to 0" is without basis because of a faulty interpretation of the results obtained.

Objection your honor!  :-DD

The reason Mr. Kirchhoff is unhappy is because you're categorically ignoring measurements of some of the voltage differences on some of the elements in the loop.


Quote
Now the big question is of course, can KVL be somehow made to work in this arrangement, and for that we need to find an equivalent circuit with lumped elements. That will prove to be difficult, because one cannot find a place where to stick a lumped transformer winding or voltage source and still satisfy all the measurements taken in the various places. We can obviously not put it in the wires between the resistors, because we measure 0V across them. It can also not be in "R" and/or in "2R". If we put a 2/3V source "inside" 2R, that would violate the "-1/3V" reading on the rightmost volt meter.

So, where is it? Apparently it is there, but we cannot pinpoint it and measurements between two identical points show different results depending on how we instrument the circuit (1/3V, 2/3V, -1/3V). For circuit theory and KVL this is a nightmare. That's why there are "equivalent circuits" e.g. for transformers which try to model physics with lumped elements full of imaginary numbers and "magic items" like ideal transformers because they relieve engineers of having to think about physics. And to make KVL work. But now and then, when Sir James Clerk Maxwell makes an appearance, everyone is baffled why their circuits don't work.

The only reason we cannot pinpoint it is because it is all the way around. Sure, it MODELS and MEASURES as if it's at the center, but if you look at Faraday, and Maxwell, it's dB/dt inside an area, and measuring it on a solid core of effective infinite length is difficult because the active element of the transformer is no different than our volt meter leads, and they too, suffer from that same effect of induced voltage.

The difficulty in probing is that the volt meter has to be on one side of the infinitely long solenoid. That doesn't mean that there isn't a certain volts per turn of induced EMF, it just means it's difficult to measure in less than 1 turn increments because the volt meter leads have to go around the solenoid an integer number of times, unless we're allowed to run the volt meter probes through the

However, if we use MaxEQ to our benefit, we can find ways to measure the voltage across a transformer turn or a partial turn without our probe leads becoming also voltage sources.

That is how on my Lewin Clock I was able to measure the partial turn voltage differences, because I understand enough to know that above my air core pancake transformer, there are certain planes in which there is no non-conservative field - and by running my probe wires along these planes I was able to measure the exact voltages that KVL predicts.

My Lewin Clock is nothing more than a continuously variable variac. It simply allows the secondary winding taps to be placed at any desired point.

If Ferrofluid wasn't so expensive, I would try the same thing submerged in ferrofluid, basically that would be a liquid core transformer, it would have a closed magnetic circuit and yet a tap could be placed anywhere and the volt meter leads would not be in a non-conservative field, and fractional turn voltages could be easily measured.

And why do I go on about fractional turn? Because here's the shell game we're playing.

First we start off with the given, "KVL IS FOR THE BIRDS." Lewin said it, it must be true.

So people like I come along and say "Ahh, but look, I have a transformer output, I put it in series with some resistors, and I considered these each as elements, measured the voltage differences around the loop, and the sum was zero..

But then the lewinites come back and say "No! You have to create the loop INSIDE a transformer, by forming a KVL ring in a toroid, using HALF turns as elements instead of WHOLE turns."

Why? Because the infinitely long nature of the toroid makes it difficult to  probe the voltage induced across a half turn. That's why. It has nothing to do with the fact that KVL works any differently on whole turns or half turns, they all work the same, it's just that it's harder to measure the voltage induced across a partial turn on a toroid using volt meters.

So the entire Lewinite argument is about making the measurement difficult, not that KVL actually fails.

But then people like me come back and come up with ways to measure partial turn voltage differences, and again, KVL holds just fine even with partial turns and correct probing.

And then we end up with this crazyness where you claim there is 0 volts across all the wires, but 1 volt across the resistors, where exactly is the voltage coming from?
Oh? It's induced? So it is there. It's just difficult to measure in certain specific situations which have been designed for it to be difficult to measure.

And that is why Dr. McDonald said that Lewin's loop is within the range of applicability for  Kirchhoff's Voltage Equations.

~~~

So at this point can we agree that KVL works fine with a toroidal transformer secondary winding as a two-terminal element in a loop of that plus resistors, like this:

(https://i.postimg.cc/1tNL2Mmn/20211122-214739.jpg)

Because all of your "KVL FAILS Proofs" seem to be based on using a two partial turns on an infinite core, can we agree that whole turns on an infinite core work fine with KVL?

Then we can get onto the nitty gritty of why you think partial turns are failing.

Ultimately, a half turn is no different than a full turn, it's just harder to measure the voltage across it on a single magnetic-circuit infinite core.
Title: Re: #562 – Electroboom!
Post by: Sredni on November 25, 2021, 02:57:07 am
Lumpable (lumped and not lumped) and unlumpable for Dummies


What makes a circuit lumpable? First of all its physical dimension have to be negligible compared to the wavelength of the electrical oscillations, and then voltages and currents 'offered' at the terminals must be well defined at any time. You want your circuit to be independent on how you measure voltage between two points or current along the same branch.

Here is a snapshot from page 2 of "Basic Circuit Theory", by Desoer and Kuh (don't let the title deceive you: this is the Bentley of circuit theory books).

(https://i.postimg.cc/76mcdQNg/Desoer-Kuh.jpg)
Source: Desoer, Kuh "Basic Circuit Theory"
https://i.postimg.cc/76mcdQNg/Desoer-Kuh.jpg

Note: in the case of Lewin's ring we know that the frequency is so low that retardation effects play no role; the condition that voltages and current be well defined is what we must be careful about. In particular, it's voltage the variable in discussion.
If we can interact with the component only through its terminals and we are not allowed to get inside, the only paths along which we can evaluate voltage (which is a path integral) are those 'outside' the component. And we require that the voltage be the same no matter how we choose the path joining the terminals.

In the case of magnetic components, we know that voltage along the path joining two points depends on the path, when we can go from one side of the magnetic flux region to the other. This is a direct consequence of one of Maxwell's equations (Faraday's law: curl E = -dB/dt) and basic vector integral calculus (Stokes theorem, the definitions of circulation and the definition of flux).

In order for voltage to be well defined, we must avoid paths that go through or 'on the other side' of the variable magnetic flux. The reason is simple: if the same starting and ending points admit two paths that are on opposite sides of the magnetic region, then the closed path formed by joining these two paths will enclose a variable flux and they must necessarily sum up to a nonzero value

Fig. two paths and the area enclosed (to be added later)

Now we know how to make voltage unique: allow only paths that cannot get into the forbidden zone. We enclose the component inside a black box (or an impenetrable wall) and we do not question what is inside. We must also ensure that our circuit path - the orange dashed line - does not contain variable magnetic flux itself.

Here is a lumpable circuit. The same circuit can be considered


(https://i.postimg.cc/8kFQxNqc/Lumpable-can-be-lumped-or-not.jpg)
Fig. the same lumpable circuit can be considered either lumped or not lumped
https://i.postimg.cc/8kFQxNqc/Lumpable-can-be-lumped-or-not.jpg

So far, so good: it looks like we have freedom to choose when we can make KVL works.
NO. There are circuits that are not lumpABLE. There are circuits where the circuit path is REQUIRED by definition or physical constraint to include the variable flux region. In this case it is not possible to find a green circuit path that connects the lumped components AND does not contain the variable flux region at its interior. Only orange paths are available.
Lewin's ring is an example of such a circuit. The resistors R1 and R2 are required to be on the opposite sides of the solenoid, i.e. the variable flux region. And you cannot find a circuit path that connects them without enclosing such forbidden zone. Voltages for points on your circuit will be path dependent.

(https://i.postimg.cc/NfhVVT6C/Unlumpable-circuit.jpg)
Fig. an unlumpable circuit: you cannot exclude dB/dt region from the circuit path
https://i.postimg.cc/NfhVVT6C/Unlumpable-circuit.jpg

And, no. You cannot model it with several tiny little transformer secondaries distributed along the perimeter, because you would forfeit the constraint that the two resistors be on the opposite sides of the variable magnetic region. It's exactly that constrain - your circuit path containing the dB/dt region - that makes Lewin's ring so special.
If you change the magnetic field region you are considering a different problem

(https://i.postimg.cc/YCKNjzg2/Spot-the-differences.png)
Fig. an unlumpable circuit: you cannot exclude dB/dt region from the circuit path
https://i.postimg.cc/YCKNjzg2/Spot-the-differences.png

In order to model it with two or four lumped coils, the magnetic field region must be split to accommodate a circuit path that does not include any of it. I used the same 'stellated' path style used by Feynman in figure 22-9 on page 22-7 of his second volume of lectures.

Title: Re: #562 – Electroboom!
Post by: jesuscf on November 25, 2021, 03:58:40 am
Lumpable (lumped and not lumped) and unlumpable for Dummies


What makes a circuit lumpable? First of all its physical dimension have to be negligible compared to the wavelength of the electrical oscillations, and then voltages and currents 'offered' at the terminals must be well defined at any time. You want your circuit to be independent on how you measure voltage between two points or current along the same branch.

Here is a snapshot from page 2 of "Basic Circuit Theory", by Desoer and Kuh (don't let the title deceive you: this is the Bentley of circuit theory books).

([url]https://i.postimg.cc/76mcdQNg/Desoer-Kuh.jpg[/url])
Source: Desoer, Kuh "Basic Circuit Theory"
[url]https://i.postimg.cc/76mcdQNg/Desoer-Kuh.jpg[/url]

Note: in the case of Lewin's ring we know that the frequency is so low that retardation effects play no role; the condition that voltages and current be well defined is what we must be careful about. In particular, it's voltage the variable in discussion.
If we can interact with the component only through its terminals and we are not allowed to get inside, the only paths along which we can evaluate voltage (which is a path integral) are those 'outside' the component. And we require that the voltage be the same no matter how we choose the path joining the terminals.

In the case of magnetic components, we know that voltage along the path joining two points depends on the path, when we can go from one side of the magnetic flux region to the other. This is a direct consequence of one of Maxwell's equations (Faraday's law: curl E = -dB/dt) and basic vector integral calculus (Stokes theorem, the definitions of circulation and the definition of flux).

In order for voltage to be well defined, we must avoid paths that go through or 'on the other side' of the variable magnetic flux. The reason is simple: if the same starting and ending points admit two paths that are on opposite sides of the magnetic region, then the closed path formed by joining these two paths will enclose a variable flux and they must necessarily sum up to a nonzero value

Fig. two paths and the area enclosed (to be added later)

Now we know how to make voltage unique: allow only paths that cannot get into the forbidden zone. We enclose the component inside a black box (or an impenetrable wall) and we do not question what is inside. We must also ensure that our circuit path - the orange dashed line - does not contain variable magnetic flux itself.

Here is a lumpable circuit. The same circuit can be considered

  • lumpED, when we avoid the forbidden paths by erecting an impenetrable wall and we only access the magnetic component through its terminals (note the circuit path that does not go into the forbidden zone, but instead jumps at the terminals). KVL works inside the green circuit path.
  • NOT lumped, when we consider a circuit path (for the same physical system that we usually address with the name 'circuit') that encloses part or all of the variable magnetic flux region. In this case any path inside the orange dashed line that represent the circuit's premises is allowed. Some of these paths - not all, but only one would suffice - can 'go on the other side' of the magnetic region, making voltage between two points NOT WELL DEFINED.

([url]https://i.postimg.cc/8kFQxNqc/Lumpable-can-be-lumped-or-not.jpg[/url])
Fig. the same lumpable circuit can be considered either lumped or not lumped
[url]https://i.postimg.cc/8kFQxNqc/Lumpable-can-be-lumped-or-not.jpg[/url]

So far, so good: it looks like we have freedom to choose when we can make KVL works.
NO. There are circuits that are not lumpABLE. There are circuits where the circuit path is REQUIRED by definition or physical constraint to include the variable flux region. In this case it is not possible to find a green circuit path that connects the lumped components AND does not contain the variable flux region at its interior. Only orange paths are available.
Lewin's ring is an example of such a circuit. The resistors R1 and R2 are required to be on the opposite sides of the solenoid, i.e. the variable flux region. And you cannot find a circuit path that connects them without enclosing such forbidden zone. Voltages for points on your circuit will be path dependent.

([url]https://i.postimg.cc/NfhVVT6C/Unlumpable-circuit.jpg[/url])
Fig. an unlumpable circuit: you cannot exclude dB/dt region from the circuit path
[url]https://i.postimg.cc/NfhVVT6C/Unlumpable-circuit.jpg[/url]

And, no. You cannot model it with several tiny little transformer secondaries distributed along the perimeter, because you would forfeit the constraint that the two resistors be on the opposite sides of the variable magnetic region. It's exactly that constrain - your circuit path containing the dB/dt region - that makes Lewin's ring so special.
If you change the magnetic field region you are considering a different problem

([url]https://i.postimg.cc/YCKNjzg2/Spot-the-differences.png[/url])
Fig. an unlumpable circuit: you cannot exclude dB/dt region from the circuit path
[url]https://i.postimg.cc/YCKNjzg2/Spot-the-differences.png[/url]

In order to model it with two or four lumped coils, the magnetic field region must be split to accommodate a circuit path that does not include any of it. I used the same 'stellated' path style used by Feynman in figure 22-9 on page 22-7 of his second volume of lectures.


Sure, now go tell the electric power industry that they can not lump a 200 km long power line and apply KVL... because reasons I guess?

Lewin's circuit is one of the easiest ones to lump: it is a very simple transformer.  It is literally in every basic electric circuits textbook!  And guess what: KVL works nicely on it.

Once again: you can lump anything if the size of the lump is small enough.  It may not be practical, or easy, but it is doable.

So now a varying magnetic field doesn't induce an EMF in the ring?  How would you explain a ring made of resistors exclusively?  What about the little wires between points A and D, very closely placed to the ring, that Lewin used to take the measurements?  Don't they do get an EMF as well?



Title: Re: #562 – Electroboom!
Post by: Jesse Gordon on November 25, 2021, 06:26:26 am
Lumpable (lumped and not lumped) and unlumpable for Dummies


What makes a circuit lumpable? First of all its physical dimension have to be negligible compared to the wavelength of the electrical oscillations, and then voltages and currents 'offered' at the terminals must be well defined at any time. You want your circuit to be independent on how you measure voltage between two points or current along the same branch.

Here is a snapshot from page 2 of "Basic Circuit Theory", by Desoer and Kuh (don't let the title deceive you: this is the Bentley of circuit theory books).

(https://i.postimg.cc/sf4j3HbF/Desoer-Kuh.jpg)
Source: Desoer, Kuh "Basic Circuit Theory"
https://i.postimg.cc/sf4j3HbF/Desoer-Kuh.jpg


Most excellent! The  Bentley of circuit theory books is very clear:

As long as the current entering one terminal equals the current exiting the other terminal and the voltage difference between the  two terminals can be UNAMBIGUOUSLY defined by PHYSICAL measurements, then KVL holds!

That's what I've been saying!

So I guess then that both V1 and V2 in the below diagram would hold for KVL since you already admitted V2 will hold for KVL and that both V1 and V2 will measure the same with my volt meter, that's an unambiguous physical measurement, right?

(https://i.postimg.cc/fTgyDNp0/20211119-030105.jpg)

https://i.postimg.cc/fTgyDNp0/20211119-030105.jpg (https://i.postimg.cc/fTgyDNp0/20211119-030105.jpg)

Quote




Note: in the case of Lewin's ring we know that the frequency is so low that retardation effects play no role; the condition that voltages and current be well defined is what we must be careful about. In particular, it's voltage the variable in discussion.
If we can interact with the component only through its terminals and we are not allowed to get inside, the only paths along which we can evaluate voltage (which is a path integral) are those 'outside' the component. And we require that the voltage be the same no matter how we choose the path joining the terminals.

In the case of magnetic components, we know that voltage along the path joining two points depends on the path, when we can go from one side of the magnetic flux region to the other. This is a direct consequence of one of Maxwell's equations (Faraday's law: curl E = -dB/dt) and basic vector integral calculus (Stokes theorem, the definitions of circulation and the definition of flux).

In order for voltage to be well defined, we must avoid paths that go through or 'on the other side' of the variable magnetic flux. The reason is simple: if the same starting and ending points admit two paths that are on opposite sides of the magnetic region, then the closed path formed by joining these two paths will enclose a variable flux and they must necessarily sum up to a nonzero value

Fig. two paths and the area enclosed (to be added later)

Now we know how to make voltage unique: allow only paths that cannot get into the forbidden zone. We enclose the component inside a black box (or an impenetrable wall) and we do not question what is inside. We must also ensure that our circuit path - the orange dashed line - does not contain variable magnetic flux itself.

Here is a lumpable circuit. The same circuit can be considered

  • lumpED, when we avoid the forbidden paths by erecting an impenetrable wall and we only access the magnetic component through its terminals (note the circuit path that does not go into the forbidden zone, but instead jumps at the terminals). KVL works inside the green circuit path.
  • NOT lumped, when we consider a circuit path (for the same physical system that we usually address with the name 'circuit') that encloses part or all of the variable magnetic flux region. In this case any path inside the orange dashed line that represent the circuit's premises is allowed. Some of these paths - not all, but only one would suffice - can 'go on the other side' of the magnetic region, making voltage between two points NOT WELL DEFINED.

(https://i.postimg.cc/8kFQxNqc/Lumpable-can-be-lumped-or-not.jpg)
Fig. the same lumpable circuit can be considered either lumped or not lumped
https://i.postimg.cc/8kFQxNqc/Lumpable-can-be-lumped-or-not.jpg

Why does the wall have to be there? How about I put my wall here, in the very center of the toroid just to keep idiots from dropping their volt meter leads through there, as shown in the diagram below - it's functionally identical to your wall, yes? And its a lot less bricks!

(https://i.postimg.cc/qqG0vgRV/20211124-213839.jpg)
https://i.postimg.cc/qqG0vgRV/20211124-213839.jpg (https://i.postimg.cc/qqG0vgRV/20211124-213839.jpg)
Above: See the crosshatched wall inside of the core.

The real answer is "Don't add undocumented turns to your transformer and expect physics to work."

Quote

So far, so good: it looks like we have freedom to choose when we can make KVL works.
NO. There are circuits that are not lumpABLE. There are circuits where the circuit path is REQUIRED by definition or physical constraint to include the variable flux region. In this case it is not possible to find a green circuit path that connects the lumped components AND does not contain the variable flux region at its interior. Only orange paths are available.
Lewin's ring is an example of such a circuit. The resistors R1 and R2 are required to be on the opposite sides of the solenoid, i.e. the variable flux region. And you cannot find a circuit path that connects them without enclosing such forbidden zone. Voltages for points on your circuit will be path dependent.

(https://i.postimg.cc/5NyZDPVS/Unlumpable-circuit.jpg)
Fig. an unlumpable circuit: you cannot exclude dB/dt region from the circuit path
https://i.postimg.cc/5NyZDPVS/Unlumpable-circuit.jpg

Actually, your ring above is COMPLETELY lumpable as a 1 turn transformer output in series with two resistors.

Theres NO REASON WHAT SOEVER that you cannot lump the transformer secondary as a one turn winding. The volt meter will give you an unambiguous physical measurement of that voltage, as you already admitted regarding my above yellow V1 and V2 diagram.

Quote

And, no. You cannot model it with several tiny little transformer secondaries distributed along the perimeter, because you would forfeit the constraint that the two resistors be on the opposite sides of the variable magnetic region. It's exactly that constrain - your circuit path containing the dB/dt region - that makes Lewin's ring so special.
If you change the magnetic field region you are considering a different problem

There's nothing special about Lewin's ring. It's still two half-turns and two resistors all in series.

Just like shown with my Lewin Clock, there are planes along which probe leads may be run where there is no non-conservative field which allows unambiguous physical measurements to be made of the two half-turns.

I know Lewin pretended his bright red solenoid was infinitely long - that is a mistake many men make but it's never true - but even if his solenoid was infinitely long, a volt meter lead could still be run through the center of the solenoid  along that plane which is free of non-conservative fields, and thus the voltage on a half-turn could still be unambiguously physically measured.

And the same thing is true for a toroid, if you drill a small hole through the center of the cross-section you can again run a wire along a plane of no non-conservative fields, so even with a toroid, the voltage across a half-turn can still be physically unambiguously measured.

Quote
(https://i.postimg.cc/x1KgZHcP/Spot-the-differences.png)
Fig. an unlumpable circuit: you cannot exclude dB/dt region from the circuit path
https://i.postimg.cc/x1KgZHcP/Spot-the-differences.png

In order to model it with two or four lumped coils, the magnetic field region must be split to accommodate a circuit path that does not include any of it. I used the same 'stellated' path style used by Feynman in figure 22-9 on page 22-7 of his second volume of lectures.


I'm beginning to think that your whole premise depends on the idea that you consider some core typologies to be impossible to probe half turns on, and I think you're entire argument is if it can't be probed, then KVL fails by definition, even though the voltages are all summing to zero as KVL states, it's just that you think you can't measure it.

But lets say for the sake of discussion that we have a transformer where we cannot actually physically unambiguously measure the voltage across the half-turns, we can still add up the voltage across the resistors and use KVL to solve for induced EMF -- AND WE WOULD BE RIGHT -- so did KVL really fail?

Can we all agree yet that integer turn secondaries on toroidal transformers are suitable as elements in KVL loops so long as we keep the bloody volt meters out of the core?

I don't see how you can disagree.

You already admitted that V1 and V2 will physically unambiguously read the same in my above yellow diagram, and you also admitted that V2 (the twisted pair instance) would hold as an element in a KVL loop.

And your very opening source says that KVL holds as long as the input and output current are the same and the voltage across the terminals is unambiguously physically measurable.

How can you deny it?
Title: Re: #562 – Electroboom!
Post by: thinkfat on November 25, 2021, 05:11:34 pm
You have nothing else but to resort to the "Moral High Ground Fallacy".  On the other hand, the KVL side has provided theoretical results and experimental results that perfectly agree with each other.  What have you provide?  Just a big pile of steaming BS.

If both your theory and experiments depend on each other, that'll do you no good.

Because the experimental data presented here was obtained by more or less blindly poking around circuits with a volt meter, and without any understanding of what is actually going on. So the measurements taken do not support in any form the claim that "KVL holds".

Talk for yourself!  Don't reflect your ignorance into other people.  When I saw Lewin's experiment for the first time I thought immediately: he is ignoring the induced voltage in the scope probes!   The other day I showed Lewin's circuit to a friend of mine and almost instantaneously he also said "he is measuring the voltages incorrectly".  I bet that is the case with the vast majority of people that know how to use an oscilloscope.

As for "If both your theory and experiments depend on each other, that'll do you no good", well, last time I checked that is how science works.  That is exactly how Maxwell equations were derived.

That's an engineers knee-jerk reflex, I know. That's how we're trained and conditioned by test equipment salesmen. I thought so as well, it must be some error in probing. It's not possible to measure two different voltages in the same spot. There must be an identifiable "transformer" in there. The "wires" must have a voltage across. That was until I understood some critical bits of Faradays Law and Maxwells equation concerning it. I watched some MIT video, asked a friend of mine who is really knowledgeable about this stuff (not a physicist but an electrical engineer) and made my conclusions. This new understanding immediately explained the results Mehdi was getting and some other peoples experiments as well.

About the "experiments". It should be abundantly clear by now what is wrong with Jesses EI-core measurements and why they are no proof for "KVL holds". What more than "the solution is in the fields" (plural) can I say. One cannot consider only the magnetic flux and simply ignore the electric field that is inevitably present as well.
Title: Re: #562 – Electroboom!
Post by: bdunham7 on November 25, 2021, 05:35:01 pm
It's not possible to measure two different voltages in the same spot.

But you aren't measuring the voltages in the same spot, are you?  The two voltmeters are in two different spots.
Title: Re: #562 – Electroboom!
Post by: Sredni on November 25, 2021, 06:53:14 pm
Why does the wall have to be there? How about I put my wall here, in the very center of the toroid just to keep idiots from dropping their volt meter leads through there, as shown in the diagram below - it's functionally identical to your wall, yes? And its a lot less bricks!
(https://i.postimg.cc/qqG0vgRV/20211124-213839.jpg)
https://i.postimg.cc/qqG0vgRV/20211124-213839.jpg (https://i.postimg.cc/qqG0vgRV/20211124-213839.jpg)
Above: See the crosshatched wall inside of the core.

Good, it seems that the top right corner of this image I posted nearly a week ago

(https://i.postimg.cc/Yq1ZDcPY/KVL-works-if-I-leave-out-the-magnetic-region.jpg)

has finally reached your brain. A little slow, but better late than never.
Of course, you still don't understand it even though I remade both images using the very same circuit. Voltage is path-dependent in both situations: with or without a wall.
What we choose to do in order to be able to apply KVL in our circuit is limit our circuit path to the region of space where voltage between any two points is the same for all paths within that region of space. In short, we PRETEND voltage is single-valued because we willingly ignore the paths along which it will give multiple values.

The bag of nails in my basement is nodding. It seems to be able to understand this.
Do you?

Quote
Actually, your ring above is COMPLETELY lumpable as a 1 turn transformer output in series with two resistors.

Prove it.
Draw the resistors in that exact position around the shaded dB/dt region, with that shape and size (the one where I write "Example where there is no room to twist it"), and then draw a green dashed line that is your circuit's path. It has to join the resistors' terminals with one another and with the lumped transformer. Note that the shaded disk should all be contained into your lumped transformer, and your green dashed circuit path must NOT INCLUDE the shaded disk.
Go ahead, I'll wait for your picture.

Quote

Just like shown with my Lewin Clock, there are planes along which probe leads may be run where there is no non-conservative field which allows unambiguous physical measurements to be made of the two half-turns.
I am not even sure this sentence makes any sense. But I will post the fields for your clock, and you will see that what you are measuring is the path integral of the conservative part of the electric field along the hands of the clock. So much for your good probing...
But let's put this aside for a while, you will comment when I have posted the pictures.

For the moment, let's see that picture where you lump Lewin's rings by enclosing the shaded disk inside your transformer but NOT inside the circuit path that connects it to the resistors.
Remember, the resistor and the disk must remain in the same position, you cannot move them nor split it.
I am waiting.

Title: Re: #562 – Electroboom!
Post by: bsfeechannel on November 26, 2021, 12:48:55 am
No, it all boils down to certain people being unable to understand what they read, and being incapable of assimilating new concepts that go beyond their limited knowledge. Like trying to explain the orbit of planets with epicycloids: it works numerically, but it betrays a fundamental ignorance of the actual physical phenomena behind the motion.

Do you really think that by now these guys haven't already figured out that their rhetoric is pointless? They know darn well that Lewin is right, Mehdi is wrong, KVL doesn't hold under a varying magnetic field, static wires are just low ohm resistors that can't generate voltages, you can measure two different voltages across the same two points in a circuit, etc., etc., etc.

So, if they are not intellectually impaired, why do they keep producing assertions that deny the evidence and reject facts and logic? Even when repeatedly debunked left and right?

As I said in an early post, stupidity is a moral issue. Even when you give them the answer they expect, they'll say you're wrong. This is because they determined arbitrarily that you're wrong. No matter what.

Although discussing with them may sometimes produce an interesting socratic-like dialog, it also may give the occasional reader the impression that their questioning is legitimate.

You have nothing else but to resort to the "Moral High Ground Fallacy".  On the other hand, the KVL side has provided theoretical results and experimental results that perfectly agree with each other.  What have you provide?  Just a big pile of steaming BS.

It really is their religion, isn't it? I hate to say it, but that seems to be what it comes down to.

Yet the only thing you managed to prove is that Lewin is right and you are wrong. Keep on trying. Who knows you manage to win converts to the cause of stubborn ignorance.
Title: Re: #562 – Electroboom!
Post by: Jesse Gordon on November 26, 2021, 04:06:42 am
No, it all boils down to certain people being unable to understand what they read, and being incapable of assimilating new concepts that go beyond their limited knowledge. Like trying to explain the orbit of planets with epicycloids: it works numerically, but it betrays a fundamental ignorance of the actual physical phenomena behind the motion.

Do you really think that by now these guys haven't already figured out that their rhetoric is pointless? They know darn well that Lewin is right, Mehdi is wrong, KVL doesn't hold under a varying magnetic field, static wires are just low ohm resistors that can't generate voltages, you can measure two different voltages across the same two points in a circuit, etc., etc., etc.

So, if they are not intellectually impaired, why do they keep producing assertions that deny the evidence and reject facts and logic? Even when repeatedly debunked left and right?

As I said in an early post, stupidity is a moral issue. Even when you give them the answer they expect, they'll say you're wrong. This is because they determined arbitrarily that you're wrong. No matter what.

Although discussing with them may sometimes produce an interesting socratic-like dialog, it also may give the occasional reader the impression that their questioning is legitimate.

You have nothing else but to resort to the "Moral High Ground Fallacy".  On the other hand, the KVL side has provided theoretical results and experimental results that perfectly agree with each other.  What have you provide?  Just a big pile of steaming BS.

It really is their religion, isn't it? I hate to say it, but that seems to be what it comes down to.

Yet the only thing you managed to prove is that Lewin is right and you are wrong.
Not by my calculations!
Quote
Keep on trying.
Thank you!
Quote
Who knows you manage to win converts to the cause of stubborn ignorance.
You realize of course both sides feel the same way about the other side, right?

And besides, if your technique works for you, it should work for me, right? HEHE  :-DD :-DD

But seriously, why not address the topic? Why not at the very least will you agree that a TWO TERMINAL transformer secondary on a toroidal transformer works fine as an element in a KVL loop?

PS: Happy Thanksgiving if you're in USA.  :popcorn:
Title: Re: #562 – Electroboom!
Post by: Jesse Gordon on November 26, 2021, 06:24:43 am
Why does the wall have to be there? How about I put my wall here, in the very center of the toroid just to keep idiots from dropping their volt meter leads through there, as shown in the diagram below - it's functionally identical to your wall, yes? And its a lot less bricks!
(https://i.postimg.cc/qqG0vgRV/20211124-213839.jpg)
https://i.postimg.cc/qqG0vgRV/20211124-213839.jpg (https://i.postimg.cc/qqG0vgRV/20211124-213839.jpg)
Above: See the crosshatched wall inside of the core.

Good, it seems that the top right corner of this image I posted nearly a week ago

(https://i.postimg.cc/R04QGyHs/KVL-works-if-I-leave-out-the-magnetic-region.jpg)


has finally reached your brain. A little slow, but better late than never.

Sadly, my understanding hasn't changed. It's just taken you this long to get a clue about what I'm trying to say.

Quote
Of course, you still don't understand it even though I remade both images using the very same circuit. Voltage is path-dependent in both situations: with or without a wall.
What we choose to do in order to be able to apply KVL in our circuit is limit our circuit path to the region of space where voltage between any two points is the same for all paths within that region of space. In short, we PRETEND voltage is single-valued because we willingly ignore the paths along which it will give multiple values.

So to clarify, if I put up a wall inside my core as seen in my diagram directly below, and this wall keeps all the idiots from dropping their volt meters in the hole, then the secondary winding measures and models as exactly a single turn secondary, right? And as long as no other elements of my loop breach that wall, then I'm golden, right?

(https://i.postimg.cc/qqG0vgRV/20211124-213839.jpg)
https://i.postimg.cc/qqG0vgRV/20211124-213839.jpg (https://i.postimg.cc/qqG0vgRV/20211124-213839.jpg)

And since your own trusted source says that KVL holds as long as the current_in = current_out and the voltage is unambiguously physically measurable, I guess we have to agree, with my boundary wall KVL then holds for the exact same reason I've been droning on about for a year now - because you have to model reality or your math isn't going to match reality. The wall just keeps the idiots from dropping their volt meters through the forbidden stargate hahahaha  :-DD :-DD

(And NO NO NO, I'm not saying it IS a stargate, I'm saying it MODELS AND MEASURES AS IF it is.)

You already agreed that KVL would hold for "V2" in the below diagram, and you also already agreed that "V1" and "V2" would unambiguously read the same with a physical measurement:

(https://i.postimg.cc/fTgyDNp0/20211119-030105.jpg)
https://i.postimg.cc/fTgyDNp0/20211119-030105.jpg (https://i.postimg.cc/fTgyDNp0/20211119-030105.jpg)

So I really don't know why you can't agree that the 2 terminal output of a toroidal transformer can work fine as an element in a KVL loop.

Quote
The bag of nails in my basement is nodding. It seems to be able to understand this.
Do you?

This box of left-handed six-faced lug nuts in my basement is shaking its head at you. It cannot believe you've had this explained so clearly to you and you still don't get it!  :-DD

Does it help you when I talk about the box of lug nuts in my basement?

Quote

Quote
Actually, your ring above is COMPLETELY lumpable as a 1 turn transformer output in series with two resistors.

Prove it.

Yipes you're insufferable when it comes to quoting out of context!

You quote me saying "your ring above..." and you put a different picture there. It's similar, but dude, put out an effort to quote in context! All you got to do is include the picture I'm talking about in the quoted text so we can all see what I'm really referring to!

I'll put it here for you:
(https://i.postimg.cc/5NyZDPVS/Unlumpable-circuit.jpg)
https://i.postimg.cc/5NyZDPVS/Unlumpable-circuit.jpg (https://i.postimg.cc/5NyZDPVS/Unlumpable-circuit.jpg)

Now, as you were saying:

Quote

Quote
Actually, your ring above is COMPLETELY lumpable as a 1 turn transformer output in series with two resistors.

Prove it.

Prove what?

You already agreed that "V1" and "V2" read the same in the following diagram - and you also already agreed that "V2" is suitable for use in a KVL loop.
Furthermore, your trusted sources say that if Iout=Iin and the voltage across the terminals is unambiguously physically measurable, then KVL holds.

And because you already agreed that V1 and V2 in diagram below read the same, that tells me they read UNAMBIGUOUSLY  so how can you not agree that KVL would hold in V1 as well?

(https://i.postimg.cc/fTgyDNp0/20211119-030105.jpg)
https://i.postimg.cc/fTgyDNp0/20211119-030105.jpg (https://i.postimg.cc/fTgyDNp0/20211119-030105.jpg)


Quote
Draw the resistors in that exact position around the shaded dB/dt region, with that shape and size (the one where I write "Example where there is no room to twist it"), and then draw a green dashed line that is your circuit's path. It has to join the resistors' terminals with one another and with the lumped transformer. Note that the shaded disk should all be contained into your lumped transformer, and your green dashed circuit path must NOT INCLUDE the shaded disk.
Go ahead, I'll wait for your picture.


What're you blathering on about?

You've already agreed that V1 and V2 will read the same in the above yellow diagram, and you've also agreed that V2 is suitable to be an element in a KVL loop.

AAAAaaand you've also agreed that KVL will hold if we put an imaginary brick wall inside the core that prevents all idiot volt meters from falling through the hole as shown in the diagram directly below:

You full well know that if I measure the voltage across that inner section of winding it will read 1 turn of voltage. Of course the voltage isn't actually all induced just inside there, but because the rest of our loop is not inside there, IT MODELS AND MEASURES AS IF IT IS ENTIRELY INDUCED INSIDE THE CORE and as such ONE TURN MODELS AND MEASURES AS EXACTLY A SINGLE TURN, which is what my claim is.

Are you actually denying that the voltage measured by V1 in diagram below will be something other than one turns worth of voltage?

(https://i.postimg.cc/qqG0vgRV/20211124-213839.jpg)
https://i.postimg.cc/qqG0vgRV/20211124-213839.jpg (https://i.postimg.cc/qqG0vgRV/20211124-213839.jpg)


Quote

Quote

Just like shown with my Lewin Clock, there are planes along which probe leads may be run where there is no non-conservative field which allows unambiguous physical measurements to be made of the two half-turns.
I am not even sure this sentence makes any sense. But I will post the fields for your clock, and you will see that what you are measuring is the path integral of the conservative part of the electric field along the hands of the clock. So much for your good probing...
But let's put this aside for a while, you will comment when I have posted the pictures.

Huh? What the tarzan are you talking about?

Quote
For the moment, let's see that picture where you lump Lewin's rings by enclosing the shaded disk inside your transformer but NOT inside the circuit path that connects it to the resistors.
Remember, the resistor and the disk must remain in the same position, you cannot move them nor split it.
I am waiting.

What exactly are you asking?

You're not very good at describing it. Maybe you should draw something if you want something drawd  :-DD

Ultimately, I'm not sure what your point is. You've already agreed with enough things it's time you step back and re-assess your angle of attack.

And the way you refuse to answer questions tells me you know you're wrong and you're just not about to admit it.

Why not just admit that a secondary winding on a toroidal transformer can be unambiguously measured and therefore KVL will hold fine with such an element?

Then we can move onto where the real dragons be, like aircore transformers and partial turns and all sorts of wonderful stuff!

The fact that you repeatedly refuse to answer that question tells me you don't know what you're talking about.

You're going off a script, hoping it makes more sense to me than to you. Dude, the reason your script looks like it stinks to you is because it stinks!
Title: Re: #562 – Electroboom!
Post by: Jesse Gordon on November 26, 2021, 06:53:29 am
About the "experiments". It should be abundantly clear by now what is wrong with Jesses EI-core measurements and why they are no proof for "KVL holds". What more than "the solution is in the fields" (plural) can I say.

What do you think of Snedri's trusted source:

Isn't it pretty clear that if the voltage across the terminals can be unambiguously physically measured that KVL will hold?
(Please read the whole page, not just what he highlighted.)

(https://i.postimg.cc/sf4j3HbF/Desoer-Kuh.jpg)

Snedri already admitted that V1 and V2 in the following diagram will physically measure the same voltage, and that V2 will hold as an element in a KVL loop:

(https://i.postimg.cc/fTgyDNp0/20211119-030105.jpg)

He also admitted that if we put up an imaginary wall to keep other stuff from going through the core, then the winding becomes suitable for being an element in a KVL loop:

(https://i.postimg.cc/qqG0vgRV/20211124-213839.jpg)

Do you really think that the voltage reading is ambiguous on my EI-Core loop so long as I don't add or remove secondary turns without documenting them?

Quote
One cannot consider only the magnetic flux and simply ignore the electric field that is inevitably present as well.

You keep mentioning that but you won't explain more.

What does this electric field permeate? What if the transformer is inside a Faraday Shield?
Or what if it is enclosed in a superconducting Faraday Shield?
Does this magic field extend deep into the briny ocean? Deep into the earth?

Is it conservative or non-conservative?

Can it be measured without passing a wire through the hole in the core?

Just how well shielded would a transformer need to be before you would expect it to play nicely as an element in a KVL loop?


I really don't see how you can think that there is a non-conservative field for other elements that do not pass through the toroid or EI-Core.

Seriously man, you're smart enough to get this. The others aren't. That's probably why they carry on as if they could never be wrong, but you realize that your position is difficult to support. And now you're on to talking about an "inevitable electric field." I would much rather you tell me about a measurable E-Field, rather than one that you believe deep in your heart must be there for things to work the way you think.

Yeah I watched some MIT youtube videos too. I talked to people too. Does that make me right?

What makes an understanding right is when it accurately predicts and models reality.

And my EI-Core transformer test showed that both half-turns measured exactly as KVL predicts.
Title: Re: #562 – Electroboom!
Post by: thinkfat on November 26, 2021, 08:05:05 am
[...]

Sorry man, it's all been explained and shown in pictures and equations. But you just ignore them. Instead you make up incomplete analogies, like the "wall inside the transformer". Go back and look at the diagram I made about the ring core transformer. It's all there in red, blue and green. If you don't understand how this transfers to your EI core, sorry, not going to spoonfeed you.

The electric field is there. It cannot be ignored. It cannot be discussed away if you don't want to neglect everything discovered in physics since 1861.
Title: Re: #562 – Electroboom!
Post by: thinkfat on November 26, 2021, 08:17:36 am
It's not possible to measure two different voltages in the same spot.

But you aren't measuring the voltages in the same spot, are you?  The two voltmeters are in two different spots.

Assuming we're talking about this:
(https://www.eevblog.com/forum/amphour/562-electroboom!/?action=dlattach;attach=1331327)

Electrically it's the same spot. The piece of wire between the resistors has 0V across it, so obviously there's no potential. You could move the probe points of the volt meters (those across "R" and "2R") until they touch, they would still show the same value, provided the wires have negligible resistance.

PS: edited for clarity.
Title: Re: #562 – Electroboom!
Post by: thinkfat on November 26, 2021, 08:34:15 am
Why does the wall have to be there? How about I put my wall here, in the very center of the toroid just to keep idiots from dropping their volt meter leads through there, as shown in the diagram below - it's functionally identical to your wall, yes? And its a lot less bricks!

(https://i.postimg.cc/qqG0vgRV/20211124-213839.jpg)
https://i.postimg.cc/qqG0vgRV/20211124-213839.jpg (https://i.postimg.cc/qqG0vgRV/20211124-213839.jpg)
Above: See the crosshatched wall inside of the core.

The real answer is "Don't add undocumented turns to your transformer and expect physics to work."

This "wall inside the transformer" is not equivalent. It makes people believe they'll be safe if their probe leads do not cross it and so they stop looking at the complete paths and instead concentrate on "forbidden planes". It has certainly not stopped you from adding "undocumented turns" in your EI core experiment.

Title: Re: #562 – Electroboom!
Post by: thinkfat on November 26, 2021, 12:27:30 pm

Your probing method inherently subtracts induced voltage from your reading, leaving you with only the ohmic losses.
[...]
You are, buy definition, only measuring ohmic losses, because you are using a method of measuring which specifically subtracts all induced voltage differences.

But there's no choice, really. Your voltmeter will only show you the sum of the electric fields along the path formed by its probe leads and whatever is between the tips. This path can either be conservative ("blue") or non-conservative ("red"). But you must treat each path as independent from all other paths. For example in my diagram, the RED path containing the rightmost voltmeter only sums up fields along its path, i.o.w the electric field across "2R" and all external electric field components around the core (green arrows) it encloses! It doesn't matter if the probe leads penetrate your "wall inside the transformer" because "2R" is also on that path, closing the loop. Do you still not see what happened with your EI core?

Quote
Now the big question is of course, can KVL be somehow made to work in this arrangement, and for that we need to find an equivalent circuit with lumped elements. That will prove to be difficult, because one cannot find a place where to stick a lumped transformer winding or voltage source and still satisfy all the measurements taken in the various places. We can obviously not put it in the wires between the resistors, because we measure 0V across them. It can also not be in "R" and/or in "2R". If we put a 2/3V source "inside" 2R, that would violate the "-1/3V" reading on the rightmost volt meter.

So, where is it? Apparently it is there, but we cannot pinpoint it and measurements between two identical points show different results depending on how we instrument the circuit (1/3V, 2/3V, -1/3V). For circuit theory and KVL this is a nightmare. That's why there are "equivalent circuits" e.g. for transformers which try to model physics with lumped elements full of imaginary numbers and "magic items" like ideal transformers because they relieve engineers of having to think about physics. And to make KVL work. But now and then, when Sir James Clerk Maxwell makes an appearance, everyone is baffled why their circuits don't work.

The only reason we cannot pinpoint it is because it is all the way around. Sure, it MODELS and MEASURES as if it's at the center, but if you look at Faraday, and Maxwell, it's dB/dt inside an area, and measuring it on a solid core of effective infinite length is difficult because the active element of the transformer is no different than our volt meter leads, and they too, suffer from that same effect of induced voltage.

Now wait a second. "We cannot pinpoint it because it is all the way around" does not rhyme with treating the red wires in your EI core experiment as lumped voltage sources, right? Because there you claim (incorrectly) that, since your volt meter leads don't "go through the core" they don't have any voltage induced, right? So the only voltage source must be the red wire, right? So which way will you have it? It cannot be both.

Quote
And then we end up with this crazyness where you claim there is 0 volts across all the wires, but 1 volt across the resistors, where exactly is the voltage coming from?
Oh? It's induced? So it is there. It's just difficult to measure in certain specific situations which have been designed for it to be difficult to measure.

So what you're saying is that Nature is treating poor engineers badly and making it difficult for them on purpose. Bad, bad Nature! I shall put you on my naughty list! No presents for you this Christmas!

SCNR  :-DD

PS: the induced potential is not difficult to measure. Just sum up all electric fields along the "R+2R" loop and there you have it: 1V. Just be careful not to accidentally add another non-conservative external electric field to your measurements and forget to account for it.
Title: Re: #562 – Electroboom!
Post by: Jesse Gordon on November 26, 2021, 07:31:47 pm
[...]

Sorry man, it's all been explained and shown in pictures and equations. But you just ignore them.

And you're ignoring observable reality, and refusing to answer questions which would allow you to see how wrong you are, so what good are your so called "explanations?"

You used to be willing to put your beliefs about reality to the test. Now you're getting like the other more experienced lewinites who like to wax eloquent but refuse to actually answer questions because they know they'll fail reality checks.

Quote
Instead you make up incomplete analogies, like the "wall inside the transformer".

Snedri came up with the virtual wall analogy. He had his outside the transformer as an infinite plane, but putting it inside like I did has the exact same effect on measurable reality.

Quote
Go back and look at the diagram I made about the ring core transformer. It's all there in red, blue and green. If you don't understand how this transfers to your EI core, sorry, not going to spoonfeed you.

The thing is, I do understand how transformers model and measure. You don't. That's why you've made like 5 wrong predictions now, because you just don't understand observable reality when it comes to transformers. But hang in here, if you don't slip off into never-never land like the others, you'll get to where you understand it.

Quote
The electric field is there. It cannot be ignored. It cannot be discussed away if you don't want to neglect everything discovered in physics since 1861.

That sounds like an argument from personal conviction, no? That's incredibly weak.

You say "The electric field is there, extending to infinity." But you haven't told me how I can measure it.

Nor have you said whether it is blocked by Faraday shielding, or earth or water or whether there is any way to detect it.

How can I measure this electric field extending to infinity as you first described it?

Speaking of making things up... I think you made that one up.

You also keep refusing to answer whether the electric field from the following transformer also extends into infinity:

(https://i.postimg.cc/5yD35kYf/20211116-110812.jpg)

Seriously though, how can I measure this electric field extending into infinity from a toroidal transformer?

I can see if you had an infinitely long solenoid, then you could argue that you had both magnetic and electric fields extending to infinity since the solenoid itself extends to infinity.

But the beauty of a toroid is that it wraps up infinity to the area the size of a wallnut, or  however big the toroid is.

As I said, seriously, what's the deal with this electric field around a toroid that extends to infinity? An unmeasurable unstoppable field?

You made it up, and that's like the 5th major misunderstanding you've demonstrated.
Title: Re: #562 – Electroboom!
Post by: Jesse Gordon on November 26, 2021, 07:42:19 pm
Why does the wall have to be there? How about I put my wall here, in the very center of the toroid just to keep idiots from dropping their volt meter leads through there, as shown in the diagram below - it's functionally identical to your wall, yes? And its a lot less bricks!

(https://i.postimg.cc/qqG0vgRV/20211124-213839.jpg)
https://i.postimg.cc/qqG0vgRV/20211124-213839.jpg (https://i.postimg.cc/qqG0vgRV/20211124-213839.jpg)
Above: See the crosshatched wall inside of the core.

The real answer is "Don't add undocumented turns to your transformer and expect physics to work."

This "wall inside the transformer" is not equivalent.

Please explain. Do you mean that assuming no probe wires breach the wall in the above diagram, there is still path dependence effecting the probes wires?

Are you saying somehow my volt meter can possibly read anything other than the unambiguous voltage of 1 turns worth of induced voltage, assuming my volt meter leads don't breach the wall?


Quote
It makes people believe they'll be safe if their probe leads do not cross it and so they stop looking at the complete paths and instead concentrate on "forbidden planes". It has certainly not stopped you from adding "undocumented turns" in your EI core experiment.

Whaddya mean? Each of my half turns were exactly a half turn. And all four voltages summed to zero.

Your pipe dream about this magic E-Field that extends to infinity has got you all confused. Please find a way to measure that or give up the idea.
Title: Re: #562 – Electroboom!
Post by: Jesse Gordon on November 26, 2021, 08:20:52 pm

Your probing method inherently subtracts induced voltage from your reading, leaving you with only the ohmic losses.
[...]
You are, buy definition, only measuring ohmic losses, because you are using a method of measuring which specifically subtracts all induced voltage differences.

But there's no choice, really.Your voltmeter will only show you the sum of the electric fields along the path formed by its probe leads and whatever is between the tips.

As I said before, that's why we MODEL and MEASURE a toroidal transformer in an integer number of turns.

If we have an EI core, then we can use half-turns.

Quote
This path can either be conservative ("blue") or non-conservative ("red"). But you must treat each path as independent from all other paths. For example in my diagram, the RED path containing the rightmost voltmeter only sums up fields along its path, i.o.w the electric field across "2R" and all external electric field components around the core (green arrows) it encloses! It doesn't matter if the probe leads penetrate your "wall inside the transformer" because "2R" is also on that path, closing the loop. Do you still not see what happened with your EI core?

So far as you continue to shield yourself from reality, it's not going to make sense. I have asked you multiple times and I ask again: In the diagram directly below, will V1 and V2 read the same voltage? Will either V1 and/or V2 be suitable for use as an element in a KVL loop?

(https://i.postimg.cc/fTgyDNp0/20211119-030105.jpg)

If you were to answer that truthfully, you'd have to say that yes they both read the same and yes a KVL loop would measure to be holding with either V1 or V2. (Heh, or both, as two different elements, which is what my EI-Core was.)

At the end of the day, how can you not see that KVL gives every appearance of holding in this case?


Quote
Quote
Now the big question is of course, can KVL be somehow made to work in this arrangement, and for that we need to find an equivalent circuit with lumped elements. That will prove to be difficult, because one cannot find a place where to stick a lumped transformer winding or voltage source and still satisfy all the measurements taken in the various places. We can obviously not put it in the wires between the resistors, because we measure 0V across them. It can also not be in "R" and/or in "2R". If we put a 2/3V source "inside" 2R, that would violate the "-1/3V" reading on the rightmost volt meter.

So, where is it? Apparently it is there, but we cannot pinpoint it and measurements between two identical points show different results depending on how we instrument the circuit (1/3V, 2/3V, -1/3V). For circuit theory and KVL this is a nightmare. That's why there are "equivalent circuits" e.g. for transformers which try to model physics with lumped elements full of imaginary numbers and "magic items" like ideal transformers because they relieve engineers of having to think about physics. And to make KVL work. But now and then, when Sir James Clerk Maxwell makes an appearance, everyone is baffled why their circuits don't work.

The only reason we cannot pinpoint it is because it is all the way around. Sure, it MODELS and MEASURES as if it's at the center, but if you look at Faraday, and Maxwell, it's dB/dt inside an area, and measuring it on a solid core of effective infinite length is difficult because the active element of the transformer is no different than our volt meter leads, and they too, suffer from that same effect of induced voltage.

Now wait a second. "We cannot pinpoint it because it is all the way around" does not rhyme with treating the red wires in your EI core experiment as lumped voltage sources, right?
Wrong.

I know this is hard for you to understand, but bear with me.

It is completely possible for the induced voltage to be induced all the way around while modeling it for the sake of KVL as a lumped voltage source BECAUSE none of the rest of the mesh passes through the core, and thus none of the rest of the mesh resides in any non-conservative field, because all paths that do not cross through the core are path-independent.


Quote
Because there you claim (incorrectly) that, since your volt meter leads don't "go through the core" they don't have any voltage induced, right?
NOOOOO! I am claiming that they MODEL AND MEASURE as if they have no voltage induced when they don't go through the core. There is no non-conservative field OUTSIDE  an (ideal) toroidal transformer core.


Quote
So the only voltage source must be the red wire, right? So which way will you have it? It cannot be both.
How can you not see that I'm talking about HOW IT MODELS AND MEASURES?

KVL works on how it models and measures.



Quote
Quote
And then we end up with this crazyness where you claim there is 0 volts across all the wires, but 1 volt across the resistors, where exactly is the voltage coming from?
Oh? It's induced? So it is there. It's just difficult to measure in certain specific situations which have been designed for it to be difficult to measure.

So what you're saying is that Nature is treating poor engineers badly and making it difficult for them on purpose. Bad, bad Nature! I shall put you on my naughty list! No presents for you this Christmas!

SCNR  :-DD

ha ha very good!  :-DD

But in all seriousness, I have clearly demonstrated that when we run our volt meter leads along planes where there are no fields, we can in fact measure induced voltages on even many topologies of air-core and open ended transformers.

Quote
PS: the induced potential is not difficult to measure. Just sum up all electric fields along the "R+2R" loop and there you have it: 1V. Just be careful not to accidentally add another non-conservative external electric field to your measurements and forget to account for it.

So can we just add up the ohmic losses, calculate the induced voltage, and sum that all up and find it to be zero, and then deduce that KVL holds, even for Lewin's loop?
After all, if he had summed up his ohmic losses he could have then flipped the sign and calculated the induced voltage, and if he had them added that all up, it would have summed to exactly zero! ha ha ha  :-DD :-DD :-DD :-DD
Title: Re: #562 – Electroboom!
Post by: Sredni on November 26, 2021, 09:29:46 pm
Sadly, my understanding hasn't changed.
...
...
...
Dude, the reason your script looks like it stinks to you is because it stinks!

Oh, goodness me. What a wondrous, copious and melliflously superfluous word salad. All to hide the fact that you cannot lump Lewin's ring. Here, let me repeat the question you are so eloquently avoiding to answer.

This is Lewin's ring: two resistors in a single loop that goes around a circular region (let's consider it of the same size as the loop, so you can see there is no 'room to twist' the wires) of variable magnetic field. The resistors are required to be on the opposite sides of the variable magnetic field region.

(https://i.postimg.cc/pLmfyHxZ/Lewin-ring-is-unlumpable.jpg)
https://i.postimg.cc/pLmfyHxZ/Lewin-ring-is-unlumpable.jpg

Please, show everybody you can draw a circuit path (make it green, meaning it's 'flux-free') that joins the resistors' terminals to the "lumped transformer secondary" terminals and DOES NOT INCLUDE the variable magnetic field region in its interior. Like I did for the lumpABLE circuit I decided to see as lumpED (in my post "Lumpable (lumped and not lumped) and not lumpable circuits for dummies").

In addition, you can also show everybody you can draw the path inside your "lumped transformer secondary" that DOES INCLUDE the variable magnetic field region (make it orange) but IS NOT part of the green circuit path.
I will show you that if you can do that you will run into contradiction.

Too bad my armchair is at the quantum mechanic's shop for repairs and I cannot fly away to another galaxy.
Title: Re: #562 – Electroboom!
Post by: thinkfat on November 26, 2021, 09:47:35 pm
Oh boy, where to even begin...

Okay, first of all: I'm not in denial of observable reality. In fact, I took the reality YOU observed, turned it into a nice, colorful diagram and explained each measurement YOU did using only Ohm, Maxwell and Faraday. I showed in which loops voltage is induced and why not in others, explained every number you obtained.

Here it is again:
(https://www.eevblog.com/forum/amphour/562-electroboom!/?action=dlattach;attach=1331327)

Second: I did not "dream up" any "magic electric field". It's existence was found and explained by James Maxwell. It's his second field equation.

Quote from: Wikipedia
The Maxwell–Faraday version of Faraday's law of induction describes how a time varying magnetic field creates ("induces") an electric field

Third: It is very obvious how to measure it: create a wire loop around any area with changing magnetic flux, create some discontinuity (make a cut, put a resistor in the loop), measure the electric field across it. The probe is just some wire coiled up. It senses non-conservative electric fields. Just the same, each turn around a transformer core "measures" this electric field.

I hope this is not too much to stomach.

You mentioned repeatedly how the magnetic flux is fully contained inside a transformers core (which is not exact but, eh, good enough). Then explain how a conductor (charge particles) outside of the core and thus outside of the magnetic flux, can interact with it. "Spooky action at a distance"? The answer "Dude, because Faraday" is not enough. What is the mechanism behind it?

Btw how's that lumped circuit coming along @Sredni asked you to come up with?
Title: Re: #562 – Electroboom!
Post by: jesuscf on November 27, 2021, 03:36:52 am
Oh, goodness me. What a wondrous, copious and melliflously superfluous word salad. All to hide the fact that you cannot lump Lewin's ring. Here, let me repeat the question you are so eloquently avoiding to answer.

Says the guy that when solving Lewin's ring, the first thing he does is to lump the EMF, and then he follows up by using KVL to find the voltages in the resistors!
Title: Re: #562 – Electroboom!
Post by: Jesse Gordon on November 27, 2021, 04:39:45 am
Sadly, my understanding hasn't changed.
...
...
...
Dude, the reason your script looks like it stinks to you is because it stinks!

Oh, goodness me. What a wondrous, copious and melliflously superfluous word salad. All to hide the fact that you cannot lump Lewin's ring. Here, let me repeat the question you are so eloquently avoiding to answer.

This is Lewin's ring: two resistors in a single loop that goes around a circular region (let's consider it of the same size as the loop, so you can see there is no 'room to twist' the wires) of variable magnetic field. The resistors are required to be on the opposite sides of the variable magnetic field region.

(https://i.postimg.cc/pLmfyHxZ/Lewin-ring-is-unlumpable.jpg)
https://i.postimg.cc/pLmfyHxZ/Lewin-ring-is-unlumpable.jpg

Please, show everybody you can draw a circuit path (make it green, meaning it's 'flux-free') that joins the resistors' terminals to the "lumped transformer secondary" terminals and DOES NOT INCLUDE the variable magnetic field region in its interior. Like I did for the lumpABLE circuit I decided to see as lumpED (in my post "Lumpable (lumped and not lumped) and not lumpable circuits for dummies").

In addition, you can also show everybody you can draw the path inside your "lumped transformer secondary" that DOES INCLUDE the variable magnetic field region (make it orange) but IS NOT part of the green circuit path.
I will show you that if you can do that you will run into contradiction.

Too bad my armchair is at the quantum mechanic's shop for repairs and I cannot fly away to another galaxy.

That's not exactly Lewin's ring, the resistors are too big, but I think I understand what you're asking - and I'll draw a better representation of the Lewin Ring and respond to that after you answer some questions.

It seems you're way behind on answering questions -- all you do is ask ask ask ask, but you never answer in the last few days. It's about time you warmed up your crystal ball to get some answers!

Here's some questions I've been asking for days and you're refusing to answer:

You have admitted that in the V1 and V2 voltages in the diagram directly below will both read the same voltage, and you have also admitted that V2 will be suitable as an element in a KVL loop:

Question One:

How can V1 not also be suitable since they both measure the same voltage? Your own trusted source says that if an unambiguous physical measurement of the voltage across the two terminals can be obtained, then KVL holds! How can V1 not work for KVL?

(https://i.postimg.cc/fTgyDNp0/20211119-030105.jpg)
https://i.postimg.cc/fTgyDNp0/20211119-030105.jpg (https://i.postimg.cc/fTgyDNp0/20211119-030105.jpg)

Question Two:
Considering your own trusted source, if a ONE TURN TRANSFORMER SECONDARY on a toroidal transformer has a voltage that is unambiguously physically measurable, then it should qualify as an element for a KVL loop, Correct?

For example, in the following transformer diagram with a safety wall in the center, why would the output winding not be suitable as an element in a KVL loop?

(https://i.postimg.cc/qqG0vgRV/20211124-213839.jpg)
https://i.postimg.cc/qqG0vgRV/20211124-213839.jpg (https://i.postimg.cc/qqG0vgRV/20211124-213839.jpg)

If you think the above secondary winding is not suitable in an element in a KVL loop, then why? It is possible to unambiguously measure the voltage, right? So then according to the following textbook, which you yourself cite, KVL should hold, right?

(https://i.postimg.cc/sf4j3HbF/Desoer-Kuh.jpg)
https://i.postimg.cc/sf4j3HbF/Desoer-Kuh.jpg (https://i.postimg.cc/sf4j3HbF/Desoer-Kuh.jpg)


If you honestly answer the above questions and I'll do my best to draw up Lewin's loop and show how I would measure the voltage across the half turns.
Title: Re: #562 – Electroboom!
Post by: Sredni on November 27, 2021, 04:52:55 am
Oh, goodness me. What a wondrous, copious and melliflously superfluous word salad. All to hide the fact that you cannot lump Lewin's ring. Here, let me repeat the question you are so eloquently avoiding to answer.
Says the guy that when solving Lewin's ring, the first thing he does is to lump the EMF, and then he follows up by using KVL to find the voltages in the resistors!

No, says the guy who uses Faraday's law and not KVL, but you can't tell the difference.

Ok, this had to be just an appendix to my post "What about the other forms of EMF? Uh, what about 'em(f)?", but since you are impatient, I will anticipate it.

Lewin's ring with a localized EMF: adding a battery
The following example shows that the localized EMFs are part of the path integral on the left of Faraday's equation (in its integral form), while the delocalized inductive EMF is the surface integral on the right. By not being part of the PATH integral, the delocalized inductive EMF is at all effects invisible in the closed circuit represented by the ring.

So, let's solve for the voltages and current in a Lewin ring with the addition of the localized EMF from a battery (shown in the following drawing as a 9V battery)

(https://i.postimg.cc/D0mW70jF/screenshot-4.png)
page 1 https://i.postimg.cc/D0mW70jF/screenshot-4.png

Now, pay attention at how the full closed path is partitioned, because it's that exhaustive partitioning that show established that "the inductive EMF is NOT on the path". Or, as Kirchhoff would have said it: "die induktive elektromotorische Krafte ist NICHT auf dem Wege".


(https://i.postimg.cc/zBGvk31W/screenshot-5.png)
page 2 https://i.postimg.cc/zBGvk31W/screenshot-5.png

Oh, look. I have started with Faraday's law, in its local, differential form. Used Stokes theorem to obtain the integral form of Faraday's law, then applied Faraday's law to obtain an equation that involves, the nonlocalized EMF that I call EMF(t), the localized EMF Vbatt(t) and the current I(t).
Then I solve this Faraday equation to find the current I(t).

No sign of KVL, at all. Because KVL is DEAD.
But the most interesting part is that the source of EMF that is not delocalised, the very localized and lumped EMF battery, shows up as a piece of the path integral on the left hand side. That is the nest where the localised EMF Kirchhoff was arguing about, are born. You know, the "electromotorischen Krafte" he writes about in his formulation.
(This will be more clear when I post the next message)
Title: Re: #562 – Electroboom!
Post by: jesuscf on November 27, 2021, 05:18:21 am
Oh, goodness me. What a wondrous, copious and melliflously superfluous word salad. All to hide the fact that you cannot lump Lewin's ring. Here, let me repeat the question you are so eloquently avoiding to answer.
Says the guy that when solving Lewin's ring, the first thing he does is to lump the EMF, and then he follows up by using KVL to find the voltages in the resistors!
No, says the guy who uses Faraday's law and not KVL, but you can't tell the difference.

A rose by any other name is still a rose!  You are the one that can not tell they are the same...
Title: Re: #562 – Electroboom!
Post by: Sredni on November 27, 2021, 05:36:03 am
It seems you're way behind on answering questions -- all you do is ask ask ask ask, but you never answer in the last few days. It's about time you warmed up your crystal ball to get some answers!
Here's some questions I've been asking for days and you're refusing to answer:

No, I have answered the questions that made some sense. But you have such big holes in your knowledge that you end up asking questions that have little to no sense. I was going to make a post about the misconceptions your posts are full of, but there are way too many. For example, I have written many many times that KVL applies in all regions of space that do not contain/cut a variable magnetic flux. So, in the lumpable circuit with the twisted wire it goes without saying that, since there is no changing flux in the small loop formed by V2, the wires and the jump at the first twist, KVL works.

But you keep asking questions that contain nonsense in them

Quote
You have admitted that in the V1 and V2 voltages in the diagram directly below will both read the same voltage, and you have also admitted that V2 will be suitable as an element in a KVL loop:

What the heck does "V2 (or V1) is suitable as an element in a KVL loop" even mean???
V2 is a voltmeter, to begin with, but let's consider it a branch. A branch alone can be part of a loop to which KVL applies (works) and AT THE SAME TIME be part of a loop to which KVL does NOT apply.
Case in point: Lewin's ring with the two 'external' voltmeters. V1 forms a loop with R1 that you can apply KVL to, but forms with R2 (and V2) loop(s) to which you cannot apply KVL to, you need Faraday.
Therefore:

Quote
Question One:
How can V1 not also be suitable since they both measure the same voltage? Your own trusted source says that if an unambiguous physical measurement of the voltage across the two terminals can be obtained, then KVL holds! How can V1 not work for KVL? 

This question betrays your confusion. I cannot answer this question because you are showing not to possess the prerequisite to formulate a rational question. V2 in the twisted circuit can be though as part of a circuit that stops at the first twist. If you limit yourself to paths that are inside that little loop, V2 will always be part of loops to which KVL can be applied (because in the universe that is the area enclosed by your circuit path there is no dB/dt).
Yes, you can define your circuit path to follow all the twists and go around the core, but that betrays your ignorance of the purpose of making the transformer's terminal so close together: it's to treat the circuit with V2 as lumpED.

V1, on the other hand, is part of a circuit whose circuit path you can immediately see going around the core. Anyway, being a lumpABLE circuit you can choose both ways. If you choose the copper as the trail of your circuit path, you include the dB/dt region inside it. There are paths that when part of a loop along with V1 will go around the core, hence KVL will not apply. Faraday's will.

Quote
Question Two:
Considering your own trusted source, if a ONE TURN TRANSFORMER SECONDARY on a toroidal transformer has a voltage that is unambiguously physically measurable, then it should qualify as an element for a KVL loop, Correct?
For example, in the following transformer diagram with a safety wall in the center, why would the output winding not be suitable as an element in a KVL loop? 

Again, the question betrays your ignorance. That is a lumpABLE circuit. You can treat it either as NOT LUMPED, by allowing paths that go around the core by considering an orange circuit path that follows the conductor around the core, or lumpED by denying this possibility by considering a green circuit path that jumps at the terminals, excluding this possibility. The wall is just a way to represent the fact that you are denying this possibility and your circuit path skips the innards of the transformer. (EDIT: to clarify: what really counts is how you define your circuit path: is it the green path that does not include the dB/dt region? Or is it the orange path that includes the dB/dt region?)
If you deny this possibility, then your voltages will be well defined (as Desoer and Kuh require for KVL to hold).
If you allow this possibility, then your voltage can be multivalued (and Desoer Kuh will say that KVL ceases to work for those loops with a part 'going the other side').

(To tell it all, we should make the terminals close together so that the circuit itself can be shrinked to a point, but I decided not to push this. )
Title: Re: #562 – Electroboom!
Post by: Jesse Gordon on November 27, 2021, 06:02:18 am
Oh boy, where to even begin...

Okay, first of all: I'm not in denial of observable reality. In fact, I took the reality YOU observed, turned it into a nice, colorful diagram and explained each measurement YOU did using only Ohm, Maxwell and Faraday. I showed in which loops voltage is induced and why not in others, explained every number you obtained.

Here it is again:
(https://www.eevblog.com/forum/amphour/562-electroboom!/?action=dlattach;attach=1331327)

Excellent, so if you don't deny observable reality, then please tell me that you agree that the two terminals of a single turn transformer secondary on a toroidal core will MEASURE AND MODEL just fine as a lumped element in a KVL loop, like this:

(https://i.postimg.cc/qqG0vgRV/20211124-213839.jpg)

If not, why do you think the V1,V2,V3 would not sum up to zero?

Quote
Second: I did not "dream up" any "magic electric field". It's existence was found and explained by James Maxwell. It's his second field equation.

Quote from: Wikipedia
The Maxwell–Faraday version of Faraday's law of induction describes how a time varying magnetic field creates ("induces") an electric field

Third: It is very obvious how to measure it: create a wire loop around any area with changing magnetic flux, create some discontinuity (make a cut, put a resistor in the loop), measure the electric field across it. The probe is just some wire coiled up. It senses non-conservative electric fields. Just the same, each turn around a transformer core "measures" this electric field.
Ahhh, you have to add in a secondary winding measure this electric field you talk about extending to infinity. Why'd I have to ask 3 times to get an answer?

You may extend your secondary transformer load wires off into infinity and say "Ahah! An E-Field at Infinity!" But I could sneak in and replace your toroidal transformer with a non-inductive DDS waveform generator and you'd still be out there at infinity gloating about your E-Field which you presumed was made by your toroidal transformer, because the signal from my DDS would be indistinguishable.

And when you do form a loop, you get v. Then you add another loop, and you get 2v. and so on. Is that like stacking 10 plates and being able to pile 10 times the food on the top plate before it spills over the sides?  :-DD :-DD

Obviously it's the dB field that induces the E-Field, and obviously there's "no" dB field outside the toroid core, so where exactly is the E-Field induced? In a conductor enclosing the dB region.

But let's say you take a toroidal transformer, and you have it shielded in the thickest bestest Faraday and magnetic shielding known to mankind, and yet you have a tiny hole going through the shielding so that a wire can pass through the hole in the toroid and come out the other side. You still have a full turn's induced voltage even though only a fraction of the turn is actually inside the Faraday and magnetic shielding.

So what do you have, a super powerful field that blasts through the best known shielding known to mankind, which is completely undetectable unless you get INSIDE the shielding?

As others have said, nature does what nature does. We do our best to model it with physics, but physics are concepts and they don't always exactly describe the fullness of what nature is doing.


But anyway, so what does this have to do with  KVL not holding when one of the elements is the secondary winding on a toroidal transformer?

Are you saying that the E-Field surrounding a toroidal transformer will cause unexpected readings if it is near a KVL loop?

If so, what do you expect? How would I observe this in real life?

When I measured EI-Core secondary widings in a loop, I found that KVL holds just fine.


Quote
I hope this is not too much to stomach.

You mentioned repeatedly how the magnetic flux is fully contained inside a transformers core (which is not exact but, eh, good enough). Then explain how a conductor (charge particles) outside of the core and thus outside of the magnetic flux, can interact with it. "Spooky action at a distance"? The answer "Dude, because Faraday" is not enough. What is the mechanism behind it?


Nature.

We've figured out a number of different very clever ways to model nature in a number of different settings, but you tell me what the ACTUAL mechanism is behind it. What REALLY IS magnetism? You tell me.

At the end of the day, we can encase a toroidal transformer in the best shielding known to man, and it's output winding still has induced upon it a full turn's worth of voltage even though only a fraction of the turn is actually inside the shielding. So you argue that the remainder of the voltage is induced outside the shielding, even though there is no measurable magnetic or electric fields outside of the shield.

Ultimately, our models aren't always pretty for certain situations. Nature does what nature does, and we come up with models which sometimes create apparent paradoxes.

And what has this got to do with whether KVL holds in a particular setting?

Are you saying that a KVL loop will not sum to zero if one of the elements is a 2 terminal toroidal transformer secondary winding?

Quote
Btw how's that lumped circuit coming along @Sredni asked you to come up with?

I'm trying to use that as bargaining currency to get him to answer some questions he's been refusing, but I doubt it'll work, he knows better than to allow his belief to be tested against reality.

By the way, you've been refusing to answer questions too. But at least you talked to me! Thank you!

Do you really think that the two-terminal secondary winding on a toroidal transformer would not work as an element in a KVL loop?

If I summed up the voltages around the loop shown in the diagram above, would they not sum to zero for some reason?

Title: Re: #562 – Electroboom!
Post by: Sredni on November 27, 2021, 06:05:15 am
If you honestly answer the above questions and I'll do my best to draw up Lewin's loop and show how I would measure the voltage across the half turns.

Oh, no. Now you answer MY question, not a question of your choice.
And my question is:

This is Lewin's ring: two resistors in a single loop that goes around a circular region (let's consider it of the same size as the loop, so you can see there is no 'room to twist' the wires) of variable magnetic field. The resistors are required to be on the opposite sides of the variable magnetic field region.

(https://i.postimg.cc/pLmfyHxZ/Lewin-ring-is-unlumpable.jpg)
https://i.postimg.cc/pLmfyHxZ/Lewin-ring-is-unlumpable.jpg

Please, show everybody you can draw a circuit path (make it green, meaning it's 'flux-free') that joins the resistors' terminals to the "lumped transformer secondary" terminals and DOES NOT INCLUDE the variable magnetic field region in its interior. Like I did for the lumpABLE circuit I decided to see as lumpED (in my post "Lumpable (lumped and not lumped) and not lumpable circuits for dummies").

In addition, you can also show everybody you can draw the path inside your "lumped transformer secondary" that DOES INCLUDE the variable magnetic field region (make it orange) but IS NOT part of the green circuit path.
I will show you that if you can do that you will run into contradiction.
Title: Re: #562 – Electroboom!
Post by: Jesse Gordon on November 27, 2021, 07:26:43 am
1: Thank you.

2: It's bedtime for me so I'll draw your diagram for you tomorrow or the next day.

3: Your answers seem like you could have done a lot better, my critique inserted below. I'm hoping you'll approach them a little more honestly, and update them. Up to you of course.


It seems you're way behind on answering questions -- all you do is ask ask ask ask, but you never answer in the last few days. It's about time you warmed up your crystal ball to get some answers!
Here's some questions I've been asking for days and you're refusing to answer:

No, I have answered the questions that made some sense. But you have such big holes in your knowledge that you end up asking questions that have little to no sense. I was going to make a post about the misconceptions your posts are full of, but there are way too many. For example, I have written many many times that KVL applies in all regions of space that do not contain/cut a variable magnetic flux. So, in the lumpable circuit with the twisted wire it goes without saying that, since there is no changing flux in the small loop formed by V2, the wires and the jump at the first twist, KVL works.

But you keep asking questions that contain nonsense in them

Quote
You have admitted that in the V1 and V2 voltages in the diagram directly below will both read the same voltage, and you have also admitted that V2 will be suitable as an element in a KVL loop:

What the heck does "V2 (or V1) is suitable as an element in a KVL loop"???
V2 is a voltmeter, to begin with, but ....

Dude, really? How many times have we talked about the secondary winding on a transformer being a 2-terminal element in a KVL loop?
Did you really not figure out that I was referring to the two secondary windings by the Vx number by each of them?

Did you think I was talking about a volt meter as an element in a KVL loop?

Do I literally need to write like a lawyer and cover every possible path by which you might try to escape from the question? Do you really want my replies to be 10 times size they need to be?

(Heh, besides, a volt meter in parallel with a secondary winding still makes a 2-terminal element, so what the tarzan, leave them connected and put the volt meter in the loop!)

'By "V1" and "V2," I am referring to the two-terminal toroidal transformer secondary windings depicted by "V1" and "V2" in the indicated diagram.'
'  a. In paragraph above, "V1" and "V2" are not referring to any volt meters, oscilloscopes, frequency counters, phase meters, or other measurement instruments not herein specifically mentioned.'
' b. By "toroidal" I mean to describe a transformer who's core is in the shape of a torus.'
' c. blah blah blah.'

Please, if you know what I'm asking, why beat around the bush and act like I just might be asking something else?
If you know what I'm asking, why try to take it to mean something else just to make me look stupid?

Bad faith antics dude.


Quote
... let's consider it a branch. A branch alone can be part of a loop to which KVL applies (works) and AT THE SAME TIME be part of a loop to which KVL does NOT apply.
Case in point: Lewin's ring with the two 'external' voltmeters. V1 forms a loop with R1 that you can apply KVL to, but forms with R2 (and V2) loop(s) to which you cannot apply KVL to, you need Faraday.
Therefore:

Quote
Question One:
How can V1 not also be suitable since they both measure the same voltage? Your own trusted source says that if an unambiguous physical measurement of the voltage across the two terminals can be obtained, then KVL holds! How can V1 not work for KVL? 

This question betray your confusion. I cannot answer this question because you are showing not to possess the prerequisite to formulate a rational question. V2 in the twisted circuit can be though as part of a circuit that stops at the first twist.

I don't care how you think of it I want to know how it will MEASURE.

Quote
If you limit yourself to paths that are inside that little loop, V2 will always be part of loops to which KVL can be applied (because in the universe that is the area enclosed by your circuit path there is no dB/dt).
Yes, you can define your circuit path to follow all the twists and go around the core, but that betrays your ignorance of the purpose of making the transformer's terminal so close together: it's to treat the circuit with V2 as lumpED.

V1, on the other hand, is part of a circuit whose circuit path you can immediately see going around the core. Anyway, being a lumpABLE circuit you can choose both ways. If you choose the copper as the trail of your circuit path, you include the dB/dt region inside it. There are paths that when part of a loop along with V1 will go around the core, hence KVL will not apply. Faraday's will.

You're not really answering my question.

How can V1  not work?

Consider explaining whether THE "V1" (Untwisted) SECONDARY WINDING would actually cause KVL to fail MEASURABLY -- in other words, as I already asked numerous times, if I made a loop out of "THE V1 SECONDARY WINDING" and two resistors, would the voltages around the loop as measured with a volt meter STILL sum to zero even though you say KVL is failing?

Quote
Quote
Question Two:
Considering your own trusted source, if a ONE TURN TRANSFORMER SECONDARY on a toroidal transformer has a voltage that is unambiguously physically measurable, then it should qualify as an element for a KVL loop, Correct?
For example, in the following transformer diagram with a safety wall in the center, why would the output winding not be suitable as an element in a KVL loop? 

Again, the question betrays your ignorance. That is a lumpABLE circuit. You can treat it either as NOT LUMPED, by allowing paths that go around the core by considering an orange circuit path that follows the conductor around the core, or lumpED by denying this possibility by considering a green circuit path that jumps at the terminals, excluding this possibility. The wall is just a way to represent the fact that you are denying this possibility and your circuit path skips the innards of the transformer.
If you deny this possibility, then your voltages will be well defined (as Desoer and Kuh require for KVL to hold).
If you allow this possiblity, then your voltage can be multivalued (and Desoer Kuh will say that KVL ceases to work for those loops with a part 'going the other side').

Ahhh, so on question 1 above, you said that if I use the V1 winding in a loop, KVL will fail.
But here you say so long as I deny the possibility that any other wires pass through the core, then KVL will hold.
So this begs the question, if I simply denied that other turns can be added in the Question 1 diagram, then both windings labeled V1 and V2 would work as 2-terminal elements in a loop and KVL would hold?

Isn't that exactly what I've been saying all along "AS LONG AS WE DON"T ADD OR REMOVE TURNS WITHOUT DOCUMENTING THEM?"

Quote
(To tell it all, we should make the terminals close together so that the circuit itself can be shrinked to a point, but I decided not to push this. )

I don't know if you're referring to your trusted source, but here's what it says:

Quote from: textbook
The key property associated with lumped elements is their small size
(compared to the wavelength corresponding to their normal frequency of operation.)
From the more general electromagnetic field point of view, lumped elements
are point singularities; that is, they have negligible physical dimensions.

It doesn't say the terminals have to be close together.

It just says that the elements need to be small enough compared to the wavelength that their size is negligible.

Negligible means that you can NEGLECT it. In other words, the elements just need to be small enough compared to the wavelength that their size can be neglected since it makes no significant difference.

I think we both agreed already that our wavelength is so much bigger than our elements and loop that we don't have to worry about that. Not sure why you bring it up.

As I said, I'll work on your diagram. I hope you can edit your answers to more directly answer my questions. It's up to you of course.
Title: Re: #562 – Electroboom!
Post by: jesuscf on November 28, 2021, 05:22:10 am
I have a simple challenge for team 'Lewin'.  But before I do so, let me summarize the results presented by Lewin in the original experiment.

The first attached figure shows a diagram of Lewin's original experiment, including the measuring setup with the measuring wires tightly placed close to the one loop ring.  R1 is 100 ohms and R2 is 900 ohms.  Assume the induced EMF is 80 mV; if we disregard the resistance of the green wires, the calculated voltage V1 is -8 mV and the calculated voltage V2 is 72 mV.  I think both team Lewin and team KVL agree on that.  Do we also agree that the left 'voltmeter' shows -8 mV for Vx and the right 'voltmeter' shows 72 mv for Vy?  Team KVL thinks that Vx=V1 and Vy=V2 because the voltage induced in the measuring wires cancels out the voltage induced in the corresponding section of the one loop ring, and KVL works just fine.  Team Lewin thinks that Vx is not equal to Vy because KVL doesn't work, so that the voltage measured between nodes A and D shows different values depending if its taken from the left or from the right.

(https://i.postimg.cc/26qB9ZdW/One-Loop.jpg)

Now consider the circuit of the second attached figure, where the original single loop ring is replaced with a three loop ring with the same diameter as the original one loop ring.  This arrangement of three loops has R1, R2, node A, and node D in the same locations as the original ring.  In the figure, the three rings are separated so it is easier to see how they relate to each other, but in reality they are tightly pack together, with virtually no gap between the three green wire loops.  Once again we have the measuring wires placed tightly close to the ring and  R1 is 100 ohms, R2 is 900 ohms, and since we have three loops now, the induced EMF is 240 mV.  This is the challenge: Calculate V1, V2, Vx, and Vy.

(https://i.postimg.cc/15K4HBxT/Three-Loops.jpg)

(An easy way of making a three loop ring is by making a one loop ring with three times the radius and then fold this bigger ring over itself three times so to end up with three concentric rings with the same radius as the original one loop ring.)


Title: Re: #562 – Electroboom!
Post by: bsfeechannel on November 28, 2021, 06:19:07 am
You realize of course both sides feel the same way about the other side, right?

There are no sides here.

That KVL doesn't hold under varying magnetic fields is an established fact, experimentally confirmed and predicted by the theory. We are not siding with Lewin or anyone or anything.
Title: Re: #562 – Electroboom!
Post by: jesuscf on November 28, 2021, 06:29:51 am
You realize of course both sides feel the same way about the other side, right?

There are no sides here.

That KVL doesn't hold under varying magnetic fields is an established fact, experimentally confirmed and predicted by the theory. We are not siding with Lewin or anyone or anything.

Really, a established fact?  Are you implying that the problem I just posted above has no theoretical solution by using KVL because the induced EMF comes from a varying magnetic field?  Or that the theoretical solution obtained with KVL will not match the experimental results?  The only established fact so far is that bsfeechannel has no idea of what he is talking about!!!
Title: Re: #562 – Electroboom!
Post by: bsfeechannel on November 28, 2021, 06:43:34 am
Oh, goodness me. What a wondrous, copious and melliflously superfluous word salad. All to hide the fact that you cannot lump Lewin's ring. Here, let me repeat the question you are so eloquently avoiding to answer.
Says the guy that when solving Lewin's ring, the first thing he does is to lump the EMF, and then he follows up by using KVL to find the voltages in the resistors!
No, says the guy who uses Faraday's law and not KVL, but you can't tell the difference.

A rose by any other name is still a rose!  You are the one that can not tell they are the same...

You see Sredni? They understand that their claim that KVL, the real KVL, always holds is false. Haven't I told you? But to save face they're now calling Faraday's law KVL.

-- 2+2 = 5.
-- No, two plus two equals four.
-- Yeah, that's what I wrote.
-- No you wrote two plus two equals five.
-- No, I didn't.
-- Yes, you did. How to you call this number: 5?
-- Four.
Title: Re: #562 – Electroboom!
Post by: jesuscf on November 28, 2021, 07:00:27 am
Oh, goodness me. What a wondrous, copious and melliflously superfluous word salad. All to hide the fact that you cannot lump Lewin's ring. Here, let me repeat the question you are so eloquently avoiding to answer.
Says the guy that when solving Lewin's ring, the first thing he does is to lump the EMF, and then he follows up by using KVL to find the voltages in the resistors!
No, says the guy who uses Faraday's law and not KVL, but you can't tell the difference.

A rose by any other name is still a rose!  You are the one that can not tell they are the same...

You see Sredni? They understand that their claim that KVL, the real KVL, always holds is false. Haven't I told you? But to save face they're now calling Faraday's law KVL.

-- 2+2 = 5.
-- No, two plus two equals four.
-- Yeah, that's what I wrote.
-- No you wrote two plus two equals five.
-- No, I didn't.
-- Yes, you did. How to you call this number: 5?
-- Four.

You should be asking Sredni for help on how to solve the problem I posted above

https://www.eevblog.com/forum/amphour/562-electroboom!/msg3840656/#msg3840656 (https://www.eevblog.com/forum/amphour/562-electroboom!/msg3840656/#msg3840656)

because I am pretty sure you lack the capacity to even attempt to solve it.


Title: Re: #562 – Electroboom!
Post by: Sredni on November 28, 2021, 08:17:46 am
I have a simple challenge for team 'Lewin'. 

It's not team 'Lewin'. It's team 'Classical Electrodynamics'.
But what's the point of the exercise?
Both 'teams' are doomed to find the same values for what is read by the voltmeters.

Anyway, if I copied it correctly - and there is no guarantee I did with all those turns making me dizzy - I would say

Vr1 = -24mV, Vr2 = +216mV
Vm1 = -136 mV, Vm2 = -56 mV

Provided I counted the 'rings' right and didn't change some sign here and there.

And if I got one sign wrong somewhere, that would proof that there can be say 200 mV in two inches of copper wire that carry a current of 100 nA?
Title: Re: #562 – Electroboom!
Post by: jesuscf on November 28, 2021, 04:11:06 pm
I have a simple challenge for team 'Lewin'. 

It's not team 'Lewin'. It's team 'Classical Electrodynamics'.
But what's the point of the exercise?
Both 'teams' are doomed to find the same values for what is read by the voltmeters.

Anyway, if I copied it correctly - and there is no guarantee I did with all those turns making me dizzy - I would say

Vr1 = -24mV, Vr2 = +216mV
Vm1 = -136 mV, Vm2 = -56 mV

Provided I counted the 'rings' right and didn't change some sign here and there.

And if I got one sign wrong somewhere, that would proof that there can be say 200 mV in two inches of copper wire that carry a current of 100 nA?

Excellent!  You got the correct results.  Now, what is the voltage between nodes A and D, VAD?
Title: Re: #562 – Electroboom!
Post by: Sredni on November 28, 2021, 04:16:56 pm
Excellent!  You got the correct results.  Now, what is the voltage between nodes A and D, VAD?

I didn't find a way to make it glow, but let's see...

IT DEPENDS ON THE PATH.

Title: Re: #562 – Electroboom!
Post by: jesuscf on November 28, 2021, 04:34:06 pm
Excellent!  You got the correct results.  Now, what is the voltage between nodes A and D, VAD?

I didn't find a way to make it glow, but let's see...

IT DEPENDS ON THE PATH.

No it doesn't depend on the path, because we are calculating it.  You can use either the voltage measured from the left or the voltage measured from the right to find exactly the same voltage between nodes A and D, VAD.
Title: Re: #562 – Electroboom!
Post by: bsfeechannel on November 28, 2021, 07:42:12 pm
The only established fact so far is that bsfeechannel has no idea of what he is talking about!!!

C'mon, man! There's no shame in being wrong. You can bet your bottom dollar that all of us held at some time in the history of our lives the same misconceptions you and Jesse Gordon are now expressing. We normally get aware of voltages when we poke circuits with the probes of our meters and we have this intuitive, but misleading, perception that voltages are generated by the components. What components do is to shape the conditions in which the electromagnetic field exists in that region of the space. Voltages are the consequence of the existence of the fields. Once you switch your "paradigm" to think in terms of fields--not just circuits--you immediately expand the capabilities of your analyses.

For some of us, the shift is very difficult. But it is worth it.
 
Title: Re: #562 – Electroboom!
Post by: jesuscf on November 28, 2021, 07:48:59 pm
The only established fact so far is that bsfeechannel has no idea of what he is talking about!!!

C'mon, man! There's no shame in being wrong. You can bet your bottom dollar that all of us held at some time in the history of our lives the same misconceptions you and Jesse Gordon are now expressing. We normally get aware of voltages when we poke circuits with the probes of our meters and we have this intuitive, but misleading, perception that voltages are generated by the components. What components do is to shape the conditions in which the electromagnetic field exists in that region of the space. Voltages are the consequence of the existence of the fields. Once you switch your "paradigm" to think in terms of fields--not just circuits--you immediately expand the capabilities of your analyses.

For some of us, the shift is very difficult. But it is worth it.

Sure, now use your vast knowledge of electromagnetic fields and help Sredni calculate the voltage VAD from the problem above.
Title: Re: #562 – Electroboom!
Post by: Jesse Gordon on November 28, 2021, 08:52:11 pm
If you honestly answer the above questions and I'll do my best to draw up Lewin's loop and show how I would measure the voltage across the half turns.

Oh, no. Now you answer MY question, not a question of your choice.
And my question is:

This is Lewin's ring: two resistors in a single loop that goes around a circular region (let's consider it of the same size as the loop, so you can see there is no 'room to twist' the wires) of variable magnetic field. The resistors are required to be on the opposite sides of the variable magnetic field region.

(https://i.postimg.cc/kX0TSBw6/Lewin-ring-is-unlumpable.jpg)
https://i.postimg.cc/kX0TSBw6/Lewin-ring-is-unlumpable.jpg

Please, show everybody you can draw a circuit path (make it green, meaning it's 'flux-free') that joins the resistors' terminals to the "lumped transformer secondary" terminals and DOES NOT INCLUDE the variable magnetic field region in its interior. Like I did for the lumpABLE circuit I decided to see as lumpED (in my post "Lumpable (lumped and not lumped) and not lumpable circuits for dummies").

In addition, you can also show everybody you can draw the path inside your "lumped transformer secondary" that DOES INCLUDE the variable magnetic field region (make it orange) but IS NOT part of the green circuit path.
I will show you that if you can do that you will run into contradiction.

OK here's how anybody can unambiguously physically measure the induced voltage in half of an air-core transformer secondary turn:

(https://i.postimg.cc/2SfmvWFM/20211128-113309.jpg)

I did not draw volt meters for the resistors because  measuring the voltage across those is simple.

The red wires represent a path on a plane which (ON THAT PLANE) do not contain a non-conservative field. SO LONG AS THE RED WIRES ARE ON THAT PLANE, the path does not matter. 

That plane passes through the entire center of the primary solenoid winding. Any paths ON THAT PLANE are path-independent.

You may consider the size of the resistors as zero  to make things simple, but the reality is that the resistors occupy a small section of the arc and thus are actually resistors with a small series bit of induced voltage, and the half-turns are actually slightly less than half-turns in series with a small amount of resistance, which is actually always how we model both of those components for high accuracy simulations.

But considering the resistors to have zero length and  the half turns to have exactly half a turn and no resistance simplifies things for this discussion and does not really change the point we're arguing about. So be diversionary if you want but that's what we got.

~~~~~~~

I guess since you're moving on to Lewin's air core solenoid setup you've given up on toroidal and E-Core transformers because you realize they DO in fact hold fine with KVL?

For example in the diagram below, if I were to actually construct and measure with a volt meter the voltage differences across each element in the places indicated by V1 through V5, would the sum be zero?

For clarity I have drawn the idiot screen to signify that I am not allowing the number or configuration of transformer turns to be modified during the midst of the experiment. Ya know what I mean?

I think you cannot deny that Σ(V1,V2,V3,V4,V5) = 0.

(https://i.postimg.cc/jdJntBXT/20211128-121506.jpg)
https://i.postimg.cc/jdJntBXT/20211128-121506.jpg (https://i.postimg.cc/jdJntBXT/20211128-121506.jpg)


Now tell me please, regarding the elements POINTED TO BY V1 through V5 above, which of them is not unambiguously physically measurable with a volt meter?

According to your trusted source below, if the voltage across the elements can all be unambiguously physically measured, then KVL will hold.

(https://i.postimg.cc/sf4j3HbF/Desoer-Kuh.jpg)
https://i.postimg.cc/sf4j3HbF/Desoer-Kuh.jpg (https://i.postimg.cc/sf4j3HbF/Desoer-Kuh.jpg)

How can you deny that KVL is failing to hold in my "KVL HOLDS" diagram above?

Do you really think that Σ(V1,V2,V3,V4,V5) ≠ 0 as measured in real life?

Please explain EXACTLY why you think KVL is failing if you think KVL is failing to hold in my above diagram.

Will the volt meters not all sum to zero? Or will they sum to zero giving the appearance that KVL holds, but for some technical reason you say it's still not holding?

If you only answer one thing, please let it be this:

Q:  Do you believe the volt meters in my "KVL HOLDS" diagram above will sum to zero?
Title: Re: #562 – Electroboom!
Post by: Jesse Gordon on November 28, 2021, 09:00:34 pm
You realize of course both sides feel the same way about the other side, right?

There are no sides here.

That KVL doesn't hold under varying magnetic fields is an established fact, experimentally confirmed and predicted by the theory. We are not siding with Lewin or anyone or anything.

Wow, do you know how weak of an argument that be?

Watch:

"Yeah, there no sides, I'm right and you're wrong.

KVL holds fine under varying magnetic fields as an established fact, experimentally confirmed by my Lewin Clock and predicted by theory.

I'm not siding with anyone or anything, you're just wrong.
"

Do you see how little that attitude actually does for the discussion?

Why not put your powers of experimentally confirmed predictive theory to work and answer this one question:

In the below diagram, will Σ(V1,V2,V3,V4,V5) = 0 as measured in real life by a volt meter?

(https://i.postimg.cc/jdJntBXT/20211128-121506.jpg)

In the above diagram, will Σ(V1,V2,V3,V4,V5) = 0 as measured by a volt meter in real life?

A simply yes or no would be not only much appreciated but a lack of one will show your lack of understanding.

There's nothing complex, it's a transformer with a couple secondary one-turn windings, connected in a loop with two resistors. Easily constructed, and easily measured.

What is your prediction? Will Σ(V1,V2,V3,V4,V5) = 0, as measured by real volt meters in a real world test? Yes or not, please if you can!
Title: Re: #562 – Electroboom!
Post by: Jesse Gordon on November 28, 2021, 09:20:36 pm
Oh boy, where to even begin...

Okay, first of all: I'm not in denial of observable reality.

Excellent!

So tell me please if you can, will Σ(V1,V2,V3,V4,V5) = 0 in the below diagram as measured with real volt meters in a real world test?

(https://i.postimg.cc/jdJntBXT/20211128-121506.jpg)
https://i.postimg.cc/jdJntBXT/20211128-121506.jpg (https://i.postimg.cc/jdJntBXT/20211128-121506.jpg)

Thank you!
Title: Re: #562 – Electroboom!
Post by: Sredni on November 28, 2021, 09:44:53 pm
Excellent!  You got the correct results.  Now, what is the voltage between nodes A and D, VAD?

I didn't find a way to make it glow, but let's see...
IT DEPENDS ON THE PATH.
No it doesn't depend on the path, because we are calculating it.

It's like trying to explain color to someone who is blind since birth.

I have already calculated "VAD" twice: its value depends on the path.
On the path set by the first voltmeter I get -136mV; on the path set by the second voltmeter I get -56mV.
I can compute on any path I want, if I know how the dB/dt region is partitioned by the path itself. Do you have any idea how easy it is for me to compute it? The only difficulty is making sure I am following that spiral labyrinth you set up hoping to get us confused.

Now, I solved your challenge.
Will you solve a few very elementary problems I pose? Or will you evade my questions as Jesse Gordon is doing?
Title: Re: #562 – Electroboom!
Post by: Sredni on November 28, 2021, 09:48:38 pm
If you honestly answer the above questions and I'll do my best to draw up Lewin's loop and show how I would measure the voltage across the half turns.

Oh, no. Now you answer MY question, not a question of your choice.
And my question is:

This is Lewin's ring: two resistors in a single loop that goes around a circular region (let's consider it of the same size as the loop, so you can see there is no 'room to twist' the wires) of variable magnetic field. The resistors are required to be on the opposite sides of the variable magnetic field region.

(https://i.postimg.cc/pLmfyHxZ/Lewin-ring-is-unlumpable.jpg)
https://i.postimg.cc/pLmfyHxZ/Lewin-ring-is-unlumpable.jpg

Please, show everybody you can draw a circuit path (make it green, meaning it's 'flux-free') that joins the resistors' terminals to the "lumped transformer secondary" terminals and DOES NOT INCLUDE the variable magnetic field region in its interior. Like I did for the lumpABLE circuit I decided to see as lumpED (in my post "Lumpable (lumped and not lumped) and not lumpable circuits for dummies").

In addition, you can also show everybody you can draw the path inside your "lumped transformer secondary" that DOES INCLUDE the variable magnetic field region (make it orange) but IS NOT part of the green circuit path.
I will show you that if you can do that you will run into contradiction.
OK here's how anybody can unambiguously physically measure the induced voltage in half of an air-core transformer secondary turn

Nice try, but you did not answer the question.
Let me repeat it for you here, in case you missed:

This is Lewin's ring: two resistors in a single loop that goes around a circular region (let's consider it of the same size as the loop, so you can see there is no 'room to twist' the wires) of variable magnetic field. The resistors are required to be on the opposite sides of the variable magnetic field region.

(https://i.postimg.cc/pLmfyHxZ/Lewin-ring-is-unlumpable.jpg)
https://i.postimg.cc/pLmfyHxZ/Lewin-ring-is-unlumpable.jpg

Please, show everybody you can draw a circuit path (make it green, meaning it's 'flux-free') that joins the resistors' terminals to the "lumped transformer secondary" terminals and DOES NOT INCLUDE the variable magnetic field region in its interior. Like I did for the lumpABLE circuit I decided to see as lumpED (in my post "Lumpable (lumped and not lumped) and not lumpable circuits for dummies").

In addition, you can also show everybody you can draw the path inside your "lumped transformer secondary" that DOES INCLUDE the variable magnetic field region (make it orange) but IS NOT part of the green circuit path.
I will show you that if you can do that you will run into contradiction.

Please, please, please, do not fly to another galaxy with your armchair before answering it.
Title: Re: #562 – Electroboom!
Post by: bsfeechannel on November 28, 2021, 10:59:19 pm
Sure, now use your vast knowledge of electromagnetic fields and help Sredni calculate the voltage VAD from the problem above.

My "vast knowledge" won't help you. I asked the same questions long ago, got the right answers, but ended up like you: confused.

I can show you the door to your enlightenment, but I can't walk you through it. You'll have to do it yourself.
Title: Re: #562 – Electroboom!
Post by: jesuscf on November 28, 2021, 11:51:55 pm
It's like trying to explain color to someone who is blind since birth.

I have already calculated "VAD" twice: its value depends on the path.
On the path set by the first voltmeter I get -136mV; on the path set by the second voltmeter I get -56mV.
I can compute on any path I want, if I know how the dB/dt region is partitioned by the path itself. Do you have any idea how easy it is for me to compute it? The only difficulty is making sure I am following that spiral labyrinth you set up hoping to get us confused.

Now, I solved your challenge.
Will you solve a few very elementary problems I pose? Or will you evade my questions as Jesse Gordon is doing?

You have solve nothing!  For a moment I thought you knew how to solve circuits, but then you come with such a ridiculous argument.  Do you even know what a 'node' is?  Of course I was expecting you to come with some sort of excuse, but this is plainly denial!  I see now that trying to reason with you is impossible.  For everyone else reading, I'll now show how to get the solution, and how simple it is to compute the voltage between the nodes A and D, VAD, a process that was not done correctly in Lewin's presentation.

In the problem I posted above the total generated EMF is 240mV.  Using KVL we now compute the loop current as:

\$
I = \frac{{EMF}}{{R_1  + R_2 }} = \frac{{240mV}}{{100\Omega  + 900\Omega }} = 0.240mA
\$

Now we can compute the voltage drop in each resistor:

\$
\begin{array}{l}
 V_2  = R_2  \cdot I = 900\Omega  \cdot 0.240mA = 216mV \\
 V_1  =  - R_1  \cdot I =  - 100\Omega  \cdot 0.240mA =  - 24mV \\
 \end{array}
\$

To solve for Vx and Vy we need to find an equivalent circuit to work with.  We know that three wire loops are generating 240mV.  So each quarter of a loop can be lumped as a voltage source of 240mV/(3*4)=20mV.  We now notice that the '+' terminal of R1 is 3/4 loop (or a 60mV voltage source) from node 'D' and the '-' terminal of R1 is also 3/4 loop (or a 60mV voltage source)  from node 'A'.   Similarly the Vx 'voltmeter' is 1/4 loop (or a 20mV voltage source) from node 'A' and 1/4 loop (or a 20mV voltage source) from node 'D'.  The same can be said to R2 and the Vy 'voltmeter'.  Being very careful with the signs, this is the equivalent circuit we need:

(https://i.postimg.cc/3JDyh4wB/Three-Loop-Equivalent-Circuit.jpg)

To solve for Vx, we apply KVL to the left loop:

\$
V_x  =  - 20mV - 60mV - ( - 24mV) - 60mV - 20mV =  - 136mV
\$

To solve for Vy, we apply KVL to the right loop:

\$
V_y  = 20mV + 60mV - 216mV + 60mV + 20mV =  - 56mV
\$

Finally, to find VAD, we can calculate it in four different ways from the equivalent circuit above by applying KVL to each one of the four branches between nodes A and D:

\[
\begin{array}{l}
 V_{AD}  = 20mV + ( - 136mV) + 20mV =  - 96mV \\
 V_{AD}  =  - 60mV - ( - 24mV) - 60mV =  - 96mV \\
 V_{AD}  = 60mV - 216mV + 60mV =  - 96mV \\
 V_{AD}  =  - 20mV + ( - 56mV) - 20mV =  - 96mV \\
 \end{array}
\]

So how many values we have for VAD?  Just one: -96mV.

I assembled and tested this circuit.  The voltages I measured are V1=-24.4mV, V2=212mV, Vx=-134mV, Vy=-56mV, and VAD=-96mV, as shown in the attached oscilloscope captured images.  As you can see, the theory and the measurements are a very good match!

I know, I know:  Almost a perfect match from theoretical computations and experimental results mean nothing for team Lewin.  For everyone else, here is the evidence, once again, that KVL works perfectly for circuits under the influence of varying external magnetic fields and that Lewin did NOT get his calculations correct in his presentation because he ignored the induced voltages in the wires when he extracted the equivalent circuit.
Title: Re: #562 – Electroboom!
Post by: jesuscf on November 29, 2021, 12:21:51 am
Sure, now use your vast knowledge of electromagnetic fields and help Sredni calculate the voltage VAD from the problem above.

My "vast knowledge" won't help you. I asked the same questions long ago, got the right answers, but ended up like you: confused.

I can show you the door to your enlightenment, but I can't walk you through it. You'll have to do it yourself.

Here, go argue with results:

https://www.eevblog.com/forum/amphour/562-electroboom!/msg3842183/#msg3842183 (https://www.eevblog.com/forum/amphour/562-electroboom!/msg3842183/#msg3842183)
Title: Re: #562 – Electroboom!
Post by: Sredni on November 29, 2021, 01:00:18 am
It's like trying to explain color to someone who is blind since birth.

I have already calculated "VAD" twice: its value depends on the path.
On the path set by the first voltmeter I get -136mV; on the path set by the second voltmeter I get -56mV.
I can compute on any path I want, if I know how the dB/dt region is partitioned by the path itself. Do you have any idea how easy it is for me to compute it? The only difficulty is making sure I am following that spiral labyrinth you set up hoping to get us confused.

Now, I solved your challenge.
Will you solve a few very elementary problems I pose? Or will you evade my questions as Jesse Gordon is doing?

You have solve nothing! 
---snip---

It's a no, then?
You confirm will evade questions just like Jesse has done till now?

By the way: your challenge was
Quote
This is the challenge: Calculate V1, V2, Vx, and Vy.

I calculated them (it took me some six-seven minutes, mostly to decide to get off the couch and find paper and pencil, and to make sure I had copied the spirals right), and you said it yourself:

Quote
Excellent!  You got the correct results.

And now you are on your victory lap as if you were the only one to get the correct results. (And all this after I wrote, in the same message I posted the solution in: "Both 'teams' are doomed to find the same values for what is read by the voltmeters."
Go figure.

I wonder if 'everyone else reading' should be made aware that your solution forecasts that a copper wire a few inches long will drop 20 mV when a current of 0.24 mA flows through it. But hey, you have tiny little batteries in your wires, right? Like they explained coils in high school.

So, to be clear, you won't try to solve the simple circuit quizzes I pose?

You must be the kind of guy than never pays up when he loses a bet.
Title: Re: #562 – Electroboom!
Post by: jesuscf on November 29, 2021, 01:21:22 am
It's like trying to explain color to someone who is blind since birth.

I have already calculated "VAD" twice: its value depends on the path.
On the path set by the first voltmeter I get -136mV; on the path set by the second voltmeter I get -56mV.
I can compute on any path I want, if I know how the dB/dt region is partitioned by the path itself. Do you have any idea how easy it is for me to compute it? The only difficulty is making sure I am following that spiral labyrinth you set up hoping to get us confused.

Now, I solved your challenge.
Will you solve a few very elementary problems I pose? Or will you evade my questions as Jesse Gordon is doing?

You have solve nothing! 
---snip---

It's a no, then?
You confirm will evade questions just like Jesse has done till now?

By the way: your challenge was
Quote
This is the challenge: Calculate V1, V2, Vx, and Vy.

I calculated them (it took me some six-seven minutes, mostly to decide to get off the couch and find paper and pencil, and to make sure I had copied the spirals right), and you said it yourself:

Quote
Excellent!  You got the correct results.

And now you are on your victory lap as if you were the only one to get the correct results. (And all this after I wrote, in the same message I posted the solution in: "Both 'teams' are doomed to find the same values for what is read by the voltmeters."
Go figure.

I wonder if 'everyone else reading' should be made aware that your solution forecasts that a copper wire a few inches long will drop 20 mV when a current of 0.24 mA flows through it. But hey, you have tiny little batteries in your wires, right? Like they explained coils in high school.

So, to be clear, you won't try to solve the simple circuit quizzes I pose?

You must be the kind of guy than never pays up when he loses a bet.

Once again, what is the value of VAD?  Go argue with results:

https://www.eevblog.com/forum/amphour/562-electroboom!/msg3842183/#msg3842183 (https://www.eevblog.com/forum/amphour/562-electroboom!/msg3842183/#msg3842183)


Title: Re: #562 – Electroboom!
Post by: Sredni on November 29, 2021, 02:03:10 am
Once again, what is the value of VAD?

Once again:

It depends on the path.

How did place the probes to measure it? Midway through the disk, because you chose an highly symmetrical setup?
How would you place the probes if the solenoid generating the field was made in triangular shape and the three-turn ring with the resistors was shaped as Mickey Mouse's head silhouette, tilted and off-center?

Title: Re: #562 – Electroboom!
Post by: jesuscf on November 29, 2021, 02:15:22 am
Once again, what is the value of VAD?

Once again:

It depends on the path.

How did place the probes to measure it? Midway through the disk, because you chose an highly symmetrical setup?
How would you place the probes in the solenoid was made in triangular shape and the ring with the resistor shaped as Mickey Mouse's head silhouette, tilted and off-center?

Nope, the voltage VAD is ALWAYS -96mV.  I asked you to calculate it, not to measure it.  For this particular configuration of the problem, it is fairly easy to setup the probe so that the external magnetic field does not induce a voltage on it.  The calculation perfectly matches the measurement.  When Lewin did his experiment with a similar shape, he did not extract the correct equivalent circuit, and then he couldn't figure out what VAD was, so he blamed KVL.
Title: Re: #562 – Electroboom!
Post by: bsfeechannel on November 29, 2021, 03:09:32 am
You must be the kind of guy than never pays up when he loses a bet.

KVLers are pathetic, to say the least.
Title: Re: #562 – Electroboom!
Post by: bsfeechannel on November 29, 2021, 03:15:50 am
Once again, what is the value of VAD?  Go argue with results:

https://www.eevblog.com/forum/amphour/562-electroboom!/msg3842183/#msg3842183 (https://www.eevblog.com/forum/amphour/562-electroboom!/msg3842183/#msg3842183)

I don't argue with data rigged to support pseudo scientific claims.
Title: Re: #562 – Electroboom!
Post by: Sredni on November 29, 2021, 05:04:24 am
Once again, what is the value of VAD?
Once again:
It depends on the path.

How did place the probes to measure it? Midway through the disk, because you chose an highly symmetrical setup?
How would you place the probes in the solenoid was made in triangular shape and the ring with the resistor shaped as Mickey Mouse's head silhouette, tilted and off-center?
Nope, the voltage VAD is ALWAYS -96mV.  I asked you to calculate it, not to measure it. 
And I am repeating you, in size 18pt, 24pt, 36pt, that it depends on the path. So, in order for me to calculate it, I need to know the path (actually it suffice to know how it partition the flux region). Because voltage is a path integral and in the presence of variable magnetic field, per Maxwell equations, it does depend on the path. I know you don't like it and you want to consider the scalar electric potential difference, instead, but if we use different definitions, you have to accept we have to use different methods.

But the really funny thing is that if I gave you a strangely shaped coil and a strangely shaped circuit *you* would not be able to compute your version of voltage, nor to find the special path your probes should be positioned in order to measure it because all your computations so far are based on the assumptions of a constant induced Eind field that follows your coil's profile.
(Someone else, who could compute the fields and eventually simulate them numerical would, but you? I doubt it)

My method, on the other hand, only require for me to know how the path you want voltage to be computed on (because - it - is - a - path - integral - that - depends - on - the - path) partions the flux region. Half and half? Two-thirds, one thirds? Three-fifths, Two-fifths? Using the integral version can be very easy.

What I am trying to tell you, and I know you will misunderstand, is that you will be in breach of the rules set by McDonald for computing the difference in the electric scalar potential (what you call 'voltage' and McDonald calls 'voltage drop').

Quote
For this particular configuration of the problem, it is fairly easy to setup the probe so that the external magnetic field does not induce a voltage on it.  The calculation perfectly matches the measurement. 

And I don't know how many times I had to repeat this: both parties will get the same results for the measurements by actual voltmeters.

Quote
When Lewin did his experiment with a similar shape, he did not extract the correct equivalent circuit, and then he couldn't figure out what VAD was, so he blamed KVL.

Because Lewin uses the definition of voltage that is used by Purcell, Ramo Whinnery Van Duzer, Brandao Faria, Haus and Melcher, Rosser, Romer, Nicholson, and a ton of other textbooks and universities. Oh, yes, and by your voltmeters and oscilloscopes. That's what they measure.

When Mouser or Farnell or Arrow will start to sell voltmeter capable of measuring the McDonald voltage, we can talk again. For the time being, the Helmoltz decomposition for quasimagnetostatic problems is a nice mathematical trick but a definition of voltage that breaks Ohm's law and makes your students revert back to the  high school mindset where "nonconservative fields are not in the final" is just a bad idea.
Title: Re: #562 – Electroboom!
Post by: Sredni on November 29, 2021, 05:22:29 am
"What about the other forms of EMF, uh, what about 'em(f)?"

A common objection raised by KVLers is that Kirchhoff Voltage Laws was originally intended to include all forms of electromotive forces, including the nonlocalized inductive EMF. They mention the high school version of KVL that puts all EMFs on one side of the equation and all voltage drops on the other. From their point of view, considering the inductive EMF as a special case, would imply having a new law for every new kind of localized EMF in the loop. The problem is that the inductive EMF is a very special kind of EMF that is not like other forms of EMF.

If we go back to the original formulation by Kirchhoff in the 1845 Annalen der Physik und Chemie,

(https://i.postimg.cc/5tzVMxGx/screenshot-2.png)
link https://i.postimg.cc/5tzVMxGx/screenshot-2.png
https://books.google.de/books?id=Ig8t8yIz20UC&pg=PA494&hl=de&source=gbs_toc_r&cad=3#v=onepage&q&f=false (https://books.google.de/books?id=Ig8t8yIz20UC&pg=PA494&hl=de&source=gbs_toc_r&cad=3#v=onepage&q&f=false):

he indeed wrote (I am using R instead of omega to denote the resistance, and n instead of nu):

Quote
2) wenn die Drahte 1, 2, n, eine geschlossene Figur bilden

          R1 I1 + R2 I2 + ... + Rn In

= der Summe aller elektromotorischen krafte, die sich auf dem Wege: 1, 2, n befinden; wo R1, R2, Rn die Widerstande der Drahte, I1 I2,... die Intensitaten der Strome bezeichnen, von denen diese durchflossen werden, alle nach einer Richtung als positiv gerechnet.

Which translates into (bold mine)

Quote
2) if the wires 1, 2, n, form a closed figure

          R1 I1 + R2 I2 + ... + Rn In =

= the sum of all electromotive forces located on the path: 1, 2, ...n;
where R1, R2, ... Rn are the resistances of the wires, and I1 I2, ...In denote the intensities of the currents that flow through them, all calculated as positive in one direction.

Automatic translation is getting better and better every day, thanks to AI progress. Here is Google Translate's output for the original German text:

(https://i.postimg.cc/CLJMvxyX/screenshot-3.png)
link https://i.postimg.cc/CLJMvxyX/screenshot-3.png

The locution "located on the path: 1, 2, n" points to sources of EMF located on the path itself and I have found at least one source confirming that Kirchhoff formulated his rules by using localized, lumped, forms of EMF. In his Physics Education paper "Explaining electromagnetic induction: a critical re-examination. The clinical value of history in physics" (J Roche 1987 Phys. Educ. 22 91), science historian J. Roche writes:

Quote
"It was shown by G Kirchhoff (1824-87) in 1849 that localised EMFs generally set up auxiliary electrostatic forces, by means of surface charges, in order to establish a uniform current around the circuit (Kirchhoff 1879 pp49-55, 1514). Since these additional fields are conservative, the sum of the net potential differences around the whole circuit will be exactly equal to the sum of the PDs across the localised underlying EMFs only. This is the substance of Kirchhoff’s second network law (Kirchhoff 1879 pp 15-16)"


At the time the sources of EMF were galvanic cells, chemical batteries and thermopiles, like the state of the art electricity source used by Ohm to estabilish his law: a thermocouple based on the Seebeck effect (Ohm experimented in 1827, was basically discredited by Georg Pohl, and later 'rediscovered' in 1841, when the Royal Society (in London) awarded him the Copley medal in recognition of his accomplishments).

According to the above text, the right hand side of Kirchhoff's original KVL equation collects (with due sign) the contribution of all the lumped EMF sources that are present along the path (on the path, on the way, "auf dem Wege"). He even specifies "all electromotive forces located on the path 1, 2, ... n", the segments into which the path (the CLOSED FIGURE) has been partitioned.


Original KVL with modern terminology
Fast forward to the twentyfirst century, where we know something more about fields and we can even extend Kirchhoff's law to work in the presence of variable magnetic fields (but only when we can lump the inductive EMF along the path).

First, let's rewrite Kirchhoff's equation by making all EMFs located on the paths 1, 2, ... n explicit, by calling them emf1, emf2, ... emfn. If one branch has no localized EMF in it, then the corresponding emf will be zero; the same can be said for the resistances. In its expanded form, Kirchhoff's original KVL becomes:

          R1 I1 + R2 I2 + ... + Rn In = emf1 + emf2 + ... + emfn

where the emf1, emf2,... emfn are the localized electromotive forces located on the segments 1, 2, ..., n of the closed path.

Now we take the localized EMFs of each branch that are on the right hand side and we bring them on the left hand side. The change in sign reflects that different sign convention we use for generators and for passive elements.

            (R1 I1 - emf1) + (R2 I2 - emf2) + ... + (Rn In - emfn) = 0

This is the modern equivalent form of KVL as Kirchhoff formulated it.

In the twentyfirst century we recognize that each term in parenthesis represents the path integral of the total electric field along each of the segments 1, 2,... n, in which the closed path has been partitioned. Therefore, in modern terms, the original formulation of KVL says

             Sum of path integrals of Etot.dl along the branches of our closed figure = 0

The right hand side is ZERO because we have already accounted for all "der electromotorischen Krafte, die sich auf dem Wege" - all the electromotive forces on the branches 1, 2,... n - and because Kirchhoff did not consider any delocalised source of inductive EMF (basically, he was experimenting with batteries and thermopiles). Since the path integral along a closed path is the circulation, in modern notation the original form of KVL can also be written as:

             circulation of Etot.dl = 0            (this is KVL as formulated by Kirchhoff)
                                         (all localized EMF appear in the path integral)


Faraday's law introduces an EMF that is not on the path
Faraday discovered, in modern terms, that the circulation of Etot.dl can be nonzero, if there is a variable magnetic flux cut by the surface formed from the closed figure 1, 2, ... n Kirchhoff was arguing about. According to Faraday's law (not Kirchhoff's law), the circulation of the total electric field along the circuit path is equal to minus the time derivative of the flux of the magnetic field B cut by the surface delimited by said closed path:

           circulation of Etot.dl = -d/dt flux of B   ( this is Faraday slaying Kirchhoff)
                                 (introduces an EMF that does not fit into
                                   the path integral)

Note that the integral on the left still accounts for all der electromotorischen krafte --- I mean, all the localized electromotive forces you can imagine (batteries, solar cells, peltier cells, thermocouples...), but it does not include the nonlocalized inductive EMF due to the flux cut by your closed circuit. Therefore, we don't need a new law for every kind of non-inductive EMF out there: they already are all accounted for in the circulation integral on the left.
The term on the right, on the other hand (pun intended), is a new addition that breaks the original Kirchhoff's law and considerably extends our knowledge of the electromagnetic field. In its local, differential form, Faraday's law states that the electric field curls (i.e. ceases to be irrotational) in the presence of a time-changing magnetic field. This is a general property of the EM field, not just 'another kind of EMF'!

The breaking of Kirchhoff's loop rule is not something Lewin came up with on his own. It's part of standard classical electrodynamics and is commonly acknowledged in EM books for physicists and engineers, for example in Haus and Melcher, Ramo Whinney vanDuzer, Brandao Faria, Rosser...
In particular, a respected introductory textbook that makes this explicit is the second volume of Berkeley Physics: Electricity and Magnetism 3rd edition by Purcell and Morin:

(https://i.postimg.cc/kGXzD9jR/purcell-faraday-2.jpg)
link: https://i.postimg.cc/kGXzD9jR/purcell-faraday-2.jpg
Source: Purcell, Morin third edition, section 7.5

...Kirchhoff Loop rule is no longer applicable when there are variable magnetic fields inside the circuit's premises.


Extended KVL and its limits:
We can even push it a little further and accept in the path integral on the left even lumped inductive EMFs: it's what most textbooks (like Hayt, for example) call "extended KVL". It works when the source of EMF can be lumped and the changing magnetic flux can be hidden inside the component: the key is to alter the circuit path so as to exclude the variable magnetic region from it. Still, this trick cannot possibly work when the EMF is due to the changing flux linked by the circuit path itself (like in the example proposed by Romer and popularized by Lewin). That will kill KVL for good, and we are forced to consider the more general law: Faraday's Law.

(And no, trying to bring the delocalized inductive EMF contribution on the left side and distributing it along the branches won't work and I'll explain why in the post about the 'tiny batteries' model)

I already posted a solved example: Lewin's ring with a battery.
Next stop: the tiny battery model and how it related to the McDonald Manouver.
Title: Re: #562 – Electroboom!
Post by: HuronKing on November 30, 2021, 03:45:06 am
For whatever its worth Sredni and bsfeechannel, I'm grateful for your efforts to continually educate about this topic even if it really seems like the KVLers are just running in circles through the madness inducing Lewin/Romer loop (puns somewhat intended).

And here is something peculiar I noticed in ElectroBoom's video about KVL

Look at what he emphatically states is NOT 'his' definition of KVL (it genuinely pained me to see him put a big X through a time-invariant form of Maxwell's Equations),
https://youtu.be/d-a9Pr2z-qg?t=253 (https://youtu.be/d-a9Pr2z-qg?t=253)

Meanwhile, look at the Feynman Lectures he claimed in previous videos, supports his made up pseudoscientific definition, Section 22-3, Eq. 22.14

https://www.feynmanlectures.caltech.edu/II_22.html (https://www.feynmanlectures.caltech.edu/II_22.html)

Look at what the line integral of the E-field around a closed path is equal to: the sum of the voltages in the loop. When there are no time-varying magnetic fields, what does that sum equal to? Zero.

So, he's saying that the line integral of the E-field around a closed loop being zero is NOT his definition of KVL... but the line integral of the E-field around a closed loop IS the sum of the voltages around the loop... and if that line integral is zero and path independent, you have KVL.

So, uhh, what is he talking about? More made up science?

And this reminds me of something someone noted last time this came up in a big way. The KVL-Always-Holders have inconsistent explanations amongst one another for why these voltmeters connected to the same point read different values. But when one applies Faraday's Law, one of Maxwell's actual equations, there is no inconsistency in explanation and multiple types of phenomena are easy to explain and model and, yes, reduce to KVL-lumped expressions (as Feynman writes is possible under specific circumstances).
Title: Re: #562 – Electroboom!
Post by: Jesse Gordon on November 30, 2021, 04:22:30 am
... The KVL-Always-Holders ...

Uhh, I guess when you're done grabbing at straws you can sweep them up and make a straw man  :-DD

I don't know of any KVL-Always-Holders. There's clearly situations where the wavelength is smaller than the apparatus and KVL either would not hold or would be difficult or impossible to demonstrate as holding. There may also other possible cases where it is impossible to demonstrate KVL holding.
We don't say that KVL holds ALWAYS. That's an easy straw man to pick apart.

What I'm saying is the same thing that Physics Professor Dr. McDonald  said: "Lewin’s circuit is within the range of applicability of Kirchhoff’s loop equations, which
can be used to predict measurements by the “voltmeters” in the experiment."

The really odd thing is the Lewinites refusal to answer simple questions, like I ask below.

Referencing the diagram below, can you answer for me:

1: In a real life experiment, with the shown transformer and resistors and with volt meters at the locations indicated, with the volt meters all triggering their reading at the same time, will the voltages of all 5 volt meters sum to be zero? (Within the accuracy limitations of the meters of course....)

1b: If YES, then KVL at the very least holds. If you say KVL is not holding, explain why it is not holding even when/though it appears to be.

1c: if NO, then what do you think the reading will be and why? Because I will like to test a real world setup and see who's right.

2: Are all the volt meters connected at places which would provide an unambiguous reading of the voltage at the points indicated?

2b: If not, which volt meter would be ambiguous and why?

Basically, my tests show me that the voltages would sum up to zero, like KVL predicts. Snedri and bsfeechannel seem to indicate that KVL is not holding, even though it appears to be, (however they seem very reluctant to actually commit one way or the other on that....)

As far as I can tell, KVL would at the very least appear to hold as measured with a real volt meter, but for some reason, they think it's still not holding, but they seem unable to explain why it appears to be holding when they think it's not.

Is it a technicality? Now I see Snedri going over the original language, I wonder if he's trying to make some argument based on a technicality whereby which he can say that KVL doesn't hold even in cases where it gives every appearance of holding.

(https://i.postimg.cc/jdJntBXT/20211128-121506.jpg)

I look forward to your answer.

Thank you!
Title: Re: #562 – Electroboom!
Post by: HuronKing on November 30, 2021, 04:59:44 am
"I don't know of any KVL-Always-Holders."

You're not terribly well acquainted with the debate then. ElectroBoom (the guy who this thread is about) literally, and rather triumphantly, declares that KVL holds in all cases, his words (with accompanying fanfare music):
https://youtu.be/Q9LuVBfwvzA?t=838

As for me, I'm not terribly interested in saving KVL or going around in infinite loops with you (which is why I wanted to say my thanks to Sredni and bsfeechannel for their seemingly inexhaustible patience). Besides, KVL is not one of Maxwell's Equations anyway. Faraday's Law is.

On the flipside, no one here is a Lewinite (kind of gross phrasing anyway, as if this is a 'battle of personalities'... but that's what ElectroBoom wanted when he titled his first video "Disagreeing with a Master"). We're just people who have studied Maxwell's Equations. They were around before Lewin was even born, other people have described them well, and they'll be here long after us.
Title: Re: #562 – Electroboom!
Post by: Sredni on November 30, 2021, 05:45:23 am
And here is something peculiar I noticed in ElectroBoom's video about KVL

Look at what he emphatically states is NOT 'his' definition of KVL (it genuinely pained me to see him put a big X through a time-invariant form of Maxwell's Equations),
https://youtu.be/d-a9Pr2z-qg?t=253 (https://youtu.be/d-a9Pr2z-qg?t=253)

Meanwhile, look at the Feynman Lectures he claimed in previous videos, supports his made up pseudoscientific definition, Section 22-3, Eq. 22.14

https://www.feynmanlectures.caltech.edu/II_22.html (https://www.feynmanlectures.caltech.edu/II_22.html)

Look at what the line integral of the E-field around a closed path is equal to: the sum of the voltages in the loop. When there are no time-varying magnetic fields, what does that sum equal to? Zero.

So, he's saying that the line integral of the E-field around a closed loop being zero is NOT his definition of KVL... but the line integral of the E-field around a closed loop IS the sum of the voltages around the loop... and if that line integral is zero and path independent, you have KVL.

So, uhh, what is he talking about? More made up science?

One thing I have noticed with youtubers is they do not allow coherence or logic to get in the way of their videos. And after they reached a certain number of subscribers they can hardly admit their errors. At least Mehdi does not seem to try to defend its blunders in the comment section, like others do (sometimes to the point of banning critics). I am waiting to see if Dave will correct at least the description of his giant errors in the "Electricity misconceptions" video, or if he will let his followers listen to that nonsense about the Poynting vector only pointing outwards at AC and that fantasy connection with the skin effect (why not the proximity effect, then?). And it seems that misquoting Feynman is the ultimate signature of quackery in the EE field.

Quote
And this reminds me of something someone noted last time this came up in a big way. The KVL-Always-Holders have inconsistent explanations amongst one another for why these voltmeters connected to the same point read different values. But when one applies Faraday's Law, one of Maxwell's actual equations, there is no inconsistency in explanation and multiple types of phenomena are easy to explain and model and, yes, reduce to KVL-lumped expressions (as Feynman writes is possible under specific circumstances).

Yes. This and another point bsfeechannel made a few pages ago: you can see KVLers see the light ("IT'S A SAILBOAT!") and turn Faradians, but I have never seen a Faradian who understood Faraday's law ever turn KVLer. That tells a lot. I mean, if you understand Faraday you usually understand the Helmoltz decomposition and you can see what McDonald is doing, so you won't be a KVLer tout-court...

I also get something out of this. A selection of guinea pigs to test for holes in my exposition, required refinement in language (lumpABLE, lumpED, NOT lumped, UNlumpable...), and a stimulus to deepen my knowledge.
For example, I have now a very clear idea about how give a definition of voltage straight from electrostatics (from my 'silent post'), I overcame my 'fear' of partial (even straight, as paradoxical as it may seems) turns, and if need be I feel confident I could operate a 1821 multiplier with embedded bismuth-copper thermocouple (given enough ice and boiling water).
I am also interested in the mechanisms of rejection, cognitive dissonance, confirmation bias, and the like.
Unfortunately the only two KVLers left have lowered the level by several notches, by becoming obnoxiously repetitive...
Title: Re: #562 – Electroboom!
Post by: jesuscf on November 30, 2021, 06:23:06 am
I am also interested in the mechanisms of rejection, cognitive dissonance, confirmation bias, and the like.
Unfortunately the only two KVLers left have lowered the level by several notches, by becoming obnoxiously repetitive...

Wow, interesting!  Sredni have you figured out yet how to calculate the voltage between nodes A and D, VAD?  Yes, because you used that voltage to find Vx and Vy, but no, because it completely obliterates all the BS you have posted in this forum about VAD being 'path dependent'!  That is literally the definition of cognitive dissonance!!!   Here, check this link to find out how is properly done, a very simple excercise:

https://www.eevblog.com/forum/amphour/562-electroboom!/msg3842183/#msg3842183 (https://www.eevblog.com/forum/amphour/562-electroboom!/msg3842183/#msg3842183)

The answer is VAD=-96mV, ALWAYS!  KVL works perfectly!

So far what I have observed with team Lewin is: they don't know how to use KVL ergo KVL doesn't work!!!



Title: Re: #562 – Electroboom!
Post by: Sredni on November 30, 2021, 06:33:20 am
GOTO 500
Title: Re: #562 – Electroboom!
Post by: Jesse Gordon on November 30, 2021, 06:41:23 am
If you honestly answer the above questions and I'll do my best to draw up Lewin's loop and show how I would measure the voltage across the half turns.

Oh, no. Now you answer MY question, not a question of your choice.
And my question is:

This is Lewin's ring: two resistors in a single loop that goes around a circular region (let's consider it of the same size as the loop, so you can see there is no 'room to twist' the wires) of variable magnetic field. The resistors are required to be on the opposite sides of the variable magnetic field region.

(https://i.postimg.cc/kX0TSBw6/Lewin-ring-is-unlumpable.jpg)
https://i.postimg.cc/kX0TSBw6/Lewin-ring-is-unlumpable.jpg

Please, show everybody you can draw a circuit path (make it green, meaning it's 'flux-free') that joins the resistors' terminals to the "lumped transformer secondary" terminals and DOES NOT INCLUDE the variable magnetic field region in its interior. Like I did for the lumpABLE circuit I decided to see as lumpED (in my post "Lumpable (lumped and not lumped) and not lumpable circuits for dummies").

In addition, you can also show everybody you can draw the path inside your "lumped transformer secondary" that DOES INCLUDE the variable magnetic field region (make it orange) but IS NOT part of the green circuit path.
I will show you that if you can do that you will run into contradiction.
(https://i.postimg.cc/2SfmvWFM/20211128-113309.jpg)
OK here's how anybody can unambiguously physically measure the induced voltage in half of an air-core transformer secondary turn

Nice try, but you did not answer the question.
Let me repeat it for you here, in case you missed:

This is Lewin's ring: two resistors in a single loop that goes around a circular region (let's consider it of the same size as the loop, so you can see there is no 'room to twist' the wires) of variable magnetic field. The resistors are required to be on the opposite sides of the variable magnetic field region.

(https://i.postimg.cc/kX0TSBw6/Lewin-ring-is-unlumpable.jpg)
https://i.postimg.cc/kX0TSBw6/Lewin-ring-is-unlumpable.jpg

Please, show everybody you can draw a circuit path (make it green, meaning it's 'flux-free') that joins the resistors' terminals to the "lumped transformer secondary" terminals and DOES NOT INCLUDE the variable magnetic field region in its interior. Like I did for the lumpABLE circuit I decided to see as lumpED (in my post "Lumpable (lumped and not lumped) and not lumpable circuits for dummies").

In addition, you can also show everybody you can draw the path inside your "lumped transformer secondary" that DOES INCLUDE the variable magnetic field region (make it orange) but IS NOT part of the green circuit path.
I will show you that if you can do that you will run into contradiction.

Please, please, please, do not fly to another galaxy with your armchair before answering it.

I don't know what mushroom induced pipe dream you're envisioning and wanting me to draw up or what imaginary constructs you want me to use.

I can't read your mind. Remember, YOU"RE the Gyspie Fortune Teller with your Crystalball!

I did show how anybody can unambigously physically measure the voltage across a half-turn on Lewin's circuit.

That's my answer. If you don't like it, tough.

For the record, you didn't answer my questions either that were part of this deal. You beat around the bush and refused to actually answer my questions. What recourse do I have? Tough beans is all I got and that's all you got if you don't like my answer.

You answered questions of YOUR choosing, not the ones I asked.

The over arching issue here is that your own trusted source says that if the voltage on the output of an element can be unambiguously physically defined, then KVL will hold.
And I have demonstrated both on paper and in the lab how the voltage on a half-turn of Lewin's circuit can be unambiguously and physically defined and measured.

I agree with Dr. McDonald PhD, Princeton Physics Professor, when he said that Lewin’s circuit is within the range of applicability of Kirchhoff’s loop equations, which
can be used to predict measurements by the “voltmeters” in the experiment.

I also agree with Dr. Belcher PhD, MIT Physics Professor, when he warmly thanked Mehdi for his contributions, but said NOTHING of Lewin's -- and when he talked at length about Mehdi's "very nice experiments" but said NOTHING of Lewin, and ended up saying "KVL holds as argued by Mehdi..."



If you want to trade another pair of questions, then that's fine, but you need to provide the diagrams for your questions, like I provide the diagrams for mine.

If you do want to trade another pair of questions, here's my next one:


In the real-world test depicted in the below diagram, will Σ(V1,V2,V3,V4,V5) = 0? (Within the accuracy limits of the volt meters of course....)

(https://i.postimg.cc/jdJntBXT/20211128-121506.jpg)
https://i.postimg.cc/jdJntBXT/20211128-121506.jpg (https://i.postimg.cc/jdJntBXT/20211128-121506.jpg)
Title: Re: #562 – Electroboom!
Post by: Sredni on November 30, 2021, 06:52:39 am
Jesse, Jesse...
I did answer your question related to the previous circuit. I also pointed out that the answer was in the top right corner of my images for Lumped and Not lumped circuits. You seem to think that, because lumpABLE circuits exists and as such can be made lumpED by choosing a suitable circuit path, then there cannot be alternatives.

There are alternatives:

Alternative 1: the lumpABLE circuit itself. You choose another circuit path (for the same set of components and connecting wires!) that does include the variable magnetic region. Your system has now path dependent voltage and is NOT lumped.

Alternative 2: there are other circuits in the universe that are UNlumpable. Lewin's ring is one example.

And the fact that you cannot come up with an answer to my question is proof that you are not able to lump it. Don't feel bad. Nobody can lump that without incurring in self-contradiction. It's not your fault. It's the circuit that's drawn like that.



Now you try to shift the attention on yet another lumpABLE circuit, putting your voltmeters in a manner that imply you prefer to use a circuit path that makes it lumpED. Yes, as a lumpABLE circuit, it can be lumpED. But this does not imply that

1. it can also be seen as NOT lumped, by choosing a different circuit path (same components, same wires, same transformer!)
2. there are other circuits out there that are UNlumpable

I really cannot be more clear than this. Try some vitamins, maybe?

But here's the deal: if you put the numerical values of the voltages read by your voltmeters, I will show you the NOT lumped version of your circuit, so that you will avoid posting it again in the same way you are avoiding my question on how would you lump Lewin's ring.

Deal?

EDIT: grammar and clarification about what values I want you to fill in.
Time to sleep, perchance to die, for me.
Title: Re: #562 – Electroboom!
Post by: jesuscf on November 30, 2021, 06:57:50 am
GOTO 500

Can you put all the BS you wrote there in the form of a equivalent circuit and compute VAD?  Because proper calculations, using KVL show that VAD= -96mV.  And guess what, proper measurement show also VAD= -96mV!

Hey, a simpler circuit may be easier for you...  In Lewin's original circuit the one with just only one loop, with an EMF of 1V, 100 ohms resistor to the left, and 900 ohms to the right, do you remember it?  What is the value of VAD in that circuit?  Need help?  Here:

GOTO 392
Title: Re: #562 – Electroboom!
Post by: Jesse Gordon on November 30, 2021, 07:14:30 am
"I don't know of any KVL-Always-Holders."

You're not terribly well acquainted with the debate then.

The part of this debate I'm involved in has nothing in particular to do with Mehdi. We've been arguing for weeks here about whether KVL holds in Lewin's (and other simple) circuits, regardless of what Mehdi Said he.

Quote
ElectroBoom (the guy who this thread is about)

Sure, the thread has that title. But that's not why I'm here. Believe it or not, I did not know Mehdi was involved in this debate until after I got involved.

About a year ago I was happily watching Lewin's Lectures when he got to that part 16 and did his trick with the solenoid. I thought "What's he doing? That makes no sense." I left a comment to his video, and of course his fans started arguing with me, so I made this video: https://youtu.be/nAsZFP8Cfxk

I didn't even know Mehdi was involved in this until after I published my video and then people were like "Oh yeah, Mehdi showed that too" so that's how I found out.

Quote
literally, and rather triumphantly, declares that KVL holds in all cases, his words (with accompanying fanfare music):
https://youtu.be/Q9LuVBfwvzA?t=838

Literally, yes, but not in context. If you watch the whole video, he's talking about a variety of different cases which he demonstrates - AND - he runs his cases by Dr. Belcher, who writes up a nice writeup about a bunch of Mehdi's experiments, and then concludes that "KVL holds as argued by Mehdi."

So the CONTEXT was that of all the different example circuits that Mehdi gave to Belcher, Belcher said that KVL holds for all of them. Which is what Dr. Belcher said.
And Dr. Belcher even quoted Dr. Feynman.

Neither Belcher nor Mehdi are saying that KVL holds for every possible circuit of every possible configuration, but rather that it holds for all the ones they've just examined.

By taking Mehdi's words out of context, you make him out to say something completely different than he was actually communicating.

Your strong negative bias shines like the sparks off Mehdi's fingertips, and smells like his burning clip leads.  :-DD

Quote
As for me, I'm not terribly interested in saving KVL or going around in infinite loops with you (which is why I wanted to say my thanks to Sredni and bsfeechannel for their seemingly inexhaustible patience). Besides, KVL is not one of Maxwell's Equations anyway. Faraday's Law is.

And that's what I get far and wide. The people arguing for Lewin being right are dreadfully reluctant to actually show that they can do real physics. See how you side stepped my question?

They claim to know it all, but can demonstrate none of it, except they are sure about one thing, and that is Lewin was right.

Quote
On the flipside, no one here is a Lewinite (kind of gross phrasing anyway,
What would you call them? They call us "KVLer's" or "KVL-ALWAYS-HOLD"ers or whatever.

I know in your one-sided view  you're right any all who disagree with you are wrong.

But that cuts both ways. There are no KVLer's here, just people who understand physics. Anyone who disagrees is a Lewinite.

See? Either one of us can step outfrom behind a "title" and just put a title on the other side. Doesn't solve much though.

Point is both sides think they are right and the other is wrong.

Quote
as if this is a 'battle of personalities'... but that's what ElectroBoom wanted when he titled his first video "Disagreeing with a Master").

We're just people who have studied Maxwell's Equations. They were around before Lewin was even born, other people have described them well, and they'll be here long after us.


Yeah, there are no KVLer's here, we're just people who know physics!  :-DD :-DD :-DD

I know that air core transformers existed long before Lewin, but I think Lewin was just who really made the party trick well known - the combination of his entertainer style and youtube publicity was what it took to spark it off.

As for how I got here - someone posted my video here, and Thinkfat then went to tell me I was wrong in the comments of my video, and then linked me back to here.

So you like Anti-KVLer better than Lewinite? I don't particularly care. We say "Lewinite" because "Lewin is right" seems to be the only thing that they are absolutely sure of.

I have asked so many times the question I asked you, and like you, they don't mind arguing for weeks, but they don't want to take the time to answer that question.

At the end of the day, in the diagram I provided below, the voltages sum to zero. KVL does hold. Why they can't admit at least that the voltages sum to zero,  I don't know, it's as if they feel it would be reverent to Lewin to admit to it.

(https://i.postimg.cc/jdJntBXT/20211128-121506.jpg)
Title: Re: #562 – Electroboom!
Post by: jesuscf on November 30, 2021, 07:16:51 am
I did show how anybody can unambiguously physically measure the voltage across a half-turn on Lewin's circuit.

And a quarter turn or any fraction of a turn by that matter!  The blockheads in team Lewin don't want to accept that.
Title: Re: #562 – Electroboom!
Post by: Jesse Gordon on November 30, 2021, 07:39:21 am
Jesse, Jesse...
I did answer your question related to the previous circuit. I also pointed out that the answer was in the top right corner of my images for Lumped and Not lumped circuits. You seem to think that, because lumpABLE circuits exists and as such can be made lumpED by choosing a suitable circuit path, then there cannot be alternatives.

There are alternatives:

Alternative 1: the lumpABLE circuit itself. You choose another circuit path (for the same set of components and connecting wires!) that does include the variable magnetic region. Your system has now path dependent voltage and is NOT lumped.

Alternative 2: there are other circuits in the universe that are UNlumpable. Lewin's ring is one example.

But I'm asking a yes or no question, and you're saying everything other than yes or no.


Quote
And the fact that you cannot come up with an answer to my question is proof that you are not able to lump it. Don't feel bad. Nobody can lump that without incurring in self-contradiction. It's not your fault. It's the circuit that's drawn like that.

No, it means you're not explaining your question. You're asking me to draw something you're picturing in your mind. I did what I thought you wanted - I illustrated how to unambiguously physically measure the voltage across the half-turns in Lewin's experiment which would allow the loop to be modeled as 4 lumped elements consisting of two half turns in series with two resistors, and if that's not what you're asking then you need to get off the mushrooms and try explaining it more clearly, that, or draw your own diagrams.

Quote

Now you try to shift the attention on yet another lumpABLE circuit, putting your voltmeters in a manner that imply you prefer to use a circuit path that makes it lumpED. Yes, as a lumpABLE circuit, it can be lumpED. But this does not imply that

1. it can also be seen as NOT lumped, by choosing a different circuit path (same components, same wires, same transformer!)
2. there are other circuits out there that are UNlumpable

I really cannot be more clear than this. Try some vitamins, maybe?

My question is a simple yes or no question. Either the volt meters will sum to zero or they won't. Or maybe you think they might some days depending on the phase of the moon.

Quote
But here's the deal: if you put the numerical values of the voltages read by your voltmeters, I will show you the NOT lumped version of your circuit, so that you will avoid posting it again in the same way you are avoiding my question on how would you lump Lewin's ring.

Deal?

EDIT: grammar and clarification about what values I want you to fill in.
Time to sleep, perchance to die, for me.

I already showed you how I would lump Lewin's ring.

I don't understand the exact meaning of the deal you're offering. But I will not agree to refrain from posting something I think is true and on topic, and it looks like you might be asking that in your deal, so no deal. I do not give up any privilege to post what I think is the truth on the matter.

If you want me to stop posting a particular diagram, you need to convince me it's false.

Deal?

I got a better deal: How about you explain to me why you think that KVL is failing even though the volt meters say it's not in my diagram below.

I've literally asked that question to 4 Anti-KVLers on this thread and exactly zero of them have actually been willing to so much as agree that the volt meters will sum to zero, much less explain why they think KVL isn't holding.

It's pretty clear by your trusted source that since all 5 of my elements have their voltage unambiguously physically defined that KVL will hold.

(https://i.postimg.cc/jdJntBXT/20211128-121506.jpg)
https://i.postimg.cc/jdJntBXT/20211128-121506.jpg (https://i.postimg.cc/jdJntBXT/20211128-121506.jpg)

Title: Re: #562 – Electroboom!
Post by: Jesse Gordon on November 30, 2021, 08:03:58 am
brilliantly debunked by Lewin.

experimentally?

Theatrically!!!!  :-DD :-DD
Title: Re: #562 – Electroboom!
Post by: Jesse Gordon on November 30, 2021, 08:47:29 am
I did show how anybody can unambiguously physically measure the voltage across a half-turn on Lewin's circuit.

And a quarter turn or any fraction of a turn by that matter!  The blockheads in team Lewin don't want to accept that.

Exactly.

And whether you want to think of the fractional turn in terms of turns under Faraday's law, or as half of the dB/dt area, it all works out the same - that dB/dt area is the area which contributes to the induced voltage across that section of winding. The dB in that area not the dB outside the area.

Then they posted that textbook page which says that KVL holds if the voltage across the terminals can be unambiguously defined by physical measurements - and by George - we seem to be unambiguously defining the voltages by physical measurements, so I say it's time we all agreed that KVL holds for Lewin's circuit!  :-DD

This is getting fascinating.  How is it possible that they know enough to avoid answering numerous questions, but not enough to know that they look clueless?
The fact that they refuse to answer so many questions tells you they know their paradigm is not coherent with itself.

At the very least, How can they not realize that to us they look just as mislead as we look to them?

And what's with the idea that they can just say "There's no team Lewin, just team truth and you're wrong" - how's that any different than us saying "There's no team KVL, we just know the truth and you're wrong?" or "There's no two sides, we're right and you're wrong?" That's some deep thinking!



I can imagine there being an incredibly intelligent person knowing TONS of stuff which I don't understand.
But I cannot imagine that person using such flimsy fallacies as seem rampant on the part of Team Lewin.
Nor can I imagine such an incredibly intelligent person being afraid to answer simple yes/no questions.

When I'm in a disagreement like this, I am constantly aware that I could be wrong and I may just not understand enough to see that I'm wrong, so I look for clues to help me understand whether maybe the other side is right even though I don't understand. One of the things I look at is their willingness and ability to answer questions at a level I can understand. In this case, zilch. Another thing I look at is the quality of their arguments - and again, zilch. Saying things like "There aren't two sides, you're just wrong," or "They are all dead inside" is not a sign of a person who can take a step back and look at a topic from more than one angle.

Of course, they could be right by the luck of the draw but not be bright enough to explain it to me.

In that case, I'm right where I was before, with my experimental evidence which shows that KVL holds just fine in Lewin's circuit.

Heh, maybe I'm right by the luck of the draw. At least I can explain why I think the way I do, and it's because I measured. That's not a bad place to start. :-DD
Title: Re: #562 – Electroboom!
Post by: HuronKing on November 30, 2021, 08:54:17 am
Quote
The part of this debate I'm involved in has nothing in particular to do with Mehdi. We've been arguing for weeks here about whether KVL holds in Lewin's (and other simple) circuits, regardless of what Mehdi Said he.

Then what I said doesn't apply to you but you seem terribly offended by my having brought it up. Why?

Quote
Literally, yes, but not in context. If you watch the whole video, he's talking about a variety of different cases which he demonstrates - AND - he runs his cases by Dr. Belcher, who writes up a nice writeup about a bunch of Mehdi's experiments, and then concludes that "KVL holds as argued by Mehdi."

So the CONTEXT was that of all the different example circuits that Mehdi gave to Belcher, Belcher said that KVL holds for all of them. Which is what Dr. Belcher said.
And Dr. Belcher even quoted Dr. Feynman.

Srendi has been through your misattributions of Belcher's words. I'm not going to rehash it with you because, again, you want people to chase you in infinite loops. He already said it.
Yes, context matters. Mehdi takes Feynman and Belcher out of context. Mehdi made his positions clear in the first and second videos he released about this. He thinks KVL holds in all cases - he's still saying that. He's wrong.

Quote
Neither Belcher nor Mehdi are saying that KVL holds for every possible circuit of every possible configuration, but rather that it holds for all the ones they've just examined.

By taking Mehdi's words out of context, you make him out to say something completely different than he was actually communicating.

Belcher wouldn't say that because he is an actual physicist who understands Maxwell's Equations. Mehdi on the other hand... this is what he actually is arguing. This is the whole reason he picked a fight with Lewin in the first place.
https://youtu.be/0TTEFF0D8SA?t=51

And in his second video, he still thinks this is true. Now, I should really be clear about one thing - Mehdi thinks he's picking a fight with Lewin but really he picked a fight with physics and won't allow himself to think he could be wrong (he says as much in the EEVBlog podcast).

Quote
Your strong negative bias shines like the sparks off Mehdi's fingertips, and smells like his burning clip leads.

I own a Full-Bridge Rectifier T-shirt and used to be a subscriber. I unsubscribed when Mehdi started preaching pseudoscience.

Quote
And that's what I get far and wide. The people arguing for Lewin being right are dreadfully reluctant to actually show that they can do real physics. See how you side stepped my question?

They claim to know it all, but can demonstrate none of it, except they are sure about one thing, and that is Lewin was right.

I've read through 20+ pages of Sredni and bsfeechannel very patiently pointing to the contradictions in the assumption that KVL must hold all the time, even in the Romer/Lewin circuit. And I read through 50+ pages 2 years ago when Mehdi started this debacle on a large-scale.

Quote
What would you call them? They call us "KVLer's" or "KVL-ALWAYS-HOLD"ers or whatever.

Maxwellians. People like Sredni aren't suggesting that KVL must hold even when it doesn't. It's not one of Maxwell's Equations - it's not sacred. So why the desperation to save it? Why is it so very important that KVL applies to the Lewin/Romer Ring? Well I don't know why. This is why Sredni is fascinated by the cognitive bias - why does everyone want to save KVL so badly?

Quote
I know that air core transformers existed long before Lewin, but I think Lewin was just who really made the party trick well known - the combination of his entertainer style and youtube publicity was what it took to spark it off.

The experiment is an excellent demonstration of the phenomena of non-conservative fields and path dependence. Lewin didn't come up with it nor did he really come up with the interpretations of the physics that describe it. Sredni has cited multiple well-respected authors and texts (it's in Feynman, Purcell, and Romer for starters). I'll add another into the mix: JD Kraus Electromagnetics, Chapters 4-10 and 8-2. Not that this makes an appeal to authority - just that it's not accurate to characterize Lewin's interpretation of KVL and Faraday's Law as the ramblings of a mad MIT scientist crackpot (as much as Lewin fits that stereotype with the wild hair and colorful clothes)... it's actually pretty mainstream in physics and applied EM textbooks.

I'm pretty amazed that just studying Faraday's Law and saying "KVL doesn't always hold" makes one a Lewin cultist. And I'm not even an anti-KVLer... just someone who recognizes the limitations of when you can use it, I guess? To invoke an old joke, does saying that F =/= 0 in a Mechanical Dynamics course mean I don't think F = 0 in a Statics course? F = ma is Newton's 2nd Law... and we have special cases where F = 0. That's all.
Title: Re: #562 – Electroboom!
Post by: thinkfat on November 30, 2021, 09:08:03 am
Then they posted that textbook page which says that KVL holds if the voltage across the terminals can be unambiguously defined by physical measurements - and by George - we seem to be unambiguously defining the voltages by physical measurements, so I say it's time we all agreed that KVL holds for Lewin's circuit!  :-DD

Except the voltages are not unambiguous:
(https://www.eevblog.com/forum/amphour/562-electroboom!/?action=dlattach;attach=1331327)
Not in the circuit above (which is essentially the Lewin ring), not even in the EI core experiment that originally brought you here.
Above, the voltage across "2R" is either "-1/3V" or 2/3V, depending on the path your voltmeter probes are going. That's not what I call "unambiguous".
Title: Re: #562 – Electroboom!
Post by: Jesse Gordon on November 30, 2021, 09:11:13 am
Quote from: unknown
All the KVLer's are gone except two, and they are taken down about 12 notches, frantically just repeating themselves

I don't know that anyone's down 12 notches, but I do see a shift.

Quote from: bsfeechannel
Quote from: armandine2
Quote from: bsfeechannel
brilliantly debunked by Lewin.
experimentally?
Yes. Try it yourself.

Ironically, I had tried it myself a year prior with my Lewin Clock: https://youtu.be/nAsZFP8Cfxk

Quote from: bsfeechannel
Lewin showed at least one circuit where KVL doesn't hold. Everyone repeated his experiment and obtained the same result. Conclusion: KVL does NOT ALWAYS hold. End of story.

I repeated his experiment with proper probing and KVL did hold for me -- all the voltages summed to zero.

Quote from: bsfeechannel
He, and everyone who repeated the experiment, measured the voltages and they didn't add up to zero. That's how he showed it.

Yeah, He [Lewin] didn't add up all the voltages, so doh, of course it didn't add up to zero. When I added up all the voltages, then it did sum to zero.

Thinkfat also learned some cool stuff about voltage drop on a shorted winding and a number of other wrong predictions he made.

So yes, I've seen a change, but it's not been in team KVL it's been in team Lewin - and the change I see is their increasing refusal to answer questions which would show the incoherence in their claims.
Title: Re: #562 – Electroboom!
Post by: Jesse Gordon on November 30, 2021, 09:23:45 am
Then they posted that textbook page which says that KVL holds if the voltage across the terminals can be unambiguously defined by physical measurements - and by George - we seem to be unambiguously defining the voltages by physical measurements, so I say it's time we all agreed that KVL holds for Lewin's circuit!  :-DD

Except the voltages are not unambiguous:
(https://www.eevblog.com/forum/amphour/562-electroboom!/?action=dlattach;attach=1331327)
Not in the circuit above (which is essentially the Lewin ring), not even in the EI core experiment that originally brought you here.
Above, the voltage across "2R" is either "-1/3V" or 2/3V, depending on the path your voltmeter probes are going. That's not what I call "unambiguous".


AHHH THANK YOU! I've been asking for days what was ambiguous about my reading of the voltage on a toroidal transformer secondary!
The answer? If I don't know how many turns are on the winding, especially if they might change without me knowing it, then the reading is ambiguous.

Ha! Dude!

Even if you take resistors... or batteries... If the number of series resistors or batteries changes without you knowing it, then even the reading of a battery voltage is ambiguous!

In fact, if things which cannot change themselves suddenly must be considered to randomly change themselves, ALL OF PHYSICS FAILS!

And yes, lacing your volt meter lead through the core is adding turns.

Soooo, let's say we have terminals on our transformer secondary windings as shown in the diagram below, and the volt meters are connected WHERE SHOWN, and no volt meter leads are allowed to fall into that hungry transformer core, then there's no ambiguity,  right? Then KVL holds, right?

Remember, we're considering the transformer secondary a lumped element. a black box. No peeking inside. It's got two terminals and that's all we need to know. The voltage BETWEEN THOSE TWO TERMINALS IS UNAMBIGUOUS. That's all that matters. What's going on inside is off limits because it's a lumped element.

(https://i.postimg.cc/jdJntBXT/20211128-121506.jpg)

Seriously, keep your probes out of the lumped elements!

If you had pure battery and resistor loop, and you were stabbing your probes and random places inside the resistors or batteries, you'd get all sorts of ambiguous readings. That's why we LUMP the elements!
Title: Re: #562 – Electroboom!
Post by: thinkfat on November 30, 2021, 10:27:26 am
Then they posted that textbook page which says that KVL holds if the voltage across the terminals can be unambiguously defined by physical measurements - and by George - we seem to be unambiguously defining the voltages by physical measurements, so I say it's time we all agreed that KVL holds for Lewin's circuit!  :-DD

Except the voltages are not unambiguous:
(https://www.eevblog.com/forum/amphour/562-electroboom!/?action=dlattach;attach=1331327)
Not in the circuit above (which is essentially the Lewin ring), not even in the EI core experiment that originally brought you here.
Above, the voltage across "2R" is either "-1/3V" or 2/3V, depending on the path your voltmeter probes are going. That's not what I call "unambiguous".


AHHH THANK YOU! I've been asking for days what was ambiguous about my reading of the voltage on a toroidal transformer secondary!
The answer? If I don't know how many turns are on the winding, especially if they might change without me knowing it, then the reading is ambiguous.
I don't quite know what you're getting at, because I see only single-turn windings, so, would you tell me where you see "transformer secondary" windings in the above diagram?
Title: Re: #562 – Electroboom!
Post by: bsfeechannel on November 30, 2021, 03:53:13 pm
For whatever its worth Sredni and bsfeechannel, I'm grateful for your efforts to continually educate about this topic even if it really seems like the KVLers are just running in circles through the madness inducing Lewin/Romer loop (puns somewhat intended).

We’re on a mission from God.

(https://encrypted-tbn0.gstatic.com/images?q=tbn:ANd9GcQ8GU93rgGOUTXsNuZOHdmCs93x4DHCGBp_zw&usqp=CAU)
Title: Re: #562 – Electroboom!
Post by: jesuscf on November 30, 2021, 04:02:31 pm
The experiment is an excellent demonstration of the phenomena of non-conservative fields and path dependence. Lewin didn't come up with it nor did he really come up with the interpretations of the physics that describe it. Sredni has cited multiple well-respected authors and texts (it's in Feynman, Purcell, and Romer for starters). I'll add another into the mix: JD Kraus Electromagnetics, Chapters 4-10 and 8-2. Not that this makes an appeal to authority - just that it's not accurate to characterize Lewin's interpretation of KVL and Faraday's Law as the ramblings of a mad MIT scientist crackpot (as much as Lewin fits that stereotype with the wild hair and colorful clothes)... it's actually pretty mainstream in physics and applied EM textbooks.

The experiment is an excellent demonstration of Lewin having no idea of what he is talking about!  Do you even understand what 'non-conservative fields' mean in this context?  Have you realized that in the circuit, the one with the wire loop and the two resistors, all the energy induced due to the external varying magnetic field is equal to all the energy consumed by the resistors?  That sounds pretty conservative to me!
Title: Re: #562 – Electroboom!
Post by: HuronKing on November 30, 2021, 04:28:25 pm
The experiment is an excellent demonstration of the phenomena of non-conservative fields and path dependence. Lewin didn't come up with it nor did he really come up with the interpretations of the physics that describe it. Sredni has cited multiple well-respected authors and texts (it's in Feynman, Purcell, and Romer for starters). I'll add another into the mix: JD Kraus Electromagnetics, Chapters 4-10 and 8-2. Not that this makes an appeal to authority - just that it's not accurate to characterize Lewin's interpretation of KVL and Faraday's Law as the ramblings of a mad MIT scientist crackpot (as much as Lewin fits that stereotype with the wild hair and colorful clothes)... it's actually pretty mainstream in physics and applied EM textbooks.

The experiment is an excellent demonstration of Lewin having no idea of what he is talking about!  Do you even understand what 'non-conservative fields' mean in this context?  Have you realized that in the circuit, the one with the wire loop and the two resistors, all the energy induced due to the external varying magnetic field is equal to all the energy consumed by the resistors?  That sounds pretty conservative to me!

Oh dear...
https://farside.ph.utexas.edu/teaching/301/lectures/node59.html (https://farside.ph.utexas.edu/teaching/301/lectures/node59.html)

Path dependence of the line integral to do work is the defining characteristic of a non-conservative field.
https://courses.lumenlearning.com/suny-osuniversityphysics/chapter/8-2-conservative-and-non-conservative-forces/ (https://courses.lumenlearning.com/suny-osuniversityphysics/chapter/8-2-conservative-and-non-conservative-forces/)

We're not even talking about basic electromagnetics anymore (this is in Kraus Chapter 4). Sredni is right - the notches have been lowered back to basic Newtonian mechanics.

Another Lewin stooge apparently (the casual nature of his murdering of Kirchoff is astounding):
http://www.physicsbootcamp.org/Nonconservative-Electric-Field.html (http://www.physicsbootcamp.org/Nonconservative-Electric-Field.html)

Amazing.
Title: Re: #562 – Electroboom!
Post by: HuronKing on November 30, 2021, 04:37:02 pm
More Lewin stoogery I guess - the conspiracy runs deep! This is bigger than the Illuminati:
https://phys.libretexts.org/Bookshelves/Electricity_and_Magnetism/Book%3A_Electricity_and_Magnetism_(Tatum)/09%3A_Magnetic_Potential/9.01%3A_Introduction_to_Magnetic_Potential (https://phys.libretexts.org/Bookshelves/Electricity_and_Magnetism/Book%3A_Electricity_and_Magnetism_(Tatum)/09%3A_Magnetic_Potential/9.01%3A_Introduction_to_Magnetic_Potential)

"The force on a charge  q  in a magnetic field is  qv×B . This force (the Lorentz force) does not depend only on the position of the particle, but also on its velocity (speed and direction). Thus the force is not conservative. This suggests that perhaps we cannot express the magnetic field merely as the gradient of a scalar potential function – and this is correct; we cannot."

Gawd, they're everywhere!
http://www.sfu.ca/phys/121/1101/lectures/lecture35.pdf (http://www.sfu.ca/phys/121/1101/lectures/lecture35.pdf)

Title: Re: #562 – Electroboom!
Post by: Sredni on November 30, 2021, 04:46:58 pm
Jesse, Jesse...
I did answer your question related to the previous circuit. I also pointed out that the answer was in the top right corner of my images for Lumped and Not lumped circuits. You seem to think that, because lumpABLE circuits exists and as such can be made lumpED by choosing a suitable circuit path, then there cannot be alternatives.
There are alternatives:

Alternative 1: the lumpABLE circuit itself. You choose another circuit path (for the same set of components and connecting wires!) that does include the variable magnetic region. Your system has now path dependent voltage and is NOT lumped.
Alternative 2: there are other circuits in the universe that are UNlumpable. Lewin's ring is one example.
But I'm asking a yes or no question, and you're saying everything other than yes or no.

Here's the picture I was talking about:

(https://i.postimg.cc/Yq1ZDcPY/KVL-works-if-I-leave-out-the-magnetic-region.jpg)

It's literally titled: "KVL-works-if-I-leave-out-the-magnetic-region.jpg"
Does the ".jpg" confuse you? Or is it the dashes?
If you look at the top right corner, it says: "KVL works here".
What part of "KVL works here" you do not understand? "KVL"? "works"? "here"?

I hope it is clear that if you choose to look at a lumpABLE circuit as a lumpED circuit, then KVL works.
The problem is that, even if KVL works there, it does not mean that KVL works for all other circuits in magnetoquasistatic conditions. I just need one example to prove it doesn't. And Lewin's ring is such an example.

You know, the circuit for which you are not able to define a circuit path that does not contain the variable magnetic region.

I am pretty sure that one of your many problems is that you don't understand the difference between the physical system (usually called "the circuit") and the actual circuit path. The one I keep talking about... you know when talking about lumped circutis with inductors I say "instead of following the coil filament we jump at the terminals" or something like that?

Quote
Quote
And the fact that you cannot come up with an answer to my question is proof that you are not able to lump it. Don't feel bad. Nobody can lump that without incurring in self-contradiction. It's not your fault. It's the circuit that's drawn like that.
No, it means you're not explaining your question. You're asking me to draw something you're picturing in your mind. I did what I thought you wanted - I illustrated how to unambiguously physically measure the voltage across the half-turns in Lewin's experiment

So, let's see.
I ask you to draw in green the circuit path that, if the circuit is lumpable, must not contain the shaded dB/dt region, and you draw the path you have to put the probes to measure something I did not even mentioned in my question?
Nice try.

Wanna try again?

Quote
Quote

Now you try to shift the attention on yet another lumpABLE circuit, putting your voltmeters in a manner that imply you prefer to use a circuit path that makes it lumpED. Yes, as a lumpABLE circuit, it can be lumpED. But this does not imply that

1. it can also be seen as NOT lumped, by choosing a different circuit path (same components, same wires, same transformer!)
2. there are other circuits out there that are UNlumpable

I really cannot be more clear than this. Try some vitamins, maybe?
My question is a simple yes or no question. Either the volt meters will sum to zero or they won't. Or maybe you think they might some days depending on the phase of the moon.

If you place your voltmeters all in the same simply connected region that does not include the variable dB/dt region, then yes of course KVL will work. It's written in the top right corner of the picture I have posted more then a week ago. You still have to understand that?

Quote
Quote
But here's the deal: if you put the numerical values of the voltages read by your voltmeters, I will show you the NOT lumped version of your circuit, so that you will avoid posting it again in the same way you are avoiding my question on how would you lump Lewin's ring.

I already showed you how I would lump Lewin's ring.


No you didn't. To show that you can lump it you need to show the circuit path that connects the resistors terminals to the lumped magnetic component(s) - one full turn? two half turns? four quarter turns? - and does not contain at its interior the shaded variable magnetic region. Can you?
So far you only proved you can't.

Title: Re: #562 – Electroboom!
Post by: thinkfat on November 30, 2021, 04:57:31 pm
The experiment is an excellent demonstration of the phenomena of non-conservative fields and path dependence. Lewin didn't come up with it nor did he really come up with the interpretations of the physics that describe it. Sredni has cited multiple well-respected authors and texts (it's in Feynman, Purcell, and Romer for starters). I'll add another into the mix: JD Kraus Electromagnetics, Chapters 4-10 and 8-2. Not that this makes an appeal to authority - just that it's not accurate to characterize Lewin's interpretation of KVL and Faraday's Law as the ramblings of a mad MIT scientist crackpot (as much as Lewin fits that stereotype with the wild hair and colorful clothes)... it's actually pretty mainstream in physics and applied EM textbooks.

The experiment is an excellent demonstration of Lewin having no idea of what he is talking about!  Do you even understand what 'non-conservative fields' mean in this context?  Have you realized that in the circuit, the one with the wire loop and the two resistors, all the energy induced due to the external varying magnetic field is equal to all the energy consumed by the resistors?  That sounds pretty conservative to me!

The very fact that there is energy available for the resistors to consume, in a closed loop, shows that the field must be non-conservative. Because in a conservative field, all paths going back to their origin end on the same potential and there is no energy available.

In anticipation of your next argument: it is not the magnetic field that is being talked about here. The non-conservative field is the electric field that results from the changing magnetic flux.

Think of it as a top-down 2D projection of a spiral staircase. The XY plane is the "geometric" plane in which your path lies, the Z axis is the electric potential. The path is closed in the XY projection, but start and end of it have different Z coordinates and each "turn" in the XY plane lifts your Z coordinate by "-dB/dt".
Title: Re: #562 – Electroboom!
Post by: jesuscf on November 30, 2021, 05:33:40 pm
The experiment is an excellent demonstration of the phenomena of non-conservative fields and path dependence. Lewin didn't come up with it nor did he really come up with the interpretations of the physics that describe it. Sredni has cited multiple well-respected authors and texts (it's in Feynman, Purcell, and Romer for starters). I'll add another into the mix: JD Kraus Electromagnetics, Chapters 4-10 and 8-2. Not that this makes an appeal to authority - just that it's not accurate to characterize Lewin's interpretation of KVL and Faraday's Law as the ramblings of a mad MIT scientist crackpot (as much as Lewin fits that stereotype with the wild hair and colorful clothes)... it's actually pretty mainstream in physics and applied EM textbooks.

The experiment is an excellent demonstration of Lewin having no idea of what he is talking about!  Do you even understand what 'non-conservative fields' mean in this context?  Have you realized that in the circuit, the one with the wire loop and the two resistors, all the energy induced due to the external varying magnetic field is equal to all the energy consumed by the resistors?  That sounds pretty conservative to me!

Oh dear...
https://farside.ph.utexas.edu/teaching/301/lectures/node59.html (https://farside.ph.utexas.edu/teaching/301/lectures/node59.html)

Path dependence of the line integral to do work is the defining characteristic of a non-conservative field.
https://courses.lumenlearning.com/suny-osuniversityphysics/chapter/8-2-conservative-and-non-conservative-forces/ (https://courses.lumenlearning.com/suny-osuniversityphysics/chapter/8-2-conservative-and-non-conservative-forces/)

We're not even talking about basic electromagnetics anymore (this is in Kraus Chapter 4). Sredni is right - the notches have been lowered back to basic Newtonian mechanics.

Another Lewin stooge apparently (the casual nature of his murdering of Kirchoff is astounding):
http://www.physicsbootcamp.org/Nonconservative-Electric-Field.html (http://www.physicsbootcamp.org/Nonconservative-Electric-Field.html)

Amazing.

Here, read this carefully, it is neither too long nor too difficult, because we are talking about a very simple circuit:

https://www.eevblog.com/forum/amphour/562-electroboom!/msg3828206/#msg3828206 (https://www.eevblog.com/forum/amphour/562-electroboom!/msg3828206/#msg3828206)

Everything else said to justify Lewin's incorrect results is BS.
Title: Re: #562 – Electroboom!
Post by: jesuscf on November 30, 2021, 06:02:49 pm
The experiment is an excellent demonstration of the phenomena of non-conservative fields and path dependence. Lewin didn't come up with it nor did he really come up with the interpretations of the physics that describe it. Sredni has cited multiple well-respected authors and texts (it's in Feynman, Purcell, and Romer for starters). I'll add another into the mix: JD Kraus Electromagnetics, Chapters 4-10 and 8-2. Not that this makes an appeal to authority - just that it's not accurate to characterize Lewin's interpretation of KVL and Faraday's Law as the ramblings of a mad MIT scientist crackpot (as much as Lewin fits that stereotype with the wild hair and colorful clothes)... it's actually pretty mainstream in physics and applied EM textbooks.

The experiment is an excellent demonstration of Lewin having no idea of what he is talking about!  Do you even understand what 'non-conservative fields' mean in this context?  Have you realized that in the circuit, the one with the wire loop and the two resistors, all the energy induced due to the external varying magnetic field is equal to all the energy consumed by the resistors?  That sounds pretty conservative to me!

The very fact that there is energy available for the resistors to consume, in a closed loop, shows that the field must be non-conservative. Because in a conservative field, all paths going back to their origin end on the same potential and there is no energy available.

In anticipation of your next argument: it is not the magnetic field that is being talked about here. The non-conservative field is the electric field that results from the changing magnetic flux.

Think of it as a top-down 2D projection of a spiral staircase. The XY plane is the "geometric" plane in which your path lies, the Z axis is the electric potential. The path is closed in the XY projection, but start and end of it have different Z coordinates and each "turn" in the XY plane lifts your Z coordinate by "-dB/dt".

And... you don't understand what 'conservative' means when taking about circuits.  It is the energy that is conservative!  In Lewin's circuit, what is the instantaneous power induced in the circuit by means of the external magnetic field?  Now, what is the instantaneous power dissipated by the resistors?  Come on, is not a hard calculation!  If the two are equal, then the fields (plural) are conservative.

Title: Re: #562 – Electroboom!
Post by: thinkfat on November 30, 2021, 06:13:55 pm
What you are talking about is conservation of energy. Not conservative or non-conservative fields.
Title: Re: #562 – Electroboom!
Post by: bsfeechannel on November 30, 2021, 06:32:33 pm
One thing I have noticed with youtubers is they do not allow coherence or logic to get in the way of their videos. And after they reached a certain number of subscribers they can hardly admit their errors.

They make the unforgivable mistake of believing their own publicity.

Quote
At least Mehdi does not seem to try to defend its blunders in the comment section, like others do (sometimes to the point of banning critics).

That’s a relief, but Mehdi has the bad habit of throwing his audience against, especially, but no limited to, physicists who directly or indirectly contradict his claims. His intention is clearly to gain credibility at the expense of truth. He tried to pull this trick against the two physicists who tried to explain the chain fountain effect. That backfired because Steve Mould, who has a comparable audience to Mehdi’s, intervened and went to great efforts to disprove Mehdi’s insistent false claims.

You can say that the debate with Steve was beneficial, but although the means can be said justified in this case, we cannot say Mehdi’s ends are always benevolent.

Quote
I am waiting to see if Dave will correct at least the description of his giant errors in the "Electricity misconceptions" video, or if he will let his followers listen to that nonsense about the Poynting vector only pointing outwards at AC and that fantasy connection with the skin effect (why not the proximity effect, then?). And it seems that misquoting Feynman is the ultimate signature of quackery in the EE field.

Along all these years in his channel, forum and twitter account, Dave has made quite a bunch of questionable and cringe-worthy claims, not only limited to electronics. However, in his favor he has this talk below early on in his long series of videos.

EEVBlog #22 (@6:39):

Quote
I got a rather interesting comment once, in fact I've had more than once in various forms, but the comment was basically: how do we know you're RIGHT? How  can we take your word for it? On, you know, all these things and all these topics? And, well, you know, it's a really good question, and the answer is you SHOULDN'T. You should never take anyone's word for it. Don't take anything i say on these blogs as gospel. Uh, you know, I've been in the industry for 20 years so, you know, I like to think I do know what I'm talking about mostly. But, you know, don't take my word for it. All my blogs, and all the things I talk about on here are designed to be food for thought. You're supposed to use your own engineering judgment and, you know, and go out and verify things. If you're, you know, if you're really interested in something, don't complain that i didn't explain it right or and you know i might have got it a bit wrong or something like that. Go out and investigate for yourself. That's what it's all about: food for thought.

This speech is what saves Dave. Mehdi, on the other hand, likes to claim he is “right” and the rest is wrong.

Quote
I also get something out of this. A selection of guinea pigs to test for holes in my exposition, required refinement in language (lumpABLE, lumpED, NOT lumped, UNlumpable...), and a stimulus to deepen my knowledge.
For example, I have now a very clear idea about how give a definition of voltage straight from electrostatics (from my 'silent post'), I overcame my 'fear' of partial (even straight, as paradoxical as it may seems) turns, and if need be I feel confident I could operate a 1821 multiplier with embedded bismuth-copper thermocouple (given enough ice and boiling water).
I am also interested in the mechanisms of rejection, cognitive dissonance, confirmation bias, and the like.
Unfortunately the only two KVLers left have lowered the level by several notches, by becoming obnoxiously repetitive...

On the KVLer’s head is where we hone our swords.
Title: Re: #562 – Electroboom!
Post by: Jesse Gordon on November 30, 2021, 08:11:25 pm
Quote
Literally, yes, but not in context. If you watch the whole video, he's talking about a variety of different cases which he demonstrates - AND - he runs his cases by Dr. Belcher, who writes up a nice writeup about a bunch of Mehdi's experiments, and then concludes that "KVL holds as argued by Mehdi."

So the CONTEXT was that of all the different example circuits that Mehdi gave to Belcher, Belcher said that KVL holds for all of them. Which is what Dr. Belcher said.
And Dr. Belcher even quoted Dr. Feynman.

Srendi has been through your misattributions of Belcher's words.

I realize that Srendi is very displeased with the fact that Dr. McDonald said "Lewin's circuit is within the range of applicability of Kirchhoff's voltage equations" and that Dr. Belcher gave "Too broad (as snedlie puts it)" of an approval.

The undeniable fact is that in his 16 pages of notes on the topic titled "Kirchhoff's Voltage Law (KVL) and Faraday's Law: ElectroBOOM's Experiments," Dr. Belcher starts out with warm gratitude to Mehdi and compliments his "Nice experiments," then goes over several different experiments that Mehdi did,  then Dr. Belcher cites Dr. Feynman and concludes "Thus with Feynman's definition, KVL Holds," and "KVL holds, as argued by Mehdi...."

And Dr. Belcher makes not ONE mention of Lewin or his experiments.

I realize that  Master Srendi has a crystalball and knows what Belcher meant even though it's not what he said, but I don't particularly trust Srendi's crystalball, even though he brags of it often.


Quote
I'm not going to rehash it with you because, again, you want people to chase you in infinite loops. He already said it.
Yes, context matters. Mehdi takes Feynman and Belcher out of context. Mehdi made his positions clear in the first and second videos he released about this. He thinks KVL holds in all cases - he's still saying that. He's wrong.

Quote
Neither Belcher nor Mehdi are saying that KVL holds for every possible circuit of every possible configuration, but rather that it holds for all the ones they've just examined.

By taking Mehdi's words out of context, you make him out to say something completely different than he was actually communicating.

Belcher wouldn't say that because he is an actual physicist who understands Maxwell's Equations. Mehdi on the other hand... this is what he actually is arguing. This is the whole reason he picked a fight with Lewin in the first place.
https://youtu.be/0TTEFF0D8SA?t=51

What Mehdi actually says at that point you link above is "Dr. Lewin teaches in one of his courses that KVL doesn't hold true in some cases and I disagree with him."

From the CONTEXT of watching the whole video, it is clear that Mehdi is not claiming that KVL holds in all possible electrical typologies and frequencies, but rather he is specifically talking about certain cases that Lewin and he experimented with.

But yeah, if you want to quote mine, find videos of people who aren't even speaking their native language, I bet you can find slight ambiguities in their out-of-context words which you can present out of context to mean something else.


Quote
So why the desperation to save it? Why is it so very important that KVL applies to the Lewin/Romer Ring? Well I don't know why. This is why Sredni is fascinated by the cognitive bias - why does everyone want to save KVL so badly?

Perhaps Team KVL is just as fascinated by Sredni's cognitive bias.

It's one thing if he wants to argue that Lewin's specific setup is difficult to probe and that as a result KVL may not appear to hold.

But Team Lewin literally says that KVL doesn't hold even when using the 2-terminal secondary winding on a toroidal transformer as an element, which obviously would work, and the textbooks say it would work with KVL.

So I ask you, why is Team Lewin so set on saying that KVL won't hold even for a loop which has as an element the two terminal output winding on a toroidal transformer?
Why is it so important?

Quote
I'm pretty amazed that just studying Faraday's Law and saying "KVL doesn't always hold" makes one a Lewin cultist. And I'm not even an anti-KVLer... just someone who recognizes the limitations of when you can use it, I guess?

As far as I know, everyone one Team KVL understands that there are circumstances where KVL doesn't hold - including, for example, frequencies which are smaller than the elements. That's just a straw man you're battering to bits with such glee.

What makes someone an anti-KVL cultist is when they argue that it doesn't hold even for cases where it does hold.

But if you simply understand Faraday's law and say that KVL doesn't always hold, then you should have no problem agreeing that KVL holds as measured by real volt meters in a real lab test, as depicted in the following diagram:

(https://i.postimg.cc/jdJntBXT/20211128-121506.jpg)

Why not admit that the volt meters would all sum to zero and KVL would at least appear to hold in the above diagram?

Oh that's right, you don't want to go around in circles. Except you are going around in circles about the topic, you just don't want to actually answer a real physics question.
Title: Re: #562 – Electroboom!
Post by: Jesse Gordon on November 30, 2021, 08:35:27 pm
Then they posted that textbook page which says that KVL holds if the voltage across the terminals can be unambiguously defined by physical measurements - and by George - we seem to be unambiguously defining the voltages by physical measurements, so I say it's time we all agreed that KVL holds for Lewin's circuit!  :-DD

Except the voltages are not unambiguous:
(https://www.eevblog.com/forum/amphour/562-electroboom!/?action=dlattach;attach=1331327)
Not in the circuit above (which is essentially the Lewin ring), not even in the EI core experiment that originally brought you here.
Above, the voltage across "2R" is either "-1/3V" or 2/3V, depending on the path your voltmeter probes are going. That's not what I call "unambiguous".


AHHH THANK YOU! I've been asking for days what was ambiguous about my reading of the voltage on a toroidal transformer secondary!
The answer? If I don't know how many turns are on the winding, especially if they might change without me knowing it, then the reading is ambiguous.
I don't quite know what you're getting at, because I see only single-turn windings, so, would you tell me where you see "transformer secondary" windings in the above diagram?

You've added another secondary winding path. The difference between 2/3v and -1/3v paths is that they wrap around the dB/dt area a different number of times, the difference is one turn.

You're literally asking me to run my volt meter through the transformer, effecting another secondary winding (which you won't allow me to document or model) and you're presenting that as proof of ambiguity.

But the same BS would work on battery packs:

You could just as well say that measuring a 9V battery pack was ambiguous because the volt meter probe could be put inside the battery and connected to some cell tap other than the end terminal.

NO! Just NO!

If we model an element as a black box with two terminals, then you're not allowed to poke your volt meter leads around inside.  That's why we have lumped elements.

The fact that a volt meter probe can be stuck inside a previously lumped element to get a different reading does not mean that reading the voltage on the actual terminals is ambiguous.

Of course moving the probes to another part of the circuit changes the reading.

But that doesn't mean that the reading AT THE TERMINALS is ambiguous.

[EDIT: I updated the diagram below to show designators]

See my diagram below? See how it has terminals AB drawn for the transformer? That's where we're allowed to probe. It's a lumped element. You can't go poking inside it any more than you can measure the voltage across a resistor by probing at some random location along it's physical length.

(https://i.postimg.cc/bNr4Wmwk/20211130-124903.jpg)

Do you really think that your ability to move your probes to any of points A,B,C,D,E,F invalidates the reading at AB? Of course AD or DE or AF may be different than AB, but that doesn't make AB ambiguous!

Do you really believe that any measurement is ambiguous because you could probe different locations and get different readings?

Of course measuring between AF like you want will give different reading than between AB. But that doesn't mean AB is ambiguous.

And here's something interesting. You obviously know that the volt meters in the diagram directly above sum to zero.

And yet you can't bring yourself to say "Hey you're right, KVL does appear to hold in that configuration." Why? It obviously appears to hold.

Do you need me to make a video demonstrating it?

Once you come to grips with reality, then you can explain why you think it's not actually holding even though it gives every appearance of holding.

I thought maybe you'd argue based on some definitions or something. But your best argument seems to be saying that reading AB is ambiguous because you could get a different value by reading AF.
Title: Re: #562 – Electroboom!
Post by: thinkfat on November 30, 2021, 09:06:08 pm
WHERE are you seeing "transformer secondaries"? You're evading answering again.
Title: Re: #562 – Electroboom!
Post by: HuronKing on November 30, 2021, 09:18:50 pm
And... you don't understand what 'conservative' means when taking about circuits.  It is the energy that is conservative!  In Lewin's circuit, what is the instantaneous power induced in the circuit by means of the external magnetic field?  Now, what is the instantaneous power dissipated by the resistors?  Come on, is not a hard calculation!  If the two are equal, then the fields (plural) are conservative.

Emphasis mine. (Sredni, are you seeing this?!)
Oh my goodness. You think that because energy is conserved that fields must also be conserved? This is just - I don't even know what to say.  :-\

Oh dear oh dear oh dear:
https://courses.lumenlearning.com/physics/chapter/7-5-nonconservative-forces/ (https://courses.lumenlearning.com/physics/chapter/7-5-nonconservative-forces/)

It's amazing that you heard me say "the Lewin circuit demonstrates path dependence of non-conservative fields" and INSTANTLY thought this must be about conservation of energy. They're... not the same. You think conserved fields and conserved energy are descriptions of the same thing?  :(

So yea, I guess you should review Newtonian mechanics before we deal with things like circulating magnetic fields:
https://www.pearson.com/content/dam/one-dot-com/one-dot-com/us/en/higher-ed/en/products-services/course-products/wolfson-3e-info/pdf/sample-chapter--ch07.pdf (https://www.pearson.com/content/dam/one-dot-com/one-dot-com/us/en/higher-ed/en/products-services/course-products/wolfson-3e-info/pdf/sample-chapter--ch07.pdf)

But I don't even know why I'm linking things for you - you're clearly not opening and reading them.
And I read your post with the circuit - Sredni asked you questions about it that you never answered.

Title: Re: #562 – Electroboom!
Post by: Jesse Gordon on November 30, 2021, 09:28:15 pm
(https://i.postimg.cc/jdJntBXT/20211128-121506.jpg)
https://i.postimg.cc/jdJntBXT/20211128-121506.jpg (https://i.postimg.cc/jdJntBXT/20211128-121506.jpg)

My question is a simple yes or no question. Either the volt meters will sum to zero or they won't. Or maybe you think they might some days depending on the phase of the moon.
If you place your voltmeters all in the same simply connected region that does not include the variable dB/dt region, then yes of course KVL will work. It's written in the top right corner of the picture I have posted more then a week ago. You still have to understand that?

And you still absolutely refuse to answer a simple yes/sometimes/no question regarding the above diagram.

Quote
So far you only proved you can't.

Speak for yourself, it describes you perfectly.

Well, I guess if you can't even answer a simple yes/sometimes/no question, then it's no surprise you can't wrap your mind around Lewin's loop.

Seriously, what's your best guess about the volt meters in the above diagram? Do you think they would sum to zero? or not? or just sometimes?

Maybe it would help you to visualize real world values. How about 100mV per turn induced emf in the transformer core, and  each resistor is 50 ohms.

Dude, lighten up and just answer the question so we can move on. Say "Yes, No, or Sometimes" and there's always the ever popular "I don't know" if that is your case, but if you don't know then you really don't know do you.

If this was chess, you're acting like you're in check. You're acting like you don't want to make a move because you know it'll put you in a bind you can't get out of.

I'm not asking you to try and follow some twisted mental gymnastics and draw a secret diagram I have in my mind.

I drew a simple real-world diagram which you can test yourself with a real transformer and real volt meter and resistors if you want, or if you already know the answer you can just give the answer.

If you can't do that, there's no way the other stuff you're asking is ever going to make sense.

Will the volt meters sum to zero in the above diagram in a real world test? Yes, Sometimes, No.

Thank you!
Title: Re: #562 – Electroboom!
Post by: Jesse Gordon on November 30, 2021, 09:36:09 pm
WHERE are you seeing "transformer secondaries"? You're evading answering again.

I edited that post to show the extra secondary in my diagram, also updated the text as well.

You're basically saying I can move my volt meter probes to measure AF instead of AB and get a different reading, therefore AB is ambiguous.

(https://i.postimg.cc/bNr4Wmwk/20211130-124903.jpg)

That's a really really weak argument, because you could just as well say that any reading is ambiguous because by moving the probes to another part of the circuit we get a different reading.

But if it's the best you got, polish it to shine as bright as you can.
Title: Re: #562 – Electroboom!
Post by: HuronKing on November 30, 2021, 09:37:43 pm
What Mehdi actually says at that point you link above is "Dr. Lewin teaches in one of his courses that KVL doesn't hold true in some cases and I disagree with him."

From the CONTEXT of watching the whole video, it is clear that Mehdi is not claiming that KVL holds in all possible electrical typologies and frequencies, but rather he is specifically talking about certain cases that Lewin and he experimented with.

But yeah, if you want to quote mine, find videos of people who aren't even speaking their native language, I bet you can find slight ambiguities in their out-of-context words which you can present out of context to mean something else.

I've watched his videos... it's kind of right there.

"Dr. Lewin teaches in one of his courses that KVL doesn't hold true in some cases and I disagree with him."

This is a wrong statement. In any context. And he makes the same statement in his conclusion video - despite the warnings Belcher gives him about how to define the integral of the E-field around inductors.

Are there any typologies that Mehdi would accept where KVL is not an appropriate description? I'm not sure that he would. KVL is one of Maxwell's Equations to Mehdi (and I highlighted how profoundly he is confused in my first post in this thread by drawing attention to what he thinks KVL is not and how it contradicts the definition in Feynman that he thinks supports his views - it's madness).

Watch out! Another Lewin cultist spotted!
https://ultimateelectronicsbook.com/electrons-at-rest/
https://ultimateelectronicsbook.com/kirchhoffs-voltage-law-kirchhoffs-current-law/

Quote
However, an electric field with a time-varying magnetic field is not a conservative field. And we use this every day to great practical effect: we intentionally put currents in a loop within time-varying magnetic fields, and use those to extract electrical energy from the time-varying magnetic field (i.e. in generators), or use it to turn electrical energy into magnetic fields (i.e. in motors).

Regardless, we usually encapsulate these electromagnetic effects into our Lumped Element Model, and go on assuming that the electric field actually is conservative.
Understandably, this can be confusing and disorienting to beginners. Ninty-nine percent of the time, it’s safe to just assume the electric field is conservative, but if you’re doing anything with changing or moving magnetic fields, you should make a little mental note to remember that it’s really not.

Damn. Another mind lost to the Lewin infection.


Title: Re: #562 – Electroboom!
Post by: thinkfat on November 30, 2021, 09:56:50 pm
WHERE are you seeing "transformer secondaries"? You're evading answering again.

I edited that post to show the extra secondary in my diagram, also updated the text as well.

You're basically saying I can move my volt meter probes to measure AF instead of AB and get a different reading, therefore AB is ambiguous.

(https://i.postimg.cc/bNr4Wmwk/20211130-124903.jpg)

That's a really really weak argument, because you could just as well say that any reading is ambiguous because by moving the probes to another part of the circuit we get a different reading.

But if it's the best you got, polish it to shine as bright as you can.

Nope. Where in MY diagram do you see "transformer secondaries". Point them out, please.
Title: Re: #562 – Electroboom!
Post by: jesuscf on November 30, 2021, 10:05:09 pm
And... you don't understand what 'conservative' means when taking about circuits.  It is the energy that is conservative!  In Lewin's circuit, what is the instantaneous power induced in the circuit by means of the external magnetic field?  Now, what is the instantaneous power dissipated by the resistors?  Come on, is not a hard calculation!  If the two are equal, then the fields (plural) are conservative.

Emphasis mine. (Sredni, are you seeing this?!)
Oh my goodness. You think that because energy is conserved that fields must also be conserved? This is just - I don't even know what to say.  :-\

Oh dear oh dear oh dear:
https://courses.lumenlearning.com/physics/chapter/7-5-nonconservative-forces/ (https://courses.lumenlearning.com/physics/chapter/7-5-nonconservative-forces/)

It's amazing that you heard me say "the Lewin circuit demonstrates path dependence of non-conservative fields" and INSTANTLY thought this must be about conservation of energy. They're... not the same. You think conserved fields and conserved energy are descriptions of the same thing?  :(

So yea, I guess you should review Newtonian mechanics before we deal with things like circulating magnetic fields:
https://www.pearson.com/content/dam/one-dot-com/one-dot-com/us/en/higher-ed/en/products-services/course-products/wolfson-3e-info/pdf/sample-chapter--ch07.pdf (https://www.pearson.com/content/dam/one-dot-com/one-dot-com/us/en/higher-ed/en/products-services/course-products/wolfson-3e-info/pdf/sample-chapter--ch07.pdf)

But I don't even know why I'm linking things for you - you're clearly not opening and reading them.
And I read your post with the circuit - Sredni asked you questions about it that you never answered.

Wow, you sound a lot like bsfeechannel.  Your ignorance hurts!  Three questions for you: 1) How many fields do we have in the loop circuit with two resistors?  2) What is the instantaneous power induced in the circuit by the external varying magnetic field?  3) What is the instantaneous power consumed by the circuit?  The answers are three real numbers.  Anything else you say is BS.

Don't ask Sredni for help, he is busy trying to calculate the voltage between nodes 'A' and 'D', VAD.
Title: Re: #562 – Electroboom!
Post by: HuronKing on November 30, 2021, 10:22:00 pm
And... you don't understand what 'conservative' means when taking about circuits.  It is the energy that is conservative!  In Lewin's circuit, what is the instantaneous power induced in the circuit by means of the external magnetic field?  Now, what is the instantaneous power dissipated by the resistors?  Come on, is not a hard calculation!  If the two are equal, then the fields (plural) are conservative.

Emphasis mine. (Sredni, are you seeing this?!)
Oh my goodness. You think that because energy is conserved that fields must also be conserved? This is just - I don't even know what to say.  :-\

Oh dear oh dear oh dear:
https://courses.lumenlearning.com/physics/chapter/7-5-nonconservative-forces/ (https://courses.lumenlearning.com/physics/chapter/7-5-nonconservative-forces/)

It's amazing that you heard me say "the Lewin circuit demonstrates path dependence of non-conservative fields" and INSTANTLY thought this must be about conservation of energy. They're... not the same. You think conserved fields and conserved energy are descriptions of the same thing?  :(

So yea, I guess you should review Newtonian mechanics before we deal with things like circulating magnetic fields:
https://www.pearson.com/content/dam/one-dot-com/one-dot-com/us/en/higher-ed/en/products-services/course-products/wolfson-3e-info/pdf/sample-chapter--ch07.pdf (https://www.pearson.com/content/dam/one-dot-com/one-dot-com/us/en/higher-ed/en/products-services/course-products/wolfson-3e-info/pdf/sample-chapter--ch07.pdf)

But I don't even know why I'm linking things for you - you're clearly not opening and reading them.
And I read your post with the circuit - Sredni asked you questions about it that you never answered.

Wow, you sound a lot like bsfeechannel.  Your ignorance hurts!  Three questions for you: 1) How many fields do we have in the loop circuit with two resistors?  2) What is the instantaneous power induced in the circuit by the external varying magnetic field?  3) What is the instantaneous power consumed by the circuit?  The answers are three real numbers.  Anything else you say is BS.

Don't ask Sredni for help, he is busy trying to calculate the voltage between nodes 'A' and 'D', VAD.

Ah you refuse to learn about energy conservation as opposed to field conservation. No wonder Faraday's Law seems like a big inscrutable mystery that only KVL can save you from.

Did you open any of those links? Have you opened anything I've linked? Have you read Kraus, chapter 4? Or chapter 10? It's easy to find online.

Let me guess, you're going to say...
"That's all BS!"

Title: Re: #562 – Electroboom!
Post by: jesuscf on November 30, 2021, 10:42:25 pm
And... you don't understand what 'conservative' means when taking about circuits.  It is the energy that is conservative!  In Lewin's circuit, what is the instantaneous power induced in the circuit by means of the external magnetic field?  Now, what is the instantaneous power dissipated by the resistors?  Come on, is not a hard calculation!  If the two are equal, then the fields (plural) are conservative.

Emphasis mine. (Sredni, are you seeing this?!)
Oh my goodness. You think that because energy is conserved that fields must also be conserved? This is just - I don't even know what to say.  :-\

Oh dear oh dear oh dear:
https://courses.lumenlearning.com/physics/chapter/7-5-nonconservative-forces/ (https://courses.lumenlearning.com/physics/chapter/7-5-nonconservative-forces/)

It's amazing that you heard me say "the Lewin circuit demonstrates path dependence of non-conservative fields" and INSTANTLY thought this must be about conservation of energy. They're... not the same. You think conserved fields and conserved energy are descriptions of the same thing?  :(

So yea, I guess you should review Newtonian mechanics before we deal with things like circulating magnetic fields:
https://www.pearson.com/content/dam/one-dot-com/one-dot-com/us/en/higher-ed/en/products-services/course-products/wolfson-3e-info/pdf/sample-chapter--ch07.pdf (https://www.pearson.com/content/dam/one-dot-com/one-dot-com/us/en/higher-ed/en/products-services/course-products/wolfson-3e-info/pdf/sample-chapter--ch07.pdf)

But I don't even know why I'm linking things for you - you're clearly not opening and reading them.
And I read your post with the circuit - Sredni asked you questions about it that you never answered.

Wow, you sound a lot like bsfeechannel.  Your ignorance hurts!  Three questions for you: 1) How many fields do we have in the loop circuit with two resistors?  2) What is the instantaneous power induced in the circuit by the external varying magnetic field?  3) What is the instantaneous power consumed by the circuit?  The answers are three real numbers.  Anything else you say is BS.

Don't ask Sredni for help, he is busy trying to calculate the voltage between nodes 'A' and 'D', VAD.

Ah you refuse to learn about energy conservation as opposed to field conservation. No wonder Faraday's Law seems like a big inscrutable mystery that only KVL can save you from.

Did you open any of those links? Have you opened anything I've linked? Have you read Kraus, chapter 4? Or chapter 10? It's easy to find online.

Let me guess, you're going to say...
"That's all BS!"

I understand your diversionary tactics and I will not fall for them.  Let us concentrate in the solution of Lewin's original problem.  Now, the three numerical answers are:

1) ____ fields.

2) ____ W.

3) ____ W.

Fill in the blanks.


Title: Re: #562 – Electroboom!
Post by: Jesse Gordon on November 30, 2021, 10:46:01 pm
WHERE are you seeing "transformer secondaries"? You're evading answering again.

I edited that post to show the extra secondary in my diagram, also updated the text as well.

You're basically saying I can move my volt meter probes to measure AF instead of AB and get a different reading, therefore AB is ambiguous.

(https://i.postimg.cc/bNr4Wmwk/20211130-124903.jpg)

That's a really really weak argument, because you could just as well say that any reading is ambiguous because by moving the probes to another part of the circuit we get a different reading.

But if it's the best you got, polish it to shine as bright as you can.

Nope. Where in MY diagram do you see "transformer secondaries". Point them out, please.

My diagram literally shows the same topology as yours including the additional secondary.

But if you want me to actually show it in your diagram, fine, I'll do it after you answer the following YES or NO question. I asked you before and you refuse to answer me.

Question: In the following diagram, in a real life physical lab test performed with real (time synchronized) volt meters with a real transformer and real resistors CONNECTED AS SHOWN, will the readings of all the volt meters sum to zero, within the accuracy and resolution limitations of the volt meters? YES or NO.
(Or if you believe SOMETIMES is the answer, then explain one scenario for a YES condition and one scenario for a NO condition WITH THE VOLT METERS CONNECTED AS SHOWN - Running additional conductors through the transformer core is not allowed - nor is removing existing conductors from through the transformer core!)

(https://i.postimg.cc/jdJntBXT/20211128-121506.jpg)

Seriously, other than intentional repositioning of probes or intentional changes of details inside my lumped elements, all the measurements above are unambiguous.
Title: Re: #562 – Electroboom!
Post by: Sredni on November 30, 2021, 10:46:28 pm
(https://i.postimg.cc/jdJntBXT/20211128-121506.jpg)
https://i.postimg.cc/jdJntBXT/20211128-121506.jpg (https://i.postimg.cc/jdJntBXT/20211128-121506.jpg)

My question is a simple yes or no question. Either the volt meters will sum to zero or they won't. Or maybe you think they might some days depending on the phase of the moon.
If you place your voltmeters all in the same simply connected region that does not include the variable dB/dt region, then yes of course KVL will work. It's written in the top right corner of the picture I have posted more then a week ago. You still have to understand that?

And you still absolutely refuse to answer a simple yes/sometimes/no question regarding the above diagram.
--snip--
Seriously, what's your best guess about the volt meters in the above diagram? Do you think they would sum to zero? or not? or just sometimes?

Maybe it would help you to visualize real world values. How about 100mV per turn induced emf in the transformer core, and  each resistor is 50 ohms.

Dude, lighten up and just answer the question so we can move on. Say "Yes, No, or Sometimes" and there's always the ever popular "I don't know" if that is your case, but if you don't know then you really don't know do you.

If this was chess, you're acting like you're in check. You're acting like you don't want to make a move because you know it'll put you in a bind you can't get out of.

I'm not asking you to try and follow some twisted mental gymnastics and draw a secret diagram I have in my mind.

I drew a simple real-world diagram which you can test yourself with a real transformer and real volt meter and resistors if you want, or if you already know the answer you can just give the answer.

If you can't do that, there's no way the other stuff you're asking is ever going to make sense.

Will the volt meters sum to zero in the above diagram in a real world test? Yes, Sometimes, No.

Thank you!

This reminds me of when I was a kid. My cousin has this motorcycle - more of a scooter - that had to be started by jumping on a sort of lever. It never started at the first jolt. It usually needed three or four jumps.
I wonder if it's the same with you.
Here. Reread my answer. I'll add a few cues.

Here's the picture I was talking about:

(https://i.postimg.cc/Yq1ZDcPY/KVL-works-if-I-leave-out-the-magnetic-region.jpg)

It's literally titled: "KVL-works-if-I-leave-out-the-magnetic-region.jpg"
Does the ".jpg" confuse you? Or is it the dashes?
If you look at the top right corner, it says: "KVL works here".
What part of "KVL works here" you do not understand? "KVL"? "works"? "here"?

I hope it is clear that if you choose to look at a lumpABLE circuit as a lumpED circuit, then KVL works.

Wrooom... wrooom.... ?

?

Maybe that tank of yours is empty?
Title: Re: #562 – Electroboom!
Post by: HuronKing on November 30, 2021, 10:49:22 pm
And... you don't understand what 'conservative' means when taking about circuits.  It is the energy that is conservative!  In Lewin's circuit, what is the instantaneous power induced in the circuit by means of the external magnetic field?  Now, what is the instantaneous power dissipated by the resistors?  Come on, is not a hard calculation!  If the two are equal, then the fields (plural) are conservative.

Emphasis mine. (Sredni, are you seeing this?!)
Oh my goodness. You think that because energy is conserved that fields must also be conserved? This is just - I don't even know what to say.  :-\

Oh dear oh dear oh dear:
https://courses.lumenlearning.com/physics/chapter/7-5-nonconservative-forces/ (https://courses.lumenlearning.com/physics/chapter/7-5-nonconservative-forces/)

It's amazing that you heard me say "the Lewin circuit demonstrates path dependence of non-conservative fields" and INSTANTLY thought this must be about conservation of energy. They're... not the same. You think conserved fields and conserved energy are descriptions of the same thing?  :(

So yea, I guess you should review Newtonian mechanics before we deal with things like circulating magnetic fields:
https://www.pearson.com/content/dam/one-dot-com/one-dot-com/us/en/higher-ed/en/products-services/course-products/wolfson-3e-info/pdf/sample-chapter--ch07.pdf (https://www.pearson.com/content/dam/one-dot-com/one-dot-com/us/en/higher-ed/en/products-services/course-products/wolfson-3e-info/pdf/sample-chapter--ch07.pdf)

But I don't even know why I'm linking things for you - you're clearly not opening and reading them.
And I read your post with the circuit - Sredni asked you questions about it that you never answered.

Wow, you sound a lot like bsfeechannel.  Your ignorance hurts!  Three questions for you: 1) How many fields do we have in the loop circuit with two resistors?  2) What is the instantaneous power induced in the circuit by the external varying magnetic field?  3) What is the instantaneous power consumed by the circuit?  The answers are three real numbers.  Anything else you say is BS.

Don't ask Sredni for help, he is busy trying to calculate the voltage between nodes 'A' and 'D', VAD.

Ah you refuse to learn about energy conservation as opposed to field conservation. No wonder Faraday's Law seems like a big inscrutable mystery that only KVL can save you from.

Did you open any of those links? Have you opened anything I've linked? Have you read Kraus, chapter 4? Or chapter 10? It's easy to find online.

Let me guess, you're going to say...
"That's all BS!"

I understand your diversionary tactics and I will not fall for them.  Let us concentrate in the solution of Lewin's original problem.  Now, the three numerical answers are:

1) ____ fields.

2) ____ W.

3) ____ W.

Fill in the blanks.

So the answer is no, no, and no.

Ah you're right. Faraday's Law fields are always conservative. You win.

...

Nah, even a lie that big is too much for me to utter without laughing in your face. Lolz - referencing textbooks that discuss the phenomena you don't get in all its wonderful mathematical detail is diversionary.  :-DD :-DD
Title: Re: #562 – Electroboom!
Post by: jesuscf on November 30, 2021, 10:54:29 pm
And... you don't understand what 'conservative' means when taking about circuits.  It is the energy that is conservative!  In Lewin's circuit, what is the instantaneous power induced in the circuit by means of the external magnetic field?  Now, what is the instantaneous power dissipated by the resistors?  Come on, is not a hard calculation!  If the two are equal, then the fields (plural) are conservative.

Emphasis mine. (Sredni, are you seeing this?!)
Oh my goodness. You think that because energy is conserved that fields must also be conserved? This is just - I don't even know what to say.  :-\

Oh dear oh dear oh dear:
https://courses.lumenlearning.com/physics/chapter/7-5-nonconservative-forces/ (https://courses.lumenlearning.com/physics/chapter/7-5-nonconservative-forces/)

It's amazing that you heard me say "the Lewin circuit demonstrates path dependence of non-conservative fields" and INSTANTLY thought this must be about conservation of energy. They're... not the same. You think conserved fields and conserved energy are descriptions of the same thing?  :(

So yea, I guess you should review Newtonian mechanics before we deal with things like circulating magnetic fields:
https://www.pearson.com/content/dam/one-dot-com/one-dot-com/us/en/higher-ed/en/products-services/course-products/wolfson-3e-info/pdf/sample-chapter--ch07.pdf (https://www.pearson.com/content/dam/one-dot-com/one-dot-com/us/en/higher-ed/en/products-services/course-products/wolfson-3e-info/pdf/sample-chapter--ch07.pdf)

But I don't even know why I'm linking things for you - you're clearly not opening and reading them.
And I read your post with the circuit - Sredni asked you questions about it that you never answered.

Wow, you sound a lot like bsfeechannel.  Your ignorance hurts!  Three questions for you: 1) How many fields do we have in the loop circuit with two resistors?  2) What is the instantaneous power induced in the circuit by the external varying magnetic field?  3) What is the instantaneous power consumed by the circuit?  The answers are three real numbers.  Anything else you say is BS.

Don't ask Sredni for help, he is busy trying to calculate the voltage between nodes 'A' and 'D', VAD.

Ah you refuse to learn about energy conservation as opposed to field conservation. No wonder Faraday's Law seems like a big inscrutable mystery that only KVL can save you from.

Did you open any of those links? Have you opened anything I've linked? Have you read Kraus, chapter 4? Or chapter 10? It's easy to find online.

Let me guess, you're going to say...
"That's all BS!"

I understand your diversionary tactics and I will not fall for them.  Let us concentrate in the solution of Lewin's original problem.  Now, the three numerical answers are:

1) ____ fields.

2) ____ W.

3) ____ W.

Fill in the blanks.

So the answer is no, no, and no.

Ah you're right. Faraday's Law fields are always conservative. You win.

...

Nah, even a lie that big is too much for me to utter without laughing in your face.  :-DD :-DD

I'll leave the questions unanswered for now.  Let see if any one else from team Lewin comes forward to help you.
Title: Re: #562 – Electroboom!
Post by: jesuscf on November 30, 2021, 10:57:32 pm
(https://i.postimg.cc/jdJntBXT/20211128-121506.jpg)
https://i.postimg.cc/jdJntBXT/20211128-121506.jpg (https://i.postimg.cc/jdJntBXT/20211128-121506.jpg)

My question is a simple yes or no question. Either the volt meters will sum to zero or they won't. Or maybe you think they might some days depending on the phase of the moon.
If you place your voltmeters all in the same simply connected region that does not include the variable dB/dt region, then yes of course KVL will work. It's written in the top right corner of the picture I have posted more then a week ago. You still have to understand that?

And you still absolutely refuse to answer a simple yes/sometimes/no question regarding the above diagram.
--snip--
Seriously, what's your best guess about the volt meters in the above diagram? Do you think they would sum to zero? or not? or just sometimes?

Maybe it would help you to visualize real world values. How about 100mV per turn induced emf in the transformer core, and  each resistor is 50 ohms.

Dude, lighten up and just answer the question so we can move on. Say "Yes, No, or Sometimes" and there's always the ever popular "I don't know" if that is your case, but if you don't know then you really don't know do you.

If this was chess, you're acting like you're in check. You're acting like you don't want to make a move because you know it'll put you in a bind you can't get out of.

I'm not asking you to try and follow some twisted mental gymnastics and draw a secret diagram I have in my mind.

I drew a simple real-world diagram which you can test yourself with a real transformer and real volt meter and resistors if you want, or if you already know the answer you can just give the answer.

If you can't do that, there's no way the other stuff you're asking is ever going to make sense.

Will the volt meters sum to zero in the above diagram in a real world test? Yes, Sometimes, No.

Thank you!

This reminds me of when I was a kid. My cousin has this motorcycle - more of a scooter - that had to be started by jumping on a sort of lever. It never started at the first jolt. It usually needed three or four jumps.
I wonder if it's the same with you.
Here. Reread my answer. I'll add a few cues.

Here's the picture I was talking about:

(https://i.postimg.cc/Yq1ZDcPY/KVL-works-if-I-leave-out-the-magnetic-region.jpg)

It's literally titled: "KVL-works-if-I-leave-out-the-magnetic-region.jpg"
Does the ".jpg" confuse you? Or is it the dashes?
If you look at the top right corner, it says: "KVL works here".
What part of "KVL works here" you do not understand? "KVL"? "works"? "here"?

I hope it is clear that if you choose to look at a lumpABLE circuit as a lumpED circuit, then KVL works.

Wrooom... wrooom.... ?

?

Maybe that tank of yours is empty?

You are back!  Have you figured out how to CALCULATE the voltage between nodes 'A' and 'D' yet?  Show us!
Title: Re: #562 – Electroboom!
Post by: Sredni on November 30, 2021, 11:02:08 pm
You are back!  Have you figured out how to CALCULATE the voltage between nodes 'A' and 'D' yet?  Show us!

Your tank is sure dry.
Because I have already told you at least four times that it depends on the path, and I have calculated it for two paths already.
Title: Re: #562 – Electroboom!
Post by: jesuscf on November 30, 2021, 11:09:35 pm
You are back!  Have you figured out how to CALCULATE the voltage between nodes 'A' and 'D' yet?  Show us!

Your tank is sure dry.
Because I have already told you at least four times that it depends on the path, and I have calculated it for two paths already.

Nope.  The answers you provided are the voltages MEASURED by the 'voltmeters' on the left and right.  CALCULATE the voltage VAD; you can use either the left branch or the right branch.
Title: Re: #562 – Electroboom!
Post by: thinkfat on November 30, 2021, 11:09:37 pm
WHERE are you seeing "transformer secondaries"? You're evading answering again.

I edited that post to show the extra secondary in my diagram, also updated the text as well.

You're basically saying I can move my volt meter probes to measure AF instead of AB and get a different reading, therefore AB is ambiguous.

(https://i.postimg.cc/bNr4Wmwk/20211130-124903.jpg)

That's a really really weak argument, because you could just as well say that any reading is ambiguous because by moving the probes to another part of the circuit we get a different reading.

But if it's the best you got, polish it to shine as bright as you can.

Nope. Where in MY diagram do you see "transformer secondaries". Point them out, please.

My diagram literally shows the same topology as yours including the additional secondary.

But if you want me to actually show it in your diagram, fine, I'll do it after you answer the following YES or NO question. I asked you before and you refuse to answer me.

Question: In the following diagram, in a real life physical lab test performed with real (time synchronized) volt meters with a real transformer and real resistors CONNECTED AS SHOWN, will the readings of all the volt meters sum to zero, within the accuracy and resolution limitations of the volt meters? YES or NO.
(Or if you believe SOMETIMES is the answer, then explain one scenario for a YES condition and one scenario for a NO condition WITH THE VOLT METERS CONNECTED AS SHOWN - Running additional conductors through the transformer core is not allowed - nor is removing existing conductors from through the transformer core!)

(https://i.postimg.cc/jdJntBXT/20211128-121506.jpg)

Seriously, other than intentional repositioning of probes or intentional changes of details inside my lumped elements, all the measurements above are unambiguous.

Nope. No more bobbing and weaving for you. No more smoke bombs. You don't answer simply because the answer would reveal the inconsistencies in your "modeling of reality". So which of the loops in MY diagram are "transformer secondaries"?
Title: Re: #562 – Electroboom!
Post by: Jesse Gordon on November 30, 2021, 11:22:45 pm
(https://i.postimg.cc/jdJntBXT/20211128-121506.jpg)
https://i.postimg.cc/jdJntBXT/20211128-121506.jpg (https://i.postimg.cc/jdJntBXT/20211128-121506.jpg)

My question is a simple yes or no question. Either the volt meters will sum to zero or they won't. Or maybe you think they might some days depending on the phase of the moon.
If you place your voltmeters all in the same simply connected region that does not include the variable dB/dt region, then yes of course KVL will work. It's written in the top right corner of the picture I have posted more then a week ago. You still have to understand that?

And you still absolutely refuse to answer a simple yes/sometimes/no question regarding the above diagram.
--snip--
Seriously, what's your best guess about the volt meters in the above diagram? Do you think they would sum to zero? or not? or just sometimes?

Maybe it would help you to visualize real world values. How about 100mV per turn induced emf in the transformer core, and  each resistor is 50 ohms.

Dude, lighten up and just answer the question so we can move on. Say "Yes, No, or Sometimes" and there's always the ever popular "I don't know" if that is your case, but if you don't know then you really don't know do you.

If this was chess, you're acting like you're in check. You're acting like you don't want to make a move because you know it'll put you in a bind you can't get out of.

I'm not asking you to try and follow some twisted mental gymnastics and draw a secret diagram I have in my mind.

I drew a simple real-world diagram which you can test yourself with a real transformer and real volt meter and resistors if you want, or if you already know the answer you can just give the answer.

If you can't do that, there's no way the other stuff you're asking is ever going to make sense.

Will the volt meters sum to zero in the above diagram in a real world test? Yes, Sometimes, No.

Thank you!

This reminds me of when I was a kid. My cousin has this motorcycle - more of a scooter - that had to be started by jumping on a sort of lever. It never started at the first jolt. It usually needed three or four jumps.
I wonder if it's the same with you.
Here. Reread my answer. I'll add a few cues.

Here's the picture I was talking about:

(https://i.postimg.cc/Yq1ZDcPY/KVL-works-if-I-leave-out-the-magnetic-region.jpg)

It's literally titled: "KVL-works-if-I-leave-out-the-magnetic-region.jpg"
Does the ".jpg" confuse you? Or is it the dashes?
If you look at the top right corner, it says: "KVL works here".
What part of "KVL works here" you do not understand? "KVL"? "works"? "here"?

I hope it is clear that if you choose to look at a lumpABLE circuit as a lumpED circuit, then KVL works.

Wrooom... wrooom.... ?

?

Maybe that tank of yours is empty?

Answering a Yes or No question with "if blah blah blah blah" probably isn't answering it.

Even a motorbike knows how to answer simple questions like that. You give it a kick, and it answers "No." you give it a few more kicks, and the answer changes from No to Almost. After a while it finally says "Yes."

I think your tank is empty. I don't need you to explain to me how to find out what reality is, I have a volt meter, I already know.

I'm asking your opinion of the following:

Question: In the following diagram, in a real life physical lab test performed with real (time synchronized) volt meters with a real transformer and real resistors CONNECTED AS SHOWN, will the readings of all the volt meters sum to zero, within the accuracy and resolution limitations of the volt meters? YES or NO.
(Or if you believe SOMETIMES is the answer, then explain one scenario for a YES condition and one scenario for a NO condition WITH THE VOLT METERS CONNECTED AS SHOWN - Running additional conductors through the transformer core is not allowed - nor is removing existing conductors from through the transformer core!)

(https://i.postimg.cc/jdJntBXT/20211128-121506.jpg)
Title: Re: #562 – Electroboom!
Post by: Sredni on November 30, 2021, 11:27:37 pm
You are back!  Have you figured out how to CALCULATE the voltage between nodes 'A' and 'D' yet?  Show us!

Your tank is sure dry.
Because I have already told you at least four times that it depends on the path, and I have calculated it for two paths already.

Nope.  The answers you provided are the voltages MEASURED by the 'voltmeters' on the left and right.  CALCULATE the voltage VAD; you can use either the left branch or the right branch.

I calculated the voltages measured by the voltmeters. These are also the voltages from probe tip to probe tip, because I do not have silly tiny batteries in my probe wires. And I can compute the voltages along the two branches of the circuit that join A and D: one branch through R1 and one branch through R2 (can't use KVL like for Lewin's ring, becase all measurement loops are cutting the flux, so I'll have to use Faraday for those as well). And I can also compute the voltage along a path that joins A and D and cuts the magnetic flux region in two equal parts. Or along a path that draws Snoopy's profile.
But what's the point? You wouldn't understand.
Title: Re: #562 – Electroboom!
Post by: thinkfat on November 30, 2021, 11:33:13 pm
It's really funny how one of you insists on calculations and the other one on voltmeter readings being "truth". You really need to clean up your act before you go on stage.
Title: Re: #562 – Electroboom!
Post by: Jesse Gordon on November 30, 2021, 11:42:21 pm
It's really funny how one of you insists on calculations and the other one on voltmeter readings being "truth". You really need to clean up your act before you go on stage.

We're grinding both angles because Team Lewin can neither calculate nor measure reality.

Correct calculations like Belcher and McDonald did agree with measurements. Team KVL is on the same page.

Team Lewin, on the other hand, is all over the board.

But the more they realize they are in check, the less willing they become to answer questions, as they find out their understanding of reality is so far off base they dare not show their hand.
Title: Re: #562 – Electroboom!
Post by: thinkfat on November 30, 2021, 11:54:56 pm
Blah blah blah. Are you going to answer or not?
Title: Re: #562 – Electroboom!
Post by: jesuscf on November 30, 2021, 11:57:35 pm
You are back!  Have you figured out how to CALCULATE the voltage between nodes 'A' and 'D' yet?  Show us!

Your tank is sure dry.
Because I have already told you at least four times that it depends on the path, and I have calculated it for two paths already.

Nope.  The answers you provided are the voltages MEASURED by the 'voltmeters' on the left and right.  CALCULATE the voltage VAD; you can use either the left branch or the right branch.

I calculated the voltages measured by the voltmeters. These are also the voltages from probe tip to probe tip, because I do not have silly tiny batteries in my probe wires. And I can compute the voltages along the two branches of the circuit that join A and D: one branch through R1 and one branch through R2 (can't use KVL like for Lewin's ring, becase all measurement loops are cutting the flux, so I'll have to use Faraday for those as well). And I can also compute the voltage along a path that joins A and D and cuts the magnetic flux region in two equal parts. Or along a path that draws Snoopy's profile.
But what's the point? You wouldn't understand.

First, let me clarify some terminology: the proper name is not 'silly tiny batteries'; it is 'voltage sources'.  Now remove the 'voltmeters', and you only have the loop and the two resistors R1 and R2 and the varying external magnetic field.  What is the voltage between nodes 'A' and 'D'?  I think you already figured it out (how else would have you found Vx and Vy?), but you don't want to contradict Lewin's incorrect answer in which you have invested so much effort...
Title: Re: #562 – Electroboom!
Post by: Jesse Gordon on December 01, 2021, 12:06:55 am
Blah blah blah. Are you going to answer or not?

Like I said, I'll answer you AFTER you answer the following question:

Question: In the following diagram, in a real life physical lab test performed with real (time synchronized) volt meters with a real transformer and real resistors CONNECTED AS SHOWN, will the readings of all the volt meters sum to zero, within the accuracy and resolution limitations of the volt meters? YES or NO.

(Or if you believe SOMETIMES is the answer, then explain one scenario for a YES condition and one scenario for a NO condition WITH THE VOLT METERS CONNECTED AS SHOWN - Running additional conductors through the transformer core is not allowed - nor is removing existing conductors from through the transformer core!)

(https://i.postimg.cc/jdJntBXT/20211128-121506.jpg)


Title: Re: #562 – Electroboom!
Post by: jesuscf on December 01, 2021, 12:09:07 am
It's really funny how one of you insists on calculations and the other one on voltmeter readings being "truth". You really need to clean up your act before you go on stage.

Hey! I have both calculations and measurements:

Calculated:

V1=-24mV
V2=216mV
Vx=-136mV
Vy=-56mV
VAD=-96mV

Measured:

V1=-24.4mV
V2=212mV
Vx=-134mV
Vy=-56mV
VAD=-96mV

I have yet to see any measurements from team Lewin.  Also, the only one doing calculations in team Lewin is Sredni.  Sadly he can not figure how to calculate VAD.
Title: Re: #562 – Electroboom!
Post by: jesuscf on December 01, 2021, 12:14:47 am
It's really funny how one of you insists on calculations and the other one on voltmeter readings being "truth". You really need to clean up your act before you go on stage.

We're grinding both angles because Team Lewin can neither calculate nor measure reality.

Correct calculations like Belcher and McDonald did agree with measurements. Team KVL is on the same page.

Team Lewin, on the other hand, is all over the board.

But the more they realize they are in check, the less willing they become to answer questions, as they find out their understanding of reality is so far off base they dare not show their hand.

I do believe that team Lewin doesn't know either how to use the oscilloscope or KVL!  By the way, I asked those two questions earlier in the thread...
Title: Re: #562 – Electroboom!
Post by: Jesse Gordon on December 01, 2021, 12:22:25 am
Well  gents,

we've come full circle ha ha ha  :-DD :-DD :-DD

Supposing you're right that I'm clueless, one thing I do know is that Team Lewin seems to think they in check.

Ever been playing chess with someone and they think they are in check and they are taking forever and don't want to move, because they think it may be checkmate after that?

Team Lewin is unwilling or unable to answer simple yes/no/sometimes to a pivotal question (reproduced below for reference) as if they believe they are in a bind.

In my opinion it has been clearly established with experimental results and mathematically that KVL holds in the below diagram as well as in Lewin's circuit, as Dr. McDonald says.

The absolute refusal on the part of Team Lewin to even admit to the appearance of KVL holding  in the below diagram further confirms their belief that they are cornered on the topic and they do see it as a pivotal question.

You might say that my question below is not pivotal, but looking at the amount of effort Team Lewin has put in trying to avoid answering "Yes" or "No" tells me that it is in fact a pivotal question.

In light of the fact that Team Lewin refuses to embrace (or even admit to) observable reality, it is clear this discussion isn't going to do any good in any short amount of time, and as I need to work if I want to get paid and this isn't what I get paid to do, I'm going to have to reduce the amount of time I enjoy here, so let's see if I can reduce my interaction to a day or two a week.

I do not get notice of  comments here, but if you PM me then I get an email notification, so if somebody actually answers my question for real, please PM  me and let me know!

In other news, I've begun to suspect that ads are playing on my youtube videos which really irks me because I don't upload videos for money and I don't make anything from it and even if I did with 15 videos and 12 subscribers it wouldn't be any money at all - so youtube is running intrusive ads and not paying me and not giving me a choice. If they allowed me to monetize then maybe I could go in and turn ads off for each video, but I can't even try that since they won't allow me to monetize.

Long and the short of it is I'm trying out Vimeo free - I dunno if they are better but I uploaded a couple test videos.

Does this work better? No ads?

https://vimeo.com/651687593

And actually, we can all probably see each other in that video ha ha ha ha  :-DD :-DD :-DD

We're all just aphids with this huge shadow  :-DD :-DD :-DD

(Actually Snedri, when I saw that aphid crawling along with a huge shadow I thought of your antics here, that's why I filmed it. But I imagine you probably see KVLer's the same way.)

Question: In the following diagram, in a real life physical lab test performed with real (time synchronized) volt meters with a real transformer and real resistors CONNECTED AS SHOWN, will the readings of all the volt meters sum to zero, within the accuracy and resolution limitations of the volt meters? YES or NO.

(Or if you believe SOMETIMES is the answer, then explain one scenario for a YES condition and one scenario for a NO condition WITH THE VOLT METERS CONNECTED AS SHOWN - Running additional conductors through the transformer core is not allowed - nor is removing existing conductors from through the transformer core!)

(https://i.postimg.cc/jdJntBXT/20211128-121506.jpg)
Title: Re: #562 – Electroboom!
Post by: Jesse Gordon on December 01, 2021, 12:26:26 am
It's really funny how one of you insists on calculations and the other one on voltmeter readings being "truth". You really need to clean up your act before you go on stage.

Hey! I have both calculations and measurements:

Calculated:

V1=-24mV
V2=216mV
Vx=-136mV
Vy=-56mV
VAD=-96mV

Measured:

V1=-24.4mV
V2=212mV
Vx=-134mV
Vy=-56mV
VAD=-96mV

I have yet to see any measurements from team Lewin.  Also, the only one doing calculations in team Lewin is Sredni.  Sadly he can not figure how to calculate VAD.

Dude! You gotta be more careful! Really?!? Turning on an actual oscilloscope? Opening up a glimpse into the real world? That's dangerous!

You'll scare the children!!  :-DD :-DD :-DD
Title: Re: #562 – Electroboom!
Post by: Sredni on December 01, 2021, 01:15:34 am
Well  gents,
we've come full circle ha ha ha  :-DD :-DD :-DD
Supposing you're right that I'm clueless, one thing I do know is that Team Lewin seems to think they in check.

Another victory lap.
You are restless.

Quote
Team Lewin is unwilling or unable to answer simple yes/no/sometimes to a pivotal question (reproduced below for reference) as if they believe they are in a bind.

The projection is strong in you.
Here is your answer. I wrote YES very big, so you can't miss it.
Read it all. Enjoy.

(https://i.postimg.cc/MTKM29p2/KVL-works-YEEESSSSS-Sorta.jpg)
https://i.postimg.cc/MTKM29p2/KVL-works-YEEESSSSS-Sorta.jpg

Title: Re: #562 – Electroboom!
Post by: Sredni on December 01, 2021, 01:25:25 am
You are back!  Have you figured out how to CALCULATE the voltage between nodes 'A' and 'D' yet?  Show us!

Your tank is sure dry.
Because I have already told you at least four times that it depends on the path, and I have calculated it for two paths already.

Nope.  The answers you provided are the voltages MEASURED by the 'voltmeters' on the left and right.  CALCULATE the voltage VAD; you can use either the left branch or the right branch.

I calculated the voltages measured by the voltmeters. These are also the voltages from probe tip to probe tip, because I do not have silly tiny batteries in my probe wires. And I can compute the voltages along the two branches of the circuit that join A and D: one branch through R1 and one branch through R2 (can't use KVL like for Lewin's ring, becase all measurement loops are cutting the flux, so I'll have to use Faraday for those as well). And I can also compute the voltage along a path that joins A and D and cuts the magnetic flux region in two equal parts. Or along a path that draws Snoopy's profile.
But what's the point? You wouldn't understand.

First, let me clarify some terminology: the proper name is not 'silly tiny batteries'; it is 'voltage sources'.  Now remove the 'voltmeters', and you only have the loop and the two resistors R1 and R2 and the varying external magnetic field.  What is the voltage between nodes 'A' and 'D'?  I think you already figured it out (how else would have you found Vx and Vy?), but you don't want to contradict Lewin's incorrect answer in which you have invested so much effort...

You really, really, really don't get it eh?
Ok. Here is my computed value for Vad - I call it VBA, just because I can

VBA(along left voltmeter) = -136 mV
VBA(along right voltmeter) = -56 mV
VBA(along circuit branch with R1) = +24 mV
VBA(along circuit branch with R2) = -216 mV
VBA(along line that splits the core in two) = -96 mV
VBA(along line that splits the core in 3/8 - 5/8) = -86 mV
VBA(along line that splits the core in 5/8 - 3/8) = -116 mV
VBA(along Snoopy profile) = - 100 mV

Why so many values?
Because...

--- drum roll ---

THE ****ING VOLTAGE DEPENDS ON THE ****ING PATH!!!

Title: Re: #562 – Electroboom!
Post by: jesuscf on December 01, 2021, 01:33:24 am
You really, really, really don't get it eh?
Ok. Here is my computed value for Vad - I call it VBA, just because I can

VBA(along left voltmeter) = -136 mV
VBA(along right voltmeter) = -56 mV
VBA(along circuit branch with R1) = +24 mV
VBA(along circuit branch with R2) = -216 mV
VBA(along line that splits the core in two) = -96 mV
VBA(along line that splits the core in 3/8 - 5/8) = -86 mV
VBA(along line that splits the core in 5/8 - 3/8) = -116 mV
VBA(along Snoopy profile) = - 100 mV

Why so many values?
Because...

--- drum roll ---

THE ****ING VOLTAGE DEPENDS ON THE ****ING PATH!!!

INCORRECT

(Actually, one of them is correct, but I think you got it by accident)

Tip, if you account for the induced EMF in the wires of the path you choose, you can easily compute the unique value of VAD using

--- drum roll ---

KVL
Title: Re: #562 – Electroboom!
Post by: Sredni on December 01, 2021, 03:07:06 am
Tip, if you account for the induced EMF in the wires of the path you choose,

Well, the really funny thing is... I do account for the induced electric field, while you don't.
You don't know that yet, but the reason you can have a conservative field that admits a scalar potential is that... you have stripped away the contribute of the induced field. It will be revealed in the upcoming post:

"The tiny batteries model, or: 'I don't think that field is what you think it is.' "

that you will misunderstand, of course.

(https://i.pinimg.com/originals/f9/dc/1d/f9dc1d38e61a798936f305af4839320e.jpg)
Title: Re: #562 – Electroboom!
Post by: bsfeechannel on December 01, 2021, 03:10:01 am
Maxwellians. People like Sredni aren't suggesting that KVL must hold even when it doesn't. It's not one of Maxwell's Equations - it's not sacred. So why the desperation to save it? Why is it so very important that KVL applies to the Lewin/Romer Ring? Well I don't know why. This is why Sredni is fascinated by the cognitive bias - why does everyone want to save KVL so badly?

I had a physics teacher in high school who used to say “A law is a law and always holds”. He said that because at least my generation had grown up watching Wile E. Coyote fly off a cliff in a horizontal trajectory, stop and, only after he realized there was no ground under his feet, free fall vertically.

(https://j.gifs.com/y776x8.gif)

People were confused when exactly the laws of motion were valid. If an object is thrown horizontally, does gravity act on it all the time or only when it cannot sustain a flight? Of course gravity acts all the time. So, to avoid having to think too much, teachers convey this idea that the laws of physics always hold.

Since KVL and KCL are presented as laws (Kirchhoff himself called them theorems, not laws), that’s what they think.

When these people go to get their EE degree, the professors don’t give enough emphasis on the fact that KCL  and KVL only hold under certain conditions and that Maxwell’s equations are the complete description of the electromagnetic phenomenon.

The result is people like Dave, Mehdi, Jesse, jesuscf and others who think KVL/KCL and Maxwell’s equations are two concurrent and equivalent theories. One is a clever simplification, they reckon, used by engineers and the other, more complicated, is there to maintain the intellect of physicists occupied. They don’t understand that the “more complicated” theory exists because the “clever simplification” is incomplete.

They get an aversion for calculus and Maxwell’s equations. treat RF superficially and consider any insight on the subject as black magic. In fact they use KVL and KCL to discourage people from going down the maxwellian rabbit hole. They can’t explain any electromagnetic phenomenon if they can’t model it using lumped components.

Dave can’t understand how energy in a DC circuit flows in the fields. Of course. The only components he knows that transfer energy in the fields are capacitors and transformers. Both interrupt any galvanic connection between both sides of the circuits they connect and they block DC.

Look at his cringe-worthy post on Twitter about this.

https://twitter.com/eevblog/status/1465447319663374338?s=20 (https://twitter.com/eevblog/status/1465447319663374338?s=20)

They struggle to interpret their measurements and some of them think that what makes you an engineer is your oscilloscope (goto 39).

It’s worse when these guys, in good faith, have taught KVL and KCL as always holding for years to hundreds of thousands of viewers. As Mehdi himself said in this very interview, it must be difficult to be wrong in front of a million people.

So that’s why KVLers cling to the KVL myth.

I think that the teaching of electromagnetism in schools and colleges must be radically changed. They need to go beyond Lewin and tell students that their whole lives are a lie.
Title: Re: #562 – Electroboom!
Post by: bsfeechannel on December 01, 2021, 04:17:53 am
Well  gents,

we've come full circle ha ha ha  :-DD :-DD :-DD

Supposing you're right that I'm clueless, one thing I do know is that Team Lewin seems to think they in check.

Ever been playing chess with someone and they think they are in check and they are taking forever and don't want to move, because they think it may be checkmate after that?

Team Lewin is unwilling or unable to answer simple yes/no/sometimes to a pivotal question (reproduced below for reference) as if they believe they are in a bind.

In my opinion it has been clearly established with experimental results and mathematically that KVL holds in the below diagram as well as in Lewin's circuit, as Dr. McDonald says.

The absolute refusal on the part of Team Lewin to even admit to the appearance of KVL holding  in the below diagram further confirms their belief that they are cornered on the topic and they do see it as a pivotal question.

You might say that my question below is not pivotal, but looking at the amount of effort Team Lewin has put in trying to avoid answering "Yes" or "No" tells me that it is in fact a pivotal question.

In light of the fact that Team Lewin refuses to embrace (or even admit to) observable reality, it is clear this discussion isn't going to do any good in any short amount of time, and as I need to work if I want to get paid and this isn't what I get paid to do, I'm going to have to reduce the amount of time I enjoy here, so let's see if I can reduce my interaction to a day or two a week.

I do not get notice of  comments here, but if you PM me then I get an email notification, so if somebody actually answers my question for real, please PM  me and let me know!

In other news, I've begun to suspect that ads are playing on my youtube videos which really irks me because I don't upload videos for money and I don't make anything from it and even if I did with 15 videos and 12 subscribers it wouldn't be any money at all - so youtube is running intrusive ads and not paying me and not giving me a choice. If they allowed me to monetize then maybe I could go in and turn ads off for each video, but I can't even try that since they won't allow me to monetize.

Long and the short of it is I'm trying out Vimeo free - I dunno if they are better but I uploaded a couple test videos.

Does this work better? No ads?

https://vimeo.com/651687593

And actually, we can all probably see each other in that video ha ha ha ha  :-DD :-DD :-DD

We're all just aphids with this huge shadow  :-DD :-DD :-DD

(Actually Snedri, when I saw that aphid crawling along with a huge shadow I thought of your antics here, that's why I filmed it. But I imagine you probably see KVLer's the same way.)

Question: In the following diagram, in a real life physical lab test performed with real (time synchronized) volt meters with a real transformer and real resistors CONNECTED AS SHOWN, will the readings of all the volt meters sum to zero, within the accuracy and resolution limitations of the volt meters? YES or NO.

(Or if you believe SOMETIMES is the answer, then explain one scenario for a YES condition and one scenario for a NO condition WITH THE VOLT METERS CONNECTED AS SHOWN - Running additional conductors through the transformer core is not allowed - nor is removing existing conductors from through the transformer core!)

(https://i.postimg.cc/jdJntBXT/20211128-121506.jpg)

TL;DR  I lost, but I won't admit it. I'll even claim victory, like any good KVLer.
Title: Re: #562 – Electroboom!
Post by: jesuscf on December 01, 2021, 05:14:31 am
Maxwellians. People like Sredni aren't suggesting that KVL must hold even when it doesn't. It's not one of Maxwell's Equations - it's not sacred. So why the desperation to save it? Why is it so very important that KVL applies to the Lewin/Romer Ring? Well I don't know why. This is why Sredni is fascinated by the cognitive bias - why does everyone want to save KVL so badly?

I had a physics teacher in high school who used to say “A law is a law and always holds”. He said that because at least my generation had grown up watching Wile E. Coyote fly off a cliff in a horizontal trajectory, stop and, only after he realized there was no ground under his feet, free fall vertically.

People were confused when exactly the laws of motion were valid. If an object is thrown horizontally, does gravity act on it all the time or only when it cannot sustain a flight? Of course gravity acts all the time. So, to avoid having to think too much, teachers convey this idea that the laws of physics always hold.

Since KVL and KCL are presented as laws (Kirchhoff himself called them theorems, not laws), that’s what they think.

When these people go to get their EE degree, the professors don’t give enough emphasis on the fact that KCL  and KVL only hold under certain conditions and that Maxwell’s equations are the complete description of the electromagnetic phenomenon.

The result is people like Dave, Mehdi, Jesse, jesuscf and others who think KVL/KCL and Maxwell’s equations are two concurrent and equivalent theories. One is a clever simplification, they reckon, used by engineers and the other, more complicated, is there to maintain the intellect of physicists occupied. They don’t understand that the “more complicated” theory exists because the “clever simplification” is incomplete.

They get an aversion for calculus and Maxwell’s equations. treat RF superficially and consider any insight on the subject as black magic. In fact they use KVL and KCL to discourage people from going down the maxwellian rabbit hole. They can’t explain any electromagnetic phenomenon if they can’t model it using lumped components.

Dave can’t understand how energy in a DC circuit flows in the fields. Of course. The only components he knows that transfer energy in the fields are capacitors and transformers. Both interrupt any galvanic connection between both sides of the circuits they connect and they block DC.

Look at his cringe-worthy post on Twitter about this.

https://twitter.com/eevblog/status/1465447319663374338?s=20 (https://twitter.com/eevblog/status/1465447319663374338?s=20)

They struggle to interpret their measurements and some of them think that what makes you an engineer is your oscilloscope (goto 39).

It’s worse when these guys, in good faith, have taught KVL and KCL as always holding for years to hundreds of thousands of viewers. As Mehdi himself said in this very interview, it must be difficult to be wrong in front of a million people.

So that’s why KVLers cling to the KVL myth.

I think that the teaching of electromagnetism in schools and colleges must be radically changed. They need to go beyond Lewin and tell students that their whole lives are a lie.

Hey bsfeechannel, do you have an electrical engineering degree (or similar) from an accredited institution?  I am asking because the Dunning-Kruger is very strong in you.

EDIT: By the way, I have to thank you for putting me in the same group with Dave, Mehdi, and Jesse.  I am pretty sure I am not as good as they are, but nevertheless it is a great honor to me.
Title: Re: #562 – Electroboom!
Post by: jesuscf on December 01, 2021, 05:26:04 am
Tip, if you account for the induced EMF in the wires of the path you choose,

Well, the really funny thing is... I do account for the induced electric field, while you don't.
You don't know that yet, but the reason you can have a conservative field that admits a scalar potential is that... you have stripped away the contribute of the induced field. It will be revealed in the upcoming post:

"The tiny batteries model, or: 'I don't think that field is what you think it is.' "

that you will misunderstand, of course.

Yes, yes, I know the drill.  You will go to the barn, ask your servants to feed the bulls, wait a few hours, and then dump the byproduct of all that fiber rich feed into this forum, with lots of hand drawn components and equations, so to obfuscate and/or divert from the topic.  Instead you should be thinking hard on why the voltage between nodes 'A' and 'D', VAD is unique and independent of the path you choose to calculate it.
Title: Re: #562 – Electroboom!
Post by: bsfeechannel on December 01, 2021, 05:41:53 am
Hey bsfeechannel, do you have an electrical engineering degree (or similar) from an accredited institution?  I am asking because the Dunning-Kruger is very strong in you.

Hey jesuscf, you showed a few posts ago your absolute stupidity about conservative and non-conservative fields. Who are you kidding?

Without that fundamental concept, you will not understand what Lewin, Belcher, McDonald, Maxwell, Faraday, or any of us are talking about.
Title: Re: #562 – Electroboom!
Post by: jesuscf on December 01, 2021, 05:55:23 am
Hey bsfeechannel, do you have an electrical engineering degree (or similar) from an accredited institution?  I am asking because the Dunning-Kruger is very strong in you.

Hey jesuscf, you showed a few posts ago your absolute stupidity about conservative and non-conservative fields. Who are you kidding?

Without that fundamental concept, you will not understand what Lewin, Belcher, McDonald, Maxwell, Faraday, or any of us are talking about.

Really?  So, you should not have any problem answering these questions then:  1) How many fields do we have in the loop circuit with two resistors?  2) What is the instantaneous power induced in the circuit by the external varying magnetic field?  3) What is the instantaneous power consumed by the circuit?  The answers are three real numbers.

1) ____ fields.

2) ____ W.

3) ____ W.

I'll be waiting!

EDIT: Sorry, I got distracted.  So, do you have a degree or not?
Title: Re: #562 – Electroboom!
Post by: bsfeechannel on December 01, 2021, 06:24:44 am
Really?  So, you should not have any problem answering these questions then:

No, absolutely not.

Quote
  1) How many fields do we have in the loop circuit with two resistors?  2) What is the instantaneous power induced in the circuit by the external varying magnetic field?  3) What is the instantaneous power consumed by the circuit?  The answers are three real numbers.

1) ____ fields.

2) ____ W.

3) ____ W.

I'll be waiting!

And the answer is we are not talking about the "number of fields", whatever you mean by that, or how many watts the circuit is consuming. Or how the circuit is getting its energy, which is what I presume you mean by "induced power", although this is an interesting topic we could discuss.

And here you see the proof that you don't know what any of us, including Belcher, McDonald, Lewin, and many others cited along the thread, are talking about. If you knew, you wouldn't be making such childish assertions and non pertinent questions.

Quote
EDIT: Sorry, I got distracted.  So, do you have a degree or not?

I've already answered that question.
Title: Re: #562 – Electroboom!
Post by: thinkfat on December 01, 2021, 08:06:59 am
Regarding the "tiny voltage sources", the funny thing (which is a bit sad also) is that all the "experimental proof" by Jesse and e.g. Cyriel Mabilde are just cunning demonstrations of how to create paths that enclose a variable amount of magnetic flux. The sad part is that they're unable to see it and will keep claiming it is proof of "KVL holds". But in reality they're not measuring a gradual voltage build-up in the "Lewin Ring", but just the EMF induced in their measurement loop. This under the condition that there is only negligible current flowing through the ring and the measurement loop, of course.

Title: Re: #562 – Electroboom!
Post by: Jesse Gordon on December 01, 2021, 08:48:27 am
Well  gents,
we've come full circle ha ha ha  :-DD :-DD :-DD
Supposing you're right that I'm clueless, one thing I do know is that Team Lewin seems to think they in check.

Another victory lap.
You are restless.

Quote
Team Lewin is unwilling or unable to answer simple yes/no/sometimes to a pivotal question (reproduced below for reference) as if they believe they are in a bind.

The projection is strong in you.
Here is your answer. I wrote YES very big, so you can't miss it.
Read it all. Enjoy.

(https://i.postimg.cc/43tS42YM/KVL-works-YEEESSSSS-Sorta.jpg)
https://i.postimg.cc/43tS42YM/KVL-works-YEEESSSSS-Sorta.jpg

Again, you ignored the question I asked, asked a question of your own choosing, then answered that emphatically yes.

Why not just answer directly?

Fortunately, your above example bears enough resemblance to my diagram below that I'm just going to take you as having said YES to my question too, that in the diagram below, KVL holds -- because I'm using a toroid, and the "other side" of a toroid is inside the hole, and my KVL loop is not inside there, only some of the private internals of some of it's elements.

In other words, my entire loop is on one side of the core. Since it's a toroid, one side is outside and the other side is inside so it's easy.

(https://i.postimg.cc/jdJntBXT/20211128-121506.jpg)
https://i.postimg.cc/jdJntBXT/20211128-121506.jpg (https://i.postimg.cc/jdJntBXT/20211128-121506.jpg)

Was that so hard?

@thinkfat, See, I told you that Sredni agrees with me that the secondaries of toroidal and EI-Core transformers work just fine as lumped elements in a loop and KVL holds.

Of course before that, he said no, but I guess he changed his mind. But before that he said yes, so I guess he's changed his mind a couple times. I hear his crystalball rumbling, I bet it's gonna change again  :-DD

This also means that my "KVL Holds in an iron core transformer" video also is valid, since the loop exists entirely outside of the transformer, and the two red wires are just secondary turns:
https://youtu.be/iDWv8QJrzUo

So what's the big deal? Why all the balking about this one?


Can we all agree now that KVL holds fine when one or more of the elements is a 2-terminal secondary winding on a toroidal or EI-Core transformer?

Then we can move onto Lewin's Loop.
Title: Re: #562 – Electroboom!
Post by: Jesse Gordon on December 01, 2021, 08:51:32 am
TL;DR  I lost, but I won't admit it. I'll even claim victory, like any good KVLer.

Spoken like a true Anti-KVLer who lost but won't admit it!

Look how silly that game is. For all the effort you put in, why not just answer the following question? Sredni already agreed with me that the answer to my below question is YES!

Question: In the following diagram, in a real life physical lab test performed with real (time synchronized) volt meters with a real transformer and real resistors CONNECTED AS SHOWN, will the readings of all the volt meters sum to zero, within the accuracy and resolution limitations of the volt meters? YES or NO.

(Or if you believe SOMETIMES is the answer, then explain one scenario for a YES condition and one scenario for a NO condition WITH THE VOLT MEvTERS CONNECTED AS SHOWN - Running additional conductors through the transformer core is not allowed - nor is removing existing conductors from through the transformer core!)

(https://i.postimg.cc/jdJntBXT/20211128-121506.jpg)
Title: Re: #562 – Electroboom!
Post by: thinkfat on December 01, 2021, 09:33:24 am
(https://i.postimg.cc/jdJntBXT/20211128-121506.jpg)

You do realize that this is not a circuit that is equivalent to the "Lewin Ring", or equivalent to the circuit I gave you to solve, right?

That was my challenge to you, the task was to find the voltages across the resistors and the wires:
(https://www.eevblog.com/forum/amphour/562-electroboom!/?action=dlattach;attach=1326962;image)

And that's what you came up with:
(https://i.postimg.cc/rs3cCg2F/20211119-122528.jpg)

Which of course brings up the question why you found it necessary to add a "transformer secondary" to find the voltage across "2R".
Title: Re: #562 – Electroboom!
Post by: jesuscf on December 01, 2021, 03:04:53 pm
Regarding the "tiny voltage sources", the funny thing (which is a bit sad also) is that all the "experimental proof" by Jesse and e.g. Cyriel Mabilde are just cunning demonstrations of how to create paths that enclose a variable amount of magnetic flux. The sad part is that they're unable to see it and will keep claiming it is proof of "KVL holds". But in reality they're not measuring a gradual voltage build-up in the "Lewin Ring", but just the EMF induced in their measurement loop. This under the condition that there is only negligible current flowing through the ring and the measurement loop, of course.

How do you explain the situation where the whole ring is made of resistors, with virtually no wire?  Using the same model it is fairly easy to explain the voltage measured across each resistor:

https://www.eevblog.com/forum/amphour/562-electroboom!/msg3830852/#msg3830852 (https://www.eevblog.com/forum/amphour/562-electroboom!/msg3830852/#msg3830852)
Title: Re: #562 – Electroboom!
Post by: thinkfat on December 01, 2021, 04:07:25 pm
Regarding the "tiny voltage sources", the funny thing (which is a bit sad also) is that all the "experimental proof" by Jesse and e.g. Cyriel Mabilde are just cunning demonstrations of how to create paths that enclose a variable amount of magnetic flux. The sad part is that they're unable to see it and will keep claiming it is proof of "KVL holds". But in reality they're not measuring a gradual voltage build-up in the "Lewin Ring", but just the EMF induced in their measurement loop. This under the condition that there is only negligible current flowing through the ring and the measurement loop, of course.

How do you explain the situation where the whole ring is made of resistors, with virtually no wire?  Using the same model it is fairly easy to explain the voltage measured across each resistor:

https://www.eevblog.com/forum/amphour/562-electroboom!/msg3830852/#msg3830852 (https://www.eevblog.com/forum/amphour/562-electroboom!/msg3830852/#msg3830852)

I'd say, your calculation of V1 and V2 is 5.555mV off. Also there's no reason why the voltage across any of the resistors should be different from what Ohm's Law says.
Title: Re: #562 – Electroboom!
Post by: Kalvin on December 01, 2021, 04:26:41 pm
I would be a little concerned with my probing if I measured V1 something else than 0V across a 0 ohm wire [when |V2 + V3| > 0].

(https://i.postimg.cc/1tNL2Mmn/20211122-214739.jpg)
Title: Re: #562 – Electroboom!
Post by: thinkfat on December 01, 2021, 04:39:09 pm
I would be a little concerned with my probing if I measured V1 something else than 0V across a 0 ohm wire [when |V2 + V3| > 0].

(https://i.postimg.cc/1tNL2Mmn/20211122-214739.jpg)

Really? I wouldn't be surprised at all.
Title: Re: #562 – Electroboom!
Post by: Kalvin on December 01, 2021, 04:54:13 pm
I would be a little concerned with my probing if I measured V1 something else than 0V across a 0 ohm wire [when |V2 + V3| > 0].
<removed the image>

Really? I wouldn't be surprised at all.

I would, because that is violating the Ohm's law (U = I*R, and when R is 0 ohms, the voltage should be 0 too), which means that my probing is picking up some interference from some magnetic field affecting my measurements.
Title: Re: #562 – Electroboom!
Post by: Sredni on December 01, 2021, 05:36:32 pm
I would be a little concerned with my probing if I measured V1 something else than 0V across a 0 ohm wire [when |V2 + V3| > 0].
<removed the image>

Really? I wouldn't be surprised at all.

I would, because that is violating the Ohm's law (U = I*R, and when R is 0 ohms, the voltage should be 0 too), which means that my probing is picking up some interference from some magnetic field affecting my measurements.

The reason you are surprised is because you gazed into the aby-- into the forbidden zone where the magnetic flux changes.
There, KVL dies.
(To clarify: voltage becomes path dependent and so you can have zero voltage ALONG the conductor in compliance with Ohm's law, and nonzero voltage ACROSS the terminals as you can measure with your voltmeter or oscilloscope)

But if you hide that part of space inside a black box, you will see a nonzero voltage 'across' a generator's terminals. Not surprising at all.

This is the point KVLers seem unable to understand. KVL dies right inside every transformer's secondary. We choose not to see that by not looking inside. But it's all just make-believe.
Title: Re: #562 – Electroboom!
Post by: Kalvin on December 01, 2021, 05:59:05 pm
I would be a little concerned with my probing if I measured V1 something else than 0V across a 0 ohm wire [when |V2 + V3| > 0].
<removed the image>

Really? I wouldn't be surprised at all.

I would, because that is violating the Ohm's law (U = I*R, and when R is 0 ohms, the voltage should be 0 too), which means that my probing is picking up some interference from some magnetic field affecting my measurements.

The reason you are surprised is because you gazed into the aby-- into the forbidden zone where the magnetic flux changes.
There, KVL dies.
(To clarify: voltage becomes path dependent and so you can have zero voltage ALONG the conductor in compliance with Ohm's law, and nonzero voltage ACROSS the terminals as you can measure with your voltmeter or oscilloscope)

But if you hide that part of space inside a black box, you will see a nonzero voltage 'across' a generator's terminals. Not surprising at all.

This is the point KVLers seem unable to understand. KVL dies right inside every transformer's secondary. We choose not to see that by not looking inside. But it's all just make-believe.

I just wanted to point this out because in this particular case I know that I am measuring voltage V1 across a 0 ohm wire, but my instrument is showing some other voltage than zero violating the Ohm's law, which is a clear indication that there is something wrong in my probing, and the measurement setup is picking up interference from some magnetic field affecting my measurement.
Title: Re: #562 – Electroboom!
Post by: Sredni on December 01, 2021, 06:26:00 pm
I just wanted to point this out because in this particular case I know that I am measuring voltage V1 across a 0 ohm wire, but my instrument is showing some other voltage than zero violating the Ohm's law, which is a clear indication that there is something wrong in my probing, and the measurement setup is picking up interference from some magnetic field affecting my measurement.

What I am saying is that

1. That voltage is NOT interference: it's exactly what that piece of system is supposed to do (hence your probing is correct: you put a voltmeter across the secondary of a transformer and the 12V you measure are not 'interference', despite the fact that if you follow the magnet wires inside the device all you see is... wire)
2. Ohm's law is obeyed (if you use the correct physics, of course)

Title: Re: #562 – Electroboom!
Post by: Kalvin on December 01, 2021, 07:40:52 pm
I just wanted to point this out because in this particular case I know that I am measuring voltage V1 across a 0 ohm wire, but my instrument is showing some other voltage than zero violating the Ohm's law, which is a clear indication that there is something wrong in my probing, and the measurement setup is picking up interference from some magnetic field affecting my measurement.

What I am saying is that

1. That voltage is NOT interference: it's exactly what that piece of system is supposed to do (hence your probing is correct: you put a voltmeter across the secondary of a transformer and the 12V you measure are not 'interference', despite the fact that if you follow the magnet wires inside the device all you see is... wire)
2. Ohm's law is obeyed (if you use the correct physics, of course)

Yes, I do believe in transformers, even transformers with a single-turn secondary. :) You are totally right.
Title: Re: #562 – Electroboom!
Post by: jesuscf on December 01, 2021, 07:52:11 pm
Regarding the "tiny voltage sources", the funny thing (which is a bit sad also) is that all the "experimental proof" by Jesse and e.g. Cyriel Mabilde are just cunning demonstrations of how to create paths that enclose a variable amount of magnetic flux. The sad part is that they're unable to see it and will keep claiming it is proof of "KVL holds". But in reality they're not measuring a gradual voltage build-up in the "Lewin Ring", but just the EMF induced in their measurement loop. This under the condition that there is only negligible current flowing through the ring and the measurement loop, of course.

How do you explain the situation where the whole ring is made of resistors, with virtually no wire?  Using the same model it is fairly easy to explain the voltage measured across each resistor:

https://www.eevblog.com/forum/amphour/562-electroboom!/msg3830852/#msg3830852 (https://www.eevblog.com/forum/amphour/562-electroboom!/msg3830852/#msg3830852)

I'd say, your calculation of V1 and V2 is 5.555mV off. Also there's no reason why the voltage across any of the resistors should be different from what Ohm's Law says.

If that is the case, where is the induced EMF coming from?  Remember, there is no wire in the loop, only resistors, and the only access points to the circuit are the terminals of the resistors.
Title: Re: #562 – Electroboom!
Post by: thinkfat on December 01, 2021, 08:19:31 pm
Regarding the "tiny voltage sources", the funny thing (which is a bit sad also) is that all the "experimental proof" by Jesse and e.g. Cyriel Mabilde are just cunning demonstrations of how to create paths that enclose a variable amount of magnetic flux. The sad part is that they're unable to see it and will keep claiming it is proof of "KVL holds". But in reality they're not measuring a gradual voltage build-up in the "Lewin Ring", but just the EMF induced in their measurement loop. This under the condition that there is only negligible current flowing through the ring and the measurement loop, of course.

How do you explain the situation where the whole ring is made of resistors, with virtually no wire?  Using the same model it is fairly easy to explain the voltage measured across each resistor:

https://www.eevblog.com/forum/amphour/562-electroboom!/msg3830852/#msg3830852 (https://www.eevblog.com/forum/amphour/562-electroboom!/msg3830852/#msg3830852)

I'd say, your calculation of V1 and V2 is 5.555mV off. Also there's no reason why the voltage across any of the resistors should be different from what Ohm's Law says.

If that is the case, where is the induced EMF coming from?  Remember, there is no wire in the loop, only resistors, and the only access points to the circuit are the terminals of the resistors.

I'm not quite sure why the resistance along the path would make a difference. A series of resistors is not fundamentally different from a wire with the same resistance.
Title: Re: #562 – Electroboom!
Post by: jesuscf on December 01, 2021, 08:34:10 pm
Regarding the "tiny voltage sources", the funny thing (which is a bit sad also) is that all the "experimental proof" by Jesse and e.g. Cyriel Mabilde are just cunning demonstrations of how to create paths that enclose a variable amount of magnetic flux. The sad part is that they're unable to see it and will keep claiming it is proof of "KVL holds". But in reality they're not measuring a gradual voltage build-up in the "Lewin Ring", but just the EMF induced in their measurement loop. This under the condition that there is only negligible current flowing through the ring and the measurement loop, of course.

How do you explain the situation where the whole ring is made of resistors, with virtually no wire?  Using the same model it is fairly easy to explain the voltage measured across each resistor:

https://www.eevblog.com/forum/amphour/562-electroboom!/msg3830852/#msg3830852 (https://www.eevblog.com/forum/amphour/562-electroboom!/msg3830852/#msg3830852)

I'd say, your calculation of V1 and V2 is 5.555mV off. Also there's no reason why the voltage across any of the resistors should be different from what Ohm's Law says.

If that is the case, where is the induced EMF coming from?  Remember, there is no wire in the loop, only resistors, and the only access points to the circuit are the terminals of the resistors.

I'm not quite sure why the resistance along the path would make a difference. A series of resistors is not fundamentally different from a wire with the same resistance.

The idea of making the ring out of resistors, no wires, is to show that a resistor will behave as an induced voltage source in series with a resistance.   The net voltage across the terminals of the resistor is then the fraction of the total induced EMF (proportional to the total EMF and ratio of the length of the resistor vs the perimeter of the ring) minus the drop due to Ohms law (proportional to the resistance of the resistor and the current in the loop).
Title: Re: #562 – Electroboom!
Post by: Sredni on December 01, 2021, 09:14:59 pm
How do you explain the situation where the whole ring is made of resistors, with virtually no wire?
...
If that is the case, where is the induced EMF coming from?  Remember, there is no wire in the loop, only resistors, and the only access points to the circuit are the terminals of the resistors.

What's the matter, pretty boy? You can't find your EMF?

(Sorry, I could not resist)

Title: Re: #562 – Electroboom!
Post by: jesuscf on December 01, 2021, 09:29:35 pm
How do you explain the situation where the whole ring is made of resistors, with virtually no wire?
...
If that is the case, where is the induced EMF coming from?  Remember, there is no wire in the loop, only resistors, and the only access points to the circuit are the terminals of the resistors.

What's the matter, pretty boy? You can't find your EMF?

(Sorry, I could not resist)

I am glad I have this message to reply to!  Why?  Because I just found the solution to Lewin's problem in an Electromagnetics book!  And it doesn't look good at all for team Lewin.  The book is "Electromagnetics" by Branislav M. Notaros, pages 279 and 280 (the edition I have is from 2011).  The example is 6.6.  I have attached the pages to this message.  Sredni, now you have a bibliographical reference that can teach you how to calculate the voltage VAD.  Don't forget to pay attention to Figure 6.10(b)!

EDIT: fixed page numbers
Title: Re: #562 – Electroboom!
Post by: bsfeechannel on December 01, 2021, 10:16:17 pm
I am glad I have this message to reply to!  Why?  Because I just found the solution to Lewin's problem in an Electromagnetics book!  And it doesn't look good at all for team Lewin.  The book is "Electromagnetics" by Branislav M. Notaros, pages 279 and 280 (the edition I have is from 2011).  The example is 6.6.  I have attached the pages to this message.  Sredni, now you have a bibliographical reference that can teach you how to calculate the voltage VAD.  Don't forget to pay attention to Figure 6.10(b)!

EDIT: fixed page numbers

Notaros says that if σ1 = σ2, then vMN = 0. Incredible! You have two wire resistors with zero volts across their terminals and a current that is iind = eind / (R1 + R2).

Oh yeah, of course. What Notaros didn't tell you, because he obviously presumes you already understand the effect of non-conservative electric fields, is that this voltage is calculated along a straight line between points M and N.

Once defined this path you can model it using mundane circuit theory. No problem. No one is against that.

Build the circuit, immerse it in the varying magnetic field, place your meter right in the middle with the probes stretched along a straight line between M and N and you'll find vMN he calculated.
Title: Re: #562 – Electroboom!
Post by: Sredni on December 01, 2021, 10:31:23 pm
How do you explain the situation where the whole ring is made of resistors, with virtually no wire?
...
If that is the case, where is the induced EMF coming from?  Remember, there is no wire in the loop, only resistors, and the only access points to the circuit are the terminals of the resistors.

What's the matter, pretty boy? You can't find your EMF?

(Sorry, I could not resist)

I am glad I have this message to reply to!  Why?  Because I just found the solution to Lewin's problem in an Electromagnetics book!  And it doesn't look good at all for team Lewin.  The book is "Electromagnetics" by Branislav M. Notaros, pages 279 and 280 (the edition I have is from 2011).  The example is 6.6.  I have attached the pages to this message.  Sredni, now you have a bibliographical reference that can teach you how to calculate the voltage VAD.  Don't forget to pay attention to Figure 6.10(b)!

Soooo...  the EMF is located on top of the resistors, it seems. Half just above R1, and half just above R2. How many centimeters, exactly? The text does not say. Can you locate with a bit more accuracy? No?
Or maybe...
Maybe that's the "equivalent circuit" that allows you to "solve the problem from the circuit theory point of view" and that is one of the introductory textbooks that do not explain clearly to their easily distracted audience what they intend for V. Oh, wait, but it does explain what V is! Page 269, eq. 6.18

Eq = - grad V

(Eq is what I call Ecoul) and V is... the electric scalar potential. Only half of the potentials required to describe the total electric field. And the text also says so explicitly on page 277, formula 6.43

E(t) = - dA/dt - grad V

"We see that both potentials are needed for E..."
(the same expression I used to express Etot = Eind + Ecoul, even if recently I decided to call the scalar electric potential phi, instead of V - exactly to avoid this kind of confusion you are having)

So...

where is exactly the EMF, again?

(Lewin problem is solved as an exercise on Purcell, Morin: Berkeley Physics vol 2, Electricity and Magnetism 3rd edition)
Title: Re: #562 – Electroboom!
Post by: jesuscf on December 02, 2021, 12:53:18 am
I am glad I have this message to reply to!  Why?  Because I just found the solution to Lewin's problem in an Electromagnetics book!  And it doesn't look good at all for team Lewin.  The book is "Electromagnetics" by Branislav M. Notaros, pages 279 and 280 (the edition I have is from 2011).  The example is 6.6.  I have attached the pages to this message.  Sredni, now you have a bibliographical reference that can teach you how to calculate the voltage VAD.  Don't forget to pay attention to Figure 6.10(b)!

EDIT: fixed page numbers

Notaros says that if σ1 = σ2, then vMN = 0. Incredible! You have two wire resistors with zero volts across their terminals and a current that is iind = eind / (R1 + R2).

Oh yeah, of course. What Notaros didn't tell you, because he obviously presumes you already understand the effect of non-conservative electric fields, is that this voltage is calculated along a straight line between points M and N.

Once defined this path you can model it using mundane circuit theory. No problem. No one is against that.

Build the circuit, immerse it in the varying magnetic field, place your meter right in the middle with the probes stretched along a straight line between M and N and you'll find vMN he calculated.

Oh boy bsfeechannel, you are very special, but not in a good way!  In figure 6.10(b) what is the voltage between nodes vMN if R1 is equal to R2?
Title: Re: #562 – Electroboom!
Post by: jesuscf on December 02, 2021, 01:09:45 am
How do you explain the situation where the whole ring is made of resistors, with virtually no wire?
...
If that is the case, where is the induced EMF coming from?  Remember, there is no wire in the loop, only resistors, and the only access points to the circuit are the terminals of the resistors.

What's the matter, pretty boy? You can't find your EMF?

(Sorry, I could not resist)

I am glad I have this message to reply to!  Why?  Because I just found the solution to Lewin's problem in an Electromagnetics book!  And it doesn't look good at all for team Lewin.  The book is "Electromagnetics" by Branislav M. Notaros, pages 279 and 280 (the edition I have is from 2011).  The example is 6.6.  I have attached the pages to this message.  Sredni, now you have a bibliographical reference that can teach you how to calculate the voltage VAD.  Don't forget to pay attention to Figure 6.10(b)!

Soooo...  the EMF is located on top of the resistors, it seems. Half just above R1, and half just above R2. How many centimeters, exactly? The text does not say. Can you locate with a bit more accuracy? No?
Or maybe...
Maybe that's the "equivalent circuit" that allows you to "solve the problem from the circuit theory point of view" and that is one of the introductory textbooks that do not explain clearly to their easily distracted audience what they intend for V. Oh, wait, but it does explain what V is! Page 269, eq. 6.18

Eq = - grad V

(Eq is what I call Ecoul) and V is... the electric scalar potential. Only half of the potentials required to describe the total electric field. And the text also says so explicitly on page 277, formula 6.43

E(t) = - dA/dt - grad V

"We see that both potentials are needed for E..."
(the same expression I used to express Etot = Eind + Ecoul, even if recently I decided to call the scalar electric potential phi, instead of V - exactly to avoid this kind of confusion you are having)

So...

where is exactly the EMF, again?

(Lewin problem is solved as an exercise on Purcell, Morin: Berkeley Physics vol 2, Electricity and Magnetism 3rd edition)

Can you calculate VAD yet?  The book is giving you the KVL solution, you just have to put the values and evaluate!  Just be careful, because in Lewin's original problem, the one with one loop, the positions of R1 and R2 are swapped.  Let me fix it for you:

\$
V_{AD}  = \frac{{\left( {R_2  - R_1 } \right)EMF}}{{2\left( {R_1  + R_2 } \right)}} = \frac{{\left( {900\Omega  - 100\Omega } \right)1V}}{{2\left( {100\Omega  + 900\Omega } \right)}} = ?
\$

Or you can take a look on how I solved it also using KVL:

https://www.eevblog.com/forum/amphour/562-electroboom!/msg3828206/#msg3828206 (https://www.eevblog.com/forum/amphour/562-electroboom!/msg3828206/#msg3828206)



Title: Re: #562 – Electroboom!
Post by: bsfeechannel on December 02, 2021, 01:13:52 am
Oh boy bsfeechannel, you are very special, but not in a good way!  In figure 6.10(b) what is the voltage between nodes vMN if R1 is equal to R2?

Who cares about figure 6.10(b)? It's an equivalent circuit. It's an imaginary construct, a math trick. It doesn't exist.

The real circuit is described by figure 6.10(a). There is where you'll come a gutser.

Title: Re: #562 – Electroboom!
Post by: jesuscf on December 02, 2021, 01:27:11 am
Oh boy bsfeechannel, you are very special, but not in a good way!  In figure 6.10(b) what is the voltage between nodes vMN if R1 is equal to R2?

Who cares about figure 6.10(b)? It's an equivalent circuit. It's an imaginary construct, a math trick. It doesn't exist.

The real circuit is described by figure 6.10(a). There is where you'll come a gutser.

I see you just moved to Egypt, living by the shores of the Nile! 
Title: Re: #562 – Electroboom!
Post by: thinkfat on December 02, 2021, 08:35:31 am
https://www.youtube.com/watch?v=xMePTKuAixE (https://www.youtube.com/watch?v=xMePTKuAixE)

PS: "Team KVL", don't get your hopes up too early ;)
Title: Re: #562 – Electroboom!
Post by: jesuscf on December 02, 2021, 09:31:07 am
https://www.youtube.com/watch?v=xMePTKuAixE (https://www.youtube.com/watch?v=xMePTKuAixE)

PS: "Team KVL", don't get your hopes up too early ;)

The video is correct.  He even calculates the correct and unique voltage between VAD (VJX at 33:38, something Lewin was unable to do), followed with an explanation of what happens if use  KVL without including the induced EMF (what Lewin did).  So, what is your point exactly?
Title: Re: #562 – Electroboom!
Post by: Kalvin on December 02, 2021, 09:37:09 am
I just wanted to point this out because in this particular case I know that I am measuring voltage V1 across a 0 ohm wire, but my instrument is showing some other voltage than zero violating the Ohm's law, which is a clear indication that there is something wrong in my probing, and the measurement setup is picking up interference from some magnetic field affecting my measurement.

What I am saying is that

1. That voltage is NOT interference: it's exactly what that piece of system is supposed to do (hence your probing is correct: you put a voltmeter across the secondary of a transformer and the 12V you measure are not 'interference', despite the fact that if you follow the magnet wires inside the device all you see is... wire)
2. Ohm's law is obeyed (if you use the correct physics, of course)

This is what I came up with when thinking about this circuit and when KVL holds and doesn't hold.

In figure a) the circuit is inside a magnetic field, the voltage between points A and D depends on path, it is not possible to create a lumped model for the source creating the 1mA current, and KVL doesn't hold.

In figure b) the circuit is only partially inside a magnetic field creating a transformer with a single-turn secondary, thus it is possible to create a lumped model for the source creating the 1mA current like in figure c, and KVL holds.

Figure c is a lumped model for figure b with a (voltage) source creating the 1mA current when the magnetic field is increasing, and KVL holds.

Is my reasoning correct?

Edit: Added some clarifications.
Title: Re: #562 – Electroboom!
Post by: thinkfat on December 02, 2021, 05:46:49 pm
The video is correct.  He even calculates the correct and unique voltage between VAD (VJX at 33:38, something Lewin was unable to do), followed with an explanation of what happens if use  KVL without including the induced EMF (what Lewin did).  So, what is your point exactly?

Well, it's a Nothing Burger, how our host would be calling it. While you can calculate a voltage for Vad, it is more mathturbation than anything else.

It is not trivial to compute for other than artificial setups with simple paths, perfect symmetry or at least uniformity of the electric field, and actually observing it is also quite complicated, because all measurements would again be taken along paths through a non-conservative electric field, which adds another dimension of error.

So, you'd compute a value for Vad, making various assumptions about the fields involved, then calculate a path based on the same assumptions, then try to make your measurements and calculations match in reality. But at no point you would be sure if any of it is correct. Plus there are geometries where it is not possible to measure it.

The same guy has a good video on the merits of PD to "salvage KVL" on his channel. It is quite thorough IMHO, though a bit long winded, but still worth watching in detail.

https://youtu.be/I1kYKF2x9Ns


Title: Re: #562 – Electroboom!
Post by: jesuscf on December 02, 2021, 06:42:11 pm
The video is correct.  He even calculates the correct and unique voltage between VAD (VJX at 33:38, something Lewin was unable to do), followed with an explanation of what happens if use  KVL without including the induced EMF (what Lewin did).  So, what is your point exactly?

Well, it's a Nothing Burger, how our host would be calling it. While you can calculate a voltage for Vad, it is more mathturbation than anything else.

It is not trivial to compute for other than artificial setups with simple paths, perfect symmetry or at least uniformity of the electric field, and actually observing it is also quite complicated, because all measurements would again be taken along paths through a non-conservative electric field, which adds another dimension of error.

So, you'd compute a value for Vad, making various assumptions about the fields involved, then calculate a path based on the same assumptions, then try to make your measurements and calculations match in reality. But at no point you would be sure if any of it is correct. Plus there are geometries where it is not possible to measure it.

The same guy has a good video on the merits of PD to "salvage KVL" on his channel. It is quite thorough IMHO, though a bit long winded, but still worth watching in detail.

https://youtu.be/I1kYKF2x9Ns (https://youtu.be/I1kYKF2x9Ns)

Although the video you linked is correct, I think it goes in to too much detail and sometimes it is hard to decipher what he tries to explain.   In my opinion this video from Bob Duhamel from RSD Academy explains it better:

https://www.youtube.com/watch?v=aQB871ku6a8 (https://www.youtube.com/watch?v=aQB871ku6a8)


Title: Re: #562 – Electroboom!
Post by: bsfeechannel on December 02, 2021, 07:40:23 pm
https://www.youtube.com/watch?v=xMePTKuAixE (https://www.youtube.com/watch?v=xMePTKuAixE)

PS: "Team KVL", don't get your hopes up too early ;)

The video is correct.  He even calculates the correct and unique voltage between VAD (VJX at 33:38, something Lewin was unable to do), followed with an explanation of what happens if use  KVL without including the induced EMF (what Lewin did).  So, what is your point exactly?

The point is this:

(https://www.eevblog.com/forum/amphour/562-electroboom!/?action=dlattach;attach=1338011;image)

What he calculated was the scalar potential difference between those two points and he points out [pun unintended] how difficult or even impossible would be to measure it. The voltages measured by the voltmeters in his calculations match exactly what Lewin predicted and then he, Mehdi, and many others measured: two different voltages.

So, if you're saying the video is correct, this means that you are recognizing that Lewin is absolutely right and Mehdi is dead wrong. Congratulations.


Title: Re: #562 – Electroboom!
Post by: bsfeechannel on December 02, 2021, 08:19:12 pm
Although the video you linked is correct, I think it goes in to too much detail and sometimes it is hard to decipher what he tries to explain.   In my opinion this video from Bob Duhamel from RSD Academy explains it better:

https://www.youtube.com/watch?v=aQB871ku6a8 (https://www.youtube.com/watch?v=aQB871ku6a8)

Yeah. The video thinkfat posted is absolutely correct, thank you very much. What is not correct is the video by this RSD Academy guy. Stay away from him. He doesn't understand electromagnetism, preaches pseudoscience, and wants your money for his Patreon account.

The quackery of his explanation is evident when @3:41 he said the meter will measure zero volts. To justify this he tries to explain how a piece of static wire under the effect of an electric field produced by a varying magnetic field is going to generate voltages inside it. He forgot that the charges accumulated at the ends of the wire will generate an electric field that's is going to be in the opposite direction and equal intensity to the induced field and the resultant will be zero.

For this guy the Coulumb force is like the gravitational field for Wile E. Coyote: it only works when it is amusing.



Title: Re: #562 – Electroboom!
Post by: jesuscf on December 02, 2021, 08:33:17 pm
https://www.youtube.com/watch?v=xMePTKuAixE (https://www.youtube.com/watch?v=xMePTKuAixE)

PS: "Team KVL", don't get your hopes up too early ;)

The video is correct.  He even calculates the correct and unique voltage between VAD (VJX at 33:38, something Lewin was unable to do), followed with an explanation of what happens if use  KVL without including the induced EMF (what Lewin did).  So, what is your point exactly?

The point is this:

(https://www.eevblog.com/forum/amphour/562-electroboom!/?action=dlattach;attach=1338011;image)

What he calculated was the scalar potential difference between those two points and he points out [pun unintended] how difficult or even impossible would be to measure it. The voltages measured by the voltmeters in his calculations match exactly what Lewin predicted and then he, Mehdi, and many others measured: two different voltages.

So, if you're saying the video is correct, this means that you are recognizing that Lewin is absolutely right and Mehdi is dead wrong. Congratulations.

Just carry on watching a bit longer, and try to understand what he is saying!
Title: Re: #562 – Electroboom!
Post by: Sredni on December 02, 2021, 09:09:58 pm
https://www.youtube.com/watch?v=xMePTKuAixE (https://www.youtube.com/watch?v=xMePTKuAixE)

PS: "Team KVL", don't get your hopes up too early ;)

The video is correct.  He even calculates the correct and unique voltage between VAD (VJX at 33:38, something Lewin was unable to do), followed with an explanation of what happens if use  KVL without including the induced EMF (what Lewin did).  So, what is your point exactly?

The point is this:

(https://www.eevblog.com/forum/amphour/562-electroboom!/?action=dlattach;attach=1338011;image)

What he calculated was the scalar potential difference between those two points and he points out [pun unintended] how difficult or even impossible would be to measure it. The voltages measured by the voltmeters in his calculations match exactly what Lewin predicted and then he, Mehdi, and many others measured: two different voltages.

So, if you're saying the video is correct, this means that you are recognizing that Lewin is absolutely right and Mehdi is dead wrong. Congratulations.

Just carry on watching a bit longer, and try to understand what he is saying!

Dude, that's Trevor Kearney. If Jesse hadn't flown away to another galaxy to avoid answering my question on the circuit path for Lewin's ring, he would tell you Trevor is "Armchair Physics Nobel Prize" number one. He's probably one of the most active people on YouTube trying to debunk Mehdi and the other KVLers.  :palm:
Title: Re: #562 – Electroboom!
Post by: bsfeechannel on December 02, 2021, 09:35:48 pm
I just wanted to point this out because in this particular case I know that I am measuring voltage V1 across a 0 ohm wire, but my instrument is showing some other voltage than zero violating the Ohm's law, which is a clear indication that there is something wrong in my probing, and the measurement setup is picking up interference from some magnetic field affecting my measurement.

What I am saying is that

1. That voltage is NOT interference: it's exactly what that piece of system is supposed to do (hence your probing is correct: you put a voltmeter across the secondary of a transformer and the 12V you measure are not 'interference', despite the fact that if you follow the magnet wires inside the device all you see is... wire)
2. Ohm's law is obeyed (if you use the correct physics, of course)

This is what I came up with when thinking about this circuit and when KVL holds and doesn't hold.

In figure a) the circuit is inside a magnetic field, the voltage between points A and D depends on path, it is not possible to create a lumped model for the source creating the 1mA current, and KVL doesn't hold.

In figure b) the circuit is only partially inside a magnetic field creating a transformer with a single-turn secondary, thus it is possible to create a lumped model for the source creating the 1mA current like in figure c, and KVL holds.

Figure c is a lumped model for figure b with a (voltage) source creating the 1mA current when the magnetic field is increasing, and KVL holds.

(https://www.eevblog.com/forum/amphour/562-electroboom!/?action=dlattach;attach=1337789;image)

Is my reasoning correct?

Edit: Added some clarifications.

Absolutely correct.
Title: Re: #562 – Electroboom!
Post by: jesuscf on December 02, 2021, 09:52:24 pm
https://www.youtube.com/watch?v=xMePTKuAixE (https://www.youtube.com/watch?v=xMePTKuAixE)

PS: "Team KVL", don't get your hopes up too early ;)

The video is correct.  He even calculates the correct and unique voltage between VAD (VJX at 33:38, something Lewin was unable to do), followed with an explanation of what happens if use  KVL without including the induced EMF (what Lewin did).  So, what is your point exactly?

The point is this:

(https://www.eevblog.com/forum/amphour/562-electroboom!/?action=dlattach;attach=1338011;image)

What he calculated was the scalar potential difference between those two points and he points out [pun unintended] how difficult or even impossible would be to measure it. The voltages measured by the voltmeters in his calculations match exactly what Lewin predicted and then he, Mehdi, and many others measured: two different voltages.

So, if you're saying the video is correct, this means that you are recognizing that Lewin is absolutely right and Mehdi is dead wrong. Congratulations.

Just carry on watching a bit longer, and try to understand what he is saying!

Dude, that's Trevor Kearney. If Jesse hadn't flown away to another galaxy to avoid answering my question on the circuit path for Lewin's ring, he would tell you Trevor is "Armchair Physics Nobel Prize" number one. He's probably one of the most active people on YouTube trying to debunk Mehdi and the other KVLers.  :palm:

That guy got the correct value for VAD.  Did you get the correct value for VAD yet?
Title: Re: #562 – Electroboom!
Post by: bsfeechannel on December 02, 2021, 11:49:12 pm
Dude, that's Trevor Kearney. If Jesse hadn't flown away to another galaxy to avoid answering my question on the circuit path for Lewin's ring, he would tell you Trevor is "Armchair Physics Nobel Prize" number one. He's probably one of the most active people on YouTube trying to debunk Mehdi and the other KVLers.  :palm:

Isn't it hilarious that KVLers be so obtuse that they can't even recognize when a video was made to debunk their claims, due to their absolute lack of understanding of what is being discussed?

Congratulations, Mehdi Sadaghdar! Look what you've done! You've spawned a whole bunch of brain-damaged mock engineers like yourself.
Title: Re: #562 – Electroboom!
Post by: jesuscf on December 02, 2021, 11:54:38 pm
Dude, that's Trevor Kearney. If Jesse hadn't flown away to another galaxy to avoid answering my question on the circuit path for Lewin's ring, he would tell you Trevor is "Armchair Physics Nobel Prize" number one. He's probably one of the most active people on YouTube trying to debunk Mehdi and the other KVLers.  :palm:

Isn't it hilarious that KVLers be so obtuse that they can't even recognize when a video was made to debunk their claims, due to their absolute lack of understanding of what is being discussed?

Congratulations, Mehdi Sadaghdar! Look what you've done! You've spawned a whole bunch of brain-damaged mock engineers like yourself.

You didn't watch the video, or if you did, you didn't understand it, which will not surprise me!
Title: Re: #562 – Electroboom!
Post by: Sredni on December 03, 2021, 01:38:18 am
In figure a) the circuit is inside a magnetic field, the voltage between points A and D depends on path, it is not possible to create a lumped model for the source creating the 1mA current, and KVL doesn't hold.

In figure b) the circuit is only partially inside a magnetic field creating a transformer with a single-turn secondary, thus it is possible to create a lumped model for the source creating the 1mA current like in figure c, and KVL holds.

Figure c is a lumped model for figure b with a (voltage) source creating the 1mA current when the magnetic field is increasing, and KVL holds.

Is my reasoning correct?

Yes. There is only one fine point that require some annotations.
Technically the lumped circuit model requires for the circuit to be shrunk (or to be 'shrinkable') to a point. It's a way to make sure that the circuit path itself does not participate in any phenomena that is related to geometry. The variable flux 'exploited' by magnetic components requires a finite area - you cannot shrink your circuit to a point if it has to contain a finite area; likewise, charge separation and accumulation that is exploited by electrostatic devices requires... space. So, if your circuit path can be thought as an elastic band that can be shrunk to a point, you are guaranteed you exclude these effects from the 'inside' of your circuit - they can only be 'provided' by the components.

This is why when dealing with lumped circuits, the lumped components are sometimes shown to offer their terminals close together. All the pairs of points on the circuit path corresponding to every lumped component do not interfere with the shrinking of the circuit path. In reality, if you have a component that doesn't offer terminals side by side you can choose: extend the component's terminals or extend the circuit as I have shown in a post a few pages prior. This leaves some leverage to argue about the lumpiness of either the component or the circuit. Considering components with terminals separated by a very small gap avoids this controversy entirely.

Then, of course the symbol of a battery for the lumped component signifies that you are considering a specific instant in time (the linearly increasing flux is a didactic tool). I too, prefer to use battery to represent the contribute of the inductive EMF, but sometimes it is better to remind it.
Title: Re: #562 – Electroboom!
Post by: Kalvin on December 03, 2021, 12:53:16 pm
Here is the circuit from Lewin's video with 1V voltage source.

If the KVL holds, it possible to split the original circuit in half at D and A by inserting two antiparallel 1mA current sources between nodes D and A. After split, the currents and the voltages of the original circuit remain unchanged.

If the KVL holds, it possible to split the original circuit in half at D and A by inserting a voltage source between nodes D and A with value V(D,A) because there is no current flowing through the new voltage source. After split, the currents and the voltages of the original circuit remain unchanged.

It can be seen that KVL holds and it is possible to spilt the circuit in half at D and A.

The 1V voltage source can be located some other point in the original circuit, which will affect the equivalent circuit values, but KVL will still hold.

(https://www.eevblog.com/forum/amphour/562-electroboom!/?action=dlattach;attach=1338839;image)

Here is the circuit from Lewin's video when the solenoid is generating 1mA current flowing in the circuit. It can be seen that KVL fails when the circuit contains of non-conservative fields, and thus it is not possible to split the circuit at D and A.

(https://www.eevblog.com/forum/amphour/562-electroboom!/?action=dlattach;attach=1338845;image)
Title: Re: #562 – Electroboom!
Post by: Kalvin on December 03, 2021, 01:10:53 pm
The simple circuit analysis above suggests that Dr. Lewin had it right, and there were no probing errors involved, even if the results look counterintuitive. KVL fails when there are non-conservative fields present.
Title: Re: #562 – Electroboom!
Post by: jesuscf on December 03, 2021, 03:07:21 pm
Here is the circuit from Lewin's video with 1V voltage source.

If the KVL holds, it possible to split the original circuit in half at D and A by inserting two antiparallel 1mA current sources between nodes D and A. After split, the currents and the voltages of the original circuit remain unchanged.

If the KVL holds, it possible to split the original circuit in half at D and A by inserting a voltage source between nodes D and A with value V(D,A) because there is no current flowing through the new voltage source. After split, the currents and the voltages of the original circuit remain unchanged.

It can be seen that KVL holds and it is possible to spilt the circuit in half at D and A.

The 1V voltage source can be located some other point in the original circuit, which will affect the equivalent circuit values, but KVL will still hold.

(https://www.eevblog.com/forum/amphour/562-electroboom!/?action=dlattach;attach=1338839;image)

Here is the circuit from Lewin's video when the solenoid is generating 1mA current flowing in the circuit. It can be seen that KVL fails when the circuit contains of non-conservative fields, and thus it is not possible to split the circuit at D and A.

(https://www.eevblog.com/forum/amphour/562-electroboom!/?action=dlattach;attach=1338845;image)

I see, you too, are confused with the 'non-conservative fields' statement.  Although it may appear that in the circuit the electric field is non-conservative because it is changing over time, so it is the magnetic field.   What happens is that the the total energy in the circuit is conservative: all energy added to the circuit is consumed by the circuit.  From the circuital point of view is easier to work with instantaneous power than energy.  For example at time t when the EMF is 1V the calculated current is 1mA, then: EMF*I=I2(R1+R2).  The external varying magnetic field is adding 1mW to the circuit and the circuit is consuming 1mW.  The fields in the circuit are conservative.
Title: Re: #562 – Electroboom!
Post by: thinkfat on December 03, 2021, 03:51:15 pm
I see, you too, are confused with the 'non-conservative fields' statement.  Although it may appear that in the circuit the electric field is non-conservative because it is changing over time, so it is the magnetic field.   What happens is that the the total energy in the circuit is conservative: all energy added to the circuit is consumed by the circuit.  From the circuital point of view is easier to work with instantaneous power than energy.  For example at time t when the EMF is 1V the calculated current is 1mA, then: EMF*I=I2(R1+R2).  The external varying magnetic field is adding 1mW to the circuit and the circuit is consuming 1mW.  The fields in the circuit are conservative.
Emphasis mine.

The "fields in the circuit" are not to be confused with the fields outside of the circuit. The "non-conservative electric field" described by the Maxwell-Faraday equation is not in the circuit.

PS: "change over time" is not a sign of non-conservativeness. The "curl" is.
Title: Re: #562 – Electroboom!
Post by: bsfeechannel on December 03, 2021, 05:31:13 pm
I see, you too, are confused with the 'non-conservative fields' statement.  Although it may appear that in the circuit the electric field is non-conservative because it is changing over time, so it is the magnetic field.   What happens is that the the total energy in the circuit is conservative: all energy added to the circuit is consumed by the circuit.  From the circuital point of view is easier to work with instantaneous power than energy.  For example at time t when the EMF is 1V the calculated current is 1mA, then: EMF*I=I2(R1+R2).  The external varying magnetic field is adding 1mW to the circuit and the circuit is consuming 1mW.  The fields in the circuit are conservative.

He understands what a non conservative field is. You don't. That's why you think he's confused.
Title: Re: #562 – Electroboom!
Post by: jesuscf on December 03, 2021, 07:20:32 pm
I see, you too, are confused with the 'non-conservative fields' statement.  Although it may appear that in the circuit the electric field is non-conservative because it is changing over time, so it is the magnetic field.   What happens is that the the total energy in the circuit is conservative: all energy added to the circuit is consumed by the circuit.  From the circuital point of view is easier to work with instantaneous power than energy.  For example at time t when the EMF is 1V the calculated current is 1mA, then: EMF*I=I2(R1+R2).  The external varying magnetic field is adding 1mW to the circuit and the circuit is consuming 1mW.  The fields in the circuit are conservative.

He understands what a non conservative field is. You don't. That's why you think he's confused.

As usual, the one very confused is you, because is not 'field' (singular) but instead 'fields' (plural).
Title: Re: #562 – Electroboom!
Post by: jesuscf on December 03, 2021, 07:45:39 pm
Here is the circuit from Lewin's video with 1V voltage source.

If the KVL holds, it possible to split the original circuit in half at D and A by inserting two antiparallel 1mA current sources between nodes D and A. After split, the currents and the voltages of the original circuit remain unchanged.

If the KVL holds, it possible to split the original circuit in half at D and A by inserting a voltage source between nodes D and A with value V(D,A) because there is no current flowing through the new voltage source. After split, the currents and the voltages of the original circuit remain unchanged.

It can be seen that KVL holds and it is possible to spilt the circuit in half at D and A.

The 1V voltage source can be located some other point in the original circuit, which will affect the equivalent circuit values, but KVL will still hold.

(https://www.eevblog.com/forum/amphour/562-electroboom!/?action=dlattach;attach=1338839;image)

Here is the circuit from Lewin's video when the solenoid is generating 1mA current flowing in the circuit. It can be seen that KVL fails when the circuit contains of non-conservative fields, and thus it is not possible to split the circuit at D and A.

(https://www.eevblog.com/forum/amphour/562-electroboom!/?action=dlattach;attach=1338845;image)

By the way, your equivalent circuit for the loop with the two resistors is incorrect.  If you use the correct  equivalent circuit:

(https://www.eevblog.com/forum/amphour/562-electroboom!/?action=dlattach;attach=1339286;image)

You will see that KVL holds even if you split the circuit as in the second set of your images.
Title: Re: #562 – Electroboom!
Post by: Kalvin on December 03, 2021, 08:17:54 pm
Here is the circuit from Lewin's video with 1V voltage source.

If the KVL holds, it possible to split the original circuit in half at D and A by inserting two antiparallel 1mA current sources between nodes D and A. After split, the currents and the voltages of the original circuit remain unchanged.

If the KVL holds, it possible to split the original circuit in half at D and A by inserting a voltage source between nodes D and A with value V(D,A) because there is no current flowing through the new voltage source. After split, the currents and the voltages of the original circuit remain unchanged.

It can be seen that KVL holds and it is possible to spilt the circuit in half at D and A.

The 1V voltage source can be located some other point in the original circuit, which will affect the equivalent circuit values, but KVL will still hold.

(https://www.eevblog.com/forum/amphour/562-electroboom!/?action=dlattach;attach=1338839;image)

Here is the circuit from Lewin's video when the solenoid is generating 1mA current flowing in the circuit. It can be seen that KVL fails when the circuit contains of non-conservative fields, and thus it is not possible to split the circuit at D and A.

(https://www.eevblog.com/forum/amphour/562-electroboom!/?action=dlattach;attach=1338845;image)

By the way, your equivalent circuit for the loop with the two resistors is incorrect.  If you use the correct  equivalent circuit:

(https://www.eevblog.com/forum/amphour/562-electroboom!/?action=dlattach;attach=1339286;image)

You will see that KVL holds even if you split the circuit as in the second set of your images.

Please note that the original distributed circuit model in figure a below is not equivalent with the lumped circuit models in figures b and c. Your suggestion is identical with the lumped model in figure c with the voltage source split in half. However, your circuit is not identical with the original circuit any more, because you are transforming a non-conservative circuit into a conservative circuit by introducing those voltage sources. Thus I would say that your suggestion is not a valid transformation.

(https://www.eevblog.com/forum/amphour/562-electroboom!/?action=dlattach;attach=1337789;image)
Title: Re: #562 – Electroboom!
Post by: jesuscf on December 03, 2021, 11:13:43 pm
Here is the circuit from Lewin's video with 1V voltage source.

If the KVL holds, it possible to split the original circuit in half at D and A by inserting two antiparallel 1mA current sources between nodes D and A. After split, the currents and the voltages of the original circuit remain unchanged.

If the KVL holds, it possible to split the original circuit in half at D and A by inserting a voltage source between nodes D and A with value V(D,A) because there is no current flowing through the new voltage source. After split, the currents and the voltages of the original circuit remain unchanged.

It can be seen that KVL holds and it is possible to spilt the circuit in half at D and A.

The 1V voltage source can be located some other point in the original circuit, which will affect the equivalent circuit values, but KVL will still hold.

(https://www.eevblog.com/forum/amphour/562-electroboom!/?action=dlattach;attach=1338839;image)

Here is the circuit from Lewin's video when the solenoid is generating 1mA current flowing in the circuit. It can be seen that KVL fails when the circuit contains of non-conservative fields, and thus it is not possible to split the circuit at D and A.

(https://www.eevblog.com/forum/amphour/562-electroboom!/?action=dlattach;attach=1338845;image)

By the way, your equivalent circuit for the loop with the two resistors is incorrect.  If you use the correct  equivalent circuit:

(https://www.eevblog.com/forum/amphour/562-electroboom!/?action=dlattach;attach=1339286;image)

You will see that KVL holds even if you split the circuit as in the second set of your images.

Please note that the original distributed circuit model in figure a below is not equivalent with the lumped circuit models in figures b and c. Your suggestion is identical with the lumped model in figure c with the voltage source split in half. However, your circuit is not identical with the original circuit any more, because you are transforming a non-conservative circuit into a conservative circuit by introducing those voltage sources. Thus I would say that your suggestion is not a valid transformation.

(https://www.eevblog.com/forum/amphour/562-electroboom!/?action=dlattach;attach=1337789;image)

Check the attached pdf from Electromagnetics by Notaros, pages 279-280, example 6.6.
Title: Re: #562 – Electroboom!
Post by: Jesse Gordon on December 04, 2021, 06:52:27 am
Dude, that's Trevor Kearney. If Jesse hadn't flown away to another galaxy to avoid answering my question on the circuit path for Lewin's ring, he would tell you Trevor is "Armchair Physics Nobel Prize" number one. He's probably one of the most active people on YouTube trying to debunk Mehdi and the other KVLers.  :palm:

I didn't fly away, I told you I unequivocally proved that Team Lewin absoutely refused to answer basic questions - EVERY SINGLE (then active) MEMBER of Team Lewin absolutely refused to answer my question. What's else to discuss when Team Lewin denies observable reality?

As I said, I need to limit my time here to a couple times a week because I get paid by the hour to do other work (Self employed) and things reached a stage where it was clear that Team Lewin is holding to their beliefs even if it means denying observable reality.

As you've seen from my youtube channel comments, Trevor Kearney is a very nice gent - but you will also see that he does not have as good a grasp of the topic as he thinks.
He too refused to answer numerous questions which I asked him, and also made false predictions about reality - in fact, when I told him that KVL would hold with a loop made from two transformer secondary windings and two resistors, he didn't believe me, so that's why I made the "KVL Holds with an iron core" video.

He too refused to answer my simple question that you all refused to answer - If it measures like it's working, why is it not working? is it a technicality?

There seem to be two kinds of people - those who live and breath reality and deal with math as needed, and those who live in the virtual world of math and deal with reality when needed.

Or in your case, deny observable reality.
Title: Re: #562 – Electroboom!
Post by: Jesse Gordon on December 04, 2021, 07:01:12 am
I would be a little concerned with my probing if I measured V1 something else than 0V across a 0 ohm wire [when |V2 + V3| > 0].

(https://i.postimg.cc/1tNL2Mmn/20211122-214739.jpg)

Don't forget that not all voltage differences are ohmic.

There's batteries, capacitors, solar cells, peltier junctions, and yes, the ever popular inductive transformer.

All of those things can have voltages across them which would appear to violate ohms law if we assume that all voltage differences are strictly ohmic.

not all voltage differences are purely ohmic.

And one of those that is not strictly ohmic is the winding of a transformer.
Title: Re: #562 – Electroboom!
Post by: Jesse Gordon on December 04, 2021, 07:29:45 am
Question: In the following diagram, in a real life physical lab test performed with real (time synchronized) volt meters with a real transformer and real resistors CONNECTED AS SHOWN, will the readings of all the volt meters sum to zero, within the accuracy and resolution limitations of the volt meters? YES or NO.

(Or if you believe SOMETIMES is the answer, then explain one scenario for a YES condition and one scenario for a NO condition WITH THE VOLT METERS CONNECTED AS SHOWN - Running additional conductors through the transformer core is not allowed - nor is removing existing conductors from through the transformer core!)
(https://i.postimg.cc/jdJntBXT/20211128-121506.jpg)

You do realize that this is not a circuit that is equivalent to the "Lewin Ring", or equivalent to the circuit I gave you to solve, right?

Of course! "That" is a simple circuit that can be easily constructed and measured by anybody.

I was hoping we could find some common ground there and at least agree that KVL appears -- and perhaps does - hold on the above topology.

But like all the others, you are afraid to answer either because you don't have what it takes to test reality, or you know that your answer will undermine your belief.


Quote
That was my challenge to you, the task was to find the voltages across the resistors and the wires:
(https://i.postimg.cc/dtbDygLj/IMG-20211119-112430.jpg)

And that's what you came up with:
(https://i.postimg.cc/rs3cCg2F/20211119-122528.jpg)

Which of course brings up the question why you found it necessary to add a "transformer secondary" to find the voltage across "2R".

You call it an "added secondary" but it MEASURES AND MODELS the exact same as if the right hand volt meter was instead on the left. Doesn't matter where it is. It doesn't loop through the core, which means it's not another winding.

Remember, KVL requires two-terminal elements. If we're not using a two-terminal element, OF COURSE kvl isn't even applicable.

You want me to run a volt meter lead through the core which effectively adds another secondary winding, making it into a 3 terminal element, and as such, it's no longer applicable for KVL.

By having only ONE PATH through the core, we effectively have what MODELS AND MEASURES as a black box 2 terminal element, which is what KVL requires - and we can unambiguously measure the voltage across THOSE TWO TERMINALS and KVL holds.

Remember, a toroidal transformer MODELS AND MEASURES as if the entire induced voltage takes place at the plane through the center of the core.

How and where it is actually induced is irrelevant to KVL so long as an unambiguous voltage measurement can be physically obtained. By modeling the core as being a mythical fluxgate or stargate or whatever you guys called it, then we get a physical unambiguous voltage measurement and KVL holds.

So how about it? A yes or no on the top picture question in this comment?

Dude, you know the answer is yes, KVL will measure to hold. We should be past this. This is an easy one!

Talking about the open ended air core transformer and it's half turns would be much more interesting!
Title: Re: #562 – Electroboom!
Post by: Sredni on December 04, 2021, 07:30:26 am
I didn't fly away,
Oh, good. So you can answer my question, since I have answered yours.

Quote
I told you I unequivocally proved that Team Lewin absoutely refused to answer basic questions - EVERY SINGLE (then active) MEMBER of Team Lewin absolutely refused to answer my question. What's else to discuss when Team Lewin denies observable reality?

Maybe we did answer the question, and you are not able to accept the answer so you keep asking it over and over hoping to receive the answer you want?

Quote
He too refused to answer numerous questions which I asked him, and also made false predictions about reality - in fact, when I told him that KVL would hold with a loop made from two transformer secondary windings and two resistors, he didn't believe me, so that's why I made the "KVL Holds with an iron core" video.

He too refused to answer my simple question that you all refused to answer - If it measures like it's working, why is it not working? is it a technicality?

As I repeated about a dozen times, if a circuit is lumpABLE and you choose the circuit path in such a way that it does not include a variable magnetic field, then you can consider all voltages between any two points on that circuit path as if they were path independent. The circuit can be considered lumpED and KVL works.

IF the circuit is lumpABLE.

There are in our universe, circuits that are not lumpable: I call them UNlumpable circuits.
They are circuits where you cannot find a circuit path that does not encloses the variable flux region.
Lewin's ring IS such a circuit because it REQUIRES the two resistors to be on the opposite sides of a variable magnetic flux region.
You think Lewin's ring is lumpABLE?

Well, PROVE IT.

This is Lewin's ring: two resistors in a single loop that goes around a circular region (let's consider it of the same size as the loop, so you can see there is no 'room to twist' the wires) of variable magnetic field. The resistors are required to be on the opposite sides of the variable magnetic field region.

(https://i.postimg.cc/pLmfyHxZ/Lewin-ring-is-unlumpable.jpg)
link https://i.postimg.cc/pLmfyHxZ/Lewin-ring-is-unlumpable.jpg

Please, show everybody you can draw a circuit path (make it green, meaning it's 'flux-free') that joins the resistors' terminal to the "lumped transformer secondary" terminals and DOES NOT INCLUDE the variable magnetic field region in its interior. Like I did for the lumpABLE circuit I decided to see as lumpED (in my post "Lumpable (lumped and not lumped) and not lumpable circuits for dummies").

In addition, you can also show everybody you can draw the path inside your "lumped transformer secondary" that DOES INCLUDE the variable magnetic field region (make it orange) but IS NOT part of the green circuit path.
I will show you that if you can do that you will run into contradiction.
Title: Re: #562 – Electroboom!
Post by: jesuscf on December 04, 2021, 07:30:38 am
Dude, that's Trevor Kearney. If Jesse hadn't flown away to another galaxy to avoid answering my question on the circuit path for Lewin's ring, he would tell you Trevor is "Armchair Physics Nobel Prize" number one. He's probably one of the most active people on YouTube trying to debunk Mehdi and the other KVLers.  :palm:

I didn't fly away, I told you I unequivocally proved that Team Lewin absoutely refused to answer basic questions - EVERY SINGLE (then active) MEMBER of Team Lewin absolutely refused to answer my question. What's else to discuss when Team Lewin denies observable reality?

As I said, I need to limit my time here to a couple times a week because I get paid by the hour to do other work (Self employed) and things reached a stage where it was clear that Team Lewin is holding to their beliefs even if it means denying observable reality.

As you've seen from my youtube channel comments, Trevor Kearney is a very nice gent - but you will also see that he does not have as good a grasp of the topic as he thinks.
He too refused to answer numerous questions which I asked him, and also made false predictions about reality - in fact, when I told him that KVL would hold with a loop made from two transformer secondary windings and two resistors, he didn't believe me, so that's why I made the "KVL Holds with an iron core" video.

He too refused to answer my simple question that you all refused to answer - If it measures like it's working, why is it not working? is it a technicality?

There seem to be two kinds of people - those who live and breath reality and deal with math as needed, and those who live in the virtual world of math and deal with reality when needed.

Or in your case, deny observable reality.

What I got from the video is that Trevor Kearney was solving the problem, one way assuming that there were induced voltages in the wires, the other way assuming that KVL only included ohmic losses and he was comparing results.  Then, he is not clear at all and over complicates everything (common trademark of BS), but when I saw he got the correct answer for the voltage VAD, I thought, "good at least he got that right" and I moved on.  When I have more time I'll watch the video with more detail and decipher what he really think is going on.

Title: Re: #562 – Electroboom!
Post by: jesuscf on December 04, 2021, 07:31:36 am
I didn't fly away,
Oh, good. So you can answer my question, since I have answered yours.

Quote
I told you I unequivocally proved that Team Lewin absoutely refused to answer basic questions - EVERY SINGLE (then active) MEMBER of Team Lewin absolutely refused to answer my question. What's else to discuss when Team Lewin denies observable reality?

Maybe we did answer the question, and you are not able to accept the answer so you keep asking it over and over hoping to receive the answer you want?

Quote
He too refused to answer numerous questions which I asked him, and also made false predictions about reality - in fact, when I told him that KVL would hold with a loop made from two transformer secondary windings and two resistors, he didn't believe me, so that's why I made the "KVL Holds with an iron core" video.

He too refused to answer my simple question that you all refused to answer - If it measures like it's working, why is it not working? is it a technicality?

As I repeated about a dozen times, if a circuit is lumpABLE and you choose the circuit path in such a way that it does not include a variable magnetic field, then you can consider all voltages between any two points on that circuit path as if they were path independent. The circuit can be considered lumpED and KVL works.

IF the circuit is lumpABLE.

There are in our universe, circuits that are not lumpable: I call them UNlumpable circuits.
They are circuits where you cannot find a circuit path that does not encloses the variable flux region.
Lewin's ring IS such a circuit because it REQUIRES the two resistors to be on the opposite sides of a variable magnetic flux region.
You think Lewin's ring is lumpABLE?

Well, PROVE IT.

This is Lewin's ring: two resistors in a single loop that goes around a circular region (let's consider it of the same size as the loop, so you can see there is no 'room to twist' the wires) of variable magnetic field. The resistors are required to be on the opposite sides of the variable magnetic field region.

(https://i.postimg.cc/pLmfyHxZ/Lewin-ring-is-unlumpable.jpg)
link https://i.postimg.cc/pLmfyHxZ/Lewin-ring-is-unlumpable.jpg

Please, show everybody you can draw a circuit path (make it green, meaning it's 'flux-free') that joins the resistors' terminal to the "lumped transformer secondary" terminals and DOES NOT INCLUDE the variable magnetic field region in its interior. Like I did for the lumpABLE circuit I decided to see as lumpED (in my post "Lumpable (lumped and not lumped) and not lumpable circuits for dummies").

In addition, you can also show everybody you can draw the path inside your "lumped transformer secondary" that DOES INCLUDE the variable magnetic field region (make it orange) but IS NOT part of the green circuit path.
I will show you that if you can do that you will run into contradiction.

Did you get the voltage VAD yet?  Do you need more help?
Title: Re: #562 – Electroboom!
Post by: jesuscf on December 04, 2021, 07:44:21 am
I would be a little concerned with my probing if I measured V1 something else than 0V across a 0 ohm wire [when |V2 + V3| > 0].

(https://i.postimg.cc/1tNL2Mmn/20211122-214739.jpg)

Don't forget that not all voltage differences are ohmic.

There's batteries, capacitors, solar cells, peltier junctions, and yes, the ever popular inductive transformer.

All of those things can have voltages across them which would appear to violate ohms law if we assume that all voltage differences are strictly ohmic.

not all voltage differences are purely ohmic.

And one of those that is not strictly ohmic is the winding of a transformer.

According to team Lewin the induced EMF due to Faraday's law is 'special' (like themselves) because something-something and can not be treated just like any other EMF despite that all of them are measured in volts...
Title: Re: #562 – Electroboom!
Post by: Sredni on December 04, 2021, 07:46:11 am
Did you get the voltage VAD yet?  Do you need more help?

I have got plenty of  VAD voltages. One for every path I can think of.
Including one of the paths that gives the answer you like.
Title: Re: #562 – Electroboom!
Post by: jesuscf on December 04, 2021, 07:58:46 am
This is Lewin's ring: two resistors in a single loop that goes around a circular region (let's consider it of the same size as the loop, so you can see there is no 'room to twist' the wires) of variable magnetic field. The resistors are required to be on the opposite sides of the variable magnetic field region.

(https://i.postimg.cc/pLmfyHxZ/Lewin-ring-is-unlumpable.jpg)
link https://i.postimg.cc/pLmfyHxZ/Lewin-ring-is-unlumpable.jpg (https://i.postimg.cc/pLmfyHxZ/Lewin-ring-is-unlumpable.jpg)

Here, I fixed your drawing.  Can you compute VAD now?

(https://www.eevblog.com/forum/amphour/562-electroboom!/?action=dlattach;attach=1339748)

Title: Re: #562 – Electroboom!
Post by: Kalvin on December 04, 2021, 07:59:25 am
Check the attached pdf from Electromagnetics by Notaros, pages 279-280, example 6.6.

Notaros seems to be modeling a similar circuit so that the EMF inducing the current is shown as two lumped voltage sources in the circuit, instead of computing the induced current in the distributed circuit due to the EMF. I kind of understand Notaros wanting to model the EMF into the circuit, but doing so Notaros is actually converting a non-conservative circuit into a conservative one. To me that looks like an illegal chess move (converting a non-conservative circuit into a conservative circuit).
Title: Re: #562 – Electroboom!
Post by: jesuscf on December 04, 2021, 08:03:06 am
Check the attached pdf from Electromagnetics by Notaros, pages 279-280, example 6.6.

Notaros seems to be modeling a similar circuit so that the EMF inducing the current is shown as two lumped voltage sources in the circuit, instead of computing the induced current in the distributed circuit due to the EMF. I kind of understand Notaros wanting to model the EMF into the circuit, but doing so Notaros is actually converting a non-conservative circuit into a conservative one. To me that looks like an illegal chess move (converting a non-conservative circuit into a conservative circuit).

Ok, tell me why you think the circuit is non-conservative.
Title: Re: #562 – Electroboom!
Post by: Kalvin on December 04, 2021, 08:28:09 am
Check the attached pdf from Electromagnetics by Notaros, pages 279-280, example 6.6.

Notaros seems to be modeling a similar circuit so that the EMF inducing the current is shown as two lumped voltage sources in the circuit, instead of computing the induced current in the distributed circuit due to the EMF. I kind of understand Notaros wanting to model the EMF into the circuit, but doing so Notaros is actually converting a non-conservative circuit into a conservative one. To me that looks like an illegal chess move (converting a non-conservative circuit into a conservative circuit).

Here is the problem with Notaros and a counter-example: Where would you put those voltage sources modeling the EMF so that you can measure the correct voltages across two neighboring resistors ie. V(Ra,Rb), V(Rb,Rc), V(Rc, Rf), V(Rf, Re), V(Re, Rd), V(Rd, Ra).

(https://www.eevblog.com/forum/amphour/562-electroboom!/?action=dlattach;attach=1339760;image)
Title: Re: #562 – Electroboom!
Post by: jesuscf on December 04, 2021, 08:39:54 am
Check the attached pdf from Electromagnetics by Notaros, pages 279-280, example 6.6.

Notaros seems to be modeling a similar circuit so that the EMF inducing the current is shown as two lumped voltage sources in the circuit, instead of computing the induced current in the distributed circuit due to the EMF. I kind of understand Notaros wanting to model the EMF into the circuit, but doing so Notaros is actually converting a non-conservative circuit into a conservative one. To me that looks like an illegal chess move (converting a non-conservative circuit into a conservative circuit).

Here is the problem with Notaros and a counter-example: Where would you put those voltage sources modeling the EMF so that you can measure the correct voltages across two neighboring resistors ie. V(Ra,Rb), V(Rb,Rc), V(Rc, Rf), V(Rf, Re), V(Re, Rd), V(Rd, Ra).

(https://www.eevblog.com/forum/amphour/562-electroboom!/?action=dlattach;attach=1339760;image)

That is a very good question that has been answered in this forum many, many times: each resistor in the loop behaves both as voltage source a resistor in series.  Check this post:

https://www.eevblog.com/forum/amphour/562-electroboom!/msg3830852/#msg3830852 (https://www.eevblog.com/forum/amphour/562-electroboom!/msg3830852/#msg3830852)
Title: Re: #562 – Electroboom!
Post by: thinkfat on December 04, 2021, 08:42:19 am
That was my challenge to you, the task was to find the voltages across the resistors and the wires:
(https://i.postimg.cc/dtbDygLj/IMG-20211119-112430.jpg)

And that's what you came up with:
(https://i.postimg.cc/rs3cCg2F/20211119-122528.jpg)

Which of course brings up the question why you found it necessary to add a "transformer secondary" to find the voltage across "2R".

You call it an "added secondary" but it MEASURES AND MODELS the exact same as if the right hand volt meter was instead on the left. Doesn't matter where it is. It doesn't loop through the core, which means it's not another winding.

Remember, KVL requires two-terminal elements. If we're not using a two-terminal element, OF COURSE kvl isn't even applicable.

You want me to run a volt meter lead through the core which effectively adds another secondary winding, making it into a 3 terminal element, and as such, it's no longer applicable for KVL.

Before I go further into the details of why I think your explanation is rubbish, let me expand on "observable reality": What is observed is not reality until you've asked "Why is it so?" often enough to rule out all possible reasons but one. The idea of it being enough to just observe and not ask the "Why is it so?" question has directly lead to the creation of divine beings, or gods, to explain phenomena of nature in ancient cultures. People observed weather and the changing of seasons and as they were not able to explain and thus gods were created.

You, instead of asking the right questions, just try to confirm your beliefs. Instead of asking why the volt meters in Lewins experiment show different voltages you say "He's just a fool, he made a mistake, it's bad probing". When asked about the mechanism that induces voltage in a transformer winding you say "It's Nature".

I feel now that I did injustice to the flatearthers by likening you to them. Because, as the Physicist Sabine Hossenfelder said about them, their science is wrong, but not stupid.

Anyway, thank you for answering my question about which of the loops in my diagram you consider a "transformer secondary" and which are not. I knew the answer beforehand of course, but I needed you to confirm it with your own words so that there will be no wiggling out of it later.
Title: Re: #562 – Electroboom!
Post by: Jesse Gordon on December 04, 2021, 08:42:51 am
Dude, that's Trevor Kearney. If Jesse hadn't flown away to another galaxy to avoid answering my question on the circuit path for Lewin's ring, he would tell you Trevor is "Armchair Physics Nobel Prize" number one. He's probably one of the most active people on YouTube trying to debunk Mehdi and the other KVLers.  :palm:

Isn't it hilarious that KVLers be so obtuse that they can't even recognize when a video was made to debunk their claims, due to their absolute lack of understanding of what is being discussed?

Congratulations, Mehdi Sadaghdar! Look what you've done! You've spawned a whole bunch of brain-damaged mock engineers like yourself.

Mehdi didn't start this, Lewin did. I'd made my videos in rebuttal to Lewin before I even know anybody else had.
Title: Re: #562 – Electroboom!
Post by: Jesse Gordon on December 04, 2021, 08:54:49 am
I didn't fly away,
Oh, good. So you can answer my question, since I have answered yours.


No, you didn't answer MY question, you asked a question of your own choosing.

Quote
Quote
I told you I unequivocally proved that Team Lewin absoutely refused to answer basic questions - EVERY SINGLE (then active) MEMBER of Team Lewin absolutely refused to answer my question. What's else to discuss when Team Lewin denies observable reality?

Maybe we did answer the question, and you are not able to accept the answer so you keep asking it over and over hoping to receive the answer you want?

No, you answered a question of YOUR CHOOSING, thus refusing to answer mine.

Quote
Quote
He too refused to answer numerous questions which I asked him, and also made false predictions about reality - in fact, when I told him that KVL would hold with a loop made from two transformer secondary windings and two resistors, he didn't believe me, so that's why I made the "KVL Holds with an iron core" video.

He too refused to answer my simple question that you all refused to answer - If it measures like it's working, why is it not working? is it a technicality?

As I repeated about a dozen times, if a circuit is lumpABLE and you choose the circuit path in such a way that it does not include a variable magnetic field, then you can consider all voltages between any two points on that circuit path as if they were path independent. The circuit can be considered lumpED and KVL works.

IF the circuit is lumpABLE.

See? You didn't answer my question with a yes or no, you answered with an IF, and then you refused to tell me whether my circuit met your IF.

So I ask again. I've provided a very clear unambiguous test circuit. I've specified even the points which are considered the terminals of the lumped elements.

By the way, you talk about lumpable circuits, but you seem awfully silent on which of my elements are not lumpable. According to your own trusted source, your Pratt & Whitney of textbooks, "Lumped circuts are obtained by connecting lumped elements. Typical lumped elements are resistors, capacitors, inductors, and transformers."

When you can answer MY question as described directly below, then we can talk. Until then, you're denying observable reality. Or you simply don't know what reality is in this case.

Question: In the following diagram, in a real life physical lab test performed with real (time synchronized) volt meters with a real transformer and real resistors CONNECTED AS SHOWN, will the readings of all the volt meters sum to zero, within the accuracy and resolution limitations of the volt meters? YES or NO.

(Or if you believe SOMETIMES is the answer, then explain one scenario for a YES condition and one scenario for a NO condition WITH THE VOLT METERS CONNECTED AS SHOWN - Running additional conductors through the transformer core is not allowed - nor is removing existing conductors from through the transformer core!)

(https://i.postimg.cc/jdJntBXT/20211128-121506.jpg)


Quote
There are in our universe, circuits that are not lumpable: I call them UNlumpable circuits.
They are circuits where you cannot find a circuit path that does not encloses the variable flux region.
Lewin's ring IS such a circuit because it REQUIRES the two resistors to be on the opposite sides of a variable magnetic flux region.
You think Lewin's ring is lumpABLE?

Well, PROVE IT.

This is Lewin's ring: two resistors in a single loop that goes around a circular region (let's consider it of the same size as the loop, so you can see there is no 'room to twist' the wires) of variable magnetic field. The resistors are required to be on the opposite sides of the variable magnetic field region.

(https://i.postimg.cc/pLmfyHxZ/Lewin-ring-is-unlumpable.jpg)
link https://i.postimg.cc/pLmfyHxZ/Lewin-ring-is-unlumpable.jpg

Please, show everybody you can draw a circuit path (make it green, meaning it's 'flux-free') that joins the resistors' terminal to the "lumped transformer secondary" terminals and DOES NOT INCLUDE the variable magnetic field region in its interior. Like I did for the lumpABLE circuit I decided to see as lumpED (in my post "Lumpable (lumped and not lumped) and not lumpable circuits for dummies").

In addition, you can also show everybody you can draw the path inside your "lumped transformer secondary" that DOES INCLUDE the variable magnetic field region (make it orange) but IS NOT part of the green circuit path.
I will show you that if you can do that you will run into contradiction.

Like I said, answer the question I ASKED and then we can talk.
Title: Re: #562 – Electroboom!
Post by: Kalvin on December 04, 2021, 08:56:52 am
Check the attached pdf from Electromagnetics by Notaros, pages 279-280, example 6.6.

Notaros seems to be modeling a similar circuit so that the EMF inducing the current is shown as two lumped voltage sources in the circuit, instead of computing the induced current in the distributed circuit due to the EMF. I kind of understand Notaros wanting to model the EMF into the circuit, but doing so Notaros is actually converting a non-conservative circuit into a conservative one. To me that looks like an illegal chess move (converting a non-conservative circuit into a conservative circuit).

Here is the problem with Notaros and a counter-example: Where would you put those voltage sources modeling the EMF so that you can measure the correct voltages across two neighboring resistors ie. V(Ra,Rb), V(Rb,Rc), V(Rc, Rf), V(Rf, Re), V(Re, Rd), V(Rd, Ra).

(https://www.eevblog.com/forum/amphour/562-electroboom!/?action=dlattach;attach=1339760;image)

That is a very good question that has been answered in this forum many, many times: each resistor in the loop behaves both as voltage source a resistor in series.  Check this post:

https://www.eevblog.com/forum/amphour/562-electroboom!/msg3830852/#msg3830852 (https://www.eevblog.com/forum/amphour/562-electroboom!/msg3830852/#msg3830852)

Fair enough, but my circuit has only six resistors. Where would you put those voltage sources modeling the EMF in the circuit so that it will be possible to measure correct voltages a) across each individual resistor, and b) across two consecutive resistors?
Title: Re: #562 – Electroboom!
Post by: Jesse Gordon on December 04, 2021, 09:04:24 am
That was my challenge to you, the task was to find the voltages across the resistors and the wires:
(https://i.postimg.cc/dtbDygLj/IMG-20211119-112430.jpg)

And that's what you came up with:
(https://i.postimg.cc/rs3cCg2F/20211119-122528.jpg)

Which of course brings up the question why you found it necessary to add a "transformer secondary" to find the voltage across "2R".

You call it an "added secondary" but it MEASURES AND MODELS the exact same as if the right hand volt meter was instead on the left. Doesn't matter where it is. It doesn't loop through the core, which means it's not another winding.

Remember, KVL requires two-terminal elements. If we're not using a two-terminal element, OF COURSE kvl isn't even applicable.

You want me to run a volt meter lead through the core which effectively adds another secondary winding, making it into a 3 terminal element, and as such, it's no longer applicable for KVL.

Before I go further into the details of why I think your explanation is rubbish, let me expand on "observable reality":

What is observed is not reality

That figures.

Quote
until you've asked "Why is it so?" often enough to rule out all possible reasons but one. The idea of it being enough to just observe and not ask the "Why is it so?" question has directly lead to the creation of divine beings, or gods, to explain phenomena of nature in ancient cultures. People observed weather and the changing of seasons and as they were not able to explain and thus gods were created.

You, instead of asking the right questions, just try to confirm your beliefs. Instead of asking why the volt meters in Lewins experiment show different voltages you say "He's just a fool, he made a mistake, it's bad probing". When asked about the mechanism that induces voltage in a transformer winding you say "It's Nature".

I feel now that I did injustice to the flatearthers by likening you to them.

You realize the feeling is mutual, right?

Quote
Because, as the Physicist Sabine Hossenfelder said about them, their science is wrong, but not stupid.

Anyway, thank you for answering my question about which of the loops in my diagram you consider a "transformer secondary" and which are not. I knew the answer beforehand of course, but I needed you to confirm it with your own words so that there will be no wiggling out of it later.

Which I'm sure you will quote out of context, conveniently forgetting that I said that it's how IT MODELS AND MEASURES. Don't forget that part when you quote me, ya know?

What I can't understand is your (and all of Team Lewin's) refusal to answer my question. It's not even about Lewin's exact circuit. What's the harm in conceding that KVL at the very least appears to hold in the following circuit?

(https://i.postimg.cc/jdJntBXT/20211128-121506.jpg)
Title: Re: #562 – Electroboom!
Post by: Jesse Gordon on December 04, 2021, 09:12:49 am
Well folks that better be my two days for this week.

I don't understand why Team Lewin cannot see that a two-terminal toroidal transformer secondary makes a perfectly lumped element for use in a KVL loop.

Even the Rolls Royce of textbooks says that transformers are common lumped elements for a KVL loop. Dude.


Quote from: Rolls Royce Textbook

Lumped circuits are obtained by connecting lumped elements. Typical
lumped elements are resistors, capacitors, inductors, and transformers.



PM me if anyone actually answers my question, but I suspect they won't.

I've asked it enough times to enough Team Lewin members I suspect nobody is going to.

If there's any newcomers, here's the question again.

Question: In the following diagram, in a real life physical lab test performed with real (time synchronized) volt meters with a real transformer and real resistors CONNECTED AS SHOWN, will the readings of all the volt meters sum to zero, within the accuracy and resolution limitations of the volt meters? YES or NO.

(Or if you believe SOMETIMES is the answer, then explain one scenario for a YES condition and one scenario for a NO condition WITH THE VOLT METERS CONNECTED AS SHOWN - Running additional conductors through the transformer core is not allowed - nor is removing existing conductors from through the transformer core!)

(https://i.postimg.cc/jdJntBXT/20211128-121506.jpg)



Title: Re: #562 – Electroboom!
Post by: Kalvin on December 04, 2021, 11:08:48 am
Quote from: Kalvin
Here is the problem with Notaros and a counter-example: Where would you put those voltage sources modeling the EMF so that you can measure the correct voltages across two neighboring resistors ie. V(Ra,Rb), V(Rb,Rc), V(Rc, Rf), V(Rf, Re), V(Re, Rd), V(Rd, Ra).
<removed the image>

That is a very good question that has been answered in this forum many, many times: each resistor in the loop behaves both as voltage source a resistor in series.  Check this post:

https://www.eevblog.com/forum/amphour/562-electroboom!/msg3830852/#msg3830852 (https://www.eevblog.com/forum/amphour/562-electroboom!/msg3830852/#msg3830852)

Let's say that someone is asking you: "We all know that a magnet has two poles N and S, and it can clearly be seen that the magnetic field starts at N and ends in S. But when there is a current flowing in a wire, a magnetic field will be created around the wire. Where does this magnetic field start and end?".

Would you explain this phenomenon something like this: "Well, you can think this situation as there are lots of very small magnets around the wire forming a closed loop, and that is why it is not possible to determine the start and end of the magnetic field". And the next question from that person will be: "Ok, and where do those little magnets come from?".

Or, would you explain it like this: "The current flowing in a wire is inducing a magnetic field around the wire, forming a closed loop, and it is not really possible to find the start and end of the magnetic field". Then that person is insisting: "How is it possible that there is a magnetic field without magnets. Your explanation is totally wrong! There has to be magnets somewhere, and every kid knows that magnets are creating a magnetic field, and magnetic field start at N and end in S."
Title: Re: #562 – Electroboom!
Post by: Kalvin on December 04, 2021, 12:16:13 pm
Check the attached pdf from Electromagnetics by Notaros, pages 279-280, example 6.6.

Notaros seems to be modeling a similar circuit so that the EMF inducing the current is shown as two lumped voltage sources in the circuit, instead of computing the induced current in the distributed circuit due to the EMF. I kind of understand Notaros wanting to model the EMF into the circuit, but doing so Notaros is actually converting a non-conservative circuit into a conservative one. To me that looks like an illegal chess move (converting a non-conservative circuit into a conservative circuit).

Ok, tell me why you think the circuit is non-conservative.

I am not a physicist, so I am explaining it as follows:

1. When there is a constant current flowing in a wire, a magnetic field with a constant strength will be induced around this current carrying wire. The strength of the induced magnetic field is proportional to the current in the wire.

2. But this is not true another way around: When there is a constant magnetic field around the wire, there will not be a current induced in the wire. Only a changing magnetic field will induce a current in a wire. The magnitude of the current induced into the wire is proportional to the rate of change of the magnetic field. In order to maintain a constant current in a wire, the strength of the magnetic field shall be increasing at a constant rate.

This fact that "this is not true another way around" when a magnetic field is inducing current in a wire makes this circuit non-conservative.
Title: Re: #562 – Electroboom!
Post by: bsfeechannel on December 04, 2021, 01:50:36 pm
Check the attached pdf from Electromagnetics by Notaros, pages 279-280, example 6.6.

Notaros seems to be modeling a similar circuit so that the EMF inducing the current is shown as two lumped voltage sources in the circuit, instead of computing the induced current in the distributed circuit due to the EMF. I kind of understand Notaros wanting to model the EMF into the circuit, but doing so Notaros is actually converting a non-conservative circuit into a conservative one. To me that looks like an illegal chess move (converting a non-conservative circuit into a conservative circuit).

What Notaros did was a math trick, very common in textbooks, but that unfortunately confuses KVLers. They think that if you model some circuit with an equivalent circuit, the components of that equivalent circuit will be found in the modeled circuit.

For instance, this is a very simplified model of a transistor for small signals at low frequencies.

(https://www.researchgate.net/publication/304746021/figure/fig2/AS:379772296286211@1467556179896/The-small-signal-equivalent-circuit-of-the-subthreshold-transistor-taking-into-account.png)

KVLers think that if they open a transistor up they'll find a resistor connected in parallel with a voltage-controlled current source. Well, no. That's just a mathematical trick to help you solve the circuit. A lot of assumptions are implicit in this model. Some of them are in the model name. Signals must be small and frequencies, low, among other things (like, for example, ambient temperature, biasing, etc.)

Notaros modeling gives the same result if he decided to use Faraday's law, however there are a lot of assumptions that he doesn't mention, just as in the transistor model. One of them is that there's no real battery or generator in the path of the circuit.

KVLers can't understand that the battery they are looking for is not in the resistors or the wires. Their explanations always contradict each other. For example. When the loop with two dimensionally small resistors are connected with wires, they say that the voltage is generated in the wires, but not in the resistors. When we eliminate the wires, or when the wires are the resistors themselves, they say that itsy bitsy teeny tiny little batteries suddenly migrate inside the resistors.

What they don't tell you is when exactly these itsy bitsy teeny tiny little batteries migrate from the wires to the resistors. There's no theory to predict that. Search and you'll not find anywhere in the literature about electromagnetism.

Notaros, for example, contradicts the itsy bitsy teeny tiny little battery model, as he put two lumped generators in series with the resistors, although his loop is comprised of two resistive wires and nothing else.
Title: Re: #562 – Electroboom!
Post by: bsfeechannel on December 04, 2021, 03:19:25 pm

According to team Lewin the induced EMF due to Faraday's law is 'special'

Precisely. That's what Michael Faraday himself discovered on 28 October 1831.

Quote
(like themselves)

Nah. We are just regular people who decided to not give ears to charlatubers but set out to study electromagnetism like real men.

Quote
because something-something and can not be treated just like any other EMF despite that all of them are measured in volts...

You're right again. You're starting to see the light. Hang out with us and soon you'll understand the "something-something" and how being a KVLer really sucks.
Title: Re: #562 – Electroboom!
Post by: Kalvin on December 04, 2021, 04:07:03 pm
Check the attached pdf from Electromagnetics by Notaros, pages 279-280, example 6.6.

Notaros seems to be modeling a similar circuit so that the EMF inducing the current is shown as two lumped voltage sources in the circuit, instead of computing the induced current in the distributed circuit due to the EMF. I kind of understand Notaros wanting to model the EMF into the circuit, but doing so Notaros is actually converting a non-conservative circuit into a conservative one. To me that looks like an illegal chess move (converting a non-conservative circuit into a conservative circuit).

What Notaros did was a math trick, very common in textbooks, but that unfortunately confuses KVLers. They think that if you model some circuit with an equivalent circuit, the components of that equivalent circuit will be found in the modeled circuit.

I do not really care who is KVLer and who is not. I am just trying to understand how Dr. Lewin's circuit needs to be analyzed. So far my understanding can be summarized as follows:

1. The circuit is residing inside a magnetic field, so it should not be modeled with lumped sources.
2. Applying #1: The EMF induced by the magnetic field cannot really be modeled as lumped voltage source(s).
3. There is a current flowing in the circuit due to EMF induced by the magnetic field, and its magnitude is EMF/Rtotal.
4. Trying to incorporate the induced EMF into circuit model is an illegal chess move (see #2 and #3).
5. Although it may seem counterintuitive, but there can be a current flowing in the circuit without voltage sources in the circuit.
6. The magnetic field inducing this current is a non-conservative field, which means that KVL will not hold.
7. Consequence from #6: The voltage between two points in the circuit is path-dependent violating KVL, and there is no magic or probing error behind it.
8. Any attempt in modeling the voltage source(s) as lumped source(s), eg. transformer, will modify the original circuit in a fundamental way, and this new circuit is not identical to the original circuit anymore.
9. If it is found that KVL holds while analyzing this particular circuit, the model is probably ignoring one or more items from #1 - #8.

My understanding is that trying forcefully to model the original circuit using lumped sources (eg. transformers etc.) so that KVL will still hold has lead into this debate.
Title: Re: #562 – Electroboom!
Post by: Sredni on December 04, 2021, 04:42:32 pm

1. Almost. The circuit includes (or better, cuts) the variable magnetic flux. But the circuit itself (i.e. the components and the conductors joining them can be in a region of space where there is no magnetic field at all (excluding that generated by the flow of the current in it, but we consider that negligible in the case of Lewin's ring)
2. Correct.
3. Correct
4. Trying to incorporate the nonlocalized EMF on the ring with lumped sources will give you a voltage configuration around the ring that will not correspond to the actual physical system (there will be jumps in correspondence of the lumped sources); trying to distribute it along the ring will give you a distribution of voltage corresponding to the effects of the coloumbian field alone, as if the Eind component of the field had been removed leaving the charges (at a given instant of time) frozen where they were. In my next posts I will show how this is a 'partial' description of the system.
5. Yeah, I guess you can say so.
6. The magnetic field inducing this current is a non-conservative field, yes, but most importantly it is changing and is therefore associated with a non-conservative induced electric field Eind. It's this non-conservative component of the electric field that messes things up from the point of view of voltage (which is the path integral of the total electric field Etot = Eind + Ecoul). KVL will not hold if you consider paths that follow the curling electric field full circle - it is easier to see it in this way: KVL will not hold if your closed voltage loop is going around the changing flux region.
7. Correct. It is basically a consequence of the fact that by 'going around' you eliminate the possibility to 'compensate' the contributes of Etot to path integral going 'along the path' with contributes 'against the path'. See my answer on EE Stack Exchange for pictures.
8. Correct.
9. Correct.

Title: Re: #562 – Electroboom!
Post by: Kalvin on December 04, 2021, 04:55:26 pm
@Sredni, thank you for adding the details and corrections. Highly appreciated.  :-+
Title: Re: #562 – Electroboom!
Post by: jesuscf on December 04, 2021, 04:59:30 pm
Check the attached pdf from Electromagnetics by Notaros, pages 279-280, example 6.6.

Notaros seems to be modeling a similar circuit so that the EMF inducing the current is shown as two lumped voltage sources in the circuit, instead of computing the induced current in the distributed circuit due to the EMF. I kind of understand Notaros wanting to model the EMF into the circuit, but doing so Notaros is actually converting a non-conservative circuit into a conservative one. To me that looks like an illegal chess move (converting a non-conservative circuit into a conservative circuit).

Here is the problem with Notaros and a counter-example: Where would you put those voltage sources modeling the EMF so that you can measure the correct voltages across two neighboring resistors ie. V(Ra,Rb), V(Rb,Rc), V(Rc, Rf), V(Rf, Re), V(Re, Rd), V(Rd, Ra).

(https://www.eevblog.com/forum/amphour/562-electroboom!/?action=dlattach;attach=1339760;image)

That is a very good question that has been answered in this forum many, many times: each resistor in the loop behaves both as voltage source a resistor in series.  Check this post:

https://www.eevblog.com/forum/amphour/562-electroboom!/msg3830852/#msg3830852 (https://www.eevblog.com/forum/amphour/562-electroboom!/msg3830852/#msg3830852)

Fair enough, but my circuit has only six resistors. Where would you put those voltage sources modeling the EMF in the circuit so that it will be possible to measure correct voltages a) across each individual resistor, and b) across two consecutive resistors?

That is exactly the problem: the resistors are both behaving as a voltage source and a resistor when in a loop exposed to a varying magnetic field.  But you only have access to the terminals of the resistor, so when you measure across the terminals you'll get the net voltage: the generated EMF minus the voltage drop due to ohms law.
Title: Re: #562 – Electroboom!
Post by: Kalvin on December 04, 2021, 05:10:46 pm
That is exactly the problem: the resistors are both behaving as a voltage source and a resistor when in a loop exposed to a varying magnetic field.  But you only have access to the terminals of the resistor, so when you measure across the terminals you'll get the net voltage: the generated EMF minus the voltage drop due to ohms law.

Are you suggesting that if I have for example six resistors with identical resistance but with different physical lengths eg. 0402, 0603, 0805, 1206, and leaded (non-inductive) resistors, I should see different voltages across these resistors due to the different physical sizes because the longer resistors should have getting more EMF than the shorter ones?

Edit: For example if 1206 is three times the length of 0402, the 1206 should be getting three times the EMF?
Title: Re: #562 – Electroboom!
Post by: jesuscf on December 04, 2021, 05:14:11 pm
Check the attached pdf from Electromagnetics by Notaros, pages 279-280, example 6.6.

Notaros seems to be modeling a similar circuit so that the EMF inducing the current is shown as two lumped voltage sources in the circuit, instead of computing the induced current in the distributed circuit due to the EMF. I kind of understand Notaros wanting to model the EMF into the circuit, but doing so Notaros is actually converting a non-conservative circuit into a conservative one. To me that looks like an illegal chess move (converting a non-conservative circuit into a conservative circuit).

What Notaros did was a math trick, very common in textbooks, but that unfortunately confuses KVLers. They think that if you model some circuit with an equivalent circuit, the components of that equivalent circuit will be found in the modeled circuit.

For instance, this is a very simplified model of a transistor for small signals at low frequencies.

(https://www.researchgate.net/publication/304746021/figure/fig2/AS:379772296286211@1467556179896/The-small-signal-equivalent-circuit-of-the-subthreshold-transistor-taking-into-account.png)

KVLers think that if they open a transistor up they'll find a resistor connected in parallel with a voltage-controlled current source. Well, no. That's just a mathematical trick to help you solve the circuit. A lot of assumptions are implicit in this model. Some of them are in the model name. Signals must be small and frequencies, low, among other things (like, for example, ambient temperature, biasing, etc.)

Notaros modeling gives the same result if he decided to use Faraday's law, however there are a lot of assumptions that he doesn't mention, just as in the transistor model. One of them is that there's no real battery or generator in the path of the circuit.

KVLers can't understand that the battery they are looking for is not in the resistors or the wires. Their explanations always contradict each other. For example. When the loop with two dimensionally small resistors are connected with wires, they say that the voltage is generated in the wires, but not in the resistors. When we eliminate the wires, or when the wires are the resistors themselves, they say that itsy bitsy teeny tiny little batteries suddenly migrate inside the resistors.

What they don't tell you is when exactly these itsy bitsy teeny tiny little batteries migrate from the wires to the resistors. There's no theory to predict that. Search and you'll not find anywhere in the literature about electromagnetism,

Notaros, for example, contradicts the itsy bitsy teeny tiny little battery model, as he put two lumped generators in series with the resistors, although his loop is comprised of two resistive wires and nothing else.

No dumdum, we know the MOSFET small signal equivalent circuit comes from this equation (for an enhancement N-MOSFET):

\$i_D  = \frac{1}{2}k_n^{'} \frac{W}{L}(v_{GS}  - V_t )^2 \left( {1 + \lambda v_{DS} } \right)\$

when vGS is small and we apply superposition to get the AC component (did you understand all those word?).  What you call 'mathematical tricks' is a fundamental part of engineering which you seem to be unaware off.

A side note for team Lewin: calling that induced voltage a 'tiny battery' is incorrect.  It is not a battery, it is an induced EMF that we represented in the circuit with voltage source.

EDIT: by the way, I have said it many, many times, that the same apply to wires and resistors:  under a varying magnetic field, both wires and resistors behave like a voltage source in series with a resistor.
 
Title: Re: #562 – Electroboom!
Post by: jesuscf on December 04, 2021, 05:27:42 pm
That is exactly the problem: the resistors are both behaving as a voltage source and a resistor when in a loop exposed to a varying magnetic field.  But you only have access to the terminals of the resistor, so when you measure across the terminals you'll get the net voltage: the generated EMF minus the voltage drop due to ohms law.

Are you suggesting that if I have for example six resistors with identical resistance but with different physical lengths eg. 0402, 0603, 0805, 1206, and leaded (non-inductive) resistors, I should see different voltages across these resistors due to the different physical sizes because the longer resistors should have getting more EMF than the shorter ones?

Edit: For example if 1206 is three times the length of 0402, the 1206 should be getting three times the EMF?

Yes, for the resistors placed in a circle as you put then in the figure, the induced voltage is proportional to the length of the resistor, but the voltage drop only depends on the resistance.
Title: Re: #562 – Electroboom!
Post by: Kalvin on December 04, 2021, 05:32:07 pm
That is exactly the problem: the resistors are both behaving as a voltage source and a resistor when in a loop exposed to a varying magnetic field.  But you only have access to the terminals of the resistor, so when you measure across the terminals you'll get the net voltage: the generated EMF minus the voltage drop due to ohms law.

Are you suggesting that if I have for example six resistors with identical resistance but with different physical lengths eg. 0402, 0603, 0805, 1206, and leaded (non-inductive) resistors, I should see different voltages across these resistors due to the different physical sizes because the longer resistors should have getting more EMF than the shorter ones?

Edit: For example if 1206 is three times the length of 0402, the 1206 should be getting three times the EMF?

Yes, for the resistors placed in a circle as you put then in the figure, the induced voltage is proportional to the length of the resistor, but the voltage drop only depends on the resistance.

Just to make sure that I understand you correctly: You are saying that if I have six 100 ohm 0402 resistors and measure the voltages across these resistors one at a time, I should get different voltage values when I have six 100 ohm 1206 resistors and repeat the voltage measurement across each resistor?
Title: Re: #562 – Electroboom!
Post by: jesuscf on December 04, 2021, 05:38:50 pm
That is exactly the problem: the resistors are both behaving as a voltage source and a resistor when in a loop exposed to a varying magnetic field.  But you only have access to the terminals of the resistor, so when you measure across the terminals you'll get the net voltage: the generated EMF minus the voltage drop due to ohms law.

Are you suggesting that if I have for example six resistors with identical resistance but with different physical lengths eg. 0402, 0603, 0805, 1206, and leaded (non-inductive) resistors, I should see different voltages across these resistors due to the different physical sizes because the longer resistors should have getting more EMF than the shorter ones?

Edit: For example if 1206 is three times the length of 0402, the 1206 should be getting three times the EMF?

Yes, for the resistors placed in a circle as you put then in the figure, the induced voltage is proportional to the length of the resistor, but the voltage drop only depends on the resistance.

Just to make sure that I understand you correctly: You are saying that if I have six identical 0402 resistors and measure the voltages across these resistors, I should get different voltage values when I have six 1206 resistors with the same resistance as 0402s?

If you only have resistors and no wires, the voltage you'll measure across each resistor is zero, no matter the size of the resistor.  I you have a combination of wires and resistors, or resistors of different values or a mix of resistors of different sizes you'll need to solve the circuit as I did in the example where I made a ring of 178 x 2 ohms resistors, one 100 ohms resistor, and one 900 ohms resistor.
Title: Re: #562 – Electroboom!
Post by: bsfeechannel on December 04, 2021, 05:39:57 pm
Just to make sure that I understand you correctly: You are saying that if I have six 100 ohm 0402 resistors and measure the voltages across these resistors one at a time, I should get different voltage values when I have six 100 ohm 1206 resistors and repeat the voltage measurement across each resistor?

Bang!
Title: Re: #562 – Electroboom!
Post by: Kalvin on December 04, 2021, 05:42:49 pm
Sorry that my edits didn't quite make into jesuscf's reply.
Title: Re: #562 – Electroboom!
Post by: bsfeechannel on December 04, 2021, 05:54:14 pm
EDIT: by the way, I have said it many, many times, that the same apply to wires and resistors:  under a varying magnetic field, both wires and resistors behave like a voltage source in series with a resistor.

So, in Lewin's circuit we have four voltage sources. Two inside each resistor, and two in the wires. Right? But since in Lewin's circuit the only thing you manage to measure is the voltage drop on each resistor, because the wires, you say, is where the emf is being generated, this means that the voltage source inside the resistor is zero. Which contradicts what you've just said.

Your theory is bunk.
Title: Re: #562 – Electroboom!
Post by: thinkfat on December 04, 2021, 06:01:50 pm
Nah that's all bad probing only.
Title: Re: #562 – Electroboom!
Post by: jesuscf on December 04, 2021, 06:08:06 pm
Just to make sure that I understand you correctly: You are saying that if I have six 100 ohm 0402 resistors and measure the voltages across these resistors one at a time, I should get different voltage values when I have six 100 ohm 1206 resistors and repeat the voltage measurement across each resistor?

Bang!

What?  Did your brain just exploded?  Let me explain with numbers and equations, or as you call it 'mathematical tricks':

In the original Lewin ring, if we assume the resistors are much smaller than the ring we can approximate the voltage across each resistor as

\$I=\frac{1V}{100\Omega+900\Omega}=1mA\$

\$V_1=-1mA.100\Omega=-0.1V\$

\$V_2=1mA.900\Omega=0.9V\$

But now, let us make the 900 ohms resistor much bigger so that it occupies one quarter of the ring.  For the 100 ohms resistor we see no change, but now for the 900 resistor we have to subtract the induced EMF in the resistor itself and this is what we can measure across the terminals of the resistor:

\$V_2=1mA.900\Omega-\frac{1V}{4}=0.65V\$

EDIT: typo the last equation is for V2
Title: Re: #562 – Electroboom!
Post by: bsfeechannel on December 04, 2021, 06:24:28 pm
Just to make sure that I understand you correctly: You are saying that if I have six 100 ohm 0402 resistors and measure the voltages across these resistors one at a time, I should get different voltage values when I have six 100 ohm 1206 resistors and repeat the voltage measurement across each resistor?

Bang!

What?  Did your brain just exploded?  Let me explain with numbers and equations, or as you call it 'mathematical tricks':

In the original Lewin ring, if we assume the resistors are much smaller than the ring we can approximate the voltage across each resistor as

\$I=\frac{1V}{100\Omega+900\Omega}=1mA\$

\$V_1=-1mA.100\Omega=-0.1V\$

\$V_2=1mA.900\Omega=0.9V\$

But now, let us make the 900 ohms resistor much bigger so that it occupies one quarter of the ring.  For the 100 ohms resistor we see no change, but now for the 900 resistor we have to subtract the induced EMF in the resistor itself and this is what we can measure across the terminals of the resistor:

\$V_1=1mA.900\Omega-\frac{1V}{4}=0.65V\$

So, the voltage across the load of the secondary of a transformer depends on its size. If I have a resistor, or whatever, that is physically comparable to size of the transformer, the voltage will drop. If I choose a load whose size is much smaller than the dimensions of the loop, I will have a higher voltage.

Wow! Lewin, you son of a gun. You fooled us all.

jesuscf, you really know what you're talking about. Can you teach me electronics? Please!
Title: Re: #562 – Electroboom!
Post by: thinkfat on December 04, 2021, 06:38:42 pm
Self-destruction in 3, 2, 1, electroBOOM!

Classic.

PS: if that was the case, a transformer would never be lumpable.
Title: Re: #562 – Electroboom!
Post by: jesuscf on December 04, 2021, 06:50:45 pm
Just to make sure that I understand you correctly: You are saying that if I have six 100 ohm 0402 resistors and measure the voltages across these resistors one at a time, I should get different voltage values when I have six 100 ohm 1206 resistors and repeat the voltage measurement across each resistor?

Bang!

What?  Did your brain just exploded?  Let me explain with numbers and equations, or as you call it 'mathematical tricks':

In the original Lewin ring, if we assume the resistors are much smaller than the ring we can approximate the voltage across each resistor as

\$I=\frac{1V}{100\Omega+900\Omega}=1mA\$

\$V_1=-1mA.100\Omega=-0.1V\$

\$V_2=1mA.900\Omega=0.9V\$

But now, let us make the 900 ohms resistor much bigger so that it occupies one quarter of the ring.  For the 100 ohms resistor we see no change, but now for the 900 resistor we have to subtract the induced EMF in the resistor itself and this is what we can measure across the terminals of the resistor:

\$V_1=1mA.900\Omega-\frac{1V}{4}=0.65V\$

So, the voltage on the load on the secondary of a transformer depends on its size. If I have a resistor, or whatever, that is physically comparable to size of the transformer, the voltage will drop. If I choose a load whose size is much smaller than the dimensions of the loop, I will have a higher voltage.

Wow! Lewin, you son of a gun. You fooled us all.

jesuscf, you really know what you're talking about. Can you teach me electronics? Please!

How many turns in the secondary of the transformer?  How are the wires coming out of the transformer?  What is the internal resistance of the transformer and how it relates to the resistance of the resistor?  How is the magnetic field managed in the transformer?  But yes, the total voltage that comes out of a transformer depends on what you connect to it, where, and how!  And now lets go back to the original problem.  If you don't trust what I said about the circuit equivalent of the resistors in the original one-loop ring, just build it, measure it, and post your results.
Title: Re: #562 – Electroboom!
Post by: jesuscf on December 04, 2021, 06:52:15 pm
Self-destruction in 3, 2, 1, electroBOOM!

Classic.

PS: if that was the case, a transformer would never be lumpable.

At some point I thought you had a formal education in electrical engineering, but now I am pretty sure you don't!
Title: Re: #562 – Electroboom!
Post by: thinkfat on December 04, 2021, 07:03:12 pm
Shhh, I'm trying to imagine a transformer datasheet. I can see a graph! Load dimensions vs. output voltage!

Ngggghhhh!

Nope. Lost it.
Title: Re: #562 – Electroboom!
Post by: jesuscf on December 04, 2021, 07:06:35 pm
Shhh, I'm trying to imagine a transformer datasheet. I can see a graph! Load dimensions vs. output voltage!

Ngggghhhh!

Nope. Lost it.

Your diversionary tactics will not work with me.  Please concentrate in the original problem considered by Lewin and correctly solved by Notaros.
Title: Re: #562 – Electroboom!
Post by: Sredni on December 04, 2021, 09:49:42 pm
Check the attached pdf from Electromagnetics by Notaros, pages 279-280, example 6.6.

It is a curious coincidence that Notaros seems to have been mentored by the late Branko Popovic (he is the first one he thanks in the preface, and the age difference means he must have learned from him), the author I point to for the best definition of voltage from a didactic point of view. In fact, Popovic does a better job of defining voltage in a way that does not lend itself to confusion. Let me describe you the "Popovic manouver" using my convention for symbols, and by calling the electric scalar potential phi to make it crystal clear which is which.

On voltage and scalar potential difference

This is the definition of voltage as work done per unit charge to move from point A to point B along a given path \gamma. It is also the convention adopted by the IEC and it requires one to compute the path integral of the TOTAL electric field along the path \gamma from start point A to endpoint B (fun fact: the reason for the inversion of B and A in VA->B = VBAgamma is that when the electric fields admits a potential function we have E = - grad phi and the two minus signs cancel themselves, the rest is basic integral calculus)

(https://i.postimg.cc/Hkq7MW4b/image.png)

Since the TOTAL electric field is in general non conservative, the path integral will in general depend on the particular path joining A and B. Therefore there is not a potential function V(P) such that the path integral can be expressed as the difference between the values assumed by V in the endpoint and startpoint (which, considering the minus sign would have given an expression of the form V(B) - V(A) = VBA ) .
In this context I am using the notation "VABgamma" enclosed in quotes with the addition of a reference to the path \gamma to remind us what that voltage would be called IF we could treat it as a potential difference.
By the sheer power of superposition, we can mathematically decompose the total electric field Etot into its two components Ecoul and Eind.

Etot = Ecoul + Eind

The first is the conservative (or irrotational, or laminar) part that is due to the coloumbic force, while the latter is the nonconservative (or rotational, or solenoidal) induced electric field associated with the changing magnetic flux. Since integrals are linear operators, we can split the path integral that expresses voltage in two contributes

(https://i.postimg.cc/bJTG4xXR/image-2.png)

Now, the conservative part of the electric field Ecoul admits a potential function that we call the electric scalar potential phi (I am using this notation on purpose to make it clear it is not the non-existent potential function for the total electric field Etot), so we can express that component of voltage as a scalar potential difference phi(B) - phi](A) = phiB - phiA = phiBA

(https://i.postimg.cc/7Z25q88h/image-3.png)

Here I made the signs explicit, so that it is clear that the actual path-dependent voltage V - the one Ohm's law works with - is the sum of two components: the path-independent difference in electric scalar potential phiBA and the contribute of the inducted electric field (that can be expressed as a function of the magnetic vector potential A).
The partial component phiBA is the voltage that KVLers believe is the real and true voltage, and if we express it in this way

(https://i.postimg.cc/mZpkM2Th/image-4.png)

it is clear that we are subtracting a partial component from the actual 'full' voltage. VA->B = VBAgamma is the voltage that gives the full picture as it is expression of the one and only total electric field Etot that electrons and other charges can sense. While phiBA = phi(B) - phi](A) is just a partial component that gives you only half of the story.

As a side note, if we wish to make use of the magnetic vector potential A, the formulae become:

(https://i.postimg.cc/fRmR6cm0/image-5.png)

and (spoiler alert!)  this is the key to understand where that 'tiny batteries' model comes from.
Yeah, yeah, it's voltage sources since they are generally time-varying, but if we fix a particular instant in time we can represent them with battery symbols, they make it easier to see the polarities without cluttering the drawings.


The source(s) of confusion
Some textbook authors define voltage in the chapters for electrostatics as the difference in electric potential (because when there are no changing magnetic fields, using the conventions above we have VBAgamma = VBA = phiBA ). The problem with this approach is that instead of using a dedicated symbol such as phi for the electrostatic potential, they use V from the start - thereby allocating the symbol V for the path-independent voltage of electrostatics. Then, when they reach the quasi-static electrodynamics chapters they can't use V for the more general path-dependent voltage, and they are therefore forced to give two physical quantities to fully describe their systems (either V and A, or Ecoul and Eind). This is not a big deal, as long as one understands that in this context the path-independent component of voltage only gives a partial description of the physical system. The Helmoltz decomposition is well known and used in electrodynamics, but to completely describe your system you need to specify both the electric scalar potential \phi (that some authors call V) and the magnetic vector potential A.





KVLers, on the other hand, believe that the scalar potential difference phiBA is the only one and true voltage between points A and B, and that if you obtain a different value then it must mean you are committing a probing error. No, probing has nothing to do with the fact that the scalar potential difference alone only gives a partial description of your physical system when variable magnetic fields are present. This partial description is also what is represented by the 'tiny batteries' model of inductive components. This model and why it represents only half of the story, is the subject of another post, but a sneak preview is contained in this picture:

(https://i.postimg.cc/QdRGMwMM/Map-to-the-Ring-Quest.png)

Title: Re: #562 – Electroboom!
Post by: bsfeechannel on December 05, 2021, 02:42:37 am
If you don't trust what I said about the circuit equivalent of the resistors in the original one-loop ring, just build it, measure it, and post your results.

Look at the video posted by your fellow KVLer, fromjesse

https://www.youtube.com/watch?v=iDWv8QJrzUo&t=23 (https://www.youtube.com/watch?v=iDWv8QJrzUo&t=23)

He used big chunky 5W ceramic resistors, one is 150 Ω and the other is 47 Ω, 5% tolerance. The transformer is wound at about 200mV per turn, which gives you 1mA in the loop, and the voltages he measures are exactly what should be: around 150mV and 48 mV, respectively.

Man, I can't get enough of debunking your KVLer claims. And it's so easy since you KVLers provide all the evidence we need to.

Title: Re: #562 – Electroboom!
Post by: jesuscf on December 05, 2021, 05:40:19 am
If you don't trust what I said about the circuit equivalent of the resistors in the original one-loop ring, just build it, measure it, and post your results.

Look at the video posted by or fellow KVLer, fromjesse

He used big chunky 5W ceramic resistors, one is 150 Ω and the other is 47 Ω, 5% tolerance. The transformer is wound at about 200mV per turn, which gives you 1mA in the loop, and the voltages he measures are exactly what should be: around 150mV and 48 mV, respectively.

Man, I can't get enough of debunking your KVLer claims. And it's so easy since you KVLers provide all the evidence we need to.

What a coincidence that I just finished making a decent source to create a stronger varying magnetic field which allows for easier testing!

So, this what I just did:  I made a loop with a 100 ohms resistor to the left and a 900 ohms equivalent resistor to the right, just like in Lewin's experiment.  Except that the 900 ohm resistor to the right is made of nine 100 ohms resistors in series.   The voltage I measured in the single 100 ohm resistor is -42.4mV (using the same polarity as in Lewin's experiment).  That means that the voltage I would measure in a single 900 ohm resistor should be nine times that or 42.4*9=381 mV.  With those two voltages we can calculate a total induced EMF of 424 mV.

The radius of the ring is 8 cm.  That means that the perimeter is 2*pi*r or about 50 cm.  The nine 100 ohms resistors in series have a length of about 6.5 cm.  That means that the total induced EMF across the nine 100 ohm resistors in series should be about 55 mV.  With the voltage drop and the induced EMF in the nine 100 ohms resistors I can now calculate what I should measure between the terminals of the longer equivalent 900 ohm resistor: 381mv-55mv=326 mV.  For this 900 ohm resistor measurement, I had to be more careful not to pick up an induced voltage in the probing wires, so I used the setup shown in one of the attached pictures.

The voltage I measured is 332 mV! That is less than 2% error from the theoretical value of 326 mV!!!

EDIT: I uploaded one wrong picture, the last one, but I added the correct one in the next post.



Title: Re: #562 – Electroboom!
Post by: jesuscf on December 05, 2021, 05:44:06 am
In my previous post this image was not uploaded properly...
Title: Re: #562 – Electroboom!
Post by: bsfeechannel on December 05, 2021, 12:04:41 pm
For this 900 ohm resistor measurement, I had to be more careful not to pick up an induced voltage in the probing wires, so I used the setup shown in one of the attached pictures.

Cool! That's what I thought. Jesse measured the wrong voltage. He let his probes pick up induced voltage.

Thank you, jesuscf. We finally managed to show that Jesse's experiments are a hoax. No wonder he left the discussion. He knew he would be caught and exposed sooner or later by your imaginative intellect.
Title: Re: #562 – Electroboom!
Post by: Sredni on December 06, 2021, 06:35:56 am
This Lewin guy must be like one of those aliens from the movie "The Thing". He clearly brainwashed, or flat out replaced with his own clones, the whole of MIT.
I just found another EM textbook from MIT OCW that has the two resistors ring around a changing flux region... and it agrees with him.

(https://i.postimg.cc/ZqHW9tNj/screenshot-2.png)

It is also available online for free - surely a way to propagate these... How did Bob Duhamel of RSD Academy call these? "disagreeing with the vast majority of the scientific establishment, [...] disagreeing with the vast majority of textbooks, [...] disagreeing with the vast majority of professor of both electrical engineering and physics."

https://ocw.mit.edu/resources/res-6-002-electromagnetic-field-theory-a-problem-solving-approach-spring-2008/
https://ocw.mit.edu/resources/res-6-002-electromagnetic-field-theory-a-problem-solving-approach-spring-2008/about-this-book/
https://ocw.mit.edu/resources/res-6-002-electromagnetic-field-theory-a-problem-solving-approach-spring-2008/textbook-contents/

Not only Lewin must have brainwashed this author, but he clearly has a time machine because this textbook was published in 1979, even before Romer's paper. The sneaky little Dutch devil!

The Zahn-Romer-Lewin ring is in section "6.2.3: Transformer Action", "a) Voltages are not unique" (p. 411)


Edited to adjust a few sentences but mostly to fix the place of my 's'. I have developed a theory that the numbers of "s" in my posts is always correct. They are just scattered at random.


Title: Re: #562 – Electroboom!
Post by: jesuscf on December 06, 2021, 06:49:21 am
This Lewin guy must be like one of those aliens from the movie "The Thing". He clearly brainwashed the whole of MIT.
I just found another EM textbook from MIT OCW that has the two resistor rings around a changing flux region... and it agrees with him.
It is also available online for free - surely a way to propagate these... How did Bob Duhamel of RSD Academy call these? Nonstandard concepts that go against the science and engineering establishment? (I'll fetch the exact quote later).

https://ocw.mit.edu/resources/res-6-002-electromagnetic-field-theory-a-problem-solving-approach-spring-2008/
https://ocw.mit.edu/resources/res-6-002-electromagnetic-field-theory-a-problem-solving-approach-spring-2008/about-this-book/
https://ocw.mit.edu/resources/res-6-002-electromagnetic-field-theory-a-problem-solving-approach-spring-2008/textbook-contents/

Not only Lewin must have brainwashed this author, but he clearly has a time machine because this textbook was published in 1979, even before Romer's paper. The sneaky little Dutch devil!

The Zahn-Romer-Lewin ring is in section "6.2.3: Transformer Action", "a) Voltages are not unique" (p. 411)

At this point I am pretty sure you are just delusional.
Title: Re: #562 – Electroboom!
Post by: Sredni on December 06, 2021, 07:06:52 am
This Lewin guy must be like one of those aliens from the movie "The Thing". He clearly brainwashed the whole of MIT.
I just found another EM textbook from MIT OCW that has the two resistor rings around a changing flux region... and it agrees with him.
It is also available online for free - surely a way to propagate these... How did Bob Duhamel of RSD Academy call these? Nonstandard concepts that go against the science and engineering establishment? (I'll fetch the exact quote later).

https://ocw.mit.edu/resources/res-6-002-electromagnetic-field-theory-a-problem-solving-approach-spring-2008/
https://ocw.mit.edu/resources/res-6-002-electromagnetic-field-theory-a-problem-solving-approach-spring-2008/about-this-book/
https://ocw.mit.edu/resources/res-6-002-electromagnetic-field-theory-a-problem-solving-approach-spring-2008/textbook-contents/

Not only Lewin must have brainwashed this author, but he clearly has a time machine because this textbook was published in 1979, even before Romer's paper. The sneaky little Dutch devil!

The Zahn-Romer-Lewin ring is in section "6.2.3: Transformer Action", "a) Voltages are not unique" (p. 411)

At this point I am pretty sure you are just delusional.

You are clearly entitled to your opinion, but out of sheer curiosity...
Delusional about what?
About the fact that Lewin is just one of many university professors who know that voltage is no longer unique when magnetic fields change?
About the fact that KVLers think he is a lone wolf, a rogue scientist with an alternative theory because they never actually studied EM at uni level, even undergrad level?
About the fact that Lewin is an alien? (This might have been an exaggeration on my part, I have to admit it)

Title: Re: #562 – Electroboom!
Post by: HuronKing on December 06, 2021, 07:10:35 am
This Lewin guy must be like one of those aliens from the movie "The Thing". He clearly brainwashed the whole of MIT.
I just found another EM textbook from MIT OCW that has the two resistor rings around a changing flux region... and it agrees with him.
It is also available online for free - surely a way to propagate these... How did Bob Duhamel of RSD Academy call these? Nonstandard concepts that go against the science and engineering establishment? (I'll fetch the exact quote later).

https://ocw.mit.edu/resources/res-6-002-electromagnetic-field-theory-a-problem-solving-approach-spring-2008/
https://ocw.mit.edu/resources/res-6-002-electromagnetic-field-theory-a-problem-solving-approach-spring-2008/about-this-book/
https://ocw.mit.edu/resources/res-6-002-electromagnetic-field-theory-a-problem-solving-approach-spring-2008/textbook-contents/

Not only Lewin must have brainwashed this author, but he clearly has a time machine because this textbook was published in 1979, even before Romer's paper. The sneaky little Dutch devil!

The Zahn-Romer-Lewin ring is in section "6.2.3: Transformer Action", "a) Voltages are not unique" (p. 411)

Obviously another 'bad prober.'  ;D

It's clear from Romer's paper that he had a crisis of conscience about KVL - he tells an amusing anecdote when he says he first learned about the non-uniqueness of voltages from Rogers and Shoemaker during an oral exam. I wonder if Lewin had a similar revelation which is why he says "Kirchoff is for the birds and Faraday is not." Unfortunately for Lewin, it also appears his biggest crime was saying "Kirchoff is for the birds, and Faraday is not."

Title: Re: #562 – Electroboom!
Post by: bsfeechannel on December 06, 2021, 02:00:29 pm
The Zahn-Romer-Lewin ring is in section "6.2.3: Transformer Action", "a) Voltages are not unique" (p. 411)

Obviously another 'bad prober.'  ;D

Mehdi will say that Zahn is adherent to the "old definition of voltage" and that the majority of professors and authors now agree with his "new definition of voltage".

What is this new definition of voltage? It's one that makes Lewin look wrong and Mehdi look right before his audience comprised mostly of dimwits like himself.

When his audience realizes that he's wrong, he'll invent a new stupid explanation and he'll repeat it until it appears legitimized for them.

That's the same technique used by those who spread fake news. Truth sucks. Truth is boring. Truth requires evidence, rigor, proof. Telling you that to be a good engineer you will have to painstakingly study functions, calculus, vector calculus, Maxwell's equations sucks. Saying that, without any formal training in engineering in general and electromagnetism in specific, you can confront a seasoned engineering course professor and author at a respected technology institute is cool and takes no more than a 10 minute video quickly slapped together, intermingled with slapstick comedy jokes.

Quote
It's clear from Romer's paper that he had a crisis of conscience about KVL - he tells an amusing anecdote when he says he first learned about the non-uniqueness of voltages from Rogers and Shoemaker during an oral exam. I wonder if Lewin had a similar revelation which is why he says "Kirchoff is for the birds and Faraday is not." Unfortunately for Lewin, it also appears his biggest crime was saying "Kirchoff is for the birds, and Faraday is not."

No doubt. For many of us, the full implications of the electromagnetic phenomenon is a shocking event. One that gives room to the so called five stages to acceptance: denial, anger, bargain, depression and acceptance. At first there is an outright rejection, then a resistance. Resignation comes right before the acceptance of reality.

Title: Re: #562 – Electroboom!
Post by: thinkfat on December 06, 2021, 03:03:57 pm
It's actually much easier to cope with once you accept that the induced electric field is not IN the circuit. Then you stop looking for it and try modeling it with tiny batteries.
Title: Re: #562 – Electroboom!
Post by: jesuscf on December 06, 2021, 03:27:31 pm
This Lewin guy must be like one of those aliens from the movie "The Thing". He clearly brainwashed the whole of MIT.
I just found another EM textbook from MIT OCW that has the two resistor rings around a changing flux region... and it agrees with him.
It is also available online for free - surely a way to propagate these... How did Bob Duhamel of RSD Academy call these? Nonstandard concepts that go against the science and engineering establishment? (I'll fetch the exact quote later).

https://ocw.mit.edu/resources/res-6-002-electromagnetic-field-theory-a-problem-solving-approach-spring-2008/
https://ocw.mit.edu/resources/res-6-002-electromagnetic-field-theory-a-problem-solving-approach-spring-2008/about-this-book/
https://ocw.mit.edu/resources/res-6-002-electromagnetic-field-theory-a-problem-solving-approach-spring-2008/textbook-contents/

Not only Lewin must have brainwashed this author, but he clearly has a time machine because this textbook was published in 1979, even before Romer's paper. The sneaky little Dutch devil!

The Zahn-Romer-Lewin ring is in section "6.2.3: Transformer Action", "a) Voltages are not unique" (p. 411)

Obviously another 'bad prober.'  ;D

It's clear from Romer's paper that he had a crisis of conscience about KVL - he tells an amusing anecdote when he says he first learned about the non-uniqueness of voltages from Rogers and Shoemaker during an oral exam. I wonder if Lewin had a similar revelation which is why he says "Kirchoff is for the birds and Faraday is not." Unfortunately for Lewin, it also appears his biggest crime was saying "Kirchoff is for the birds, and Faraday is not."

Definitively another guy that doesn't understand that the voltage between nodes VAD depends only on the unique path of the original circuit, a circuit which is not changing shape or moving.  When measuring, the voltage the instrument displays depends on the path of the instrument wires, but that is not the voltage VAD, it is the voltage at the instrument!  With that understanding it is a trivial task to find the correct and unique voltage between nodes VAD which is independent of the path of the instrument wires.  So, how do you find the correct voltage between nodes 'A' and 'D', VAD?  Using KVL of course!
Title: Re: #562 – Electroboom!
Post by: Sredni on December 07, 2021, 05:20:22 pm
Quote
https://ocw.mit.edu/resources/res-6-002-electromagnetic-field-theory-a-problem-solving-approach-spring-2008/
The Zahn-Romer-Lewin ring is in section "6.2.3: Transformer Action", "a) Voltages are not unique" (p. 411)
Definitively another guy that doesn't understand that the voltage between nodes VAD depends only on the unique path of the original circuit, a circuit which is not changing shape or moving. 

Of course. The MIT is notoriously known for its scarcely prepared professors of physics and engineering.
Here is yet another professor who does not understand the basics of electromagnetism!
Shame on you, MIT!!!
Listen to the guys in a garage, instead. They have an oscilloscope, so they clearly know what they are talking about.

Quote
When measuring, the voltage the instrument displays depends on the path of the instrument wires, but that is not the voltage VAD, it is the voltage at the instrument!  With that understanding it is a trivial task to find the correct and unique voltage between nodes VAD which is independent of the path of the instrument wires.  So, how do you find the correct voltage between nodes 'A' and 'D', VAD?  Using KVL of course!

I would love to see you get the voltage on the conductors that join the resistors in this square ring, asymmetrically placed inside a square toroidal core, by simply using KVL.
Keep in mind that the induced field Eind is no longer directed along concentric circles, and your ring is now square.
Good luck using KVL.

Title: Re: #562 – Electroboom!
Post by: Sredni on December 08, 2021, 04:42:34 am

The tiny batteries model, or: "You don't know the half of it"

"You keep using that word.
I don't think it means what you think it means.
"
 -- Inigo Montoya

Here is something that KVLers will find... unconceivable. (I will come back to this post for formatting and for adding the pictures in a couple of days)

Let's start from Faraday's law, in its modern integral form

    circulation of E = - d/dt flux of B

We already established that there is no particular place on the path where we can localize the inductive EMF represented by the surface integral on the right. Even if we split the circulation of E in the path integrals along several segments we can partition the path into, none of the segment can account for the inductive EMF on the right. If we bring the whole surface integral on the left hand side of the equation we get

    circulation of E + d/dt flux of B = 0

But this won't make E conservative.
There is, though, some mathematical trick we can pull to transform the surface integral of B into a line integral of something related to B and then incorporate this something into the integrand of the original path integral of E.

SPOILER ALERT GALORE:
What we will find is the circulation of something that is not E, but something else that has zero circulation and as such admits a scalar potential (which I will call phi to avoid any confusion with voltage).

Spoiler alert part I: it turns out that 'that something else' is the conservative part of the electric field E, a quantity that I will denote with Ecoul.

Spoiler alert part II: the electric field E can be seen as the composition of its conservative (irrotational) part Ecoul with its non-conservative (solenoidal) part Eind. This is the Helmoltz decomposition of a generic vector field and it holds under very general assumption (basically always):

    E = Ecoul + Eind

Moreover, to avoid confusion, I will also use the symbol Etot, instead of just E, to the denote the electric field.    

    Etot = Ecoul + Eind

But it has to be clear that Etot(x,y,z,t) (that is E(x,y,z,t) ) is the one and only electric field an electron, or any test charge, will experience at the point (x,y,z) at time t. Electrons do not have an accountant to tell them how much of the electric field they experience is due to the changing flux and how much to the displaced charge. The just react to the whole shebang.

We can start to massage our relation starting from the the vector potential A. Since div B = 0 we can express the magnetic field B as the curl of another vector field A:

    B = curl A

Our Faraday equation will become

    circulation of Etot + d/dt flux of curl A = 0

We can now use Stokes' theorem to turn the surface integral of the curl of A into the path integral of A along the boundary of the surface

    circulation of Etot + d/dt circulation of A = 0

If the ring is stationary we can bring the time derivative inside the integral

    circulation of Etot + circulation of dA/dt = 0

Now we can incorporate the second circulation integral into the first and we obtain the circulation of a field that is conservative

    circulation of (Etot + dA/dt) = 0

But notice what the integrand is: it is Ecoul, the conservartive PART of the total electric field

    Ecoul = Etot - Eind = Etot - (-dA/dt) = Etot + dA/dt

Ecoul has zero circulation and as such admits a potential function. This potential function is the scalar electric potential phi (some call it V, generating confusion) that obeys what we could call KsPDL (Kirchhoff's scalar Potential Difference Law).
It represents a partial description of the system because phi completely describes Ecoul, but only PARTIALLY describes Etot. In order to know Etot (the actual electric field felt by the charges) we need to know BOTH potentials: the electric scalar potential phi AND the magnetic vector potential A.

    Etot = - grad phi - dA/dt

Where does the tiny batteries model come from?
The tiny batteries model is not a model of the complete physical system "ring with resistors" described by Etot, but only a model of the effects of the partial, conservative part Ecoul alone. As a matter of fact, the tiny batteries represent the term associated with the negative of the induced field Eind = -dA/dt that we need to strip from Etot to get the conservative field Ecoul

    Ecoul = Etot - Eind = Etot - (-dA/dt)

This figure show the fields inside the conductor and the resistors of Lewin's ring for the case of zero-resistance wires.

(https://i.postimg.cc/QdRGMwMM/Map-to-the-Ring-Quest.png)

Only the ring on the right expresses a conservative field, and as such it can be represented by tiny lumped components obeying K"V"L (it's KsPDL, actually). I am using tiny batteries because it makes easier to see at first glance the polarity.It would be more accurate to use tiny generators, since in general the EMF contribute is time-varying, but if we freeze the system at a particular instant in time, we can pretend we are dealing with tiny batteries.

fig model with tiny batteries, no resistance in wires

The model can be generalized to the case of resistive wires. In practice we break the ring in tiny segments for which we have two contributes: the tiny resistor that represents the conductivity of the material and the tiny battery that represent the 'EMF eraser' that will recover Ecoul from Etot.

fig model with tiny batteries and resistors

This being the model of a mathematical part of the system, it will break the laws of physics obeyed by the actual, complete, system. Case in point: Ohm's law. Consider any portion of copper conductor and we will see that a piece of metal with a resistance that is almost zero will appear to drop a voltage (actually it's a scalar electric potential difference) of, say, 0.25 volts when a current of just 1 mA flows through it.

-buffering-

Title: Re: #562 – Electroboom!
Post by: Jesse Gordon on December 13, 2021, 01:21:12 am
Regarding the "tiny voltage sources", the funny thing (which is a bit sad also) is that all the "experimental proof" by Jesse and e.g. Cyriel Mabilde are just cunning demonstrations of how to create paths that enclose a variable amount of magnetic flux. The sad part is that they're unable to see it and will keep claiming it is proof of "KVL holds". But in reality they're not measuring a gradual voltage build-up in the "Lewin Ring", but just the EMF induced in their measurement loop. This under the condition that there is only negligible current flowing through the ring and the measurement loop, of course.

The funny and sad thing is that you don't see that by measuring the voltage induced across a half of the dB/dt area we're actually unambiguously physically measuring the actual voltage induced in that half !

Think about it. Look at the circular area of dB/dt of my Lewin Clock. Now imagine we removed exactly half of that but maintained  D shaped aree at the other half with exactly the same dB/dt in that half as was there before. The dB/dt exactly the same.

Running the probe lead straight through the center of the circle does objectively unambiguously physically measure the voltage induced in that half of the winding.

And thus KVL holds my my Lewin Clock topology.
Title: Re: #562 – Electroboom!
Post by: Jesse Gordon on December 13, 2021, 01:32:04 am
For this 900 ohm resistor measurement, I had to be more careful not to pick up an induced voltage in the probing wires, so I used the setup shown in one of the attached pictures.

Cool! That's what I thought. Jesse measured the wrong voltage. He let his probes pick up induced voltage.

Nonsense. I practiced safe probing, and KVL works. Welcome to the real world kiddo  :-DD :-DD :-DD.
Quote
Thank you, jesuscf. We finally managed to show that Jesse's experiments are a hoax.
My experiment is not a hoax.  The reason you couldn't see my experiment in my garage is because you were in your mums basement at the time  :-DD :-DD :-DD.
Quote
No wonder he left the discussion. He knew he would be caught and exposed sooner or later by your imaginative intellect.
I didn't leave the discussion, unlike you, I don't live in my mom's basement so I have to work and I have friends and family to interact with so I can't be arguing on here every day.

I told you I'd limit my posting days to 2 days a week here. Since I was too busy last week that gives me 4 days here this week if I have time LOL.

Besides, I've already determined that you don't know what you're talking about by how you absolutely refused to answer the questions I asked. Sure, you would ask your own question and answer that, but you absolutely refused to actually exactly answer my question, which I will repeat for you below in case you forgot it.

Question: In the following diagram, in a real life physical lab test performed with real (time synchronized) volt meters with a real transformer and real resistors CONNECTED AS SHOWN, will the readings of all the volt meters sum to zero, within the accuracy and resolution limitations of the volt meters? YES or NO.

(Or if you believe SOMETIMES is the answer, then answer SOMETIMES and explain one scenario for a YES condition and one scenario for a NO condition WITH THE VOLT METERS AND ALL COMPONENTS CONNECTED AS SHOWN - Running additional conductors through the transformer core is not allowed - nor is removing existing conductors from through the transformer core!)

(https://i.postimg.cc/jdJntBXT/20211128-121506.jpg)


Title: Re: #562 – Electroboom!
Post by: Jesse Gordon on December 13, 2021, 02:15:30 am
Of course. The MIT is notoriously known for its scarcely prepared professors of physics and engineering.
Here is yet another professor who does not understand the basics of electromagnetism!
Shame on you, MIT!!!
Listen to the guys in a garage, instead. They have an oscilloscope, so they clearly know what they are talking about.

You know, right there is rock solid proof that you don't know what you're talking about.

Think about it. You keep going on about us "guys in their garages." That is like the purest form of argumentum ad hominem because being in a garage has absolutely nothing whatsoever to do with whether we are right or not.

The fact that you argue based on personal attacks regarding completely unrelated issues which have no bearing on whether we're right tells me that's all you got.

Since you brought up my garage, why don't you get a scope for your mom's basement and do some experiments? Your moms basement will be just as cool as my garage if you add a scope for sure.

You could even convert that 1980's CRT tv you got down there into a scope if you want..

Want digital storage? well how about analog storage? You can convert your VHS recorder to a waveform storage unit.

You already accidentally admitted to enough facts that you've really cornered yourself.

You gloriously cited this this textbook:

https://i.postimg.cc/sf4j3HbF/Desoer-Kuh.jpg

For ease of reading and searchability, I reproduce it here:

Quote from: Desoer-Kuh

Lumped circuits are obtained by connecting lumped elements. Typical
lumped elements are resistors, capacitors, inductors, and transformers.
We have encountered them in the laboratory, and we can see them in our
radio sets. The key property associated with lumped elements is their
small size (compared to the wavelength corresponding to their normal
frequency of operation.)  From the more general electromagnetic field point
of view, lumped elements are point singularities; that is, they have negligible
physical dimensions. In this way they are similar to a particle.
Lumped elements may have two terminals, as in a resistor, or more than
two terminals, as in a transformer or transistor. For two-terminal
lumped elements, it can be shown that the general laws governing the
electromagnetic field, together with the restriction on physical size indicated
above, imply that at all times the current entering one terminal is
equal to the current leaving the other terminal, and that the voltage difference
between the two terminals can be unambiguously defined by physical
measurements.  Thus, for two-terminal lumped elements, the current
through the element and the voltage across it are well-defined quantities. For
lumped elements with more than two terminals, the current entering any
terminal and the voltage across any pair of terminals are well defined at all times.

  For the remainder of this book, any interconnection of the lumped
elements such that the dimensions of the circuit are small compared with the
wavelength associated with the highest frequency of interest will be called
a lumped circuit.
  As long as this restriction on the size of the circuit holds, Kirchhoff's
current and voltage laws (to be discussed in Secs. 3 and 4) are valid. This
restriction is a consequence of the fact that Kirchhoff's laws are approximations
of Maxwell's celebrated equations, which are the general laws
of the electromagnetic field.  The approximation is analogous to the fact
that Newton's laws of classical mechanics are approximations to the laws
of relativistic mechanics.  Even though they are approximations, the laws
of Newton and Kirchhoff can be applied to a large number of practical


Clearly, the output voltage of my transformer secondaries in the diagram directly below are "unambiguously physically defined by physical
measurements" as described in the revered textbook cited directly above.

(https://i.postimg.cc/jdJntBXT/20211128-121506.jpg)
https://i.postimg.cc/jdJntBXT/20211128-121506.jpg (https://i.postimg.cc/jdJntBXT/20211128-121506.jpg)

The fact that you won't even concede that KVL holds in this case shows you really don't know what's going on, you don't have a scope, and you've got less training on the topic than I do if that's even possible LOL.

KVL clearly holds (and should hold!) both on paper and in the garage!  :-DD :-DD :-DD

What else is there even to talk about?

Might as well pull up your crystal ball. BS has one you can borrow if you like, it kept shorting out on him so he had to get a new one. Thinkfat has one you can borrow a well, it's only been used once.

(https://i.postimg.cc/5NsPHxQ4/3Gypsies.jpg)
https://i.postimg.cc/5NsPHxQ4/3Gypsies.jpg (https://i.postimg.cc/5NsPHxQ4/3Gypsies.jpg)
Title: Re: #562 – Electroboom!
Post by: bsfeechannel on December 13, 2021, 02:17:11 am
Nonsense. I practiced safe probing, and KVL works. Welcome to the real world kiddo  :-DD :-DD :-DD.

"Safe probing" is for the weak. Real men measure the "interference".
Title: Re: #562 – Electroboom!
Post by: jesuscf on December 13, 2021, 03:26:04 am
Since this thread has sprung back to live, let me share some information to further demolish team's Lewin position.  Check out this marvelous quote from an old book:

Quote
Let the components of the electromotive intensity be X, Y, Z.
The electromotive intensity at any point is the resultant force on a unit of positive electricity placed at that point.  It may arise (1) from electrostatic action, in which case if V is the potential,
 \$X=-\frac{dV}{dx},Y=-\frac{dV}{dy},Z=-\frac{dV}{dz};    (1)\$
or (2) from electromagnetic induction, the laws of which we shall afterwards examine; or (3) from thermoelectric or electrochemical action at the point itself, tending to produce a current in a given direction.
We shall in general suppose that X, Y, Z represent the components of the actual electromotive intensity at the point, whatever be the origin of the force, but we shall occasionally examine the result of supposing it entirely due to variation of potential.

Where that quote comes from? From this book: A Treatise On Electricity and Magnetism by James Clerk Maxwell (page 418, Dover edition from 1954 based in third edition published in 1891)!  So, should we treat the EMF due to electromagnetic induction differently from the point of view of KVL to any other EMF?  I think Maxwell is telling us 'no' way back in time from 1873 when he wrote the book!  If you continue reading, the rest of the explanation looks pretty much like KVL and KCL to me, which he introduced earlier in the same book starting in page 406.




Title: Re: #562 – Electroboom!
Post by: Jesse Gordon on December 13, 2021, 03:39:41 am
Nonsense. I practiced safe probing, and KVL works. Welcome to the real world kiddo  :-DD :-DD :-DD.
"Safe probing" is for the weak. Real men measure the "interference".

You know, you guys have shamed people for using scopes in their garages for a while now, I think it's high time you told us where you're doing your experiments. At least we're doing experiments. It's high time you tell us something about your degrees and experience.

You told us how you like to take calculators apart but that it's too noisy in your "lab" so you have to use a text to speech program for your video. "Wow, look at the amount of corrosion on those self tapping screws! Look what we have here, the leakage may have effected the tracks. ....  This is called multiplexing, so I read on Wikipedia. ... Now the calculator is restored to its glory! Good night and stay beautiful! "  (https://youtu.be/-qJebvO23Y8)

And here's you repairing a $20 kitchen scale: https://youtu.be/y3BRjA6jOyA

And here's you repairing a camera which you got from a dumpster: https://youtu.be/vhG3WvryEW0

And you fixing an old worn out electric drill motor: https://youtu.be/bDv9G7ezGss

Those are all super cool projects, but look. You and I have more in common than you think. You're just another guy like me who takes things apart and looks on wikipedia and does our best to figure out what's going on and definitely gets old junk out of dumpsters and from friends and fixes it up.

If either one of us had nice big fat degrees, we wouldn't be dumpster diving and fixing up common easily replaced household appliances. We probably wouldn't even be here arguing about this.

Hey, you used to have a scope: https://youtu.be/mT6FGdfeCNo?t=275

Again, just like me, we both have (or do you still have?) a cheapo scope. (Mine's Siglent which is another cheapo brand similar to your Regol.) (My vintage CRT analog scopes are Tek, but the DSO's are Siglent.)

If we were big degreed professionals, we'd have Tek/HP/Agilent/Keysight scopes.

But dood, we're both hobbiests. And that's a wonderful thing to be. Why not admit it?

The reason we do things in our garages is because that's what we have. Neither one of us pulls stuff from the dumpster then walks into our PHD level lab complex and goes into a clean room to fix whatever we pulled from the dumpster.

So what's your story? You were taking EE and dropped out? Was the scope yours, or did it belong to the school?

Why pass yourself off as something you're not? Who you really are is every bit as cool, and it's OK that there's things you don't understand -- goodness there's things I don't understand, like why you won't answer my question below about my transformer loop.

In cased you missed my question, I'd really appreciate you answering it.

Question: In the following diagram, in a real life physical lab test performed with real (time synchronized) volt meters with a real transformer and real resistors CONNECTED AS SHOWN, will the readings of all the volt meters sum to zero, within the accuracy and resolution limitations of the volt meters? YES or NO.

(Or if you believe SOMETIMES is the answer, then answer SOMETIMES and explain one scenario for a YES condition and one scenario for a NO condition WITH THE VOLT METERS AND ALL COMPONENTS CONNECTED AS SHOWN - Running additional conductors through the transformer core is not allowed - nor is removing existing conductors from through the transformer core!)

(https://i.postimg.cc/jdJntBXT/20211128-121506.jpg)


Title: Re: #562 – Electroboom!
Post by: Jesse Gordon on December 13, 2021, 03:49:04 am
For this 900 ohm resistor measurement, I had to be more careful not to pick up an induced voltage in the probing wires, so I used the setup shown in one of the attached pictures.

Cool! That's what I thought. Jesse measured the wrong voltage. He let his probes pick up induced voltage.

Thank you, jesuscf. We finally managed to show that Jesse's experiments are a hoax. No wonder he left the discussion. He knew he would be caught and exposed sooner or later by your imaginative intellect.

By the way, which  measurement are you saying was where I measured the wrong voltage? The iron-core or the Lewin Clock?
Title: Re: #562 – Electroboom!
Post by: Jesse Gordon on December 13, 2021, 04:04:04 am
If you don't trust what I said about the circuit equivalent of the resistors in the original one-loop ring, just build it, measure it, and post your results.

Look at the video posted by your fellow KVLer, fromjesse

https://www.youtube.com/watch?v=iDWv8QJrzUo&t=23 (https://www.youtube.com/watch?v=iDWv8QJrzUo&t=23)

He used big chunky 5W ceramic resistors, one is 150 Ω and the other is 47 Ω, 5% tolerance. The transformer is wound at about 200mV per turn, which gives you 1mA in the loop, and the voltages he measures are exactly what should be: around 150mV and 48 mV, respectively.

Man, I can't get enough of debunking your KVLer claims. And it's so easy since you KVLers provide all the evidence we need to.

And then in another post:

For this 900 ohm resistor measurement, I had to be more careful not to pick up an induced voltage in the probing wires, so I used the setup shown in one of the attached pictures.

Cool! That's what I thought. Jesse measured the wrong voltage. He let his probes pick up induced voltage.

Thank you, jesuscf. We finally managed to show that Jesse's experiments are a hoax. No wonder he left the discussion. He knew he would be caught and exposed sooner or later by your imaginative intellect.

What do you mean you finally managed to show what again? in other words, you've secretly believed it was a hoax this whole time, and now you think you got proof of it? If it was a hoax you could have demonstrated it in an instant with your scope.

But there was nothing hoaxical about my measurement. I used a real transformer and a real volt meter and measured exactly as I claimed and if you did the same thing you'd get the same results.

Are you seriously comparing my iron-core measurements to the open-pancake air core measurements?

You are clueless aren't you!
Title: Re: #562 – Electroboom!
Post by: Jesse Gordon on December 13, 2021, 04:30:51 am
Since this thread has sprung back to live, let me share some information to further demolish team's Lewin position.  Check out this marvelous quote from an old book:

Quote
Let the components of the electromotive intensity be X, Y, Z.
The electromotive intensity at any point is the resultant force on a unit of positive electricity placed at that point.  It may arise (1) from electrostatic action, in which case if V is the potential,
 \$X=-\frac{dV}{dx},Y=-\frac{dV}{dy},Z=-\frac{dV}{dz};    (1)\$
or (2) from electromagnetic induction, the laws of which we shall afterwards examine; or (3) from thermoelectric or electrochemical action at the point itself, tending to produce a current in a given direction.
We shall in general suppose that X, Y, Z represent the components of the actual electromotive intensity at the point, whatever be the origin of the force, but we shall occasionally examine the result of supposing it entirely due to variation of potential.

Where that quote comes from? From this book: A Treatise On Electricity and Magnetism by James Clerk Maxwell (page 418, Dover edition from 1954 based in third edition published in 1891)!  So, should we treat the EMF due to electromagnetic induction differently from the point of view of KVL to any other EMF?  I think Maxwell is telling us 'no' way back in time from 1873 when he wrote the book!  If you continue reading, the rest of the explanation looks pretty much like KVL and KCL to me, which he introduced earlier in the same book starting in page 406.

Indeed. And why wouldn't he? It makes sense. Doing otherwise makes no sense.

There's just no logical reason for Team Lewin to keep on going.

If an element is small compared to the wavelength, and the current going in one terminal equals the that going out the other, and the voltage can be physically unambiguously physically defined by measurement, then there's no reason KVL doesn't hold.

Btw, great job on the mini Lewin Ring setup!
Title: Re: #562 – Electroboom!
Post by: Sredni on December 13, 2021, 06:36:00 am
Ah, good, the "Treatise".
Are we going to bring back ether, as well?

And yes, the EMF due to induction is a very special kind of EMF.
All the other ones fall into the circulation integral.

And, Jesse, when I say "listen to the guys in a garage" I am not comparing them to me, but to the university professors who wrote books and papers where this problem was carefully analyzed.
Purcell, Morin, Roche, Ramo, Whinnery, VanDuzer, Romer, Rosser, Zahn, Haus, Melcher, Nicholson, Brandao Faria, ...
Title: Re: #562 – Electroboom!
Post by: jesuscf on December 13, 2021, 06:51:02 am
Ah, good, the "Treatise".
Are we going to bring back ether, as well?

And yes, the EMF due to induction is a very special kind of EMF.
All the other ones fall into the circulation integral.

And then you wonder why I say you are delusional!  Dude, it is Maxwell himself (from "Maxwell equations" nonetheless) saying that the inductive EMF is to be treated exactly the same as the other kinds of EMF!!!

Here is another historic reference for you to digest.  The attached image is from the paper Complex Quantities and their use in Electrical Engineering; Charles Proteus Steinmetz; Proceedings of the International Electrical Congress, Chicago, IL; AIEE Proceedings, 1893; pp.33-74.  Notice how the author is using KVL with induced EMFs.

(https://www.eevblog.com/forum/amphour/562-electroboom!/?action=dlattach;attach=1348229;image)
Title: Re: #562 – Electroboom!
Post by: Sredni on December 13, 2021, 06:56:52 am
You are still stuck at the difference between KVL and extended KVL?
Oh, dear.

Wanna see how I can treat induction EMF as any other EMF?
Simple, I localize it, just like with lumped circuits with inductors, transformers, motors...
You're still stuck there?

It's written on Hayt's book, the one you quoted three or four years ago.
Title: Re: #562 – Electroboom!
Post by: jesuscf on December 13, 2021, 07:13:38 am
You are still stuck at the difference between KVL and extended KVL?
Oh, dear.

Wanna see how I can treat induction EMF as any other EMF?
Simple, I localize it, just like with lumped circuits with inductors, transformers, motors...
You're still stuck there?

It's written on Hayt's book, the one you quoted three or four years ago.

Now it is official: according to Maxwell (the guy from "Maxwell equations") and Steinmetz (the guy from "phasor analysis") you don't know what you are talking about!

By the way, I think you didn't understand that Hayt was explaining the contribution of inductive EMFs to KVL using Faraday's law from an historic perspective.  I thought that was pretty clear from the text.
Title: Re: #562 – Electroboom!
Post by: jesuscf on December 13, 2021, 05:02:35 pm
Ah, good, the "Treatise".
Are we going to bring back ether, as well?

And yes, the EMF due to induction is a very special kind of EMF.
All the other ones fall into the circulation integral.

And, Jesse, when I say "listen to the guys in a garage" I am not comparing them to me, but to the university professors who wrote books and papers where this problem was carefully analyzed.
Purcell, Morin, Roche, Ramo, Whinnery, VanDuzer, Romer, Rosser, Zahn, Haus, Melcher, Nicholson, Brandao Faria, ...

I see you edited you message.  Since I noticed that you just dump references without reading and/or understanding them, and that you also quite often just lie, allow me to check a couple of these references:

Electricity and Magnetism by Purcell E.M., Morin D.J:

They have Problem 7.4 similar to Lewin's problem.  No solution to the problem given!  The problem asks "What will each voltmeter read?" but it is not asking what is the voltage between nodes A and D!  I have the impression that the problem wants to point out that you have to be careful when you measure voltages and the instrument probes are under the effect of an external varying magnetic field...

Fields and Waves in Communication Electronics by Simon Ramo, John R. Whinnery, Theodore Van Duzer:

At the beginning of chapter 4 they say this: "Kirchhoff's two laws provide the basis for classical circuit theory.  We begin with the voltage law as a way of reviewing the basic element values of lumped-circuit theory.  The law states that for any closed loop of a circuit, the algebraic sum of voltages for the individual branches is zero... the basis for this is Faraday's law for a closed path, written as..."  And then, they proceed to use KVL with induced EMFs to solve a bunch of stuff.  How is that supposed to support your position?
Title: Re: #562 – Electroboom!
Post by: jesuscf on December 13, 2021, 05:33:29 pm
Since this thread has sprung back to live, let me share some information to further demolish team's Lewin position.  Check out this marvelous quote from an old book:

Quote
Let the components of the electromotive intensity be X, Y, Z.
The electromotive intensity at any point is the resultant force on a unit of positive electricity placed at that point.  It may arise (1) from electrostatic action, in which case if V is the potential,
 \$X=-\frac{dV}{dx},Y=-\frac{dV}{dy},Z=-\frac{dV}{dz};    (1)\$
or (2) from electromagnetic induction, the laws of which we shall afterwards examine; or (3) from thermoelectric or electrochemical action at the point itself, tending to produce a current in a given direction.
We shall in general suppose that X, Y, Z represent the components of the actual electromotive intensity at the point, whatever be the origin of the force, but we shall occasionally examine the result of supposing it entirely due to variation of potential.

Where that quote comes from? From this book: A Treatise On Electricity and Magnetism by James Clerk Maxwell (page 418, Dover edition from 1954 based in third edition published in 1891)!  So, should we treat the EMF due to electromagnetic induction differently from the point of view of KVL to any other EMF?  I think Maxwell is telling us 'no' way back in time from 1873 when he wrote the book!  If you continue reading, the rest of the explanation looks pretty much like KVL and KCL to me, which he introduced earlier in the same book starting in page 406.

Indeed. And why wouldn't he? It makes sense. Doing otherwise makes no sense.

There's just no logical reason for Team Lewin to keep on going.

If an element is small compared to the wavelength, and the current going in one terminal equals the that going out the other, and the voltage can be physically unambiguously physically defined by measurement, then there's no reason KVL doesn't hold.

Btw, great job on the mini Lewin Ring setup!

Yes, most textbooks put emphasis in the fact that the circuit has to be much smaller when compared to the wavelength of the fields before using Kirchoff's laws... them proceed to explain transmission lines were the wavelengths are comparable to the size of the circuit!  How they solve the problem?  Easy, hash the equivalent circuit into lots of smaller pieces!  It may not be pretty or easy to solve, but it certainly works!  Thankfully in many cases the circuit under consideration is in an steady state regime, where we can again use simple lumped representations for very large parts of the circuit.

I am pretty happy with my ring setup.  For its simplicity, it works surprisingly well.  I have made lots of experiments already.  Lately, I have been adding capacitors to the ring circuit and using my fancy multimeters (BM869, DMM4050, and Fluke45) at home to measure the voltages, showing in each case that KVL works as expected (consistently, with less that 2% error).  At work I have easy access to dozens of HP34401A, DMM4040, and FLuke45 multimeters. I am pretty tempted to measure all the voltages at once using a stack of those... but that will have to wait until work slows down a bit.

Title: Re: #562 – Electroboom!
Post by: Sredni on December 13, 2021, 11:39:12 pm
I see you edited you message. 
I try not to make an excessive number of posts.

Quote
Since I noticed that you just dump references without reading and/or understanding them, and that you also quite often just lie, allow me to check a couple of these references:

Electricity and Magnetism by Purcell E.M., Morin D.J:
They have Problem 7.4 similar to Lewin's problem.  No solution to the problem given!  The problem asks "What will each voltmeter read?" but it is not asking what is the voltage between nodes A and D!  I have the impression that the problem wants to point out that you have to be careful when you measure voltages and the instrument probes are under the effect of an external varying magnetic field...
page 358
(https://i.postimg.cc/kGXzD9jR/purcell-faraday-2.jpg)

A little excerpt (bold mine) which some have difficulty in understanding:

"...Kirchhoff Loop rule is no longer valid"

I wonder what that could mean... Why, oh why couldn't they be any clearer???
But let's see the solution of the problem on page 710 (again, bold mine)

"The moral of this problem is that if a setup contains changing flux, it makes no sense to talk about the voltage difference (that is, the value of −integral of E·ds) between two points. It is necessary to state the path over which −integral of E · ds is calculated."

Still of the opinion it does not support my position, which is that KVL is no longer valid and voltage depends on path?
 :palm:

Quote
Fields and Waves in Communication Electronics by Simon Ramo, John R. Whinnery, Theodore Van Duzer:

At the beginning of chapter 4 they say this: "Kirchhoff's two laws provide the basis for classical circuit theory.  We begin with the voltage law as a way of reviewing the basic element values of lumped-circuit theory.  The law states that for any closed loop of a circuit, the algebraic sum of voltages for the individual branches is zero... the basis for this is Faraday's law for a closed path, written as..."  And then, they proceed to use KVL with induced EMFs to solve a bunch of stuff.  How is that supposed to support your position?

I'll show you how. Page 174 of the third edition

"let us take a closed integral of electric field along the conductor of the coil, returning by the path across the terminals. Since the contribution along the part of the path which follows the conductor is zero, all the voltage appears ACROSS the terminals.

I'll translate this for you: voltage ALONG the filament of the coil = 0; voltage ACROSS the terminals = V not zero.
KVL cannot survive two values of voltage between the same two points. Unless you use a trick and pretend that there is only one voltage, that ACROSS the coil, and pretend its a potential difference.

Read page 179

"In the above we seem to be treating voltage as potential difference when we take voltage of a node with respect to the chosen reference, but note that this is only after the circuit is defined and we are only breaking up the integral of E.dl into its contributions over the various branches. As illustrated in the preceding section, we do have to define the path carefully whenever there are inductances or other elements with contribution to voltage from Faraday's law."

But hey, and this is related to the above, have you noticed that Jesse is repeatedly posting the same image of a circuit with a transformer, two outputs on a circuit with resistors and a lot of voltmeters? Suppose one turn around the core gives you an emf of 1V.
Can you place the values of your "path independent voltage" inside all those voltmeters? Because I have a hunch you cannot do it, using the voltage that according to you builds up in the coil.
I can easily do it using my definition of voltage, in accordance with Purcell, Ramo etc, Haus etc, Brandao Faria, Zahn, etc... but I wonder if you have any idea of what 'your voltage' (or the 'McDonald voltage', if you prefer to call it that way) is, in that context.
(Pretty sure Jesse doesn't know either, because I've asked and he refused to answer)


Edit: further clarifies what "voltage" I want you to place inside those voltmeters.
I will place the values of "my" path dependent voltage once either you or Jesse post your solution.
Title: Re: #562 – Electroboom!
Post by: jesuscf on December 14, 2021, 12:31:29 am
I see you edited you message. 
I try not to make an excessive number of posts.

Quote
Since I noticed that you just dump references without reading and/or understanding them, and that you also quite often just lie, allow me to check a couple of these references:

Electricity and Magnetism by Purcell E.M., Morin D.J:
They have Problem 7.4 similar to Lewin's problem.  No solution to the problem given!  The problem asks "What will each voltmeter read?" but it is not asking what is the voltage between nodes A and D!  I have the impression that the problem wants to point out that you have to be careful when you measure voltages and the instrument probes are under the effect of an external varying magnetic field...
page 358
(http://link: [url=https://i.postimg.cc/kGXzD9jR/purcell-faraday-2.jpg]https://i.postimg.cc/kGXzD9jR/purcell-faraday-2.jpg[/url])

A little excerpt (bold mine) which some have difficulty in understanding:

"...Kirchhoff Loop rule is no longer valid"

I wonder what that could mean... Why, oh why couldn't they be any clearer???
But let's see the solution of the problem on page 710 (again, bold mine)

"The moral of this problem is that if a setup contains changing flux, it makes no sense to talk about the voltage difference (that is, the value of −integral of E·ds) between two points. It is necessary to state the path over which −integral of E · ds is calculated."

Still of the opinion it does not support my position, which is that KVL is no longer valid and voltage depends on path?
 :palm:

Quote
Fields and Waves in Communication Electronics by Simon Ramo, John R. Whinnery, Theodore Van Duzer:

At the beginning of chapter 4 they say this: "Kirchhoff's two laws provide the basis for classical circuit theory.  We begin with the voltage law as a way of reviewing the basic element values of lumped-circuit theory.  The law states that for any closed loop of a circuit, the algebraic sum of voltages for the individual branches is zero... the basis for this is Faraday's law for a closed path, written as..."  And then, they proceed to use KVL with induced EMFs to solve a bunch of stuff.  How is that supposed to support your position?

I'll show you how. Page 174 of the third edition

"let us take a closed integral of electric field along the conductor of the coil, returning by the path across the terminals. Since the contribution along the part of the path which follows the conductor is zero, all the voltage appears ACROSS the terminals.

I'll translate this for you: voltage ALONG the filament of the coil = 0; voltage ACROSS the terminals = V not zero.
KVL cannot survive two values of voltage between the same two points. Unless you use a trick and pretend that there is only one voltage, that ACROSS the coil, and pretend its a potential difference.

Read page 179

"In the above we seem to be treating voltage as potential difference when we take voltage of a node with respect to the chosen reference, but note that this is only after the circuit is defined and we are only breaking up the integral of E.dl into its contributions over the various branches. As illustrated in the preceding section, we do have to define the path carefully whenever there are inductances or other elements with contribution to voltage from Faraday's law."

But hey, and this is related to the above, have you noticed that Jesse is repeatedly posting the same image of a circuit with a transformer, two outputs on a circuit with resistors and a lot of voltmeters? Suppose one turn around the core gives you an emf of 1V.
Can you place the values of your "path independent voltage" inside all those voltmeters? Because I have a hunch you cannot do it, using the voltage that according to you builds up in the coil.
I can easily do it using my definition of voltage, in accordance with Purcell, Ramo etc, Haus etc, Brandao Faria, Zahn, etc... but I wonder if you have any idea of what 'your voltage' (or the 'McDonald voltage', if you prefer to call it that way) is, in that context.
(Pretty sure Jesse doesn't know either, because I've asked and he refused to answer)


Edit: further clarifies what "voltage" I want you to place inside those voltmeters.
I will place the values of "my" path dependent voltage once either you or Jesse post your solution.

Once again Purcel, the same as Hayt, is progressing through the subject in a sequential historical way when he writes:

Quote
A consequence of Faraday’s law of induction is that Kirchhoff’s loop rule (which states that \$\int_{}^{}E.ds=0\$ around a closed path) is no longer valid in situations where there is a changing magnetic field. Faraday has taken us beyond the comfortable realm of conservative electric fields.

If we take that in a different context, it will directly contradict what Maxwell says:

Quote
Let the components of the electromotive intensity be X, Y, Z.
The electromotive intensity at any point is the resultant force on a unit of positive electricity placed at that point.  It may arise (1) from electrostatic action, in which case if V is the potential,
 \$X=-\frac{dV}{dx},Y=-\frac{dV}{dy},Z=-\frac{dV}{dz};    (1)\$
or (2) from electromagnetic induction, the laws of which we shall afterwards examine; or (3) from thermoelectric or electrochemical action at the point itself, tending to produce a current in a given direction.
We shall in general suppose that X, Y, Z represent the components of the actual electromotive intensity at the point, whatever be the origin of the force, but we shall occasionally examine the result of supposing it entirely due to variation of potential.

As for the second quote:

"let us take a closed integral of electric field along the conductor of the coil, returning by the path across the terminals. Since the contribution along the part of the path which follows the conductor is zero, all the voltage appears ACROSS the terminals."

That is ABSOLUTELY incorrect as the induced voltage is distributed through the conductor as many experiments have demonstrated.

EDIT: You may also want to check page 199 of Fields and Waves in Communication Electronics where they explicitly contradict the statement above by considering the effect of the electric field between the turns of the inductor!







Title: Re: #562 – Electroboom!
Post by: bsfeechannel on December 14, 2021, 01:31:24 am
Since this thread has sprung back to live, let me share some information to further demolish team's Lewin position. 

Aaww, the cute little KVLey demolished the argument of the big bad Lewin-Newey. Poor Lewin-Newey.
Title: Re: #562 – Electroboom!
Post by: jesuscf on December 14, 2021, 02:02:51 am
Since this thread has sprung back to live, let me share some information to further demolish team's Lewin position. 

Aaww, the cute little KVLey demolished the argument of the big bad Lewin-Newey. Poor Lewin-Newey.

I see your brain has already melted down...
Title: Re: #562 – Electroboom!
Post by: Sredni on December 14, 2021, 05:41:24 am
Once again Purcel, the same as Hayt, is progressing through the subject in a sequential historical way when he writes:

Quote
A consequence of Faraday’s law of induction is that Kirchhoff’s loop rule (which states that \$\int_{}^{}E.ds=0\$ around a closed path) is no longer valid in situations where there is a changing magnetic field. Faraday has taken us beyond the comfortable realm of conservative electric fields.

If we take that in a different context,


What different context???
The context is "in situations where there is a changing magnetic field".

Quote
it will directly contradict what Maxwell says:

Quote
Let the components of the electromotive intensity be X, Y, Z.
The electromotive intensity at any point is the resultant force on a unit of positive electricity placed at that point.  It may arise (1) from electrostatic action, in which case if V is the potential,
 \$X=-\frac{dV}{dx},Y=-\frac{dV}{dy},Z=-\frac{dV}{dz};    (1)\$
or (2) from electromagnetic induction, the laws of which we shall afterwards examine; or (3) from thermoelectric or electrochemical action at the point itself, tending to produce a current in a given direction.
We shall in general suppose that X, Y, Z represent the components of the actual electromotive intensity at the point, whatever be the origin of the force, but we shall occasionally examine the result of supposing it entirely due to variation of potential.

Apart from the fact that Maxwell is simply enumerating the EMF available at the time: electrochemical piles/electrostatic machines,  thermopiles and em induction (remember, if you consider localized sources you can apply what today we call "extended KVL") - what if it actually contradicted Maxwell?
Maxwell believed in the existence of the luminiferous ether. He was wrong.
What we today call "Maxwell's equations" is the result of the work of Heaviside, using the notation expressed in the treatise would be maddening. Expliciting all gradients, curls, divergences...
A lot of thing has changed since 1865.

Quote
As for the second quote:

"let us take a closed integral of electric field along the conductor of the coil, returning by the path across the terminals. Since the contribution along the part of the path which follows the conductor is zero, all the voltage appears ACROSS the terminals."

That is ABSOLUTELY incorrect as the induced voltage is distributed through the conductor as many experiments have demonstrated.

EDIT: You may also want to check page 199 of Fields and Waves in Communication Electronics where they explicitly contradict the statement above by considering the effect of the electric field between the turns of the inductor!

Sure, page 199 comes after page 198. Where the section "Circuit which are not small compared with wavelength" starts and we are considering "4.10 Distributed effects and retardation".

I am trying to hold back that pun.
Title: Re: #562 – Electroboom!
Post by: jesuscf on December 14, 2021, 06:33:00 am

Quote
Let the components of the electromotive intensity be X, Y, Z.
The electromotive intensity at any point is the resultant force on a unit of positive electricity placed at that point.  It may arise (1) from electrostatic action, in which case if V is the potential,
 \$X=-\frac{dV}{dx},Y=-\frac{dV}{dy},Z=-\frac{dV}{dz};    (1)\$
or (2) from electromagnetic induction, the laws of which we shall afterwards examine; or (3) from thermoelectric or electrochemical action at the point itself, tending to produce a current in a given direction.
We shall in general suppose that X, Y, Z represent the components of the actual electromotive intensity at the point, whatever be the origin of the force, but we shall occasionally examine the result of supposing it entirely due to variation of potential.

Apart from the fact that Maxwell is simply enumerating the EMF available at the time: electrochemical piles/electrostatic machines,  thermopiles and em induction (remember, if you consider localized sources you can apply what today we call "extended KVL") - what if it actually contradicted Maxwell?
Maxwell believed in the existence of the luminiferous ether. He was wrong.
What we today call "Maxwell's equations" is the result of the work of Heaviside, using the notation expressed in the treatise would be maddening. Expliciting all gradients, curls, divergences...
A lot of thing has changed since 1865.

But one thing that has not changed since the time of Maxwell is that KVL explicitly includes electromagnetic induction EMFs which are not to be treated differently from any other EMFs, "whatever be the origin of the force"!
Title: Re: #562 – Electroboom!
Post by: bsfeechannel on December 14, 2021, 02:17:17 pm

Quote
Let the components of the electromotive intensity be X, Y, Z.
The electromotive intensity at any point is the resultant force on a unit of positive electricity placed at that point.  It may arise (1) from electrostatic action, in which case if V is the potential,
 \$X=-\frac{dV}{dx},Y=-\frac{dV}{dy},Z=-\frac{dV}{dz};    (1)\$
or (2) from electromagnetic induction, the laws of which we shall afterwards examine; or (3) from thermoelectric or electrochemical action at the point itself, tending to produce a current in a given direction.
We shall in general suppose that X, Y, Z represent the components of the actual electromotive intensity at the point, whatever be the origin of the force, but we shall occasionally examine the result of supposing it entirely due to variation of potential.

Apart from the fact that Maxwell is simply enumerating the EMF available at the time: electrochemical piles/electrostatic machines,  thermopiles and em induction (remember, if you consider localized sources you can apply what today we call "extended KVL") - what if it actually contradicted Maxwell?
Maxwell believed in the existence of the luminiferous ether. He was wrong.
What we today call "Maxwell's equations" is the result of the work of Heaviside, using the notation expressed in the treatise would be maddening. Expliciting all gradients, curls, divergences...
A lot of thing has changed since 1865.

But one thing that has not changed since the time of Maxwell is that KVL explicitly includes electromagnetic induction EMFs which are not to be treated differently from any other EMFs, "whatever be the origin of the force"!

If the induced EMF is not in the path of the circuit, yes, you can include it in KVL. If not, KVL fails.
Title: Re: #562 – Electroboom!
Post by: jesuscf on December 14, 2021, 04:03:04 pm

Quote
Let the components of the electromotive intensity be X, Y, Z.
The electromotive intensity at any point is the resultant force on a unit of positive electricity placed at that point.  It may arise (1) from electrostatic action, in which case if V is the potential,
 \$X=-\frac{dV}{dx},Y=-\frac{dV}{dy},Z=-\frac{dV}{dz};    (1)\$
or (2) from electromagnetic induction, the laws of which we shall afterwards examine; or (3) from thermoelectric or electrochemical action at the point itself, tending to produce a current in a given direction.
We shall in general suppose that X, Y, Z represent the components of the actual electromotive intensity at the point, whatever be the origin of the force, but we shall occasionally examine the result of supposing it entirely due to variation of potential.

Apart from the fact that Maxwell is simply enumerating the EMF available at the time: electrochemical piles/electrostatic machines,  thermopiles and em induction (remember, if you consider localized sources you can apply what today we call "extended KVL") - what if it actually contradicted Maxwell?
Maxwell believed in the existence of the luminiferous ether. He was wrong.
What we today call "Maxwell's equations" is the result of the work of Heaviside, using the notation expressed in the treatise would be maddening. Expliciting all gradients, curls, divergences...
A lot of thing has changed since 1865.

But one thing that has not changed since the time of Maxwell is that KVL explicitly includes electromagnetic induction EMFs which are not to be treated differently from any other EMFs, "whatever be the origin of the force"!

If the induced EMF is not in the path of the circuit, yes, you can include it in KVL. If not, KVL fails.

You don't even now what is KVL!
Title: Re: #562 – Electroboom!
Post by: bsfeechannel on December 14, 2021, 05:07:54 pm
You don't even now what is KVL!

KVL is a special case of Faraday's law when you don't have a time-varying magnetic field in the surface enclosed by the path of the circuit.
Title: Re: #562 – Electroboom!
Post by: jesuscf on December 14, 2021, 05:15:37 pm
You don't even now what is KVL!

KVL is a special case of Faraday's law when you don't have a time-varying magnetic field in the surface enclosed by the path of the circuit.

Your ignorance is laughable!  :-DD
Title: Re: #562 – Electroboom!
Post by: bsfeechannel on December 15, 2021, 02:06:12 pm
Your ignorance is laughable!  :-DD

You'll stop laughing when you finally understand electromagnetism.
Title: Re: #562 – Electroboom!
Post by: Jesse Gordon on December 28, 2021, 04:41:01 am
And, Jesse, when I say "listen to the guys in a garage" I am not comparing them to me, but to the university professors who wrote books and papers where this problem was carefully analyzed.
Purcell, Morin, Roche, Ramo, Whinnery, VanDuzer, Romer, Rosser, Zahn, Haus, Melcher, Nicholson, Brandao Faria, ...

Ahhhh, and when I read your articulate posts so artfully penned from your garage, I would be comparing them then to Maxwell, Feynman, Belcher, McDonald, etc., ..., .

See how useless that is?
Title: Re: #562 – Electroboom!
Post by: Jesse Gordon on December 28, 2021, 05:02:03 am
have you noticed that Jesse is repeatedly posting the same image of a circuit with a transformer, two outputs on a circuit with resistors and a lot of voltmeters? Suppose one turn around the core gives you an emf of 1V.
Can you place the values of your "path independent voltage" inside all those voltmeters?

What does that even mean? We're talking about KVL here. You use self contained lumped two-terminal elements here. You are trying to put KVL on trial for what goes  on inside of a black-box lumped element.

And according to Team Lewin's fully endorsed top of the line textbook, transformer windings are completely acceptable:

Quote from: Desoer-Kuh

Lumped circuits are obtained by connecting lumped elements. Typical
lumped elements are resistors, capacitors, inductors, and transformers.
We have encountered them in the laboratory, and we can see them in our
radio sets. The key property associated with lumped elements is their
small size (compared to the wavelength corresponding to their normal
frequency of operation.)  From the more general electromagnetic field point
of view, lumped elements are point singularities; that is, they have negligible
physical dimensions. In this way they are similar to a particle.
Lumped elements may have two terminals, as in a resistor, or more than
two terminals, as in a transformer or transistor. For two-terminal
lumped elements, it can be shown that the general laws governing the
electromagnetic field, together with the restriction on physical size indicated
above, imply that at all times the current entering one terminal is
equal to the current leaving the other terminal, and that the voltage difference
between the two terminals can be unambiguously defined by physical
measurements.  Thus, for two-terminal lumped elements, the current
through the element and the voltage across it are well-defined quantities. For
lumped elements with more than two terminals, the current entering any
terminal and the voltage across any pair of terminals are well defined at all times.

  For the remainder of this book, any interconnection of the lumped
elements such that the dimensions of the circuit are small compared with the
wavelength associated with the highest frequency of interest will be called
a lumped circuit.
  As long as this restriction on the size of the circuit holds, Kirchhoff's
current and voltage laws (to be discussed in Secs. 3 and 4) are valid. This
restriction is a consequence of the fact that Kirchhoff's laws are approximations
of Maxwell's celebrated equations, which are the general laws
of the electromagnetic field.  The approximation is analogous to the fact
that Newton's laws of classical mechanics are approximations to the laws
of relativistic mechanics.  Even though they are approximations, the laws
of Newton and Kirchhoff can be applied to a large number of practical

(Glowies Mine HEHE)

Team Lewin provided this very citation and assured me that this was the Bently or Rolls Royce or whatever of textbooks.

It is utterly clear that as long as the restrictions it describes are conformed to, transformer and inductor windings are perfectly valid as elements in a loop for KVL.

Quote
(Pretty sure Jesse doesn't know either, because I've asked and he refused to answer)

If I refused to answer any of your questions it was for one of two reasons: 1: It was incoherent or unambiguous words which did not actually form an unambiguous question.
Or 2: At some point I realized you were just throwing muddy questions at the wall in hopes one would stick while refusing to answer a single one of mine so I began question bargaining which meant I had to refuse to answer questions just to get you to answer a couple of mine. Doesn't mean I didn't know the answers though.

And you still have not answered MY question as I asked it. Sure you asked yourself  your own question and answered it, but you have not answered MY question as I asked it:

Question: In the following diagram, in a real life physical lab test performed with real (time synchronized) volt meters with a real transformer and real resistors CONNECTED AS SHOWN, will the readings of all the volt meters sum to zero, within the accuracy and resolution limitations of the volt meters? YES or NO.

(Or if you believe SOMETIMES is the answer, then answer SOMETIMES and explain one scenario for a YES condition and one scenario for a NO condition WITH THE VOLT METERS AND ALL COMPONENTS CONNECTED AS SHOWN - Running additional conductors through the transformer core is not allowed - nor is removing existing conductors from through the transformer core!)

(https://i.postimg.cc/jdJntBXT/20211128-121506.jpg)


Title: Re: #562 – Electroboom!
Post by: Jesse Gordon on December 28, 2021, 05:31:31 am
If the induced EMF is not in the path of the circuit, yes, you can include it in KVL. If not, KVL fails.

So where does that leave the venerable transformer secondary? Is the voltage across the terminals of a transformer secondary considered "Induced EMF which is in the path of the circuit?"

I think what you mean to say is that IF there is UNDOCUMENTED EMF across any part of the loop, then KVL fails.
That undocumented voltage could be from the photovoltaic effect, the electrochemical effect, the seebeck effect, or yes, even magnetically induced voltage, and KVL will fail.

But if all element voltages are accurately measured then KVL holds.

The only real question we must answer is whether "the voltage difference
between the two terminals can be unambiguously defined by physical
measurements." (Desoer-Kuh)

If the answer is yes then the answer is yes, KVL holds.

And the fact is that for closed-magnetic-circuit-core transformers, the output voltage of a secondary winding (heh as well as the input voltage of the primary) can almost always be unambiguously physically measured.

And in many open-magnetic-circuit type core transformers the voltage difference can also be unambiguously physically measured, and so KVL holds there too.

I honestly cannot understand why Team Lewin won't admit to even the undeniable and move on to the finer points.

Question: In the following diagram, in a real life physical lab test performed with real (time synchronized) volt meters with a real transformer and real resistors CONNECTED AS SHOWN, will the readings of all the volt meters sum to zero, within the accuracy and resolution limitations of the volt meters? YES or NO.

(Or if you believe SOMETIMES is the answer, then answer SOMETIMES and explain one scenario for a YES condition and one scenario for a NO condition WITH THE VOLT METERS AND ALL COMPONENTS CONNECTED AS SHOWN - Running additional conductors through the transformer core is not allowed - nor is removing existing conductors from through the transformer core!)

(https://i.postimg.cc/jdJntBXT/20211128-121506.jpg)


It's time you admit that the answer to the above question is YES: So long as the voltage across the terminals of the element can be unambiguously defined by physical measurement, then KVL will hold.

And besides, actual lab results match KVL. It's obviously holding!
Title: Re: #562 – Electroboom!
Post by: Jesse Gordon on December 28, 2021, 05:51:46 am

Quote
Let the components of the electromotive intensity be X, Y, Z.
The electromotive intensity at any point is the resultant force on a unit of positive electricity placed at that point.  It may arise (1) from electrostatic action, in which case if V is the potential,
 \$X=-\frac{dV}{dx},Y=-\frac{dV}{dy},Z=-\frac{dV}{dz};    (1)\$
or (2) from electromagnetic induction, the laws of which we shall afterwards examine; or (3) from thermoelectric or electrochemical action at the point itself, tending to produce a current in a given direction.
We shall in general suppose that X, Y, Z represent the components of the actual electromotive intensity at the point, whatever be the origin of the force, but we shall occasionally examine the result of supposing it entirely due to variation of potential.

Apart from the fact that Maxwell is simply enumerating the EMF available at the time: electrochemical piles/electrostatic machines,  thermopiles and em induction (remember, if you consider localized sources you can apply what today we call "extended KVL") - what if it actually contradicted Maxwell?
Maxwell believed in the existence of the luminiferous ether. He was wrong.
What we today call "Maxwell's equations" is the result of the work of Heaviside, using the notation expressed in the treatise would be maddening. Expliciting all gradients, curls, divergences...
A lot of thing has changed since 1865.

But one thing that has not changed since the time of Maxwell is that KVL explicitly includes electromagnetic induction EMFs which are not to be treated differently from any other EMFs, "whatever be the origin of the force"!

I'm pretty sure Team Lewin doesn't even know what they are arguing for, much less what they are arguing about.

Otherwise they'd be willing to concede the obvious stuff and move on to the finer details, but even that cannot happen.

It's been fascinating for sure.

But what baffles me is Team Lewin's refusal to draw a definitive line in reasoning and say "Here's where you're wrong. (Insert reason)" ("You" being Team KVL.)

The only thing they really seem certain of is that Lewin cannot be wrong. Maybe former students? Maybe fellow Dutchmen? I've read that the Dutch are both very loyal to their own as well as very tenacious in negotiations with others.

At the very least I'd have expected Team Lewin to say "Yes, the volt meters in your diagram will sum to zero, and KVL does appear to work, but due to a certain technicality of a certain definition, we say KVL isn't holding.."

But we can't even get that much from them, which means they don't even understand the topic much less what they are arguing for.

Team Lewin also seems to ignore the fact that some PhD Physics Professors also disagree with them -- it doesn't take a genius to see that not all PhD Physics Professors agree on the topic so there must be room for discussion.. But no, Lewin defines reality and we're supposed to ignore the others.
Title: Re: #562 – Electroboom!
Post by: ogden on December 28, 2021, 06:38:30 am
But no, Lewin defines reality and we're supposed to ignore the others.

It is waste to argue with true believers because they simply ignore anything that does not support their beliefs.
Title: Re: #562 – Electroboom!
Post by: bsfeechannel on December 29, 2021, 03:39:21 am
I think what you mean to say is that IF there is UNDOCUMENTED EMF across any part of the loop, then KVL fails.

What if it is written off, instead of undocumented? I don't know, by some very basic electrostatic phenomenon that you're ignoring along the path you chose?

I'm pretty sure Team Lewin doesn't even know what they are arguing for, much less what they are arguing about.

You're ignorant about the electromagnetic phenomenon. But that's not a problem. The problem is that you don't even know it.

This is what I call recursive ignorance. It is the kind of ignorance that is so deep that it doesn't allow you to realize that it exists.

Quote
But what baffles me is Team Lewin's refusal to draw a definitive line in reasoning and say "Here's where you're wrong. (Insert reason)" ("You" being Team KVL.)

This whole thread is a monument erected to your ignorance of electromagnetism. So, just read the thread and you'll instantly get where you're wrong.

Quote
The only thing they really seem certain of is that Lewin cannot be wrong. Maybe former students? Maybe fellow Dutchmen? I've read that the Dutch are both very loyal to their own as well as very tenacious in negotiations with others.

Lewin is not wrong because we independently learned the theory (I did decades ago) and repeated the experiments, and the results predicted by the theory are precisely the same.

Quote
Team Lewin also seems to ignore the fact that some PhD Physics Professors also disagree with them -- it doesn't take a genius to see that not all PhD Physics Professors agree on the topic so there must be room for discussion..

But none of them said Lewin is wrong. And yes, KVLers proved to be no geniuses up to now.

Quote
But no, Lewin defines reality and we're supposed to ignore the others.

What defines reality is Nature. Lewin just happened to be the harbinger of it to you. Learn to live with that.

It is waste to argue with true believers because they simply ignore anything that does not support their beliefs.

We're engineers. We don't believe. We learn engineering and test things in the lab, and what we say is the result of that.
Title: Re: #562 – Electroboom!
Post by: Sredni on December 29, 2021, 02:18:53 pm
have you noticed that Jesse is repeatedly posting the same image of a circuit with a transformer, two outputs on a circuit with resistors and a lot of voltmeters? Suppose one turn around the core gives you an emf of 1V.
Can you place the values of your "path independent voltage" inside all those voltmeters?

What does that even mean?


It means that you cannot even put numbers of your 'McDonald voltage' in those voltmeters without a numerical simulation. That's the problem of using the scalar potential difference as definition of voltage. And if you choose to use minus the path integral of Etot.dl, then you will have to face that there are configurations of voltmeters where KVL dies.
On the other hand, I have zero problems in putting values into those voltmeters that will be what you will read in the real world. And I have zero problems in admitting that for some configurations KVL will die.

Quote
And according to Team Lewin's fully endorsed top of the line textbook, transformer windings are completely acceptable:
Quote from: Desoer-Kuh

Lumped circuits are obtained by connecting lumped elements.

It just shows you do not understand what you read. Go back a few pages to my posts about "lumpED lumpABLE and UNlumpable" circuits, I cannot repeat everything every time you reset to default (like you do on your channel, when you keep mentioning that sentence by Belcher which I have extensively shown it was not referred to Lewin's ring).

Quote
Quote
(Pretty sure Jesse doesn't know either, because I've asked and he refused to answer)

If I refused to answer any of your questions it was for one of two reasons: 1: It was incoherent or unambiguous words which did not actually form an unambiguous question.
Or 2: At some point I realized you were just throwing muddy questions at the wall in hopes one would stick while refusing to answer a single one of mine so I began question bargaining which meant I had to refuse to answer questions just to get you to answer a couple of mine. Doesn't mean I didn't know the answers though.

and
3. you don't know how to answer.
I see that you have banned another user on your YT channel.

(https://i.postimg.cc/KjvfpC5f/screenshot.png)
https://i.postimg.cc/KjvfpC5f/screenshot.png

I wonder why you do that.  :-//

Quote
And you still have not answered MY question as I asked it.

I did answer your question. And I showed you that even if you can find configurations where KVL works, certain circuits also admits configurations where KVL does not work. And I showed you, by using the unlumpable Lewin ring, that even in such a case you can devise a configuration of elements and voltmeters where KVL holds. So, your example does not mean anything.

Here, let me try to explain with an analogy:

In the presence of water there are situations where you cannot walk.
Of course, if you are on the shore, or on a pier, or on a boat, you can walk even if you are close to water.
The problem is that this is not always true because when you are not on the shore, not on a pier, not on a boat, but you are in the middle of the ocean, you cannot walk.

Your voltmeters configuration is like a boat in the middle of the ocean. You keep insisting "see, I'm in the presence of water the middle of the ocean and I can walk, therefore we can walk on water even when we are in the middle of the ocean".
No, you can't.
But you do not understand that, and keep proposing that circuit as if it could represent some difficulty...

Quote
Question: In the following diagram, in a real life physical lab test performed with real (time synchronized) volt meters with a real transformer and real resistors CONNECTED AS SHOWN, will the readings of all the volt meters sum to zero, within the accuracy and resolution limitations of the volt meters? YES or NO.

(Or if you believe SOMETIMES is the answer, then answer SOMETIMES and explain one scenario for a YES condition and one scenario for a NO condition WITH THE VOLT METERS AND ALL COMPONENTS CONNECTED AS SHOWN - Running additional conductors through the transformer core is not allowed - nor is removing existing conductors from through the transformer core!)

(https://i.postimg.cc/jdJntBXT/20211128-121506.jpg)

You are showing a boat in the middle of the ocean. Yes you can walk on the bridge of that boat.
But you still cannot walk on water.
Is it clear now?
(It is a rhetorical question)

Now, would you care to put values in those voltmeters in that 'oh, so tricky' circuit of yours?


Edit: grammar
Title: Re: #562 – Electroboom!
Post by: Sredni on December 29, 2021, 03:54:34 pm
But what baffles me is Team Lewin's refusal to draw a definitive line in reasoning and say "Here's where you're wrong. (Insert reason)" ("You" being Team KVL.)

The only thing they really seem certain of is that Lewin cannot be wrong. Maybe former students? Maybe fellow Dutchmen? I've read that the Dutch are both very loyal to their own as well as very tenacious in negotiations with others.

At the very least I'd have expected Team Lewin to say "Yes, the volt meters in your diagram will sum to zero, and KVL does appear to work, but due to a certain technicality of a certain definition, we say KVL isn't holding.."

But we can't even get that much from them, which means they don't even understand the topic much less what they are arguing for.

Team Lewin also seems to ignore the fact that some PhD Physics Professors also disagree with them -- it doesn't take a genius to see that not all PhD Physics Professors agree on the topic so there must be room for discussion.. But no, Lewin defines reality and we're supposed to ignore the others.

Lewin is just one of many, many, many other professors who know how to correctly apply classical electrodynamics. What we (well, I at least) call the Romer-Lewin ring is mentioned in a 1969 teaser by Moorcroft, followed by a discussion published the year after

    D. R. Moorcroft
    Faraday's Law, potential and voltage: discussion of a teaser
    received July 1969, published on American Journal of Physics 38, 376 - March 1970
    doi 10.1119/1.1976329

there, Moorcroft proposed a distinction between the definitions of potential difference (the scalar potential difference) and voltage (minus the path integral of the total electric field Etot.dl).

What voltmeters measure and an application to "the ring" is also discussed in Romer's paper - with the same conclusions Lewin arrive at.

    Robert H. Romer
    What do voltmeters measure? Faraday's law in multiply connected regions
    American Journal of Physics vol 50, no 12, December 1982

The ring, the electric field configuration, its charge distribution and even the Poynting vector field is discussed in a paper by Mark Heald

    Mark A. Heald
    Energy Flow in Circuits with Faraday EMF
    American Journal of Physics, 56 (6), June 1988

same exact conclusion as Lewin's

The same can be said about the paper by Roche

    J. Roche
    Explaining electromagnetic induction: a critical re-examination.
    The clinical value of history in physics.
    Physics Education, Volume 22, Number 2

What a voltmeter measures in the case of a loop with a single resistor is treated in

    Howard W. Nicholson
    What does the voltmeter read?
    Am. J. Phys. 73 (12), December 2005

And then there are books, lots of books from many authors (Purcell, Haus and Melcher, Ramo Whinnery VanDuzer, Brandao Faria, ...)

And videos, like the videos from MIT or those from Micheal Melloch of Purdue University.

   Electric and Magnetic Fields (https://www.youtube.com/channel/UC5ZsENSXOVzI0zHt4i2bZFw)   
    Prof. Micheal R. Melloch
    School of ECE - Purdue University

videos of the same author that you yourself linked in your channel, evidently without understanding they were on the same position as Lewin's.

I think I understand where this fixation for Lewin comes from: he expressed the limitations of KVL in terms that even people who have no idea what a path integral is can understand. They understood the sentence "KVL is for the bird" and nothing of what is behind and now Lewin is their boogey man.

They do not even understand what Belcher and McDonald say, they only get a few sentences here and there. Like you with that sentence by Belcher that is about the RLC lumped circuit. Or with what McDonald proposes, ie using the scalar potential difference as definition of voltage - for which you do not understand the implications (what would be the McDonald voltage in the McDonald voltmeters of your circuit? You cannot even answer that).
Belcher, McDonald and Notaros require both fields or both potentials to correctly describe Lewin's ring. KVLers ignore this simple fact and think the coulombian field alone (sometimes the induced field alone) or the scalar potential alone suffice.
Ignorance is bliss, they say.

Title: Re: #562 – Electroboom!
Post by: jesuscf on December 29, 2021, 06:08:13 pm
Lewin is just one of many, many, many other professors who know how to correctly apply classical electrodynamics. What we (well, I at least) call the Romer-Lewin ring is mentioned in a 1969 teaser by Moorcroft, followed by a discussion published the year after

Sredni (the king of cherry picking!), have you calculated the voltage between nodes A and D, VAD yet?  Have you figured what is the difference between the voltage between two nodes and the voltage measured by a 'voltmeter'?

Lewin, and team Lewin by that matter, are so wrong that they even disagree with what Maxwell explicitly says.  Let start with definition of KVL as stated by Maxwell in "A Treatise on Electricity & Magnetism" from 1873:

[Page 406 article 282b]

The theory of conjugate conductors has been investigated by Kirchhoff, who has stated the conditions of a linear system in the following manner, in which the consideration of the potential is avoided.

(1) (Condition of 'continuity.')  At any point of the system the sum of all the currents which flow towards that point is zero.

(2) In any complete circuit formed by the conductors the sum of the electromotive forces taken round the circuit is equal to the sum of the products of the current in each conductor multiplied by the resistance of that conductor.


Point (1) above is clearly KCL and point (2) is clearly KVL.  That is followed latter on by this about electromotive forces:

[Page 418 article 297]

Let the components of the electromotive intensity be X, Y, Z.
The electromotive intensity at any point is the resultant force on a unit of positive electricity placed at that point.  It may arise (1) from electrostatic action, in which case if V is the potential,

 \$X=-\frac{dV}{dx},Y=-\frac{dV}{dy},Z=-\frac{dV}{dz};    (1)\$

or (2) from electromagnetic induction, the laws of which we shall afterwards examine; or (3) from thermoelectric or electrochemical action at the point itself, tending to produce a current in a given direction.
We shall in general suppose that X, Y, Z represent the components of the actual electromotive intensity at the point, whatever be the origin of the force, but we shall occasionally examine the result of supposing it entirely due to variation of potential.

We shall in general suppose that X, Y, Z represent the components of the actual electromotive intensity at the point, whatever be the origin of the force, but we shall occasionally examine the result of supposing it entirely due the variation of potential.


Now, whom should we believe?

1) Maxwell who has been proven right over and over for more than 150 years?

2) Lewin who has been proven wrong over and over again for many years, both mathematically and experimentally?

The correct option is of course 1).  Unless you are Sredni, who will manage to find a way to denigrate Maxwell because what he wrote doesn't agree with his bullshit!





Title: Re: #562 – Electroboom!
Post by: Sredni on December 29, 2021, 06:31:16 pm
Lewin is just one of many, many, many other professors who know how to correctly apply classical electrodynamics. What we (well, I at least) call the Romer-Lewin ring is mentioned in a 1969 teaser by Moorcroft, followed by a discussion published the year after
Sredni (the king of cherry picking!), have you calculated the voltage between nodes A and D, VAD yet?  Have you figured what is the difference between the voltage between two nodes and the voltage measured by a 'voltmeter'?

Yes I did, and for several paths.
The fact that you either did not understand that, or just simply revert back to factory defaults and keep asking the same questions again and again because you don't like the answer is irrelevant.

Quote
Lewin, and team Lewin by that matter, are so wrong that they even disagree with what Maxwell explicitly says.

Maxwell believed in the luminiferous ether.
Do you believe in the luminiferouts ether?
(Or should I ask "do you know what the luminiferous ether is"?)

Quote
Let start with definition of KVL as stated by Maxwell in "A Treatise on Electricity & Magnetism" from 1873:
[Page 406 article 282b]
The theory of conjugate conductors has been investigated by Kirchhoff, who has stated the conditions of a linear system in the following manner, in which the consideration of the potential is avoided.
(1) (Condition of 'continuity.')  At any point of the system the sum of all the currents which flow towards that point is zero.
(2) In any complete circuit formed by the conductors the sum of the electromotive forces taken round the circuit

I am not a native English speaker. Could you please enlighten me about what "taken round the circuit" means in English?

Title: Re: #562 – Electroboom!
Post by: jesuscf on December 29, 2021, 07:09:52 pm

Yes I did, and for several paths.
The fact that you either did not understand that, or just simply revert back to factory defaults and keep asking the same questions again and again because you don't like the answer is irrelevant.

When you calculate the voltage VAD, there is only one path, the path of the ring circuit, which is neither changing nor moving!  The other paths you mention do not exist!

Maxwell believed in the luminiferous ether.
Do you believe in the luminiferouts ether?
(Or should I ask "do you know what the luminiferous ether is"?)

And here you are!  Denigrating Maxwell as predicted.  Also, I am pretty sure Maxwell contemplated the possibility of a "luminiferous ether", but he was not sold on it unless solid evidence of it was provided.  (I think I read that somewhere in one of his writings...)

I am not a native English speaker. Could you please enlighten me about what "taken round the circuit" means in English?

You need to imagine as if Sean Connery is reading the writings of Maxwell!  It may make more sense to you then!  :-DD
Title: Re: #562 – Electroboom!
Post by: Sredni on December 30, 2021, 12:41:46 am

Yes I did, and for several paths.
The fact that you either did not understand that, or just simply revert back to factory defaults and keep asking the same questions again and again because you don't like the answer is irrelevant.

When you calculate the voltage VAD, there is only one path, the path of the ring circuit, which is neither changing nor moving!

Of course you mean the path on the right (A ---> R1 ---> D)?
Or the one on the left (A ---> R2 ---> D)?
Or the one along an imaginary line that splits the circular circuit in half? And if the circuit is not symmetric?

Now paths do not exists because the circuit is stationary
Every day something new.


Quote
Maxwell believed in the luminiferous ether.
Do you believe in the luminiferouts ether?
(Or should I ask "do you know what the luminiferous ether is"?)

And here you are!  Denigrating Maxwell as predicted

Not denigrating. Just stating a fact.
Newton believed time and space were absolutes. Is this denigrating Newton?

Quote
Also, I am pretty sure Maxwell contemplated the possibility of a "luminiferous ether", but he was not sold on it unless solid evidence of it was provided.  (I think I read that somewhere in one of his writings...)

https://en.wikipedia.org/wiki/Luminiferous_aether

"Albeit compatible with Maxwell's equations, electromagnetic induction of electric fields could not be demonstrated in vacuum, because all methods of detecting electric fields required electrically charged matter.
In addition, Maxwell's equations required that all electromagnetic waves in vacuum propagate at a fixed speed, c. As this can only occur in one reference frame in Newtonian physics (see Galilean relativity), the aether was hypothesized as the absolute and unique frame of reference in which Maxwell's equations hold. That is, the aether must be "still" universally, otherwise c would vary along with any variations that might occur in its supportive medium. Maxwell himself proposed several mechanical models of aether based on wheels and gears, and George Francis FitzGerald even constructed a working model of one of them.
"

Quote
I am not a native English speaker. Could you please enlighten me about what "taken round the circuit" means in English?
You need to imagine as if Sean Connery is reading the writings of Maxwell!  It may make more sense to you then!  :-DD

Yeah, like Jesse, you can't answer and just try to deflect.
Title: Re: #562 – Electroboom!
Post by: jesuscf on December 30, 2021, 01:26:45 am

Yes I did, and for several paths.
The fact that you either did not understand that, or just simply revert back to factory defaults and keep asking the same questions again and again because you don't like the answer is irrelevant.

When you calculate the voltage VAD, there is only one path, the path of the ring circuit, which is neither changing nor moving!

Of course you mean the path on the right (A ---> R1 ---> D)?
Or the one on the left (A ---> R2 ---> D)?
Or the one along an imaginary line that splits the circular circuit in half? And if the circuit is not symmetric?

Now paths do not exists because the circuit is stationary
Every day something new.

Here, from Engineering Electromagnetics by Hayt , 7th edition, page 308:

"We need to define emf as used in (1) or (2). The emf is obviously a scalar, and (perhaps no so obviously) a dimensional check shows that it is measured in volts.  We define the emf as:

 \$emf=\oint_{}^{}E \cdot dL\$

and note that is is the voltage about a specific closed path."

See that little circle "round" the integral, the one you often omit from your long posts?  That is the path we are talking about, the path of the wires and components of the circuit.

Maxwell believed in the luminiferous ether.
Do you believe in the luminiferouts ether?
(Or should I ask "do you know what the luminiferous ether is"?)

And here you are!  Denigrating Maxwell as predicted

Not denigrating. Just stating a fact.
Newton believed time and space were absolutes. Is this denigrating Newton?

Really?  So why is relevant to this discussion whether or not Maxwell believed in the "luminiferous ether" or not?  You are guilty here of not just one, but two fallacies: denigration and deflection.

I am not a native English speaker. Could you please enlighten me about what "taken round the circuit" means in English?
Quote
You need to imagine as if Sean Connery is reading the writings of Maxwell!  It may make more sense to you then!  :-DD

Yeah, like Jesse, you can't answer and just try to deflect.

Sorry, I thought you were joking or something.  Who would ask such a basic question?  Here:

\$emf=\oint_{}^{}E \cdot dL\$

See that little circle "round" the integral?  That is what Maxwell is talking about.

Title: Re: #562 – Electroboom!
Post by: Sredni on December 30, 2021, 11:26:35 am
Maxwell believed in the luminiferous ether.
Do you believe in the luminiferouts ether?
(Or should I ask "do you know what the luminiferous ether is"?)
And here you are!  Denigrating Maxwell as predicted
Not denigrating. Just stating a fact.
Newton believed time and space were absolutes. Is this denigrating Newton?
Really?  So why is relevant to this discussion whether or not Maxwell believed in the "luminiferous ether" or not?  You are guilty here of not just one, but two fallacies: denigration and deflection.

You are the one who believe Maxwell is infallible and his Treatise is gospel. I simply pointed out that the Treatise reflects the time it was written. A time where Maxwell's equations as we know them were yet to be penned. A time where people, including Maxwell, believed in the luminiferous ether. You have quoted a passage of the Treatise where Maxwell is illustrating the sources of emf as known at the time, so what? He states explicitly:

"In any complete circuit formed by the conductors the sum of the electromotive forces taken round the circuit..."

And I am wondering: if the emf is a scalar value that results from computing the path integral of that one closed loop (as that little circle implies), why is Maxwell using the plural? You said it yourself: there is only one loop [note], so there must be only one value for the integral computed along that one loop.

I tell you why he is using the plural.
Because in that sentence he was considering the lumped emfs encountered by going around the circuit. That is what KVL was about at the time (and still is now, since its generalization outside of garages is known as Faraday's law).


note I also wonder... what if the circuit has more than one loop linking dB/dt... What single one loop would you choose to compute the one and only value of Vad?

Title: Re: #562 – Electroboom!
Post by: ogden on December 30, 2021, 03:32:04 pm
As Dr.Lewin ignores emf literally in half of his circuit stating that voltage on two wires A1-A2 and D1-D2 is 0V, let's remove both from circuit completely. What we have left are two resistors. Let's take 10 Ohm wire and make full turn out of it, connect tiny 100 Ohms resistor as load. What we have in result - transformer with lossy secondary and load. Voltages on both equal obviously. It would be rather stupid to measure current through 100 Ohms load, then knowing internal resistance (10 Ohms) of transformer secondary, apply ohms law to calculate voltage :D But wait... this is what Dr.Lewin effectively does in his video "Kirchhoff's Loop Rule Is For The Birds".
Title: Re: #562 – Electroboom!
Post by: jesuscf on December 30, 2021, 05:16:38 pm
You are the one who believe Maxwell is infallible and his Treatise is gospel. I simply pointed out that the Treatise reflects the time it was written. A time where Maxwell's equations as we know them were yet to be penned. A time where people, including Maxwell, believed in the luminiferous ether. You have quoted a passage of the Treatise where Maxwell is illustrating the sources of emf as known at the time, so what? He states explicitly:

Interesting.  Maxwell is fallible, but Lewin, whom struggles to solve the simplest of circuits, is infallible them!


"In any complete circuit formed by the conductors the sum of the electromotive forces taken round the circuit..."

And I am wondering: if the emf is a scalar value that results from computing the path integral of that one closed loop (as that little circle implies), why is Maxwell using the plural? You said it yourself: there is only one loop [note], so there must be only one value for the integral computed along that one loop.

I tell you why he is using the plural.
Because in that sentence he was considering the lumped emfs encountered by going around the circuit. That is what KVL was about at the time (and still is now, since its generalization outside of garages is known as Faraday's law).

So, according to your logic in Lewin's ring, the wire on the top has half the induced EMF and the wire at the bottom has half the induced EMF.  So, can you now can compute the voltage VAD?

Also, according to Maxwell (and he says it explicitly) Faraday's law is NOT a generalization of KVL.  "Electromagnetic induction, the laws of which we shall afterwards examine" is just one more EMF that must be added to other EMFs when using KVL, "whatever be the origin of the force".

note I also wonder... what if the circuit has more than one loop linking dB/dt... What single one loop would you choose to compute the one and only value of Vad?

Then you use KVL and KCL to the complete and correct circuit to computer VAD, as Lewin should had done when he added the 'voltmeters' to left and right of the ring.  In that case he went from one loop to three loops, ignored the two extra loops, disregarded the induced voltages in the wires, calculated the voltages in the resistors using KVL, drew the wrong equivalent circuit, failed to calculate the voltage VAD, and proclaimed with grandiose fanfare that KVL doesn't work.  When confronted on his massive mess, the only recourse he had was to redefine KVL.  To bad for him, the definition of modern KVL was explicitly stated by Maxwell all the way back in 1873 (or earlier) which includes electromagnetic induction as one of the EMFs you have to consider.
Title: Re: #562 – Electroboom!
Post by: bsfeechannel on December 31, 2021, 01:24:06 am
3. you don't know how to answer.
I see that you have banned ano]ther user on your YT channel.

(https://i.postimg.cc/KjvfpC5f/screenshot.png)
https://i.postimg.cc/KjvfpC5f/screenshot.png

I wonder why you do that.  :-//

If you don't understand it, get rid of it. He doesn't have the minimum background to understand the what the guy said. But you cannot expect anything different from these people who worship their own stupidity.

They do not even understand what Belcher and McDonald say, they only get a few sentences here and there.

This is called confirmation bias, one of the most widely committed cognitive biases. They only get snippets of text, out of context, that, on the surface, seem to, but don't, support their beliefs and values. Confirmation bias is also present in the interpretation of results. Only the results that seem to confirm their bias are selected out of context; the rest, which debunk their claims, is ignored. You find this tendency in politics, religion, ideologies, but it is mortal in engineering and science.

Maxwell's equations as we know them today are the result of almost one century of research and development by brilliant scientists, including the already mentioned Faraday, Kirchhoff and Maxwell, but also countless others, like Örsted, Gauss, Ampère, Hertz, Lorentz, Heaviside, Kelvin and Einstein, for example.

It takes several years of painstaking study to master a minimal understanding of it.

You can clearly see that they do not have the appropriate background to sustain an argument. The idea of a non-conservative field, which is key to understanding the path dependency of voltage, is foreign to them. Their understanding of the vectorial nature of the electric and magnetic fields, if existent, is precarious, to say the least.
Title: Re: #562 – Electroboom!
Post by: jesuscf on December 31, 2021, 02:04:14 am
It takes several years of painstaking study to master a minimal understanding of it.

Maybe for you.  Hence the importance of a formal education, where with the proper guidance not only the correct knowledge is acquired in a timely manner, but the BS that characterizes team Lewin is weeded out.

You can clearly see that they do not have the appropriate background to sustain an argument. The idea of a non-conservative field, which is key to understanding the path dependency of voltage, is foreign to them. Their understanding of the vectorial nature of the electric and magnetic fields, if existent, is precarious, to say the least.

You say we don't have the background???  What about the perfect match of experimental results and theoretical solutions?  Where are your experiments and solutions?  You just show fallacy after fallacy and a tremendous display of ignorance.  Very well genius: explain what is it a non-conservative field, in the context of Lewin's ring!  I bet it is going to be hilarious as everything you have posted previously.  At some point I am going to compile in one enormous list all the stupid sh*t you have say in all your posts.  But please, keep adding!
Title: Re: #562 – Electroboom!
Post by: Sredni on December 31, 2021, 02:23:53 am
As Dr.Lewin ignores emf literally in half of his circuit stating that voltage on two wires A1-A2 and D1-D2 is 0V,

He is not ignoring the emf. He is accounting for the coulombian field generated by the charge that accumulates at the resistors' interfaces. That charge will generate an electric field that will cancel almost all the induced field in the conductor, leaving only a residual field compatible with Ohm's law. Which is nearly zero in an excellent conductor like copper.

Quote
let's remove both from circuit completely. What we have left are two resistors. Let's take 10 Ohm wire and make full turn out of it, connect tiny 100 Ohms resistor as load. What we have in result - transformer with lossy secondary and load. Voltages on both equal obviously.

(highlight mine). See, this is the limit you, and all KVLers, cannot overcome: you cannot conceive that voltage can have different values between the same two points, depending on the path joining them.

Quote
It would be rather stupid to measure current through 100 Ohms load, then knowing internal resistance (10 Ohms) of transformer secondary, apply ohms law to calculate voltage :D But wait... this is what Dr.Lewin effectively does in his video "Kirchhoff's Loop Rule Is For The Birds".

Even worse than that. This is what Belcher effectively does in his note about most books getting Faraday's law wrong:

(https://i.postimg.cc/BQL3xBkn/screenshot.pngg)
https://i.postimg.cc/BQL3xBkn/screenshot.png (https://i.postimg.cc/BQL3xBkn/screenshot.png)

You might object that this pdf has been edited by Lewin, but notice this detail at the end of the second sentence: it says "(see sketch next page)", while the sketch in the edited note is on the same page. It means that this example was present in the original note, written by Belcher alone. Could it be that Lewin changed the solution proposed by Belcher? I don't think so, because the figure is showing the same superposition of fields as shown by Belcher in the note he wrote for Mehdi:

(https://i.postimg.cc/SRnyVCmR/screenshot.png)
Source: Belcher's note available on Electroboom's channel

So... is Belcher ignoring the obvious? Is he doing something rather stupid?
Wanna see another professor doing the same as Belcher, and Lewin? Micheal Melloch of Purdue University - the same Micheal Melloch whose videos Jesse Gordon has linked in the comments of his videos on Lewin's ring. Too bad he is doing what Lewin (and Belcher!) is doing:

Quote
Faraday's Law Example 4 (Electromagnetic Induction) 
https://www.youtube.com/watch?v=sAsxEBkaty8 (https://www.youtube.com/watch?v=sAsxEBkaty8)
This is the fourth of four examples of the application of Faraday's Law. A current is ramped in a solenoid to produce circles of electric field intensity around the solenoid. The solenoid is surrounded by a conducting ring where the left half of the ring has a higher conductivity than the right half of the ring. What is observed is charge buildup at the boundaries between the two materials.
 

The charge buildup at the boundaries between the materials is what you KVLers are unable to see.
It's like glass for birds, or storks.
Title: Re: #562 – Electroboom!
Post by: Sredni on December 31, 2021, 02:54:22 am
You are the one who believe Maxwell is infallible and his Treatise is gospel. I simply pointed out that the Treatise reflects the time it was written. A time where Maxwell's equations as we know them were yet to be penned. A time where people, including Maxwell, believed in the luminiferous ether. You have quoted a passage of the Treatise where Maxwell is illustrating the sources of emf as known at the time, so what? He states explicitly:

Interesting.  Maxwell is fallible, but Lewin, whom struggles to solve the simplest of circuits, is infallible them!


Again with the boogeman Lewin. He is not alone in correctly applying Faraday. There are so many other professors, including Belcher, that do what Lewin does: accounting for both the EMF contribution of the induced field Eind and the scalar potential difference of the conservative field Ecoul. And, being human, the are fallible too.
But at this point in time they can base their view of classical electrodynamics on about 150 years of careful refinement by some of the most brilliant minds of the whole planet. So, I'll take that above what Maxwell (which was an absolute genius, but was also pioneering a new field) had to say in 1865. The ether is one glaring example.
A more refined view of classical electrodynamics in a less glaring one, and in fact...

Quote
Also, according to Maxwell (and he says it explicitly) Faraday's law is NOT a generalization of KVL.  "Electromagnetic induction, the laws of which we shall afterwards examine" is just one more EMF that must be added to other EMFs when using KVL, "whatever be the origin of the force".

...you fail to see why we should move forward from the interpretations field theory at the time of the Americal Civil War.

Quote
So, according to your logic in Lewin's ring, the wire on the top has half the induced EMF and the wire at the bottom has half the induced EMF.  So, can you now can compute the voltage VAD?

There is the EMF contribution of Eind, but there is also the scalar potential contribution of Ecoul. And guess what? The pretty much cancel each other in the conductor leaving an almost zero net electric field inside. Perfectly compliant with Ohm's law. The same superposition leaves a perfectly Ohm compliant resultant E field in both resistors. But being the resistance different, the field inside will be different as well.

Now, let's do 2+2: on branch 1 we have 0 field in the upper arc of conductor, strong E1 field inside the R1 resistor, and 0 field in the lower arc of conductor. The voltage is the same as that along the resistor alone: Vad1 = E1*deltaL, where deltaL is the length of the resistor.
In branch 2, for the same reasons we have Vad2 = E2*deltaL.
Since the net electric field is different in the two resistors, we get Vad1 != Vad2

Quote
note I also wonder... what if the circuit has more than one loop linking dB/dt... What single one loop would you choose to compute the one and only value of Vad?

Then you use KVL and KCL to the complete and correct circuit to compute...

Sure, you use KVL to prove KVL.
Title: Re: #562 – Electroboom!
Post by: jesuscf on December 31, 2021, 03:17:58 am
Wanna see another professor doing the same as Belcher, and Lewin? Micheal Melloch of Purdue University - the same Micheal Melloch whose videos Jesse Gordon has linked in the comments of his videos on Lewin's ring. Too bad he is doing what Lewin (and Belcher!) is doing:

Quote
Faraday's Law Example 4 (Electromagnetic Induction) 
https://www.youtube.com/watch?v=sAsxEBkaty8 (https://www.youtube.com/watch?v=sAsxEBkaty8)
This is the fourth of four examples of the application of Faraday's Law. A current is ramped in a solenoid to produce circles of electric field intensity around the solenoid. The solenoid is surrounded by a conducting ring where the left half of the ring has a higher conductivity than the right half of the ring. What is observed is charge buildup at the boundaries between the two materials.
 

The charge buildup at the boundaries between the materials is what you KVLers are unable to see.
It's like glass for birds, or storks.

Excellent, I was waiting for something like this.  Now, here is a question for you:  what would happen if you replace the resistors with capacitors.  Say, instead of the 100 ohm resistor place  a 100nF capacitor, and instead of the 900 ohm resistor place a 10nF capacitor.  What would be the voltage between nodes A and D, VAD at steady state?   Spoiler alert: I did that experiment a few weeks ago and it looks very, very bad for team Lewin!
Title: Re: #562 – Electroboom!
Post by: Sredni on December 31, 2021, 03:36:23 am
Wanna see another professor doing the same as Belcher, and Lewin? Micheal Melloch of Purdue University - the same Micheal Melloch whose videos Jesse Gordon has linked in the comments of his videos on Lewin's ring. Too bad he is doing what Lewin (and Belcher!) is doing:

Quote
Faraday's Law Example 4 (Electromagnetic Induction) 
https://www.youtube.com/watch?v=sAsxEBkaty8 (https://www.youtube.com/watch?v=sAsxEBkaty8)
This is the fourth of four examples of the application of Faraday's Law. A current is ramped in a solenoid to produce circles of electric field intensity around the solenoid. The solenoid is surrounded by a conducting ring where the left half of the ring has a higher conductivity than the right half of the ring. What is observed is charge buildup at the boundaries between the two materials.
 

The charge buildup at the boundaries between the materials is what you KVLers are unable to see.
It's like glass for birds, or storks.

Excellent, I was waiting for something like this.  Now, here is a question for you:  what would happen if you replace the resistors with capacitors.  Say, instead of the 100 ohm resistor place  a 100nF capacitor, and instead of the 900 ohm resistor place a 10nF capacitor.  What would be the voltage between nodes A and D, VAD at steady state?   Spoiler alert: I did that experiment a few weeks ago and it looks very, very bad for team Lewin!

This is another common trait of believers: once they are cornered,  they change the topic to something new, so that the debunking has to start from scratch again and they can bask in the illusion that this time they could be right.

First is the ring with two resistors joined by conductors. Then the ring with two resistors and a battery. Then the ring with two resistors without conductors in between. Then the uniform resistive ring. Then the perfectly conducting ring. I have seen in the other thread the ring with a transistor in the loop. Now it's the two capacitors in a loop. What is next? A Josephson junction?

Two capacitors in a loop can lead to some nasty paradoxes. Will this invalidate the easily explained ring of just two resistors? Logic dictates that no, it won't. But believers are renowned for not being very good in applying logic.
Title: Re: #562 – Electroboom!
Post by: jesuscf on December 31, 2021, 03:39:14 am
The pretty much cancel each other in the conductor leaving an almost zero net electric field inside.

Right there is where you are wrong!  In Lewin's ring the the induced emf is evenly distributed in the conductors and resistors.  He failed to include that emf in the equivalent circuit, that is why he incorrectly concluded that KVL doesn't work.  Once again: under the influence of a varying magnetic field a piece of conductor must be modeled as an equivalent voltage source in series with a resistance (or more generally an impedance).  The voltage of of the source is derived from the induced emf, and the resistor (or impedance) is derived from the properties of the conductor.  By the way, the same applies to anything that 'conducts': wires, resistors, inductors, and even capacitors!

Quote
Then you use KVL and KCL to the complete and correct circuit to compute...

Sure, you use KVL to prove KVL.

Nope, you use a voltage measuring instrument to prove KVL.  But of course, in order to do that, you must know how to use the instrument!
Title: Re: #562 – Electroboom!
Post by: HuronKing on December 31, 2021, 03:44:45 am

But at this point in time they can base their view of classical electrodynamics on about 150 years of careful refinement by some of the most brilliant minds of the whole planet. So, I'll take that above what Maxwell (which was an absolute genius, but was also pioneering a new field) had to say in 1865. The ether is one glaring example.
A more refined view of classical electrodynamics in a less glaring one, and in fact...


Even in the context of Maxwell's work, there were things corrected in the decades following his treatise by Oliver Heaviside,

Quote
In his Treatise, Maxwell had built his theory around the vector and scalar potentials A and Ψ. They did not locate the energy correctly, however, and Heaviside regarded them as quite distant from the real workings of the field. He proceeded to work back from his energy-flow formula to find a new set of basic equations, equivalent to those in Maxwell’s Treatise but based directly on E and H and so better suited to treating energy flow. By combining two of Maxwell’s expressions relating the vector potential A to the fields E and H, Heaviside derived what he called the “second circuital law,” which related the curl of E directly to the rate of change of H—a fitting partner, he said, for Maxwell’s “first circuital law” relating the curl of H to E and its rate of change (see the box on page 53). By combining them with Maxwell’s expressions for the divergence of the electric displacement D and the magnetic induction B, Heaviside arrived at the compact set of four vector relations we now know as Maxwell’s equations.
https://physicstoday.scitation.org/doi/10.1063/PT.3.1788

But even the great Oliver Heaviside who gave us the modern formulation of Maxwell's Equations then went on to pretty famously dispute special relativity when he learned about it.

In the context of this discussion it's remarkable how much the KVL-Always-Holder depends on the appeal to authority and the quote-mining. It reminds me of anti-evolution proponents I've come across who run around quoting Darwin as if The Origin of Species is still, somehow, the state of the art understanding of the Theory of Evolution. And as I discovered many pages ago - vector calculus and non-conservative fields are utterly mysterious to the KVL-Always-Holder.
Title: Re: #562 – Electroboom!
Post by: Sredni on December 31, 2021, 03:55:40 am
The pretty much cancel each other in the conductor leaving an almost zero net electric field inside.

Right there is where you are wrong!  In Lewin's ring the the induced emf is evenly distributed in the conductors and resistors.  He failed to include that emf in the equivalent circuit, that is why he incorrectly concluded that KVL doesn't work.

So... is Belcher wrong?

(https://i.postimg.cc/SRnyVCmR/screenshot.png)
Source: Belcher's note available on Electroboom's channel

Because he too gets (basically) zero net field in the copper wires.
Title: Re: #562 – Electroboom!
Post by: jesuscf on December 31, 2021, 03:57:11 am
Wanna see another professor doing the same as Belcher, and Lewin? Micheal Melloch of Purdue University - the same Micheal Melloch whose videos Jesse Gordon has linked in the comments of his videos on Lewin's ring. Too bad he is doing what Lewin (and Belcher!) is doing:

Quote
Faraday's Law Example 4 (Electromagnetic Induction) 
https://www.youtube.com/watch?v=sAsxEBkaty8 (https://www.youtube.com/watch?v=sAsxEBkaty8)
This is the fourth of four examples of the application of Faraday's Law. A current is ramped in a solenoid to produce circles of electric field intensity around the solenoid. The solenoid is surrounded by a conducting ring where the left half of the ring has a higher conductivity than the right half of the ring. What is observed is charge buildup at the boundaries between the two materials.
 

The charge buildup at the boundaries between the materials is what you KVLers are unable to see.
It's like glass for birds, or storks.

Excellent, I was waiting for something like this.  Now, here is a question for you:  what would happen if you replace the resistors with capacitors.  Say, instead of the 100 ohm resistor place  a 100nF capacitor, and instead of the 900 ohm resistor place a 10nF capacitor.  What would be the voltage between nodes A and D, VAD at steady state?   Spoiler alert: I did that experiment a few weeks ago and it looks very, very bad for team Lewin!

This is another common trait of believers: once they are cornered that change the topics to something new, so that the debunking has to start from scratch and they can bask in the illusion that this time they could be right.

First is the ring with two resistors joined by conductors. Then the ring with two resistors and a battery. Then the ring with two resistors without conductors in between. Then the uniform resistive ring. Then the perfectly conducting ring. I have seen in the other thread the ring with a transitor in the loop. Now it's the two capacitors in a loop. What is next? A Josephson junction?

Two capacitors in a loop can lead to some nasty paradoxes. Will this invalidate the easily explained ring of just two resistors? Logic dictates that no, it won't. But believers are renowned for not being very good in applying logic.

That is where you are massively wrong.  If your theory is correct it must predict what happens when you change something in the experiment, and that prediction must perfectly match experimental results.  That is what KVL does every single time!!!  So go ahead, make your prediction!

By the way, if these small changes to the circuit upset you, I have another circuit that will completely  freak you out!  (And KVL still works!!!).
Title: Re: #562 – Electroboom!
Post by: jesuscf on December 31, 2021, 04:04:43 am
The pretty much cancel each other in the conductor leaving an almost zero net electric field inside.

Right there is where you are wrong!  In Lewin's ring the the induced emf is evenly distributed in the conductors and resistors.  He failed to include that emf in the equivalent circuit, that is why he incorrectly concluded that KVL doesn't work.

So... is Belcher wrong?

(https://i.postimg.cc/SRnyVCmR/screenshot.png)
Source: Belcher's note available on Electroboom's channel

Because he too gets (basically) zero net field in the copper wires.

You are confusing electric field with voltage.
Title: Re: #562 – Electroboom!
Post by: jesuscf on December 31, 2021, 04:09:18 am

But at this point in time they can base their view of classical electrodynamics on about 150 years of careful refinement by some of the most brilliant minds of the whole planet. So, I'll take that above what Maxwell (which was an absolute genius, but was also pioneering a new field) had to say in 1865. The ether is one glaring example.
A more refined view of classical electrodynamics in a less glaring one, and in fact...


Even in the context of Maxwell's work, there were things corrected in the decades following his treatise by Oliver Heaviside,


 I am pretty sure Heaviside says exactly the same as Maxwell about EMFs and Faraday's law.
Title: Re: #562 – Electroboom!
Post by: bsfeechannel on December 31, 2021, 07:23:35 am
Maybe for you.  Hence the importance of a formal education,

Which you don't have.

Quote
where with the proper guidance not only the correct knowledge is acquired in a timely manner,

Yeah. Become an electronics engineer in just six months. Apply here!

Quote
but the BS that characterizes team Lewin is weeded out.

If you really had a formal education in electronics engineering...

Quote
Very well genius: explain what is it a non-conservative field, in the context of Lewin's ring!

...you wouldn't be asking me to explain to you what a non-conservative field is.
Title: Re: #562 – Electroboom!
Post by: HuronKing on December 31, 2021, 08:07:36 am


Quote
Very well genius: explain what is it a non-conservative field, in the context of Lewin's ring!

...you wouldn't be asking me to explain to you what a non-conservative field is.

Robert Romer's paper explains exactly what is non-conservative about the Lewin ring. I suggest jesuscf rereads it. I had to take a semester long course in vector calculus to understand this. Kudos to anyone who can truly understand the mathematics (and all the prerequisites) required for this in a couple of weeks! Even more so if they retain this knowledge. Some of the EE students I teach have already forgotten a lot of the vector calculus by the time they get to my lab courses on power machinery.
http://www.fisica.uns.edu.ar/albert/archivos/15/119/420063006_tp_y_guias.pdf (http://www.fisica.uns.edu.ar/albert/archivos/15/119/420063006_tp_y_guias.pdf)
Title: Re: #562 – Electroboom!
Post by: thinkfat on December 31, 2021, 09:23:25 am

That is where you are massively wrong.  If your theory is correct it must predict what happens when you change something in the experiment, and that prediction must perfectly match experimental results.  That is what KVL does every single time!!!  So go ahead, make your prediction!

On that thought, why don't you retry your experiment with the resistor ring with a slight change, so that the magnetic flux geometry isn't in complete symmetry with your ring, but slightly off-center? Trevor Kearney suggested this as a challenge to those who support the idea of the scalar PD between two points being the "unique" voltage in the "Lewin Ring".

You will find that the "tiny voltage sources" in your circuit suddenly have a different value in every little piece of the wire and that for anything but the most trivial geometries you will need a numeric EM field solver and a model perfectly matching you circuit, to calculate that voltage. That in itself is not problematic, what is the much bigger problem is that you will not be able to quantify the uncertainty in your calculation, because you cannot know how close your numeric model is to reality.

The next problem will be to actually measure the "unique voltage" as a proof for your models accuracy, because now your measurement setup will have to use real-world wires and all the electric fields you measure will of course depend on the path of those wires relative to the magnetic flux. So what do you do now? You already don't know to what degree you can trust your calculation (or simulation) and then you don't know if any discrepancy between the voltage you measure and your calculation is due to "bad probing" or "bad modeling". For an engineer, that's not a good point to be.

And it doesn't end there. The next problem will be repeatability, as any small change in geometry will throw off your calculation and measurements again.

The beauty of the Maxwell-Faraday equation is that is requires no knowledge about the magnetic flux or path geometry. It just requires that the path be closed (that's what the "circle" in the Integral sign means) and then gives a perfect relation between the "voltage" you measure and the time-varying magnetic flux in the area enclosed by said path and the only uncertainty is how precisely you can measure the voltage. The only snail to swallow is accepting that voltages can be non-unique and will depend on the path you measure them on.
Title: Re: #562 – Electroboom!
Post by: ogden on December 31, 2021, 11:16:51 am
So, according to your logic in Lewin's ring, the wire on the top has half the induced EMF and the wire at the bottom has half the induced EMF.  So, can you now can compute the voltage VAD?

There is the EMF contribution of Eind, but there is also the scalar potential contribution of Ecoul. And guess what? The pretty much cancel each other in the conductor leaving an almost zero net electric field inside. Perfectly compliant with Ohm's law. The same superposition leaves a perfectly Ohm compliant resultant E field in both resistors. But being the resistance different, the field inside will be different as well.

Now, let's do 2+2: on branch 1 we have 0 field in the upper arc of conductor, strong E1 field inside the R1 resistor, and 0 field in the lower arc of conductor. The voltage is the same as that along the resistor alone: Vad1 = E1*deltaL, where deltaL is the length of the resistor.
In branch 2, for the same reasons we have Vad2 = E2*deltaL. Since the net electric field is different in the two resistors, we get Vad1 != Vad2

Hopefully you do agree that battery is EMF source, right? If solenoid does not influence voltmeter leads in Romer's/Lewin's circuit then it does not matter what kind of EMF source we are using, right? So, I take 1.5V button-cell battery with internal 100 Ohm resistance and bigger 1.5V battery with 10 Ohms internal resistance, connect them "in series" (yes, small battery is reversed). You say that somehow we can measure different voltages between two points where batteries are connected together - we place voltmeter next to button cell battery, it shows one voltage then we move voltmeter to other side next to bigger battery and we have different reading? Wow :-DD Yes, this is what Romer and Lewin are trying to prove, just with other kind of EMF source rather than batteries.
Title: Re: #562 – Electroboom!
Post by: Sredni on December 31, 2021, 01:31:55 pm
Hopefully you do agree that battery is EMF source, right?

A localized EMF source, to be precise.

Quote
If solenoid does not influence voltmeter leads in Romer's/Lewin's circuit then it does not matter what kind of EMF source we are using, right?

It matters quite a lot if the difference in between localized and distributed sources.

Quote
So, I take 1.5V button-cell battery with internal 100 Ohm resistance and bigger 1.5V battery with 10 Ohms internal resistance, connect them "in series" (yes, small battery is reversed). You say that somehow we can measure different voltages between two points where batteries are connected together - we place voltmeter next to button cell battery, it shows one voltage then we move voltmeter to other side next to bigger battery and we have different reading? Wow :-DD Yes, this is what Romer and Lewin are trying to prove, just with other kind of EMF source rather than batteries.

You can redo your experiment for localized EMF sources using two localized inductive EMF sources. One lumped secondary on the left, one lulmped secondary on the right and you will see that voltage won't be path dependent.
Title: Re: #562 – Electroboom!
Post by: Sredni on December 31, 2021, 01:44:26 pm
If your theory is correct it must predict what happens when you change something in the experiment, and that prediction must perfectly match experimental results.

Again: this is not my theory.
I wish it was. But not even in a thousand years I would be able to come up with such elegant, coherent and comprehensive theory. I can barely see beyond the shoulders of those giants who crafted it.

Yes, I can find what happens when you put two capacitors in a loop around a magnetic core. But the idealization of perfect conductors in between them could lead to problems. I suppose I can add a small finite resistance in the wires, and I will have a look a this problem because I find it interesting.
I cannot but notice that you did not specify the sort of excitation: is it a pulse (I guess it will lead to a short-lived oscillation), or a sinusoidal signal (we will end up with a capacitive divider, if there are no surprises), or worse a linearly changing magnetic flux that would try to force a constant current in the capacitors (this I have to think about)?
My take is that you did the experiments with a sinusoidal excitation. Is that correct?

But why is it that it's always "team Lewin" that has to solve your problems, while you guys never answer questions?
Title: Re: #562 – Electroboom!
Post by: ogden on December 31, 2021, 02:20:30 pm
Hopefully you do agree that battery is EMF source, right?
A localized EMF source, to be precise.

There is no problem to make two "distributed' batteries that resembles physical layout of Dr.Levin's test circuit. So what?

Quote
You can redo your experiment for localized EMF sources using two localized inductive EMF sources. One lumped secondary on the left, one lulmped secondary on the right and you will see that voltage won't be path dependent.

How about more than half-turn of Lewin's transformer? Let's say, one full turn for each "resistor"? According to your logic it automagically becomes localized EMF source and voltages are not path-dependent anymore?
Title: Re: #562 – Electroboom!
Post by: Sredni on December 31, 2021, 03:20:17 pm
There is no problem to make two "distributed' batteries that resembles physical layout of Dr.Levin's test circuit. So what?

The problem is that if you replace the battery with say 100 tiny batteries, you are still using lumped localized batteries. You will no longer have simple conducting wires connecting the resistors.

Quote
How about more than half-turn of Lewin's transformer? Let's say, one full turn for each "resistor"? According to your logic it automagically becomes localized EMF source and voltages are not path-dependent anymore?

You need to see where the variable magnetic region is, with respect to the circuit path.
Inside.
Outside.
They are different concepts.

(https://i.postimg.cc/YCKNjzg2/Spot-the-differences.png)



Title: Re: #562 – Electroboom!
Post by: jesuscf on December 31, 2021, 03:21:42 pm
Maybe for you.  Hence the importance of a formal education,

Which you don't have.

Quote
where with the proper guidance not only the correct knowledge is acquired in a timely manner,

Yeah. Become an electronics engineer in just six months. Apply here!

Quote
but the BS that characterizes team Lewin is weeded out.

If you really had a formal education in electronics engineering...

Quote
Very well genius: explain what is it a non-conservative field, in the context of Lewin's ring!

...you wouldn't be asking me to explain to you what a non-conservative field is.

I know what a non-conservative field is.  Now, you tell me, let see if we are talking about the same thing.
Title: Re: #562 – Electroboom!
Post by: jesuscf on December 31, 2021, 03:23:45 pm


Quote
Very well genius: explain what is it a non-conservative field, in the context of Lewin's ring!

...you wouldn't be asking me to explain to you what a non-conservative field is.

Robert Romer's paper explains exactly what is non-conservative about the Lewin ring. I suggest jesuscf rereads it. I had to take a semester long course in vector calculus to understand this. Kudos to anyone who can truly understand the mathematics (and all the prerequisites) required for this in a couple of weeks! Even more so if they retain this knowledge. Some of the EE students I teach have already forgotten a lot of the vector calculus by the time they get to my lab courses on power machinery.
http://www.fisica.uns.edu.ar/albert/archivos/15/119/420063006_tp_y_guias.pdf (http://www.fisica.uns.edu.ar/albert/archivos/15/119/420063006_tp_y_guias.pdf)

Ok, then in three small sentences or less explain where the non-conservative field is in Lewin's ring.
Title: Re: #562 – Electroboom!
Post by: jesuscf on December 31, 2021, 03:36:46 pm

That is where you are massively wrong.  If your theory is correct it must predict what happens when you change something in the experiment, and that prediction must perfectly match experimental results.  That is what KVL does every single time!!!  So go ahead, make your prediction!

On that thought, why don't you retry your experiment with the resistor ring with a slight change, so that the magnetic flux geometry isn't in complete symmetry with your ring, but slightly off-center? Trevor Kearney suggested this as a challenge to those who support the idea of the scalar PD between two points being the "unique" voltage in the "Lewin Ring".

You will find that the "tiny voltage sources" in your circuit suddenly have a different value in every little piece of the wire and that for anything but the most trivial geometries you will need a numeric EM field solver and a model perfectly matching you circuit, to calculate that voltage. That in itself is not problematic, what is the much bigger problem is that you will not be able to quantify the uncertainty in your calculation, because you cannot know how close your numeric model is to reality.

The next problem will be to actually measure the "unique voltage" as a proof for your models accuracy, because now your measurement setup will have to use real-world wires and all the electric fields you measure will of course depend on the path of those wires relative to the magnetic flux. So what do you do now? You already don't know to what degree you can trust your calculation (or simulation) and then you don't know if any discrepancy between the voltage you measure and your calculation is due to "bad probing" or "bad modeling". For an engineer, that's not a good point to be.

And it doesn't end there. The next problem will be repeatability, as any small change in geometry will throw off your calculation and measurements again.

The beauty of the Maxwell-Faraday equation is that is requires no knowledge about the magnetic flux or path geometry. It just requires that the path be closed (that's what the "circle" in the Integral sign means) and then gives a perfect relation between the "voltage" you measure and the time-varying magnetic flux in the area enclosed by said path and the only uncertainty is how precisely you can measure the voltage. The only snail to swallow is accepting that voltages can be non-unique and will depend on the path you measure them on.

But will KVL hold after everything you listed above?  Of course yes!  Furthermore, the voltage potential difference between two arbitrary nodes will be a unique scalar at any fixed time t.   Do you understand the difference of the voltage between two nodes and the voltage you measure between those two nodes?  As soon as you measure the voltage in any circuit, the instrument is modifying the circuit, and if you don't account for that, the results you get may be so baffling as to make you proclaim that "KVL is for the birds".
Title: Re: #562 – Electroboom!
Post by: thinkfat on December 31, 2021, 03:40:41 pm
Ok, then in three small sentences or less explain where the non-conservative field is in Lewin's ring.

\${\nabla \times E = -\frac{\partial B}{\partial t}}\$
Hint: left side of the equation.
Title: Re: #562 – Electroboom!
Post by: jesuscf on December 31, 2021, 03:43:41 pm
If your theory is correct it must predict what happens when you change something in the experiment, and that prediction must perfectly match experimental results.

Again: this is not my theory.
I wish it was. But not even in a thousand years I would be able to come up with such elegant, coherent and comprehensive theory. I can barely see beyond the shoulders of those giants who crafted it.

Yes, I can find what happens when you put two capacitors in a loop around a magnetic core. But the idealization of perfect conductors in between them could lead to problems. I suppose I can add a small finite resistance in the wires, and I will have a look a this problem because I find it interesting.
I cannot but notice that you did not specify the sort of excitation: is it a pulse (I guess it will lead to a short-lived oscillation), or a sinusoidal signal (we will end up with a capacitive divider, if there are no surprises), or worse a linearly changing magnetic flux that would try to force a constant current in the capacitors (this I have to think about)?
My take is that you did the experiments with a sinusoidal excitation. Is that correct?

Yes, please make the excitation sinusoidal.  It is easier to calculate (actually trivial), test, and measure.  Assume the amplitude of the induced emf is again 1V.

But why is it that it's always "team Lewin" that has to solve your problems, while you guys never answer questions?

Because team Lewin doesn't make experiments or perform measurements.


Title: Re: #562 – Electroboom!
Post by: ogden on December 31, 2021, 03:44:11 pm
There is no problem to make two "distributed' batteries that resembles physical layout of Dr.Levin's test circuit. So what?

The problem is that if you replace the battery with say 100 tiny batteries, you are still using lumped localized batteries. You will no longer have simple conducting wires connecting the resistors.

No. I suggested to make long, two curved batteries that resemble experiment configuration. Anyway result of curl integral does not change - you have long battery, tiny battery with long terminals or 100 tiny batteries in series. I do not see where you are going-to with your argument anyway.

Quote
Quote
How about more than half-turn of Lewin's transformer? Let's say, one full turn for each "resistor"? According to your logic it automagically becomes localized EMF source and voltages are not path-dependent anymore?

You need to see where the variable magnetic region is, with respect to the circuit path.
Inside.
Outside.
They are different concepts.

Not wat I did mean. Think transformer where you connect (short) one to another two secondary windings, one made out of 10 Ohms wire single turn, another out of 100 Ohms wire turn. Connections are made at one side of transformer. Question: voltages in those two windings are path-dependent or not?
Title: Re: #562 – Electroboom!
Post by: jesuscf on December 31, 2021, 03:51:42 pm
Ok, then in three small sentences or less explain where the non-conservative field is in Lewin's ring.

\${\nabla \times E = -\frac{\partial B}{\partial t}}\$
Hint: left side of the equation.

Yes, but the right part of the equation makes it conservative again!  The only way it will be non-conservative is if you do this:

\${\nabla \times E = 0}\$

Which is what Lewin define as KVL for any circuit (but only works with DC).  But Maxwell says that for KVL you need to include all EMFs, including induced EMFs!
Title: Re: #562 – Electroboom!
Post by: thinkfat on December 31, 2021, 04:29:33 pm

That is where you are massively wrong.  If your theory is correct it must predict what happens when you change something in the experiment, and that prediction must perfectly match experimental results.  That is what KVL does every single time!!!  So go ahead, make your prediction!

On that thought, why don't you retry your experiment with the resistor ring with a slight change, so that the magnetic flux geometry isn't in complete symmetry with your ring, but slightly off-center? Trevor Kearney suggested this as a challenge to those who support the idea of the scalar PD between two points being the "unique" voltage in the "Lewin Ring".

You will find that the "tiny voltage sources" in your circuit suddenly have a different value in every little piece of the wire and that for anything but the most trivial geometries you will need a numeric EM field solver and a model perfectly matching you circuit, to calculate that voltage. That in itself is not problematic, what is the much bigger problem is that you will not be able to quantify the uncertainty in your calculation, because you cannot know how close your numeric model is to reality.

The next problem will be to actually measure the "unique voltage" as a proof for your models accuracy, because now your measurement setup will have to use real-world wires and all the electric fields you measure will of course depend on the path of those wires relative to the magnetic flux. So what do you do now? You already don't know to what degree you can trust your calculation (or simulation) and then you don't know if any discrepancy between the voltage you measure and your calculation is due to "bad probing" or "bad modeling". For an engineer, that's not a good point to be.

And it doesn't end there. The next problem will be repeatability, as any small change in geometry will throw off your calculation and measurements again.

The beauty of the Maxwell-Faraday equation is that is requires no knowledge about the magnetic flux or path geometry. It just requires that the path be closed (that's what the "circle" in the Integral sign means) and then gives a perfect relation between the "voltage" you measure and the time-varying magnetic flux in the area enclosed by said path and the only uncertainty is how precisely you can measure the voltage. The only snail to swallow is accepting that voltages can be non-unique and will depend on the path you measure them on.

But will KVL hold after everything you listed above?  Of course yes!  Furthermore, the voltage potential difference between two arbitrary nodes will be a unique scalar at any fixed time t.   Do you understand the difference of the voltage between two nodes and the voltage you measure between those two nodes?  As soon as you measure the voltage in any circuit, the instrument is modifying the circuit, and if you don't account for that, the results you get may be so baffling as to make you proclaim that "KVL is for the birds".

My question would be: do you really believe that Dr. Lewin is such a massive fool that he didn't understand that the volt meters (or oscilloscopes) he used and the paths they were connected through were part of the circuit? His experiment was particularly crafted to display path dependence of voltages. Dozens of others have recreated it and came to the same observation. It isn't even new or original, I'm sure that other MIT video that has been referenced here (the one with the ring core) predates Lewins setup by 10 years or so.

With regards to whether KVL holds in that circuit or not - it doesn't, in the circuit Lewin chose for his experiment. It may very well hold in the circuit you created, but it is a different circuit, by your own words, because the electric fields are observed along different paths. It is also very, very peculiar and so full of uncertainties that it will be very hard for you to claim that the results you obtained are "exact" and "true". Your measurements were one or two percent off from your calculation as I recall, but can you say where that error came from? Probing? Calculation? Assumptions? Resistor tolerance? Volt meter error? That's why I called it a Nothing-Burger.
Title: Re: #562 – Electroboom!
Post by: jesuscf on December 31, 2021, 04:45:27 pm
My question would be: do you really believe that Dr. Lewin is such a massive fool that he didn't understand that the volt meters (or oscilloscopes) he used and the paths they were connected through were part of the circuit?

Yes.
Title: Re: #562 – Electroboom!
Post by: thinkfat on December 31, 2021, 04:59:28 pm
Ok, then in three small sentences or less explain where the non-conservative field is in Lewin's ring.

\${\nabla \times E = -\frac{\partial B}{\partial t}}\$
Hint: left side of the equation.

Yes, but the right part of the equation makes it conservative again!  The only way it will be non-conservative is if you do this:

\${\nabla \times E = 0}\$

Ah, no. The right part doesn't "make" the left part conservative. The right part just says "there is a time-varying magnetic flux" and the left part says "there's an electric field with a curl", and the curl is what makes the left part non-conservative. As to answer the "where", the "curled" electric field is in a plain perpendicular to the magnetic flux vector.
Title: Re: #562 – Electroboom!
Post by: jesuscf on December 31, 2021, 05:05:00 pm
With regards to whether KVL holds in that circuit or not - it doesn't, in the circuit Lewin chose for his experiment. It may very well hold in the circuit you created, but it is a different circuit, by your own words, because the electric fields are observed along different paths. It is also very, very peculiar and so full of uncertainties that it will be very hard for you to claim that the results you obtained are "exact" and "true". Your measurements were one or two percent off from your calculation as I recall, but can you say where that error came from? Probing? Calculation? Assumptions? Resistor tolerance? Volt meter error? That's why I called it a Nothing-Burger.

The main source of error, I believe, is probing.  One has to be very careful for the varying magnetic field not to affect the measuring instruments.  I find it interesting that you are willing to dismiss my experiment because I got a less than 2% discrepancy with the theoretical KVL calculation, but you are willing to immediately accept Lewin's results when he doesn't even provide any actual numerical measurements from his experiment.

Also, have you done the experiment yourself?  Do you have any measurements? 
Title: Re: #562 – Electroboom!
Post by: jesuscf on December 31, 2021, 05:06:25 pm
Ok, then in three small sentences or less explain where the non-conservative field is in Lewin's ring.

\${\nabla \times E = -\frac{\partial B}{\partial t}}\$
Hint: left side of the equation.

Yes, but the right part of the equation makes it conservative again!  The only way it will be non-conservative is if you do this:

\${\nabla \times E = 0}\$

Ah, no. The right part doesn't "make" the left part conservative. The right part just says "there is a time-varying magnetic flux" and the left part says "there's an electric field with a curl", and the curl is what makes the left part non-conservative. As to answer the "where", the "curled" electric field is in a plain perpendicular to the magnetic flux vector.

Yes it does!  That is why KVL do work.
Title: Re: #562 – Electroboom!
Post by: HuronKing on December 31, 2021, 05:25:48 pm
Ok, then in three small sentences or less explain where the non-conservative field is in Lewin's ring.

\${\nabla \times E = -\frac{\partial B}{\partial t}}\$
Hint: left side of the equation.

Yes, but the right part of the equation makes it conservative again!  The only way it will be non-conservative is if you do this:

\${\nabla \times E = 0}\$

Ah, no. The right part doesn't "make" the left part conservative. The right part just says "there is a time-varying magnetic flux" and the left part says "there's an electric field with a curl", and the curl is what makes the left part non-conservative. As to answer the "where", the "curled" electric field is in a plain perpendicular to the magnetic flux vector.

Yes it does!  That is why KVL do work.

Emphasis mine. Is there a typo here? Are you saying that the curl of E being 0 means the field is non-conservative?

Have you taken a course in vector calculus or are you self-taught on this subject?
Title: Re: #562 – Electroboom!
Post by: jesuscf on December 31, 2021, 05:46:49 pm
Ok, then in three small sentences or less explain where the non-conservative field is in Lewin's ring.

\${\nabla \times E = -\frac{\partial B}{\partial t}}\$
Hint: left side of the equation.

Yes, but the right part of the equation makes it conservative again!  The only way it will be non-conservative is if you do this:

\${\nabla \times E = 0}\$

Ah, no. The right part doesn't "make" the left part conservative. The right part just says "there is a time-varying magnetic flux" and the left part says "there's an electric field with a curl", and the curl is what makes the left part non-conservative. As to answer the "where", the "curled" electric field is in a plain perpendicular to the magnetic flux vector.

Yes it does!  That is why KVL do work.

Emphasis mine. Is there a typo here? Are you saying that the curl of E being 0 means the field is non-conservative?

Have you taken a course in vector calculus or are you self-taught on this subject?

Yes, the old good "Straw Man Fallacy".  Here, read it from Hayt yourself, from the attached pdf.
Title: Re: #562 – Electroboom!
Post by: thinkfat on December 31, 2021, 05:47:31 pm
With regards to whether KVL holds in that circuit or not - it doesn't, in the circuit Lewin chose for his experiment. It may very well hold in the circuit you created, but it is a different circuit, by your own words, because the electric fields are observed along different paths. It is also very, very peculiar and so full of uncertainties that it will be very hard for you to claim that the results you obtained are "exact" and "true". Your measurements were one or two percent off from your calculation as I recall, but can you say where that error came from? Probing? Calculation? Assumptions? Resistor tolerance? Volt meter error? That's why I called it a Nothing-Burger.

The main source of error, I believe, is probing.  One has to be very careful for the varying magnetic field not to affect the measuring instruments.  I find it interesting that you are willing to dismiss my experiment because I got a less than 2% discrepancy with the theoretical KVL calculation, but you are willing to immediately accept Lewin's results when he doesn't even provide any actual numerical measurements from his experiment.

Also, have you done the experiment yourself?  Do you have any measurements?

I don't dismiss your experiment because of those 2%, but because you cannot say where they come from. You "believe" the error is from probing. That means you're absolutely sure that your calculation (and thus assumptions) are correct. Why? Your approach to modeling was trivially assuming complete uniformity in distribution of the scalar PD. How can you be sure of that?

I can surely repeat the experiment as soon as I've found my stash of ring cores that is hidden somewhere in the pile of boxes from the last move. But I'm reasonably sure that I'll just observe what others have already found.
Title: Re: #562 – Electroboom!
Post by: HuronKing on December 31, 2021, 06:08:31 pm
Ok, then in three small sentences or less explain where the non-conservative field is in Lewin's ring.

\${\nabla \times E = -\frac{\partial B}{\partial t}}\$
Hint: left side of the equation.

Yes, but the right part of the equation makes it conservative again!  The only way it will be non-conservative is if you do this:

\${\nabla \times E = 0}\$

Ah, no. The right part doesn't "make" the left part conservative. The right part just says "there is a time-varying magnetic flux" and the left part says "there's an electric field with a curl", and the curl is what makes the left part non-conservative. As to answer the "where", the "curled" electric field is in a plain perpendicular to the magnetic flux vector.

Yes it does!  That is why KVL do work.

Emphasis mine. Is there a typo here? Are you saying that the curl of E being 0 means the field is non-conservative?

Have you taken a course in vector calculus or are you self-taught on this subject?

Yes, the old good "Straw Man Fallacy".  Here, read it from Hayt yourself, from the attached pdf.

Have you read it? No strawman here. I seriously have no idea WTF you're talking about because on the very page you cited, Hayt writes:
"Any field that satisfies an equation of the form of Eq. (20), (i.e., where the closed
line integral of the field is zero) is said to be a conservative field."

You wrote,
"The only way it will be non-conservative is if you do this:

∇×E=0"

Which is the opposite of what Hayt wrote. Is this a typo? Or have you self-taught yourself vector calculus? Those are the only two possibilities I can think of for such a statement.
Title: Re: #562 – Electroboom!
Post by: HuronKing on December 31, 2021, 06:12:28 pm
Hayt goes on to write,
"The integral is zero if ρ1 = 1, 2, 3,... , etc., but it is not zero for other values of ρ1,
or for most other closed paths, and the given field is not conservative. A conservative
field must yield a zero value for the line integral around every possible closed path."

There are paths in the Romer-Lewin ring whose line integrals are non-zero. Proof by counterexample then is that the field is non-conservative, KVL doesn't hold (as it requires 0 line integral around every path, as Feynman writes), and voltage is non-unique in the presence of non-conservative fields.

How is this even a debate?
Title: Re: #562 – Electroboom!
Post by: HuronKing on December 31, 2021, 06:19:51 pm
I should mention that, strictly speaking, a vector field with zero curl CAN be non-conservative. That is, zero curl does not imply the field is conservative.

https://mathinsight.org/path_dependent_zero_curl

This is why I, personally, much prefer the integral representation of Maxwell's Eqs than the differential form.

I wish I had this website when I went to college. Their examples and details are quite nice:
https://mathinsight.org/conservative_vector_field_determine
Title: Re: #562 – Electroboom!
Post by: jesuscf on December 31, 2021, 06:21:19 pm
Ok, then in three small sentences or less explain where the non-conservative field is in Lewin's ring.

\${\nabla \times E = -\frac{\partial B}{\partial t}}\$
Hint: left side of the equation.

Yes, but the right part of the equation makes it conservative again!  The only way it will be non-conservative is if you do this:

\${\nabla \times E = 0}\$

Ah, no. The right part doesn't "make" the left part conservative. The right part just says "there is a time-varying magnetic flux" and the left part says "there's an electric field with a curl", and the curl is what makes the left part non-conservative. As to answer the "where", the "curled" electric field is in a plain perpendicular to the magnetic flux vector.

Yes it does!  That is why KVL do work.

Emphasis mine. Is there a typo here? Are you saying that the curl of E being 0 means the field is non-conservative?

Have you taken a course in vector calculus or are you self-taught on this subject?

Yes, the old good "Straw Man Fallacy".  Here, read it from Hayt yourself, from the attached pdf.

Have you read it? No strawman here. I seriously have no idea WTF you're talking about because on the very page you cited, Hayt writes:
"Any field that satisfies an equation of the form of Eq. (20), (i.e., where the closed
line integral of the field is zero) is said to be a conservative field."

You wrote,
"The only way it will be non-conservative is if you do this:

∇×E=0"

Which is the opposite of what Hayt wrote. Is this a typo? Or have you self-taught yourself vector calculus? Those are the only two possibilities I can think of for such a statement.

What I mean is that, over the influence of an external magnetic field, if you add the voltage drops from the resistors you'll get a number that is not zero.  That is where the induced emf comes into play.  The sum of the voltage drops in the resistors is equal to the induced emf.  Energy in is equal to energy out.  The circuit is conservative and KVL works.  Is that clear now?
Title: Re: #562 – Electroboom!
Post by: jesuscf on December 31, 2021, 06:23:59 pm
Hayt goes on to write,
"The integral is zero if ρ1 = 1, 2, 3,... , etc., but it is not zero for other values of ρ1,
or for most other closed paths, and the given field is not conservative. A conservative
field must yield a zero value for the line integral around every possible closed path."

There are paths in the Romer-Lewin ring whose line integrals are non-zero. Proof by counterexample then is that the field is non-conservative, KVL doesn't hold (as it requires 0 line integral around every path, as Feynman writes), and voltage is non-unique in the presence of non-conservative fields.

How is this even a debate?

The only path allowed is the path of the circuit composed of wires and resistors.  The integral does not apply to any other arbitrary path.
Title: Re: #562 – Electroboom!
Post by: jesuscf on December 31, 2021, 06:26:20 pm
I should mention that, strictly speaking, a vector field with zero curl CAN be non-conservative. That is, zero curl does not imply the field is conservative.

https://mathinsight.org/path_dependent_zero_curl

This is why I, personally, much prefer the integral representation of Maxwell's Eqs than the differential form.

I wish I had this website when I went to college. Their examples and details are quite nice:
https://mathinsight.org/conservative_vector_field_determine

I agree.  I also prefer the integral representation.
Title: Re: #562 – Electroboom!
Post by: jesuscf on December 31, 2021, 06:31:50 pm
With regards to whether KVL holds in that circuit or not - it doesn't, in the circuit Lewin chose for his experiment. It may very well hold in the circuit you created, but it is a different circuit, by your own words, because the electric fields are observed along different paths. It is also very, very peculiar and so full of uncertainties that it will be very hard for you to claim that the results you obtained are "exact" and "true". Your measurements were one or two percent off from your calculation as I recall, but can you say where that error came from? Probing? Calculation? Assumptions? Resistor tolerance? Volt meter error? That's why I called it a Nothing-Burger.

The main source of error, I believe, is probing.  One has to be very careful for the varying magnetic field not to affect the measuring instruments.  I find it interesting that you are willing to dismiss my experiment because I got a less than 2% discrepancy with the theoretical KVL calculation, but you are willing to immediately accept Lewin's results when he doesn't even provide any actual numerical measurements from his experiment.

Also, have you done the experiment yourself?  Do you have any measurements?

I don't dismiss your experiment because of those 2%, but because you cannot say where they come from. You "believe" the error is from probing. That means you're absolutely sure that your calculation (and thus assumptions) are correct. Why? Your approach to modeling was trivially assuming complete uniformity in distribution of the scalar PD. How can you be sure of that?

I can surely repeat the experiment as soon as I've found my stash of ring cores that is hidden somewhere in the pile of boxes from the last move. But I'm reasonably sure that I'll just observe what others have already found.

Wait a minute!  Are you admitting that any arbitrary part of the circuit under the influence of an external varying magnetic field has a measurable induced emf?  Can you please explain that to Sredni?
Title: Re: #562 – Electroboom!
Post by: thinkfat on December 31, 2021, 06:45:22 pm
Hayt goes on to write,
"The integral is zero if ρ1 = 1, 2, 3,... , etc., but it is not zero for other values of ρ1,
or for most other closed paths, and the given field is not conservative. A conservative
field must yield a zero value for the line integral around every possible closed path."

There are paths in the Romer-Lewin ring whose line integrals are non-zero. Proof by counterexample then is that the field is non-conservative, KVL doesn't hold (as it requires 0 line integral around every path, as Feynman writes), and voltage is non-unique in the presence of non-conservative fields.

How is this even a debate?

The only path allowed is the path of the circuit composed of wires and resistors.  The integral does not apply to any other arbitrary path.

For Dr. Lewins experiment, it obviously needs to be pointed out that also the paths involving the measurement equipment are important. People seem to forget that, but it is key to understanding it. And of course the equation applies to every possible path through the circuit. How could it be any different.
Title: Re: #562 – Electroboom!
Post by: HuronKing on December 31, 2021, 07:28:42 pm
What I mean is that, over the influence of an external magnetic field, if you add the voltage drops from the resistors you'll get a number that is not zero.  That is where the induced emf comes into play.  The sum of the voltage drops in the resistors is equal to the induced emf.  Energy in is equal to energy out.  The circuit is conservative and KVL works.  Is that clear now?

This is not what conservative fields mean. Path-dependence is the key. I linked this to you pages and pages ago.
https://courses.lumenlearning.com/suny-osuniversityphysics/chapter/8-2-conservative-and-non-conservative-forces/

Don't know how much simpler it can get.

And also why I asked if you were self-taught on this subject - you are confusing terminology and making an inconsistent theory.

Quote
The only path allowed is the path of the circuit composed of wires and resistors.  The integral does not apply to any other arbitrary path.

This is very explicitly not what Hayt or any of the other published authors on this subject have written (when time-varying fields are present inside the contour of the loop).
KVL predicts that going clockwise around the loop or counterclockwise around the loop doesn't matter - the voltage should be the same, i.e. the line integral should be zero around ANY path of the wires and no work is done.

That doesn't just mean ANY configuration of wires (which you're admitting here your theory won't work for any arbitrary arrangement of wires, so KVL is dead?). Romer is perfectly okay predicting what happens when the wires move, that's what Figure 6 of his paper shows. But also the DIRECTION of the path matters. Of course it should - it's an integration of vectors, which have direction. Romer makes note of this in his Figure 3 and copious amounts of the mathematical literature on line integration discusses this.

This isn't at all surprising in a complete interpretation of Faraday's Law. The periodic output waveform from Romer's Figure 5 makes perfect sense - when the direction of the magnetic flux changes the sign of the voltages across the resistors also changes.

Title: Re: #562 – Electroboom!
Post by: bdunham7 on December 31, 2021, 07:33:57 pm
For Dr. Lewins experiment, it obviously needs to be pointed out that also the paths involving the measurement equipment are important. People seem to forget that, but it is key to understanding it. And of course the equation applies to every possible path through the circuit. How could it be any different.

Not pointing it out and hoping people missed it--perhaps expecting the curled e-field resulting from the changing magnetic flux to somehow stop at the perimeter of the inner ring--was the sole basis for this being a memorable, 'miind-blowing' demonstration. 
Title: Re: #562 – Electroboom!
Post by: jesuscf on December 31, 2021, 07:56:59 pm
What I mean is that, over the influence of an external magnetic field, if you add the voltage drops from the resistors you'll get a number that is not zero.  That is where the induced emf comes into play.  The sum of the voltage drops in the resistors is equal to the induced emf.  Energy in is equal to energy out.  The circuit is conservative and KVL works.  Is that clear now?

This is not what conservative fields mean. Path-dependence is the key. I linked this to you pages and pages ago.
https://courses.lumenlearning.com/suny-osuniversityphysics/chapter/8-2-conservative-and-non-conservative-forces/

Don't know how much simpler it can get.

And also why I asked if you were self-taught on this subject - you are confusing terminology and making an inconsistent theory.


In Lewin's ring can you please calculate the power added to the circuit by the external varying magnetic field (via Faraday's law) and compare it to the power consumed by the resistors?  Is energy conserved in the circuit or not?
Title: Re: #562 – Electroboom!
Post by: HuronKing on December 31, 2021, 09:12:30 pm
What I mean is that, over the influence of an external magnetic field, if you add the voltage drops from the resistors you'll get a number that is not zero.  That is where the induced emf comes into play.  The sum of the voltage drops in the resistors is equal to the induced emf.  Energy in is equal to energy out.  The circuit is conservative and KVL works.  Is that clear now?

This is not what conservative fields mean. Path-dependence is the key. I linked this to you pages and pages ago.
https://courses.lumenlearning.com/suny-osuniversityphysics/chapter/8-2-conservative-and-non-conservative-forces/

Don't know how much simpler it can get.

And also why I asked if you were self-taught on this subject - you are confusing terminology and making an inconsistent theory.


In Lewin's ring can you please calculate the power added to the circuit by the external varying magnetic field (via Faraday's law) and compare it to the power consumed by the resistors?  Is energy conserved in the circuit or not?

Your question is nonsensical in the context of this discussion. I also sent you this link.
https://courses.lumenlearning.com/physics/chapter/7-5-nonconservative-forces/

If you'd read this, or had any formal training in mathematics or physics (I'm happy to admit I received formal education, have you? I really am interested to know if you've ever passed exams on these topics), you'd see how ridiculous this question is when we are talking about non-conservative forces and fields. From the link above:

"An important characteristic is that the work done by a nonconservative force adds or removes mechanical energy from a system. Friction, for example, creates thermal energy that dissipates, removing energy from the system. Furthermore, even if the thermal energy is retained or captured, it cannot be fully converted back to work, so it is lost or not recoverable in that sense as well."

Are you asking if the energy consumed in the resistors to the Romer system is the same as the energy supplied by the field? Yes... obviously.

But that has diddly-squat to do with whether the work done in one path versus another path with the same start and end point is the same. Figure 1 in the link above make this point as simple as possible - probably too simple since friction can't supply energy and doesn't circulate like EM-fields can but alas this is the baseline I see.

Hayt makes this same distinction in the pages you posted earlier for Figure 4.4. And that's the question you should be asking. Is the work done on a charge the same in every possible path (which includes direction!) in the Romer ring? Romer's analysis shows that it is not... and so the voltage between A-B depends on the path... which is the whole reason KVL dies in this experiment.

I think you ought to read Hayt's Chapter 9. And if you can't make it that far, read Hayt's Chapter 7.6.
"Although we shall consider the scalar magnetic potential to a much greater extent in Chapter 8, when we introduce magnetic materials and discuss the magnetic circuit, one difference between V and Vm should be pointed out now: Vm is not a single-valued function of position. The electric potential V is single-valued; once a zero reference is assigned, there is only one value of V associated with each point in space. Such is not the case with Vm."

And Hayt goes on to prove it using a coaxial cable and ties the whole thing together beautifully on P.212.

I don't know how much more explicitly you want it. In Romer's ring, since the voltage is the contribution of both the electric potential and the magnetic potential (whose contribution is a current flow from induction), and the magnetic potential is multi-valued depending on the path of integration, then there is no unique, single-valued voltage in the network.

Let KVL die. Let yourself graduate from Hayt's Chapter 4 and embrace the entirety of Applied EM.

*Edit*
Corrected an ambiguity in my phrasing to make it clear that the magnetic potential is in amperes and gives rise to a current. Covid and flu booster can make one miss some things!
Title: Re: #562 – Electroboom!
Post by: jesuscf on December 31, 2021, 09:47:10 pm
Your question is nonsensical in the context of this discussion. I also sent you this link.
https://courses.lumenlearning.com/physics/chapter/7-5-nonconservative-forces/

If you'd read this, or had any formal training in mathematics or physics (I'm happy to admit I received formal education, have you? I really am interested to know if you've ever passed exams on these topics), you'd see how ridiculous this question is when we are talking about non-conservative forces and fields. From the link above:

It makes perfect sense, because the derivation of Kirchhoff's circuital laws are based in the principle of conservation of energy.  And yes, I read the link you posted, and it completely agrees with what I am saying.

(Yes, I have received formal education: 5 years undergrad, 2 years masters, and 4 years PhD.  What about you?)

Are you asking if the energy consumed in the resistors to the Romer system is the same as the energy supplied by the field? Yes... obviously.

Therefore the fields in the circuit are conservative, aren't they?

I don't know how much more explicitly you want it. In Romer's ring, since the voltage is the contribution of both the electric potential and the magnetic potential (whose contribution is a current flow from induction), and the magnetic potential is multi-valued depending on the path of integration, then there is no unique, single-valued voltage in the network.

You are mixing magnetic circuits with electric circuits here.  In the wire ring with two resistors we don't have a magnetic circuit.  But since you are at it, please add a thermal circuit, and why not, a hydraulic circuit as well!  All of those can be solved with laws equivalent to Ohms law, KVL, and KCL.
Title: Re: #562 – Electroboom!
Post by: HuronKing on December 31, 2021, 10:32:22 pm
It makes perfect sense, because the derivation of Kirchhoff's circuital laws are based in the principle of conservation of energy.  And yes, I read the link you posted, and it completely agrees with what I am saying.

The KVL law predicts that voltage in a closed path must be zero regardless of path. That's not true in the presence of non-conservative fields, period.

Quote
(Yes, I have received formal education: 5 years undergrad, 2 years masters, and 4 years PhD.  What about you?)

MSEE and 2 years Ph.D in progress and P.E. license.

Quote
Therefore the fields in the circuit are conservative, aren't they?

Go directly to jail. Do not pass GO. Do not collect $200.

How can you read Hayt, have a formal education (I assume in EE), and come away with that conclusion about magnetic fields? It's astounding to me.

Here, try this maybe?
https://phys.libretexts.org/Bookshelves/Electricity_and_Magnetism/Book%3A_Electricity_and_Magnetism_(Tatum)/09%3A_Magnetic_Potential/9.01%3A_Introduction_to_Magnetic_Potential

Quote
You are mixing magnetic circuits with electric circuits here.  In the wire ring with two resistors we don't have a magnetic circuit.  But since you are at it, please add a thermal circuit, and why not, a hydraulic circuit as well!  All of those can be solved with laws equivalent to Ohms law, KVL, and KCL.

So the contribution of the magnetic field does not matter to you? No wonder you can't see the non-conservative interactions at play.
Hayt is using magnetic circuits as part of his progression in getting to the complete form of Faraday's Law. Chapter 4 is about electrostatic fields, where all the fields are conservative, work done on any path is zero, and KVL can happily be applied.

But trouble starts to happen when the magnetic fields come in and start to wobble a little. Hayt writes on P.212,
Quote
We should remember that the electrostatic potential V is a conservative field; the magnetic scalar potential Vm is not a conservative field.

That is achieved by simply taking the closed loop line integrals of H (H and B are related directly by permeability constant so its trivial to do the unit conversions) and seeing that they no longer have unique values.

So what happens when we take the derivative of those B-fields and those derivative are not zero and then we integrate them?

By Chapter 9, KVL is gone. Hayt writes (P.280),

Quote
If B is not a function of time, (5) and (6) evidently reduce to the electrostatic equations

Which are just the KVL equations of Chapter 4. If B IS a function of time, then those equations can't reduce - KVL doesn't exist here, the closed loop contours are now entirely path dependent.

See, the trick is in seeing that magnetic forces on a charge in a magnetic field are not, by their nature, conservative. But, if they don't vary with time - everything is cool. Their derivative is zero and they disappear. Yet, if they DO vary with time - then all the nasty complications of path-dependency analyzed in Chapter 7 MUST be taken into account. Chapter 9 does this - but KVL cannot live in Chapter 9 because KVL is said, repeatedly by Hayt, to be path independent.

Faraday's Law is not. And can never be. Voltages in the presence of time-varying fields are not unique.
Title: Re: #562 – Electroboom!
Post by: jesuscf on December 31, 2021, 10:43:49 pm
It makes perfect sense, because the derivation of Kirchhoff's circuital laws are based in the principle of conservation of energy.  And yes, I read the link you posted, and it completely agrees with what I am saying.

The KVL law predicts that voltage in a closed path must be zero regardless of path. That's not true in the presence of non-conservative fields, period.

Quote
(Yes, I have received formal education: 5 years undergrad, 2 years masters, and 4 years PhD.  What about you?)

MSEE and 2 years Ph.D in progress and P.E. license.

Quote
Therefore the fields in the circuit are conservative, aren't they?

Go directly to jail. Do not pass GO. Do not collect $200.

How can you read Hayt, have a formal education (I assume in EE), and come away with that conclusion about magnetic fields? It's astounding to me.

Here, try this maybe?
https://phys.libretexts.org/Bookshelves/Electricity_and_Magnetism/Book%3A_Electricity_and_Magnetism_(Tatum)/09%3A_Magnetic_Potential/9.01%3A_Introduction_to_Magnetic_Potential

Quote
You are mixing magnetic circuits with electric circuits here.  In the wire ring with two resistors we don't have a magnetic circuit.  But since you are at it, please add a thermal circuit, and why not, a hydraulic circuit as well!  All of those can be solved with laws equivalent to Ohms law, KVL, and KCL.

So the contribution of the magnetic field does not matter to you? No wonder you can't see the non-conservative interactions at play.
Hayt is using magnetic circuits as part of his progression in getting to the complete form of Faraday's Law. Chapter 4 is about electrostatic fields, where all the fields are conservative, work done on any path is zero, and KVL can happily be applied.

But trouble starts to happen when the magnetic fields come in and start to wobble a little. Hayt writes on P.212,
Quote
We should remember that the electrostatic potential V is a conservative field; the magnetic scalar potential Vm is not a conservative field.

That is achieved by simply taking the closed loop line integrals of H (H and B are related directly by permeability constant so its trivial to do the unit conversions) and seeing that they no longer have unique values.

So what happens when we take the derivative of those B-fields and those derivative are not zero and then we integrate them?

By Chapter 9, KVL is gone. Hayt writes (P.280),

Quote
If B is not a function of time, (5) and (6) evidently reduce to the electrostatic equations

Which are just the KVL equations of Chapter 4. If B IS a function of time, then those equations can't reduce - KVL doesn't exist here, the closed loop contours are now entirely path dependent.

See, the trick is in seeing that magnetic forces on a charge in a magnetic field are not, by their nature, conservative. But, if they don't vary with time - everything is cool. Their derivative is zero and they disappear. Yet, if they DO vary with time - then all the nasty complications of path-dependency analyzed in Chapter 7 MUST be taken into account. Chapter 9 does this - but KVL cannot live in Chapter 9 because KVL is said, repeatedly by Hayt, to be path independent.

Faraday's Law is not. And can never be. Voltages in the presence of time-varying fields are not unique.

You are very confused by what you are reading.  For instance, can you provide a guesstimate of the reluctance of the ring circuit?  Assume the wiring is either copper or aluminum.   How would that affect the electric behavior of the ring circuit?
Title: Re: #562 – Electroboom!
Post by: HuronKing on December 31, 2021, 11:12:01 pm

You are very confused by what you are reading.  For instance, can you provide a guesstimate of the reluctance of the ring circuit?  Assume the wiring is either copper or aluminum.   How would that affect the electric behavior of the ring circuit?

And phhhewwwwww away we go. It's funny you should ask this though as if it's some kind of gotcha question. Hayt has a discussion of this too:

Quote
In an electric circuit, the voltage source is a part of the closed path; in the magnetic circuit, the current-carrying coil will surround or link the magnetic circuit. In tracing a magnetic circuit, we will not be able to identify a pair of terminals at which the magnetomotive force is applied. The analogy is closer here to a pair of coupled circuits in which induced voltages exist (and in which we will see in Chapter 9 that the closed line integral of E is also not zero).

So, the MMF of equation 44 on P.257 is entirely dependent on the number of loops of the coil enclosing the circuit. In other words, the value of the MMF changes based on how many times you circumscribe the closed loop path.
Hayt defines on P.256 that the Reluctance of a circuit is a function of the magnetic scalar potential (which we saw from Chapter 7.6 is path dependent). The units are A-t/Wb. So, the Reluctance is also path-dependent.

And I like how Hayt tosses in, for good measure, a reminder that closed line integral of E is NOT zero in this circuit or the circuits of Chapter 9 which is also explicitly the definition of a non-conservative field. I think he mentions this because on P.256 he writes after showing the closed loop integral of E dot dl,

Quote
In other words, Kirchhoff’s voltage law states that the rise in potential through the source is exactly equal to the fall in potential through the load.

Once more for the people in the back? What does he write on the very next page of P.257?

Quote
The analogy is closer here to a pair of coupled circuits in which induced voltages exist (and in which we will see in Chapter 9 that the closed line integral of E is also not zero)

Can we stop with this fiction that Hayt somehow agrees with the cockamamie proposition that KVL holds in all cases? He doesn't. None of the published authors do - because it's wrong.
Title: Re: #562 – Electroboom!
Post by: jesuscf on December 31, 2021, 11:20:07 pm

You are very confused by what you are reading.  For instance, can you provide a guesstimate of the reluctance of the ring circuit?  Assume the wiring is either copper or aluminum.   How would that affect the electric behavior of the ring circuit?

And phhhewwwwww away we go. It's funny you should ask this though as if it's some kind of gotcha question. Hayt has a discussion of this too:

Quote
In an electric circuit, the voltage source is a part of the closed path; in the magnetic circuit, the current-carrying coil will surround or link the magnetic circuit. In tracing a magnetic circuit, we will not be able to identify a pair of terminals at which the magnetomotive force is applied. The analogy is closer here to a pair of coupled circuits in which induced voltages exist (and in which we will see in Chapter 9 that the closed line integral of E is also not zero).

So, the MMF of equation 44 on P.257 is entirely dependent on the number of loops of the coil enclosing the circuit. In other words, the value of the MMF changes based on how many times you circumscribe the closed loop path.
Hayt defines on P.256 that the Reluctance of a circuit is a function of the magnetic scalar potential (which we saw from Chapter 7.6 is path dependent). The units are A-t/Wb. So, the Reluctance is also path-dependent.

And I like how Hayt tosses in, for good measure, a reminder that closed line integral of E is NOT zero in this circuit or the circuits of Chapter 9 which is also explicitly the definition of a non-conservative field. I think he mentions this because on P.256 he writes after showing the closed loop integral of E dot dl,

Quote
In other words, Kirchhoff’s voltage law states that the rise in potential through the source is exactly equal to the fall in potential through the load.

Once more for the people in the back? What does he write on the very next page of P.257?

Quote
The analogy is closer here to a pair of coupled circuits in which induced voltages exist (and in which we will see in Chapter 9 that the closed line integral of E is also not zero)

Can we stop with this fiction that Hayt somehow agrees with the cockamamie proposition that KVL holds in all cases? He doesn't. None of the published authors do - because it's wrong.

Once again, we are not dealing with a magnetic circuit here.
Title: Re: #562 – Electroboom!
Post by: HuronKing on December 31, 2021, 11:26:42 pm
Once again, we are not dealing with a magnetic circuit here.

That is why you fail. I've taken you, step-by-step, through Hayt's progression from electrostatics, into magnetic circuits (where the non-conservative properties of magnetism are explored), and how they directly lead to the conclusions of Chapter 9, which Hayt teased in Chapter 8, and I quote again,

Quote
The analogy is closer here to a pair of coupled circuits in which induced voltages exist (and in which we will see in Chapter 9 that the closed line integral of E is also not zero)

Your answer is the equivalent of saying,
"Nuh uh!"

A Ph.D. is telling me that the properties of magnetic fields do not apply to the Romer ring. I guess Hayt was just navel-gazing in Chapters 7 and 8 and none of that stuff has anything to do with Chapter 9...

I'm truly astounded (and I guess I now understand Lewin's comments in his lecture about being accused of cheating on the experiment). Unless you have something substantive, I am not replying to your comments anymore.
Title: Re: #562 – Electroboom!
Post by: jesuscf on December 31, 2021, 11:54:54 pm
Once again, we are not dealing with a magnetic circuit here.

That is why you fail. I've taken you, step-by-step, through Hayt's progression from electrostatics, into magnetic circuits (where the non-conservative properties of magnetism are explored), and how they directly lead to the conclusions of Chapter 9, which Hayt teased in Chapter 8, and I quote again,

Quote
The analogy is closer here to a pair of coupled circuits in which induced voltages exist (and in which we will see in Chapter 9 that the closed line integral of E is also not zero)

Your answer is the equivalent of saying,
"Nuh uh!"

A Ph.D. is telling me that the properties of magnetic fields do not apply to the Romer ring. I guess Hayt was just navel-gazing in Chapters 7 and 8 and none of that stuff has anything to do with Chapter 9...

I'm truly astounded (and I guess I now understand Lewin's comments in his lecture about being accused of cheating on the experiment). Unless you have something substantive, I am not replying to your comments anymore.

Great, from now on I will try not reply to your messages either as I am tired of your fallacious diatribe.
Title: Re: #562 – Electroboom!
Post by: HuronKing on January 01, 2022, 12:13:47 am
Great, from now on I will try not reply to your messages either as I am tired of your fallacious diatribe.

LoL. You're the one who trotted out Hayt. You don't even know what's in it as I took you page by page. As for fallacies, you never actually said anything about what I wrote is fallacious. However, there is absolutely nothing more fallacious than this statement, here once more in all its glory:

Quote
Yes, but the right part of the equation makes it conservative again!  The only way it will be non-conservative is if you do this:

∇×E=0

When asked for clarification, you told me this is not a typo.

The statement that this equation is non-conservative is so astoundingly, blisteringly, wrong that it entirely sums up the position of the KVL-Always-Holder.
After all this, you still can't tell the difference between a conservative electrostatic field and a non-conservative magnetostatic field - it's hopeless to try to go beyond this and analyze what happens if one takes the derivative of this non-conservative field.

And this one,
Quote
Therefore the fields in the circuit are conservative, aren't they?

And you're a Ph.D? Of electrical engineering (I assume?)?!?!
Title: Re: #562 – Electroboom!
Post by: bsfeechannel on January 01, 2022, 06:37:36 am
Not pointing it out and hoping people missed it--perhaps expecting the curled e-field resulting from the changing magnetic flux to somehow stop at the perimeter of the inner ring--was the sole basis for this being a memorable, 'miind-blowing' demonstration.

The sole basis for the memorable mind blowing demonstration was to be sure that people would not pay attention when he demonstrated in his very lecture 16 @27:00 that the electric field outside the solenoid has no influence whatsoever over the probes (In the previous lecture 15, he went in more detail about this). And that's because inside any closed path that does not include the solenoid the induced electric field is, wait for it, wait for it, conservative.

And it is conservative because the magnetic field outside the solenoid is precisely dick.

Remember curl E = - dB/dt. If B = 0, curlE = 0.

What I like about Lewin's super demo is how he covered all the bases. He left no escape route for those who were not prepared. After his demo you either grow a pair and learn this thing properly or you'll be immediately identified as a dilettante.
Title: Re: #562 – Electroboom!
Post by: thinkfat on January 01, 2022, 08:49:42 am
The funny thing about this whole "energy conservative" stuff is, there has to be a source of energy in the first place, for the circuit to consume it. This source of energy can be either lumped or not. If the physical circuit allows for it, you can lump the energy source into a two-terminal component and put it actually into the circuit. In other circuits, like in Dr. Lewins experiment, you can not do it. You're forced to admit that the energy source is external to the circuit and has no discrete source, and this is where KVL stops working.

The reason is simply because such circuit is not energy conservative. The energy source is not IN the circuit. This breaks the very foundation on which KVL is based.

PS: before anyone gets any funny ideas: No, Scalar PD is not the savior of KVL here. You can calculate a theoretical "half-way" voltage between the two points where the Oscilloscopes are connected in Dr. Lewins circuit, but you cannot replace the EMF with a voltage source connected to those points without disturbing the results.
Title: Re: #562 – Electroboom!
Post by: jesuscf on January 01, 2022, 10:00:02 am
The funny thing about this whole "energy conservative" stuff is, there has to be a source of energy in the first place, for the circuit to consume it. This source of energy can be either lumped or not. If the physical circuit allows for it, you can lump the energy source into a two-terminal component and put it actually into the circuit. In other circuits, like in Dr. Lewins experiment, you can not do it. You're forced to admit that the energy source is external to the circuit and has no discrete source, and this is where KVL stops working.

The reason is simply because such circuit is not energy conservative. The energy source is not IN the circuit. This breaks the very foundation on which KVL is based.

I see you are a lot more confused that I thought you were.  Let me try to explain, in the simplest possible terms what is going on:

This is how Lewin calculated (correctly by the way) the loop current through the circuit:

\$I=\frac{emf}{R_{1}+R_{2}}\$

Or, rearranging a bit, so that you can clearly see what is going on:

\$I^{2}R_{1}+I^{2}R_{2}=emf\cdot I\$

Can you read what the equation above is telling you?  If you are not happy with the units of the equation, multiply both sides by some arbitrary time to find the energy added or consumed by the circuit during that time.




Title: Re: #562 – Electroboom!
Post by: thinkfat on January 01, 2022, 10:34:22 am
The funny thing about this whole "energy conservative" stuff is, there has to be a source of energy in the first place, for the circuit to consume it. This source of energy can be either lumped or not. If the physical circuit allows for it, you can lump the energy source into a two-terminal component and put it actually into the circuit. In other circuits, like in Dr. Lewins experiment, you can not do it. You're forced to admit that the energy source is external to the circuit and has no discrete source, and this is where KVL stops working.

The reason is simply because such circuit is not energy conservative. The energy source is not IN the circuit. This breaks the very foundation on which KVL is based.

I see you are a lot more confused that I thought you were.  Let me try to explain, in the simplest possible terms what is going on:

This is how Lewin calculated (correctly by the way) the loop current through the circuit:

\$I=\frac{emf}{R_{1}+R_{2}}\$

Or, rearranging a bit, so that you can clearly see what is going on:

\$I^{2}R_{1}+I^{2}R_{2}=emf\cdot I\$

Can you read what the equation above is telling you?  If you are not happy with the units of the equation, multiply both sides by some arbitrary time to find the energy added or consumed by the circuit during that time.

Just tell me where in the circuit of Dr. Lewin you find "emf" and we're good. Note: you're not allowed to change the measurement results.
Title: Re: #562 – Electroboom!
Post by: ogden on January 01, 2022, 12:10:07 pm
And it is conservative because the magnetic field outside the solenoid is precisely dick.

Solenoid EM fields are miraculously nonexistant for voltmeter leads running next to the test circuit parts. Right.  :-DD
Title: Re: #562 – Electroboom!
Post by: jesuscf on January 01, 2022, 04:24:39 pm
The funny thing about this whole "energy conservative" stuff is, there has to be a source of energy in the first place, for the circuit to consume it. This source of energy can be either lumped or not. If the physical circuit allows for it, you can lump the energy source into a two-terminal component and put it actually into the circuit. In other circuits, like in Dr. Lewins experiment, you can not do it. You're forced to admit that the energy source is external to the circuit and has no discrete source, and this is where KVL stops working.

The reason is simply because such circuit is not energy conservative. The energy source is not IN the circuit. This breaks the very foundation on which KVL is based.

I see you are a lot more confused that I thought you were.  Let me try to explain, in the simplest possible terms what is going on:

This is how Lewin calculated (correctly by the way) the loop current through the circuit:

\$I=\frac{emf}{R_{1}+R_{2}}\$

Or, rearranging a bit, so that you can clearly see what is going on:

\$I^{2}R_{1}+I^{2}R_{2}=emf\cdot I\$

Can you read what the equation above is telling you?  If you are not happy with the units of the equation, multiply both sides by some arbitrary time to find the energy added or consumed by the circuit during that time.

Just tell me where in the circuit of Dr. Lewin you find "emf" and we're good. Note: you're not allowed to change the measurement results.

First let me clarify that in the equation above both the emf and the current 'I' are a function of time, so in order to find the correct amount of energy added to the circuit, one must add the instantaneous energy values (or find the integral) over that time.  Nevertheless, a dimensional analysis as I did in the quote above, in my opinion could be very useful to understand what is going on.

In the case of Lewin's circuit the induced emf is uniformly distributed across the elements of circuit: the two pieces of wire, one at the top and and one at the bottom, as well as the two resistors, one to the left and one to the right.  When drawing the equivalent circuit, it is very important to considered the emf contributions of these four elements.  If the dimensions of the resistors are much smaller than the dimensions of the wires, a good approximation would be to considered the emf contribution of the wires only.
Title: Re: #562 – Electroboom!
Post by: bsfeechannel on January 02, 2022, 12:52:19 am
And it is conservative because the magnetic field outside the solenoid is precisely dick.

Solenoid EM fields are miraculously nonexistant for voltmeter leads running next to the test circuit parts. Right.  :-DD

KVLers are a lot happier than engineers. They keep laughing when we show that they don't understand electromagnetism.

Watch Lewin's lecture 15 where he probes a solenoid with a calibrated Hall sensor.

https://www.youtube.com/watch?v=MXuZ1SRjpqk (https://www.youtube.com/watch?v=MXuZ1SRjpqk)

So, in Lewin's setup, where he puts the ring midway from the ends of the solenoid, there's no varying magnetic field outside. Therefore there will be no non-conservative electric field that could interfere with the probes connected to the circuit.

Now you can stop laughing and weep together with us for all the unfortunate KVLers out there.
Title: Re: #562 – Electroboom!
Post by: ogden on January 02, 2022, 10:20:10 am
Watch Lewin's lecture 15 where he probes a solenoid with a calibrated Hall sensor.

So, in Lewin's setup, where he puts the ring midway from the ends of the solenoid, there's no varying magnetic field outside. Therefore there will be no non-conservative electric field that could interfere with the probes connected to the circuit.

Seems, you did not get what I was saying. Rephrased sentence: Solenoid EM fields that causes EMF in the test circuit, are miraculously nonexistant for voltmeter leads running next to the test circuit parts.

[edit] Those who are impatient waiting for answer from "Dr.Lewin's science guru", can try to find answer empirically. All what's needed - AC mains transformer, insulated wire and AC voltmeter. Try to measure EMF for 1) single turn - tightly wrapped around core, then 2) big enough loop that would imitate voltmeter leads not receiving any EMF. 3) compare. Sample configuration for supposedly most popular E-E core transformer attached. Note that test wire is going through outer leg of "E". Comments about your test results are welcome. Do not hesitate to make even bigger loops than shown in M2 pic.
Title: Re: #562 – Electroboom!
Post by: jesuscf on January 02, 2022, 08:53:08 pm
Watch Lewin's lecture 15 where he probes a solenoid with a calibrated Hall sensor.

So, in Lewin's setup, where he puts the ring midway from the ends of the solenoid, there's no varying magnetic field outside. Therefore there will be no non-conservative electric field that could interfere with the probes connected to the circuit.

Seems, you did not get what I was saying. Rephrased sentence: Solenoid EM fields that causes EMF in the test circuit, are miraculously nonexistant for voltmeter leads running next to the test circuit parts.

[edit] Those who are impatient waiting for answer from "Dr.Lewin's science guru", can try to find answer empirically. All what's needed - AC mains transformer, insulated wire and AC voltmeter. Try to measure EMF for 1) single turn - tightly wrapped around core, then 2) big enough loop that would imitate voltmeter leads not receiving any EMF. 3) compare. Sample configuration for supposedly most popular E-E core transformer attached. Note that test wire is going through outer leg of "E". Comments about your test results are welcome. Do not hesitate to make even bigger loops than shown in M2 pic.

More likely than not, you'll not get an answer from "Dr.Lewin's science guru".  I just got my Fluke 187 working again, so with the help of an small audio transformer, a function generator (sine wave, 1 kHz, 20Vpp), and my Brymen BM869s, I got what is shown in the attached picture.

Title: Re: #562 – Electroboom!
Post by: thinkfat on January 02, 2022, 09:41:37 pm
I'm not quite sure what you two are trying to demonstrate there. Why should the size/diameter/area of the loop make any difference, as long as it loops through the core?
How about you make a loop that does not enclose any part of the core and see if it picks up anything?
Title: Re: #562 – Electroboom!
Post by: ogden on January 02, 2022, 10:20:37 pm
I just got my Fluke 187 working again, so with the help of an small audio transformer, a function generator (sine wave, 1 kHz, 20Vpp), and my Brymen BM869s, I got what is shown in the attached picture.

Results as expected - close to identical. Surprisingly quick! Respect 8)

Why should the size/diameter/area of the loop make any difference, as long as it loops through the core?

Bingo. Size/diameter/area does not matter in Maxwell-Faraday equation. It also means that two circuits show in attachment are equivalent and will show equal readings on voltmeter. Those who are familiar with Romer's/Lewin's experiment and still believe that it is not "probing error", can take a moment to sink it in. Yes, it comes from Romer's paper, edited using windows paint (lol).

[edit] All this just to frustrate students and fellow scientists with resistive divider which *itself* receives EMF from EM induction. Obviously KVL holds. EMF = I*(R1 + R2), V1 = EMF * R1/(R1+R2), V2 = EMF * R2/(R1+R2)
Title: Re: #562 – Electroboom!
Post by: thinkfat on January 02, 2022, 10:50:28 pm
Why should the size/diameter/area of the loop make any difference, as long as it loops through the core?

Bingo. Size/diameter/area does not matter in Maxwell-Faraday equation. It also means that two circuits show in attachment are equivalent and will show equal readings on voltmeter. Those who are familiar with Romer's/Lewin's experiment and still believe that it is not "probing error", can take a moment to sink it in. Yes, it comes from Romer's paper, edited using windows paint (lol).

[edit] All this just to frustrate students and fellow scientists with resistive divider which *itself* receives EMF from EM induction. Obviously KVL holds. EMF = I*(R1 + R2), V1 = EMF * R1/(R1+R2), V2 = EMF * R2/(R1+R2)

Err, yes.. But that was entirely not the point of Lewin's experiment.
Title: Re: #562 – Electroboom!
Post by: ogden on January 02, 2022, 10:58:34 pm
Bingo. Size/diameter/area does not matter in Maxwell-Faraday equation. It also means that two circuits show in attachment are equivalent and will show equal readings on voltmeter. Those who are familiar with Romer's/Lewin's experiment and still believe that it is not "probing error", can take a moment to sink it in. Yes, it comes from Romer's paper, edited using windows paint (lol).

[edit] All this just to frustrate students and fellow scientists with resistive divider which *itself* receives EMF from EM induction. Obviously KVL holds. EMF = I*(R1 + R2), V1 = EMF * R1/(R1+R2), V2 = EMF * R2/(R1+R2)

Err, yes.. But that was entirely not the point of Lewin's experiment.

AFAIK point of Lewin's experiment was to prove that voltages in his test circuit are "path-dependent" and "KVL is for birds". Both statements are simply BS. Old man frustrated himself with overcomplicated experiment. [edit] You can't agree to both - my equations *and* Lewin's equations (attached)
Title: Re: #562 – Electroboom!
Post by: jesuscf on January 03, 2022, 12:02:27 am
Bingo. Size/diameter/area does not matter in Maxwell-Faraday equation. It also means that two circuits show in attachment are equivalent and will show equal readings on voltmeter. Those who are familiar with Romer's/Lewin's experiment and still believe that it is not "probing error", can take a moment to sink it in. Yes, it comes from Romer's paper, edited using windows paint (lol).

[edit] All this just to frustrate students and fellow scientists with resistive divider which *itself* receives EMF from EM induction. Obviously KVL holds. EMF = I*(R1 + R2), V1 = EMF * R1/(R1+R2), V2 = EMF * R2/(R1+R2)

Err, yes.. But that was entirely not the point of Lewin's experiment.

AFAIK point of Lewin's experiment was to prove that voltages in his test circuit are "path-dependent" and "KVL is for birds". Both statements are simply BS. Old man frustrated himself with overcomplicated experiment. [edit] You can't agree to both - my equations *and* Lewin's equations (attached)

Here, I fixed Lewin's blackboard to match reality.
Title: Re: #562 – Electroboom!
Post by: Sredni on January 03, 2022, 12:50:21 am
Bingo. Size/diameter/area does not matter in Maxwell-Faraday equation. It also means that two circuits show in attachment are equivalent and will show equal readings on voltmeter. Those who are familiar with Romer's/Lewin's experiment and still believe that it is not "probing error", can take a moment to sink it in. Yes, it comes from Romer's paper, edited using windows paint (lol).

[edit] All this just to frustrate students and fellow scientists with resistive divider which *itself* receives EMF from EM induction. Obviously KVL holds. EMF = I*(R1 + R2), V1 = EMF * R1/(R1+R2), V2 = EMF * R2/(R1+R2)

Err, yes.. But that was entirely not the point of Lewin's experiment.

AFAIK point of Lewin's experiment was to prove that voltages in his test circuit are "path-dependent" and "KVL is for birds". Both statements are simply BS. Old man frustrated himself with overcomplicated experiment. [edit] You can't agree to both - my equations *and* Lewin's equations (attached)

Voltages ARE path-dependent. What you fail to see is that in order to see the path dependency you have to enclose a variable flux region between the paths. In the case of Lewin's ring

(https://i.postimg.cc/VvFWycbH/Voltage-can-be-path-dependent.jpg)
https://i.postimg.cc/VvFWycbH/Voltage-can-be-path-dependent.jpg

you are just considering two green paths which give equal voltages and come to the conclusion that all paths always give equal voltages.
When you cannot physically put your wires inside the dB/dt region, you are left with two possible sets of paths (the green and purple ones) that give two possible and different values.
Here, a few more pictures:

https://electronics.stackexchange.com/questions/551244/what-would-a-voltmeter-measure-if-you-had-an-electromotive-force-generated-by-a
Title: Re: #562 – Electroboom!
Post by: bdunham7 on January 03, 2022, 01:33:57 am
Voltages ARE path-dependent. What you fail to see is that in order to see the path dependency you have to enclose a variable flux region between the paths. In the case of Lewin's ring

Yes, the voltage generated in the voltmeter test leads depends on the path--i.e. where you put them.  >:D

(https://www.eevblog.com/forum/amphour/562-electroboom!/?action=dlattach;attach=1367822;image)
Title: Re: #562 – Electroboom!
Post by: Sredni on January 03, 2022, 02:38:45 am
Voltages ARE path-dependent. What you fail to see is that in order to see the path dependency you have to enclose a variable flux region between the paths. In the case of Lewin's ring

Yes, the voltage generated in the voltmeter test leads depends on the path--i.e. where you put them.  >:D

(https://www.eevblog.com/forum/amphour/562-electroboom!/?action=dlattach;attach=1367822;image)

Well, I would have drawn that purple, to match the picture below but... what do you think you have accomplished pointing that out?
The circuit below already showed path dependent voltages. That you have drawn is  a 0V path - it is measuring the voltage along the transformer coil. I simply reused a previously linked figure but if you wish you can draw ten more 0V path. What's your point? As I wrote above, in order to see path dependency you need to enclose the dB/dt region inside your paths.
Title: Re: #562 – Electroboom!
Post by: bsfeechannel on January 03, 2022, 06:01:56 am
Those who are familiar with Romer's/Lewin's experiment and still believe that it is not "probing error", can take a moment to sink it in.

OK. Cool. Since you guys are quick to answer and even do experiments. Help me here, please. What will V1 read?

(https://www.eevblog.com/forum/amphour/562-electroboom!/?action=dlattach;attach=1367984;image)

EDIT: Awwww! Now I get it.  :palm:

I need time not to see the probing error, but to absorb the impact of the stupidity of KVLers.

Non, mon chouchou! That's not how you check if there's interference with your probing. Let me show you:

(https://www.eevblog.com/forum/amphour/562-electroboom!/?action=dlattach;attach=1368059;image)

I can't believe I have to teach you basic practical engineering.

If you guys had some formal training in the field, you must have found your degree in the dumpster.
Title: Re: #562 – Electroboom!
Post by: bsfeechannel on January 03, 2022, 07:07:32 am
Here, I fixed Lewin's blackboard to match reality.

Here, I corrected your blunder.

(https://www.eevblog.com/forum/amphour/562-electroboom!/?action=dlattach;attach=1368065;image)

We have to thank Lewin and his superdemo. He really made the dilettantes unhappy.
Title: Re: #562 – Electroboom!
Post by: ogden on January 03, 2022, 07:18:06 am
EDIT: Awwww! Now I get it.  :palm:

I need time not to see the probing error, but to absorb the impact of the stupidity of KVLers.

Right. You do it by looking incredibly stupid yourself. Voltmeter leads of the circuit below are part of loop receiving EMF or not?

Hint: yes indeed. EM induction does not care - wire is resistor or voltmeter leads. It does cause EMF in *BOTH*. There is no magical "path directivity", just same old good Faraday's law in action Lewin's preachers pretend to know so well.
Title: Re: #562 – Electroboom!
Post by: ogden on January 03, 2022, 05:44:43 pm
Voltages ARE path-dependent. What you fail to see is that in order to see the path dependency you have to enclose a variable flux region between the paths. In the case of Lewin's ring
Yes, the voltage generated in the voltmeter test leads depends on the path--i.e. where you put them.  >:D

Well, I would have drawn that purple, to match the picture below but... what do you think you have accomplished pointing that out?
The circuit below already showed path dependent voltages. That you have drawn is  a 0V path - it is measuring the voltage along the transformer coil.

So you are saying - to measure voltage for given path, one shall route voltmeter leads along that path? In case I have only one voltmeter, I need to place voltmeter and it's leads on right side of Lewin's experiment to measure voltage on right side resistor, as soon as I move voltmeter and leads to left side - I suddenly measure voltage on left side resistor? This is not because of electromagnetic induction but "path dependency"?

I wonder - how one can practically demonstrate path-dependency using transformer with let's say, 100 turns of secondary? He shall wind 100 turns of voltmeter leads on transformer core or what? Would be good to get description of experiment so those who are interested, can repeat. Thank you.
Title: Re: #562 – Electroboom!
Post by: jesuscf on January 03, 2022, 06:04:40 pm
Voltages ARE path-dependent. What you fail to see is that in order to see the path dependency you have to enclose a variable flux region between the paths. In the case of Lewin's ring
Yes, the voltage generated in the voltmeter test leads depends on the path--i.e. where you put them.  >:D

Well, I would have drawn that purple, to match the picture below but... what do you think you have accomplished pointing that out?
The circuit below already showed path dependent voltages. That you have drawn is  a 0V path - it is measuring the voltage along the transformer coil.

So you are saying - to measure voltage for given path, one shall route voltmeter leads along that path? In case I have only one voltmeter, I need to place voltmeter and it's leads on right side of Lewin's experiment to measure voltage on right side resistor, as soon as I move voltmeter and leads to left side - I suddenly measure voltage on left side resistor? This is not because of electromagnetic induction but "path dependency"?

I wonder - how one can practically demonstrate path-dependency using transformer with let's say, 100 turns of secondary? He shall wind 100 turns of voltmeter leads on transformer core or what? Would be good to get description of experiment so those who are interested, can repeat. Thank you.

You make a very good point here.  Sredni's "path dependency" is less and less noticeable the more turns on the secondary.  I did the experiment with a three turn secondary and posted the results in this forum.  Sredni almost calculated the voltage between nodes A and D, VAD but suddenly realized that if he did the calculation he would show that KVL works perfectly, and backed up quickly...  Cyriel Mabilde in Youtube did a similar experiment too with a five turn secondary and also demonstrated that KVL works perfectly.  Now I am waiting for some insight from Sredni of what would happen if we replace the resistors in the loop with capacitors, but he is MIA...
Title: Re: #562 – Electroboom!
Post by: Sredni on January 04, 2022, 04:30:08 pm
You make a very good point here.  Sredni's "path dependency" is less and less noticeable the more turns on the secondary.  I did the experiment with a three turn secondary and posted the results in this forum.  Sredni almost calculated the voltage between nodes A and D, VAD but suddenly realized that if he did the calculation he would show that KVL works perfectly, and backed up quickly... 

You clearly have a very fuzzy recollection of those events.
I computed all the values in your silly circuits in less than 15 minutes and got the results right. Including the voltage along a diameter in the case of perfectly circular and concentric geometry.
You still cannot understand that voltage IS path dependent.

Quote
Cyriel Mabilde in Youtube did a similar experiment too with a five turn secondary and also demonstrated that KVL works perfectly.  Now I am waiting for some insight from Sredni of what would happen if we replace the resistors in the loop with capacitors, but he is MIA...

Mabilde, from the depth of his garage, is another KVLer who cannot imagine a path dependent quantity. And this is an old movie that is being rerun over and over. The KVLers propose their 'killing' experiments that should make us "Armchair Nobel prize physicists" fly away to another galaxy. Then we post the solutions according to classical ED (it's not 'our' theory, it's plain old classical electrodynamics) and you fade to silence for a while, except coming back with muddy recollections of events.
It happened with the 'two secondaries is series', it happened with the straight partial coil, it happened with the multiturn coil, it happened with that sentence by Belcher (Jesse is still touting it in his boilerplate answer on his channel and he is forced to ban users who do not agree with him to make them 'fly away to another galaxy')...


...and now we are at the ring with two capacitors. As if these capacitors could change something.

So, here, are the results for the following values of capacitors

    C1 = 4.7 uF , C2 = 22 uF
    freq = 50 Hz
    emf = 374 mV

I get - from simulation and without even invoking MEAS, just by eyeballing the plots

    VcapL = 308.5 mV,           VcapH = 65.5 mV

Guess what I measure with a true RMS multimeter?

    VcapL = 308 mV,            VcapH = 66 mV

And nothing, ok almost nothing, in the copper joining the caps.

So, what are the revolutionary results that you said were bad news for 'team Lewin'?
Title: Re: #562 – Electroboom!
Post by: Sredni on January 04, 2022, 04:36:40 pm
So you are saying - to measure voltage for given path, one shall route voltmeter leads along that path?

I am saying that when I measure a voltage I want to make sure there is not a variable magnetic flux region inside my measurement loop.

When I measure from the outside of the ring, since the dB/dt is well inside the ring, there is no dB/dt region inside my measurement loop, so if we look at it in 2D in the area enclosed by the loop voltage is not path-dependent and the value along the branch I test is ALSO the value across the branch I test which is ALSO the value across the probes and voltmeter, which is ALSO the value shown by the voltmeter.
Title: Re: #562 – Electroboom!
Post by: ogden on January 04, 2022, 04:55:46 pm
When I measure from the outside of the ring, since the dB/dt is well inside the ring, there is no dB/dt region inside my measurement loop, so if we look at it in 2D in the area enclosed by the loop voltage is not path-dependent and the value along the branch I test is ALSO the value across the branch I test which is ALSO the value across the probes and voltmeter, which is ALSO the value shown by the voltmeter.

I wonder - how dB/dt discerns between wires of resistor and voltmeter leads? You think that EM induction miraculously stops at first wire it encounters, acts only on circuit but not on voltmeter leads? Then please explain how multi-turn transformers work? Also you did not answer my 2nd question: how one can practically demonstrate path-dependency using transformer with let's say, 100 turns of secondary.
Title: Re: #562 – Electroboom!
Post by: Sredni on January 04, 2022, 05:18:09 pm
When I measure from the outside of the ring, since the dB/dt is well inside the ring, there is no dB/dt region inside my measurement loop, so if we look at it in 2D in the area enclosed by the loop voltage is not path-dependent and the value along the branch I test is ALSO the value across the branch I test which is ALSO the value across the probes and voltmeter, which is ALSO the value shown by the voltmeter.

I wonder - how dB/dt discerns between wires of resistor and voltmeter leads? You think that EMF miraculously stops at first wire it encounters, acts only on circuit but not on voltmeter leads? Then please explain how multi-turn transformers work?

It doesn't.
Maybe one day you will learn about current dividers.
Try to compute the current that will flow in your 100meg (or even 10meg) internal voltmeter resistance when you shunt your voltmeter with the branch you are measuring (900 ohm or 100 ohm in the case of Lewin's ring).

The measurement loop that does not include the dB/dt region will give you the correct voltage (EDIT: with a nanovolt order-of-magnitude load effect).
The measurement loop that include the dB/dt region will give you a measurement that is the voltage of the distance branch plus (or minus, depending on orientation) the linked EMF.

Quote
Also you did not answer my 2nd question: how one can practically demonstrate path-dependency using transformer with let's say, 100 turns of secondary.

Another killer question, I suppose.
I did it for a generic autotransformer. You probably have to look in the last 30 or so pages to find it.
When you measure from the outside you can look at the measure in two ways:

the voltmeter shows the actual voltage in the gap between taps
the voltmeter shows the voltage along the portion of filament between the taps (which is the ohmic drop that is nearly zero volts) plus (or minus depedending on orientation) the linked emf (emf of one turn times integer number of turns linked).
Title: Re: #562 – Electroboom!
Post by: jesuscf on January 04, 2022, 05:39:56 pm
You clearly have a very fuzzy recollection of those events.
I computed all the values in your silly circuits in less than 15 minutes and got the results right. Including the voltage along a diameter in the case of perfectly circular and concentric geometry.
You still cannot understand that voltage IS path dependent.

The voltage the voltmeter shows depends on the induced emf in the probes of the multimeter.  Is that what you call path dependent?   The voltage at the tips of the voltmeter probes, the ones connected to nodes A and D, VAD, is not path dependent because the ring circuit is neither changing nor moving!  You don't seem to understand that Vvoltmeter_screen != VAD. Unless you get rid of the induced voltage in the voltmeter probes or account for it somehow (using KVL of course) you'll get the wrong value for VAD.  Nevertheless, that should not prevent you for calculating the voltage VAD correctly

Mabilde, from the depth of his garage, is another KVLer who cannot imagine a path dependent quantity. And this is an old movie that is being rerun over and over. The KVLers propose their 'killing' experiments that should make us "Armchair Nobel prize physicists" fly away to another galaxy. Then we post the solutions according to classical ED (it's not 'our' theory, it's plain old classical electrodynamics) and you fade to silence for a while, except coming back with muddy recollections of events.
It happened with the 'two secondaries is series', it happened with the straight partial coil, it happened with the multiturn coil, it happened with that sentence by Belcher (Jesse is still touting it in his boilerplate answer on his channel and he is forced to ban users who do not agree with him to make them 'fly away to another galaxy')...

All of these are trivial experiments, that show that KVL works perfectly.  Also, what is wrong about doing experiments in your garage?  Both Mabilde, I, and probably you too, have equipment at home to do this experiments that is several orders of magnitude better than all the equipment the big wigs of physics had in the 19th century when doing similar tests.

...and now we are at the ring with two capacitors. As if these capacitors could change something.

So, here, are the results for the following values of capacitors

    C1 = 4.7 uF , C2 = 22 uF
    freq = 50 Hz
    emf = 374 mV

I get - from simulation and without even invoking MEAS, just by eyeballing the plots

    VcapL = 308.5 mV,           VcapH = 65.5 mV

Guess what I measure with a true RMS multimeter?

    VcapL = 308 mV,            VcapH = 66 mV

And nothing, ok almost nothing, in the copper joining the caps.

So, what are the revolutionary results that you said were bad news for 'team Lewin'?

Everything is bad news for team Lewin, because team Lewin is convinced that the induced voltage in an arbitrary length of wire is zero.  Can you show us your calculations or are you using a circuit simulator?  Did you measure VAD?  Any pictures of your setup?

Do you think there is an induced voltage in the capacitors via Faraday's law?

Title: Re: #562 – Electroboom!
Post by: Sredni on January 04, 2022, 06:47:36 pm
I don't think I have to repeat what I already wrote in the last thirty or so pages. Re-read them if you want.
You touted this 'two capacitor' example as the (yet another) definitive circuit that would jave brought 'team Lewin' to the ground.
It turned out not to be at all different from the original ring with two resistors.

In steady state resistances are exchanged with reactances. The induced field drives charges on the plates of the capacitors and instead of a conduction current we end up with a displacement current.
The setup is the same as that I used for the ring with the two resistors: a ring with two resistors around a toroidal core. There are no surprises. Everything is as it should be: a drop of 308 mV on the 4.7uF cap, negligible - basically zero - drops in the copper section, and a drop of 66 mV in the 22 uF cap.
This time I didn't attach the scope's probes, but I would have seen the same 180 degrees phase inversion.

If you want to waste time, you can compute the reactances, and the current.

I am asking you again: why did you insist this example was something special? It was a dud.

Title: Re: #562 – Electroboom!
Post by: jesuscf on January 04, 2022, 07:39:09 pm
I don't think I have to repeat what I already wrote in the last thirty or so pages. Re-read them if you want.
You touted this 'two capacitor' example as the (yet another) definitive circuit that would jave brought 'team Lewin' to the ground.
It turned out not to be at all different from the original ring with two resistors.

In steady state resistances are exchanged with reactances. The induced field drives charges on the plates of the capacitors and instead of a conduction current we end up with a displacement current.
The setup is the same as that I used for the ring with the two resistors: a ring with two resistors around a toroidal core. There are no surprises. Everything is as it should be: a drop of 308 mV on the 4.7uF cap, negligible - basically zero - drops in the copper section, and a drop of 66 mV in the 22 uF cap.
This time I didn't attach the scope's probes, but I would have seen the same 180 degrees phase inversion.

If you want to waste time, you can compute the reactances, and the current.

I am asking you again: why did you insist this example was something special? It was a dud.

Will a ring made exclusively out of capacitors show an induced emf?  No wires, only capacitors in series replacing the ring, as well as the two original resistors on each side.  What would be the voltage VAD in this case?  Do you agree that now that there are not 'Lewin zero volt' short circuits anywhere?


Title: Re: #562 – Electroboom!
Post by: Sredni on January 04, 2022, 08:09:57 pm
I am asking you again: why did you insist this example was something special? It was a dud.

Will a ring made exclusively out of capacitors show an induced emf?  No wires, only capacitors in series replacing the ring, as well as the two original resistors on each side.  What would be the voltage VAD in this case?  Do you agree that now that there are not 'Lewin zero volt' short circuits anywhere?

And here we go again.
After the nth "killer question" is answered, not a comment.
And then "killer question" n+1 takes its place.
Answer that and the cycle repeats.

What if we made a ring of alternating capacitors and thermistors?
And a ring with thermocouples orbiting a neutron star?
What about a ring made of anti-matter?
And what if we changed the gravitational constant of the universe?

You are just wasting people's time.
Title: Re: #562 – Electroboom!
Post by: jesuscf on January 04, 2022, 08:45:32 pm
I am asking you again: why did you insist this example was something special? It was a dud.

Will a ring made exclusively out of capacitors show an induced emf?  No wires, only capacitors in series replacing the ring, as well as the two original resistors on each side.  What would be the voltage VAD in this case?  Do you agree that now that there are not 'Lewin zero volt' short circuits anywhere?

And here we go again.
After the nth "killer question" is answered, not a comment.
And then "killer question" n+1 takes its place.
Answer that and the cycle repeats.

What if we made a ring of alternating capacitors and thermistors?
And a ring with thermocouples orbiting a neutron star?
What about a ring made of anti-matter?
And what if we changed the gravitational constant of the universe?

You are just wasting people's time.

Why so belligerent?  Are your servants on holidays?

Let me answer my question for you:  yes.  A ring made out of capacitors behaves as a ring made of wires or a ring made out of resistors: it picks up an induced emf.    Also yes, I did the experiment;  I connected 100 x 1uF capacitors in series to make a ring with the two resistors, 100 ohms and 900 ohms, and KVL works!  Why is this important?  Because it takes away Lewin's argument that the voltage across a arbitrary segment of wire under a varying magnetic field "shows zero volts because its zero resistance".  What if there are no wires?  What is the voltage between nodes A and D, VAD, then?  When Lewin argues that the voltage between the terminals of the opposing resistors in the ring is zero or that the induced voltage in the voltmeter probes is also zero, he is incorrect, and that is the source of this controversy.  The guy messed up and he won't admit it.  He even went so far as of trying to change the definition of KVL.  Too bad for Lewin that Maxwell himself wrote the definition of KVL in his 1873 book, which contradicts what he says.

If you think I am wasting your time, just don't read my posts or reply to them.  It is that simple.

(https://www.eevblog.com/forum/amphour/562-electroboom!/?action=dlattach;attach=1369835;image)

Title: Re: #562 – Electroboom!
Post by: bsfeechannel on January 04, 2022, 10:12:24 pm
When Lewin argues that the voltage between the terminals of the opposing resistors in the ring is zero or that the induced voltage in the voltmeter probes is also zero, he is incorrect, and that is the source of this controversy.  The guy messed up and he won't admit it.  He even went so far as of trying to change the definition of KVL.  Too bad for Lewin that Maxwell himself wrote the definition of KVL in his 1873 book, which contradicts what he says.

What you omitted from §297 is the following:
Quote
Let the components of the current at any point be u, v, w.

[...]

By Ohm's Law the current is proportional to the electromotive intensity. Hence X, Y and Z must be linear functions of u, v, w. We may therefore assume as the equations of Resistance,

X = R1u + Q3v + P2w,
Y = P3u + R2v + Q1w,
Z = Q2u + P1v + R3w.

So, the resultant EMF inside a wire no matter its multiple origins must be its current multiplied by its resistance. So much for modelling a wire as a resistance in series with a battery. This agrees perfectly with what is being said all the time: the wires and resistors in Lewin's circuit only have a voltage that is their respective resistance times the current intensity and nothing else.

So, nice try pale face. Invoking the spirit of dead Maxwell won't help you. He was no fool.

But this sums up perfectly what KVLism is: an empty rethoric full of omissions, misinformation, disinformation and sheer stupidity.

As for Lewin causing "controversy". That serves you right. Next time don't chose ignorance over knowledge.
Title: Re: #562 – Electroboom!
Post by: jesuscf on January 04, 2022, 11:30:17 pm
So, the resultant EMF inside a wire no matter its multiple origins must be its current multiplied by its resistance. So much for modelling a wire as a resistance in series with a battery. This agrees perfectly with what is being said all the time: the wires and resistors in Lewin's circuit only have a voltage that is their respective resistance times the current intensity and nothing else.

Do you mean a wire in general, or a wire with an induced emf from a varying magnetic field?  Just asking, because you seem pretty confused with what you are reading.
Title: Re: #562 – Electroboom!
Post by: ogden on January 05, 2022, 04:18:00 am
Also you did not answer my 2nd question: how one can practically demonstrate path-dependency using transformer with let's say, 100 turns of secondary.
When you measure from the outside you can look at the measure in two ways:

the voltmeter shows the actual voltage in the gap between taps
the voltmeter shows the voltage along the portion of filament between the taps (which is the ohmic drop that is nearly zero volts) plus (or minus depedending on orientation) the linked emf (emf of one turn times integer number of turns linked).

Sorry, I do not see how this demonstrates path-dependency. Let's say - I measure 10VAC on 100-turn secondary of my transformer. Please tell how to set-up 2nd AC voltmeter that would demonstrate "path-dependency", measure 0VAC while connected to same terminals as 1st voltmeter measuring 10VAC? Do you agree that 100 turns of 2nd voltmeter test leads on transformer needed?
Title: Re: #562 – Electroboom!
Post by: Siwastaja on January 06, 2022, 10:15:54 am
This has been one heck of a ride.

Really the first thing I need to get off my chest is, I really appreciate the work by bsfeechannel and Sredni, and later thinkfat. Thank you for not giving up, and I also need to apologize. I don't remember what I exactly posted on the previous round (is it already two years or what?) but it was about bsfeechannel's writing style and completely uncalled for. Seeing how much pure stupidity you have to deal with, I admire the coolheadness you can still maintain.

Thank you for revisiting this again, because the fact that time has passed helps to see it in new light. It's also amusing to see how many have "converted". For the rest, quality is indeed going down.

Additionally to thinkfat doing full 180, my real eye openers were actually helpful comments by jesuscf and Jesse.

These guys repeatedly called all the math used gibberish, confusing, etc. etc., and it really hit home because I agree with them. Math as shown by Sredni is pretty much gibberish to me, as well. But there is one big difference; while hate admitting being wrong as well, I hate intellectual dishonesty even more. I simply can't go there.

Which gets us into the matter of credentials as discussed repeatedly. Personally, still in high school I really shone in math. Those integrals were a breeze. Coming to university however, the first few courses were still with full scores, but then my scores started dropping. It seems, while integrals as taught in high school were easy, now when multiple integral symbols with those small extra symbols like circles started to appear, number of different alphabets and writing styles to signify different entities blew through the roof, it become harder. I was able to pass exams still but had fundamental issues digesting it.

It didn't help that my curriculum was kind of special mix, originally designed to mass-produce design engineers for still growing Nokia (which later colossally failed), but I'm not sure if that mix worked that well. Only one course in very fundamental electromagnetism (dealing with these subjects) so while we had all the related math, it was disconnected. I never learned the Maxwell equations well. Then we mostly had signal processing, some computer science, some electronics, etc.

Finally, I failed the exam in vector analysis. This was the first time me literally failing math. In the end, I admitted the fact that I'm just not a math wizard, and I don't need to be. Different people have different skills. I know enough math to know what I need to know, so that I can ask for help from the right people if the task requires it. This also makes me appreciate those who can deal with complex math. Their skills and understanding in electromagnetics have enabled all the nice modern things we have, so that more simple-minded engineers like myself can just buy a lumped module which deals with all the voodoo internally.

And now we get back to the eye opening moments. The bullshit generated by J & J in this thread could work for some readers, say, a carpenter or plumber with no university math background.

But for me, it doesn't work. Sredni's math might look gibberish-y enough for me so that I understand how J & J feel about it, but on the other hand, I still remember enough of the Vector Analysis and Electromagnetic fields and waves that I can see chances are very high Sredni knows exactly what he's talking about. Even if that is unsure, it's certainly sure that J & J have absolutely no idea about the math involved whatsoever, even if they are capable of copy-pasting some terms and putting them together to form sentences cargo cult way - think about "lumpable paths" & co.

So, finaly I asked myself what is the real dispute here, behind the endless loop of fog screens.

It is the question:

Is Lewin's original circuit lumpable?

Because non-lumpability is hard to prove (like nonexistence of God), requiring mathmematical concepts not everybody seemingly understand or agree about, the burden of proof has to be turned around: prove that KVL works.

And I think the only sustainable, intellectual honest way of debating about this would be this very process:
* Show an equivalent circuit -> show that the real circuit behaves like that equivalent circuit
* Now disputing this is easy; just show how the given equivalent circuit does not work. Single datapoint suffices.
* Now the author is required to come up with a new equivalent circuit, until it works - or doesn't.

And this is what I have not seen. I'm serious. I have not seen a circuit diagram, showing an equivalent circuit, and Kirchoff based calculations that get the result matching with the experiment.

Instead, I have seen videos of special 3D arrangements made for the task.

So I think I know where the issue lies.

It's the fact that a 2D circuit drawing with nodes, KVL applied, is not sufficient alone. To get it working, extra "hidden" information is added; not necessarily even hidden, not everyone is dishonest with this, but nevertheless it seems the exact 3D construction is the crucial part. This cannot be conveyed with simple circuit mesh diagram, otherwise it would have been done already.

So the model is not a equivalent circuit that can be drawn as a diagram; no, the model is a photograph or usually even a video showing a careful 3D construction. Only within this model, the KVL seems to hold.

As soon as you change this model - while the 2D circuit diagram keeps the same! - the model breaks up. This is then called "bad probing".

But Kirchoff laws and simple 2D circuit diagrams with lumped components were never supposed to cover such complex cases. Sure, you can force this approach, by adding new layers of information (like, add a photograph describing the exact layout required, to the point of showing correct ways of probing), but then the question is, is this way of modeling beneficial? Or maybe going to the lower physical level with wider generalizations, using known, true and tested principles - for example, Maxwell equations on paper, or modern EM field solvers/simulators - would make more sense?

In any case, it's quite a mystery how this discussion actually started, and though I originally contributed this to Lewin's "flashy" way, I now think it's more because of how Mehdi represented it. After all, many have seen the Lewin's lecture, and it did mean nothing until Mehdi "set up the stage" so to speak, introducing the concept of "being wrong", which is actually quite ridiculous if you think about it deeper.

Lewin's lecture isn't that special; it's completely normal to see lecturers show examples "how not to do something". It's quite a stretch to think this shows that "the lecturer doesn't know how to do it". No shit Sherlock, that's the whole point. I could understand this from someone who has never attended university lectures, but it's surprising to see from Mehdi.

Really, the eye opener should be the fact that "correct probing" requires carefully thought out geometric constructions. This is not what I mean when I, as a practical engineer, talk about correct probing. For me, correct probing means avoiding extra loop caused by the scope's ground clip, instead using the small springy thing to connect to the terminals of the small output capacitor, for example, directly. But this is only possible if the thing to be measured is a small physical point (like a 0805 capacitor). In other words: a lumped circuit, or a close approximation of it! If you are measuring between wide area of circuit, under influence of external field, then obviously the presence of leads cannot be avoided in any way. They become parts of the circuit. The definition that "correct probing" is the one that happens to give consistent results per equivalent circuit + KVL is of course backwards. We should use capable enough models so they can model the real world, not the other way around (constraint the real world until it matches with the simple model), because latter limits out capability of building useful circuits. We can do better.
Title: Re: #562 – Electroboom!
Post by: jesuscf on January 06, 2022, 03:55:14 pm
This has been one heck of a ride.

Really the first thing I need to get off my chest is, I really appreciate the work by bsfeechannel and Sredni, and later thinkfat. Thank you for not giving up, and I also need to apologize. I don't remember what I exactly posted on the previous round (is it already two years or what?) but it was about bsfeechannel's writing style and completely uncalled for. Seeing how much pure stupidity you have to deal with, I admire the coolheadness you can still maintain.

Thank you for revisiting this again, because the fact that time has passed helps to see it in new light. It's also amusing to see how many have "converted". For the rest, quality is indeed going down.

Additionally to thinkfat doing full 180, my real eye openers were actually helpful comments by jesuscf and Jesse.

These guys repeatedly called all the math used gibberish, confusing, etc. etc., and it really hit home because I agree with them. Math as shown by Sredni is pretty much gibberish to me, as well. But there is one big difference; while hate admitting being wrong as well, I hate intellectual dishonesty even more. I simply can't go there.

Which gets us into the matter of credentials as discussed repeatedly. Personally, still in high school I really shone in math. Those integrals were a breeze. Coming to university however, the first few courses were still with full scores, but then my scores started dropping. It seems, while integrals as taught in high school were easy, now when multiple integral symbols with those small extra symbols like circles started to appear, number of different alphabets and writing styles to signify different entities blew through the roof, it become harder. I was able to pass exams still but had fundamental issues digesting it.

It didn't help that my curriculum was kind of special mix, originally designed to mass-produce design engineers for still growing Nokia (which later colossally failed), but I'm not sure if that mix worked that well. Only one course in very fundamental electromagnetism (dealing with these subjects) so while we had all the related math, it was disconnected. I never learned the Maxwell equations well. Then we mostly had signal processing, some computer science, some electronics, etc.

Finally, I failed the exam in vector analysis. This was the first time me literally failing math. In the end, I admitted the fact that I'm just not a math wizard, and I don't need to be. Different people have different skills. I know enough math to know what I need to know, so that I can ask for help from the right people if the task requires it. This also makes me appreciate those who can deal with complex math. Their skills and understanding in electromagnetics have enabled all the nice modern things we have, so that more simple-minded engineers like myself can just buy a lumped module which deals with all the voodoo internally.

And now we get back to the eye opening moments. The bullshit generated by J & J in this thread could work for some readers, say, a carpenter or plumber with no university math background.

But for me, it doesn't work. Sredni's math might look gibberish-y enough for me so that I understand how J & J feel about it, but on the other hand, I still remember enough of the Vector Analysis and Electromagnetic fields and waves that I can see chances are very high Sredni knows exactly what he's talking about. Even if that is unsure, it's certainly sure that J & J have absolutely no idea about the math involved whatsoever, even if they are capable of copy-pasting some terms and putting them together to form sentences cargo cult way - think about "lumpable paths" & co.

So, finaly I asked myself what is the real dispute here, behind the endless loop of fog screens.

It is the question:

Is Lewin's original circuit lumpable?

Because non-lumpability is hard to prove (like nonexistence of God), requiring mathmematical concepts not everybody seemingly understand or agree about, the burden of proof has to be turned around: prove that KVL works.

And I think the only sustainable, intellectual honest way of debating about this would be this very process:
* Show an equivalent circuit -> show that the real circuit behaves like that equivalent circuit
* Now disputing this is easy; just show how the given equivalent circuit does not work. Single datapoint suffices.
* Now the author is required to come up with a new equivalent circuit, until it works - or doesn't.

And this is what I have not seen. I'm serious. I have not seen a circuit diagram, showing an equivalent circuit, and Kirchoff based calculations that get the result matching with the experiment.

Instead, I have seen videos of special 3D arrangements made for the task.

So I think I know where the issue lies.

It's the fact that a 2D circuit drawing with nodes, KVL applied, is not sufficient alone. To get it working, extra "hidden" information is added; not necessarily even hidden, not everyone is dishonest with this, but nevertheless it seems the exact 3D construction is the crucial part. This cannot be conveyed with simple circuit mesh diagram, otherwise it would have been done already.

So the model is not a equivalent circuit that can be drawn as a diagram; no, the model is a photograph or usually even a video showing a careful 3D construction. Only within this model, the KVL seems to hold.

As soon as you change this model - while the 2D circuit diagram keeps the same! - the model breaks up. This is then called "bad probing".

But Kirchoff laws and simple 2D circuit diagrams with lumped components were never supposed to cover such complex cases. Sure, you can force this approach, by adding new layers of information (like, add a photograph describing the exact layout required, to the point of showing correct ways of probing), but then the question is, is this way of modeling beneficial? Or maybe going to the lower physical level with wider generalizations, using known, true and tested principles - for example, Maxwell equations on paper, or modern EM field solvers/simulators - would make more sense?

In any case, it's quite a mystery how this discussion actually started, and though I originally contributed this to Lewin's "flashy" way, I now think it's more because of how Mehdi represented it. After all, many have seen the Lewin's lecture, and it did mean nothing until Mehdi "set up the stage" so to speak, introducing the concept of "being wrong", which is actually quite ridiculous if you think about it deeper.

Lewin's lecture isn't that special; it's completely normal to see lecturers show examples "how not to do something". It's quite a stretch to think this shows that "the lecturer doesn't know how to do it". No shit Sherlock, that's the whole point. I could understand this from someone who has never attended university lectures, but it's surprising to see from Mehdi.

Really, the eye opener should be the fact that "correct probing" requires carefully thought out geometric constructions. This is not what I mean when I, as a practical engineer, talk about correct probing. For me, correct probing means avoiding extra loop caused by the scope's ground clip, instead using the small springy thing to connect to the terminals of the small output capacitor, for example, directly. But this is only possible if the thing to be measured is a small physical point (like a 0805 capacitor). In other words: a lumped circuit, or a close approximation of it! If you are measuring between wide area of circuit, under influence of external field, then obviously the presence of leads cannot be avoided in any way. They become parts of the circuit. The definition that "correct probing" is the one that happens to give consistent results per equivalent circuit + KVL is of course backwards. We should use capable enough models so they can model the real world, not the other way around (constraint the real world until it matches with the simple model), because latter limits out capability of building useful circuits. We can do better.


Summarizing a bit:

1) This extremely simple circuit is perfectly lumpable.  There is nothing to it really.

2) This circuit has been solved correctly in electromagnetics textbooks by experts in the field.

3) Both Jesse and I have done experiments that demonstrate that KVL and measurements results match perfectly.  Just need to be carefull when measuring.

4) Many people have debunked Lewin's claims, including Mehdi, RDS accademy, and Mabilde.  Neither Jesse nor I are the first ones.



If you don't know how to use KVL, it doesn't mean that KVL doesn't work!

 




Title: Re: #562 – Electroboom!
Post by: Siwastaja on January 06, 2022, 04:58:33 pm
If you don't know how to use KVL, it doesn't mean that KVL doesn't work!

This is a pretty good conclusion, actually.

Any tool can be used to solve any arbitrarily generated problem, to prove anything, as evidenced by all those weird videos showing a specific way of doing a thing which has no practical application (and which is also not the same experiment as Lewin's, so why do it to prove anything about Lewin's experiment is beyond me :-//).

Look at it this way: Lewin has shown an experiment where clearly the measured outcome doesn't match with KVL. This is undisputed. You guys are spending great effort in building significantly different experiments (to the point they look something completely else even to a plumber's or carpenter's eye) that shows that in these experiments, KVL holds. Yet when suggested to add small modifications to the experiments, you won't do it.

What's the point? Like, world is literally full of circuits where KVL works, no one says it isn't a useful tool that works in most everyday circuits. It's the workhorse of basic calculations in the whole electronics industry. What's the deal? My current interpretation is, it's all because Mehdi "set up the stage" to make it like an attack against some good thing which is KVL. The rest just follows. This is ridiculous. Look, it's originally a university lecture. It's not aimed at practical engineers. The target audience are people who can deal with such stuff, analytically, using math, and don't take an offense on behalf of their favorite tool.

So practical engineers like myself continue using KVL every day. It is a really suitable tool for so many circuits we engineers design. Actually, I have never had to resort to either expensive EM simulation packages, or hand-calculating with Faraday's law. I'm pretty sure latter is very rarely done in modern days circuit design. For me, assuming lumped components, and radio stuff only inside radio modules which I keep within the suggested physical separation away from the rest of the circuit, using online transmission line impedance calculators and Spice simulations has been enough.

Again, I don't have to master everything, that's why we have teams of people.

But this all gives the false impression that the underlying physical laws are not important. No, there are and always will be difficult cases where deeper understanding, and better tools than Kirchoff laws are required. You or me might never encounter them. But Lewin's original experiment sure is one, that's undisputed - the result does not match the calculation. Now you can claim that the experiment is wrong ("wrong probing" is part of the experiment), and for some sake of "KVL justice" it needs to be modified using "good probing"; but sorry, that's only an opinion of yours. In my opinion, no experiment is wrong, if the aim is to gain understanding. Here, it helps gain understanding that the actual physical definition of voltage is pretty weird to us practical engineers!

It's actually quite similar to the "Arduino hobbyist fallacy" when Arduino was blog-credible over a decade ago. I was constantly lectured at that my skills of designing circuits are superfluous because you can always buy an Arduino shield which does that. Well, that is seemingly true to many, but someone has to design the products and circuits that do not exist at yet. Someone has to design those Arduino shields.

Similarly, someone has to design those RF modules, those 5G base stations, GPS satellites, and so on. These will always contain subsections where KVL does not work.

If you work there, you can't say "hey, I modified that circuit to look - and perform - physically different to be able to use KVL, because KVL is good if you do that". No, KVL has no such value. It has only value as a tool.

Then again, if my circuits end up being inside strong varying magnetic fields, I can just choose to call that "interference", and try to follow the rules-of-thumb of EMC design. Again, if the field is strong enough to matter even after that, I can call "foul play", clearly there is a radiating source that exceeds the EMC regulations. Again, Faraday avoided :phew:.

And a carpenter doesn't need to know shit about metallurgy, yet is still able to hammer in nails. But someone's got to do it.
Title: Re: #562 – Electroboom!
Post by: jesuscf on January 06, 2022, 05:10:19 pm
If you don't know how to use KVL, it doesn't mean that KVL doesn't work!
Look at it this way: Lewin has shown an experiment where clearly the measured outcome doesn't match with KVL. This is undisputed. You guys are spending great effort in building significantly different experiments (to the point they look something completely else even to a plumber's or carpenter's eye) that shows that in these experiments, KVL holds. Yet when suggested to add small modifications to the experiments, you won't do it.

Except that the measured outcome of Lewin's experiment DOES match KVL calculations!  Just draw the correct equivalent circuit, solve, and presto: the KVL solution matches experimental measurements!!!  Maybe I am pointing out the obvious, but if you don't solve the correct circuit you'll get an incorrect answer.
Title: Re: #562 – Electroboom!
Post by: Siwastaja on January 06, 2022, 05:45:28 pm
Except that the measured outcome of Lewin's experiment DOES match KVL calculations!  Just draw the correct equivalent circuit, solve, and presto: the KVL solution matches experimental measurements!!!  Maybe I am pointing out the obvious, but if you don't solve the correct circuit you'll get an incorrect answer.

Maybe the issue is that you take this too seriously? Like, fit the KVL no matter what. When the actual goal was to learn beyond KVL. If you just look past the "dispute" which started after Mehdi's video, and look why the university course exists in the first place...

I mean, the whole idea of that university course is to teach underlying laws of electromagnetism. They need an example where the existing knowledge (how equivalent circuit is formed) is not a suitable tool for the job anyway, in order to teach the more general physical laws.

I'm sure you'll (or maybe not you, but someone) be able to make KVL seemingly work in organizing the equivalent circuit just right. Why wouldn't you - your schematic can, after all, contain infinite number of circuit elements, so you can make it perform any circuit function, including what Lewin's voltmeters showed. The question is, now your equivalent circuit encodes information about the external field (what if the field looks like Bart Simpson's face?), and about the placement of measurement probes, to get the correct result. I'm still no mathematician, but if this was the preferred way of calculation, it would be used. Instead, at some point, when the limits of equivalent circuits and Kirchoff laws are getting closer and closer, transition to more advanced (more fundamental) methods are fruitful, finally to save work.

But if you only have a hammer...

I think this is, in essence, what that MacDonald guy was saying in the famous excerpt quoted in this thread a few times.
Title: Re: #562 – Electroboom!
Post by: HuronKing on January 06, 2022, 06:09:28 pm
@Siwastaja

I am incredibly impressed with your post. You're absolutely correct that while these ideas are 'fundamental' they are not necessarily simple. I also had a similar experience to yours with vector calculus and such. I had a tough time in my 'pure' mathematics classes learning vector calculus. It actually didn't 'click' for me until I took Applied EM from an incredibly good instructor who was an expert in RF engineering and waveguide antennas. He showed me the way how these vectors and integrals can make physical sense in nature.

2) This circuit has been solved correctly in electromagnetics textbooks by experts in the field.

Hmm...
Quote
A consequence of Faraday’s law of induction is that Kirchhoff’s loop rule (which states that [integral]E · ds = 0 around a closed path) is no longer valid in situations where there is a changing magnetic field. Faraday has taken us beyond the comfortable realm of conservative electric fields. The voltage difference between two points now depends on the path between them. Problem 7.4 provides an instructive example of this fact.

Purcell & Morin, Electricity and Magnetism, 3rd Edition, Chapter 7.6


Quote
3) Both Jesse and I have done experiments that demonstrate that KVL and measurements results match perfectly.  Just need to be carefull when measuring.

So induced voltage is path dependent.

Quote
4) Many people have debunked Lewin's claims, including Mehdi, RDS accademy, and Mabilde.  Neither Jesse nor I are the first ones.

And one, or all, of these debunkers has said that KVL holds 'in all cases.' That's wrong.

Title: Re: #562 – Electroboom!
Post by: jesuscf on January 06, 2022, 06:35:53 pm
Quote
A consequence of Faraday’s law of induction is that Kirchhoff’s loop rule (which states that [integral]E · ds = 0 around a closed path) is no longer valid in situations where there is a changing magnetic field. Faraday has taken us beyond the comfortable realm of conservative electric fields. The voltage difference between two points now depends on the path between them. Problem 7.4 provides an instructive example of this fact.

Purcell & Morin, Electricity and Magnetism, 3rd Edition, Chapter 7.6

Check example 6.6 from Electromagnetics by Notaros.  That is how Lewin's circuit should have been solved.  Pay attention to Figure 6.10b where the equivalent circuit to be solved is correctly presented.

EDIT: " [integral]E · ds = 0 around a closed path" is not the general definition of KVL.  KVL must include all EMFs, included magnetically induced EMFs.  Says who you may ask?  Maxwell himself!
Title: Re: #562 – Electroboom!
Post by: Sredni on January 07, 2022, 12:15:54 pm
Check example 6.6 from Electromagnetics by Notaros.  That is how Lewin's circuit should have been solved.  Pay attention to Figure 6.10b where the equivalent circuit to be solved is correctly presented.

EDIT: " [integral]E · ds = 0 around a closed path" is not the general definition of KVL.  KVL must include all EMFs, included magnetically induced EMFs.  Says who you may ask?  Maxwell himself!


Soooo...  the EMF is located on top of the resistors, it seems. Half just above R1, and half just above R2. How many centimeters, exactly? The text does not say. Can you locate with a bit more accuracy? No?
Or maybe...
Maybe that's the "equivalent circuit" that allows you to "solve the problem from the circuit theory point of view" and that is one of the introductory textbooks that do not explain clearly to their easily distracted audience what they intend for V. Oh, wait, but it does explain what V is! Page 269, eq. 6.18

Eq = - grad V

(Eq is what I call Ecoul) and V is... the electric scalar potential. Only half of the potentials required to describe the total electric field. And the text also says so explicitly on page 277, formula 6.43

E(t) = - dA/dt - grad V

"We see that both potentials are needed for E..."
(the same expression I used to express Etot = Eind + Ecoul, even if recently I decided to call the scalar electric potential phi, instead of V - exactly to avoid this kind of confusion you are having)

So...

where is exactly the EMF, again?

(Lewin problem is solved as an exercise on Purcell, Morin: Berkeley Physics vol 2, Electricity and Magnetism 3rd edition)

(Yes, I have checked with the kitchen. They do have deja-vus)
Title: Re: #562 – Electroboom!
Post by: jesuscf on January 07, 2022, 03:37:57 pm
Check example 6.6 from Electromagnetics by Notaros.  That is how Lewin's circuit should have been solved.  Pay attention to Figure 6.10b where the equivalent circuit to be solved is correctly presented.

EDIT: " [integral]E · ds = 0 around a closed path" is not the general definition of KVL.  KVL must include all EMFs, included magnetically induced EMFs.  Says who you may ask?  Maxwell himself!


Soooo...  the EMF is located on top of the resistors, it seems. Half just above R1, and half just above R2. How many centimeters, exactly? The text does not say. Can you locate with a bit more accuracy? No?
Or maybe...
Maybe that's the "equivalent circuit" that allows you to "solve the problem from the circuit theory point of view" and that is one of the introductory textbooks that do not explain clearly to their easily distracted audience what they intend for V. Oh, wait, but it does explain what V is! Page 269, eq. 6.18

Eq = - grad V

(Eq is what I call Ecoul) and V is... the electric scalar potential. Only half of the potentials required to describe the total electric field. And the text also says so explicitly on page 277, formula 6.43

E(t) = - dA/dt - grad V

"We see that both potentials are needed for E..."
(the same expression I used to express Etot = Eind + Ecoul, even if recently I decided to call the scalar electric potential phi, instead of V - exactly to avoid this kind of confusion you are having)

So...

where is exactly the EMF, again?

(Lewin problem is solved as an exercise on Purcell, Morin: Berkeley Physics vol 2, Electricity and Magnetism 3rd edition)

(Yes, I have checked with the kitchen. They do have deja-vus)

What confusion are you talking about?  Every single circuit I tried matches the KVL solution.  You seem to be the one that is very confused.  Here, once again: the induced EMF in Lewin's circuit and the circuit of example 6.6 from Electromagnetics by Notaros is distributed uniformly across the whole circuit, on every component and piece of wire connecting them.  The correct way of lumping the induced EMF in order to solve for the voltage VAD is shown by Notaros.   Lewin on the other hand, didn't represent the induced EMF in his equivalent circuit at all!  No wonder he thinks KVL doesn't work.
 
Title: Re: #562 – Electroboom!
Post by: ogden on January 07, 2022, 03:48:45 pm
Also you did not answer my 2nd question: how one can practically demonstrate path-dependency using transformer with let's say, 100 turns of secondary.
When you measure from the outside you can look at the measure in two ways:

the voltmeter shows the actual voltage in the gap between taps
the voltmeter shows the voltage along the portion of filament between the taps (which is the ohmic drop that is nearly zero volts) plus (or minus depedending on orientation) the linked emf (emf of one turn times integer number of turns linked).

Sorry, I do not see how this demonstrates path-dependency. Let's say - I measure 10VAC on 100-turn secondary of my transformer. Please tell how to set-up 2nd AC voltmeter that would demonstrate "path-dependency", measure 0VAC while connected to same terminals as 1st voltmeter measuring 10VAC? Do you agree that 100 turns of 2nd voltmeter test leads on transformer needed?

Lewin's team, please come down to the Earth and answer this simple question - about how to demonstrate path-dependency on real transformer with 100-turn secondary using two AC voltmeters one showing nominal AC volts, another 0V. Many will be pleased to discover something they do not know yet. [disclaimer] I am usually not that persistent, but I feel like Killer Question (© Sredni) has been asked.
Title: Re: #562 – Electroboom!
Post by: Sredni on January 07, 2022, 06:39:47 pm
Yes, the current question is always the killer question.
Until it gets answered. They it becomes the forgotten question.
And killer question n+1 makes it appearance.
Title: Re: #562 – Electroboom!
Post by: Siwastaja on January 07, 2022, 06:52:09 pm
Lewin's team, please come down to the Earth and answer this simple question - about how to demonstrate path-dependency on real transformer with 100-turn secondary using two AC voltmeters one showing nominal AC volts, another 0V. Many will be pleased to discover something they do not know yet. [disclaimer] I am usually not that persistent, but I feel like Killer Question (© Sredni) has been asked.

It didn't occur to you that university stuff is not supposed to be down to Earth. It's not supposed to be dumbed down and simplified. This is about the theoretical basis, which has to be understood when designing RF layouts, or when designing EM field solver simulators. Someone have to do those, even if you only buy a transformer or pre-certified RF module and use them within the datasheet conditions.

And indeed, designing an actual RF communication circuit would be a perfect example where Kirchoff laws do not apply. This is obvious to anyone seeing an antenna, a wire that goes nowhere, yet radiates energy out. Still, I'm sure if you try enough, you'll be able to somehow make use of Kirchoff laws by modeling the transmitter and receiver pair as a schematic. After all, you can use infinite number of circuit elements. But is it the right way?

If I understood your question right, one of the many possible answers to your question would be, by piercing through the possible packaging and looping the probe wire 100 turns around the core. This is completely unfruitful. No one uses Maxwell equations in everyday engineering with everyday off-the-shelf transformer, the whole idea of that component is that it performs as a lumped circuit, a black box.

This was never anyone's intention. You have been only fighting against made-up strawman for years. Come on! Let the academics do the academic discussion their way; the track record is excellent. We "practical engineers" should learn something about it, even if we don't go fully there. Forget the ego for a while.
Title: Re: #562 – Electroboom!
Post by: ogden on January 07, 2022, 08:27:22 pm
Yes, the current question is always the killer question.
Until it gets answered. They it becomes the forgotten question.
And killer question n+1 makes it appearance.

Lewin's team, please come down to the Earth and answer this simple question - about how to demonstrate path-dependency on real transformer with 100-turn secondary using two AC voltmeters one showing nominal AC volts, another 0V. Many will be pleased to discover something they do not know yet. [disclaimer] I am usually not that persistent, but I feel like Killer Question (© Sredni) has been asked.

It didn't occur to you that university stuff is not supposed to be down to Earth. It's not supposed to be dumbed down and simplified. This is about the theoretical basis, which has to be understood when designing RF layouts, or when designing EM field solver simulators. Someone have to do those, even if you only buy a transformer or pre-certified RF module and use them within the datasheet conditions.

And indeed, designing an actual RF communication circuit would be a perfect example where Kirchoff laws do not apply. This is obvious to anyone seeing an antenna, a wire that goes nowhere, yet radiates energy out. Still, I'm sure if you try enough, you'll be able to somehow make use of Kirchoff laws by modeling the transmitter and receiver pair as a schematic. After all, you can use infinite number of circuit elements. But is it the right way?

If I understood your question right, one of the many possible answers to your question would be, by piercing through the possible packaging and looping the probe wire 100 turns around the core. This is completely unfruitful. No one uses Maxwell equations in everyday engineering with everyday off-the-shelf transformer, the whole idea of that component is that it performs as a lumped circuit, a black box.

This was never anyone's intention. You have been only fighting against made-up strawman for years. Come on! Let the academics do the academic discussion their way; the track record is excellent. We "practical engineers" should learn something about it, even if we don't go fully there. Forget the ego for a while.

You both - please, please tell exact, easily repeatable steps of experiment KVL'ers shall do - to test path-dependency themselves. All you can come-up until now is just  :blah: :blah: :blah: :blah: :blah: :blah: :blah: :blah: :blah: :blah: :blah:

[edit] Sample (toroid) transformer picture attached. Assume it is 12 ACV transformer so KVL'ers can get it easily if interested.
Title: Re: #562 – Electroboom!
Post by: thinkfat on January 07, 2022, 08:31:35 pm
Thing is, if you had some basic understanding of the matter, you'd know that your question is irrelevant to the matter.
Title: Re: #562 – Electroboom!
Post by: ogden on January 07, 2022, 08:41:06 pm
Thing is, if you had some basic understanding of the matter, you'd know that your question is irrelevant to the matter.

Really? - Most pathetic excuse of no explanation ever seen. If you are not able to explain your "science" to your grandmother - you do not know it well enough.
Title: Re: #562 – Electroboom!
Post by: thinkfat on January 07, 2022, 08:44:10 pm
Thing is, if you had some basic understanding of the matter, you'd know that your question is irrelevant to the matter.

Really? - Most pathetic excuse of no explanation ever seen. If you are not able to explain your "science" to your grandmother - you do not know it well enough.

I'm not spoonfeeding you. The answers you're looking for are all in this thread. There's no value in repeating them over and over.
Title: Re: #562 – Electroboom!
Post by: ogden on January 07, 2022, 08:50:02 pm
Thing is, if you had some basic understanding of the matter, you'd know that your question is irrelevant to the matter.

Really? - Most pathetic excuse of no explanation ever seen. If you are not able to explain your "science" to your grandmother - you do not know it well enough.

I'm not spoonfeeding you. The answers you're looking for are all in this thread. There's no value in repeating them over and over.

I am asking SPECIFIC question about transformer with secondary winding of 100 turns which I am sure is not answered in this thread. Do not play stupid games please.
Title: Re: #562 – Electroboom!
Post by: Siwastaja on January 08, 2022, 09:54:32 am
I already answered to that specific question - by looping the measurement wire through the core -, but it doesn't prevent it being re-asked repeatedly, just like before.

These people are indeed like broken records. Or maybe you want to hear the answer from Sredni or bsfeechannel? Maybe I'm not qualified?

In any case, the question is irrelevant, even if answered. A smoke screen. Playing games.

This is incredibly sad. I may have overestimated the EEVBlog forum community quality. On the other hand, the well known excellent posters on this forum avoid this topic like plague, and I well understand why.

Bulk of the appeal of this forum is completely built upon the contribution of ~10-20 excellent contributors (none of whom have posted regarding this matter, because, I guess, they are just facepalming).

Now I have realized how mediocre "practical engineers" can be totally manipulated into participating such needless and ridiculous war against science and education. Incredibly sad. (To be fair, I count myself in those "mediocre practical engineers". We can get a lot done. We may be more productive in building actual physical products that we forget that when scientists, physicists, mathematicians talk, we should at least listen and hone our basic scientific skills. Most of us are lacking there even if we managed to sit through university introductory lectures. That's a fact.)

So get off the high horse and read, read, and reread until you understand. I'm not the only one who did full 180 and needed to apologize. It's not easy.

The key difference in Lewin's excercise and ogden's transformer circuit is that the former circuit is completely placed within the varying field, making it impossible to route the probe wires outside of the field. The transformer? The field is contained within the core, and you can conveniently measure from the taps that are bought outside of the field.
Title: Re: #562 – Electroboom!
Post by: ogden on January 08, 2022, 11:08:29 am
I already answered to that specific question - by looping the measurement wire through the core -, but it doesn't prevent it being re-asked repeatedly, just like before.
These people are indeed like broken records. Or maybe you want to hear the answer from Sredni or bsfeechannel? Maybe I'm not qualified?

Yes I was expecting to continue discussion with Sredni or bsfeechannel, not you. As soon as Lewin's followers agree that path-dependency can be proven using multiple windings of measurement wire which are essentially another secondary winding on transformer, I say that same exact phenomena is true in Romer's/Lewin's experiment and there is no such thing as path-dependency. It is just EMF induced in two wire loops! One can connect them such a way that EMF's cancels out - there will be no current flowing, thus 0V reading on voltmeter.

Quote
The key difference in Lewin's excercise and ogden's transformer circuit is that the former circuit is completely placed within the varying field, making it impossible to route the probe wires outside of the field.

It was said already - Maxwell-Faraday equation does not care about size/diameter/area of wire loop. Hopefully you agree. It was proven as well. (https://www.eevblog.com/forum/amphour/562-electroboom!/msg3913808/#msg3913808) Probe wires in Lewin's experiment are routed *around* varying field, same way as circuit wires. It is fallacy to think that circuit receives EMF in Lewin's experiment, but probe wires miraculously avoid Faraday's Law of Induction. Fact that so many fail to understand such a simple logic is mindboggling. No wonder that most eevblog contributors avoid those threads because they are busy of laughing loud and facepalming.
Title: Re: #562 – Electroboom!
Post by: thinkfat on January 08, 2022, 02:18:39 pm
The key difference in Lewin's excercise and ogden's transformer circuit is that the former circuit is completely placed within the varying field, making it impossible to route the probe wires outside of the field. The transformer? The field is contained within the core, and you can conveniently measure from the taps that are bought outside of the field.

Actually, no. The probe wires are all outside of the magnetic flux region in Dr. Lewins experiment.
Title: Re: #562 – Electroboom!
Post by: Siwastaja on January 08, 2022, 02:24:44 pm
So many diversions have been discussed that I clearly need to watch the original experiment again, and maybe that would be a good thing to do for everyone else as well, seeing that all questions seem to be about different experiments and different circuits.
Title: Re: #562 – Electroboom!
Post by: bsfeechannel on January 08, 2022, 03:41:34 pm
As soon as Lewin's followers agree

I learned electromagnetism in the 1980's long before Lewin's lectures and articles about the subject. We are not here to defend Lewin. We are here to denounce the stupid assertions and outright lies KVLiars make.

Quote
that path-directivity can be proven

"Path-directivity", he said.

Man, come on. Stop it. You're not fooling anyone. You don't know the theory, nor the practice, and not even the terminology, because you don't know jack squat what it means.

We've been trying to save you from this embarrassment for quite a few years now. Just don't do it anymore. OK?
Title: Re: #562 – Electroboom!
Post by: bsfeechannel on January 08, 2022, 04:04:42 pm
Summarizing a bit:

4) Many people have debunked Lewin's claims, including Mehdi, RDS accademy, and Mabilde.  Neither Jesse nor I are the first ones.

What you didn't say is that Mehdi, RDS, Mabilde and Jesse debunk each other. Mehdi said Lewin measured two different voltages across the same two points of the circuit due to bad probing. Mabilde said it was not bad probing, contradicting Mehdi, but it is because there's a "hidden" EMF in the probes. The RDS guy said the problem is otherwise: the ohm's law, according to him, predicts the presence of a voltage across a static wire that's not the product of its current times its resistance. Contradicting Mehdi and Mabilde. Jesse, the poor bugger, doesn't even have a theory to contradict the others, so he doesn't know what is really at stake, nor what to look for, nor how to interpret the results of his "experiment".

In common, all of the above showed without any doubt that, yes, voltages around a circuit immersed in a varying magnetic field are path-dependent. Jesse's setup even shows that dynamically. He changes the position, i.e. the path, of the hands of his clock and the meter displays a different voltage every time.

Quote
If you don't know how to use KVL, it doesn't mean that KVL doesn't work!

No one said KVL doesn't work. Sometimes it works. Sometimes it doesn't. It works when there's no varying magnetic field inside the area delimited by the boundaries of the circuit, and doesn't work otherwise.
Title: Re: #562 – Electroboom!
Post by: bsfeechannel on January 08, 2022, 04:39:56 pm
This is incredibly sad. I may have overestimated the EEVBlog forum community quality.

[...]

Now I have realized how mediocre "practical engineers" can be totally manipulated into participating such needless and ridiculous war against science and education. Incredibly sad.

[...]

So get off the high horse and read, read, and reread until you understand. I'm not the only one who did full 180 and needed to apologize. It's not easy.

It's a joy when we finally get it. Things fall into place. But then it's sad, exactly for the same reason.
Title: Re: #562 – Electroboom!
Post by: ogden on January 08, 2022, 04:44:47 pm
The key difference in Lewin's excercise and ogden's transformer circuit is that the former circuit is completely placed within the varying field, making it impossible to route the probe wires outside of the field. The transformer? The field is contained within the core, and you can conveniently measure from the taps that are bought outside of the field.

Actually, no. The probe wires are all outside of the magnetic flux region in Dr. Lewins experiment.

Not only probe wires are outside of magnetic flux region of Dr.Lewin's solenoid, but whole test circuit as well. You can do "probes outside of the field" experiment yourself, like this: https://www.eevblog.com/forum/amphour/562-electroboom!/msg3913808/#msg3913808 (https://www.eevblog.com/forum/amphour/562-electroboom!/msg3913808/#msg3913808)

"Path-directivity", he said.
Man, come on. Stop it. You're not fooling anyone. You don't know the theory, nor the practice, and not even the terminology, because you don't know jack squat what it means.
I am not natural englisch speaker, also you most likely got what I did mean by saying so, but instead of just correcting, you did not miss opportunity to insult, again.
Title: Re: #562 – Electroboom!
Post by: jesuscf on January 08, 2022, 05:21:30 pm
Summarizing a bit:

4) Many people have debunked Lewin's claims, including Mehdi, RDS accademy, and Mabilde.  Neither Jesse nor I are the first ones.

What you didn't say is that Mehdi, RDS, Mabilde and Jesse debunk each other. Mehdi said Lewin measured two different voltages across the same two points of the circuit due to bad probing. Mabilde said it was not bad probing, contradicting Mehdi, but it is because there's a "hidden" EMF in the probes. The RDS guy said the problem is otherwise: the ohm's law, according to him, predicts the presence of a voltage across a static wire that's not the product of its current times its resistance. Contradicting Mehdi and Mabilde. Jesse, the poor bugger, doesn't even have a theory to contradict the others, so he doesn't know what is really at stake, nor what to look for, nor how to interpret the results of his "experiment".

In common, all of the above showed without any doubt that, yes, voltages around a circuit immersed in a varying magnetic field are path-dependent. Jesse's setup even shows that dynamically. He changes the position, i.e. the path, of the hands of his clock and the meter displays a different voltage every time.

Quote
If you don't know how to use KVL, it doesn't mean that KVL doesn't work!

No one said KVL doesn't work. Sometimes it works. Sometimes it doesn't. It works when there's no varying magnetic field inside the area delimited by the boundaries of the circuit, and doesn't work otherwise.

You are definitively delusional!  Mehdi, Bob Duhamel, Mabilde, Jesse, and I agree without contradiction among other things, that Lewin didn't probe the circuit correctly.  This 'straw man' you are trying to create only highlights your deep ignorance about this subject matter.  Lewin does say KVL doesn't work in the particular experiment that concerns us.  But indeed KVL works as I have demonstrated may times already.
Title: Re: #562 – Electroboom!
Post by: bsfeechannel on January 08, 2022, 06:21:03 pm
You are definitively delusional!  Mehdi, Bob Duhamel, Mabilde, Jesse, and I agree without contradiction among other things, that Lewin didn't probe the circuit correctly.  This 'straw man' you are trying to create only highlights your deep ignorance about this subject matter.  Lewin does say KVL doesn't work in the particular experiment that concerns us.  But indeed KVL works as I have demonstrated may times already.

The only thing you agree with is that Lewin is, according to you, wrong, but none of you agree exactly why. And your experiments exhibit exactly the same outcome of Lewin's experiment: it is possible to measure two different voltages at the same time across the same two points of a circuit subjected to a varying magnetic field inside the area delimited by its borders.  In none of your experiments you could measure exactly the same voltage for any position of the probes or the meters.

So ¯\_(ツ)_/¯
Title: Re: #562 – Electroboom!
Post by: jesuscf on January 08, 2022, 06:38:23 pm
You are definitively delusional!  Mehdi, Bob Duhamel, Mabilde, Jesse, and I agree without contradiction among other things, that Lewin didn't probe the circuit correctly.  This 'straw man' you are trying to create only highlights your deep ignorance about this subject matter.  Lewin does say KVL doesn't work in the particular experiment that concerns us.  But indeed KVL works as I have demonstrated may times already.

The only thing you agree with is that Lewin is, according to you, wrong, but none of you agree exactly why. And your experiments exhibit exactly the same outcome of Lewin's experiment: it is possible to measure two different voltages at the same time across the same two points of a circuit subjected to a varying magnetic field inside the area delimited by its borders.  In none of your experiments you could measure exactly the same voltage for any position of the probes or the meters.

So ¯\_(ツ)_/¯

Another one for the collection of bsfeechannel's laughable statements!  :-DD   So, here are some questions for you: 

1) What is the calculated voltage between nodes A and D, VAD, in Lewin's circuit?

2) Is it the voltage VAD the same as the voltages measure by the voltmeters in Lewin's circuit?

3) If you are given the measured voltage of the voltmeter as well a the location of the voltmeter's probes in Lewin's circuit, can you determine the correct value of VAD?

4) If the answer to 3) is yes, how would you do it?

As usual, I expect nothing from bsfeechannel, so if anybody feels like pitching in, go for it.
Title: Re: #562 – Electroboom!
Post by: bsfeechannel on January 08, 2022, 06:52:51 pm
I am not natural englisch speaker,

No excuse. You had plenty of time to understand what the path-dependence of voltage means because you have been exposed to this concept for quite a few years now.

Quote
also you most likely got what I did mean by saying so

We have corrected you multiple times and you refused to accept the correction. Why would you accept it now?

Quote
, but instead of just correcting, you did not miss opportunity to insult, again.

Ignorance is a moral issue. Don't blame me for yours.
Title: Re: #562 – Electroboom!
Post by: jesuscf on January 08, 2022, 06:56:40 pm
I am not natural englisch speaker,

No excuse. You had plenty of time to understand what the path-dependence of voltage means because you have been exposed to this concept for quite a few years now.

Quote
also you most likely got what I did mean by saying so

We have corrected you multiple times and you refused to accept the correction. Why would you accept it now?

Quote
, but instead of just correcting, you did not miss opportunity to insult, again.

Ignorance is a moral issue. Don't blame me for yours.

Look who is talking!  bsfeechannel:  Mr. Laughable ignorance himself!!!  Stop wasting time, calculate the voltage VAD!
Title: Re: #562 – Electroboom!
Post by: Siwastaja on January 08, 2022, 07:55:20 pm
1) What is the calculated voltage between nodes A and D, VAD, in Lewin's circuit?

We have heard this "question" so many times I think we are reaching a situation which could be considered spam, forbidden by the forum rules. Basically copy-pasting your arguments is not helpful to anyone.

You have been given calculated values many times in this thread, also reasoning behind the numbers. It seems clear you just don't like the answer, or agree with it. Which is fine. But...

Now with sensible adults, discussion goes like this:
1 You ask X
2 You get answered
3 You don't agree
4 You tell why you don't agree so discussion can go on
5a In the end, either you accept being wrong, or you are right. Somebody learns something;
or
5b There is no resolution. End of story. No endless loop. Everybody can go do something else.

Not this:
1 You ask X
2 You get answered
3 You don't agree
4 goto 1, infinitely, word for word. + use the fact that others do not participate in your sick game and copy-paste their answers, as some kind of proof for a victory dance.

I see this as borderline harassment, nothing else. This is not only embarrassing because it's so obvious, but I'm sure everybody feels bad, yourself included, no? Please consider stopping this behavior and start behaving like a decent human being, accepting the fact that you are in disagreement, but trying to go forward.
Title: Re: #562 – Electroboom!
Post by: ogden on January 08, 2022, 08:19:28 pm
Ignorance is a moral issue. Don't blame me for yours.

... Said master of ignorance and personal attacks. Why didn't you address other part of my post, claim that everything - probes *and* test circuit are outside of magnetic flux area? Afraid of being cornered same way as with question about 100-turn transformer?
Title: Re: #562 – Electroboom!
Post by: ogden on January 08, 2022, 08:39:10 pm
1) What is the calculated voltage between nodes A and D, VAD, in Lewin's circuit?

We have heard this "question" so many times I think we are reaching a situation which could be considered spam, forbidden by the forum rules. Basically copy-pasting your arguments is not helpful to anyone.

You have been given calculated values many times in this thread, also reasoning behind the numbers. It seems clear you just don't like the answer, or agree with it. Which is fine. But...

Now with sensible adults, discussion goes like this:
1 You ask X
2 You get answered
3 You don't agree
4 You tell why you don't agree so discussion can go on
5a In the end, either you accept being wrong, or you are right. Somebody learns something;
or
5b There is no resolution. End of story. No endless loop. Everybody can go do something else.

Not this:
1 You ask X
2 You get answered
3 You don't agree
4 goto 1, infinitely, word for word. + use the fact that others do not participate in your sick game and copy-paste their answers, as some kind of proof for a victory dance.

I see this as borderline harassment, nothing else. This is not only embarrassing because it's so obvious, but I'm sure everybody feels bad, yourself included, no? Please consider stopping this behavior and start behaving like a decent human being, accepting the fact that you are in disagreement, but trying to go forward.

As you are claiming that this is recursive - you shall not have any difficulties of finding post with answer to this ever repeating question.
Title: Re: #562 – Electroboom!
Post by: jesuscf on January 08, 2022, 08:40:40 pm
1) What is the calculated voltage between nodes A and D, VAD, in Lewin's circuit?

We have heard this "question" so many times I think we are reaching a situation which could be considered spam, forbidden by the forum rules. Basically copy-pasting your arguments is not helpful to anyone.

You have been given calculated values many times in this thread, also reasoning behind the numbers. It seems clear you just don't like the answer, or agree with it. Which is fine. But...

Now with sensible adults, discussion goes like this:
1 You ask X
2 You get answered
3 You don't agree
4 You tell why you don't agree so discussion can go on
5a In the end, either you accept being wrong, or you are right. Somebody learns something;
or
5b There is no resolution. End of story. No endless loop. Everybody can go do something else.

Not this:
1 You ask X
2 You get answered
3 You don't agree
4 goto 1, infinitely, word for word. + use the fact that others do not participate in your sick game and copy-paste their answers, as some kind of proof for a victory dance.

I see this as borderline harassment, nothing else. This is not only embarrassing because it's so obvious, but I'm sure everybody feels bad, yourself included, no? Please consider stopping this behavior and start behaving like a decent human being, accepting the fact that you are in disagreement, but trying to go forward.

Do you know why I keep asking?  Because if you answer it correctly then you'll admit that KVL works in Lewin's circuit, which is the topic of this forum.  So, see if you can calculate VAD and then measure VAD and they match...  So start by calculating VAD first, then we can talk.
Title: Re: #562 – Electroboom!
Post by: Sredni on January 09, 2022, 03:35:13 am
Do you know why I keep asking?  Because if you answer it correctly then you'll admit that KVL works in Lewin's circuit, which is the topic of this forum. 

Ah, yes. The ultimate killer question.

Quote
So, see if you can calculate VAD and then measure VAD and they match...  So start by calculating VAD first, then we can talk.

Oh, for Fuchs' sakes!


1) What is the calculated voltage between nodes A and D, VAD, in Lewin's circuit?

It.
Depends.
On.
The.
Path.

Here are a few paths for which VAD, which I call VBA, has the value +0.9V.
Among this paths there are the left circuit branch and the path composed by probes and left voltmeter.

(https://i.postimg.cc/sDY8LD67/voltages-on-the-left.jpg)

And here are a few paths for which VAD, which I call VBA, assumes the value -0.1V.
Among these paths there are the right circuit branch and the path composed by probes and right voltmeter.

(https://i.postimg.cc/FztWP20T/voltages-on-the-right.jpg)

Mind you, my hand was uncertain and I draw a wobbly Lewin circuit, even off center with respect to the EMF generating coil. Still, I can put values on all those voltages, including the voltages across the resistors themselves.
Can you do the same with your KVL-obeying voltages?
Please, do so. Provide the voltages across the resistors and across the probes. Or shut up.

For paths cutting the variable magnetic field region, I can get all values comprised between +0.9V and -0.1V including 0V, 0.5V and 0.4V (what you would call the 'true' voltage in the circular, symmetric and axially aligned Lewin ring - but is it the same in this irregular off-center circuit? You tell me. Or shut up).


2) Is it the voltage VAD the same as the voltages measure by the voltmeters in Lewin's circuit?

Voltage VBA for the left branch of the circuit is the same as the voltage measured by the left voltmeter.
It is also the same as the voltages along paths gamma 1 to 7.
Voltage VBA for the right branch of the circuit is the same as the voltage measured by the right voltmeter.
It is also the same as the voltages along paths gamma 8 to 14.

Here is the total electric field inside the circuit. I am also giving the dimensions of the asymmetric ring. Let's say the inner solenoid perimeter is tangent to the straight line joining A and B and has a diameter of 2 cm. Or, if you prefer, draw the circuit to scale and put your own length.

(https://i.postimg.cc/52t3mxZx/electric-field-inside-the-ring.jpg)

If you feel the total field inside is wrong, by all means provide your solution. Or shut up.

And here is how I compute the voltages along the two branches of circuit from path integrals

(https://i.postimg.cc/KztrY4g5/compute-voltages-as-path-integrals.jpg)

I have thrown in, for good measure, the reason why people say KVL no longer works in this circuit: if you sum all voltages along the closed path (be they drops or localized EMFs) you end up with a nonzero total. The total is the EMF linked by the closed path itself. But let's NOT talk about that. Let's focus on the voltages and on how you compute the voltages across the resistors and across the probes for this asymmetric and off-center circuit.
In my world the arcs of conductor and the probes obey Ohm's law and therefore develop no voltage. Put your numbers in, now.

Incidentally, if the segment AB is tangent to the solenoid, VBA along the straight line joining A and B will be 0.9V, in this circuit.


3) If you are given the measured voltage of the voltmeter as well a the location of the voltmeter's probes in Lewin's circuit, can you determine the correct value of VAD?

Yes. But remember:
in my world the voltage depends on the path. So I will be able to tell you what the voltmeter will read and how it relates with the voltages along the left and the right branches of the circuits.


4) If the answer to 3) is yes, how would you do it? 

By applying Faraday's law when my loops include the variable magnetic region, and by applying KVL when they do not.

Now, please, tell me what are the values of the voltages across the resistors and the probes.
We all know we agree on what the voltmeters will measure. Show us how you compute the voltages across the probes and resistors in this asymmetric and off center configuration. Copy the profile of one circuit and report back when you have the results.

EDIT: added missing bold face (pun intended).
Title: Re: #562 – Electroboom!
Post by: jesuscf on January 09, 2022, 04:41:00 am

1) What is the calculated voltage between nodes A and D, VAD, in Lewin's circuit?

It.
Depends.
On.
The.
Path.

Here are a few paths for which VAD, which I call VBA, has the value +0.9V.
Among this paths there are the left circuit branch and the path composed by probes and left voltmeter.

(https://i.postimg.cc/sDY8LD67/voltages-on-the-left.jpg)

Yes, it depends on the path of the circuit formed by the wires and components.  An unique path that is not changing geometry or moving throughout space. The dashed paths you drew in the figure above are not valid paths because they are incapable of circulating current; they are made of... air?  You can not apply Faraday's law or more generally KVL on those made up paths.  The line integral must follow the circuit.  At this point you are even in conflict with what is correct in Lewin's lectures.  It is a waste of time to argue with you on anything else when you can not grasp this very simple concept.
Title: Re: #562 – Electroboom!
Post by: Sredni on January 09, 2022, 04:54:21 am
Yes, it depends on the path of the circuit formed by the wires and components.  An unique path that is not changing geometry or moving throughout space. The dashed paths you drew in the figure above are not valid paths because they are incapable of circulating current; they are made of... air?  You can not apply Faraday's law or more generally KVL on those made up paths.  The line integral must follow the circuit.  At this point you are even in conflict with what is correct in Lewin's lectures.  It is a waste of time to argue with you on anything else when you can not grasp this very simple concept.

 :palm:


Don't let your ignorance of the concept itself of voltage hinder you.
Show us the values for the voltages across the resistors and across the probes in the above asymmetric and offcentered Lewin ring.
All paths are made of copper, resistive material and voltmetrium. All in matter, the way you like.

Now, please, tell us what those values are.
Time to put up or shut up.
Title: Re: #562 – Electroboom!
Post by: jesuscf on January 09, 2022, 05:28:55 am
Yes, it depends on the path of the circuit formed by the wires and components.  An unique path that is not changing geometry or moving throughout space. The dashed paths you drew in the figure above are not valid paths because they are incapable of circulating current; they are made of... air?  You can not apply Faraday's law or more generally KVL on those made up paths.  The line integral must follow the circuit.  At this point you are even in conflict with what is correct in Lewin's lectures.  It is a waste of time to argue with you on anything else when you can not grasp this very simple concept.

 :palm:


Don't let your ignorance of the concept itself of voltage hinder you.
Show us the values for the voltages across the resistors and across the probes in the above asymmetric and offcentered Lewin ring.
All paths are made of copper, resistive material and voltmetrium. All in matter, the way you like.

Now, please, tell us what those values are.
Time to put up or shut up.

What you are trying to do here is the good old "moving-the-goalposts" fallacy.  You are trying to distract from the original question; your plan is not going to work.

Back to Lewin's circuit which is perfectly symmetric, with no extra 'dashed' paths.  Let us concentrate on that fixed circuit, with no extra wires of any kind.  If you assume the voltage between nodes A and D, VAD, is unique at some instant of time (which by the way it is true), can you calculate that voltage?
Title: Re: #562 – Electroboom!
Post by: ogden on January 09, 2022, 07:29:53 am
To prove that "KVL dies", Lewin's cultists suddenly ignore Faraday's law of Electromagnetic Induction they claim to know so well and EMF as such. With same success one can prove that "KVL dies" for simple circuit as chemical battery with load resistor connected. As internal resistance of the battery is so small that can be ignored - we obviously assume it as zero. After all Lewin ignores wire resistance in his experiment as well. Then according to Lewin's approach we use Ohms law to claim that there is no field inside battery, U = IR, but as R=0, then U=0. Then we integrate fields around the circuit to conclude that KVL is for birds :-DD
Title: Re: #562 – Electroboom!
Post by: bsfeechannel on January 09, 2022, 07:30:48 am
Back to Lewin's circuit which is perfectly symmetric, with no extra 'dashed' paths.  Let us concentrate on that fixed circuit, with no extra wires of any kind.  If you assume the voltage between nodes A and D, VAD, is unique at some instant of time (which by the way it is true), can you calculate that voltage?

Before we go back to Lewin's circuit, let's consider friction. You know, friction is a non-conservative force field. Your car consumes fuel basically to overcome friction. Let's suppose that your car makes 10km with 1 liter of fuel. If you want to go from Amarillo, TX, to Santa Fé, NM, you have several options. You can go via Tucumcari and you'll drive 450 km. But if you want to go by way of Raton, NM, it'll take 622 km. If you choose Clovis, NM, it'll be 515 km.

How many liters of fuel you'll need to go from Amarillo to Santa Fé? There's no single correct answer because it depends on the path. Tucumcari: 45 l; Raton: 62.5 l; Clovis: 51.5 l.

So it is with a non-conservative electric field. The energy needed to move a test charge from two points will depend on the path. Since voltage is energy per unit charge, there's where you get your path dependency. That's soooooooo intuitive.
Title: Re: #562 – Electroboom!
Post by: bsfeechannel on January 09, 2022, 07:39:04 am
Lewin's cultists suddenly ignore Faraday's law of Electromagnetic Induction

There are two kinds of people here: those who, in the words of Siwastaja, "get off the high horse and read, read, and reread until you understand", and those who are too lazy to learn electromagnetism and think that attacking those who can teach it will redeem them from their profound stupidity.
Title: Re: #562 – Electroboom!
Post by: jesuscf on January 09, 2022, 07:48:51 am
Lewin's cultists suddenly ignore Faraday's law of Electromagnetic Induction

There are two kinds of people here: those who, in the words of Siwastaja, "get off the high horse and read, read, and reread until you understand", and those who are too lazy to learn electromagnetism and think that attacking those who can teach it will redeem them from their profound stupidity.

Nope!  There is also bsfeechannel, a fine example of the Dunning-Kruger effect.
Title: Re: #562 – Electroboom!
Post by: bsfeechannel on January 09, 2022, 07:51:00 am
Nope!  There is also bsfeechannel, a fine example of the Dunning-Kruger effect.

We both are ignorant, but you don't know it.
Title: Re: #562 – Electroboom!
Post by: ogden on January 09, 2022, 09:00:02 am
If you assume the voltage between nodes A and D, VAD, is unique at some instant of time (which by the way it is true), can you calculate that voltage?

Don't bother. Usual tactics of cultists - to simply ignore anything that would somehow let them agree with you.

In case of 1V EMF, resistors 100 and 900 Ohms, 1mA current is flowing through loop. Voltage on resistors 0.1V and 0.9V accordingly. So far everybody agree.

Lewin's cultists truly believe that 1/4 of the transformer wire turn do not have 1/4 of EMF on it because when they try to measure using voltmeter, by placing voltmeter leads next to wire they measure, they see 0V - as expected. It is obvious that Faradays' law of induction do not care - it is test circuit or voltmeter leads, but Lewin's cultists are stubborn, they claim that they know better and for some reason voltmeter leads are not influenced by time-varying magnetic field of experiment, only circuit. In short - fact that Lewin and his cultists struggle to properly measure EMF, do not mean it does not exist.

Back to the voltage between A-D: If circuit is symmetric, all four wires of the circuit loop equal length, resistors so small that we ignore EMF inside them, then each wire receives 1/4 of EMF, 1V/4 = 0.25V. Voltage between A and D will be +0.25V-0.1V+0.25V = 0.4V if calculated using 100 Ohm resistor side. Also we can calculate voltage using 900 Ohm resistor side, -0.25+0.9V-0.25=0.4V.

If we try to measure voltage between A-D, by orienting probe leads along the circuit loop, each probe lead will receive +0.25V or -0.25V EMF depending on which side of the solenoid they are. If we put voltmeter and it's leads on one side of solenoid, then we see 0.4V-0.25V-0.25V = -0.1V, on the other side we will see 0.4V+0.25V+0.25V=0.9V. Visualization of what I am telling, attached.
Title: Re: #562 – Electroboom!
Post by: Siwastaja on January 09, 2022, 09:35:30 am
As you are claiming that this is recursive - you shall not have any difficulties of finding post with answer to this ever repeating question.

Oh, now I see the tactic clearly.

I remember the exact post quite well - Sredni calculated, IIRC, six different values, one of which matched jesuscf's expectation. jesuscf definitely saw it, and remembers it (assuming he doesn't have actual brain damage, which is, I think, a fair assumption).

But, I have "any difficulties" finding it. 33 pages of smoke screens and bullshit burying it so deep I can't find it. Tried to look it up, gave up. Sheer number of pages prevents the "show all pages" feature, preventing successful text search. It is simply too much work. Besides, if I look it up, this will result in nothing but more smoke screens. Your whole tactic is to just waste others time.

See? Now you can claim your victory.

I didn't know you are this dirty. I'm sad for you. Goodbye.
Title: Re: #562 – Electroboom!
Post by: ogden on January 09, 2022, 09:57:29 am
As you are claiming that this is recursive - you shall not have any difficulties of finding post with answer to this ever repeating question.

Oh, now I see the tactic clearly.

Tactic? Victory? Geez  :palm: You may consider to cool down.

When you reply to someone "I told you long time ago" - perhaps you both know what you are talking about. Problem is that this is not private chat room for you two, this is public forum, huge thread with 30+ pages indeed. There are other readers, many most likely did not read every post. Be courteous to at least newcomers interested in the subject or whatever they find in this thread.
Title: Re: #562 – Electroboom!
Post by: emece67 on January 09, 2022, 12:19:10 pm
.
Title: Re: #562 – Electroboom!
Post by: Sredni on January 09, 2022, 01:18:15 pm
Now, please, tell us what those values are.
Time to put up or shut up.

What you are trying to do here is the good old "moving-the-goalposts" fallacy.  You are trying to distract from the original question; your plan is not going to work.

Back to Lewin's circuit which is perfectly symmetric, with no extra 'dashed' paths.  Let us concentrate on that fixed circuit, with no extra wires of any kind.  If you assume the voltage between nodes A and D, VAD, is unique at some instant of time (which by the way it is true), can you calculate that voltage?

You remind me of that guy who claimed he could read.
But he could only read from his own only book. Not other books.

Hic Rhodus, hic salta.

What happened to your beautiful theory? It only applies to circular concentric setups?
Title: Re: #562 – Electroboom!
Post by: Sredni on January 09, 2022, 02:48:11 pm
Yes I was expecting to continue discussion with Sredni or bsfeechannel, not you. As soon as Lewin's followers agree that path-dependency can be proven using multiple windings of measurement wire which are essentially another secondary winding on transformer

But are they?
The other winding has the voltmeter's internal resistance embedded into it. Do you really think it is equivalent to another secondary winding of a transformer?

If I put a loop of wire around a core with an EMF of 1V I can get 1V across a load of say 10 ohms and 100 mA flowing in it.
If I put a loop formed by my voltmeters and its two probes around a core with an EMF of 1V and I attach the prove tips to a 10 ohms load... what do I get? Still 1V across the load? Still 100 mA flowing through it?
No?
Why not?
Ah, now you consider the internal resistance of the voltmeter a second load?
But are you sure there is a net voltage across the wires?

Look what happens to the charge when you short the voltmeters tips around a core:

(https://i.postimg.cc/CKB4Y14c/screenshot-2.png)

You can see it by putting some numbers (I used 1mohm for the short and 10Gohm for the open to get nice numbers, but feel free to put actual values if you wish.

(https://i.postimg.cc/R0pwDnJP/screenshot-3.png)

Well, that charge that has changed place is what kills the 'induced voltage' in the conducting arcs. You need to study a bit to understand why. Here's a hint for you: "superposition".

Regarding path depedency and how to measure the voltage ALONG a coil, it's obvious that it's easier to show it for a single coil

(https://i.postimg.cc/zftL0ZHK/screenshot-4.png)

(https://i.postimg.cc/15s9n7fQ/screenshot-5.png)

why bothering adding the unnecessary complication of multiple loops? They will only make pictures harder to draw and follow, but the concept will still be the same: making sure there is no dB/dt region in the measurement loop. And besides, a KVLer will be in the same position if they were asked to show a picture that illustrates their 'path-independent' measure for a multiloop transformer with multiple layers of windings. What do you do? Drill through the core and the other wires?

Title: Re: #562 – Electroboom!
Post by: jesuscf on January 09, 2022, 04:50:54 pm
Now, please, tell us what those values are.
Time to put up or shut up.

What you are trying to do here is the good old "moving-the-goalposts" fallacy.  You are trying to distract from the original question; your plan is not going to work.

Back to Lewin's circuit which is perfectly symmetric, with no extra 'dashed' paths.  Let us concentrate on that fixed circuit, with no extra wires of any kind.  If you assume the voltage between nodes A and D, VAD, is unique at some instant of time (which by the way it is true), can you calculate that voltage?

You remind me of that guy who claimed he could read.
But he could only read from his own only book. Not other books.

Hic Rhodus, hic salta.

What happened to your beautiful theory? It only applies to circular concentric setups?

What I described applies to any circuit!  It may not be easy but it is doable!  I will not be distracted with a harder problem because that is your tactic.  Let us concentrate in easy symmetric setups like Lewin's circuit, the one where he says KVL doesn't work.  Did you figure out how to calculate VAD yet?  You may want to read the post from Ogden, he explains it clearly, and it is very easy.

By the way, where are the calculations for the two capacitor problem?  It looks to me that you used SPICE to solve that one.  How did you represent the induced EMF in the equivalent circuit?




Title: Re: #562 – Electroboom!
Post by: emece67 on January 09, 2022, 05:04:57 pm
.
Title: Re: #562 – Electroboom!
Post by: jesuscf on January 09, 2022, 05:35:49 pm
What I described applies to any circuit!  It may not be easy but it is doable!  I will not be distracted with a harder problem because that is your tactic.  Let us concentrate in easy symmetric setups like Lewin's circuit, the one where he says KVL doesn't work.  Did you figure out how to calculate VAD yet?  You may want to read the post from Ogden, he explains it clearly, and it is very easy.

Apparently he figured it out. Here: https://www.eevblog.com/forum/amphour/562-electroboom!/msg3927425/#msg3927425 (https://www.eevblog.com/forum/amphour/562-electroboom!/msg3927425/#msg3927425)

Can you explain us where is his calculation wrong?

I have seen Ogden's calculations, but such calculation relies heavily on symmetry. Albeit knowing that it may be harder, how is that calculation applied to a non symmetric circuit?

Regards.

There is only one answer.  The voltage VAD is unique, because there is only one path to consider, the path of the circuit.  Sredni keeps saying that the voltage is path dependent, with made up paths that don't follow the circuit, and them he proceeds to calculate infinite answers because he has infinite made up paths.

The real question here should be: will KVL work on an loop that is not circular with an asymmetrically placed magnetic flux?  Of course it will!  That is what Faraday's law tell us.

EDIT: Trevor Kearny has a really nice video with a very ingenious solution for a non-symmetric problem here:

https://www.youtube.com/watch?v=FR8k12j7_Eo (https://www.youtube.com/watch?v=FR8k12j7_Eo)

Team Lewin should watch this video too, maybe they can learn something!



Title: Re: #562 – Electroboom!
Post by: emece67 on January 09, 2022, 05:46:21 pm
.
Title: Re: #562 – Electroboom!
Post by: jesuscf on January 09, 2022, 06:09:25 pm
There is only one answer.  The voltage VAD is unique, because there is only one path to consider, the path of the circuit.

Then how will you measure such VAD?

The real question here should be: will KVL work on an loop that is not circular with an asymmetrically placed magnetic flux?  Of course it will!  That is what Faraday's law tell us.

Then how will you calculate VAD in an asymmetrical circuit?

It looks like I edited my post after you replied.  Here, watch this video by Trevor Kearney:

https://www.youtube.com/watch?v=FR8k12j7_Eo (https://www.youtube.com/watch?v=FR8k12j7_Eo)


Title: Re: #562 – Electroboom!
Post by: ogden on January 09, 2022, 06:15:48 pm
Yes I was expecting to continue discussion with Sredni or bsfeechannel, not you. As soon as Lewin's followers agree that path-dependency can be proven using multiple windings of measurement wire which are essentially another secondary winding on transformer

But are they?
The other winding has the voltmeter's internal resistance embedded into it. Do you really think it is equivalent to another secondary winding of a transformer?

I do not see you are answering my question which was very SPECIFIC - about transformer with 100 turns of secondary winding. Also if you use voltmeter leads with resistance embedded into them - stop doing that! Get proper tools.

[edit] Same fallacy again and again.  :palm:  You can't draw mythical dotted line which separates circuit wires from probe wires and claim that Faradays's law of Induction stops where you choose:

(https://i.postimg.cc/zftL0ZHK/screenshot-4.png)

Seems you missed this:
https://www.eevblog.com/forum/amphour/562-electroboom!/msg3913808/#msg3913808 (https://www.eevblog.com/forum/amphour/562-electroboom!/msg3913808/#msg3913808)
Title: Re: #562 – Electroboom!
Post by: ogden on January 09, 2022, 06:51:33 pm
Back to the voltage between A-D: If circuit is symmetric, all four wires of the circuit loop equal length, resistors so small that we ignore EMF inside them, then each wire receives 1/4 of EMF, 1V/4 = 0.25V. Voltage between A and D will be +0.25V-0.1V+0.25V = 0.4V if calculated using 100 Ohm resistor side. Also we can calculate voltage using 900 Ohm resistor side, -0.25+0.9V-0.25=0.4V.
How would you compute Vad if:
  • the circuit is not symmetric;
  • and the four wires have different lengths.

I am glad you asked. Asymmetric to circuit/probe_leads magnetic field is outside of scope for Dr.Lewin's experiment, Thou jesuscf provided link to video (https://youtu.be/FR8k12j7_Eo?t=1) explaining this problem.

In case you ask about calculation of voltage between two asymmetric points of Dr.Lewin's experiment, then it is not that hard. You can even try to solve it yourself. Let's say circumference of wire is 10cm, resistors still way much smaller so we ignore dimensions of both. We know that EMF induced is 1V meaning 0.1V per cm. Voltage on resistors still 0.1V and 0.9V. Let's say we move A&D points 0.5cm off-center to the right, in result we have two wires of 3cm with 100Ohms resistor on the left side and two wires 2cm each, with 900 Ohms resistor on right side of circuit loop. Voltage calculation using left side path would be +0.3-0.1+0.3=0.5V and -0.2+0.9-0.2=0.5V for right side path.

Zero wire on right side and 2*5cm on the left? - No problem! +0.5-0.1+0.5=0.9V on the left and -0.0+0.9-0.0=0.9V on the right. It also means measurement points are exactly on right side resistor terminals, so we can calculate answer using Ohms law U=IR as well.

Quote
And also, how will you measure Vad (supposed that it is even possible to directly measure it)?

Measuring asymmetrical circuit is another can of worms which is, sorry again, outside of scope of Dr.Lewin's experiment, but I can tell you how to measure for (symmetric) Dr.Lewin's experiment: run voltmeter leads properly so magnetic flux do not impact them. There is only one way to run probe leads for Lewin's experiment - directly between AD, such a way that they split magnetic flux area in half, in case of symmetric circuit which Lewin's experiment for sure is. Mabilde not only tells how to do it but also shows it (https://youtu.be/JpVoT101Azg?t=761).
Title: Re: #562 – Electroboom!
Post by: Sredni on January 09, 2022, 07:38:44 pm
Now, please, tell us what those values are.
Time to put up or shut up.

What you are trying to do here is the good old "moving-the-goalposts" fallacy.  You are trying to distract from the original question; your plan is not going to work.

Back to Lewin's circuit which is perfectly symmetric, with no extra 'dashed' paths.  Let us concentrate on that fixed circuit, with no extra wires of any kind.  If you assume the voltage between nodes A and D, VAD, is unique at some instant of time (which by the way it is true), can you calculate that voltage?

You remind me of that guy who claimed he could read.
But he could only read from his own only book. Not other books.

Hic Rhodus, hic salta.

What happened to your beautiful theory? It only applies to circular concentric setups?

What I described applies to any circuit!  It may not be easy but it is doable!


Oh, I know it is doable. Numerically. And Notaros, McDonald and Belcher know how to do it, as well.
But I believe most KVLers have no friggin' idea on how to do it.
Can you do it?

Quote
I will not be distracted with a harder problem because that is your tactic. 

No, my tactic is to show you that the EM method of using both the electrical scalar potential and the magnetic vector potential is not the brightest of the ideas when you are dealing with circuits. And if you were able to solve that problem you would know what Notaros and McDonald mean. And I could also show you how you do NOT use this method in every day life.

Quote
Let us concentrate in easy symmetric setups like Lewin's circuit, the one where he says KVL doesn't work.  Did you figure out how to calculate VAD yet?

See? I showed you so many VAD so many times that I've lost count. And you still pretend I didn't.

Quote
By the way, where are the calculations for the two capacitor problem?  It looks to me that you used SPICE to solve that one.  How did you represent the induced EMF in the equivalent circuit?

Do you really think it's that difficult to compute the reactances and use those instead of resistances?
Wow. To me, it's so easy it's basically a waste of time doing it.
But I will do it when  you show me what are the actual voltages across the resistors and the probe wires in the circuit I've drawn.

Can't you do it?
Title: Re: #562 – Electroboom!
Post by: jesuscf on January 09, 2022, 08:05:41 pm
Now, please, tell us what those values are.
Time to put up or shut up.

What you are trying to do here is the good old "moving-the-goalposts" fallacy.  You are trying to distract from the original question; your plan is not going to work.

Back to Lewin's circuit which is perfectly symmetric, with no extra 'dashed' paths.  Let us concentrate on that fixed circuit, with no extra wires of any kind.  If you assume the voltage between nodes A and D, VAD, is unique at some instant of time (which by the way it is true), can you calculate that voltage?

You remind me of that guy who claimed he could read.
But he could only read from his own only book. Not other books.

Hic Rhodus, hic salta.

What happened to your beautiful theory? It only applies to circular concentric setups?

What I described applies to any circuit!  It may not be easy but it is doable!


Oh, I know it is doable. Numerically. And Notaros, McDonald and Belcher know how to do it, as well.
But I believe most KVLers have no friggin' idea on how to do it.
Can you do it?

Quote
I will not be distracted with a harder problem because that is your tactic. 

No, my tactic is to show you that the EM method of using both the electrical scalar potential and the magnetic vector potential is not the brightest of the ideas when you are dealing with circuits. And if you were able to solve that problem you would know what Notaros and McDonald mean. And I could also show you how you do NOT use this method in every day life.

Quote
Let us concentrate in easy symmetric setups like Lewin's circuit, the one where he says KVL doesn't work.  Did you figure out how to calculate VAD yet?

See? I showed you so many VAD so many times that I've lost count. And you still pretend I didn't.

Quote
By the way, where are the calculations for the two capacitor problem?  It looks to me that you used SPICE to solve that one.  How did you represent the induced EMF in the equivalent circuit?

Do you really think it's that difficult to compute the reactances and use those instead of resistances?
Wow. To me, it's so easy it's basically a waste of time doing it.
But I will do it when  you show me what are the actual voltages across the resistors and the probe wires in the circuit I've drawn.

Can't you do it?

This video from Trevor Kearney debunks many, if not all your claims:

https://www.youtube.com/watch?v=FR8k12j7_Eo (https://www.youtube.com/watch?v=FR8k12j7_Eo)

Have you watched it yet?  Pay attention on how many potential difference voltages between nodes A and B he calculates: one.

EDIT: there is a typo in the video above.  When calculating the voltage VAB, Trevor Kearney wrote -0.17202 but the correct calculation is:

VAB=0.3798V-(0.1 A x 5 ohm)=-0.1202V


Title: Re: #562 – Electroboom!
Post by: Sredni on January 09, 2022, 09:15:00 pm
I do not see you are answering my question which was very SPECIFIC - about transformer with 100 turns of secondary winding. Also if you use voltmeter leads with resistance embedded into them - stop doing that! Get proper tools.


Right. You have voltmeters with zero internal resistance. Where did you buy those? Duckburg?

Quote
[edit] Same fallacy again and again.  :palm:  You can't draw mythical dotted line which separates circuit wires from probe wires and claim that Faradays's law of Induction stops where you choose:

(https://i.postimg.cc/zftL0ZHK/screenshot-4.png)


You do not understand what you see. Any path you can imagine within that region of space will NOT be able to form a loop (with the branch of circuit you want to measure the voltage of) that will cut the variable flux. That is why KVL will work in that region of space.
In 2D it's easy to see it. In 3D the three-dimensional region of space will be more complicated, but you can still find it.

Quote
Seems you missed this:
https://www.eevblog.com/forum/amphour/562-electroboom!/msg3913808/#msg3913808 (https://www.eevblog.com/forum/amphour/562-electroboom!/msg3913808/#msg3913808)

Are those the zero internal resistance voltmeters?
Title: Re: #562 – Electroboom!
Post by: jesuscf on January 09, 2022, 09:30:45 pm
Are those the zero internal resistance voltmeters?

When measuring voltage what you want is infinite internal resistance voltmeters.  Anyhow, I'll assume what you wrote is a typo.

For the BM869s: input impedance is 10Mohm in the mV range.

For the Fluke 187, directly from the user manual:

"When measuring voltage, the meter acts like a 10 MΩ
(10,000,000 Ω) impedance in parallel with the circuit. This
loading effect can cause measurement errors in high-impedance
circuits. In most cases, the error is negligible
(0.1% or less) if the circuit impedance is 10 kΩ (10,000 Ω)
or less."
Title: Re: #562 – Electroboom!
Post by: Sredni on January 09, 2022, 09:39:46 pm
This video from Trevor Kearney debunks many, if not all your claims:
https://www.youtube.com/watch?v=FR8k12j7_Eo (https://www.youtube.com/watch?v=FR8k12j7_Eo)
Have you watched it yet?  Pay attention on how many potential difference voltages between nodes A and B he calculates: one.

This is the second time I write this in this thread.

Dude, that's Trevor Kearney. He's probably the most active "Lewin defender" on youtube (go read his posts on Electroboom's channel if you don't believe me). He's "Armchair Physics Nobel Laureate" number one in fromjesse's scale. It's funny that you bring his videos in your defense (like fromjesse linking those videos from Purdue university, not understanding that professor Melloch has the same views as Lewin).
Trevor is computing the ELECTRIC SCALAR POTENTIAL difference. One of the two components of the actual voltage.

And, no, in the case of my wobbly circuit, not even him will be able to find a closed analytical solution. You have to go numerical. But that's not the point. The point is that you are not able to find 'your' voltage between any two points of a simple resistive circuit linking a variable magnetic flux. Not a very useful theory, isn't it. Especially considering that you should apply that even to simple resistive circuits NOT linking a variable magnetic flux, but just being in the same universe as a variable magnetic flux region. (Let me guess: you have no idea what I am talking about, right?)


ADDENDUM
Yes, I really meant "zero internal resistance voltmeters" because I did consider the 10 meg internal resistance of my voltmeter when I showed you that running a voltmeter with its probes around a magnetic core is NOT the same thing as running a zero resistance wire around it. But you did not understand that either
Title: Re: #562 – Electroboom!
Post by: jesuscf on January 09, 2022, 09:57:30 pm
This video from Trevor Kearney debunks many, if not all your claims:
https://www.youtube.com/watch?v=FR8k12j7_Eo (https://www.youtube.com/watch?v=FR8k12j7_Eo)
Have you watched it yet?  Pay attention on how many potential difference voltages between nodes A and B he calculates: one.

This is the second time I write this in this thread.

Dude, that's Trevor Kearney. He's probably the most active "Lewin defender" on youtube (go read his posts on Electroboom's channel if you don't believe me). He's "Armchair Physics Nobel Laureate" number one in fromjesse's scale. It's funny that you bring his videos in your defense (like fromjesse linking those videos from Purdue university, not understanding that professor Melloch has the same views as Lewin).
Trevor is computing the ELECTRIC SCALAR POTENTIAL difference. One of the two components of the actual voltage.

And, no, in the case of my wobbly circuit, not even him will be able to find a closed analytical solution. You have to go numerical. But that's not the point. The point is that you are not able to find 'your' voltage between any two points of a simple resistive circuit linking a variable magnetic flux. Not a very useful theory, isn't it. Especially considering that you should apply that even to simple resistive circuits NOT linking a variable magnetic flux, but just being in the same universe as a variable magnetic flux region. (Let me guess: you have no idea what I am talking about, right?)


ADDENDUM
Yes, I really meant "zero internal resistance voltmeters" because I did consider the 10 meg internal resistance of my voltmeter when I showed you that running a voltmeter with its probes around a magnetic core is NOT the same thing as running a zero resistance wire around it. But you did not understand that either

Really?  It looks to me that you didn't watch the video!  Are you in denial again?  Here, I added to his right hand side circuit equivalent, the equivalent circuit for the left hand side:

(https://www.eevblog.com/forum/amphour/562-electroboom!/?action=dlattach;attach=1374995;image)

Let us calculate the voltage from the right:

\$V_{AB}  = 0.3798V - 0.1A \times 5\Omega  =  - 0.1202V\$

Now the voltage from the left:

\$V_{AB}  = 0.1A \times 5\Omega  - 0.6202V =  - 0.1202V\$

Are the calculated values identical?  Yes!

Does KVL hold? Yes!

\$ - 0.3798V + 0.1A \times 5\Omega + 0.1A \times 5\Omega  - 0.6202V = 0 \$

What BS excuse are you going to craft now?


Title: Re: #562 – Electroboom!
Post by: ogden on January 09, 2022, 10:19:55 pm
Same fallacy again and again.  :palm:  You can't draw mythical dotted line which separates circuit wires from probe wires and claim that Faradays's law of Induction stops where you choose:

(https://i.postimg.cc/zftL0ZHK/screenshot-4.png)


You do not understand what you see. Any path you can imagine within that region of space will NOT be able to form a loop (with the branch of circuit you want to measure the voltage of) that will cut the variable flux. That is why KVL will work in that region of space.
In 2D it's easy to see it. In 3D the three-dimensional region of space will be more complicated, but you can still find it.

So you say that circuit containing just open wire loop which you drew around variable flux area is subject of Faraday's law of induction, but probe leads which also you drew around variable flux area, just farther away than wire - do not? This is yes/no question.
Title: Re: #562 – Electroboom!
Post by: thinkfat on January 09, 2022, 11:11:17 pm
Same fallacy again and again.  :palm:  You can't draw mythical dotted line which separates circuit wires from probe wires and claim that Faradays's law of Induction stops where you choose:

(https://i.postimg.cc/zftL0ZHK/screenshot-4.png)


You do not understand what you see. Any path you can imagine within that region of space will NOT be able to form a loop (with the branch of circuit you want to measure the voltage of) that will cut the variable flux. That is why KVL will work in that region of space.
In 2D it's easy to see it. In 3D the three-dimensional region of space will be more complicated, but you can still find it.

So you say that circuit containing just open wire loop which you drew around variable flux area is subject of Faraday's law of induction, but probe leads which also you drew around variable flux area, just farther away than wire - do not? This is yes/no question.

It may surprise you, but the answer to this would actually be: no.  :-DD

And still the volt meter would read 0V.
Title: Re: #562 – Electroboom!
Post by: ogden on January 09, 2022, 11:42:57 pm
So you say that circuit containing just open wire loop which you drew around variable flux area is subject of Faraday's law of induction, but probe leads which also you drew around variable flux area, just farther away than wire - do not? This is yes/no question.

It may surprise you, but the answer to this would actually be: no.  :-DD

And still the volt meter would read 0V.

Your answer surprised, in a good way. EMF in wire and EMF in voltmeter leads are equal, for particular setup. Voltmeter essentially measure difference between two equal EMF sources, thus shall read 0V. [edit] It is easier to imagine by rearranging test setup for voltmeter having single probe wire only.
Title: Re: #562 – Electroboom!
Post by: bsfeechannel on January 10, 2022, 01:54:01 am
Lewin's cultists truly believe that 1/4 of the transformer wire turn do not have 1/4 of EMF on it because when they try to measure using voltmeter, by placing voltmeter leads next to wire they measure, they see 0V - as expected.

If, as expected, the voltage measured across a piece of wire is zero, it is because it IS zero. Any other conjecture about that voltage is absolutely moronic in the context of our discussion.

Quote
It is obvious that Faradays' law of induction do not care - it is test circuit or voltmeter leads, but Lewin's cultists are stubborn, they claim that they know better and for some reason voltmeter leads are not influenced by time-varying magnetic field of experiment, only circuit.

Faraday's law of induction doesn't care about wires, leads, probes, whatever. They're irrelevant. They're not even mentioned in the law.

What Faraday's law of induction says is that the TOTAL EMF around a CLOSED ARBITRARY path, any path, whatever path in whatever portion of the SPACE, is proportional to the rate of change of the magnetic field INSIDE the area, whatever area, delimited by the aforementioned closed PATH.

So, if your meter, your probes and your wire under test constitute a closed PATH that delimit an AREA where the magnetic field doesn't change or, better yet, where there's none, the EMF is NULLE (in Latvian), i.e. ZERO, нуль, صفر, 零, μηδέν.

So there's no EMF, no belief, no cultism, no Lewin. It is only the theory, which by the way was extensively reviewed and tested by the most brilliant minds of our recent times, agreeing with practice.

Lewin, the Lewin cultism, the belief, and the phantasmagorical EMF that you claim is present in the wires is just an artificial construct that only exists in the brain-damaged mind of a KVLiar. No scientist, professor or author, dead or alive, will agree with you.

Quote
In short - fact that Lewin and his cultists struggle to properly measure EMF, do not mean it does not exist.

We, you and all the other cabrones like Mehdi, Mabilde, Jesse and the RSD dude, measure and the voltage is zero (or to be precise it is the current times the resistance of the wire), exactly like the theory predicts.

I told you in the other discussion we had three years ago that you don't understand electromagnetism because you're a "circuity" guy. You're limited to wires. No wonder, Kirchhoff was obsessed with them. They're the essence of his laws. Maxwell's equations made us understand the phenomenon of electricity and magnetism beyond wires. The universe is plenty of an unfathomably large number of electromagnetic events and it is not even wired.

So free your mind and give Lewin a kiss in the cheek.
Title: Re: #562 – Electroboom!
Post by: jesuscf on January 10, 2022, 02:53:19 am

Quote
In short - fact that Lewin and his cultists struggle to properly measure EMF, do not mean it does not exist.

We, you and all the other cabrones like Mehdi, Mabilde, Jesse and the RSD dude, measure and the voltage is zero (or to be precise it is the current times the resistance of the wire), exactly like the theory predicts.

Hey bsfeechannel, I may be asking you to do something impossible, but can you watch this video from Trevor Kearney and try to understand it?  Pay particular attention starting at around minute 13:00 when Trevor Kearney explains how to properly measure the voltage between nodes A and B, VAB, the way Lewin should have done it in his circuit between nodes A and D.

https://www.youtube.com/watch?v=FR8k12j7_Eo (https://www.youtube.com/watch?v=FR8k12j7_Eo)
Title: Re: #562 – Electroboom!
Post by: bsfeechannel on January 10, 2022, 03:47:39 am
Does KVL hold? Yes!

\$ - 0.3798V + 0.1A \times 5\Omega + 0.1A \times 5\Omega  - 0.6202V = 0 \$

What BS excuse are you going to craft now?

How about the one given by Maxwell?

Quote
The  theory  of  conjugate  conductors  has  been  investigated  by
Kirchhoff,  who  has  stated  the  conditions  of  a  linear  system  in
the  following  manner,  in  which  the  consideration  of  the  potential
is  avoided.


(1)  (Condition  of  'continuity.')  At  any  point  of  the  system
the  sum  of  all  the  currents  which  flow  towards  that  point  is
zero.

(2)  In  any  complete  circuit  formed  by  the  conductors  the  sum
of  the  electromotive  forces  taken  round  the  circuit  is  equal  to
the  sum  of  the  products  of  the  current  in  each  conductor  multi-
plied by  the  resistance  of  that  conductor.

So, if the consideration of potential is avoided, then we need to eliminate the potential difference between points A and D from your beautiful equation, which becomes:

\$ 0.1A \times 5\Omega + 0.1A \times 5\Omega = 1V ≠ 0 \$

KVL fails.

And this agrees exactly with what he says in the video @12:19:

Quote
It is always worth reminding ourselves that our voltmeter connected between nodes A and B, and the conditions are that the voltmeter measurement PATH does not intersect the time varying field, the voltmeter will not indicate the potential difference between A and B, rather it will indicate the OHMIC voltage difference between nodes A and B.
Title: Re: #562 – Electroboom!
Post by: jesuscf on January 10, 2022, 04:25:41 am
Does KVL hold? Yes!

\$ - 0.3798V + 0.1A \times 5\Omega + 0.1A \times 5\Omega  - 0.6202V = 0 \$

What BS excuse are you going to craft now?

How about the one given by Maxwell?

Quote
The  theory  of  conjugate  conductors  has  been  investigated  by
Kirchhoff,  who  has  stated  the  conditions  of  a  linear  system  in
the  following  manner,  in  which  the  consideration  of  the  potential
is  avoided.


(1)  (Condition  of  'continuity.')  At  any  point  of  the  system
the  sum  of  all  the  currents  which  flow  towards  that  point  is
zero.

(2)  In  any  complete  circuit  formed  by  the  conductors  the  sum
of  the  electromotive  forces  taken  round  the  circuit  is  equal  to
the  sum  of  the  products  of  the  current  in  each  conductor  multi-
plied by  the  resistance  of  that  conductor.

So, if the consideration of potential is avoided, then we need to eliminate the potential difference between points A and D from your beautiful equation, which becomes:

\$ 0.1A \times 5\Omega + 0.1A \times 5\Omega = 1V ≠ 0 \$

KVL fails.

And this agrees exactly with what he says in the video @12:19:

Quote
It is always worth reminding ourselves that our voltmeter connected between nodes A and B, and the conditions are that the voltmeter measurement PATH does not intersect the time varying field, the voltmeter will not indicate the potential difference between A and B, rather it will indicate the OHMIC voltage difference between nodes A and B.

As I predicted, you didn't understand, by a long, long shot!  Either that or we have another member of team Lewin in denial!  :-DD

1) Half_Circuit != Complete_Circuit.  But more important:

2) Do you even know what this quote means "is always worth reminding ourselves that our voltmeter connected between nodes A and B, and the conditions are that the voltmeter measurement path does not intersect the time varying field, the voltmeter will not indicate the potential difference between A and B, rather it will indicate the ohmic voltage difference between nodes A and B."  Well, it means that if you measure with the voltmeter without cancelling the induced EMF in the probes due to the varying magnetic field,  what you read in the multimeter is the voltage drop in the resistance.  This is the mistake that Lewin made in his demonstration.  But wait, there is more:

3) Starting at minute 13:00 Trevor Kearney says : "...if it were physically possible to align a voltmeter measurement path with the physical path A-X-B, the indication would then be the same as the calculated potential difference...".  Do you know why?  Because there will not be induced EMF in the voltmeter probes!  This is exactly what I have shown a bunch of times with my experiments.  And this is also what Lewin failed to do or even understand in his experiment.


Title: Re: #562 – Electroboom!
Post by: Sredni on January 10, 2022, 05:04:22 am
This video from Trevor Kearney debunks many, if not all your claims:
https://www.youtube.com/watch?v=FR8k12j7_Eo (https://www.youtube.com/watch?v=FR8k12j7_Eo)
Have you watched it yet?  Pay attention on how many potential difference voltages between nodes A and B he calculates: one.

This is the second time I write this in this thread.

Dude, that's Trevor Kearney. He's probably the most active "Lewin defender" on youtube (go read his posts on Electroboom's channel if you don't believe me). He's "Armchair Physics Nobel Laureate" number one in fromjesse's scale. It's funny that you bring his videos in your defense (like fromjesse linking those videos from Purdue university, not understanding that professor Melloch has the same views as Lewin).
Trevor is computing the ELECTRIC SCALAR POTENTIAL difference. One of the two components of the actual voltage.

And, no, in the case of my wobbly circuit, not even him will be able to find a closed analytical solution. You have to go numerical. But that's not the point. The point is that you are not able to find 'your' voltage between any two points of a simple resistive circuit linking a variable magnetic flux. Not a very useful theory, isn't it. Especially considering that you should apply that even to simple resistive circuits NOT linking a variable magnetic flux, but just being in the same universe as a variable magnetic flux region. (Let me guess: you have no idea what I am talking about, right?)


ADDENDUM
Yes, I really meant "zero internal resistance voltmeters" because I did consider the 10 meg internal resistance of my voltmeter when I showed you that running a voltmeter with its probes around a magnetic core is NOT the same thing as running a zero resistance wire around it. But you did not understand that either

Really?  It looks to me that you didn't watch the video!  Are you in denial again?  Here, I added to his right hand side circuit equivalent, the equivalent circuit for the left hand side:

(https://www.eevblog.com/forum/amphour/562-electroboom!/?action=dlattach;attach=1374995;image)

Let us calculate the voltage from the right:

\$V_{AB}  = 0.3798V - 0.1A \times 5\Omega  =  - 0.1202V\$

Now the voltage from the left:

\$V_{AB}  = 0.1A \times 5\Omega  - 0.6202V =  - 0.1202V\$

Are the calculated values identical?  Yes!

Does KVL hold? Yes!

\$ - 0.3798V + 0.1A \times 5\Omega + 0.1A \times 5\Omega  - 0.6202V = 0 \$

What BS excuse are you going to craft now?

Ok, I had not watched the video, because I cannot see images and videos from my laptop. Now I logged in (for the first time) with my phone and while posting is a PITA (is this why you quote entire posts? Can't select parts of text?), at least I can see videos and images to the posts I respond.

Yes, if you take A UNIFORM RESISTIVE RING the voltages you measure from the exterior between two points on a diameter are bound to be equal because they are given by

    Resistance of arc * current in loop

And if the arcs have the same length you end up with the same voltage. The flux off axis requires the development of surface charge that will compensate for the nonuniformity of the induced field in the region occupied by the ring.

This is not the two resistor lewin ring but it's okay, I'll go along with it.

Now PICK TWO POINTS THAT ARE NOT ON A DIAMETER, and show us how KVL is working.

For example pick A and B such that going from A to B will require three quarter of circumference and going from B to A will require a quarter.

Compute the voltages now.
Title: Re: #562 – Electroboom!
Post by: bsfeechannel on January 10, 2022, 05:33:13 am
Well, it means that if you measure with the voltmeter without cancelling the induced EMF in the probes due to the varying magnetic field,  what you read in the multimeter is the voltage drop in the resistance.  This is the mistake that Lewin made in his demonstration.  But wait, there is more:

There's no EMF induced because the circuit the probes establish with the resistance and the meter has no varying magnetic field inside the area delimited by the closed path constituted by them. This is Faraday's law. That's why the only voltage that you'll measure is the voltage across the resistor, which is the current through it times its resistance.

Lewin probed this area with a calibrated hall sensor in his previous lecture and showed beyond any doubt that the area was free from varying magnetic fields.

Everyone repeated his experiment and obtained the same results.

That's why he's right.

Quote
3) Starting at minute 13:00 Trevor Kearney says : "...if it were physically possible to align a voltmeter measurement path with the physical path A-X-B, the indication would then be the same as the calculated potential difference...".  Do you know why?

Yes.

Quote
Because there will not be induced EMF in the voltmeter probes! 

It's because the probes of your meter would be in such a convenient path where the total EMF produced by the two loops they establish with each resistor will be the same and will coincide in value with the scalar potential difference between the two points.

Quote
This is exactly what I have shown a bunch of times with my experiments.

Congratulations! You showed that not only you can measure two different voltages across the same two points of a circuit whose area is subjected to a varying magnetic field, but also that you can measure a third voltage.

I'll recommend you for the Nobel Prize of Physics of 2022.

Quote
  And this is also what Lewin failed to do or even understand in his experiment.

Poor Lewin.  Oh, well, at least this didn't disprove the fact that you CAN measure two different voltages across the same two points of a circuit whose area is subjected to a varying magnetic field and that KVL is bird food.
Title: Re: #562 – Electroboom!
Post by: bsfeechannel on January 10, 2022, 05:36:56 am
Oh, now I see the tactic clearly.

Their tactic is to be debunked multiple times in every single page of the thread. Endless amusement.
Title: Re: #562 – Electroboom!
Post by: jesuscf on January 10, 2022, 05:49:18 am
This video from Trevor Kearney debunks many, if not all your claims:
https://www.youtube.com/watch?v=FR8k12j7_Eo (https://www.youtube.com/watch?v=FR8k12j7_Eo)
Have you watched it yet?  Pay attention on how many potential difference voltages between nodes A and B he calculates: one.

This is the second time I write this in this thread.

Dude, that's Trevor Kearney. He's probably the most active "Lewin defender" on youtube (go read his posts on Electroboom's channel if you don't believe me). He's "Armchair Physics Nobel Laureate" number one in fromjesse's scale. It's funny that you bring his videos in your defense (like fromjesse linking those videos from Purdue university, not understanding that professor Melloch has the same views as Lewin).
Trevor is computing the ELECTRIC SCALAR POTENTIAL difference. One of the two components of the actual voltage.

And, no, in the case of my wobbly circuit, not even him will be able to find a closed analytical solution. You have to go numerical. But that's not the point. The point is that you are not able to find 'your' voltage between any two points of a simple resistive circuit linking a variable magnetic flux. Not a very useful theory, isn't it. Especially considering that you should apply that even to simple resistive circuits NOT linking a variable magnetic flux, but just being in the same universe as a variable magnetic flux region. (Let me guess: you have no idea what I am talking about, right?)


ADDENDUM
Yes, I really meant "zero internal resistance voltmeters" because I did consider the 10 meg internal resistance of my voltmeter when I showed you that running a voltmeter with its probes around a magnetic core is NOT the same thing as running a zero resistance wire around it. But you did not understand that either

Really?  It looks to me that you didn't watch the video!  Are you in denial again?  Here, I added to his right hand side circuit equivalent, the equivalent circuit for the left hand side:

(https://www.eevblog.com/forum/amphour/562-electroboom!/?action=dlattach;attach=1374995;image)

Let us calculate the voltage from the right:

\$V_{AB}  = 0.3798V - 0.1A \times 5\Omega  =  - 0.1202V\$

Now the voltage from the left:

\$V_{AB}  = 0.1A \times 5\Omega  - 0.6202V =  - 0.1202V\$

Are the calculated values identical?  Yes!

Does KVL hold? Yes!

\$ - 0.3798V + 0.1A \times 5\Omega + 0.1A \times 5\Omega  - 0.6202V = 0 \$

What BS excuse are you going to craft now?

Ok, I had not watched the video, because I cannot see images and videos from my laptop. Now I logged in (for the first time) with my phone and while posting is a PITA (is this why you quote entire posts? Can't select parts of text?), at least I can see videos and images to the posts I respond.

Yes, if you take A UNIFORM RESISTIVE RING the voltages you measure from the exterior between two points on a diameter are bound to be equal because they are given by

    Resistance of arc * current in loop

And if the arcs have the same length you end up with the same voltage. The flux off axis requires the development of surface charge that will compensate for the nonuniformity of the induced field in the region occupied by the ring.

This is not the two resistor lewin ring but it's okay, I'll go along with it.

Now PICK TWO POINTS THAT ARE NOT ON A DIAMETER, and show us how KVL is working.

For example pick A and B such that going from A to B will require three quarter of circumference and going from B to A will require a quarter.

Compute the voltages now.

Sure, why not.  I picked the nodes A and C as shown in the figure:

(https://www.eevblog.com/forum/amphour/562-electroboom!/?action=dlattach;attach=1375328;image)

Since the circuit is symmetric along the horizontal axis with respect to the magnetic flux, the induced EMF between nodes A and C on the top right quarter is half what Trevor Kearney calculated in the video, or 0.3798V/2=0.1899V.  Also the resistance is now also half or 2.5 ohms.  Conversely, the induced EMF in the rest of the ring is 1V-0.1899V=0.8101V and the resistance is 7.5 ohms.  This is the equivalent circuit we need to solve using KVL:

(https://www.eevblog.com/forum/amphour/562-electroboom!/?action=dlattach;attach=1375334;image)

You can calculate either using the left or the right of the equivalent circuit.  The result is the same:

\$
\begin{array}{l}
 V_{AC}  = 0.1899V - 0.1A \times 2.5\Omega  =  - 0.0601V \\
 V_{AC}  = 0.1A \times 7.5\Omega  - 0.8101V =  - 0.0601V \\
 \end{array}
\$


Title: Re: #562 – Electroboom!
Post by: ogden on January 10, 2022, 06:00:04 am
Now everybody can see what cultism does to the brain. He states:

Faraday's law of induction doesn't care about wires, leads, probes, whatever. They're irrelevant. They're not even mentioned in the law.

Actual law:
Quote
Faraday's law states that there is emf on the conductive loop when the magnetic flux through the surface enclosed by the loop varies in time.

If that is not conclusive enough, one can watch Dr.Lewin's YT video about Faraday's law (https://youtu.be/nGQbA2jwkWI?t=1).

Whole post for the records, in case bsfeechannel wakes-up and finds that he contradicted even to himself:

Lewin's cultists truly believe that 1/4 of the transformer wire turn do not have 1/4 of EMF on it because when they try to measure using voltmeter, by placing voltmeter leads next to wire they measure, they see 0V - as expected.

If, as expected, the voltage measured across a piece of wire is zero, it is because it IS zero. Any other conjecture about that voltage is absolutely moronic in the context of our discussion.

Quote
It is obvious that Faradays' law of induction do not care - it is test circuit or voltmeter leads, but Lewin's cultists are stubborn, they claim that they know better and for some reason voltmeter leads are not influenced by time-varying magnetic field of experiment, only circuit.

Faraday's law of induction doesn't care about wires, leads, probes, whatever. They're irrelevant. They're not even mentioned in the law.

What Faraday's law of induction says is that the TOTAL EMF around a CLOSED ARBITRARY path, any path, whatever path in whatever portion of the SPACE, is proportional to the rate of change of the magnetic field INSIDE the area, whatever area, delimited by the aforementioned closed PATH.

So, if your meter, your probes and your wire under test constitute a closed PATH that delimit an AREA where the magnetic field doesn't change or, better yet, where there's none, the EMF is NULLE (in Latvian), i.e. ZERO, нуль, صفر, 零, μηδέν.

So there's no EMF, no belief, no cultism, no Lewin. It is only the theory, which by the way was extensively reviewed and tested by the most brilliant minds of our recent times, agreeing with practice.

Lewin, the Lewin cultism, the belief, and the phantasmagorical EMF that you claim is present in the wires is just an artificial construct that only exists in the brain-damaged mind of a KVLiar. No scientist, professor or author, dead or alive, will agree with you.

Quote
In short - fact that Lewin and his cultists struggle to properly measure EMF, do not mean it does not exist.

We, you and all the other cabrones like Mehdi, Mabilde, Jesse and the RSD dude, measure and the voltage is zero (or to be precise it is the current times the resistance of the wire), exactly like the theory predicts.

I told you in the other discussion we had three years ago that you don't understand electromagnetism because you're a "circuity" guy. You're limited to wires. No wonder, Kirchhoff was obsessed with them. They're the essence of his laws. Maxwell's equations made us understand the phenomenon of electricity and magnetism beyond wires. The universe is plenty of an unfathomably large number of electromagnetic events and it is not even wired.

So free your mind and give Lewin a kiss in the cheek.
Title: Re: #562 – Electroboom!
Post by: jesuscf on January 10, 2022, 06:46:51 am
Now everybody can see what cultism does to the brain. He states:

Faraday's law of induction doesn't care about wires, leads, probes, whatever. They're irrelevant. They're not even mentioned in the law.

Actual law:
Quote
Faraday's law states that there is emf on the conductive loop when the magnetic flux through the surface enclosed by the loop varies in time.

If that is not conclusive enough, one can watch Dr.Lewin's YT video about Faraday's law (https://youtu.be/nGQbA2jwkWI?t=1).


You are correct: team Lewin is even contradicting basic concepts that Lewin taught correctly!
Title: Re: #562 – Electroboom!
Post by: bsfeechannel on January 10, 2022, 07:26:04 am
Now everybody can see what cultism does to the brain.

Yes. And in the case of the KVLiars, the cognitive impairment seems to be irreversible.

Quote
Actual law:
Faraday's law states that there is emf on the conductive loop when the magnetic flux through the surface enclosed by the loop varies in time.

Since the EMF will be there even if you don't have a conductor (otherwise propagation would be impossible), the most accepted version of Faraday's law is:

The electromotive force around a closed path is equal to the negative of the time rate of change of the magnetic flux enclosed by the path.

Instead of the conductive loop you can also have a conductive plate or block. Faraday's law doesn't care.

Quote
If that is not conclusive enough, one can watch Dr.Lewin's YT video about Faraday's law (https://youtu.be/nGQbA2jwkWI?t=1).

Here you can see how we are independent and we verify things ourselves, unlike KVLiars who take whatever stupid assertion that comes out of Mehdi's or uncle RSD's butt at face value.

Quote
Whole post for the records, in case bsfeechannel wakes-up

I never sleep.

Quote
and finds that he contradicted even to himself:

More likely to find KVLiars spreading misconceptions and pseudo-science.
Title: Re: #562 – Electroboom!
Post by: Sredni on January 10, 2022, 10:28:06 am

Quote
Compute the voltages now.

Sure, why not.  I picked the nodes A and C as shown in the figure:

(https://www.eevblog.com/forum/amphour/562-electroboom!/?action=dlattach;attach=1375328;image)

Since the circuit is symmetric along the horizontal axis with respect to the magnetic flux, the induced EMF between nodes A and C on the top right quarter is half what Trevor Kearney calculated in the video, or 0.3798V/2=0.1899V.  Also the resistance is now also half or 2.5 ohms.  Conversely, the induced EMF in the rest of the ring is 1V-0.1899V=0.8101V and the resistance is 7.5 ohms

That is not voltage. That is scalar potential difference.
It turns out that Trevor uses V to denote the scalar potential difference (what I now call phi) and U_path to denote voltage (what I call V_gamma).

You can ask himself, if you want.
Comment in his video asking:

"Hi Trevor, is this video a demonstration that Lewin is wrong in saying that voltage is path dependent? Or are you computing the scalar potential difference which happens to be independent of path, but is different from voltage?"

And see what he replies.

Chances are that he will link you to his last video "Scalar potential difference a questionable choice for the true voltage between the test nodes in Dr Lewin's Experiment"

https://www.youtube.com/watch?v=I1kYKF2x9Ns (https://www.youtube.com/watch?v=I1kYKF2x9Ns)

(In case you are wondering, no I haven't seen all of Trevor's videos from start to finish. I love his passion and I know we share the same positions from what he writes in the comments on YouTube, bit his videos are too long and too low volume for me to watch from start to finish - I just skim through them. But here is a screenshot that tells the whole story:

(https://i.postimg.cc/cCR5wB1k/Trevor-s-PD-vs-Voltage.png)

see, how he makes a distinction between scalar potential difference and voltage?)
Title: Re: #562 – Electroboom!
Post by: Sredni on January 10, 2022, 10:33:15 am

I never sleep.

Brother!
Title: Re: #562 – Electroboom!
Post by: jesuscf on January 10, 2022, 03:44:09 pm

Quote
Compute the voltages now.

Sure, why not.  I picked the nodes A and C as shown in the figure:

(https://www.eevblog.com/forum/amphour/562-electroboom!/?action=dlattach;attach=1375328;image)

Since the circuit is symmetric along the horizontal axis with respect to the magnetic flux, the induced EMF between nodes A and C on the top right quarter is half what Trevor Kearney calculated in the video, or 0.3798V/2=0.1899V.  Also the resistance is now also half or 2.5 ohms.  Conversely, the induced EMF in the rest of the ring is 1V-0.1899V=0.8101V and the resistance is 7.5 ohms

That is not voltage. That is scalar potential difference.
It turns out that Trevor uses V to denote the scalar potential difference (what I now call phi) and U_path to denote voltage (what I call V_gamma).

You can ask himself, if you want.
Comment in his video asking:

"Hi Trevor, is this video a demonstration that Lewin is wrong in saying that voltage is path dependent? Or are you computing the scalar potential difference which happens to be independent of path, but is different from voltage?"

And see what he replies.

Chances are that he will link you to his last video "Scalar potential difference a questionable choice for the true voltage between the test nodes in Dr Lewin's Experiment"

https://www.youtube.com/watch?v=I1kYKF2x9Ns (https://www.youtube.com/watch?v=I1kYKF2x9Ns)

(In case you are wondering, no I haven't seen all of Trevor's videos from start to finish. I love his passion and I know we share the same positions from what he writes in the comments on YouTube, bit his videos are too long and too low volume for me to watch from start to finish - I just skim through them. But here is a screenshot that tells the whole story:

(https://i.postimg.cc/cCR5wB1k/Trevor-s-PD-vs-Voltage.png)

see, how he makes a distinction between scalar potential difference and voltage?)

In that problem the scalar potential difference is equal to the true voltage between nodes A and C because the magnetic flux is defined as:

\$\varphi (t) = 1.0t[Wb]\$

So the magnitude of the induced EMF is a constant 1V, which is not a function of time.  We are solving a DC circuit!
Title: Re: #562 – Electroboom!
Post by: thinkfat on January 10, 2022, 04:34:16 pm
In that problem the scalar potential difference is equal to the true voltage between nodes A and C because the magnetic flux is defined as:

\$\varphi (t) = 1.0t[Wb]\$

So the magnitude of the induced EMF is a constant 1V, which is not a function of time.  We are solving a DC circuit!

What does that have to do with anything? It's just a construct to make the calculation easier to follow.
Title: Re: #562 – Electroboom!
Post by: jesuscf on January 10, 2022, 06:46:31 pm
In that problem the scalar potential difference is equal to the true voltage between nodes A and C because the magnetic flux is defined as:

\$\varphi (t) = 1.0t[Wb]\$

So the magnitude of the induced EMF is a constant 1V, which is not a function of time.  We are solving a DC circuit!

What does that have to do with anything? It's just a construct to make the calculation easier to follow.

So is my calculation of the potential difference between nodes A and C correct or no?   Is that the voltage VAC or not?  Just asking, because I see yet another 'moving the goalposts' fallacy coming from Sredni.
Title: Re: #562 – Electroboom!
Post by: Sredni on January 10, 2022, 07:20:44 pm
I did not even check, but if it's scalar potential difference you are after, and you get the same result as Trevor then it might be correct.
But it is only half of the story

Voltage = scalar PD + induced voltage

The scalar PD obeys kvl because it's the result of the conservative field generated by the displaced surface and interface charge. In Lewin's ring it is largely due to the charge accumulated at the resistors' ends.
You just need to sum the induced voltage to get the 'true voltage', ie (minus) the path integral of the TOTAL electric field.


Note in the case of the symmetric  uniform resistive ring  things are a bit different in that Etot = Eind and you only need surface charge when you place the ring off-center.
Title: Re: #562 – Electroboom!
Post by: bsfeechannel on January 11, 2022, 03:48:49 am

I never sleep.

Brother!

Don't worry. I have a life.

So is my calculation of the potential difference between nodes A and C correct or no?   Is that the voltage VAC or not?  Just asking, because I see yet another 'moving the goalposts' fallacy coming from Sredni.

If you think that the scalar potential difference will save you, think again.

(https://www.eevblog.com/forum/amphour/562-electroboom!/?action=dlattach;attach=1376111;image)
Title: Re: #562 – Electroboom!
Post by: ogden on January 11, 2022, 04:58:35 am
If you think that the scalar potential difference will save you, think again.

(https://www.eevblog.com/forum/amphour/562-electroboom!/?action=dlattach;attach=1376111;image)

To prove that KVL is for birds, Lewin's cultists simply omit energy source in their circuit schematics - as if current is appearing out of nowhere. Very clever! BTW 0.5V + 0.5V = 1V which is voltage of "invisible EMF source", so... how to say... you did just prove KVL! Congrats! Glad you can learn too. Next time don't forget to include transformer secondary in your schematics. Hopefully you know the symbol.
Title: Re: #562 – Electroboom!
Post by: bsfeechannel on January 11, 2022, 05:42:07 am
To prove that KVL is for birds, Lewin's cultists simply omit energy source in their circuit schematics - as if current is appearing out of nowhere.

This is not a concern of KVL. It only states that the voltages around a loop must add up to zero. End of story. In the case of Lewin's circuit they don't.

Quote
Very clever! BTW 0.5V + 0.5V = 1V

Which is even worse, since 1 V is farther away from 0 V than 0.5 V is.

Title: Re: #562 – Electroboom!
Post by: ogden on January 11, 2022, 05:57:08 am
To prove that KVL is for birds, Lewin's cultists simply omit energy source in their circuit schematics - as if current is appearing out of nowhere.

This is not a concern of KVL. It only states that the voltages around a loop must add up to zero. End of story. In the case of Lewin's circuit they don't.

Quote
Very clever! BTW 0.5V + 0.5V = 1V

Which is even worse, since 1 V is farther away from 0 V than 0.5 V is.

Next thing Lewin's cultists do - choose convenient to their fallacy, polarities. They can easily claim that 1V chemical battery in series with 1K resistor sum-up to 2V because I*R added to 1V of the battery equals 2V, so KVL per their logic fail.  :palm:
Title: Re: #562 – Electroboom!
Post by: jesuscf on January 11, 2022, 06:37:52 am
To prove that KVL is for birds, Lewin's cultists simply omit energy source in their circuit schematics - as if current is appearing out of nowhere.

This is not a concern of KVL. It only states that the voltages around a loop must add up to zero. End of story. In the case of Lewin's circuit they don't.

Quote
Very clever! BTW 0.5V + 0.5V = 1V

Which is even worse, since 1 V is farther away from 0 V than 0.5 V is.

Next thing Lewin's cultists do - choose convenient to their fallacy, polarities. They can easily claim that 1V chemical battery in series with 1K resistor sum-up to 2V because I*R added to 1V of the battery equals 2V, so KVL per their logic fail.  :palm:

They will always come up with a new fallacy, I'll bet on it!
Title: Re: #562 – Electroboom!
Post by: Sredni on January 11, 2022, 07:52:46 am
If you think that the scalar potential difference will save you, think again.

(https://www.eevblog.com/forum/amphour/562-electroboom!/?action=dlattach;attach=1376111;image)

To prove that KVL is for birds, Lewin's cultists simply omit energy source in their circuit schematics - as if current is appearing out of nowhere. Very clever! BTW 0.5V + 0.5V = 1V which is voltage of "invisible EMF source", so... how to say... you did just prove KVL! Congrats! Glad you can learn too. Next time don't forget to include transformer secondary in your schematics. Hopefully you know the symbol.

...aaaaand we are back to square one.

 5  + 3 = 8.  Vs 5 + 3 - 8 = 0

and the inability of KVLers of seeing a difference between lumped and unlumpable circuits.

But hold that thought about current coming out of nowhere.
In the case of the perfectly symmetric uniform resistive ring we have Etot = Eind (this is exact in the 2D case). That is, since Ecoul=0, the scalar potential difference is zero everywhere on the ring.
And in fact KVLers proudly show that their radial measures of "voltage" are all zero.
In their view K"V"L holds because the sum of all zero "voltages" is zero.

But now its their turn to ignore the energy source.
They have all "voltages" zero, and "current coming out of nowhere". So they are forced to switch definition of voltage and invent the distributed battery+resistor element.

  1-1+1-1+1-1+1-1+1-1+1-1+1-1+1-1=0

Does that mean that the radial measure that shows zero "voltage" is not measuring the true voltage, now?
Title: Re: #562 – Electroboom!
Post by: thinkfat on January 11, 2022, 08:34:29 am
(https://www.eevblog.com/forum/amphour/562-electroboom!/?action=dlattach;attach=1376111;image)

So, if the scalar PD between A and D is the "true voltage" in the Lewin ring, then what @bsfeechannel showed above is a proper model for Dr. Lewins setup, right?
That means one can replace the induced EMF with a voltage source between A and D with the "true voltage" and that is then the energy source of the circuit?
But if this is really a proper model for Dr. Lewins circuit, it must also reproduce the meter readings from the experiment, right?
Only it doesn't. Hm. So, apparently it is not a proper model for Dr. Lewins circuit.

Maybe it's a proper model for the @jesuscf circuit? Apparently so, because he measured it with a few percent of error. He cannot say where this error comes from, but lets not go there now. But he also admits that if the probing wires were to follow a slightly different path, the error would increase.
So, @jesuscf found one single, very special path on which the \$V_{AD}\$ coincided with the computed scalar PD and yes, if in his circuit he replaced the induced EMF with a voltage source between A and D, the outcome would not change. But if he connected a second voltmeter between A and D on any other path, the whole model would collapse.


Title: Re: #562 – Electroboom!
Post by: jesuscf on January 11, 2022, 03:38:31 pm
(https://www.eevblog.com/forum/amphour/562-electroboom!/?action=dlattach;attach=1376111;image)

So, if the scalar PD between A and D is the "true voltage" in the Lewin ring, then what @bsfeechannel showed above is a proper model for Dr. Lewins setup, right?
That means one can replace the induced EMF with a voltage source between A and D with the "true voltage" and that is then the energy source of the circuit?
But if this is really a proper model for Dr. Lewins circuit, it must also reproduce the meter readings from the experiment, right?
Only it doesn't. Hm. So, apparently it is not a proper model for Dr. Lewins circuit.

Maybe it's a proper model for the @jesuscf circuit? Apparently so, because he measured it with a few percent of error. He cannot say where this error comes from, but lets not go there now. But he also admits that if the probing wires were to follow a slightly different path, the error would increase.
So, @jesuscf found one single, very special path on which the \$V_{AD}\$ coincided with the computed scalar PD and yes, if in his circuit he replaced the induced EMF with a voltage source between A and D, the outcome would not change. But if he connected a second voltmeter between A and D on any other path, the whole model would collapse.

It looks to me that you never learnt how to do basic circuit analysis!  Also, are you implying that I forged the results of my experiments?  There is a very simple way for you to prove it: repeat the experiment yourself and post the results here.
Title: Re: #562 – Electroboom!
Post by: jesuscf on January 11, 2022, 03:51:18 pm
If you think that the scalar potential difference will save you, think again.

(https://www.eevblog.com/forum/amphour/562-electroboom!/?action=dlattach;attach=1376111;image)

To prove that KVL is for birds, Lewin's cultists simply omit energy source in their circuit schematics - as if current is appearing out of nowhere. Very clever! BTW 0.5V + 0.5V = 1V which is voltage of "invisible EMF source", so... how to say... you did just prove KVL! Congrats! Glad you can learn too. Next time don't forget to include transformer secondary in your schematics. Hopefully you know the symbol.

...aaaaand we are back to square one.

 5  + 3 = 8.  Vs 5 + 3 - 8 = 0

and the inability of KVLers of seeing a difference between lumped and unlumpable circuits.

But hold that thought about current coming out of nowhere.
In the case of the perfectly symmetric uniform resistive ring we have Etot = Eind (this is exact in the 2D case). That is, since Ecoul=0, the scalar potential difference is zero everywhere on the ring.
And in fact KVLers proudly show that their radial measures of "voltage" are all zero.
In their view K"V"L holds because the sum of all zero "voltages" is zero.

But now its their turn to ignore the energy source.
They have all "voltages" zero, and "current coming out of nowhere". So they are forced to switch definition of voltage and invent the distributed battery+resistor element.

  1-1+1-1+1-1+1-1+1-1+1-1+1-1+1-1=0

Does that mean that the radial measure that shows zero "voltage" is not measuring the true voltage, now?

Yes, jump to conclusions without getting the correct equivalent circuit of the modified setup and solving it first.  That is the favorite procedure of team Lewin!
Title: Re: #562 – Electroboom!
Post by: jesuscf on January 11, 2022, 04:11:36 pm

I never sleep.

Brother!

Don't worry. I have a life.

So is my calculation of the potential difference between nodes A and C correct or no?   Is that the voltage VAC or not?  Just asking, because I see yet another 'moving the goalposts' fallacy coming from Sredni.

If you think that the scalar potential difference will save you, think again.

(https://www.eevblog.com/forum/amphour/562-electroboom!/?action=dlattach;attach=1376111;image)

Here, I fixed it for you.  I am pretty sure you'll never understand it!

(https://www.eevblog.com/forum/amphour/562-electroboom!/?action=dlattach;attach=1376441;image)
Title: Re: #562 – Electroboom!
Post by: ogden on January 11, 2022, 04:57:52 pm
So, if the scalar PD between A and D is the "true voltage" in the Lewin ring, then what @bsfeechannel showed above is a proper model for Dr. Lewins setup, right?
No. It is just proper copy of Dr.Lewin's improper model which omits EMF source in the circuit schematics. Just because some could not find proof of EMF by measuring improperly probing segment of the loop, does not mean it does not exist.

Quote
That means one can replace the induced EMF with a voltage source between A and D with the "true voltage" and that is then the energy source of the circuit?
No. EMF is induced in the conductive elements of the loop. You can't calculate or measure voltage between two points of the circuit and declare it as voltage *source*. It could be just voltage *drop* with source somewhere else. You really shall consider to learn (more about) Electromagnetic Induction and Faraday's law, start with https://youtu.be/nGQbA2jwkWI?t=1 (https://youtu.be/nGQbA2jwkWI?t=1).
Title: Re: #562 – Electroboom!
Post by: thinkfat on January 11, 2022, 05:20:32 pm
Okay. So, none of you, neither ogden or jesuscf are able to provide an equivalent circuit that reproduces Dr. Lewins experiment, right? And your whole defence to why you cannot do it boils down to "Lewin is an idiot!" :palm:

PS: before you go about bringing "stray magnetic flux" into the game again, watch this video maybe:

https://youtu.be/u6ud7JD0fV4

Ring core transformer, magnetic flux well confined inside the core. But the outcome is the same.
Title: Re: #562 – Electroboom!
Post by: jesuscf on January 11, 2022, 06:19:50 pm
Okay. So, none of you, neither ogden or jesuscf are able to provide an equivalent circuit that reproduces Dr. Lewins experiment, right? And your whole defence to why you cannot do it boils down to "Lewin is an idiot!" :palm:

PS: before you go about bringing "stray magnetic flux" into the game again, watch this video maybe:

https://youtu.be/u6ud7JD0fV4

Ring core transformer, magnetic flux well confined inside the core. But the outcome is the same.

You don't understand how Faraday's law work!  :-DD  If the circuit formed by the probes of your instrument go around the 'confined' varying magnetic core, there is going to be an induced EMF in the probes unless you align the probes carefully.  That is what is happening in the video you posted; there is definitively an induced EMF in the oscilloscope probes.  They are making exactly the same mistake Lewin made!

Watch this video from Trevor Kearney and pay attention to everything what he says.  In particular, since you don't seem to understand how is done, look carefully how he derives the equivalent circuit for the RHS of the setup.  That is what Lewin should have done.

https://youtu.be/FR8k12j7_Eo



Title: Re: #562 – Electroboom!
Post by: Sredni on January 11, 2022, 06:44:14 pm
Okay. So, none of you, neither ogden or jesuscf are able to provide an equivalent circuit that reproduces Dr. Lewins experiment, right? And your whole defence to why you cannot do it boils down to "Lewin is an idiot!" :palm:

PS: before you go about bringing "stray magnetic flux" into the game again, watch this video maybe:

https://youtu.be/u6ud7JD0fV4

Ring core transformer, magnetic flux well confined inside the core. But the outcome is the same.

You don't understand how Faraday's law work!  :-DD  If the circuit formed by the probes of your instrument go around the 'confined' varying magnetic core, there is going to be an induced EMF in the probes unless you align the probes carefully.

What you KVLers keep missing is that this emf makes the charge in the probes move and accumulate at the voltmeter's internal resistance and the field that is generated by that charge will cancel the induced field in the probes. Leaving nearly nothing in the probes.

Quote
  That is what is happening in the video you posted; there is definitively an induced EMF in the oscilloscope probes.  They are making exactly the same mistake Lewin made!

Sure. The MIT has always been a nest of incompetent physicists and engineers.
They should hire you, instead!

Quote
Watch this video from Trevor Kearney and pay attention to everything what he says.  In particular, since you don't seem to understand how is done, look carefully how he derives the equivalent circuit for the RHS of the setup.  That is what Lewin should have done.

https://youtu.be/FR8k12j7_Eo

Did you ask Trevor if he thinks Lewin is in error?
Are you afraid of asking?
Because I am pretty sure you are misunderstanding his point of view.
Title: Re: #562 – Electroboom!
Post by: thinkfat on January 11, 2022, 06:57:30 pm
Okay. So, none of you, neither ogden or jesuscf are able to provide an equivalent circuit that reproduces Dr. Lewins experiment, right? And your whole defence to why you cannot do it boils down to "Lewin is an idiot!" :palm:

PS: before you go about bringing "stray magnetic flux" into the game again, watch this video maybe:

https://youtu.be/u6ud7JD0fV4

Ring core transformer, magnetic flux well confined inside the core. But the outcome is the same.

You don't understand how Faraday's law work!  :-DD  If the circuit formed by the probes of your instrument go around the 'confined' varying magnetic core, there is going to be an induced EMF in the probes unless you align the probes carefully.  That is what is happening in the video you posted; there is definitively an induced EMF in the oscilloscope probes.  They are making exactly the same mistake Lewin made!

Watch this video from Trevor Kearney and pay attention to everything what he says.  In particular, since you don't seem to understand how is done, look carefully how he derives the equivalent circuit for the RHS of the setup.  That is what Lewin should have done.

https://youtu.be/FR8k12j7_Eo

And you neither understand Faradays' Law nor do you understand what a path integral is, and in particular what Faradays' Law says about closed paths:

There is no EMF induced in the loops formed by the probe wires and the resistors they connect to, because the paths \$C_1, C_2\$ don't enclose the magnetic flux region. This is a claim anyone can easily verify: Just take your oscilloscope probe, connect the ground lead to the tip and move it around the core, see if you can pick up any significant EMF.  Also, it would be quite miraculous how any induced EMF caused by "stray magnetic flux" would exactly make the two voltages reproduce the ratio between the resistors, to conveniently coincide with what is expected through Ohms law.

So, the result of the experiment can be exactly predicted by application of Faraday's Law. You just have to understand it first.
Title: Re: #562 – Electroboom!
Post by: ogden on January 11, 2022, 07:00:20 pm
Okay. So, none of you, neither ogden or jesuscf are able to provide an equivalent circuit that reproduces Dr. Lewins experiment, right? And your whole defence to why you cannot do it boils down to "Lewin is an idiot!" :palm:

Can you read? https://www.eevblog.com/forum/amphour/562-electroboom!/msg3927647/#msg3927647 (https://www.eevblog.com/forum/amphour/562-electroboom!/msg3927647/#msg3927647). Dr.Lewin frustrated himself with overcomplicated experiment, drew wrong conclusions out of measurement results. If you see someone making error - you immediately label him an idiot? Dr.Lewin is brilliant teacher with loads of educating videos, I suggested one for you as well.
Title: Re: #562 – Electroboom!
Post by: ogden on January 11, 2022, 07:13:28 pm
There is no EMF induced in the loops formed by the probe wires and the resistors they connect to, because the paths \$C_1, C_2\$ don't enclose the magnetic flux region. This is a claim anyone can easily verify: Just take your oscilloscope probe, connect the ground lead to the tip and move it around the core, see if you can pick up any significant EMF.  Also, it would be quite miraculous how any induced EMF caused by "stray magnetic flux" would exactly make the two voltages reproduce the ratio between the resistors, to conveniently coincide with what is expected through Ohms law.

Here we go again. One of many fallacies of Lewin's cultists. Think about various values of R2, ranging from 900 Ohms till open (no resistor):

1. R1 = 100 Ohms and R2 = open circuit. Do you agree that EMF is induced in voltmeter leads? You shall. Because essentially there is no R2.
2. R1 = 100 Ohms and R2 = 900 Ohms. Now voltmeter leads miraculously do not get any EMF - as soon as you put resistor of *any* resistance closer to solenoid than Voltmeter?
3. What if you exchange R2 = 900 Ohms and voltmeter?  Then what?
Title: Re: #562 – Electroboom!
Post by: thinkfat on January 11, 2022, 07:48:50 pm
Okay. So, none of you, neither ogden or jesuscf are able to provide an equivalent circuit that reproduces Dr. Lewins experiment, right? And your whole defence to why you cannot do it boils down to "Lewin is an idiot!" :palm:

Can you read? https://www.eevblog.com/forum/amphour/562-electroboom!/msg3927647/#msg3927647 (https://www.eevblog.com/forum/amphour/562-electroboom!/msg3927647/#msg3927647). Dr.Lewin frustrated himself with overcomplicated experiment, drew wrong conclusions out of measurement results. If you see someone making error - you immediately label him an idiot? Dr.Lewin is brilliant teacher with loads of educating videos, I suggested one for you as well.

I specifically asked jesuscf if he thinks Dr. Lewin is an idiot and he confirmed it.
Title: Re: #562 – Electroboom!
Post by: ogden on January 11, 2022, 08:02:24 pm
What you KVLers keep missing is that this emf makes the charge in the probes move and accumulate at the voltmeter's internal resistance and the field that is generated by that charge will cancel the induced field in the probes. Leaving nearly nothing in the probes.

It is huge surprise to hear from you - that emf is indeed generated in the probe leads. Good. As emf generated in probe lead and wire connecting probing point and resistor on same side as voltmeter, are of same polarity - they cancel out meaning  voltage we see on voltmeter is equal to voltage drop on resistor. Glad you can learn too. Congrats!

I specifically asked jesuscf if he thinks Dr. Lewin is an idiot and he confirmed it.
Maybe you catch him out of context? Anyway I strongly disagree (https://youtu.be/77ZF50ve6rs?t=3) :)
Title: Re: #562 – Electroboom!
Post by: thinkfat on January 11, 2022, 08:05:43 pm
There is no EMF induced in the loops formed by the probe wires and the resistors they connect to, because the paths \$C_1, C_2\$ don't enclose the magnetic flux region. This is a claim anyone can easily verify: Just take your oscilloscope probe, connect the ground lead to the tip and move it around the core, see if you can pick up any significant EMF.  Also, it would be quite miraculous how any induced EMF caused by "stray magnetic flux" would exactly make the two voltages reproduce the ratio between the resistors, to conveniently coincide with what is expected through Ohms law.

Here we go again. One of many fallacies of Lewin's cultists. Think about various values of R2, ranging from 900 Ohms till open (no resistor):

1. R1 = 100 Ohms and R2 = open circuit. Do you agree that EMF is induced in voltmeter leads? You shall. Because essentially there is no R2.
2. R1 = 100 Ohms and R2 = 900 Ohms. Now voltmeter leads miraculously do not get any EMF - as soon as you put resistor of *any* resistance closer to solenoid than Voltmeter?
3. What if you exchange R2 = 900 Ohms and voltmeter?  Then what?

You said you understood circuit theory. Then you should know how each loop (mesh) in a circuit is to be evaluated separately, to later sum up all contributions. So, in Dr. Lewins circuit there are three loops, only one of which encloses an area with time-varying magnetic flux. Or do you suddenly want to argue that circuit theory cannot be applied any more? But KVL still works?
Title: Re: #562 – Electroboom!
Post by: ogden on January 11, 2022, 08:26:19 pm
There is no EMF induced in the loops formed by the probe wires and the resistors they connect to, because the paths \$C_1, C_2\$ don't enclose the magnetic flux region. This is a claim anyone can easily verify: Just take your oscilloscope probe, connect the ground lead to the tip and move it around the core, see if you can pick up any significant EMF.  Also, it would be quite miraculous how any induced EMF caused by "stray magnetic flux" would exactly make the two voltages reproduce the ratio between the resistors, to conveniently coincide with what is expected through Ohms law.

Here we go again. One of many fallacies of Lewin's cultists. Think about various values of R2, ranging from 900 Ohms till open (no resistor):

1. R1 = 100 Ohms and R2 = open circuit. Do you agree that EMF is induced in voltmeter leads? You shall. Because essentially there is no R2.
2. R1 = 100 Ohms and R2 = 900 Ohms. Now voltmeter leads miraculously do not get any EMF - as soon as you put resistor of *any* resistance closer to solenoid than Voltmeter?
3. What if you exchange R2 = 900 Ohms and voltmeter?  Then what?

You said you understood circuit theory. Then you should know how each loop (mesh) in a circuit is to be evaluated separately, to later sum up all contributions. So, in Dr. Lewins circuit there are three loops, only one of which encloses an area with time-varying magnetic flux. Or do you suddenly want to argue that circuit theory cannot be applied any more? But KVL still works?

Could be good we first clear question I asked you in my previous post (https://www.eevblog.com/forum/amphour/562-electroboom!/msg3933425/#msg3933425) because it lets you really question "three loops, only one of which encloses an area with time-varying magnetic flux", but it seems you just take it for a granted as usually true believers do. You are just believer or you can think too? Visualization attached for case 3.
Title: Re: #562 – Electroboom!
Post by: thinkfat on January 11, 2022, 08:57:57 pm
There is no EMF induced in the loops formed by the probe wires and the resistors they connect to, because the paths \$C_1, C_2\$ don't enclose the magnetic flux region. This is a claim anyone can easily verify: Just take your oscilloscope probe, connect the ground lead to the tip and move it around the core, see if you can pick up any significant EMF.  Also, it would be quite miraculous how any induced EMF caused by "stray magnetic flux" would exactly make the two voltages reproduce the ratio between the resistors, to conveniently coincide with what is expected through Ohms law.

Here we go again. One of many fallacies of Lewin's cultists. Think about various values of R2, ranging from 900 Ohms till open (no resistor):

1. R1 = 100 Ohms and R2 = open circuit. Do you agree that EMF is induced in voltmeter leads? You shall. Because essentially there is no R2.
2. R1 = 100 Ohms and R2 = 900 Ohms. Now voltmeter leads miraculously do not get any EMF - as soon as you put resistor of *any* resistance closer to solenoid than Voltmeter?
3. What if you exchange R2 = 900 Ohms and voltmeter?  Then what?

You said you understood circuit theory. Then you should know how each loop (mesh) in a circuit is to be evaluated separately, to later sum up all contributions. So, in Dr. Lewins circuit there are three loops, only one of which encloses an area with time-varying magnetic flux. Or do you suddenly want to argue that circuit theory cannot be applied any more? But KVL still works?

Could be good we first clear question I asked you in my previous post (https://www.eevblog.com/forum/amphour/562-electroboom!/msg3933425/#msg3933425) because it lets you really question "three loops, only one of which encloses an area with time-varying magnetic flux", but it seems you just take it for a granted as usually true believers do. You are just believer or you can think too? Visualization attached for case 3.

What's that supposed to change? The loop formed by the voltmeter and R2 still doesn't enclose any time-varying magnetic flux. You'd just have to analyze the circuit a little differently, but the result would stay the same.

PS: I found this video quite enlightening:
https://youtu.be/OmlnGei1xo8 (https://youtu.be/OmlnGei1xo8)
Title: Re: #562 – Electroboom!
Post by: ogden on January 11, 2022, 09:21:38 pm
There is no EMF induced in the loops formed by the probe wires and the resistors they connect to, because the paths \$C_1, C_2\$ don't enclose the magnetic flux region. This is a claim anyone can easily verify: Just take your oscilloscope probe, connect the ground lead to the tip and move it around the core, see if you can pick up any significant EMF.  Also, it would be quite miraculous how any induced EMF caused by "stray magnetic flux" would exactly make the two voltages reproduce the ratio between the resistors, to conveniently coincide with what is expected through Ohms law.

Here we go again. One of many fallacies of Lewin's cultists. Think about various values of R2, ranging from 900 Ohms till open (no resistor):

1. R1 = 100 Ohms and R2 = open circuit. Do you agree that EMF is induced in voltmeter leads? You shall. Because essentially there is no R2.
2. R1 = 100 Ohms and R2 = 900 Ohms. Now voltmeter leads miraculously do not get any EMF - as soon as you put resistor of *any* resistance closer to solenoid than Voltmeter?
3. What if you exchange R2 = 900 Ohms and voltmeter?  Then what?

You said you understood circuit theory. Then you should know how each loop (mesh) in a circuit is to be evaluated separately, to later sum up all contributions. So, in Dr. Lewins circuit there are three loops, only one of which encloses an area with time-varying magnetic flux. Or do you suddenly want to argue that circuit theory cannot be applied any more? But KVL still works?

Could be good we first clear question I asked you in my previous post (https://www.eevblog.com/forum/amphour/562-electroboom!/msg3933425/#msg3933425) because it lets you really question "three loops, only one of which encloses an area with time-varying magnetic flux", but it seems you just take it for a granted as usually true believers do. You are just believer or you can think too? Visualization attached for case 3.

What's that supposed to change? The loop formed by the voltmeter and R2 still doesn't enclose any time-varying magnetic flux. You'd just have to analyze the circuit a little differently, but the result would stay the same.

It was supposed to let you actually think. Please be so kind, spread your wisdom. If as you say wires of outer loop having R2 do not receive any EMF, then I am all ears of your little different analysis for case #3. I am sure many others will be interested to hear it as well.

[edit] The best part of that video is at the end :)
Title: Re: #562 – Electroboom!
Post by: thinkfat on January 11, 2022, 09:39:00 pm
There is no EMF induced in the loops formed by the probe wires and the resistors they connect to, because the paths \$C_1, C_2\$ don't enclose the magnetic flux region. This is a claim anyone can easily verify: Just take your oscilloscope probe, connect the ground lead to the tip and move it around the core, see if you can pick up any significant EMF.  Also, it would be quite miraculous how any induced EMF caused by "stray magnetic flux" would exactly make the two voltages reproduce the ratio between the resistors, to conveniently coincide with what is expected through Ohms law.

Here we go again. One of many fallacies of Lewin's cultists. Think about various values of R2, ranging from 900 Ohms till open (no resistor):

1. R1 = 100 Ohms and R2 = open circuit. Do you agree that EMF is induced in voltmeter leads? You shall. Because essentially there is no R2.
2. R1 = 100 Ohms and R2 = 900 Ohms. Now voltmeter leads miraculously do not get any EMF - as soon as you put resistor of *any* resistance closer to solenoid than Voltmeter?
3. What if you exchange R2 = 900 Ohms and voltmeter?  Then what?

You said you understood circuit theory. Then you should know how each loop (mesh) in a circuit is to be evaluated separately, to later sum up all contributions. So, in Dr. Lewins circuit there are three loops, only one of which encloses an area with time-varying magnetic flux. Or do you suddenly want to argue that circuit theory cannot be applied any more? But KVL still works?

Could be good we first clear question I asked you in my previous post (https://www.eevblog.com/forum/amphour/562-electroboom!/msg3933425/#msg3933425) because it lets you really question "three loops, only one of which encloses an area with time-varying magnetic flux", but it seems you just take it for a granted as usually true believers do. You are just believer or you can think too? Visualization attached for case 3.

What's that supposed to change? The loop formed by the voltmeter and R2 still doesn't enclose any time-varying magnetic flux. You'd just have to analyze the circuit a little differently, but the result would stay the same.

It was supposed to let you actually think. Please be so kind, spread your wisdom. If as you say wires of outer loop having R2 do not receive any EMF, then I am all ears of your little different analysis for case #3. I am sure many others will be interested to hear it as well.

Quote
PS: I found this video quite enlightening:
https://youtu.be/OmlnGei1xo8 (https://youtu.be/OmlnGei1xo8)
You can't say that there is no EMF in the wires of outer loop, at the time time agreeing to this video!  :-DD

What I say is that \$ \oint E \cdot dl = 0\$ for the measurement loop. Remember that the dispute is about Faradays' Law and whether volt meters conncted to the same nodes can show different values, depending on the path of measurement. Nothing else was claimed by Dr. Lewin.
Title: Re: #562 – Electroboom!
Post by: Sredni on January 11, 2022, 09:55:43 pm
I wonder if Ogden is capable of computing and comparing the currents in the loop R1 R2 alone, in the loop R1 voltmeter alone and then in the full circuit with R1 R2 and the voltmeter.

Assume 10 meg for the voltmeter internal resistance.
Title: Re: #562 – Electroboom!
Post by: ogden on January 11, 2022, 10:01:00 pm
What I say is that \$ \oint E \cdot dl = 0\$ for the measurement loop.
Typical tactics of Lewin's team - distraction and/or goalpost shifting.

I wonder if Ogden is capable of computing and comparing the currents in the loop R1 R2 alone, in the loop R1 voltmeter alone and then in the full circuit with R1 R2 and the voltmeter.
Sure. (https://www.eevblog.com/forum/amphour/562-electroboom!/msg3927647/#msg3927647) It's because I state that there is no outer loop which miraculously avoids EM induction so it does not matter - you put voltmeter or resistor on outside loop. I am sure you agree because recently stated that EMF is induced in outer wires as well.
Title: Re: #562 – Electroboom!
Post by: Sredni on January 11, 2022, 10:03:20 pm
Then please by all means
Compute them and compare the three cases
Title: Re: #562 – Electroboom!
Post by: ogden on January 11, 2022, 10:14:08 pm
Then please by all means
Compute them and compare the three cases

Be my guest.
Questions:

Quote
1. R1 = 100 Ohms and R2 = open circuit. Do you agree that EMF is induced in voltmeter leads? You shall. Because essentially there is no R2.
2. R1 = 100 Ohms and R2 = 900 Ohms. Now voltmeter leads miraculously do not get any EMF - as soon as you put resistor of *any* resistance closer to solenoid than Voltmeter?
3. What if you exchange R2 = 900 Ohms and voltmeter?  Then what?

Answers:
1. Voltmeter will indicate value close to EMF, ~1V. I say "close" because R1=100 Ohms way smaller than typical 10 Megs of voltmeters, but still not zero.
2. Essentially Lewin's circuit (https://www.eevblog.com/forum/amphour/562-electroboom!/msg3927647/#msg3927647). Yes indeed EMF is generated induced in all wires & leads.
3. It does not matter where you put R2 - on inside or outside loop. EM induction do not miraculously pick inner loop, it acts on all wires unless they are magnetically shielded from time-varying magnetic field. So Lewin's circuit again (https://www.eevblog.com/forum/amphour/562-electroboom!/msg3927647/#msg3927647).
Title: Re: #562 – Electroboom!
Post by: thinkfat on January 11, 2022, 10:25:10 pm
What I say is that \$ \oint E \cdot dl = 0\$ for the measurement loop.
Typical tactics of Lewin's team - distraction and/or goalpost shifting.

Huh? I've never claimed anything but what is obvious from the Faraday-Maxwell equation (\$\oint E \cdot dl = -\frac{dB}{dt}\$) and this one only ever makes statements about closed paths. It was jesuscf who started the whole mess with scalar PD and voltages that are supposedly unique, but only until you measure them, when they suddenly become path dependent again.
Title: Re: #562 – Electroboom!
Post by: Sredni on January 11, 2022, 10:25:59 pm
@ogden

So you are not able to compute the current in those three simple circuits?

And no, emf is not 'generated in wires'. The induced electric field is in all space, but when you put a good conductor in that space, the free charge inside will start to move, and things change.

Do you think charge will cease to generate an electric field when you move it?
Title: Re: #562 – Electroboom!
Post by: ogden on January 11, 2022, 10:38:28 pm
So you are not able to compute the current in those three simple circuits?

I can try, but what's the point. Really. Explain please your aim. What will do calculation of current running through 10 MOhm voltmeters measuring 1mA-capable circuit other than pissing me off? Whatever. You can claim your victory. I wont waste time for stupid requests anyway. Take care.
Title: Re: #562 – Electroboom!
Post by: thinkfat on January 11, 2022, 10:47:09 pm
I specifically asked jesuscf if he thinks Dr. Lewin is an idiot and he confirmed it.
Maybe you catch him out of context? Anyway I strongly disagree (https://youtu.be/77ZF50ve6rs?t=3) :)

Hm.

My question would be: do you really believe that Dr. Lewin is such a massive fool that he didn't understand that the volt meters (or oscilloscopes) he used and the paths they were connected through were part of the circuit?

Yes.
Title: Re: #562 – Electroboom!
Post by: ogden on January 11, 2022, 10:52:59 pm
I specifically asked jesuscf if he thinks Dr. Lewin is an idiot and he confirmed it.
Maybe you catch him out of context? Anyway I strongly disagree (https://youtu.be/77ZF50ve6rs?t=3) :)
Hm.
I strongly disagree with jesuscf in this regard. Hopefully now this is clear and you can stop to bring this up. Better watch some Dr.Lewin's video. [edit] Wait.. I already stated that Dr.Lewin is brilliant teacher yet you are trying to prove something  :wtf:
Title: Re: #562 – Electroboom!
Post by: jesuscf on January 11, 2022, 10:56:25 pm
Okay. So, none of you, neither ogden or jesuscf are able to provide an equivalent circuit that reproduces Dr. Lewins experiment, right? And your whole defence to why you cannot do it boils down to "Lewin is an idiot!" :palm:

Can you read? https://www.eevblog.com/forum/amphour/562-electroboom!/msg3927647/#msg3927647 (https://www.eevblog.com/forum/amphour/562-electroboom!/msg3927647/#msg3927647). Dr.Lewin frustrated himself with overcomplicated experiment, drew wrong conclusions out of measurement results. If you see someone making error - you immediately label him an idiot? Dr.Lewin is brilliant teacher with loads of educating videos, I suggested one for you as well.

I specifically asked jesuscf if he thinks Dr. Lewin is an idiot and he confirmed it.

I don't remember saying that.  Can point me where I said it?

EDIT: I see it.  You can interpret that in many ways, but I didn't say Lewin is an idiot.  I just said 'yes' to your convoluted question.  But anyhow, believe what you want, that doesn't change reality.  Also, just for the record 'fool' != 'idiot'.
Title: Re: #562 – Electroboom!
Post by: jesuscf on January 12, 2022, 01:20:07 am
Okay. So, none of you, neither ogden or jesuscf are able to provide an equivalent circuit that reproduces Dr. Lewins experiment, right? And your whole defence to why you cannot do it boils down to "Lewin is an idiot!" :palm:

PS: before you go about bringing "stray magnetic flux" into the game again, watch this video maybe:

https://youtu.be/u6ud7JD0fV4 (https://youtu.be/u6ud7JD0fV4)

Ring core transformer, magnetic flux well confined inside the core. But the outcome is the same.

You don't understand how Faraday's law work!  :-DD  If the circuit formed by the probes of your instrument go around the 'confined' varying magnetic core, there is going to be an induced EMF in the probes unless you align the probes carefully.

What you KVLers keep missing is that this emf makes the charge in the probes move and accumulate at the voltmeter's internal resistance and the field that is generated by that charge will cancel the induced field in the probes. Leaving nearly nothing in the probes.

Quote
  That is what is happening in the video you posted; there is definitively an induced EMF in the oscilloscope probes.  They are making exactly the same mistake Lewin made!

Sure. The MIT has always been a nest of incompetent physicists and engineers.
They should hire you, instead!

Quote
Watch this video from Trevor Kearney and pay attention to everything what he says.  In particular, since you don't seem to understand how is done, look carefully how he derives the equivalent circuit for the RHS of the setup.  That is what Lewin should have done.

https://youtu.be/FR8k12j7_Eo (https://youtu.be/FR8k12j7_Eo)

Did you ask Trevor if he thinks Lewin is in error?
Are you afraid of asking?
Because I am pretty sure you are misunderstanding his point of view.

Why should I ask Trevor Kearney a question he has already answer in his videos?  You should go and watch his videos and try to understand them!  Once you have done that, you'll know for sure who is correct.

Also, do you remember that time you asked me if I had a toroid?  Well, I have one now!  This beauty just arrived:

(https://www.eevblog.com/forum/amphour/562-electroboom!/?action=dlattach;attach=1376789;image)
Title: Re: #562 – Electroboom!
Post by: jesuscf on January 12, 2022, 03:48:46 am
Okay. So, none of you, neither ogden or jesuscf are able to provide an equivalent circuit that reproduces Dr. Lewins experiment, right? And your whole defence to why you cannot do it boils down to "Lewin is an idiot!" :palm:

PS: before you go about bringing "stray magnetic flux" into the game again, watch this video maybe:

https://youtu.be/u6ud7JD0fV4 (https://youtu.be/u6ud7JD0fV4)

Ring core transformer, magnetic flux well confined inside the core. But the outcome is the same.

Here, allow me to debunk the MIT guys from the video link you posted above.

With the toroid core I just received, I prepared a setup similar to the one in the video.  Here is a sketch of the circuit:

(https://www.eevblog.com/forum/amphour/562-electroboom!/?action=dlattach;attach=1376876;image)

Instead of making a hole through the toroid to pass a wire to measure the voltage from node A to D without an induced EMF in the voltmeter probes, I used the balanced circuit composed by the two 10kOhm resistors.  Whatever voltage is induced in the left 10kOhm resistor is canceled by the voltage induced in the right 10kOhm resistor and the net EMF induced in the red probe wire is zero.  The total measured EMF is 67.6mV (RMS).  The equivalent circuit we need to solve is (sorry, the polarity of the sources is swapped in the circuit diagram, so fix the polarity before getting the equations):

(https://www.eevblog.com/forum/amphour/562-electroboom!/?action=dlattach;attach=1376882;image)

As usual, calculate the current first:

\$
\begin{array}{l}
 EMF = 67.6mV \\
 I = \frac{{67.6mV}}{{100\Omega  + 910\Omega }} = 66.93\mu A \\
 \end{array}
\$

With the current we can calculate the voltage drop in the resistors:

\$
\begin{array}{l}
 V_{R1}  = 66.93\mu A \times 100\Omega  = 6.693mV \\
 V_{R2}  = 66.93\mu A \times 910\Omega  = 60.91mV \\
 \end{array}
\$

Using the left half or the right half of the circuit we can calculate the voltage VAD (there is an error in the circuit diagram, the polarity of the sources is the other way around):

\$
\begin{array}{l}
 V_{AD}  = 33.8mV - 66.93\mu A \times 100\Omega  = 27.11mV \\
 V_{AD}  = 66.93\mu A \times 910\Omega  - 33.8mV = 27.11mV \\
 \end{array}
\$

These are the voltages I measured:

\$
\begin{array}{l}
 V_{R1}  = 6.44mV \\
 V_{R2}  = 61.15mV \\
 V_{AD}  = 27.16mV \\
 \end{array}
\$

Look at that: KVL works again!!!  :-DD

Here is a picture of the measured VR1 and VR2 voltages:

(https://www.eevblog.com/forum/amphour/562-electroboom!/?action=dlattach;attach=1376888;image)

Here is a picture of the measured VAD voltage:

(https://www.eevblog.com/forum/amphour/562-electroboom!/?action=dlattach;attach=1376894;image)

Here is a view of the setup from the left (yes, those are chopsticks):

(https://www.eevblog.com/forum/amphour/562-electroboom!/?action=dlattach;attach=1376900;image)

Here is a view of the setup from the right:

(https://www.eevblog.com/forum/amphour/562-electroboom!/?action=dlattach;attach=1376906;image)

Hey team Lewin, show us your experiments please!  We are waiting!




Title: Re: #562 – Electroboom!
Post by: ogden on January 12, 2022, 06:43:03 am
Nice job! Appreciate your work and dedication. Especially nice is 2*10K solution which could spark new controversy - because it is "inner loop" 8)

Meanwhile those who are not yet made their mind, can read detailed article about controversy, Do not miss to watch all the videos linked:
https://www.electronicdesign.com/technologies/analog/article/21808474/when-kirchhoffs-law-doesnt-work (https://www.electronicdesign.com/technologies/analog/article/21808474/when-kirchhoffs-law-doesnt-work)

For those who want to repeat experiment - many toroidal inductors out of your scrap box can be ok for the job.
Title: Re: #562 – Electroboom!
Post by: bsfeechannel on January 12, 2022, 07:47:02 am
Look at that: KVL works again!!!  :-DD

Yes. KVL holds for meshes I and II,  because they do not encircle a varying magnetic flux anymore, but you created a third mesh and behold there KVL fails. Guess why. Besides, points A and D still exhibit different voltages as clearly shown by your meters, which means that along the closed path of R1 an R2, the  voltages still do not add up to zero and KVL fails again.
(https://www.eevblog.com/forum/amphour/562-electroboom!/?action=dlattach;attach=1376999;image)
Kirchhoff hates you.
Title: Re: #562 – Electroboom!
Post by: Sredni on January 12, 2022, 09:06:38 am
Okay. So, none of you, neither ogden or jesuscf are able to provide an equivalent circuit that reproduces Dr. Lewins experiment, right? And your whole defence to why you cannot do it boils down to "Lewin is an idiot!" :palm:

PS: before you go about bringing "stray magnetic flux" into the game again, watch this video maybe:

https://youtu.be/u6ud7JD0fV4 (https://youtu.be/u6ud7JD0fV4)

Ring core transformer, magnetic flux well confined inside the core. But the outcome is the same.

You don't understand how Faraday's law work!  :-DD  If the circuit formed by the probes of your instrument go around the 'confined' varying magnetic core, there is going to be an induced EMF in the probes unless you align the probes carefully.

What you KVLers keep missing is that this emf makes the charge in the probes move and accumulate at the voltmeter's internal resistance and the field that is generated by that charge will cancel the induced field in the probes. Leaving nearly nothing in the probes.

Quote
  That is what is happening in the video you posted; there is definitively an induced EMF in the oscilloscope probes.  They are making exactly the same mistake Lewin made!

Sure. The MIT has always been a nest of incompetent physicists and engineers.
They should hire you, instead!

Quote
Watch this video from Trevor Kearney and pay attention to everything what he says.  In particular, since you don't seem to understand how is done, look carefully how he derives the equivalent circuit for the RHS of the setup.  That is what Lewin should have done.

https://youtu.be/FR8k12j7_Eo (https://youtu.be/FR8k12j7_Eo)

Did you ask Trevor if he thinks Lewin is in error?
Are you afraid of asking?
Because I am pretty sure you are misunderstanding his point of view.

Why should I ask Trevor Kearney a question he has already answer in his videos?  You should go and watch his videos and try to understand them!  Once you have done that, you'll know for sure who is correct.

Also, do you remember that time you asked me if I had a toroid?  Well, I have one now!  This beauty just arrived:

(https://www.eevblog.com/forum/amphour/562-electroboom!/?action=dlattach;attach=1376789;image)

Good. So now you can tell me what are the voltages in the arcs of the ring. I mean from A to R1, from R1 to D, from D to R2, and from R2 to A.
Title: Re: #562 – Electroboom!
Post by: thinkfat on January 12, 2022, 10:48:13 am
Okay. So, none of you, neither ogden or jesuscf are able to provide an equivalent circuit that reproduces Dr. Lewins experiment, right? And your whole defence to why you cannot do it boils down to "Lewin is an idiot!" :palm:

Can you read? https://www.eevblog.com/forum/amphour/562-electroboom!/msg3927647/#msg3927647 (https://www.eevblog.com/forum/amphour/562-electroboom!/msg3927647/#msg3927647). Dr.Lewin frustrated himself with overcomplicated experiment, drew wrong conclusions out of measurement results. If you see someone making error - you immediately label him an idiot? Dr.Lewin is brilliant teacher with loads of educating videos, I suggested one for you as well.

I specifically asked jesuscf if he thinks Dr. Lewin is an idiot and he confirmed it.

I don't remember saying that.  Can point me where I said it?

EDIT: I see it.  You can interpret that in many ways, but I didn't say Lewin is an idiot.  I just said 'yes' to your convoluted question.  But anyhow, believe what you want, that doesn't change reality.  Also, just for the record 'fool' != 'idiot'.

Convoluted, huh? The question was really just what you quoted in your response. I think you understood it quite well. What lead to the question was you quite correctly stating that connecting a measurement instrument to a circuit changes the circuit. So I asked if you thought Lewin was such a "massive fool" that he didn't understand this.

You answering "Yes" means you hold Lewin to be "massive fool who doesn't understand his own experiment". Which is of course completely, entirely different from calling him an idiot.
Title: Re: #562 – Electroboom!
Post by: thinkfat on January 12, 2022, 11:10:41 am
...

What do you mean with "debunked the MIT guys"? Apparently you have perfectly recreated their experiment. VR1 and VR2 still show different voltages and they are not going to change when you connect your volt meters directly to A and D instead of to the resistor terminals. So, now you have three voltages for VAD, on three different paths. I'd call that a success.
Title: Re: #562 – Electroboom!
Post by: Sredni on January 12, 2022, 01:43:48 pm
Nice job! Appreciate your work and dedication. Especially nice is 2*10K solution which could spark new controversy - because it is "inner loop" 8)

Meanwhile those who are not yet made their mind, can read detailed article about controversy, Do not miss to watch all the videos linked:
https://www.electronicdesign.com/technologies/analog/article/21808474/when-kirchhoffs-law-doesnt-work (https://www.electronicdesign.com/technologies/analog/article/21808474/when-kirchhoffs-law-doesnt-work)

For those who want to repeat experiment - many toroidal inductors out of your scrap box can be ok for the job.

I was well aware of this article, and as much as I like Rako's articles on EDN, he is dead wrong in thinking this is a probing issue.
He should know better, because he knows how to compute path integrals - unlike the vast majority of the garage gang.
At the end he quotes one of the comments where someone tries to explains where he is wrong, but he seems not to have anything to say about it. I wonder why.

Also, really bad taste in suggesting that Lewin's personal life could mean anything about the physics he discusses.
The only error Lewin committed in his videos are slips of the tongue regarding terminology. He sometimes said potential difference instead of voltage (he recently changed his boilerplate to fix that, upon suggestions by Mick Vall) and then there's that thing about the emf: in the ring with lumped resistors the induced Eind field gets obliterated in the conductive arc, while the field inside the resistors' adds up to a voltage equivalent to the emf linked by the circular path. That's why Lewin says the emf is at the resistors.

Chances are, Rako gets partial inductance wrong, as well. (And I could make an ever bigger name who does get that wrong, but I'm keeping it to myself)
Title: Re: #562 – Electroboom!
Post by: HuronKing on January 12, 2022, 02:33:44 pm

Also, really bad taste in suggesting that Lewin's personal life could mean anything about the physics he discusses.
The only error Lewin committed in his videos are slips of the tongue regarding terminology.

It's really freaking weird too how these types of articles and comments focus so much on Lewin's politeness, tact, 'arrogance,' blah blah blah, or lack thereof, or whatever.

Meanwhile, I've met many of these same kind of engineers who pride themselves on their "blunt tactlessness" and "telling it like it is, man!"

And in the annals of the history of physics and engineering, we have quite a few characters who were pretty famously cantankerous or wryly abrasive but no less absolutely correct  - like Heaviside.

Lastly, it's also really weird how much attention is paid to how funny ElectroBoom is and how not-funny Lewin was... despite Lewin developing a reputation as one of the most entertaining physics educators ever to teach at MIT. Clowning in his lectures and telling jokes is trademark of his style. In fact, NONE of this would've even been controversial if he hadn't uttered the phrase 'Kirchhoff is for the birds and Faraday is not.'

I betcha if he had hidden the truth behind the calculus and jargon no one who doesn't understand path-dependent line integrals and non-conservative fields would've even noticed.
Title: Re: #562 – Electroboom!
Post by: jesuscf on January 12, 2022, 03:05:17 pm
Nice job! Appreciate your work and dedication. Especially nice is 2*10K solution which could spark new controversy - because it is "inner loop" 8)

Meanwhile those who are not yet made their mind, can read detailed article about controversy, Do not miss to watch all the videos linked:
https://www.electronicdesign.com/technologies/analog/article/21808474/when-kirchhoffs-law-doesnt-work (https://www.electronicdesign.com/technologies/analog/article/21808474/when-kirchhoffs-law-doesnt-work)

For those who want to repeat experiment - many toroidal inductors out of your scrap box can be ok for the job.

I am very thankful for your appreciation.  The reason I bought the toroid was to test the 2*10k circuit idea; it works really well.

Yes, I read the Electronic Design article some time ago.  It is good you posted the link so other people can get the full picture of this controversy.

I looked and looked for a big toroid with circular cross section, but I couldn't find any.  Decide to buy the one with rectangular cross are instead, which worked out OK for the setup.  With smaller toroids it could be difficult to solder the inner connecting wire.
Title: Re: #562 – Electroboom!
Post by: jesuscf on January 12, 2022, 03:08:15 pm
Look at that: KVL works again!!!  :-DD

Yes. KVL holds for meshes I and II,  because they do not encircle a varying magnetic flux anymore, but you created a third mesh and behold there KVL fails. Guess why. Besides, points A and D still exhibit different voltages as clearly shown by your meters, which means that along the closed path of R1 an R2, the  voltages still do not add up to zero and KVL fails again.
(https://www.eevblog.com/forum/amphour/562-electroboom!/?action=dlattach;attach=1376999;image)
Kirchhoff hates you.

I see you don't know how mesh analysis work either.  Lewin has a video on that, well, actually more like 5 videos on that.  So go watch them then come back to what you said above and see if you can find where you are wrong.
Title: Re: #562 – Electroboom!
Post by: jesuscf on January 12, 2022, 03:11:54 pm
Okay. So, none of you, neither ogden or jesuscf are able to provide an equivalent circuit that reproduces Dr. Lewins experiment, right? And your whole defence to why you cannot do it boils down to "Lewin is an idiot!" :palm:

PS: before you go about bringing "stray magnetic flux" into the game again, watch this video maybe:

https://youtu.be/u6ud7JD0fV4 (https://youtu.be/u6ud7JD0fV4)

Ring core transformer, magnetic flux well confined inside the core. But the outcome is the same.

You don't understand how Faraday's law work!  :-DD  If the circuit formed by the probes of your instrument go around the 'confined' varying magnetic core, there is going to be an induced EMF in the probes unless you align the probes carefully.

What you KVLers keep missing is that this emf makes the charge in the probes move and accumulate at the voltmeter's internal resistance and the field that is generated by that charge will cancel the induced field in the probes. Leaving nearly nothing in the probes.

Quote
  That is what is happening in the video you posted; there is definitively an induced EMF in the oscilloscope probes.  They are making exactly the same mistake Lewin made!

Sure. The MIT has always been a nest of incompetent physicists and engineers.
They should hire you, instead!

Quote
Watch this video from Trevor Kearney and pay attention to everything what he says.  In particular, since you don't seem to understand how is done, look carefully how he derives the equivalent circuit for the RHS of the setup.  That is what Lewin should have done.

https://youtu.be/FR8k12j7_Eo (https://youtu.be/FR8k12j7_Eo)

Did you ask Trevor if he thinks Lewin is in error?
Are you afraid of asking?
Because I am pretty sure you are misunderstanding his point of view.

Why should I ask Trevor Kearney a question he has already answer in his videos?  You should go and watch his videos and try to understand them!  Once you have done that, you'll know for sure who is correct.

Also, do you remember that time you asked me if I had a toroid?  Well, I have one now!  This beauty just arrived:

(https://www.eevblog.com/forum/amphour/562-electroboom!/?action=dlattach;attach=1376789;image)

Good. So now you can tell me what are the voltages in the arcs of the ring. I mean from A to R1, from R1 to D, from D to R2, and from R2 to A.

VA_to_R1=16.9mV
VR1_to_D=16.9mV
VD_to_R2=16.9mV
VR2_to_A=16.9mV
Title: Re: #562 – Electroboom!
Post by: jesuscf on January 12, 2022, 03:14:41 pm
Okay. So, none of you, neither ogden or jesuscf are able to provide an equivalent circuit that reproduces Dr. Lewins experiment, right? And your whole defence to why you cannot do it boils down to "Lewin is an idiot!" :palm:

Can you read? https://www.eevblog.com/forum/amphour/562-electroboom!/msg3927647/#msg3927647 (https://www.eevblog.com/forum/amphour/562-electroboom!/msg3927647/#msg3927647). Dr.Lewin frustrated himself with overcomplicated experiment, drew wrong conclusions out of measurement results. If you see someone making error - you immediately label him an idiot? Dr.Lewin is brilliant teacher with loads of educating videos, I suggested one for you as well.

I specifically asked jesuscf if he thinks Dr. Lewin is an idiot and he confirmed it.

I don't remember saying that.  Can point me where I said it?

EDIT: I see it.  You can interpret that in many ways, but I didn't say Lewin is an idiot.  I just said 'yes' to your convoluted question.  But anyhow, believe what you want, that doesn't change reality.  Also, just for the record 'fool' != 'idiot'.

Convoluted, huh? The question was really just what you quoted in your response. I think you understood it quite well. What lead to the question was you quite correctly stating that connecting a measurement instrument to a circuit changes the circuit. So I asked if you thought Lewin was such a "massive fool" that he didn't understand this.

You answering "Yes" means you hold Lewin to be "massive fool who doesn't understand his own experiment". Which is of course completely, entirely different from calling him an idiot.

I see you are using the "look a squirrel!" fallacy.  (Sorry, I forgot the formal name of the fallacy)
Title: Re: #562 – Electroboom!
Post by: jesuscf on January 12, 2022, 03:21:41 pm
...

What do you mean with "debunked the MIT guys"? Apparently you have perfectly recreated their experiment. VR1 and VR2 still show different voltages and they are not going to change when you connect your volt meters directly to A and D instead of to the resistor terminals. So, now you have three voltages for VAD, on three different paths. I'd call that a success.

There is only one correctly measured VAD, which matches the KVL computation.  The other two measurements are not for VAD;  one is for VR1 and the other is for VR2.   Saying otherwise is incorrect.

VAD!=VR1
VAD!=VR2

Title: Re: #562 – Electroboom!
Post by: Sredni on January 12, 2022, 03:26:42 pm

Good. So now you can tell me what are the voltages in the arcs of the ring. I mean from A to R1, from R1 to D, from D to R2, and from R2 to A.

VA_to_R1=16.9mV
VR1_to_D=16.9mV
VD_to_R2=16.9mV
VR2_to_A=16.9mV

That's funny. You seem to think that the induced electric field is rotationally symmetric around the square section center and perfectly centered with your supposedly perfectly circular ring.

That's also expected.

And it's wrong.

Did you not pay attention to Trevor Kearney's video?

EDIT: in case you are wondering about why distinguish between potential difference and voltage, maybe you can open Hayt, and see if the seventh edition has at the beginning of chapter 9 the following sentence

Quote from: Hayt
The departure from static results is clearly shown by (3), for an electric field intensity resulting from a static charge distribution must lead to zero potential difference about a closed path. In electrostatics, the line integral leads to a potential difference; with time-varying fields, the result is an emf or a voltage.

and try to understand why they make this distinction.
Title: Re: #562 – Electroboom!
Post by: bsfeechannel on January 12, 2022, 03:36:45 pm
Meanwhile those who are not yet made their mind, can read detailed article about controversy, Do not miss to watch all the videos linked:
https://www.electronicdesign.com/technologies/analog/article/21808474/when-kirchhoffs-law-doesnt-work (https://www.electronicdesign.com/technologies/analog/article/21808474/when-kirchhoffs-law-doesnt-work)

This is old news. We've already debunked Mehdi, Mabilde and a string of people who fall for the stupid claims perpetrated by the sick minds of KVLiars.
Title: Re: #562 – Electroboom!
Post by: thinkfat on January 12, 2022, 03:45:33 pm
I see you are using the "look a squirrel!" fallacy.  (Sorry, I forgot the formal name of the fallacy)

Here:
https://en.wikipedia.org/wiki/List_of_fallacies

I'm sure you can pull something out of your nose.
Title: Re: #562 – Electroboom!
Post by: jesuscf on January 12, 2022, 04:00:34 pm
I see you are using the "look a squirrel!" fallacy.  (Sorry, I forgot the formal name of the fallacy)

Here:
https://en.wikipedia.org/wiki/List_of_fallacies

I'm sure you can pull something out of your nose.

Sure I did:

"Fallacy of quoting out of context (contextotomy, contextomy; quotation mining) – selective excerpting of words from their original context to distort the intended meaning."

Thanks.
Title: Re: #562 – Electroboom!
Post by: bsfeechannel on January 12, 2022, 04:40:45 pm

Also, really bad taste in suggesting that Lewin's personal life could mean anything about the physics he discusses.
The only error Lewin committed in his videos are slips of the tongue regarding terminology.

It's really freaking weird too how these types of articles and comments focus so much on Lewin's politeness, tact, 'arrogance,' blah blah blah, or lack thereof, or whatever.

Meanwhile, I've met many of these same kind of engineers who pride themselves on their "blunt tactlessness" and "telling it like it is, man!"

And in the annals of the history of physics and engineering, we have quite a few characters who were pretty famously cantankerous or wryly abrasive but no less absolutely correct  - like Heaviside.

Lastly, it's also really weird how much attention is paid to how funny ElectroBoom is and how not-funny Lewin was... despite Lewin developing a reputation as one of the most entertaining physics educators ever to teach at MIT. Clowning in his lectures and telling jokes is trademark of his style. In fact, NONE of this would've even been controversial if he hadn't uttered the phrase 'Kirchhoff is for the birds and Faraday is not.'

I betcha if he had hidden the truth behind the calculus and jargon no one who doesn't understand path-dependent line integrals and non-conservative fields would've even noticed.

The article also gives too much importance to popularity, which is not a very good parameter to ascertain the techical credibility of an engineer or a scientist.

(https://encrypted-tbn0.gstatic.com/images?q=tbn:ANd9GcT3aTabQrRyChwC7v17fgANsdck0V3X6czw0g&usqp=CAU)

As for politeness, I like  one quote (https://lkml.org/lkml/2012/7/6/495) from Linus Torvalds who once suggested that stupid people should be "retroactively aborted" as they wouldn't be likely able to survive babyhood due to their inability "to find a tit to suck on".

He justified his behavior by saying that he saw political correctness destroy multiple projects out there and he didn't want that for Linux kernel.

I've been in the industry long enough to understand what he's talking about.

But since KVLiars love stupidity, and they fail every time, like KVL in a circuit under a varying magnetic field, I tend to belive that they are also incapable of satisfying even their most basic instincts.

Perhaps one redeeming point of the article is that the author admits in the end that KL fail sometimes, contradicting Mehdi, who stupidly asserts that "Kirchhoff always holds".

Title: Re: #562 – Electroboom!
Post by: thinkfat on January 12, 2022, 04:51:49 pm
I see you are using the "look a squirrel!" fallacy.  (Sorry, I forgot the formal name of the fallacy)

Here:
https://en.wikipedia.org/wiki/List_of_fallacies

I'm sure you can pull something out of your nose.

Sure I did:

"Fallacy of quoting out of context (contextotomy, contextomy; quotation mining) – selective excerpting of words from their original context to distort the intended meaning."

Thanks.

Given that I explicitly provided the context of the quote to be fully transparent - yep, that is indeed from your nose.

Also, arguing that "massive fool != idiot" -> Logic chopping.
Title: Re: #562 – Electroboom!
Post by: jesuscf on January 12, 2022, 06:08:26 pm

Also, really bad taste in suggesting that Lewin's personal life could mean anything about the physics he discusses.
The only error Lewin committed in his videos are slips of the tongue regarding terminology.

It's really freaking weird too how these types of articles and comments focus so much on Lewin's politeness, tact, 'arrogance,' blah blah blah, or lack thereof, or whatever.

Meanwhile, I've met many of these same kind of engineers who pride themselves on their "blunt tactlessness" and "telling it like it is, man!"

And in the annals of the history of physics and engineering, we have quite a few characters who were pretty famously cantankerous or wryly abrasive but no less absolutely correct  - like Heaviside.

Lastly, it's also really weird how much attention is paid to how funny ElectroBoom is and how not-funny Lewin was... despite Lewin developing a reputation as one of the most entertaining physics educators ever to teach at MIT. Clowning in his lectures and telling jokes is trademark of his style. In fact, NONE of this would've even been controversial if he hadn't uttered the phrase 'Kirchhoff is for the birds and Faraday is not.'

I betcha if he had hidden the truth behind the calculus and jargon no one who doesn't understand path-dependent line integrals and non-conservative fields would've even noticed.

The article also gives too much importance to popularity, which is not a very good parameter to ascertain the techical credibility of an engineer or a scientist.

(https://encrypted-tbn0.gstatic.com/images?q=tbn:ANd9GcT3aTabQrRyChwC7v17fgANsdck0V3X6czw0g&usqp=CAU)

As for politeness, I like  one quote (https://lkml.org/lkml/2012/7/6/495) from Linus Torvalds who once suggested that stupid people should be "retroactively aborted" as they wouldn't be likely able to survive babyhood due to their inability "to find a tit to suck on".

He justified his behavior by saying that he saw political correctness destroy multiple projects out there and he didn't want that for Linux kernel.

I've been in the industry long enough to understand what he's talking about.

But since KVLiars love stupidity, and they fail every time, like KVL in a circuit under a varying magnetic field, I tend to belive that they are also incapable of satisfying even their most basic instincts.

Perhaps one redeeming point of the article is that the author admits in the end that KL fail sometimes, contradicting Mehdi, who stupidly asserts that "Kirchhoff always holds".

Reading bsfeechannel reminds me of Nathan Thompson.

Reading Sredni reminds me of Nathan Oakley.
Title: Re: #562 – Electroboom!
Post by: Sredni on January 12, 2022, 06:25:47 pm
Reading books is what you should do, instead..
Start from Hayt, chapter 9 and try to understand why the author's differentiate between potential difference and voltage.

The books in your picture look alarmingly unread.
Title: Re: #562 – Electroboom!
Post by: ogden on January 12, 2022, 07:22:52 pm
Engineer: "That model does not look right"
Scientist: "Eureka!"

When engineer say "you have to place wire here to get voltage X but have to place wire there to get voltage Y"- he gets fired.
When scientist say exactly same words - he gets worshipped.
Go figure.
Title: Re: #562 – Electroboom!
Post by: bsfeechannel on January 12, 2022, 11:28:00 pm
I see you don't know how mesh analysis work either.

Translation: you got me again, damn it!
Title: Re: #562 – Electroboom!
Post by: jesuscf on January 13, 2022, 12:25:33 am
Reading books is what you should do, instead..
Start from Hayt, chapter 9 and try to understand why the author's differentiate between potential difference and voltage.

Got it:  Magnetic potential difference units are amperes.  Electric potential difference units are volts.

The books in your picture look alarmingly unread.

Maxwell's book is new.  As for Hayt's book:

(https://www.eevblog.com/forum/amphour/562-electroboom!/?action=dlattach;attach=1377895;image)

Title: Re: #562 – Electroboom!
Post by: bsfeechannel on January 13, 2022, 12:40:08 am
Reading books is what you should do, instead..
Start from Hayt, chapter 9 and try to understand why the author's differentiate between potential difference and voltage.

The books in your picture look alarmingly unread.

Do you expect them to read books and on top of that understand what they read? You're asking too much. KVLism is based on ignorance. The more you're ignorant, the better. Because then they can claim with even more propriety that they have "debunked" some reputable scientist, engineer or scholar.

When they resort to books, it is only to select the passages that, out of context, appear to support their false claims, and rig their experiments until they get the results they think will prove them "right".

The books they show are only to impress the casual reader of the thread, or to serve as an amulet to ward off the evil people that show their favorite ignorant blogger is fundamentally wrong.
Title: Re: #562 – Electroboom!
Post by: jesuscf on January 13, 2022, 12:42:27 am
Engineer: "That model does not look right"
Scientist: "Eureka!"

When engineer say "you have to place wire here to get voltage X but have to place wire there to get voltage Y"- he gets fired.
When scientist say exactly same words - he gets worshipped.
Go figure.

That is so true!  :-DD
Title: Re: #562 – Electroboom!
Post by: jesuscf on January 13, 2022, 12:45:08 am
Reading books is what you should do, instead..
Start from Hayt, chapter 9 and try to understand why the author's differentiate between potential difference and voltage.

The books in your picture look alarmingly unread.

Do you expect them to read books and on top of that understand what they read? You're asking too much. KVLism is based on ignorance. The more you're ignorant, the better. Because then they can claim with even more propriety that they have "debunked" some reputable scientist, engineer or scholar.

When they resort to books, it is only to select the passages that, out of context, appear to support their false claims, and rig their experiments until they get the results they think will prove them "right".

The books they show are only to impress the casual reader of the thread, or to serve as an amulet to ward off the evil people that show their favorite ignorant blogger is fundamentally wrong.

Have you learn how to apply mesh analysis yet?  Do you want any book recommendations?
Title: Re: #562 – Electroboom!
Post by: Sredni on January 13, 2022, 01:04:00 am
Reading books is what you should do, instead..
Start from Hayt, chapter 9 and try to understand why the author's differentiate between potential difference and voltage.

Got it:  Magnetic potential difference units are amperes.  Electric potential difference units are volts.


Non sequitur and mutatio controversiae in a single sentence. Not bad.
But try again.

Magnetic potential difference has nothing to do with the separation of the concepts of
    - electric potential difference
    - electric voltage

Quote

Maxwell's book is new.  As for Hayt's book:

(https://www.eevblog.com/forum/amphour/562-electroboom!/?action=dlattach;attach=1377895;image)

They still look alarming unread.
You have four different editions of Hayt and you did not find any reference on a certain closed path integral being zero in one case, and being nonzero in the other?

Did you have to buy four edition because your books consistently miss pages?
Title: Re: #562 – Electroboom!
Post by: bsfeechannel on January 13, 2022, 02:56:19 am
Did you have to buy four edition because your books consistently miss pages?

You're being too polite. What they miss is parts of their brains.
Title: Re: #562 – Electroboom!
Post by: bsfeechannel on January 13, 2022, 04:03:07 am
I see you don't know how mesh analysis work either.  Lewin has a video on that, well, actually more like 5 videos on that.  So go watch them then come back to what you said above and see if you can find where you are wrong.

Well, I learned mesh analysis in the 1970's when I had my first formal training in electronics. All my teachers were engineers. In the last 50 years or so things may have changed. I don't know, they may have changed the definition of voltage, Kirchhoff's laws, Faraday's law, who knows?

I'll check Lewin's lectures about that to see if I'm still up to date, thank you.

While I do that, let's talk about your circuit, which by the way is not Lewin's circuit. I took the liberty to edit one of your pictures so that we can understand what is going on with it.

(https://www.eevblog.com/forum/amphour/562-electroboom!/?action=dlattach;attach=1378054;image)

In the picture you see four stickmen seen from above (they have ears) extending their arms along the various paths of your circuit. The red dotted line shows what is hidden by the toroid. The yellow guys are the ones that, when stretching their arms, end up "embracing" the area where the varying magnetic field B is. The guys in blue don't.

Let's take the voltages along the paths of the yellow guys first. According to your measurements, VR1= 6.44 mV, and VR2=61.15 mV. These resistors form a complete circuit and their voltages add up to 6.44 mV + 61.15 mV = 67.59 mV. You haven't measured the voltages across the two 10k resistors of the inner ring, but we can presume that the voltage on each of them will be 33.8 mV, because it'll be the EMF (67.6 mV) divided by two. Their voltages will obviously add up to 67.6 mV.

Now the blue guys. According to the other picture the voltage that you insist on calling VAD, which is not of course, because you are measuring the voltage between two points of two different rings, is 27.16 mV. If we add up the voltages around their arms, we get, for the left guy 6.44 mV + 27.16 mV - 33.8 mV = -200 µV. It's not zero, either because I'm using a presumed voltage for the 10k resistor (because you didn't provide it), or because of multimeter imprecision. Whatever the case, we can see that it is two orders of magnitude less than the other voltages.  For the blue guy on the right we have, likewise, 61.15 mV - 33.8 mV - 27.16 mV = 190 µV, which is not surprising, since we have -200µV for the other blue guy. This imbalance is within 5% tolerance for the resistors you used.

Anyway, you can clearly see that the voltages along the arms of the yellow guys tend to accumulate, while the voltages for the blue guys they tend to cancel each other out.

What is the difference between them? As I said, the yellow guys are encircling a varying magnetic field with their arms, while the blue guys are not.

Since KVL ONLY HOLDS where the voltages cancel each other out, we can conclude that KVL holds for the blue guys, but not for the yellow guys. Connecting the dots, KVL ONLY HOLDS for voltages that are NOT circling around a varying magnetic field, and fails when they are.

You see, all of that we concluded by simple observation. Now, we can try to predict an outcome with what we learned. If you repeat the same experiment and now place a voltmeter like the picture below (in red), we can expect pretty much that the voltage we will measure will either be zero or much lower than the voltages we previously measured. Why? Because we will have a blue guy there, who will not be able to embrace a varying magnetic field.

(https://www.eevblog.com/forum/amphour/562-electroboom!/?action=dlattach;attach=1378060;image)
Title: Re: #562 – Electroboom!
Post by: jesuscf on January 13, 2022, 04:46:35 am
You see, all of that we concluded by simple observation. Now, we can try to predict an outcome with what we learned. If you repeat the same experiment and now place a voltmeter like the picture below (in red), we can expect pretty much that the voltage we will measure will either be zero or much lower that the voltages we previously measured. Why? Because we will have a blue guy there, who will not be able to embrace a varying magnetic field.
(https://www.eevblog.com/forum/amphour/562-electroboom!/?action=dlattach;attach=1378060;image)

If you place the multimeter as you show in the image, what you get is the voltage drop in the resistance of the wire which is very small.  The induced EMF in the ring wire is cancelled by the induced EMF in the voltmeter probes.  Please watch Trevor Kearney's video until you understand what is going.  He explains it very nicely, starting at around minute 12:00 and then explains what to do around minute 13:00 in order to measure the induced EMF in that portion of the wire.  Here is the link:

https://youtu.be/FR8k12j7_Eo (https://youtu.be/FR8k12j7_Eo)

Alternatively whatch Jesse's "Lewin's clock", where he takes the precautions pointed out by Trevor Kearney in order to measure the induced EMF in the wires.  This the link:

https://youtu.be/nAsZFP8Cfxk (https://youtu.be/nAsZFP8Cfxk)

Finally, you should seriously review your understanding of mesh analysis.
 




Title: Re: #562 – Electroboom!
Post by: thinkfat on January 13, 2022, 06:25:54 am
Engineer: "That model does not look right"
Scientist: "Eureka!"

When engineer say "you have to place wire here to get voltage X but have to place wire there to get voltage Y"- he gets fired.
When scientist say exactly same words - he gets worshipped.
Go figure.

Oh, but that's exactly what jesuscf says: "You have to place the wires _there_ to get voltage \$V_{AD}\$. Everything else is incorrect!"
And then he goes on to prove that KVL works because KVL works!
Title: Re: #562 – Electroboom!
Post by: bsfeechannel on January 13, 2022, 08:03:03 am
If you place the multimeter as you show in the image, what you get is the voltage drop in the resistance of the wire which is very small.

That's right. That's what we are going to measure. We are after voltages here, because we need to ascertain if Kirchhoff's VOLTAGE law holds for Lewin's circuit.

Quote
The induced EMF in the ring wire is cancelled by the induced EMF in the voltmeter probes.

OK. Let's suppose for a moment that the only thing going on in the wires (and probes, which are also wires) is the induced EMF. If instead of the meter, I hook up an arbitrary resistive load, for instance an LED, we won't have anything done there, because all the voltage that would be supposedly available by the EMF in the wire, will be cancelled by the leads of the LED, which will take the place of the probes. So for all intents and purposes, the voltage across the wire will be ideally zero, or, in practice, just the resistive voltage drop.

No wonder no one connects a load across a wire on a transformer. EDIT: the way the meter is connected in my previous message, of course.

Since the EMF is cancelled, as you say, for things connected in parallel with wires, let's see how it behaves for the resistors in series with them.

When we have a battery connected to a load, the voltages, and their respective electric fields, across the battery and the resistors are in opposite directions around the circuit. So, when we add them up, they will cancel each other out as dictated by Kirchhoff's VOLTAGE law. Or, in Kirchhoff's own words, the EMF (the voltage across the battery) will have to be equal to the current through the resistor times its resistance.

(https://www.eevblog.com/forum/amphour/562-electroboom!/?action=dlattach;attach=1378114;image)

However, when we replace the battery with a wire and subject the entire circuit to a varying magnetic field, the electric field inside the resistor and inside the wire, and their associated EMFs, will be in the same direction around the circuit. So they do not cancel each other out in the equations. Quite the opposite, they'll accumulate. If I suppose that the induced EMF will have to be equal to the current through the resistor times its resistance, when I go around the loop I will be effectively counting them twice, because they have the same direction.

(https://www.eevblog.com/forum/amphour/562-electroboom!/?action=dlattach;attach=1378120;image)

So it is pretty obvious that something else is going on. We need to have in the wire an electric field that has the opposite direction of the electric field in the resistor along the circuit. Just like we had, when there was only a battery. And we do. It is the electrostatic, or coulombic electric field that is generated by the displacement of charges produced by the induced electric field in the wire.

So now the puzzle is solved. What the wire is doing there is to produce the necessary coulombic field, which is conservative, by the way, to cancel the field in the resistor along the circuit in the equations, and as an aside, provide a return path for the current.

However we have a caveat. Since the coulombic field has the same magnitude as the induced field in the wire, and they are in opposite directions, they will cancel each other out, and the field inside the wire will be zero, rendering part of the induced EMF there as effectively zero. The "rest" of the EMF plus the coulombic field will be in the same direction inside the resistor and they will add up to be equal to the total induced EMF. That's a pretty cool mechanism that  works very conveniently.

(https://www.eevblog.com/forum/amphour/562-electroboom!/?action=dlattach;attach=1378162;image)

So yeah, the meter is telling the truth. The total EMF in the wire is just the residual voltage necessary to satisfy Ohm's law. KVL unfortunately for this case (or for this path, if you prefer) kills itself so as to make sense from the point of view of the fields. It survives, however, outside the magnetic field. But inside, for-fluxing-get it. The energy is obviously coming from whatever is generating the varying magnetic field, just like in a transformer or loop antenna, or any arbitrary loop (conductive or not) subjected to an "interference" ( which etymologically means bringing [something] into ).

EDIT: last image replaced, grammar and typos corrected.
Title: Re: #562 – Electroboom!
Post by: Sredni on January 13, 2022, 12:16:37 pm
Also, do you remember that time you asked me if I had a toroid?  Well, I have one now!

The induced field generated by a toroidal core is not circularly symmetric around the section of the core. It's more like the magnetic field generated by a single loop of current:

(https://i.stack.imgur.com/rsmwJ.jpg)

The induced field is stronger inside, being maximum on the axis of the toroid, where all 'cross sections' of the core contribute constructively and decreases faster than 1/r at the exterior. Therefore, your circular ring - even if it is perfectly circular and perfectly concentring to a perfectly circular cross-section core - will not experience the same Eind field in all points of its circumference.
The infinite long solenoid, with its perfectly circular Eind field lines, on the other hand, allows you to pull the trick of considering the contribute on arcs of equal length to be the same. But a toroidal core? No.
Therefore your answer about having

Quote
VA_to_R1=16.9mV
VR1_to_D=16.9mV
VD_to_R2=16.9mV
VR2_to_A=16.9mV

in the approximately circular ring around your square section core is... you guessed from the preview... WRONG.

And this is where I wanted to take you - and Jesse - some ten-fifteen pages ago when I asked you to specifiy your 'McDonald voltages' for all parts of that 'killer question' circuit that Jesse kept reposting. But you guys ran away like politicians from a truth serum.
Like you seemingly did when I asked you to explain why Hayt - which defines voltage as the path integral of the electric field - makes a difference between potential difference (to be used in the static case) and voltage/emf (to be used in the dynamic case).

Title: Re: #562 – Electroboom!
Post by: emece67 on January 13, 2022, 06:18:44 pm
.
Title: Re: #562 – Electroboom!
Post by: jesuscf on January 13, 2022, 06:41:12 pm
Also, do you remember that time you asked me if I had a toroid?  Well, I have one now!

The induced field generated by a toroidal core is not circularly symmetric around the section of the core. It's more like the magnetic field generated by a single loop of current:

(https://i.stack.imgur.com/rsmwJ.jpg)

The induced field is stronger inside, being maximum on the axis of the toroid, where all 'cross sections' of the core contribute constructively and decreases faster than 1/r at the exterior. Therefore, your circular ring - even if it is perfectly circular and perfectly concentring to a perfectly circular cross-section core - will not experience the same Eind field in all points of its circumference.
The infinite long solenoid, with its perfectly circular Eind field lines, on the other hand, allows you to pull the trick of considering the contribute on arcs of equal length to be the same. But a toroidal core? No.
Therefore your answer about having

Quote
VA_to_R1=16.9mV
VR1_to_D=16.9mV
VD_to_R2=16.9mV
VR2_to_A=16.9mV

in the approximately circular ring around your square section core is... you guessed from the preview... WRONG.

And this is where I wanted to take you - and Jesse - some ten-fifteen pages ago when I asked you to specifiy your 'McDonald voltages' for all parts of that 'killer question' circuit that Jesse kept reposting. But you guys ran away like politicians from a truth serum.
Like you seemingly did when I asked you to explain why Hayt - which defines voltage as the path integral of the electric field - makes a difference between potential difference (to be used in the static case) and voltage/emf (to be used in the dynamic case).

You seem to agree that in my setup VA_to_R1=VR2_to_A and VR1_to_D=VD_to_R2.  Is that correct?
Title: Re: #562 – Electroboom!
Post by: jesuscf on January 13, 2022, 06:42:25 pm
In case anybody is wondering about drilling ferrite cores, keep in mind that they are really hard to drill  (that's is, it took me ~1 h to drill 6.5 mm |O )

Also very brittle!
Title: Re: #562 – Electroboom!
Post by: Sredni on January 13, 2022, 10:41:12 pm
Also, do you remember that time you asked me if I had a toroid?  Well, I have one now!

The induced field generated by a toroidal core is not circularly symmetric around the section of the core. It's more like the magnetic field generated by a single loop of current:

(https://i.stack.imgur.com/rsmwJ.jpg)

The induced field is stronger inside, being maximum on the axis of the toroid, where all 'cross sections' of the core contribute constructively and decreases faster than 1/r at the exterior. Therefore, your circular ring - even if it is perfectly circular and perfectly concentring to a perfectly circular cross-section core - will not experience the same Eind field in all points of its circumference.
The infinite long solenoid, with its perfectly circular Eind field lines, on the other hand, allows you to pull the trick of considering the contribute on arcs of equal length to be the same. But a toroidal core? No.
Therefore your answer about having

Quote
VA_to_R1=16.9mV
VR1_to_D=16.9mV
VD_to_R2=16.9mV
VR2_to_A=16.9mV

in the approximately circular ring around your square section core is... you guessed from the preview... WRONG.

And this is where I wanted to take you - and Jesse - some ten-fifteen pages ago when I asked you to specifiy your 'McDonald voltages' for all parts of that 'killer question' circuit that Jesse kept reposting. But you guys ran away like politicians from a truth serum.
Like you seemingly did when I asked you to explain why Hayt - which defines voltage as the path integral of the electric field - makes a difference between potential difference (to be used in the static case) and voltage/emf (to be used in the dynamic case).

You seem to agree that in my setup VA_to_R1=VR2_to_A and VR1_to_D=VD_to_R2.  Is that correct?

In my world they are all identical and zero, if we use perfect conductors. In the real world they are equal to the current flowing in the ring times the resistance of the length of the arc, whatever that maybe. I can easily calculate their value no matter how the ring is shaped and placed.

In your world, if by V you intend the scalar potential difference... Good luck! Unless you have perfectly symmetric rings perfectly centered around a perfectly symmetric core, you need to resort to numerical computation to get their values.

Moreover, when variable magnetic fields are present, the scalar potential difference does not give the full story. It is a partial result in that it describe only a part of the actual electric field in the ring.

Have you started reading Hayt, yet?
Title: Re: #562 – Electroboom!
Post by: ogden on January 13, 2022, 11:08:20 pm
In my world they are all identical and zero, if we use perfect conductors. In the real world they are equal to the current flowing in the ring times the resistance of the length of the arc, whatever that maybe. I can easily calculate their value no matter how the ring is shaped and placed.

Engineer just measured 12VAC on transformer output. Wannabe scientist: as resistance of transformer secondary winding is zero, then voltage on terminals of transformer secondary is I*R meaning - zero. [edit] Those guys claim to know Faraday's law? C'mon.
Title: Re: #562 – Electroboom!
Post by: Sredni on January 14, 2022, 12:30:14 am
In my world they are all identical and zero, if we use perfect conductors. In the real world they are equal to the current flowing in the ring times the resistance of the length of the arc, whatever that maybe. I can easily calculate their value no matter how the ring is shaped and placed.

Engineer just measured 12VAC on transformer output. Wannabe scientist: as resistance of transformer secondary winding is zero, then voltage on terminals of transformer secondary is I*R meaning - zero. [edit] Those guys claim to know Faraday's law? C'mon.

We really are back to square one, aren't we?
In my world voltage is path dependent so I can have zero voltage ALONG the coil filament, and 12 V ACROSS the coil's terminal.

This is what the math says. And this is what the physics says. The amount of fuel per passenger when you go from LA to NY is different for different paths: if your path goes through Chicago it's one number; if you go through Lima, Peru, it's another.
Apparently this simple concept is inconceivable to KVLers.
Title: Re: #562 – Electroboom!
Post by: jesuscf on January 14, 2022, 06:26:59 am
In my world they are all identical and zero, if we use perfect conductors. In the real world they are equal to the current flowing in the ring times the resistance of the length of the arc, whatever that maybe. I can easily calculate their value no matter how the ring is shaped and placed.

I see.  I have nothing to discuss with you then.
Title: Re: #562 – Electroboom!
Post by: thinkfat on January 14, 2022, 07:35:34 am
In my world they are all identical and zero, if we use perfect conductors. In the real world they are equal to the current flowing in the ring times the resistance of the length of the arc, whatever that maybe. I can easily calculate their value no matter how the ring is shaped and placed.

Engineer just measured 12VAC on transformer output. Wannabe scientist: as resistance of transformer secondary winding is zero, then voltage on terminals of transformer secondary is I*R meaning - zero. [edit] Those guys claim to know Faraday's law? C'mon.

Dumb Engineer doesn't want to care about what's happening inside the transformer. He just measures across the transformer terminals and is happy. Engineer who knows Faradays Law understands why his measurements are sometimes giving unintuitive results. This is also the Engineer who's designs pass EMC testing.
Title: Re: #562 – Electroboom!
Post by: Siwastaja on January 14, 2022, 09:24:19 am
This is also the Engineer who's designs pass EMC testing.

Yeah. And EMC is still quite manageable; the mediocre engineer who only knows Kirchoff laws, will be able to try the 5-10 most commonly known "rules of thumb" of EMC design, and likely gets a pass. They don't calculate or simulate EMC; just experimental improvements, until emissions are below the threshold.

Similarly to modifying the layout and probe wiring until the results match within 2%, but with the difference that emissions do not need to match within 2%, getting under the bar is enough.

But then, try to design a modern-day radio communication device. Not a pre-certified module, the full design. Now you just can't avoid understanding the physics anymore.

And to be fair, I can't do it. But I'm grateful to those who can.
Title: Re: #562 – Electroboom!
Post by: Sredni on January 14, 2022, 03:51:23 pm
In my world they are all identical and zero, if we use perfect conductors. In the real world they are equal to the current flowing in the ring times the resistance of the length of the arc, whatever that maybe. I can easily calculate their value no matter how the ring is shaped and placed.

I see.  I have nothing to discuss with you then.

This is a common trait of you KVLers: when you are faced with your self-contradictions you either censor your critics (like fromjesse, RSD Academy on their channels, where they can be censors) or all of a sudden you just find out there are other, more important thing to do and flee (like Jesse Gordon here, where he has no power to censor critics, or Mabilde on his channel, or you now). Except they always find the time to answer the easy question or starting from zero, ignoring all the inconsistencies that made em stop discussing with the other critics.

Anyway, you mentioned the magnetic potential. Hayt calls the scalar magnetic potential Vm and look what he has to say about it (this is from my fourth edition, but I am pretty sure you can find it on one of the editions you have at hand):

(https://i.postimg.cc/W4p8jpHn/01-magnetic-potential-multivaluedness.jpg)

(https://i.postimg.cc/7YYZgpdR/02-magnetic-scalar-potential-vs-electric.jpg)

Oh, look: in the electrostatics case, the voltage is also scalar electric potential difference and as such is independent of path (thanks to that circulation equal to zero). In the magnetostatic case, on the other hand, the fact that the corresponding circulation is NOT zero, but I, the magnetic scalar potential ends up being a non-conservative field. The path integral becomes path-dependent and multivalued: to get the actual value YOU NEED TO SPECIFY THE PATH FROM a TO b.

And now look what happens to the electric potential in the presence of a changing magnetic flux:

(https://i.postimg.cc/RZpdSxxx/03-quasistatics-and-multivaluedness-of-v.jpg)

Oh, look! The circulation of the electric field that in the electrostatic case was zero is now equal to -d flux/dt, and this - just like in the case of the scalar magnetic potential - makes the integral (voltage) multivalued and path dependent. YOU NEED TO SPECIFY THE PATH FROM a TO b.

This is written on the book you have what? five copies of?
Title: Re: #562 – Electroboom!
Post by: ogden on January 14, 2022, 07:18:57 pm
Dumb Engineer doesn't want to care about what's happening inside the transformer. He just measures across the transformer terminals and is happy. Engineer who knows Faradays Law understands why his measurements are sometimes giving unintuitive results. This is also the Engineer who's designs pass EMC testing.

It does not matter - he is dumb engineer or dumb scientist. If he do not use proper model for analysis, he do not get proper results. Problem with all this engineers vs scientists tribe war is - both tribes do not even agree about common model which will be used for analysis. Scientist tribe say that segment of the loop can't be "lumped element" because "we say so". Engineers who say that EMF is induced in any kind of conductor, not necessarily loop - are labelled with swear words. Go figure.